Вы находитесь на странице: 1из 536

Free eBooks on @neetquestionpaper2020

ACE
for AIPMT
AIIMS / BHU / JIPMER / AMU

Vol. 2

Must for Based on


Class 12 NCERT
Free eBooks on @neetquestionpaper2020

• Head Office : B-32, Shivalik Main Road, Malviya Nagar, New Delhi-110017

• Sales Office : B-48, Shivalik Main Road, Malviya Nagar, New Delhi-110017
Tel. : 011-26691021 / 26691713

Typeset by Disha DTP Team

DISHA PUBLICATION
ALL RIGHTS RESERVED

© Copyright Author
No part of this publication may be reproduced in any form without prior permission of the author and the publisher. The
author and the publisher do not take any legal responsibility for any errors or misrepresentations that might have crept in.
We have tried and made our best efforts to provide accurate up-to-date information in this book.

For further information about the books from DISHA


Log on to www.dishapublication.com or email to info@dishapublication.com
Free eBooks on @neetquestionpaper2020

INDEX
TOPIC-WISE AIPMT 2014 & 2015 SOLVED PAPERS V2-1 – V2-8

16. Electric Charges and Fields ............................................................... 533-576


Electric charges: Conductors and insulators, Basic properties of charge, Coulomb’s law-forces between
two point charges, forces between multiple charges; superposition principle and continuous charge
distribution. Electric field: Electric field due to a point charge, Electric field lines, Electric dipole, Electric field
due to a dipole, Torque on a dipole in a uniform electric field. Electric flux, Gauss’s law and its applications
to find field due to infinitely long, uniformly charged straight wire, uniformly charged infinite plane sheet and
uniformly charged thin spherical shell.

17. Electrostatic Potential and Capacitance .......................................... 577-622


Electric potential and its calculation for a point charge, electric dipole and system of charges; Equipotential
surfaces, Electrical potential energy of a system of two point charges in an electrostatic field; Dielectrics and
polarisation; Capacitors and capacitance; Combination of capacitors in series and in parallel; Capacitance
of a parallel plate capacitor with and without dielectric medium between the plates, Energy stored in a
capacitor; Van de Graff generator

18. Current Electricity................................................................................ 623-690


Electric current; Drift velocity, Mobility, Ohm’s law, Electrical resistance, Resistances of different materials, V-
I characteristics of ohmic and nonohmic conductors, Electrical energy and power, Electrical resistivity,
Colour code for resistors; Series and parallel combinations of resistors; Temperature dependence of
resistivity; Electric Cell and its Internal resistance, potential difference and emf of a cell, combination of cells
in series and in parallel; Kirchhoff’s laws and their applications. W heatstone bridge, Metre bridge;
Potentiometer - principle and its applications; Joule's law of heating, electric energy and power, electro-
chemical cells, thermoelectric effect

19. Moving Charges and Magnetism ...................................................... 691-734


Biot - Savart law and its application to current carrying circular loop; Ampere’s circuital law and its applications
to infinitely long current carrying straight wire and solenoid; Force on a moving charge in uniform magnetic
and electric fields; Cyclotron; Force on a current-carrying conductor in a uniform magnetic field; Force
between two parallel current-carrying conductors-definition of ampere; Torque experienced by a current
loop in uniform magnetic field; Moving coil galvanometer, its current sensitivity and conversion to ammeter
and voltmeter.

20. Magnetism and Matter ........................................................................ 735-764


Bar magnet; Current loop as a magnetic dipole and its magnetic dipole moment. Bar magnet as an
equivalent solenoid, Magnetic field lines; Gauss's laws in magnetism; Earth’s magnetic field and magnetic
elements; Para-, dia- and ferro- magnetic substances; Magnetisation and magnetic intensity; Magnetic
susceptibility and permeability, Hysteresis, Electromagnets and permanent magnets.

21. Electromagnetic Induction ................................................................. 765-786


Experiments of Farday and Henry; Magnetic flux; Electromagnetic induction; Faraday’s law, induced emf
and current; Lenz’s law, Eddy currents; Self and mutual inductance; AC generator and DC motor.

22. Alternating Current .............................................................................. 787-816


Alternating current, peak and rms value of alternating current/ voltage; Reactance and impedance; LC, LR,
CR and LCR series circuit, Resonance; Quality factor; Power in AC circuits, Wattless current; Transformer.
Free eBooks on @neetquestionpaper2020

23. Electromagnetic Waves ...................................................................... 817-828


Displacement current; Electromagnetic waves and their characteristics; Electromagnetic spectrum (radio
waves, microwaves, infrared, visible, ultraviolet, X-rays, gamma rays); Applications of e.m. waves.

24. Ray Optics and Optical Instruments ................................................ 831-884


Reflection and refraction of light at plane and spherical surfaces; Mirror formula; Total internal reflection and
its applications; Deviation and dispersion of light by a prism; Lens formula; Magnification; Power of a lens;
Combination of thin lenses in contact; Microscope and Telescope and their magnifying powers.

25. Wave Optics .......................................................................................... 885-914


Wavefront and Huygens principle; Laws of reflection and refraction using Huygens principle; Interference;
Young’s double slit experiment and expression for fringe width; Coherent sources and sustained interference
of light; Diffraction due to a single slit, width of central maxima; Resolving power of microscopes and
telescopes, Polarisation, plane polarized light; Brewster’s law.

26. Dual Nature of Radiation and Matter ................................................ 915-944


Emission of electron; Photoelectric effect, Hertz and Lenard’s observations; Einstein’s photoelectric equation;
Particle and wave nature of light; Matter waves, de Broglie relation; Davisson-Germer experiment.

27. Atoms ..................................................................................................... 945-962


Alpha-particle scattering experiment; Rutherford’s model of atom; Bohr model; Energy levels; Atomic spectra,
line spectra of hydrogen atom; de Broglie's explanation of Bohr's second postulate of quantisation.

28. Nuclei ..................................................................................................... 963-990


Composition and size of nucleus, atomic masses, isotopes, isobars; isotones; Radioactivity-alpha, beta
and gamma particles/rays and their properties; Radioactive decay law; Mass-energy relation, mass defect;
Binding energy per nucleon and its variation with mass number; Nuclear force; Nuclear-reaction fission
and fusion.

29. Semiconductor Electronics : Materials, Devices and


Simple Circuits ................................................................................... 991-1032
Solids, conductors, insulators and Semiconductors; semiconductor diode : I-V characteristics in forward
and reverse bias; Diode as a rectifier; I-V characteristics of LED, photodiode, solar cell, and Zener diode;
Zener diode as a voltage regulator. Junction transistor, transistor action, characteristics of a transistor;
Transistor as an amplifier (common emitter configuration) and oscillator. Logic gates (OR, AND, NOT,
NAND, NOR and XOR).

30. Communication Systems ...............................................................1033-1056


Basic elements of a communication system; Propagation of electromagnetic waves in the atmosphere; Sky
and space wave propagation, Need for modulation, Amplitude and frequency modulation, Bandwidth of
signals, Bandwidth of Transmission medium.
Free eBooks on @neetquestionpaper2020

Topic-wise AIPMT 2014 & 2015 Solved Papers

Chapter 16 : Electric Charges and Fields 5. A parallel plate air capacitor of capacitance C is connected
1. The electric field in a certain region is acting radially outward to a cell of emf V and then disconnected from it. A dielectric
and is given by E = Ar. A charge contained in a sphere of slab of dielectric constant K, which can just fill the air gap of
radius 'a' centred at the origin of the field, will be given by the capacitor, is now inserted in it. Which of the following is
[2015] incorrect ? [2015]
(a) A e0 a2 (b) 4 pe0 Aa3 (a) The energy stored in the capacitor decreases K times.
(c) e0 Aa3 (d) 4 pe0 Aa2 1 æ1 ö
(b) The chance in energy stored is CV 2 ç – 1÷
Chapter 17 : Electrostatic Potential and Capacitance 2 èK ø
2. Two thin dielectric slabs of dielectric constants K1 and K2 (c) The charge on the capacitor is not conserved.
(K1 < K2) are inserted between plates of a parallel plate (d) The potential difference between the plates decreases
capacitor, as shown in the figure. The variation of electric K times.
field ‘E’ between the plates with distance ‘d’ as measured Chapter 18 : Current Electricity
from plate P is correctly shown by : [2014] 6. Two cities are 150 km apart. Electric power is sent from one
city to another city through copper wires. The fall of potential
P +
+


Q per km is 8 volt and the average resistance per km is 0.5 W.
+ – The power loss in the wires is : [2014]
+ –
+ – (a) 19.2 W (b) 19.2 kW
+ –
+ – (c) 19.2 J (d) 12.2 kW
+ –
+ – 7. The resistances in the two arms of the meter bridge are 5W
K1 K2 and RW, respectively. When the resistance R is shunted
with an equal resistance, the new balance point is at 1.6 l1.
The resistance ‘R’ is : [2014]
E E

(a) (b) 5W RW
0 0
d d

G
E
E
A B
(c) (d) l1 100 – l1
0 (a) 10W (b) 15W
d 0 d (c) 20W (d) 25W
3. A conducting sphere of radius R is given a charge Q. The 8. A potentiometer circuit has been set up for finding the
electric potential and the electric field at the centre of the internal resistance of a given cell. The main battery used
sphere respectively are: [2014] across the potentiometer wire, has an emf of 2.0 V and a
Q negligible internal resistance. The potentiometer wire itself
(a) Zero and
4p e 0 R 2 is 4m long, When the resistace R, connected across the
Q given cell, has values of
(b) and Zero (i) infinity (ii) 9.5W
4p e 0 R The balancing lengths’, on the potentiometer wire are found
to be 3 m and 2.85 m, respectively. The value of internal
Q Q
(c) and resistance of the cell is [2014]
4p e 0 R 4p e 0 R 2 (a) 0.25W (b) 0.95W
(d) Both are zero (c) 0.5W (d) 0.75W
9. In an ammeter 0.2% of main current passes through the
4. In a region, the potential is represented by
galvanometer. If resistance of galvanometer is G, the
V(x, y, z) = 6x – 8xy – 8y + 6yz, where V is in volts and x, y, z
resistance of ammeter will be : [2014]
are in metres. The electric force experienced by a charge of 2
coulomb situated at point (1, 1, 1) is : [2014] 1 499
(a) G (b) G
(a) 499 500
6 5 N (b) 30 N
1 500
(c) 24 N (d) 4 35 N (c) G (d) G
500 499
Free eBooks on @neetquestionpaper2020

V2-2 AIPMT 2014-2015 SOLVED P APER


10. Across a metallic conductor of non-uniform cross section a ur ur
constant potential difference is applied. The quantity which
remains constant along the conductor is : [2015]
(a)
m I $
B=– 0
4p R
mi ´ 2k$ ( ) (b)
m I $
B=– 0
4p R
(
pi + 2k$ )
(a) current (b) drift velocity
ur m0 I ur m0 I
11.
(c) electric field (d) current density
A potentiometer wire has length 4 m and resistance 8W. The
(c) B=
4p R
(
p$i – 2k$ ) (d) B=
4p R
(
p$i + 2k$ )
resistance that must be connected in series with the wire Chapter 20 : Magnetism and Matter
and an accumulator of e.m.f. 2V, so as to get a potential
gradient 1 mV per cm on the wire is [2015] 16. Following figures show the arrangement of bar magnets in
(a) 40 W (b) 44 W different configurations. Each magnet has magnet ic dipole
r
(c) 48 W (d) 32 W moment m . Which configuration has highest net magnetic
12. A, B and C are voltmeters of resistance R, 1.5 R and 3R dipole moment ? [2014]
respectively as shown in the figure. When some potential
difference is applied between X and Y, the voltmeter readings N
are VA, VB and VC respectively. Then [2015]
N S
B
A. B. S N
A S S N
X Y
C
N
(a) VA ¹ VB = VC (b) VA = VB ¹ VC N
(c) VA ¹ VB ¹ VC (d) VA = VB = VC C. 30º D.
Chapter 19 : Moving Charges and Magnetism S 60º
N
13. Two identical long conducting wires AOB and COD are S N
placed at right angle to each other, with one above other (a) A (b) B
such that ‘O’ is their common point for the two. The wires (c) C (d) D
carry I1 and I2 currents respectively. Point ‘P’ is lying at
distance ‘d’ from ‘O’ along a direction perpendicular to the Chapter 21 : Electromagnetic Induction
plane containing the wires. The magnetic field at the point 17. A thin semicircular conducting ring (PQR) of radius ‘r’ is
‘P’ will be : [2014] falling with its plane vertical in a horizontal magnetic field B,
m 0 æ I1 ö m0 as shown in figure. The potential difference developed
(a) (b) (I1 + I2 ) across the ring when its speed is v, is : [2014]
2pd çè I2 ÷ø 2pd

m0 2 m0 2 2 1/ 2 Q B
(c) (I1 - I22 ) (d) (I1 ´ I 2 )
2pd 2 pd
14. An electron moving in a circular orbit of radius r makes n r
rotations per second. The magnetic field produced at the P R
centre has magnitude: [2015]
m0 n 2 e (a) Zero
(a) Zero (b)
r (b) Bvpr2 /2 and P is at higher potnetial
m0 ne m0 ne (c) prBv and R is at higher potnetial
(c) (d)
2r 2pr (d) 2rBv and R is at higher potential
15. A wire carrying current I has the shape as shown in adjoining
18. A conducting square frame of side ‘a’ and a long staight
figure. Linear parts of the wire are very long and parallel to
wire carrying current I are located in the same plane as shown
X-axis while semicircular portion of radius R is lying in Y-Z
in the figure. The frame moves to the right with a constant
plane. Magnetic field at point O is : [2015]
velocity ‘V’. The emf induced in the frame will be proportional
Z to [2015]
X

I
R l
Y
O V

I
a
X
Free eBooks on @neetquestionpaper2020

AIPMT 2014-2015 SOLVED P APER V2-3

1 1 26. Two identical thin plano-convex glass lenses (refractive


(a) (b) index 1.5) each having radius of curvature of 20 cm are placed
(2x – a)2 (2x + a)2 with their convex surfaces in contact at the centre. The
1 intervening space is filled with oil of refractive index 1.7.
1
(c) (d) The focal length of the combination is [2015]
(2x – a)(2x + a) x2 (a) –25 cm (b) –50 cm
Chapter 22 : Alternating Current (c) 50 cm (d) –20 cm
19. A transformer having efficiency of 90% is working on 200V Chapter 25 : Wave Optics
and 3kW power supply. If the current in the secondary coil 27. A beam of light of l = 600 nm from a distant source falls on
is 6A, the voltage across the secondary coil and the current a single slit 1 mm wide and the resulting diffraction pattern
in the primary coil respectively are : [2014] is observed on a screen 2 m away. The distance between
(a) 300 V, 15A (b) 450 V, 15A first dark fringes on either side of the central bright fringe is:
(c) 450V, 13.5A (d) 600V, 15A [2014]
20. A resistance 'R' draws power 'P' when connected to an AC (a) 1.2 cm (b) 1.2 mm
source. If an inductance is now placed in series with the (c) 2.4 cm (d) 2.4 mm
resistance, such that the impedance of the circuit becomes 28. In the Young’s double-slit experiment, the intensity of light
'Z', the power drawn will be [2015] at a point on the screen where the path difference is l is K,
(l being the wave length of light used). The intensity at a
R æRö point where the path difference is l/4, will be: [2014]
(a) P (b) Pç ÷
Z èZø (a) K (b) K/4
2 (c) K/2 (d) Zero
æRö 29. In a double slit experiment, the two slits are 1 mm apart and
(c) P (d) Pç ÷
èZø the screen is placed 1 m away. A monochromatic light
Chapter 23 : Electromagnetic Waves wavelength 500 nm is used. What will be the width of each
21. Light with an energy flux of 25 × 104 Wm– 2 falls on a perfectly slit for obtaining ten maxima of double slit within the central
reflecting surface at normal incidence. If the surface area is maxima of single slit pattern ? [2015]
15 cm2, the average force exerted on the surface is :[2014] (a) 0.1 mm (b) 0.5 mm
(a) 1.25 × 10– 6 N (b) 2.50 × 10– 6 N (c) 0.02 mm (d) 0.2 mm
(c) 1.20 × 10 N– 6 (d) 3.0 × 10– 6 N 30. For a parallel beam of monochromatic light of wavelength
22. A radiation of energy ‘E’ falls normally on a perfectly 'l', diffraction is produced by a single slit whose width 'a' is
reflecting surface. The momentum transferred to the surface of the wavelength of the light. If 'D' is the distance of the
is (C = Velocity of light) [2015] screen from the slit, the width of the central maxima will be
2E 2E Dl Da
(a) (b) (a) (b) [2015]
C C2 a l

E
2Da 2Dl
E (c) (d)
(c) (d) l a
C2 C
Chapter 26 : Dual Nature of Radiation and Matter
Chapter 24 : Ray Optics and Optical Instruments
31. When the energy of the incident radiation is incredased by
23. If the focal length of objective lens is increased then 20%, the kinetic energy of the photoelectrons emitted from
magnefying power of : [2014] a metal surface increased from 0.5 eV to 0.8 eV. The work
(a) microscope will increase but that of telescope decrease. function of the metal is : [2014]
(b) microscope and telescope both will increase. (a) 0.65 eV (b) 1.0 eV
(c) microscope and telescope both will decrease (c) 1.3 eV (d) 1.5 eV
(d) microscope will decrease but that of telescope increase. 32. If the kinetic energy of the particle is increased to 16 times
24. The angle of a prism is ‘A’. One of its refracting surfaces is its previous value, the percentage change in the de-Broglie
silvered. Light rays falling at an angle of incidence 2A on wavelength of the particle is : [2014]
the first surface returns back through the same path after (a) 25 (b) 75
suffering reflection at the silvered surface. The refractive (c) 60 (d) 50
index m, of the prism is : [2014] 33. Which of the following figures represent the variation of
(a) 2 sin A (b) 2 cos A particle momentum and the associated de-Broglie
1 wavelength? [2015]
(c) cos A (d) tan A p
2 p
25. The refracting angle of a prism is ‘A’, and refractive index of
the material of the prism is cot(A/2). The angle of minimum
(a) (b)
deviation is : [2015]
(a) 180° – 2A (b) 90° – A
(c) 180° + 2A (d) 180° – 3A l
l
Free eBooks on @neetquestionpaper2020

V2-4 AIPMT 2014-2015 SOLVED P APER


p p 5 3
(a) R Al (b) R Al
3 5
(c) (d) 1/3 1/3
æ 13 ö æ 53 ö
(c) ç ÷ R Al (d) ç ÷ R Al
è 53 ø è 13 ø
l l
Chapter 29 : Semiconductor Electronics : Materials,
34. A certain metallic surface is illuminated with monochromatic Devices and Simple Circuits
light of wavelength l. The stopping potential for photo- 41. The given graph represents V - I characteristic for a
electric current for this light is 3V0. If the same surface is semiconductor device.
illuminated with light of wavelength 2l, the stopping
potential is V0. The threshold wavelength for this surface
for photo-electric effect is [2015] I
A
l
(a) 4l (b) V
4
B
l
(c) (d) 6l
6 Which of the following statement is correct ? [2014]
Chapter 27 : Atoms (a) It is V - I characteristic for solar cell where, point A
35. Hydrogen atom in ground state is excited by a represents open circuit voltage and point B short circuit
monochromatic radiation of l = 975 Å. Number of spectral current.
lines in the resulting spectrum emitted will be [2014] (b) It is a for a solar cell and point A and B represent open
(a) 3 (b) 2 circuit voltage and current, respectively.
(c) 6 (d) 10 (c) It is for a photodiode and points A and B represent
36. Consider 3rd orbit of He+ (Helium), using non-relativistic open circuit voltage and current, respectively.
approach, the speed of electron in this orbit will be [given K (d) It is for a LED and points A and B represent open circuit
= 9 × 109 constant, Z = 2 and h (Plank's Constant) voltage and short circuit current, respectively.
= 6.6 × 10–34 J s] [2015] 42. The barrier potential of a p-n junction depends on:
(a) 1.46 × 106 m/s (b) 0.73 × 106 m/s [2014]
(c) 3.0 × 108 m/s (d) 2.92 × 106 m/s (A) type of semi conductor material
37. Two particles of masses m1, m2 move with initial velocities (B) amount of doping
u1 and u2. On collision, one of the particles get excited to (C) temperature
higher level, after absorbing energy e. If final velocities of Which one of the following is correct ?
particles be v1 and v2 then we must have [2015] (a) (A) and (B) only (b) (B) only
1 1 1 1 (c) (B) and (C) only (d) (A), (B) and (C)
(a) m1u12 + m2u 22 = m1v12 + m2 v22 – e 43. Which logic gate is represented by the following
2 2 2 2
combination of logic gate ? [2015]
1 1 1 1
(b) m1u12 + m 2 u 22 – e = m1v12 + m 2 V22 Y1
2 2 2 2 A
1 2 2 1 2 2 1 1 Y
(c) m1 u1 + m 2 u 2 + e = m12 v12 + m 22 v22
2 2 2 2
B
(d) m12 u1 + m22 u 2 – e = m12 v1 + m22 v 2 Y2
Chapter 28 : Nuclei (a) NAND (b) AND
38. The Binding energy per nucleon of 73 Li and 42 He nuclei are (c) NOR (d) OR
5.60 MeV and 7.06 MeV, respectively. 44. If in a p-n junction, a square input signal of 10 V is applied
as shown, then the output across RL will be [2015]
In the nuclear reaction 73 Li + 11H ® 42 He + Q , the value of
+5V
energy Q released is : [2014]
(a) 19.6 MeV (b) – 2.4 MeV RL
(c) 8.4 MeV (d) 17.3 MeV
39. A radio isotope ‘X’ with a half life 1.4 × 109 years decays to –5V
‘Y’ which is stable. A sample of the rock from a cave was 10V
found to contain ‘X’ and ‘Y’ in the ratio 1 : 7. The age of the
rock is : [2014] (a) (b)
(a) 1.96 × 109 years (b) 3.92 × 109 years –5V
(c) 4.20 × 109 years (d) 8.40 × 109 years
5V
27
40. If radius of the 12 Al nucleus is taken to be RAl, then the (c) (d)
radius of 125
nucleus is nearly: [2015] –10V
53 Te
Free eBooks on @neetquestionpaper2020

AIPMT 2014-2015 SOLVED P APER V2-5

HINTS & SOLUTIONS


Chapter 16 : Electric Charges and Fields æ 3 - 2.85 ö
1. (b) Net flux emmited from a spherical surface of radius a =ç ÷ ´ (9.5) W = 0.5 W
according to Gauss’s theorem è 2.85 ø
q q 9. (c) As 0.2% of main current passes th rough the
fnet = in or, (Aa) (4pa2) = in
e0 e0 998
galvanometer hence I current through the shunt.
So, qin = 4pe0 A a3 1000
Chapter 17 : Electrostatic Potential and Capacitance 998I
1 1000 S
2. (c) Electric field, E µ As K1 < K2 so E1 > E2
K
Hence graph (c) correctly dipicts the variation of electric
field E with distance d.
3. (b) Due to conducting sphere G
At centre, electric field E = 0 I 2I
1000
Q
And electric potential V = æ 2I ö æ 998I ö G
4p Î0 R ç ÷G = ç ÷S Þ S=
è 1000 ø è 1000 ø 499
r ¶V $ ¶V $ ¶V $ Total resistance of Ammeter
4. (d) E=- i- j- k
¶x ¶y ¶z
æ G ö
ç ÷G
= -[(6 - 8y)iˆ + (-8x - 8 + 6z)ˆj + (6y)k]
ˆ SG
= è
499 ø
=
G
r R=
At (1, 1, 1), E = 2i$ + 10$j - 6k$
S+G æ G ö 500
ç ÷+G
r è 499 ø
Þ (E) = 22 + 102 + 62 = 140 = 2 35 10. (a) Here, metallic conductor can be considered as the
r combination of various conductors connected in series.
\ F = qE = 2 ´ 2 35 = 4 35 And in series combination current remains same.
Q
5. (c) Capacitance of the capacitor, C =
V
After inserting the dielectric, new capacitance C1 = K.C i
New potential difference
V V
V1 = 11. (d) Total potential difference across potentiometer wire
K
= 10–3 × 400 volt = 0.4 volt
1 2 Q2 v
ui = cv = (Q Q = cv) 1mv
2 2C potential gradient = = 10–3 v/cm = 10–1 m
cm
Q 2 Q 2 C2 V 2 æ ui ö Let resistance of RW connected in series.
uf = = = =ç ÷
2f 2kc 2KC è k ø 2V
1 2 ì1 ü
Du = uf – ui = cv í – 1ý i
2 îk þ
+0.4V
As the capacitor is isolated, so change will remain
conserved p.d. between two plates of the capacitor RW 8W
Q V 2 10 –1 ´ 4 1
L= = So, = = ÞR + 8 = 40 or,, R = 32 W
KC K R +8 8 20
Chapter 18 : Current Electricity 12. (d) Effective resistance of B and
6. (b) Total resistance R = (0.5 W/km) × (150 km) = 75 W R ×R 1.5R ´ 3R 4.5R 2
Total voltage drop = (8 V/km) × (150 km) = 1200 V = B C = = =R
(DV)2 (1200) 2 R B + R C 1.5R + 3R 4.5R
Power loss = = W i.e., equal to resistance of voltmeter A.
R 75 1.5R
= 19200 W = 19.2 kW
7. (b) This is a balanced wheatstone bridge condition, A B
5 l1 5 1.6l1
= and = ÞR = 15 W R
R 100 - l1 R / 2 100 - 1.6l1 C
8. (c) Internal resistance of the cell, 3Ris same so, VB = VC and
In parallel potential difference
æ E-Vö æ l1 - l 2 ö in series current is same. So, VA = VB = VC
r= ç ÷R = ç ÷R
è V ø è l2 ø
Free eBooks on @neetquestionpaper2020

V2-6 AIPMT 2014-2015 SOLVED P APER


Chapter 19 : Moving Charges and Magnetism
13. (d) Net magnetic field, B = B12 + B22
2 2
æ m0 I1 ö æ m0 I2 ö æ m 0 I1 m 0 I2 ö
= ç 2pd ÷ + ç 2pd ÷ çQ B1 = 2 pd and B2 = 2 pd ÷ As induced current in ring produces magnetic field in
è ø è ø è ø upward direction hence R is at higher potential.
m0 18. (c) Emf induced in side 1 of frame e1 = B1Vl
= I12 + I22 mo I
2pd B1 =
14. (c) Radius of circular orbit = r 2p (x – a/ 2)
No. of rotations per second = n Emf induced in side 2 of frame e2 = B2 Vl
1 O r moI
i.e., T = B2 =
n 2p (x + a/ 2)
Magnetic field at its centre, Bc =?
As we know, current x
e e I
i= = 1 2
T (1 / n)
= en = equivalent current
a v
Magnetic field at the centre of circular orbit, x–
2
m 0 i m0 ne a
Bc = = a
2r 2r x+
15. (b) Megnetic field due to segment ‘1’ 2
Emf induced in square frame
uur m I ur
$ = – m 0 I k$ = B
B1 = 0 [sin 90° + sin 0°] (–k)
4 pR 4pR
3 () m0 I m0 I
e = B1Vl – B2Vl = 2p (x – a / 2) lv – 2p (x + a/ 2) lv
Magnetic field due to segment 2
1
B2 =
4R
( )
m0 I $ – m0 I $
–i =
4pR
pi ( ) or, e µ
(2x – a)(2 x + a)
Z Chapter 22 : Alternating Current
Vs Is V (6)
19. (b) Efficiency h = Þ 0.9 = s
2 Vp Ip 3 ´ 103
Þ Vs = 450 V
O Y As VpIp = 3000 so
1
I 3000 3000
I
3 Ip = = A = 15A
Vp 200
Pure resistor L-R series circuit
X
ur
\ B at centre R R L
20. (d)
ur ur ur ur
Bc = B1 + B2 + B3 =
– m0I $
4 pR
pi + 2k$ ( ) V
Vs
V
V
Chapter 20 : Magnetism and Matter Phasor diagram
q R
16. (c) Net magnetic dipole moment = 2 Mcos q
2
q R
As value of cos is maximum in case (c) hence net XL cos q =
2 Z Z
magnetic dipole moment is maximum for option (c).
Chapter 21 : Electromagnetic Induction
17. (d) Rate of decreasing of area of semicircular ring Z = impedance
For pure resistor circuit, power
dA
= = (2r)V V2
dt P= Þ V 2 = PR
From Faraday’s law of electromagnetic induction R
dq dA For L-R series circuit, power
e= - = -B = - B(2rV) 2
dt dt 1V2 V 2 R PR æ Rö
P = cos q = . = .R =Pç ÷
Z Z Z Z2 è Zø
Free eBooks on @neetquestionpaper2020

AIPMT 2014-2015 SOLVED P APER V2-7

Chapter 23 : Electromagnetic Waves Þ f1 = 40cm


Dp 2IA
21. (b) Average force Fav = = (Q Power = F.V)
Dt c 1 æ 1.7 ö æ 1 1 ö 100
=ç – 1÷ ç – Þ f2 = – cm
4
2 ´ 25 ´ 10 ´ 15 ´ 10 -4 f2 è 1 ø è –20 +20 ÷ø 7
= = 2.50 × 10– 6 N
3 ´ 108 1 æ 1.5 öæ 1 1 ö
and =ç – 1÷ç – ÷ Þ f3 = 40 cm
E f3 è 1 øè ¥ –20 ø
22. (a) Momentum of light falling on reflecting surface p =
C 1 1 1 1 1 1 1 1
As surface is perfectly reflecting so momentum reflect = + + Þ = + +
f eq f1 f 2 f3 f eq 40 –100 / 7 40
E E E \ feq = –50 cm
=P
p1 = – C C Therefore, the focal length of the combination is
C
– 50 cm.
So, momentum transferred Chapter 25 : Wave Optics
E æ E ö 2E 27. (d) Given: D = 2m; d = 1 mm = 1 × 10– 3 m
– – l = 600 nm = 600 × 10– 6 m
C çè C ÷ø
= P – P1 = =
C Width of central bright fringe (= 2b)
Chapter 24 : Ray Optics and Optical Instrument
LD 1 2lD 2 ´ 600 ´ 10 - 6 ´ 2
µ = = m = 2.4 × 10– 3 m = 2.4 mm
23. (d) Magnifying power of microscope = d 1 ´ 10 - 3
f 0f e f0
Hence with increase f0 magnifyig power of microscope 28. (c) For path difference l, phase difference = 2p rad.
decreases. l p
For path difference , phase difference = rad.
f0 4 2
Magnifying power of telescope = µ f0 As K = 4I0 so intensity at given point where path
fe
Hence with increase f0 magnifying power of telescope l
difference is
increases. 4
2 æ pö æ p ö K
K¢ = 4I0 cos ç ÷ ç cos = cos 45º÷ = 2I0 =
è 4ø è 4 ø 2
24. (b) 29. (d) Here, distance between two slits,
d = 1mm = 10–3m
distance of screen from slits, D = 1 m
wavelength of monochromatic light used,
l = 500nm = 500 × 10–9m
sin i width of each slit a = ?
According to Snell’s law m =
sin r 2lD
Þ (1) sin 2A = (m) sin A Þ m = 2 cos A Width of central maxima in single slit pattern =
a
25. (a) As we know, the refractive index of the material of the
prism lD
Fringe width in double slit experiment b =
æ d + Aö d
sin ç m
è 2 ÷ø So, required condition
10 l D 2 l D
=
m=
sin (A/ 2) d a
æ A + dm ö d 1
sin ç Þ a= = ´ 10–3 m = 0.2 mm
è 2 ÷ø cos (A/ 2) 5D 5
cot A/2 = = 2D l
sin A / 2 sin (A / 2) 30. (d) Linear width of central maxima = D(2q) = 2Dq
[Q µ = cot (A/2)] a
æ dm + A ö
Þ Sin çè ÷ = sin(90° + A/2)
2 ø
Þ dmin = 180° – 2A
26. (b) Using lens maker’s formula,
q
q
1 æ 1 1 ö
= (m – 1) ç – ÷
f è R1 R 2 ø
D
1 æ 1.5 öæ 1 1 ö
=ç – 1÷ç – ÷
f1 è 1 øè ¥ –20 ø
n = 1.5 n = 1.5
n = 1.7
Free eBooks on @neetquestionpaper2020

V2-8 AIPMT 2014-2015 SOLVED P APER


Chapter 26 : Dual Nature of Radiation and Matter 2
31. (b) According to Einstein’s photoelectric equation, V = (2.19 × 106) (Z = 2 & n = 3)
hv = f0 + Kmax 3
We have V = 1.46 × 106 m/s
hv = f0 + 0.5 ...(i) 37. (b) By law of conservation of energy,
and 1.2hv = f0 + 0.8 ...(ii) K.Ef = K.Ei – excitation energy (e)
Therefore, from above two equations f0 = 1.0 eV.
1 1 1 1
h ormv12 + mv 22 = m1u12 + m 2 u 22 = e
32. (b) As we know l = 2 2 2 2
P
h Chapter 30 : Nuclei
= (Q P = 2mKE) 38. (d) BE of 2He4 = 4 × 7.06 = 28.24 MeV
2mK
l1 K2 16K 4 BE of 73 Li = 7 × 5.60 = 39.20 MeV
or = = =
l2 K1 K 1 7 1
3 Li +1 H ® 2 He 4 + 2He 4 + Q
Therefore the percentage chanbe in de-Broglie
39.20 28.24 ´ 2(= 56.48 MeV)
1- 4
wavelength = ´ 100 = - 75% Therefore, Q = 56.48 – 39.20 = 17.28 MeV.
4
h 1 Nx 1
39. (c) As = (Given)
33. (a) According to De-broglie p = or P µ Ny 7
l l
where Nx 1 æ1ö
3
P = particle momentum; l = de-Broglie wavelength Þ = =ç ÷
h = Plank’s constant Nx + Ny 8 è 2 ø
Therefore, age of the rock
1 t = 3T1/2 = 3 × 1.4 × 109 yrs = 4.2 × 109 yrs.
P µ represents rectangular hyperbola.
l 40. (a) As we know, R = R0 (A)1/3
34. (a) As we know, where A = mass number
hc RAI = R0 (27)1/3 = 3R0
eVs = –Y
l 5
hc RTe = R0 (125)1/3 = 5R0 =R
3eVo = –Y ...(1)
3 AI
l Chapter 29 : Semiconductor Electronics : Materials,
hc Devices, Simple Circuits and Communication System
eVo = –Y ...(2) 41. (a) The given graph represents V-I characteristics of solar
2l
3hc cell.
3eVo = – 3Y ...(3) 42. (d) The barrier potential of a p-n junction depends on
2l
Multiplying eqn. (2) by (3) and subtracting it from eqn amount of doping, type of semiconductor material and
(1) temperature.
43. (b) First two gates are NOT gates and the last gate is NOR
hc gate.
Y=
4l
So, threshold wavelength, Thus, y1 = A, y2 = B and y = y1 + y2
hc hc The truth table corresponding to this is as follows:
l th = = = 4l
Y hc / 4l
Chapter 27 : Atoms A B y1 = A y 2 = B y1 + y 2 y = y1 + y2 A. B
35. (c) For the l = 975 Å
0 0 1 1 1 0 0
1 æ 1 1 ö
= Rç - ÷ 0 1 1 0 1 0 0
l ç n2 n 2 ÷
è 1 2ø 1 0 0 1 1 0 0
where R is the Rydberg constant 1 1 0 0 0 1 1
Solving we get n 2 = n = 4 (Q n1 = 1 ground state)
Therefore number of spectral lines Thus the combination of gate represents AND gate.
44. (c) Here P-N junction diode rectifies half of the ac wave
n(n - 1) 4(4 - 1) i.e., acts as half wave rectifier. During + ve half cycle
= = =6
2 2 Diode ® forward biased output across will be
36. (a) Speed of electron in nth orbit
5V
2
2p KZe
Vn =
nh
Z
V = (2.19 × 106 m/s) During –ve half cycle Diode ® reverse biased output
n
will not obtained.
Free eBooks on @neetquestionpaper2020

16
Electric Charges
and Fields
ELECTRIC CHARGE Insulators can be charged but do not conduct electric charge.
Charge is something associated with matter due to which it Insulators do not have 'free electrons' that is why insulators do
produces and experiences electric and magnetic effects. not conduct electricity.
There are two types of charges : Induced charge can be lesser or equal to inducing charge (but
(i) Positive charge and (ii) Negative charge never greater) and its max. value is given by
Positive and negative charges : Positive charge means the Q' = – Q (1 – 1/k), where 'Q' is inducing charge and 'K' is the
deficiency of electons while negative charge means excess of dielectric const. of the material of the uncharged body.
electrons. In any neutral body the net charge is equal to zero For metals k = ¥ Þ Q' = – Q.
i.e., the sum of positive charges is equal to the sum of negative
METHODS OF CHARGING
charges.
(i) By friction : By rubbing two suitable bodies, given in box
+ Mass M. – – Mass M'' one is charged by +ve and another by –ve charge in equal
Mass M' –
+

amount.
Electron Electron –
+ Electron
< Proton – > Proton – +ve –ve
= Proton
+ – – Glass rod Silk
+ –
– – – Fur Ebonite rod
Positively charged Uncharged body Negatively charged
body M' < M M body M'' > M
Dry hair Comb
Charge is a scalar quantity and its SI unit is coulomb (C). Wool Amber
CONDUCTORS AND INSULATORS
The materials which allow electric charge (or electricity) to Note : Electric charges remain confined only to the
flow freely through them are called conductors. Metals are very
rubbed portion of a non-conductor but in case of a conductor,
good conductors of electricity. Silver, copper and aluminium are
some of the best conductors of electricity. Our skin is also a they spread up throughout the conductor.
conductor of electricity. Graphite is the only non-metal which is a
(ii) By conduction : Charging a neutral body by touching it
conductor of electricity.
with a charged body is called charging by conduction.
All metals, alloys and graphite have 'free electrons', which can · It is important to note that when the bodies are charged
move freely throughout the conductor. These free electrons make by conduction, a charged and an uncharged bodies
metals, alloys and graphite good conductor of electricity. are brought into contact and then seperated, the two
Aqueous solutions of electrolytes are also conductors. bodies may or may not have equal charges.
The materials which do not allow electric charge to flow through · If the two bodies are identical the charges on the two
them are called nonconductors or insulators. will be equal.
For example, most plastics, rubber, non-metals (except graphite), · If the two bodies are not identical, the charges will be
dry wood, wax, mica, porcelain, dry air etc., are insulators. different.
· The potential of the two bodies will always be the same.
Free eBooks on @neetquestionpaper2020

534 Physi cs
(iii) By induction : Charging a body without bringing it in (iv) Charge is unaffected by motion. This is also called charge
contact with a charged body is called charging by invariance with motion
induction. Mathematically, (q)at rest = (q)in motion
A +++ – – B ++ (v) Quantisation of charge. A charge is an aggregate of small
+ – + unit of charges, each unit being known as fundamental or

+ ++ –– ++ – elementary charge which is equal to e = 1.6 × 10–19 C. This
e
principle states that charge on any body exists as integral
mutliple of electronic charge.
First rearrangement of charge takes place in metal rod B. i.e. q = ne where n is an integer.
When the rod B is connected to earth, electrons flow from According to the concept of quantisation of charges, the
earth to the rod B thus making it -vely charged charge q cannot go below e. On macroscopic scale, this is
The magnitude of elementary positive or negative charge as good as taking limit q0 ® 0.
(electron) is same and is equal to 1.6 × 10–19 C. Quantisation of electric charge is a basic (unexplained) law
of nature. It is important to note that there is no analogous
Properties of Electric Charge law of quantisation of mass.
(i) Similar charges repel and dissimilar charges attract each Recent studies on high energy physics have indicated the
other. presence of graphs with charge 2e/3, e/3. But since these
cannot be isolated and are present in groups with total charge,
F therefore the concept of elementary charge is still valid.
+ + + + + + + + COULOMB’S LAW
F Attractive The force of attraction or repulsion between two point charges
ive

+ F force (q1 and q2) at finite separation (r) is directly proportional to


F
Repuls

+
the product of charges and inversely proportional to the square
force

+
+ of distance between the charges and is directed along the line
joining the two charges.
qq 1 q1q2 q1 q2
i.e., F µ 1 2 or F = . 2
In rare situation you may find similar charged bodies r2 4pe r r
attracting each other. Suppose a big positive charged body where e is the permittivity of medium between the charges.
is placed near a small positively charged body then because If e 0 is the permittivity of free space, then relative permittivity of
of induction, opposite charge produced on the small body
medium or dielectric constant (K), is given by
makes it to attract the other body.
e
(ii) A charged body attracts light uncharged bodies, due to er ( or K ) =
polarisation of uncharged body. e0
Wall The permittivity of free space

---
e 0 = 8.86 ´ 10-12 C 2 N -1m -2
+
Positively 1 1
+ and = = 9 × 109 Nm2 C –2.
charged + 4pe 0 4 ´ 3.14 ´ 8.86 ´ 10- 2
balloon
- +
Fig : When a positively charged balloon is placed in
Also
1
4 pe 0
= 1 in CGS system of unit.

contact with the wall, an opposite charge is induced with Coulomb’s law may also be expressed as
the wall, the balloon stick to the wall due to electrostatic 1 qq
F= . 1 2
attraction 4pe0 e r r 2
(iii) Charge is conserved i.e., the charge can neither be created Let F0 be the force between two charges placed in vacuum then
nor be destroyed but it may simply be transferred from one
1 q1q2
body to the other. F0 = .
Thus we may say that the total charge in the universe is 4pe 0 r 2
constant or we may say that charges can be created or r
destroyed in equal and opposite pair. For example
® e-
g (Energy ³ 1.02MeV) ¾¾ + e+ Medium with dielectric
Electron Positron • •
(Pair-production process) +q1 constant K –q2
· Positron is an antiparticle of electron. It has same mass
as that of electron but equal negative charge. æ 1 q1q2 e ö
= K çèQ F = 4pe . r 2 and e = K ÷ø
F0
Hence
e- + e+ ¾
¾® g (Pair-annihilation process) F 0
Free eBooks on @neetquestionpaper2020

Electric Charges and Fields 535


Therefore we can conclude that the force between two
Coulomb's law is valid if r ³ 10 -15 m and if charges
charges becomes 1/K times when placed in a medium of
dielectric constant K. are point charges
The value of K for different media
Medium Dielectric Cons tant (K) FORCE FOR CONTINUOUS CHARGE DISTRIBUTION
A ir 1.006 A small element having charge dq is considered on the body. The
force on the charge q1 is calculated as follows :
Vacuu m 1.00
ur 1 q1dq
W ater 1.00026 d F1 = . rˆ
M ica 3 to 6 4pÎ0 r 2
M etals ¥
r r
Now the total force F1 is calculated by integrating dF1 under
Dielectric : A dielectric is an insulator. It is of two types - proper limits.
(i) Polar dielectric and r r 1 q1dq q1 dq
(ii) Non-polar dielectric. i.e., F1 = ò dF1 = ò 2
rˆ = ò rˆ
4p Î0 r 4p Î0 r
Significance of Permittivity Constant or Dielectric Constant :
Permittivity constant is a measure of the inverse degree of where r̂ is a variable unit vector which points from each dq,
permission of the medium for the charges to interact. towards the location of charge q1 (where dq is a small charge
Dielectric strength : The maximum value of electric field that can element)
be applied to the dielectric without its electric breakdown is called Types of Charge Distribution
its dielectric strength.
(i) Volume charge distribution : If a charge, Q is uniformly
Difference between electrostatic force and gravitational force : distributed through a volume V, the charge per unit volume
Electrostatic force Gravitational force r (volume charge density) is defined by

1. Much stronger 1. Much weaker as compared Q


r= ; r has unit coulomb/m3.
to electrostatic force V
2. Can be attractive 2. Only attractive . . .
. . .
or repulsive dQ = rdV . .. dq
. .. .
3. Depends on the nature of 3. Does not depend on the . .. ..
Total charge is
medium between charges nature of medium . uniformly distributed
. .V
in a cube
between masses

Both electric and gravitational forces follow inverse (ii) Surface charge distribution : If a charge Q is uniformly
distributed on a surface of area A, the surface charge density
square law. s , is defined by the following equation
Vector Form of Coulomb’s Law : Q
s = , s has unit coulomb / m2
uuur 1 q1q 2 uur 1 q1q 2 A
F12 = r12 = ˆr12
4pe 0 k r3 4pe 0 k r 2
Total charge Q, which
SUPERPOSITION PRINCIPLE FOR DISCRETE CHARGE is uniformly
DISTRIBUTION : FORCE BETWEEN MULTIPLE dQ = s dA distributed over disc
CHARGES
r13
r12 q2 q3 (iii) Linear charge distribution : If a charge q is uniformly
q5 distributed along a line of length l, the linear charge density
q4 l, is defined by
q1
q6 Q
O q7 l = , l has unit coloumb/m.
Discrete charge l
distribution r17 dQ = ldl
r
The electric force F1 on q1 due to a number of charges placed in
air or vacuum is given by
dl
r 1 n q1 q i 1 é q1 q 2 q1 q 3 ù Total charge Q uniformly
F1 = å 2 rˆ1i = ê 2 rˆ12 + 2 rˆ13 + ......ú Q distributed in a tube
4pe 0 i =1 r1i 4pe 0 êë r 12 r13 úû
Free eBooks on @neetquestionpaper2020

536 Physi cs
l If the charge is non uniformly distributed over a volume, where m is linear charge density i.e., m = Q / l.
surface, or line we would have to express the charge l+ a l+a
densities as q µ
so, F = ò dF = ò . dx
4p Îo x 2
dQ dQ dQ a a
r= , s= , l= ,
dV dA dl l+ a
qµ 1
where dQ is the amount of charge in a small volume, surface =
4p Î0 ò x2
dx newton
or length element. a
l In general, when there is a distribution of direct and
Keep in Memory
continuous charge bodies,we should follow the following
steps to find force on a charge q due to all the charges : 1. When the distance between the two charges placed in
(1) Fix the origin of the coordinate system on charge q. vacuum or a medium is increased K-times then the force
(2) Draw the forces on q due to the surrounding charges between them decreases K2-times. i.e., if F0 and F be the
considering one charge at a time. initial and final forces between them, then
(3) Resolve the force in x and y-axis respectively and
Fo
find SFx and SFy F=
K2
2. When the distance between the two charges placed in
(4) The resultant force is F = (SFx )2 + (SFy )2 and the
vacuum or a medium is decreased K-times then the force
between them increases K2-times. i.e., if Fo and F be the
SFx initial and final forces then
direction is given by tan q = .
SFy F = K2Fo
Calculation of electric force in some situations : 3. When a medium of dielectric constant K is placed between
(a) Force on one charge due to two other charges : the two charges then the force between them decreases by
Resultant force on q due to q1 and q2 are obtained by K-times. i.e., if Fo and F be the forces in vacuum and the
vector addition of individual forces medium respectively, then
Fo
q1 F=
K
4. When a medium of dielectric constant K between the
charges is replaced by another medium of dielectric constant
F1 F K' then the force decreases or increases by (K/K') times
according as K' is greater than K or K' is less than K.
+q q Example 1.
F2 q2
If we supply a charge to a soap bubble then it will expand.
r r r Why?
F = F1 + F2 Solution :
r Since we know like charges repel and try to get away from
r r
F = | F1 |2 + | F2 |2 each other which is at outer surface of the conductor. So a
soap bubble expand.
r
| F1 |
The direction of F is given by tan q = r Example 2.
| F2 |
Calculate the net charge on a substance consisting of
(b) Force due to linear charge distribution : (a) 5 × 1014 electrons (b) a combination of 7 × 1013
Let AB is a long (length l) thin rod with uniform protons and 4 × 1013 electrons.
distribution of total charge Q. Solution :
dx a (a) The charge of one electron is –1.6×10–19C. So net
•q charge on a substance consisting of 5 × 1014 electrons
A dQ B
is
x
5 × 1014 × (–1.6 × 10–19C) = -8 ´ 10 -5 C = -80mC.
We calculate force of these charges i.e. Q on q which
(b) Similarly the net charge on a substance consisting of a
is situated at a distance a from the edge of rod AB.
combination of 7 × 1013 protons and 4 × 1013 electron
Let, dQ is a small charge element in rod AB at a distance
is
x from q .
The force on q due to this element will be [7 ´ 1013 ´ (1.6 ´ 10 -19 C)] + [ 4 ´ 1013 (-1.6 ´ 10 -19 - C]
= + 4.8 mC.
q dQ q mdx
dF = = (Q the charge on one proton is + 1.6 × 1019C)
4pe o x 2 4pe o x 2
Free eBooks on @neetquestionpaper2020

Electric Charges and Fields 537


Example 3. 1 1
Two protons in a molecule is separated by a distance q= i.e. th of the magnitude of either unit charge.
4 4
3 × 10–10 m. Find the electrostatic force exerted by one Stability : If q is displaced slightly towards A, force of
proton on the other. attraction due to A exceeds the force of attraction due to B.
Solution : Therefore, q will get displaced further towards A. Hence
the equilibrium of q is unstable.
3 ´ 10 - 10 m
However, if q is displaced in a direction ^ to A, net force
–19 –19
1.6 × 10 C 1.6 × 10 C would bring q back to its normal position. Therefore, the
According to coulomb’s law, the electrostatic force F equilibrium will be stable.
between two charges q1 and q2, which are seperated by Example 6.
distance r is Four identical point charges each of magnitude q are
placed at the corners of a square of side a. Find the net
1 q q electrostatic force on any of the charge.
´ 1 2
F=
4pe o r2 Solution :
Here, q1 = q2 = 1.6 × 10–19C , r = 3 × 10–10 m Let the concerned charge be at C then charge at C will
experience the force due to charges at A, B and D. Let these
(1.6 ´ 10-19 ) 2
r r r
so, F = 9 ´ 109 = 2.56 ´ 10 -9 C (Repulsive) forces respectively be FA , FB and FD thus forces are given as
-20
9 ´ 10
A Bq
Example 4. q y
When a piece of polythene is rubbed with wool, a charge
of 2 × 10–7 C is developed on polythene. What is the
amount of mass, which is transferred to polythene?
Solution : Cq
FD x
D
q q
No. of electrons transferred, n = FA
e FB
æ qö
Mass transferred = m e ´ n = m e ´ ç ÷ r 1 q2 q2 æ î ĵ ö÷
è eø FA = along AC = ç -
4pe 0 AC 2
4pe 0 2a 2 ç 2 2 ÷ø
è
æ 2 ´10 -7 ö
= 9.1´10 -31 ´ ç ÷ r 1 q2 q2
ç 1.6 ´10 -19 ÷ FB = along BC = ( - ĵ)
è ø 4pe 0 BC 2 4πε 0 a 2
= 11.38 ´10 -19 kg
r 1 q2 q2
Example 5. FD = along DC = (î )
Two negative charges of unit magnitude each and a 4pe 0 DC 2 4πε 0 a 2
positive charge q are placed along a straight line. At what r r r r
position and for what value of q will the system be in Fnet = FA + FB + FD
equilibrium? Check whether it is stable, unstable or q2 éæ 1 ö æ 1 öù
neutral equilibrium? = êî çç + 1÷÷ - ĵ çç + 1÷÷ú
Solution :
4 πε 0 a 2
êë è 2 2 ø è2 2 øúû
Let the charge + q be held at a distance x1 from unit negative r æ 1 ö q2 æ1 ö q
2
charge at A, and at a distance x2 from unit negative charge Fnet = 2ç + 1÷ = ç + 2 ÷
ç ÷ 4pe a 2 è2 ø 4pe 0 a 2
at B. è2 2 ø 0

–1 +q –1 Example 7.
Electric force between two point charges q and Q at rest is
A B F. Now if a charge – q is placed next to q what will be the
x1 x2 (a) force on Q due to q (b) total force on Q ?
q (-1) q (-1) Solution :
For equilibrium of q, =
4 p e o x12 4 p e o x 22 (a) As electric force between two body interaction, i.e., force
\ x1 = x2 i.e. q must be equidistant from A and B. between two particles, is independent of presence or
For equilibrium of unit negative charge at B. absence of other particles, the force between Q and q
Force on B due to charge at A + force on B due to q = 0 will remain unchanged, i.e., F.
(b) An electric force is proportional to the magnitude of
(-1) (-1) q (-1)
2
+ =0 charges, total force on Q will be given by
4 p e o ( x1 + x 2 ) 4 p e o x 22 F¢ Qq ¢ q ¢ 0
= = = = 0 [as q' = q + (– q) = 0]
1 -(-q) F Qq q q
Þ = (Q x1 = x 2 )
2
4 p e o (2 x 2 ) 4 p e o x 22 i.e., the resultant or total force on Q will be zero.
Free eBooks on @neetquestionpaper2020

538 Physi cs

16.1
Solve following problems with the help of above text and 10. A positive point charge Q is brought near an isolated
examples. metal cube then
1. When a body is charged by induction, then the body (a) the cube becomes negatively charged.
(a) becomes neutral (b) the cube becomes positively charged.
(b) does not lose any charge (c) the interior becomes positively charged and the
(c) loses whole of the charge on it
surface becomes negatively charged.
(d) loses part of the charge on it
(d) the interior remains charge free and the surface gets
2. If a body is positively charged, then it has
nonuniform charge distribution.
(a) excess of electrons
11. A large nonconducting sheet M is given a uniform charge
(b) excess of protons
(c) deficiency of electrons density. Two uncharged small metal rods A and B are placed
(d) deficiency of neutrons near the sheet as shown in figure. Then
M
3. On charging by conduction, mass of a body may
(a) increase
A B
(b) decreases
(c) increase or decrease
(d) None of these
4. Coulomb’s law is true for
(a) atomic distances (= 10–11 m)
(b) nuclear distances (= 10–15 m) (a) M attracts A (b) M attracts B
(c) charged as well as uncharged particles (c) A attracts B (d) All of the above
(d) all the distances 12. What happens when some charge is placed on a soap
5. Two charges are placed a certain distance apart. A metallic bubble?
sheet is placed between them. What will happen to the (a) Its radius decreases (b) Its radius increases
force between the charges? (c) The bubble collapses (d) None of these
(a) It will increase 13. Quantisation of charge implies
(b) It will decrease (a) charge cannot be destroyed
(c) It will remain unchanged (b) charge exists on particles
(d) Either (a) or (b) (c) there is a minimum permissible charge on a particle
6. Which of the following is best insulator? (d) charge, which is a fraction of a coulomb is not
(a) Carbon (b) Paper possible.
(c) Graphite (d) Ebonite 14. Two charges are placed a certain distance apart in air. If a
7. The distinction between conductors, insulators and glass slab is introduced between them, the force between
semiconductors is largely concerned with them will
(a) their ability to conduct current (a) increase (b) decrease
(b) the type of crystal lattice
(c) remain the same (d) be zero
(c) binding energy of their electrons
15. A positively charged rod is brought near an uncharged
(d) relative widths of their energy gaps
conductor. If the rod is then suddenly withdrawn, the charge
8. A clyindrical conductor is placed near another positively
left on the conductor will be
charged conductor. The net charge acquired by the
cylindrical conductor will be (a) positive (b) negative
(a) positive only (c) zero (d) cannot say
(b) negative only 16. Two identical metal spheres A and B are supported on
(c) zero insulating stands and placed in contact. What kind of
(d) either positive or negative charges will A and B develop when a negatively charged
9. Two spheres A and B of exactly same mass are given ebonite rod is brought near A?
equal positive and negative charges respectively. Their (a) A will have a positive charge and B will have a
masses after charging negative charge
(a) remains unaffected (b) A will have a negative charge and B will have a
(b) mass of A > mass of B positive charge
(c) mass of A < mass of B (c) Both A and B will have positive charges
(d) Nothing can be said (d) Both A and B will have negative charges
Free eBooks on @neetquestionpaper2020

Electric Charges and Fields 539

17. The ratio of electric force between two electrons to two 19. The dielectric constant of a metal is
protons separated by the same distance in air is (a) ¥ (b) 0
(a) 100 (c) 1 (d) None of these
(b) 106 20. A metallic particle having no net charge is placed near a
(c) 104 finite metal plate carrying a positive charge. The electric
(d) None of these force on the particle will be
18. Out of gravitational, electrostatic, vander waal and (a) Towards the plate (b) Away from the plate
nuclear forces, which are able to provide attractive force (c) Parallel to the plate (d) Zero
between two neutrons? 21. If a body is charged by rubbing, its weight
(a) Electrostatic and gravitational (a) remains precisely constant
(b) Electrostatic and nuclear (b) increases slightly
(c) Vander wall and nuclear (c) decreases slightly
(d) Nuclear and gravitational (d) may increase slightly or may decrease slightly

ANSWER KEY

1. (b) 2. (c) 3. (c) 4. (d) 5. (b) 6. (d) 7. (d) 8. (c) 9. (c) 10. (d) 11. (d) 12. (b)
13. (d) 14. (b) 15. (c) 16. (a) 17. (a) 18. (d) 19. (a) 20. (d) 21. (d)

ELECTRIC FIELD uur uur uur uur


E = E1 + E2 + E3 + ........
The space around an electric charge, where it exerts a force on
another charge is an electric field. where E1 , E 2 , E 3 ,........ are the electric field intensities due to
Electric force, like the gravitational force acts between the bodies
charges q1, q2, q3 ......respectively.
that are not in contact with each other. To understand these
forces, we involve the concept of force field. When a mass is ELECTRIC LINES OF FORCE
present somewhere, the properties of space in vicinity can be These are the imaginary lines of force and the tangent at any
considered to be so altered in such a way that another mass point on the lines of force gives the direction of the electric field
brought to this region will experience a force there. The space at that point.
where alteration is caused by a mass is called its Gravitational Properties of Electric Lines of Froce
field and any other mass is thought of as interacting with the (i) The lines of force diverge out from a positive charge and
field and not directly with the mass responsible for it. converge at a negative charge. i.e. the lines of force are
Similarly an electric charge produces an electric field around it always directed from higher to lower potential.
so that it interacts with any other charges present there. One
reason it is preferable not to think of two charges as exerting
forces upon each other directly is that if one of them is
changed in magnitude or position, the consequent change in + –
the forces each experiences does not occur immediately but
takes a definite time to be established. This delay cannot be
understood on the basis of coulomb law but can be explained
by assuming (using field concept) that changes in field travel (ii) The electric lines of force contract length wise indicating
with a finite speed. (» 3 × 108 m / sec). unlike charges attract each other and expand laterally
indicating like charges repel each other.
Electric field can be represented by field lines or line of force.
The direction of the field at any point is taken as the direction of
the force on a positive charge at the point.
Electric field intensity due to a charge q at any position F
r +2q –q
( r ) from that chargeur is defined as
uur F q0
E =
q0 r
ur
uur F 1 q (iii) The number of lines that originate from or terminate on a
E (r) = Limit = ˆr
q0 ® 0 q 0 4pe r 2 q charge is proportional to the magnitude of charge.
O
| q1 | N1
where F is the force experienced by a small positive test charge i.e., | q | = N
2 2
q0 due to charge q. Its SI unit is NC–1. It is a vector quantity.
(iv) Two electric lines of force never intersect each other.
If there are more charges responsible for the field, then
Free eBooks on @neetquestionpaper2020

540 Physi cs
(v) They begin from positive charge and end on negative charge The net field strength due to entire charge distribution is given
i.e., they do not make closed loop (while magnetic field by
lines form closed loop). uur 1 dq ^
4pe ò r 2
E = r

where the integration extends over the entire charge distribution.


+ –

Electric field intensity due to a point charge q, at a


distance (r1 + r2) where r1 is the thickness of medium of dielectric
(vi) Where the electric lines of force are
constant K1 and r2 is the thickness of medium of dielectric constant
(a) close together, the field is strong (see fig.1)
K2 as shown in fig. is given by
(b) far apart, the field is weak (see fig.2)
r1 r2
Electric field lines
q K2
Strong field K1
1 q
Fig. 1 E=
(
4 p Î0 r K + r K
)
2
1 1 2 2

Weak field
CALCULATION OF ELECTRIC FIELD INTENSITY FOR
Fig. 2
A DISTRIBUTION OF DIRECT AND CONTINOUS
CHARGE
(vii) Electric lines of force generate or terminate at charges
/surfaces at right angles. 1. Fix origin of the coordinate system where electric field
intenstiy is to be found.
- 2. Draw the direction of electric field intensity due to the
- -
surrounding charges considering one charge at a time.
-
3. Resolve the electric field intensity in x and y-axis
- -
- - respectively and find SEx and SEy
-
- - 4. The resultant intensity is E = (SE x ) 2 + (SE y ) 2
-
SE y ur
d and tan q = where q is the angle between E and
SE x
Fixed point charge
near infinite metal plate
x-axis.
5. To find the force acting on the charge placed at the origin,
Electric Field for Continuous Charge Distribution : the formula F = qE is used.
If the charge distribution is continuous, then the electric field Energy density : Energy in unit volume of electric field is called
strength at any point may be calculated by dividing the charge 1
into infinitesimal elements. If dq is the small element of charge energy density and is given by u = eo E 2 ,
r 2
within the charge distribution, then the electric field dE at point where E = electric field and eo= permitivity of vacuum
P at a distance r from charge element dq is Electric Field due to Various Charge Distribution :
uur 1 dq ^ (i) Electric Field due to an isolated point charge
dE = r;
4pe r 2
q x E
Non conducting sphere (dq is small charge element) E=k
2
x +q P
r (ii) A circular ring of radius R with uniformly distributed charge
Qx Q
E =k
+
R P 2 2 3/2
(R + x )
Continuous R E
charge Q
distribution When x >> R, E = k 2 x P
x
[The charge on ring behaves
dq = ldl (line charge density) as point charge] +
= s ds (surface charge density) R Q
= rdv (volume charge density) E is max when x = ± . Also Emax =
2 6 3pe 0 R 2
Free eBooks on @neetquestionpaper2020

Electric Charges and Fields 541


(iii) A circular disc of radius R with uniformly distributed (vii) Due to a solid non conducting sphere of uniformly
charge with surface charge density s distributed charges with charge density r
E
R
E R
Q x
p
x
Qx
Ecentre = 0 Ein = k
2kQ é x ù s é x ù
R3
E= ê1- ú = ê1- ú
R 2 êë x 2 + R2 úû 2e0 êë x 2 + R2 úû Esurface = k
Q
Eout
Q
=k 2
(iv) An infinite sheet of uniformly distributed charges with R2 x
surface charge density s (viii) Due to a solid non-conducting
cylinder with linear charge denisty l
2l x
Eaxis = 0, E in = k 2 ,
R
E
2l 2l R
p E surface = k , E out = k
R x

s
E= 1
2e 0 In above cases, k =
4p Î0
(v) A finite length of charge with linear charge density l
+ Keep in Memory
+ Ey
+ 1. If the electric lines of force are parallel and equally spaced,
+ b the field is uniform.
+ x 2. If E0 and E be the electric field intensity at a point due to a
a P
+ Ex point charge or a charge distribution in vacuum and in a
+ medium of dielectric constant K then
+ E = KE0
+ 3. If E and E' be the electric field intensity at a point in the two
kl kl media having dielectric constant K and K' then
Ex = [sin a + sin b] and E y = [cos a - cos b]
x x
E' K'
Special case : =
E K
p 4. The electric field intensity at a point due to a ring with
For Infinite length of charge, a = b =
2 uniform charge distribution doesn't depend upon the radius
2 kl of the ring if the distance between the point and the centre
\ Ex = and E y = 0 of the ring is much greater than the radius of the ring. The
x
(vi) Due to a spherical shell of uniformly distributed charges ring simply behaves as a point charge.
with surface charge density s 5. The electric field intensity inside a hollow sphere is zero
æ KQ ö
+

Q + but has a finite value at the surface ç = and outside it


è R 2 ÷ø
+

+ E
+

KQ
+
+ (= ; x being the distance of the point from the centre of
R x2
+
+ the sphere).
+
+ R x 6. The electric field intensity at a point outside a hollow sphere
+

(or spherical shell) does not depend upon the radius of the
Ein = 0 (x < R)
sphere. It just behaves as a point charge.
Q s 7. The electric field intensity at the centre of a non-conducting
Esurface = k = (x = R) solid sphere with uniform charge distribution is zero. At
R2 e0
other points inside it, the electric field varies directly with
the distance from the centre (i.e. E µ x; x being the distance
Q
Eout = k of the point from the centre). On the surface, it is constant
x2 but varies inversely with the square of the distance from
Free eBooks on @neetquestionpaper2020

542 Physi cs
1 Eliminating t from equation (i) & (ii),
the centre (i.e. E µ 2 ). Note that the field doesn't depend
x qE x 2
on the radius of the sphere for a point outside it. It simply y= . i.e. y µ x2.
2m u 2
behaves as a point charge. This shows that the path of charged particle in perpendicular
8. The electric field intensity at a point on the axis of non- field is parabola.
conducting solid cylinder is zero. It varies directly with the If the width of the region in which the electric field exists be l
distance from the axis inside it (i.e. E µ x). On the surface, it then
is constant and varies inversely with the distance from the (i) the particle will leave the field at E vy
1 a distance from its original path
axis for a point outside it (i.e. E µ ). q
x in the direction of field, given by vx
y
2
MOTION OF A CHARGED PARTICLE IN AN ELECTRIC qE l u
y= .
FIELD 2m u 2 l
Let a charged particle of mass m and charge q be placed in an (ii) The particle will leave the region in the direction of the
tangent drawn to the parabola at the point of escape.
uniform electric field E , then electric force on the charge particle
(iii) The velocity of the particle at the point of escape is given
is
uur uur by
F = qE
uur v = v 2x + v 2y , where v x = u and
ur qE
\ acceleration, a = (constant) qE
m vy = u y + a yt = 0 + t
(a) The velocity of the charged particle at time t is, m
qE qEt qEl æ lö
v = u + at = at = t (Particle initially at rest) = = çèQ t = ÷ø
m m mu u
qE 2
or v = t æ qE l ö
m \ v = u2 + ç ÷
1 1 qE 2 è mu ø
(b) Distance travelled by particle is S = at 2 = t (iv) The direction of the particle in which it leaves the field is
2 2 m
given by
1 q 2 E 2t 2 vy qEl qEl
(c) Kinetic energy gained by particle, K = mv 2 = tan q = = =
2 2m vx mu.u mu 2
If a charged particle is entering the electric field in perpendicular
æ qE l ö
direction. Þ q = tan -1 ç ÷
Y è mu 2 ø
Example 8.
A thin half ring of radius R = 20 cm is uniformly charged
E with a total charge q = 0.70 millicoloumb (mC). Find the
P (x, y) magnitude of electric field strength at the curvature centre
of this half ring.
O Solution :
X
U Consider a small element of length dl of the wire with centre
O, as shown in fig.
Let E = E ˆj and the particle enters the field with speed u along
+
+
x-axis.
+

qE
+

Acceleration along Y-axis, a y =


m dl R dq dEx
q X
The initial component of velocity along y-axis is zero. Hence the
+ +

O
deflection of the particle along y-axis after time t is
dE y
1 qE 2 1 dE
+

y = u yt + . t = 0 + a yt2 ; +
2 m 2 + +
The charge on this element is
1 qE 2
\y = . t …… (i) q dl q ´ (R dq) qdq
2 m dq = = =
pR pR p
Distance covered by particle in x-axis,
x = ut …… (ii) (Q acceleration ax = 0) The electric field due to the current element at O is
1 dq
dE = .
4p Î0 R 2
Free eBooks on @neetquestionpaper2020

Electric Charges and Fields 543


From symmetry, net field at O is Solution :

ò
E = d Ex 1 1
From s = u t + a t 2 , s = a t 2
2 2
(Q u = 0)
(Since the net electric field along y-axis vanishes as each
current element nulify the effect due to diametrically 2s
t=
opposite similar current element) a
+ p /2
1 dq cos q 1 q cos q dq 1 qE
=ò As s is same, t µ where a =
4 p Î0 -pò/2 p R 2
=
4 p Î0 R 2 a M

1´ q t2 a1 qe / Me Mp
= 2 2
(sin q)p- p/2/2 = = =
4 p Î0 R t1 a2 qp / Mp Me
2q (Because E is same in both cases and qe = qp in magnitude).
E= 2 2
4 p Î0 R Example 11.
2 ´ 7 ´ 10 -10 ´ 9 ´ 109
Calculate the electric field strength required to just
= support a water drop of mass 10–7 kg and having charge
22
(0.2)2 1.6 × 10–19 C.
7
Solution :
= 100 V/m
Here, m = 10–7 kg, q = 1.6 × 10–19 C
Example 9.
Step 1 : Let E be the electric field strength required to
Point charge q moves from point P to point S along the
support the water drop.
path PQRS (as shown in fig.) in a uniform electric field
E pointing co-parallel to the positive direction of X-axis. Force acting on the water drop due to electric field E is
The coordinates of the points P, Q, R and S are (a, b, 0), F = qE = 1.6 × 10–19 E
(2 a, 0, 0), (a, –b, 0) and (0, 0, 0) respectively. Weight of drop acting downward,
Y W = mg = 10–7 × 9.8 newton.
Step 2 : Drop will be supported if F and W are equal and
P opposite.
E i.e., 1.6 × 10–19 E = 9.8 × 10–7
S Q
9.8 ´ 10 -7
or E=
R 1.6 ´ 10 -19
= 6.125 × 1012 N C–1.
The workdone by the field in the above case is given by
Example 12.
the expression
Can a metal sphere of radius 1cm hold a charge of
(a) q E A (b) – q E A
1 coulomb.
2 2 Solution :
(c) q E A 2 (d) q E [(2 a) + b ]
Solution : (b) Electric field at the surface of the sphere.
r r r r KQ
W = F.S = q E i .S E=
r r r R2
Now WP®Q= q E i · (a i + b j ) = q E a 9 ´ 109 ´ 1
r r r =
WQ®R = q E i · (– a i + b j ) = – q E a (1 ´ 10-2 )2
(workdone against field is taken as negative) V
r r r = 9 × 1013
WR®S = q E i · (– a i + b j ) = – q E a m
This field is much greater than the dielectric strength of air
W = WP ® Q + WQ+ R + WR ®S (3 × 106 v/m), the air near the sphere will get ionised and
= q E a - q E a - q E a = -q E a charge will leak out. Thus a sphere of radius 1 cm cannot
hold a charge of 1 coulomb in air.
Example 10. Example 13.
An electron of mass Me, initially at rest, moves through a
A thin non-conducting ring of radius R has linear charge
certain distance in a uniform electric field in time t1. A
proton of mass Mp also initially at rest, takes time t2 to density l = l0 cos f , where l0 is a constant, f is the
move through an equal distance in this uniform electric azimuthal angle. Find the magnitude of the electric field
field. Neglecting the effect of gravity, find the ratio t2/t1. strength at the centre of the ring.
Free eBooks on @neetquestionpaper2020

544 Physi cs
Solution : Electric field along x-axis due to this element
The situation is shown in figure. dEX = dE cos f
The half ring on the right hand side will be positive while on 1 Rl0 cos f df
= ´ ´ cos f
the half left side will be negative. The reason being that 4pe 0 R2
cos f for first and fourth quadrants is positive while for 2nd l0
and 3rd quadrants is negative. = (cos2 f df )
4pe 0 R
l 0 é1 + cos 2f ù
=
4pe 0 R êë 2 ú df
û
Electric field due to positive part along x-axis,
+p / 2
h l0 é1 + cos 2f ù
E1 =
4pe 0 R ò ê
ë 2 ú df
û
- p /2
l0 p l
or E1 = ´ = 0
4pe 0 R 2 8e 0 R
Similarly, the electric field due to negative charge along
Consider a small element dx of the ring. x-axis
Here dx = R cos f l0
Charge on small element dq = l = dx = l0 cos f (R df) E2 = 8e R
0
1 Rl0 cos f df l0 l0 l0
\ dE = ´
4pe 0 R2 \ Enet = E1 + E2 = 8e R + 8e R = 4e R
0 0 0

16.2
Solve following problems with the help of above text and 5. A hollow sphere of charge does not have electric field at
examples : (a) outer point (b) interior point
1. Figure shows the electric lines of force emerging from a (c) beyond 2 m (d) beyond 100 m
charged body. If the electric field at A and B are EA and EB 6. If one penetrates a uniformly charged solid sphere, the
respectively and if the displacement between A and B is r, electric field E
then (a) increases
(b) decreases
(c) is zero at all points
A r B (d) remains same at the surface
7. If one penetrates a uniformly charged spherical cloud,
electric field strength
(a) EA > EB (b) EA < EB (a) decreases directly as the distance from the centre
(c) EA = EB/r (d) EA = EB/r2 (b) increases directly as the distance from the centre
2. An electric field can deflect (c) remains constant
(a) neutrons (b) X-rays (d) None of these
(c) g-rays (d) a-particles 8. Electric lines of force about a negative point charge are
3. If an electron has an initial velocity in a direction different (a) circular anticlockwise
from that of an electric field, the path of the electron is (b) circular clockwise
(a) a straight line (b) a circle (c) radial, inwards
(c) an ellipse (d) a parabola (d) radial, outwards
4. If a charge is moving along the direction of coulomb’s 9. In a region with a uniform electric field, the number of
force of an electric field, lines of force per unit area is E. If a spherical metallic
(a) work is done by the electric field conductor is placed in the area, the field inside the
(b) energy is used from some outside source conductor will be
(c) strength of field decreases (a) zero (b) E
(d) energy of the system is decreased (c) more than E (d) less than E
Free eBooks on @neetquestionpaper2020

Electric Charges and Fields 545

10. Figure shows some of the electr ic field lines 13. An electron and a proton are situated in a uniform electric
corresponding to an electric field. The figure suggests field. The ratio of their acceleration will be
that (a) zero
(b) unity
(c) ratio of masses of proton and electron
A B C
(d) ratio of masses of electron and proton
14. Electric lines of force
(a) exist everywhere
(b) exist only in the immediate vicinity of electric
charges
(a) EA > EB > EC (b) EA = EB = EC
(c) exist only when both positive and negative charges
(c) EA = EC > EB (d) EA = EC < EB
are near one another
11. A point charge q is rotated along a circle in the electric
(d) are imaginary
field generated by another point charge Q. The work
done by the electric field on the rotating charge in one 15. A thin, metallic spherical shell contains a charge Q on it.
complete revolution is A point charge q is placed at the cente of the shell and
another charge q1 is placed outside it as shown in figure.
(a) zero
All the three charges are positive. The force on the charge
(b) positive
at the centre is
(c) negative
(d) zero if the charge Q is at the centre and nonzero Q
otherwise.
12. The electric charge, in uniform motion, produces q q1
(a) electric field only (b) magnetic field only
(c) both ‘a’ and ‘b’ (d) neither ‘a’ nor ‘b’ (a) towards left (b) towards right
(c) upward (d) zero

ANSWER KEY

1. (a) 2. (d) 3. (d) 4. (a) 5. (b) 6 (b) 7. (a) 8. (c) 9. (a) 10. (c) 11. (a) 12. (c)
13. (c) 14. (d) 15. (d)

ELECTRIC DIPOLE q2 q2

Two equal and opposite charges separated by a finite distance


constitutean electric dipole. If –q and +q are charges at distance
W= ò dW = ò pE sin q dq
q1 q1
2l apart, then dipole moment, 2l If q1 = 0 i.e., equilibrium position, then
p = q ´ 2l
–q +q q2
Its SI unit is coulomb metre.
Its direction is from –q to +q. It is a vector quantity.
W= ò pE sin qdq = pE(1 - cos q2 )
0
The torque t on a dipole in uniform electric field as shown in
uur uur Workdone in rotating an electric dipole in uniform electric field
figure is given by, t = qE ´ 2l sin q = p ´ E from q1 to q2 is W = pE (cosq1 – cosq2)
So t is maximum, when dipole is ^ to field & minimum (=0) when Potential energy of an electric dipole in an electric field is,
dipole is parallel or antiparallel to field.
U = – p .E i.e. U = –pE cosq
If p = p x î + p y ˆj + p z k̂ and E = E x î + E y ˆj + E z k̂
r r
where q is the angle between E and p .
+q
qE
ˆj We can also write
î k̂ 2l
Then t = p x py pz U = px Ex + p yEy + pz Ez
qE q
Ex Ey Ez Electric Field due to an Electric Dipole
–q E
The work done in rotating the dipole from equilibrium through (a) Along the axial line (or end-on position)
an angle dq is given by 2l
P
dW = td q = pE sin q d q E
–q +q
and from q1 ® q2, p x
Free eBooks on @neetquestionpaper2020

546 Physi cs
6. The electric field intensity at a point due to an electric dipole
p and E are parallel varies inversely with the cube of the distance of the point
1 2 px from its centre if the distance is much greater than the length
E ax = . when x >> l of the dipole.
4 p Î0 ( x 2 - l 2 )2
7. The electric field at a point due to a small dipole in end-on
(b) Along equatorial line (or broad-side on position) position is double of its value in broad side-on position,
p i.e. EEnd-on = 2EBroad side-on
E
x 8. For a small dipole, the electric field tends from infinity at a
point very close to the axis of the dipole to zero at a point at
–q l l +q
infinity.
1 p 9. The force between two dipoles increases (or decreases) by
Eeq = . 2 when x >>l K4 -times as the distance between them decreases (or
4p Î0 ( x + l 2 )3 / 2
r increases) by K-times.
r
When p and E are anti parallel then, 10. Time period of a dipole in uniform electric field is
Eax = 2 Eeq
(c) At any point (from the dipole) I
T = 2p
pE
Ey E Ex
b where I = moment of inertia of the dipole about the axis of rotation.

x p Example 14.
Calculate the electric field intensity due to a dipole of
q length 10 cm and having a charge of 500 m C at a point on
–q p +q
the axis distant 20 cm from one of the charges in air.
Solution :
1 p 1
E= 3cos 2 q + 1 ; tan b = tan q Given : q = 500 × 10–6C, a = 10 cm or a/2 = 5 × 10–2 m,
4p Î0 x 3 2 r = (20 + 5) cm = 25 × 10–2 m,
Electric field intensity due to a point charge varies inversely as p = q × a = (500 × 10–6) × (10 × 10–2) = 5 × 10–5 C-m
cube of the distance and in case of quadrupole it varies inversely The electric intensity on the axial line of the dipole is given
as the fourth power of distance from the quadrupole. by
Electric Force between Two Dipoles 1 2r p
E axial = ´
The electrostatic force between two dipoles of dipole moments 4p e o é 2ù
2
2 æaö
p1 and p2 lying at a seperation r is êr - ç ÷ ú
êë è 2 ø úû
1 6 p1 p2
F= when dipoles are placed co-axially
4p Î0 r 4 2 ´ (25 ´ 10 -2 ) ´ (5 ´ 10 -5 )
Eaxial = (9 ´ 109 ) ´
1 3 p1 p2 10 -8 [(25) 2 - (5) 2 ]2
F= when dipoles are placed perpendicular to
4p Î0 r 4 = 3.25 ´ 107 NC -1
each other. Example 15.
Keep in Memory Consider two objects of masses M and 3M seperated by a
distance l . Charge q and –q are placed on them
1. The dipole moment of a dipole has a direction from the respectively and they are lying in an electric field E.
negative charge to the positive charge. Find the angular frequency of oscillation (S.H.M.)
2. If the separation between the charges of the dipole is Solution :
increased (or decreased) K-times, the dipole moment 3Ma + 0 3
increases (or decreases) by K-times. x cm = = a
4M 4
3. The torque experienced by a dipole placed in a uniform (3M)
qE
electric field has value always lying between zero and pE,
where p is the dipole moment and E, the uniform electric a/4
q
field. It varies directly with the separation between the O
3a
charges of the dipole. 4
4. The work done in rotating a dipole in a uniform electric field
qE (M)
varies from zero (minimum) to 2pE (maximum). Also , it varies
directly with the separation between the charges of the (Q xcm is the position of centre of mass of system)
dipole. Since net force is zero, the centre of mass will not move but
5. The potential energy of the dipole in a uniform electric field the dipole will rotate about the centre of mass due to torque
always lies between +pE and –pE. t = qE (a / 4) sin q + qE (3a / 4) sin q
Free eBooks on @neetquestionpaper2020

Electric Charges and Fields 547


t = q Ea sin q …… (1)

d 2q +q
Also, t = Icma = – I cm …… (2) –q
dt 2
For small oscillations sin q = q 2cm 2mm
2
d q
Þ I cm = - qEa sin q = –qEaq;
dt 2 Line charge
From eq. (1) & (2), Solution :
We know that electric field intensity at a distance r from the
line charge of density l is given by
d 2q æ qEa ö
= -ç q l
dt2
è Icm ÷ø E=
2pe 0 r
2 2
æ aö æ 3a ö so, the field intensity on negative charge is given by
Icm = 3M ç ÷ + M ç ÷
è 4ø è 4ø 4.0 ´ 10-4
E1 = ´ (2 ´ 9 ´ 109 ) = 3.6 × 108 N/C
3Ma 2 + 9Ma 2 12Ma 2 3 0.02
= = = Ma 2 \ Force on negative charge,
16 16 4
F1 = qE1 = (2 × 10–8) (3.6 × 108) = 7.2 N
qEa 4qE 4qE This is directed towards line charge.
\ w2 = = or w =
3 3Ma 3Ma Similarly, field intensity on positive charge
Ma 2
4 4.0 ´ 10-4
E2 = ´ (2 ´ 9 ´ 109 ) = 3.27 × 108 N/C
Example 16. 0.022
An electric dipole consists of charges 2.0 × 10 –8 C Force on positive charge, F2 = qE2.
separated by distance of 2mm. It is placed near a long line \ F2 = (2 × 10–8) × (3.27 × 108) = 6.54 N (away)
charge of density 4.0 × 10–4 cm–1 as shown in figure, such So, net force F on dipole,
that the negative charge is at a distance of 2 cm from the F = F1 – F2 = (7.2 – 6.54) N = 0.66 N
line charge. Calculate the force acting on the dipole. The force is towards the line charge.

16.3
Solve following problems with the help of above text and 4. An electric dipole is kept in a non-uniform electric field. It
examples : experiences
1. Debye is the unit of (a) a force and a torque
(a) electric flux (b) electric dipole moment
(b) a force but not a torque
(c) electric potential (d) electric field intensity
2. An electric dipole will experience a net force when it is (c) a torque but no force
placed in (d) neither a force nor a torque
(a) a uniform electric field
5. An electric dipole is placed at the centre of a sphere.
(b) a non-uniform electric field
(c) both (a) and (b) Select the correct option. [more than one option may be
(d) None of these correct]
3. An electric dipole is kept in a uniform electric field. it (a) The flux of the electric field through the sphere is
experiences zero.
(a) a force and a torque
(b) The electric field is zero at every point of the sphere.
(b) a force, but no torque
(c) a torque but no force (c) The electric field is not zero anywhere on the sphere.
(d) neither a force nor a torque (d) The electric field is zero on a circle on the sphere.

ANSWER KEY

1. (b) 2. (b) 3. (c) 4. (a) 5. (a, c)


Free eBooks on @neetquestionpaper2020

548 Physi cs
ELECTRIC FLUX situation, where there is a high degree of symmetry such as
Electric flux is a measure of the number of electric field lines spherical, cylindrical etc.
passing through the surface. If surface is not open & encloses (i) The net electric flux through any closed surface depends
some net charge, then net number of lines that go through the only on the charge inside that surface. In the figures, the
surface is proportional to net charge within the surface. net flux through S is q1/eo, the net flux through S’ is (q2 +q3
For uniform electric field when the angle between area vector )/eo and the net flux, through S" is zero.
ur ur
( ) ( )
A and electric field E has the same value throughout the
uur ur q2
area, f = E × A Þ f = EA cos q
q3
q1
^n
S' S"
q S
(c)
E (a) (b)
(ii) A point charge Q is located outside a closed surface S. In
this case note that the number of lines entering the surface
For uniform electric field when the angle between the area vector equals to the number of lines leaving the surface. In other
and electric field is not constant throughout the area words the net flux through a closed surface is zero, if there
n^ is no charge inside.
E
ld
dA
c fie
tri s
ec line
El

uur uur uur uur Q


d f = E × dA Þ f = ò E × dA Þ f = ò EdA cos q = E ò dA cos q

Keep in Memory S
(iii) The net flux across surface A is zero
1. The electric flux is a scalar although it is a product of two
ur ur A
vectors E and A (because it is a scalar product of the two).
+q
2. The electric flux has values lying between –EA and +EA, –q
where E and A are the electric field and the area of cross-
section of the surface.
Q in
GAUSS'S LAW i.e., f = ò E. dS = = 0 because Qin = – q + q = 0
eo
It states that, the net electric flux through a closed surface in
vacuum is equal to 1/eo times the net charge enclosed within Applications of Gauss’s Law :
the surface. (i) To determine electric field due to a point charge :
r uur rr Qin
i.e., j = Ñ
ò E.dA = Ñò E.ndA = e o
s r
where Qin represents the net charge inside the gaussian surface E
S. Q dA
. .. . .
........................................ Gaussian
.. .................. ......... .... surface
.. ..dA.... .............. ..
n
The point charge Q is at the centre of spherical surface
.. . .. .. . .... . E
........ ........... .. ...
shown in figure. Gaussian surface and E is parallel to d A
.
.. ....... .. . . (direction normal to Gaussian surface ) at every point on
. ..........
the Gaussian surface.
Gaussian
surface ur ur Q
Closed surface of irregular shape which enclosed total charge Qin so, fc = Ñò E.dA = Ñò EdA = eo
In principle, Gauss's law can always be used to calculate the
(Q A = 4pr )
Q
electric field of a system of charges or a continuous distribution ÞE= 2
of charge . But in practice it is useful only in a limited number of 4pe o r 2
Free eBooks on @neetquestionpaper2020

Electric Charges and Fields 549


(ii) To determine electric field due to a cylindrically symmetric Solution :
charge distribution : According to Gauss’s theorem,
We calculate the electric 1
field at a distance r from a Gaussian Total electric flux f= × total enclosed charge
+ eo
uniform positive line surface
+ 1
charge of infinite length + E = ´q.
whose charge per unit l + eo
dA
length is l = constant. The + Since cube has six faces, hence electric flux linked with
flux through the plane + each face = (1/6f) = q/6eo.
+ E
Example 18.
surfaces of the Gaussian +
r The following figure shows a surface S which is enclosing
cylinder is zero, since E is dA
r –2q charge. The charge +q is kept outside the surface S.
parallel to the plane of end surface ( E is perpendicular to Calculate the net outward/ inward flux from the surface S.
r
dA ). The total charge inside the Gaussian suface is l l,
where l is linear charge density and l is the length of cylinder. –2q
Now applying Gauss’s law and noting E is parallel to d A
+q
everywhere on cylindrical surface, we find that
ur ur
fc = Ñ
ò E.dA = (flux)both ends + (flux)cylindrical surface S
Gaussian Solution :
surface
According to Gauss’s law, the net flux is
= 0 + E . 2A
1 -2 q
f= × net charge enclosed by closed surface =
Qin ll l eo eo
2E . p r l = = or E =
eo eo 2pe o r (Because +q is outside the surface S, so net flux due to +q
is zero)
Keep in Memory Example 19.
In which of the following figures, the electric flux is
1. The closed imaginary surfaces drawn around a charge are maximum?
called Gaussian surfaces.
2. If the flux emerging out of a Gaussian surface is zero then it
is not necessary that the intensity of electric field is zero. q q
q
3. In the Gauss's law,
ur uuur Q
Ñò E. dA = Îin0 A B C
Solution :
ur
E is the resultant electric field due to all charges lying According to Gauss’s law, the electric flux linked with a
inside or outside the Gaussian surface, but Qin is the charge closed surface depends only on net charge enclosed by
lying only inside the surface. that surface. It does not depend on the shape and size of
that closed surface. Hence electric flux linked in above three
4. The net flux of the electric field through a closed surface figures are same i.e., fA = fB = fC.
due to all the charges lying inside or outside the surface is
equal to the flux due to the charges only enclosed by the
Example 20.
surface.
A point charge +q is placed at mid point of a cube of side
5. The electric flux through any closed surface does not ‘L’. What is the electric flux emerging from the cube ?
depend on the dimensions of the surface but it depends
Solution :
only on the net charge enclosed by the surface.
According to Gauss’s law,
Example 17. q
fnet closed surface =
eo
A charge q is enclosed in a cube. What is the electric flux
associated with one of the faces of cube? Net charge enclosed by closed surface
=
eo
Free eBooks on @neetquestionpaper2020

550 Physi cs

16.4
Solve following problems with the help of above text and electrons leaving it. On closing the switch, the flux of the
examples : electric field through the closed surface
1. Positive electric flux indicates that electric lines of force [more than one option may be correct]
are directed S
(a) is increased
(a) outwards (b) inwards
(c) either (a) or (b) (d) None of these (b) is decreased
Cell
2. Negative electric flux indicates that electric lines of force
(c) remains unchanged
are directed
(a) outwards (b) inwards (d) remains zero
(c) either (a) or (b) (d) None of these 7. Figure below shows a closed surface which intersects a
3. If the flux of the electric field thorugh a closed surface is conducting sphere. If a positive charge is placed at the
zero, then [more than one option may be correct] point P, the flux of the electric field through the closed
(a) the electric field must be zero everywhere on the surface
surface
(b) the electric field may be zero everywhere on the
surface P
(c) the charge inside the surface must be zero
(d) the charge in the vicinity of the surface must be zero Conducting
Closed sphere
4. Electric flux over a surface in an electric field may be
surface
(a) positive (b) negative
(a) will remain zero (b) will become positive
(c) zero (d) All of the above
(c) will become negative (d) will become undefined
5. Mark the correct option(s).
8. Figure shows a charge q placed at the centre of a
(a) Gauss’s law is valid only for symmetrical charge hemisphere. A second charge Q is placed at one of the
distributions. positions A, B, C and D. In which position(s) of this second
(b) Gauss’s law is valid only for charge placed in charge, the flux of the electric field through the hemisphere
vacuum. remains unchanged?
(c) The electric field calculated by Gauss’s law is the [more than one option may be correct]
field due to the charges inside the Gaussian surface.
(d) The flux of the electric field through a closed surface D B
due to all the charges is equal to the flux due to the
charges enclosed by the surface.
6. A closed surface S is constructed around a conducting
wire connected to a battery and a switch. As the switch is C q A
closed, the free electrons in the wire start moving along
the wire. In any time interval, the number of electrons (a) A (b) B
entering the closed surface S is equal to the number of (c) C (d) D

ANSWER KEY

1. (a) 2. (b) 3. (b, c) 4. (d) 5. (d, c) 6. (c, d) 7. (b) 8. (a, c)


Free eBooks on @neetquestionpaper2020

Electric Charges and Fields 551

Very Short / Short Answer Questions


2 5 2
15. Four point charges qA = C , qB = - C , qC = C and qD
1. Sketch the electric lines of force due to (i) q > 0 and 4 4 4
(ii) q < 0 5
= - C are located at the corners of a square ABCD of side
2. How does the force between two point charges change if 4
the dielectric constant of the medium in which they are 10 cm. What is the force on charge placed at the centre of
kept increases? the square?
3. There are two charges +1mC and +5mC. What is the ratio 16. An electric dipole is kept in a uniform electric field. Derive
of the forces acting on them? an expression for the net torque acting on it and wirte its
4. The surface density on the copper sphere is s . What will direction. State the conditions under which the dipole is in
be the electric field strength on the surface of the spheres? (i) stable and (ii) unstable equilibrium.
5. Derive an expression for the electric field intensity at an [Delhi Board - 2012 COMPTT.]
equatorial point of an electric dipole. 17. Two charged conducting spheres of radii r 1 and r2 are
6. An electric dipole is placed at rest in a uniform electric connected to each other by a wire. Find the ratio of electric
field, and released. How will it move? fields at the surface of the two spheres.
7. Distinguish between a dielectric and a conductor? [Delhi Board - 2013]
[Delhi Board - 2012 COMPTT.] Multiple Choice Questions
8. What is the direction of the electric field at the surface of
a charged conductor having charge density s < 0? 18. An electric charge, + Q is placed on the surface of a solid,
[Delhi Board - 2012 COMPTT.]. conduction sphere of radius a. The distance measured from
9. Why is the electric field inside a hollow charged spherical the centre of the sphere is denoted as r. Then
conductor zero? [Delhi Board - 2012 COMPTT.] (a) the charge gets distributed uniformly through the
10. Why do the electric field lines not form closed loops? volume of the sphere
[Outside Delhi - 2010 COMPTT.] (b) the electrostatic potential has the same value for r < a
11. A small metal sphere carrying charge + Q is located at the (c) an equal and opposite charge gets induce in the
centre of a spherical cavity inside a large uncharged bottom half of the sphere
metallic spherical shell as shown in the figure. Use Gauss's
(d) the electric field is given by 1/(4p Î0 r2 ) for r < a
law to find the expressions for the electric field at points
P1 and P2. [Outside Delhi - 2012 COMPTT.] 19. A dielectric sphere is placed in a uniform electric field
Metal directed along the positive y-axis. Which one of the
following represents the correct equipotential surface?
+Q
P1
Free y
space (a)
P2
r x
12. An electric dipole of dipole moment P, is placed in a
r y
uniform electric field E . Deduce the expression for the
r (b)
torque t acting on it.[Outside Delhi - 2012 COMPTT.]
x
Long Answer Questions
y
13. Define electric flux. A sphere S1 of radius r1 encloses a (c)
charge Q. If there is another concentric sphere S 2 of radius
r2 (r 2 > r1) and there is no additional charges between S1 x
and S2, find the ratio of electric flux through S1 and S2. y
14. State Gauss theorem in electrostatics and write its
mathematical form. Using it, derive an expression for electric
field at a point near a thin infinite plane sheet of electric (d)
x
charge. How does this electric field change with a uniformly
thick sheet of charge?
Free eBooks on @neetquestionpaper2020

552 Physi cs
20. A charge q is placed at O in the cavity in a spherical A B C D E
uncharged conductor. Point S is outside the conductor. (a) + – + 0 +
If the charge is displaced from O towards S, still remaining (b) + – + + 0
within the cavity, (c) + – + 0 0
(d) – + – 0 0
22. The figure shows two charged particles on x-axis. The
particles are free to move. However at one point, a third
charged particle can be placed such that all three particles
would be in equilibrium.
1. Is the third particle positively or negatively charged?
(a) electric field at S will increase 2. Is the point the third particle is placed to the left, right
(b) electric field at S will decrease or between the two charged?
(c) electric field at S will first increase and then decrease
(d) electric field at S will not change
21. Five styrofoam balls are suspended from insulating threads.
Several experiments are performed on the balls and the (a) negative, to their right
following observations are made (b) positive, between them
(i) ball A repels C and attracts B (c) negative, to their left
(ii) ball D attracts B and has no effect on E (d) positive, to their right
(iii) a negatively charged rod attracts both A and E. 23. An electric dipole, consisting of two opposite charges of
An electrically neutral styrofoam ball gets attracted if placed 2 × 10–6 C each separated by a distance 3 cm is placed in an
nearby a charged body due to induced charge. What are electric field of 2 × 105 N/C. Torque acting on the dipole is
the charges, if any, on each ball ? (a) 12 × 10–1 N – m (b) 12 × 10–2 N – m
//////////////// //////////////// //////////////// (c) 12 × 10–3 N – m (d) 12 × 10–4 N – m
24. The electric field strength at a distance r from a charge q is
E. What will be electric field strength if the distance of the
observation point is increased to 2 r?
//////////////// //////////////// (a) E/2 (b) E/4
A B C
(c) E/6 (d) None of the above
25. The surface density on the copper sphere is s. The electric
field strength on the surface of the sphere is
(a) s (b) s/2
D E (c) s / 2eo (d) s / eo
Free eBooks on @neetquestionpaper2020

Electric Charges and Fields 553

1. If a charge q is placed at the centre of the line joining two 7. The electric potential V at any point (x, y, z), all in meters in
equal charges Q such that the system is in equilibrium then space is given by V = 4x2 volt. The electric field at the point
the value of q is [CBSE PMT 1995] (1, 0, 2) in volt/meter is [CBSE-PMT 2011M]
(a) Q/2 (b) –Q/2 (a) 8 along positive X-axis (b) 16 along negative X-axis
(c) Q/4 (d) –Q/4 (c) 16 along positive X-axis (d) 8 along negative X-axis
2. The electric intensity due to a dipole of length 10 cm and 8. Three charges, each +q, are placed at the corners of an
having a charge of 500 mC, at a point on the axis at a distance isosceles triangle ABC of sides BC and AC, 2a. D and E are
20 cm from one of the charges in air, is the mid points of BC and CA. The work done in taking a
[CBSE PMT 2001] charge Q from D to E is [CBSE-PMT 2011M]
(a) 6.25 × 107 N/C (b) 9.28 × 107 N/C 3qQ
(a) A
(c) 13.1 × 1011 N/C (d) 20.5 × 107 N/C 8p Î0 a
r
3. If a dipole of dipole moment p is placed in a uniform electric qQ E
r (b) 4 p Î0 a
field E , then torque acting on it is given by
[CBSE PMT 2001] (c) zero
r rr r r r 3qQ B C
(a) t = p.E (b) t = p ´ E (d) D
4 p Î0 a
r r r r r r
(c) t = p + E (d) t = p - E 9. Two metallic spheres of radii 1 cm and 3 cm are given
4. Two positive ions, each carrying a charge q, are separated charges of –1×10–2 C and 5×10–2 C, respectively. If these
by a distance d. If F is the force of repulsion between the are connected by a conducting wire, the final charge on the
ions, the number of electrons missing from each ion will be bigger sphere is [CBSE-PMT 2012M]
(e being the charge of an electron) [CBSE-PMT 2010] (a) 2 × 10–2 C (b) 3 × 10–2 C
(c) 4 × 10–2 C (d) 1 × 10–2 C
4pe0 Fd 2 4pe0 Fe 2 10. An electric dipole of moment ´p´ is placed in an electric
(a) (b)
e2 d2 field of intensity ´E´. The dipole acquires a position such
that the axis of the dipole makes an angle q with the direction
4pe0 Fd 2 4pe 0 Fd 2 of the field. Assuming that the potential energy of the dipole
(c) (d) to be zero when = 90°, the torque and the potential energy
e2 q2
of the dipole will respectively be [CBSE-PMT 2012S]
5. A square surface of side L meter in the plane of the paper is
(a) p E sin q, – p E cos q (b) p E sin q, –2 p E cos q
placed in a uniform electric field E (volt/m) acting along the
(c) p E sin q, 2 p E cos q (d) p E cos q, – p E cos q
same plane at an angle q with the horizontal side of the
11. Two pith balls carrying equal charges are suspended from
square as shown in Figure. The electric flux linked to the
a common point by strings of equal length. The equilibrium
surface, in units of volt. m, is [CBSE-PMT 2010]
separation between them is r. Now the strings are rigidly
E clamped at half the height. The equilibrium separation
between the balls now become [NEET 2013]
q

y
y/2
(a) EL2 (b) EL2 cos q

(c) EL2 sin q (d) zero æ r ö æ 2r ö


6. A charge Q is enclosed by a Gaussian spherical surface of (a) ç3 ÷ (b) ç ÷
è 2ø è 3ø
radius R. If the radius is doubled, then the outward electric
flux will [CBSE-PMT 2011S] 2
æ 2r ö æ r ö
(a) increase four times (b) be reduced to half (c) ç 3 ÷ (d) ç ÷
(c) remain the same (d) be doubled è ø è 2ø
Free eBooks on @neetquestionpaper2020

554 Physi cs
12. A charged particle q is placed at the centre O of cube of 17. A thin spherical shell of radus R has charge Q spread
length L (A B C D E F G H). Another same charge q is uniformly over its surface. Which of the following graphs
placed at a distance L from O. Then the electric flux through most closely represents the electric field E(r) produced by
ABCD is [AIEEE 2002] the shell in the range 0 £ r < ¥, where r is the distance from
E F the centre of the shell? [AIEEE 2008]
D
C E(r)
O
q q
H
G
A
B (a)
L r
O R
(a) q /4pÎ0L (b) zero
(c) q/2 pÎ0L (d) q/3pÎ0 L
13. If the electric flux entering and leaving an enclosed surface E(r)
respectively is f1 and f2, the electric charge inside the
surface will be [AIEEE 2003]
(a) (f2 + f2) × eo (b) (f2 – f2) × eo
(c) (f1 + f2) × eo (d) (f2 – f1) × eo (b) r
O R
14. Three charges –q1 , +q2 and –q3 are place as shown in the
fig. The x - component of the force on –q1 is proportional to
E(r)
[AIEEE 2003]

–q3
(c) r
O R

E(r)
–q1 +q2
q2 q3 q2 q3
(a) - cos q (b) + sin q
2 2
a2
2 (d) r
b a b
O R
q2 q3 q2 q3
(c) + cos q (d)
sin q -
b 2
a2 b a2
2 18. A charge particle ‘q’ is shot towards another charged particle
‘Q’ which is fixed, with a speed ‘v’. It approaches ‘Q’ upto
15. A charged ball B hangs from a silk thread S, which makes an a closest distance r and then returns. If q were given a
angle q with a large charged conducting sheet P, as shown speed of ‘2v’ the closest distances of approach would be
in the figure. The surface charge density s of the sheet is [AIEEE 2009]
proportional to [AIEEE 2005] (a) r/2 (b) 2 r
(c) r (d) r/4
P 19. Four charges equal to -Q are placed at the four corners of a
q square and a charge q is at its centre. If the system is in
S equilibrium the value of q is [AIEEE 2009]
Q Q
(a) - (1 + 2 2 ) (b) (1 + 2 2 )
2 4
B
(a) cot q (b) cos q Q Q
(c) - (1 + 2 2 ) (d) (1 + 2 2 )
(c) tan q (d) sin q 4 2
16. An electric dipole is placed at an angle of 30° to a 20. A charged oil drop is suspended in a uniform field of 3×104
non-uniform electric field. The dipole will experience V/m so that it neither falls nor rises. The charge on the drop
[AIEEE 2006] will be (Take the mass of the charge = 9.9 × 10–15 kg and
g = 10 m/s2) [AIEEE 2009]
(a) a translational force only in the direction of the field –18
(a) 1.6 × 10 C
(b) a translational force only in a direction normal to the
direction of the field (b) 3.2 × 10–18 C
(c) a torque as well as a translational force (c) 3.3 × 10–18 C
(d) a torque only (d) 4.8 × 10–18 C
Free eBooks on @neetquestionpaper2020

Electric Charges and Fields 555


(a) Statement-1 is false, Statement-2 is ture
Q
21. Let P (r ) = r be the charge density distribution for a (b) Statement-1 is true, Statement-2 is false
pR 4 (c) Statement-1 is ture, Statement-2 is true, Statement-2 is
solid sphere of radius R and total charge Q. For a point ‘p’ the correct explanation for statement-1
inside the sphere at distance r1 from the centre of the sphere, (d) Statement-1 is ture, Statement-2 is true, Statement-2 is
the magnitude of electric field is [AIEEE 2009] not the correct explanation of Statement-1
Q Qr12 25. In an uniformly charged sphere of total charge Q and radius
(a) (b) R, the electric field E is plotted as function of distance from
4p Î0 r12 4p Î0 R 4
the centre. The graph which would correspond to the above
Qr12 will be [AIEEE 2012]
(c) (d) Zero
3p Î0 R 4 E E
22. A thin semi-circular ring of radius r has a positive charge q
ur
distributed uniformly over it. The net field E at the centre
(a) (b)
O is [AIEEE 2010]
j R r R r
E E

i (c) (d)
O
q ˆj q ˆj R r R r
(a) 2 2 (b) - 2 2
4p e 0 r 4p e 0 r 26. Two charges, each equal to q, are kept at x = – a and x = a on
q ˆ q ˆj q
(c) - 2 2 j (d) thex-axis. A particleof massm and charge q0 = is placed
2p e 0 r 2p e 0 r 2
2
2
23. Let there be a spherically symmetric charge distribution at the origin. If charge q0 is given a small displacement
æ5 rö (y <<a) along the y-axis, the net force acting on the particle
with charge density varying as r(r ) = r0 çè - ÷ø upto is proportional to [JEE Main 2013]
4 R
(a) y (b) –y
r = R , and r(r ) = 0 for r > R , where r is the distance from
1 1
the origin. The electric field at a distance r(r < R) from the (c) (d) –
origin is given by [AIEEE 2010] y y

r0 r æ 5 r ö 4pr0 r æ 5 r ö 27. Two point charges +q and –q are held fixed at (–d, 0) and
(a) ç - ÷ (b) ç - ÷ (d, 0) respectively of a x – y coordinate system. Then
4e 0 è 3 R ø 3e 0 è 3 R ø
[IIT JEE 1995 S]
4r0 r æ 5 r ö r0 r æ 5 r ö (a) the electric field E at all points on the axis has the same
(c) çè - ÷ø (d) ç - ÷
4ε 0 4 R 3ε 0 è 4 R ø direction
24. This question has statement-1 and statement-2 of the four (b) work has to be done in bringing a test charge from ¥
choices given after the statements, choose the one that to the orgin
best describes the two statements. (c) electric field at all points on y-axis is along x-axis
An insulating solid sphere of radius R has a uniform positive (d) the dipole moment is 2qd along the x-axis
charge density r. As a result of this uniform charge 28. Six charges of equal magnitude, 3 positive and 3 negative
distribution, there is a finite value of electric potential at the are to be placed on PQRSTU corners of a regular hexagon,
centre of the sphere, at the surface of the sphere and also at such that field at the centre is double that of what it would
a point outside the sphere. The electric potential at infinite have been if only one +ve charge is placed at R. What is
is zero. [AIEEE 2012] charge configuration? [IIT JEE 2004 S]
Statement-1 : When a charge q is taken from the centre of
P Q
the surface of the sphere its potential energy changes by
qr U O R
3e 0 .
Statement-2 : The electric field at a distance r ( r < R) from T S
rr
(a) + , +, +, –, –, – (b) –, + , +, +, –, –
the centre of the sphere is 3e . (c) –, + , +, –, +, – (d) + , –, +, –, +, –
0
Free eBooks on @neetquestionpaper2020

556 Physi cs
29. A Gaussian surface in the figure is shown by dotted line. 33. A disk of radius a / 4 having a uniformly distributed charge
The electric field on the surface will be [IIT JEE 2004 S] 6 C is placed in the x - y plane with its centre at (–a / 2, 0, 0).
A rod of length a carrying a uniformly distributed charge
8 C is placed on the x - axis from x = a /4 to x = 5a / 4. Two
q1 q2 point charges – 7 C and 3 C are placed at (a / 4, – a /4, 0) and
(– 3a /4, 3a / 4, 0), respectively. Consider a cubical surface
–q1
formed by six surfaces x = ± a / 2, y = ± a / 2, z = ± a / 2. The
electric flux through this cubical surface is [IIT-JEE 2009]
(a) due to q1 and q2 only y
(b) due to q2 only
(c) zero
(d) due to all x
30. Three infinitely long charge sheets are placed as shown in
figure. The electric field at point P is [IIT JEE 2005 S]
–2 C 2C
Z (a) (b) e0
e0
s 10 C 12 C
Z = 3a (c) (d)
e0 e0
P
r
-2s 34. Consider an electric field E = E0 xˆ where E0 is a constant.
Z=a
The flux through the shaded area (as shown in the figure)
x due to this field is [IIT-JEE 2011]
-s z
Z = -a
(a,0,a) (a,a,a)
2s 2s
(a) k̂ (b) - k̂
e0 e0
4s 4s y
(d) -
k̂ k̂ (0,0,0) (0,a,0)
(c) x
e0 e0
31. A spherical portion has been removed from a solid sphere (a) 2E0a2 (b) 2E0 a 2
having a charge distributed uniformly in its volume as shown E0a 2
in the figure. The electric field inside the emptied space is (c) E0 a2 (d)
2
[IIT JEE 2007]
35. A spherical metal shell A of radius RA and a solid metal
sphere B of radius RB(<RA) are kept far apart and each is
given charge ‘+Q’. Now they are connected by a thin metal
wire. Then [IIT-JEE 2011]
(a) E inside
A =0 (b) QA > QB
s A RB
(a) zero everywhere (c) = (d) E on
A
surface on surface
< EB
s B RA
(b) non-zero and uniform
36. Which of the field patterns given below is valid for electric
(c) non-uniform field as well as for magnetic field? [IIT-JEE 2011]
(d) zero only at its center
32. Consider a neutral conducting sphere. A positive point
charge is placed outside the sphere. The net charge on the (a) (b)
sphere is then [IIT JEE 2007]
(a) negative and distributed uniformly over the surface
of the sphere
(b) negative and appears only at the point on the sphere
closest to the point charge
(c) negative and distributed non-uniformly over the entire (c) (d)
surface of the sphere
(d) zero
Free eBooks on @neetquestionpaper2020

Electric Charges and Fields 557


37. Which of the following statement(s) is/are correct? 39. An infinitely long solid cylinder of radius R has a uniform
[IIT-JEE 2011] volume charge density r. It has a spherical cavity of radius
(a) If the electric field due to a point charge varies as r –2.5 R/2 with its centre on the axis of the cylinder, as shown in
instead of r –2, then the Gauss law will still be valid. the figure. The magnitude of the electric field at the point P,
(b) The Gauss law can be used to calculate the field which is at a distance 2R from the axis of the cylinder, is
distribution around an electric dipole. 23rR
given by the expression . The value of k is
(c) If the electric field between two point charges is zero 16Ke 0
somewhere, then the sign of the two charges is the [IIT-JEE 2012]
same.
(d) The work done by the external force in moving a unit
positive charge from point A at potential VA to point B
at potential VB is (VB - VA).
38. A cubical region of side a has its centre at the origin.
It encloses three fixed point charges, –q at (0, –a/4, 0), +3q
at (0, 0, 0) and –q at (0, +a/4, 0). Choose the correct
options(s) [IIT-JEE 2012]
z
a

–q
–q 3q (a) 6 (b) 5
(c) 7 (d) 4
x
40. Two non-conducting solid spheres of radii R and 2R, having
(a) The net electric flux crossing the plane x = +a/2
uniform volume charge densities r1 and r2 respectively,
is equal to the net electric flux crossing the plane
touch each other. The net electric field at a distance 2R from
x = –a/2
the centre of the smaller sphere, along the line joining the
(b) The net electric flux crossing the plane y = +a/2 is r
more than the net electric flux crossing the plane centres of the spheres, is zero. The ratio 1 can be
r2
y = –a/2.
[JEE Advance 2013]
q
(c) The net electric flux crossing the entire region is e 32
0 (a) –4 (b) -
(d) The net electric flux crossing the plane z = +a/2 is 25
equal to the net electric flux crossing the plane
32
x = +a/2. (c) (d) 4
25
Free eBooks on @neetquestionpaper2020

558 Physi cs

1. The electric field strength at a distance r from a charge q is 11. There is an electric field E in X-direction. If work done in
E. What will be electric field strength if the distance of the moving a charge 0.2 C through a distance of 2m along a line
observation point is increased to 2 r? making an angle of 60 degree with X-axis is 4.0 joule, what
(a) E/2 (b) E/4 is the value of E?
(c) E/6 (d) None of these
(a) 3 newton per coulomb
2. The surface density on the copper sphere is s. The electric
field strength on the surface of the sphere is (b) 4 netwon per coulomb
(a) s (b) s/2 (c) 5 newton per coulomb
(c) Q / 2eo (d) Q / eo (d) None of these
3. What is the angle between the electric dipole moment and 12. A particle of mass m and charge q is placed at rest in a
the electric field due to it on the axial line? uniform electric field E and then released. The kinetic energy
(a) 0º (b) 90º attained by the particle after moving a distance y is
(c) 180º (d) None of these (a) q E y2 (b) q E2 y
4. An infinite parallel plane sheet of a metal is charged to
(c) q E y (d) q2 E y
charge density s coulomb per square metre in a medium of
dielectric constant K. Intensity of electric field near the 13. If electric field in a region is radially outward with magnitude
metallic surface will be E = Ar, the charge contained in a sphere of radius r centred
at the origin is
s s
(a) E= (b) E=
eo K 2e o 1
(a) A r3 (b) 4peoAr 3
s Ks 4 p eo
(c) E= (d) E =
2e o K 2e o
1 A 4 p eo A
5. In a region of space having a uniform electric field E, a (c) (d)
hemispherical bowl of radius r is placed. The electric flux f 4 p e o r3 r3
through the bowl is
14. If a charge is moved against the coulomb force of an electric
(a) 2pRE (b) 4pR2E
2
field, then
(c) 2pR E (d) pR2E
(a) work is done by the electric field
6. A cylinder of radius R and length l is placed in a uniform
electric field E parallel to the axis of the cylinder. The total (b) energy is used from some outside source
flux over the curved surface of the cylinder is (c) the strength of the field is decreased
(a) zero (b) pR2E (d) the energy of the system is decreased
(c) 2pR E 2 (d) E / pR2 15. A hollow insulated conduction sphere is given a positive
7. Two thin infinite parallel sheets have uniform surface charge of 10 mC. What will be the electric field at the centre
densities of charge + s and – s. Electric field in the space of the sphere if its radius is 2 metres?
between the two sheets is (a) Zero (b) 5 mCm–2
(a) s/eo (b) s/2eo (c) 20 mCm–2 (d) 8 mCm–2
(c) 2s/eo (d) zero
16. An electric dipole has the magnitude of its charge as q and
8. What is the value of E in the space outside the sheets?
its dipole moment is p. It is placed in uniform electric field E.
(a) s/eo (b) s/2eo
If its dipole moment is along the direction of the field, the
(c) E ¹ 0 (d) 2s/eo
force on it and its potential energy are respectively.
9. An electric dipole when placed in a uniform electric field E
(a) q.E and max. (b) 2 q.E and min.
will have minimum potential energy if the dipole moment
makes the following angle with E. (c) q.E and min (d) zero and min.
(a) p (b) p/2 17. If the dipole of moment 2.57 × 10–17 cm is placed into an
(c) zero (d) 3 p/2 electric field of magnitude 3.0 × 104 N/C such that the fields
10. At the centre of a cubical box + Q charge is placed. The lines are aligned at 30° with the line joining P to the dipole,
value of total flux that is coming out a wall is what torque acts on the dipole?
(a) Q / eo (b) Q / 3eo (a) 7.7 × 10–13 Nm (b) 3.855 × 10–13 Nm
(c) Q / 4eo (d) Q / 6eo (c) 3.855 × 10–15 Nm (d) 7.7 × 10–15 Nm
Free eBooks on @neetquestionpaper2020

Electric Charges and Fields 559


18. An electric dipole is placed at an angle of 30° with an electric 25. Identify the wrong statement in the following. Coulomb's
field of intensity 2 × 105 NC–1, It experiences a torque of law correctly describes the electric force that
4 Nm. Calculate the charge on the dipole if the dipole length (a) binds the electrons of an atom to its nucleus
is 2 cm. (b) binds the protons and neutrons in the nucleus of an
(a) 8 mC (b) 4 mC atom
(c) 8 mC (d) 2 mC (c) binds atoms together to form molecules
19. Charge Q is distributed to two different metallic spheres (d) binds atoms and molecules together to form solids
having radii R and 2R such that both spheres have equal 26. Two particle of equal mass m and charge q are placed at a
surface charge density, then charge on large sphere is distance of 16 cm. They do not experience any force. The
4Q Q q
(a) (b) value of is
5 5 m
3Q 5Q
(c) (d) pe0
5 4 (a) l (b)
20. Two point charges q1 = 4mC and q2 = 9 mC are placed 20 cm G
apart. The electric field due to them will be zero on the line
joining them at a distance of G
(c) 4pe 0 (d) 4pe0 G
(a) 8 cm from q1 (b) 8 cm form q2
80 80 27. The electric field intensity just sufficient to balance the
(c) cm from q1 (d) cm from q 2
13 13 earth’s gravitational attraction on an electron will be: (given
21. Three charge q, Q and 4q are placed in a straight line of mass and charge of an electron respectively are
1
length l at points distant 0, and l respectively from one 9.1 ´10-31 kg and 1.6 ´ 10 -19 C .)
2 (a) –5.6 × 10–11 N/C (b) –4.8 × 10–15 N / C
end. In order to make the net froce on q zero, the charge Q (c) –1.6 × 10–19 N/C (d) –3.2 × 10–19 N / C
must be equal to 28. The spatial distribution of electric field due to charges
(a) –q (b) – 2q (A, B) is shown in figure. Which one of the following
-q statements is correct ?
(c) (d) q
2
22. Among two discs A and B, first have radius 10 cm and charge
10–6 µC and second have radius 30 cm and charge 10–5C. A B
When they are touched, charge on both q A and q B
respectively will, be
(a) q A = 2.75μC, q B =3.15 μC (a) A is +ve and B –ve, |A| > |B|
(b) A is –ve and B +ve, |A| = |B|
(b) q A = 1.09 μC, q B = 1.53μC
(c) Both are +ve but A > B
(c) q A = q B = 5.5μC (d) Both are –ve but A > B
(d) None of these r
29. In a medium of dielectric constant K, the electric field is E .
23. Two charges are at a distance d apart. If a copper plate of
d If Î0 is permittivity of the free space, the electric
thickness is kept between them, the effective force will displacement vector is
2
be r r
(a) F/2 (b) zero KE E
(a) (b)
Î0 K Î0
(c) 2F (d) 2F
r
24. The charge given to any conductor resides on its outer Î0 E r
(c) (d) K Î0 E
surface, because K
(a) the free charge tends to be in its minimum potential 30. A drop of oil of density r and radius r carries a charge q
energy state when placed in an electric field E, it moves upwards with a
(b) the free charge tends to be in its minimum kinetic energy velocity v. If r0 is the density of air, h be the viscosity of the
state air, then which of the following force is directed upwards?
(c) the free charge tends to be in its maximum potential (a) q E (b) 6π ηr v
energy state.
(d) the free charge tends to be in its maximum kinetic energy 4 3
(c) p r (r - r 0 )g (d) None of these
state 3
Free eBooks on @neetquestionpaper2020

560 Physi cs
31. Two conducting spheres of radii r 1 and r2 are charged to 36. A simple pendulum has a length l , mass of bob m. The bob
the same surface charge density. The ratio of electric fields is given a charge q. The pendulum is suspended between
near their surface is the vertical plates of the charged parallel plate capacitor. If
E is the field strength between the plates, then time period
(a) r12 / r22 (b) r22 / r12 T equal to
+ -
(c) r1 / r2 (d) 1 : 1
32. A charge q is placed at the centre of the open end of a
cylindrical vessel. The flux of the electric field through the
surface of the vessel is

q + -

l l
(a) 2p (b) 2p
g qE
g+
m
(a) zero (b) q/e o
(c) l (d) l
2p 2p
(c) q/2eo (d) 2q/eo qE 2
g- æ qE ö
33. ABC is an equilateral triangle. Charges +q are placed at m g2 + ç ÷
each corner as shown as fig. The electric intensity at centre èmø
O will be 37. Two identical thin rings, each of radius a meter, are coaxially
placed at a distance R meter apart. If Q1 coulomb and Q2
+q A coulomb are respectively the charges uniformly spread on
the two rings, the work done in moving a charge q coulomb
from the centre of one ring to that of the other is
r
q(Q1 - Q2 )( 2 - 1)
O (a) Zero (b)
r r 4 2 pe0 a
+q +q q 2(Q1 + Q 2 ) q(Q1 + Q 2 )( 2 + 1)
B C (c) (d)
4pe 0a 4 2pe 0 a
38. A metallic sphere is placed in a uniform electric field. The
1 q 1 q
(a) (b) line of force follow the path (s) shown in the figure as
4 p Îo r 4 p Îo r 2
1 1
2 2
1 3q
(c) (d) zero 3 3
4 p Îo r 2
4 4
34. The insulation property of air breaks down when the electric
field is 3 × 106 Vm–1. The maximum charge that can be (a) 1 (b) 2
given to a sphere of diameter 5 m is approximately (c) 3 (d) 4
39. A soap bubble is given negative charge, its radius will
(a) 2 × 10–2 C
(a) increase (b) decrease
(b) 2 × 10–3 C
(c) remain unchanged (d) fluctuate
(c) 2 × 10–4 C 40. The E-r curve for an infinite linear charge distribution will
(d) 2 × 10–5 C be
35. An electric dipole is placed along the x-axis at the origin O.
A point P is at a distance of 20 cm from this origin such that
OP makes an angle p/3 with the x-axis. If the electric field at E
(a) E (b)
P makes an angle q with the x-axis, the value of q would be

æ 3ö r r
p p
(a) (b) + tan -1 ç ÷
3 3 ç 2 ÷
è ø

æ 3ö (c) E (d) E
2p
(c) (d) tan -1ç ÷
3 ç 2 ÷
è ø r r
Free eBooks on @neetquestionpaper2020

Electric Charges and Fields 561


41. A and B are two identically spherical charged bodies which t (1) (2) (3) (4)
repel each other with force F, kept at a finite distance. A
third uncharged sphere of the same size is brought in contact
with sphere B and removed. It is then kept at mid point of A O 3p
p 2p
and B. Find the magnitude of force on C.
F F
(a) (b) Fig. 2
2 8
(a) curve (1) (b) curve (2)
(c) F (d) Zero
(c) curve (3) (d) curve (4)
42. An electric dipole of moment P is placed in a uniform 45. Force between two identical charges placed at a distance
of r in vacuum is F. Now a slab of dielectric of dielectric
electric field E such that P points along E . If the dipole contrant 4 is inserted between these two charges. If the
is slightly rotated about an axis perpendicular to the plane thickness of the slab is r/2, then the force between the
charges will become
containing E and P and passing through the centre of
3
the dipole, the dipole executes simple harmonic motion. (a) F (b) F
Consider I to be the moment of inertia of the dipole about 5
the axis of rotation. What is the time period of such 4 F
oscillation? (c) F (d)
9 2
(a) ( pE / I) (b) 2p ( I / pE) 46. A charge +q is at a distance L/2 above a square of side L.
Then what is the flux linked with the surface?
(c) 2p ( I / 2pE) (d) None of these
q 2q
43. There exists a non-uniform electric field along x-axis as (a) 4e 0 (b) 3e 0
shown in the figure below. The field increases at a uniform
rate along +ve x-axis. A dipole is placed inside the field as q 6q
shown. Which one of the following is correct for the dipole? (c) 6e 0 (d) e0
47. Positive and negative point charges of equal magnitude
–q
a æ aö æ -a ö
+q x-axis are kept at ç 0, 0, ÷ and ç 0, 0, ÷ respectively. The work
è 2ø è 2 ø
done by the electric field when another positive point charge
(a) Dipole moves along positive x-axis and undergoes a is moved from (–a, 0, 0) to (0, a, 0) is
clockwise rotation (a) positive
(b) Dipole moves along negative x-axis and undergoes a (b) negative
clockwise rotation (c) zero
(c) Dipole moves along positive x-axis and undergoes a
(d) depends on the path connecting the initial and final
anticlockwise rotation
positions
(d) Dipole moves along negative x-axis and undergoes a r
anticlockwise rotation 48. A short electric dipole of dipole moment p is placed at a
44. The electric dipole is situated is an electric field as shown distance r from the centre of a solid metallic sphere of radius
in fig 1. The dipole and electric field are both in the plane of a (<< r) as shown in the figure. The electric field intensity at
the paper. The dipole is rotated about an axis perpendicular the centre of sphere C due to induced charge on the sphere
to plane of paper passing through A in anticlockwise is
direction. If the angle of rotation (q) is measured with respect –q O +q
to the direction of electric field, then the torque (t) a
C
experienced by the dipole for different values of the angle
of rotation q will be represented in fig. 2, by
+q
r
q 1 2p
E (a) zero (b) along CO
4pe 0 r 3
1 2p 1 p
–q (c) along OC (d) along CO
Fig. 1
4pe 0 r 3 4pe0 r3
Free eBooks on @neetquestionpaper2020

562 Physi cs
49. A positive charge +Q is fixed at a point A. Another positively 53. A glass rod rubbed with silk is used to charge a gold leaf
charged particle of mass m and charge +q is projected from electroscope and the leaves are observed to diverge. The
a point B with velocity u as shown in the figure. The point B electroscope thus charged is exposed to X-rays for a short
is at the large distance from A and at distance d from the line period. Then
AC. The initial velocity is parallel to the line AC. The point (a) the divergence of leaves will not be affected
C is at very large distance from A. The minimum distance (in
(b) the leaves will diverge further
meter) of +q from +Q during the motion is d (1 + A) . Find
(c) the leaves will collapse
the value of A.
(d) the leaves will melt
[TakeQq = 4pe0mu2d and d = ( 2 - 1) meter.] 54. A solid conducting sphere of radius a has a net positive
B charge 2Q. A conducting spherical shell of inner radius b
u and outer radius c is concentric with the solid sphere and
has a net charge – Q. The surface charge density on the
d inner and outer surfaces of the spherical shell will be

A 2Q Q
C (a) - ,
2
4pb 4pc 2
+Q
(a) 3 (b) 2 a
Q Q b
(c) 4 (d) 5 (b) - 2
,
4pb 4pc 2 c
50. A solid sphere of radius R has a charge Q distributed in its
volume with a charge density r = k r a, where k and a are Q
constants and r is the distance from its centre. (c) 0,
4pc 2
If the electric field at r = R is 1 times that at r = R, find the (d) None of these
2 8
55. An uncharged sphere of metal is placed in between charged
value of a. plates as shown. The lines of force look like
(a) 2 (b) 3
(c) 5 (d) 6 +++++++
51. The electrostatic potential (fr) of a spherical symmetric
system kept at origin, is shown in the figure, and given as

q
– –––– ––
A
fr = (r ³ R0 ) fr
4pe 0 r
+++++++
q
fr = (r £ R0 )
4pe 0 R0 R0 r
Which of the following option(s) is/are correct – –––– ––
B
(a) For spherical region r £ R0 total electrostatic energy
stored is zero +++++++
(b) Within r = 2R0 total charge is q.

(c) There will be no charge anywhere except at r = R0


– –––– ––
C
(d) Electric field is discontinuous at r = R0
52. A solid sphereical conductor of radius R has a spherical +++++++
cavity of radius a (a < R) at its centre. A charge + Q is kept
at the centre. The cahrge at the inner surface, outer and at a
position r (a < r < R) are respectively
(a) + Q, – Q, 0 – –––– ––
D
(b) – Q, + Q, 0
(a) A (b) B
(c) 0, – Q, 0
(d) + Q , 0, 0 (c) C (d) D
Free eBooks on @neetquestionpaper2020

Electric Charges and Fields 563


Directions (Qs. 56 to 61) : Each question contains STATEMENT- Directions for Qs. (62 to 70) : Read the following passage(s)
1 and STATEMENT-2. Choose the correct answer (ONLY ONE carefully and answer the questions that follows:
option is correct ) from the following- PASSAGE -1
(a) Statement-1 is true, Statement-2 is true; Statement -2 is a Related to the following diagram of two charges, +Q and – 4Q.
correct explanation for Statement-1
(b) Statement-1 is true, Statement-2 is true; Statement -2 is
NOT a correct explanation for Statement-1
62. The net electric field is zero near which point?
(c) Statement-1 is true, Statement-2 is false
(a) A (b) B
(d) Statement-1 is false, Statement-2 is true
(c) C (d) D
56. Statement-1 : Consider two identical charges placed
63. At which point does the net electric field vector point to
distance 2d apart, along x-axis.
the left?
(a) A (b) B
(c) C (d) D
64. At which point would a small positive charge q feel the
greatest force?
The equilibrium of a positive test charge placed at the (a) A (b) B
point O midway between them is stable for displacements (c) C (d) D
along the x-axis. PASSAGE -2
Statement-2: Force on test charge is zero. A thin insulating wire is stretched along the diameter of an
57. Statement-1 : A deuteron and an a-particle are placed in an insulated circular loop of radius R. A small bead of mass m and
electric field. If F1 and F2 be the forces acting on them and charge –q is threaded on to the wire. Two small identical charges
a1 and a2 be their accelerations respectively then, a1 = a2. are tied to the hoop at points opposite to each other, so that the
Statement-2 : Forces will be same in electric field. diameter passing through them is perpendicular to the thread
58. Statement-1 : Four point charges q1, q2, q3 and q4 are as (see figure). The bead is released at a point which is a distance x0
shown in figure. The flux over the shown Gaussian surface from the centre of the loop. Assume that x0 << R.
depends only on charges q1 and q2.

65. The resultant force acting on the charged bead is


Statement-2 : Electric field at all points on Gaussian surface k 2 Q q x0 k 2 Q q x0
(a) F = - 2 . (b) F = 2 .
depends only on charges q1 and q2. x0 + R 2 r x0 + R 2 r
59. Statement-1 : The positive charge particle is placed in front x0
k 2Qq k 2 Q q x0
of a spherical uncharged conductor. The number of lines of (c) F = - (d) F = 2 . .
forces terminating on the sphere will be more than those -R x 20 r 2
x0 - R 2 r
emerging from it. 66. The exact equation of motion of the bead along the thread
Statement-2 : The surface charge density at a point on the is
sphere nearest to the point charge will be negative and d2x k 2Qq x d2 x kQq x
(a) m =- (b) m =-
maximum in magnitude compared to other points on the
(x ) (x )
2 2 3/2 2 3/2
dt 2
+R dt 2
+ R2
sphere.
60.. Statement-1 : A uniformly charged disc has a pin hole at its d2 x k 2 Qq x d2 x k Qq x
(c) m =- (d) m =-
centre. The electric field at the centre of the disc is zero.
(x ) (x )
2 2 3/2 2 3/2
dt 2 dt 2
-R - R2
Statement-2 : Disc can be supposed to be made up of many
67. The time when will the velocity of the bead vanish for the
rings. Also electric field at the centre of uniformly charged
first time is
ring is zero.
61. Statement-1 : When a conductor is placed in an external p2 m R3 p2 m R3
(a) (b)
electrostatic field, the net electric field inside the conductor k 2Qq k Qq
becomes zero after a small instant of time.
Statement-2 : It is not possible to set up an electric field p2 m R3 2 p2 m R 3
(c) (d)
inside a conductor. k 4Qq k Qq
Free eBooks on @neetquestionpaper2020

564 Physi cs
PASSAGE -3 69. In an electric field due to a point charge +Q a spherical
Gauss’s law relates the net flux f of an electric field through a closed surface is drawn as shown by dotted circle. The
closed surface to the net charge qen that is enclosed by that electric flux through the surface drawn is zero by Gauss law.
surface. A conducing sphere is inserted intersecting the previously
r r drawn Gaussian surface. The electric flux through the
Î0 f = qen or Î0 ò Ñ E.dA < q en surface
Charge outside the surface no matter how larger or how close it
may be is not included in the qen in Gauss law. The exact form of
location of the charge inside the Gaussian surface is also of no
concern. The electric field on the left hand side of equation,
however, is the net electric field resulting from all charge, both +Q
those inside and those outside the Gaussian surface. This may
seem to be inconsistent, but keeping in mind that the electric
field due to a charge outside the Gaussian surface contributes
zero net flux through the surface, because as many field lines due (a) still remains zero
to that charge enter the surface as leave it. (b) non-zero and positive
68. Figure shows five charged lumps of plastic. The cross- (c) non-zero and negative
section of Gaussian surface S is indicated. Assuming (d) becomes infinite
q1 = q4 = 3.1 nC, q2 = q5 = – 5.9 nC, and q3 = – 3.1 nC, the net 70. Electric field at point A depends on
electric flux through the surface is
A
q2
q1 q3 q1

q5
q4 (a) q1
q2
(b) q2
(c) both q1 and q2
(a) – 670 Nm2/C (b) + 670 Nm2/C (d) None of these
(c) – 360 Nm2/C (d) + 360 Nm2/C
Free eBooks on @neetquestionpaper2020

Electric Charges and Fields 565

15. (c) Positively charged rod induces negative charge on


Exercise 16.1
the side of the sphere closer to the rod and an equal
positive charge on the side away from the rod.When
1. (b) Charging by induction involves transfer of charges
the rod is withdrawn, the negative charges move to
from one part to the other of the body. No loss of
neutralise the equal positive charge. The charge left is
charge is involved.
zero.
2. (c) Positive charge is due to deficiency of electrons.
3. (c) On charging by conduction, body may gain mass, if it 16. (a) In contact, A and B behave as one body. Therefore,
acquires negative charge. It may lose mass, if it by induction, positive charge developes on A and
acquires positive charge. negative charge developes on farther side of B.
4. (d) Coulomb’s law is true for all distances small and large.
Fe
Hence it is called a long range force. 17. (a) = 100 = 1 , because force depends only on
Fp
F0
5. (b) Fm = . For metals K = ¥ , therefore, Fm ® 0 , in charges, distance and nature of medium.
K
the thickness of sheet. Hence force between the 18. (d) Attractive force between two neutrons may be due to
charges will decrease. gravitational forces or due to nuclear forces.
6. (d) Ebonite is the best insulator. 19. (a) Metal is a good conductor. So, its K = ¥ .
7. (d) According to band theory, distinction between 21. (d) The weight can be increased slightly, if it acquire
conductors, insulators and semiconductors is based negative charge & weight can be decreased slightly, if
on relative width of energy gaps between valence band it acquires positive charge.
and conduction band.
8. (c) Net charge acquired by induction is zero, as there is
Exercise 16.2
only transfer of electrons from one part of body to the
other.
9. (c) A loses electrons and B gains electrons. Therefore, 1. (a) Figure indicates the presence of some positive charge
mass of A < mass of B. to the left of A.
11. (d) There may be two case \ EA > EB (Q rA < rB)

M M 2. (d) An electric field can deflect only a charged particle.


+ –
+ –
3. (d) The path is a parabola, because initial velocity can be
+ – resolved into two rectangular components, one along
r r
(i) + – + – + (ii) – + – + – E and other ^ to E . The former decreases at a constant
+ – + – + – + – + –
– – –
+ – + + – rate and latter is unaffected. The resultant path is
+ – therefore a parabola.
+ –
4. (a) Work is done by the electric field because q = 0º,
12. (b) The radius of soap bubble increases because of
W = F s cos q ® positive.
outward force acting on the bubble due to charging.
5. (b) E = 0, at any point inside the sphere.
13. (d) q = ± n e shows that minimum value of q = ± 1e.
6. (b) E decreases as we move from the surface of the sphere
where e = 1.6×10–19 to its centre.
Coulomb = Charge of one electron 7. (a) A spherical cloud behaves as a solid sphere. Therefore,
1q1 q 2 Fu E decreases directly with the decreasing distance from
14. (b) Fg = 2
= the centre.
4 pÎ0 r K
8. (c) For a single negative point charge, electric lines of
As K (for glass) > 1. \ Fg < Fu force are radial and inwards.
Free eBooks on @neetquestionpaper2020

566 Physi cs
9. (a) Electric field inside a conductor is zero.
s
11. (a) Because it is moving in a equipotential surface, hence 4. E= .
e0
the work done in complete rotation is zero.
6. The dipole shall not move as net force on it is zero.
12. (c) An electric charge in uniform motion produces both
an electric and a magnetic field. 13. Flux through S2 is same.
\ Ratio is 1 : 1
13. (c) ae F / m e m p ( e E) m p
= e = = .
a m Fp / m p m e (e E ) m e 1
15. Net force on the charge C of the centre is zero.
4
14. (d) Electric lines of force are imaginary.
18. (b) 19. (b) 20. (d)
15. (d) The charge q, which is kept at the centre of metallic
spherical shell transfered to the outer surface of shell 21. (c) 22. (b) 23. (c)
& inside the shell the electric field is zero & hence 24. (b) 25. (d)
force is also zero. Exercise 2 : PAST Competition MCQs
Exercise 16.3
1. (d) Let q charge is situated at the mid position of the line
AB. The distance between AB is x. A and B be the
1. (b) Debye is the unit of electric dipole moment.
positions of charges Q and Q respectively.
2. (b) Net force (as well as torque) are experienced by the
dipole in a non uniform electric field. A C B
Q Q
x q x
3. (c) In a uniform electric field, net force = 0, but torque
2 2
¹ 0. x
4. (a) In a non uniform electric field, net force ¹ 0 and net
x x
torque ¹ 0. Let AC = , BC =
2 2
Exercise 16.4 The force on A due to charge q at C,
® 1 Q.q
1. (a) Positive electric flux is due to + charge. So lines of F CA = . along AC
force are directed outwards. 4pe 0 ( x / 2) 2
2. (b) Negative electric flux is due to negative chage. The force on A due to charge Q at B
Therefore, lines of force are directed inwards.
5. (d) Though electric flux is scalar, yet its value may be
® 1 Q2 ®
F AB = . 2 along BA
positive, negative or zero. 4pe0 x
8. (b) The positive & negative charges developed on The system is in equilibrium, then two oppositely
different parts of conducting sphare due to induction. directed force must be equal, i.e., total force on A is
Hence the flux through the closed surface is positive. equal to zero.
® ® ® ®
Positive
F CA + F AB = 0 Þ F CA = - F AB
charge
Negative
1 4Q.q –1 Q 2
+ – charge . 2 = .
+ –
– 4 pe 0 x 4 pe 0 x 2
+ –
+ –
Q
conducting Þq=-
closed sphere 4
surface 2. (a) Given : Length of the dipole (2l) = 10cm = 0.1m or
l = 0.05 m
Exercise 1 : NCERT Based Questions
Chargeon thedipole(q) = 500 mC = 500 × 10–6 C and
distance of the point on the axis from the mid-point of
F the dipole (r) = 20 + 5 = 25 cm = 0.25 m.
2. Force in a medium = Fm =
k We know that the electric field intensity due to dipole
3. The ratio of forces = 1 : 1 on the given point (E)
Free eBooks on @neetquestionpaper2020

Electric Charges and Fields 567


Q – x then
1 2(q.2l ) r
= ´ where Q = 4 × 10–2 C
4pe 0 (r 2 - l 2 ) 2
v1 = v2
2(500 ´ 10 -6 ´ 0.1) ´ 0.25 kx k (Q - x)
= 9 ´109 ´ =
2 2 2
[(0.25) - (0.05) ] 1cm 3cm
3x = Q – x Þ 4x = Q
225 ´ 10 3
= = 6.25 ´ 10 7 N / C (k = 1 for air)
3.6 ´ 10 -3 Q 4 ´ 10-2
x= = C = 1 ´ 10-2
r 4 4
3. (b) Given : Dipole moment of the dipole = p and uniform
® Q¢ = Q – x = 3 × 10– 2C
electric field = E . We know that dipole moment (p) = 10. (a) The torque on the dipole is given as
q.a (where q is the charge and a is dipole length). And
® t = PE sin q
when a dipole of dipole moment p is placed in uniform
The potential energy of the dipole in the electric field
®
electric field E , then Torque (t) = Either force × is given as
perpendicular distance between the two forces = U = – PE cos q
r r r
qaEsinq or t = pE sin q or t = p ´ E (vector form)
11. (a)
4. (c) Let n be the number of electrons missing.

1 q2
F= × 2
4pe 0 d 2 Þ q = 4pe 0 d F = ne

4pe 0 Fd 2
\ n=
e2 kq 2
Fe r/2 2
From figure, tan q = Þ = r
5. (d) Electric flux, f = EA cos q , where q mg y mg
= angle between E and normal to the surface.
kq 2
p [Q F = from Coulomb’s law]
Here q = r2
2
Þ f=0 y r' 1
Þ r3 µ y Þ r'3 µ Þ = 1/3
2 r 2
Q in
6. (c) By Gauss’s theorem, f =
Î0 r
Thus, the net flux depends only on the charge Þ r' = 3
2
enclosed by the surface. Hence, there will be no effect
12. (b) The flux for both the charges exactly cancels the effect
on the net flux if the radius of the surface is doubled.
of each other.
r é dv dv dv ˆ ù 13. (d)
7. (d) E = - ê iˆ + ˆj + k ú = –8 ´ iˆ volt/meter
ë dx dy dz û 14. (b) The force body diagram
r
\ E (1,0,2) = -8iˆ V / m
–q3
8. (c) AC = BC q
qA
VD = VE a F2
(–q1)
We have, +q2
E q F1
W = Q (VE – VD)
b
ÞW=0 q q
B D C
1 q1q3 1 q1q 2
9. (b) At equilibrium potential of both sphere becomes same F1 = . ; F2 = .
4pe o a 2 4pe o b2
if charge of sp here one x and other sphere
Free eBooks on @neetquestionpaper2020

568 Physi cs

q1 é q3 q ù 1 kQq 1 kqQ r
sin q + 2 ú 18. (d) mv2 = Þ m(2v)2 = Þ r' =
Fx = F1 sin q + F2 =
4pe o êë a 2 b2 û 2 r 2 r' 4
19. (b) Net field along AB
æq q ö
Þ Fx µ çç 3 sin q + 2 ÷÷
èa 2
b2 ø kQ ´ 2 kQ kq
= + - =0
2 2 2
a ( 2a ) æ a ö
15. (c) çè ÷
P 2ø
T cos q
q T –Q –Q B
s
F = Eq = q
e0K q
T sin q

A –Q –Q
mg

Q 2 Q Q
s Þ + = 2q Þ q = (2 2 + 1)
T sin q = .q ...(i) 1 2 4
e0 K
mg
T cos q = mg ....(ii) 20. (c) For equilibrium, qE = mg Þ q =
E
Dividing (i) by (ii),
9.9 ´ 10-15 ´ 10
sq q= = 3.3 ´ 10-18 C
tan q = 3 ´ 10 4
e0K. mg
\ s µ tan q 21. (b)
16. (c) +q R
F1
E1 x dx

F2
E2
–q

The electric field will be different at the location of Let us consider a spherical shell of thickness dx and
force on the two charges. Therefore the two charges
radius x. The volume of this spherical shell = 4px 2 dx .
will be unequal. This will result in a force as well as
The charge enclosed within shell
torque.
17. (a) The electric field inside a thin spherical shell of radius é Q.x ù 2 4Q 3
R has charge Q spread uniformly over its surface is = ê 4 ú [4px dx] = 4 x dx
ë pR û R
zero.
The charge enclosed in a sphere of radius r1 is
Q +
++ + +
r1 r1
4Q é x 4 ù
+

R 4Q Q 4
ò
+

3
= x dx = ê ú = 4 r1
+ + +
+++

Q R4 4 4
R êë úû R
0
E=k 2 0
r
++

\ The electric field at point p inside the sphere at a


+ +

E=0
distance r1 from the centre of the sphere is
+

+ + +
++
é Q 4ù
r
Q 1 êë R 4 1 úû 1 Q 2
Outside the shell the electric field is E = k 2 . These E= = r1
r 4p Î0 r12 4 p Î0 R 4
characteristics are represented by graph (a).
Free eBooks on @neetquestionpaper2020

Electric Charges and Fields 569


22. (c) Let us consider a differential element dl. charge on
this element. é 5 r3 1 r 4 ù 3æ5 r ö
j = 4pr0 ê . - . ú = pr0r ç - ÷
êë 4 3 R 4 úû è3 Rø

+
+

+
+ dl
+
1 q
+ dq \ Electric field at r, E = . 2
4p Î0 r
+
+ dE q
+
cos q
i
O 1 pr0 r 3 æ 5 r ö r0 r æ 5 r ö
dE = . ç - ÷= ç - ÷
4pÎ0 r 2 è 3 R ø 4 Î0 è 3 R ø
dE sin q
æ q ö q 24. (a) Statement 1 is dimensionally wrong while from Gauss'
dq = ç ÷ dl = (rd q) (Qdl = rd q) law,
è pr ø pr
4
æqö r. pr 3
= ç ÷dq rr
è pø E (4pr 2 ) = 3 ÞE=
e0 3e 0
Electric field at O due to dq is
gives statement 2 is correct.
1 dq 1 q 25. (c) Electric field inside the uniformly charged sphere varies
dE = . = . dq
4p Î0 r 2 4 p Î0 pr 2
kQ
The component dE cos q will be counter balanced by linearly, E =.r , (r £ R), while outside the sphere,
R3
another element on left portion. Hence resultant field it varies as inverse square of distance,
at O is the resultant of the component dE sinq only.
kQ
p
q E= ; (r ³ R ) which is correctly represented in
\ E = ò dE sin q = ò 4 p2 r 2 Î sin qd q r2
0 0 option (c).
q
=
2 2
[ - cos q]p0 = q
(+1 + 1)
y
4p r Î0 2 2
4p r Î0 – q/2
q F q q F
= 2 2 26. (a) x
2p r Î0 q q
a a
The direction of E is towards negative y-axis.
r q
\ E=- 2 2 ˆj Þ F sin q F sin q Þ Fnet = 2F cosq
2p r Î0
23. (a) Let us consider a spherical shell of radius x and 2Fcos q
thickness dx.
Charge on this shell æ qö
2kq ç ÷
è 2ø y
Fnet = .
( )
æ5 xö 2 2
dq = r.4px 2 dx = r0 ç 4 - R ÷ .4px dx y2 + a2 y2 + a2
è ø
\ Total charge in the spherical region from centre to r
(r < R ) is æ qö
2kq ç ÷ y
r è 2ø kq2 y
æ5 xö Fnet = Þ
q = ò dq = 4 pr0 ò ç - ÷ x 2 dx ( y 2 + a 2 )3/ 2 a3
0
è4 Rø
So, F µ y
dx
x 27. (c) If we take a point M on the X-axis as shown in the
figure, then the net electric field is in X-direction.
\ Option (a) is incorrect.
Free eBooks on @neetquestionpaper2020

570 Physi cs
If we take a point N on Y-axis, we find net electric us consider an arbitrary point P in the small sphere.
field along +X direction. The same will be true for any The electric field due to charge on big sphere,
point on Y-axis. (c) is a correct option.
Wµ0 = q (Vµ - V0 ) = q(0 - 0) = 0
\ (b) is incorrect. The direction of dipole moment is O
from –ve to +ve. Therefore (d) is incorrect. a
P O'
28. (c) Let us place the charges as shown on the regular
hexagon
® s ®
P Q E1 = OP
+
3e 0

O Also the electric field due to small sphere


U +R
® s ®
E2 = PO '
3e 0
+T S
\ The total electric field

s ® ® s ®
E due to each charge at O is same in magnitude, the ® ® ®
arrow shows the direction of E due to each charge E = E1 + E 2 = [OP + PO] = OO
3e0 3e0
r r r r
ER + E U = E0 where E 0 is electric field due to +ve
This will have a finite value which will be uniform.
charge placed at R
32. (d) When a positive point charge is placed outside a
29. (d) The electric field deduced by Gauss’ law is due to all conducting sphere, a rearrangement of charge takes
the charges. place on the surface. But the total charge on the sphere
is zero as no charge has left or entered the sphere.
30. (b)
net charge = 0

++
+
+
+
+q
Ù
E s away from +ve plate along – k
Ù 33. (a) y z
E –2s towards –ve plate along – k
Ù
E –s towards –ve plate along – k
Electric field due to all the three uniform surface
have same direction which add upon give x
2C
s Ù 2s Ù s Ù 4s Ù 3C –7 C
( - k) + ( - k) + ( - k) = ( - k)
2e 0 2e 0 2e 0 2e 0
From the figure it is clear that the charge enclosed in the
2s Ù
= ( - k) cubical surface is 3 C + 2 C - 7 C = -2 C. Therefore the
e0
electric flux through the cube is
Ù
in the direction opposite to k . qin -2 C
f= = [qin = -2C ]
31. (b) Let us consider a uniformly charged solid sphere e0 e0
without any cavity. Let the charge per unit volume be r
s and O be the centre of the sphere. Let us consider a 34. (c) Given E = Eo xˆ
uniformly charged sphere of negative charged density This shows that the electric field acts along + x direction
s having its centre at O'. Also let OO' be equal to a. Let and is a constant. The area vector makes an angle
Free eBooks on @neetquestionpaper2020

Electric Charges and Fields 571


of 45° with the electric field. Therefore the electric (b) is not correct
flux through the shaded portion whose area is (c) is correct as between two point charges we will get a
r r point where the electric field due to the two point
a´ 2a = 2 a 2 is f = E. A = EA cos q = E0 ( 2 a2 ) cos 45°
charges cancel out each other.
2 1
= E0 ( 2 a ) ´ = E0a2 (d) is correct when the work done is without accelerating
2 the charge.
z
a
38. (a, c, d) The electric flux passing through x = + ,
2
a a
(a,0,a) (a,a,a) q = 45° x= - , z = + is same due to symmetry..
2a 2 2
E The net electric flux through the cubical region is
q A
y - q + 3q - q q
(0,0,0) (0,a,0) =
e0 e0 .

39. (a) We suppose that the cavity is filled up by a positive


as well as negative volume charge of r. So the electric
x field now produced at P is the superposition of two
35. (a,b,c,d) electric fields.
Electric field inside a spherical metallic shell with charge on (i) The electric field created due to the infinitely
the surface is always zero. Therefore option [a] is correct. long solid cylinder is
When the shells are connected with a thin metal wire then
electric potentials will be equal, say V. rR
E1 = directed towards the +Y direction
4e 0
1 QA 1 QB
\ = =V (ii) The electric field created due to the spherical
4p Î0 RA 4p Î0 RB
negative charge density
As RA > RB therefore QA > AB. option [b] is also correct.
rR
E2 = directed towards the –Y direction.
QA 96e 0
2
s A 4 pR A R2 Q R2 4pÎ0 R A V \ The net electric field is
As = = A2 ´ A = 2B ´
sB QB RB Q B RA 4 p Î0 R B V
2
4 pR B 1 é 23rR ù
E = E1 - E 2 = ê ú
6 ë 16e0 û
s A RB
\ s = Option (c) is also correct 40. (d)
B RA
Electric field E1 due to smaller sphere at P is
sA s 4
Also E A = & EB = B r1 ´ p R 3
Î0 Î0 1 3
E1 =
4pe 0 (2R)2
E A s A RB
= = <1
EB s B RA + + + + +
\ EA < EB Option (d) is also correct + + + + + + +
+ + ® ®

36. (c) The pattern of field lines shown in option (c) is correct
+ + + + E2 P E1 + + + +
+ + + + + + + +
because + + + 2R R + + + +
+ +
(i) a current carrying toroid produces magnetic field r1
+ + + + + +
lines of such pattern + + + + +

(ii) a changing magnetic field with respect to time in r2


a region perpendicular to the paper produces
induced electric field lines of such pattern. 1 r pR rR
E1 = ´ 1 = 1
37. (c,d) 4p Î0 3 4 Î0 ´3
(a) is not correct because it is valid only when E µ r–2 Electric field E2 due to bigger sphere at P is
Free eBooks on @neetquestionpaper2020

572 Physi cs

r 2R 9. (c) U p = –p• E = –p E cos q


E2 =
3Î 0 (Up)minimum = – pE, when q = 0°
r1R r R 10. (d) According to Gauss’ Law
As E1 = E 2 \ = 2
4 Î0 ´3 3 Î0 Q enclosed by closed surface
ò E.ds = eo
= flux
r1 so total flux = Q/eo
=4
r2 Since cube has six face, so flux coming out through
option (d) is correct. one wall or one face is Q/6eo.
11. (d) Work done, W = F s cosq = (q E) s cos q
Exercise 3 : Conceptual & Applied MCQs
12. (c) K.E. acquired = work done
1. (b) As new distance = 2 r and electric field due to single = force × distance = q E × y = q E y
1 q q
charge, E µ , 13. (b) E= Þ Ar = Þ q = 4pe o Ar3
2 2 2
r 4 pe o r 4p Î0 r

therefore, new intensity = E/4. 14. (b) To overcome electrostatic repulsion forces, work will
have to be done by external agency.
2. (d) According to Gauss's theorem,
15. (a) Charge resides on the outer surface of a conducting
q é hollow sphere of radius R. We consider a spherical
E Ñò ds = Here Ñ
ò ds = 4pR ùû
2
Î0 ë surface of radius r < R.
By Gauss theorem
q / 4 pR 2 ++
++ +++
\E = [Q q / 4pR 2 = s] +
+
+
Î0 + +
+ R
+ +
+
or E = s / e o + O S +
+
r + r
E +
+
3. (a) Electric field E is along the dipole axis. +
+ +
+ +
\ q = 0º. ++
+ + ++
4. (c) Electric field intensity due to thin infinite plane sheet rr 1 1
of charge, ò
s
E.ds = ´ charge enclosed or E ´ 4pr 2 =
e0 e0
´0

s s ÞE= 0
E= =
2 e 2 e0 K i.e electric field inside a hollow sphere is zero.
e 16. (d) When the dipole is in the direction of field then net
[Where dielectric constant, K = e ] force is qE + (–qE) = 0
0
E
5. (c) f = E (ds) cos q = E (2p r 2 ) cos 0º = 2 p r 2 E .
–q a q
6. (a) For the curved surface, q = 90º
\ f = E ds cos90º = 0 . and its potential energy is minimum = – p.E.
= – qaE
s æ -s ö = s = s/e
7. (a) E = E1 - E 2 = -ç ÷ o
2 e o çè 2 e o ÷ø 2 eo

The field intensity in between sheets having equal 17. (b) ( æ


è )
Nö æ 1ö
t = 2.57 ´ 10-17 Cm ç 3.0 ´ 104 ÷ ç ÷
C ø è 2ø
and opposite uniform surface densities of charge
becomes constant. ie, an uniform electric field is = 3.855 ´ 10 -13 Nm.
produced and it is independent of the distance between
the sheets. r r r
18. (d) Torque, t = p ´ E = pE sin q
8. (c) Inside a charged conducting surface E = 0, but on or 4 = p × 2 × 105 × sin 30°
outside the surface E ¹ 0.
\ Electric intensity is discontinuous across a charged 4
or, p = 5
= 4 ´ 10-5 Cm
conducting surface. 2 ´ 10 ´ sin 30°
Free eBooks on @neetquestionpaper2020

Electric Charges and Fields 573


Dipole moment, p = q × l 25. (b) Nuclear force binds the protons and neutrons in the
nucleus of an atom.
p 4 ´10-5 r r
q= = = 2 ´ 10-3 C = 2mC 26. (d) They will not experience any force if | FG |=| Fe |
l 0.02
19. (a) Let q and q' be the charges on spheres of radii R and m2 1 q2 q
2R respectively. ÞG = . Þ = 4pe 0 G
(16 ´ 10 -2 )2 -
4pe 0 (16 ´ 10 )
2 2 m
Given q + q' = Q …(i)
Surface charge densities are 27. (a) – eE = mg

q' uur 9.1 ´ 10 -31 ´ 10


q E =- -11
= -5.6 ´10 N / C
s= s=
2 and 4p (2R) 2 1.6 ´ 10 -19
4pR
Given s = s' 28. (a) Since lines of force starts from A and ends at B, so A is
+ve and B is –ve. Lines of forces are more crowded near
q q' A, so A > B.
\ = or, q' = 4q
2
4 pR 4 p(2R) 2
29. (d) Electric displacement vector, D = eE
From eq. (i), q' = Q – q or, 4q = Q – q
or, Q = 5q …(ii) As, e = e 0 K \ D = e0 K E

Q 4Q 30. (a) Force due to electric field (F = q E) acts upwards.


\ q' = Q – q = Q - = .
5 5 31. (d) As s1 = s 2
20. (a) EP = 0
Q1 Q2 Q1 Q2
\ 2
= 2 or =
4 9 4 p r1 4 p r2 4 p e o r12 4 p e o r22
Þ =
2 2
x (20 - x)
\ E1 = E 2 or E1 / E 2 = 1 Þ E : E 2 = 1:1
20 - x 3 32. (a) The flux is zero according to Gauss’ Law because it is
Þ =
x 2 a open surface which enclosed a charge q.
Þ 40 – 2x = 3x 33. (d) Unit positive charge at O will be repelled equally by
Þ x = 8 cm three charges at the three corners of triangle.
r
21. (a) (Fnet )q = 0 By symmetry, resultant E at O would be zero.

kQ E ´ r 2 3 ´ 106 ´ (2.5)2
Qq 4q 2 34. (b) E= ÞQ= =
Þ k +k =0 r2 k 9 ´ 109
ælö
2
l2
ç ÷
è2ø = 2 ´ 10-3 C

l/2 l/2 35. (b) From figure, q = p + a , where


q Q 4q 3

1 E 2 æ p sin q ö æ 4p Î0 r 3 ö 1
where k = tan a = =ç ÷ç ÷ = tan q
4pe0 E1 è 4 p Î0 r 3 ø è 2p cos q ø 2

Þ 4Qq + 4q2 = 0 æ pö 1 p 3
tan ç q - ÷ = tan =
Þ Q=–q è 3ø 2 3 2
22. (c) The charge on disc A is 10–6 mC. The charge on disc p æ 3ö
or q- = tan -1 ç ÷
B is 10 × 10–6 mC.
The total charge on both = 11 mC. 3 è 2 ø
When touched, this charge will be distributed equally
æ 3ö
i.e. 5.5 mC on each disc. or a = tan -1 ç ÷
ç 2 ÷
23. (b) The dielectric constant for metal is infinity, the force è ø
between the two charges would be reduced to zero. p æ 3ö
or q= + tan -1ç ÷
24. (a) 3 ç 2 ÷
è ø
Free eBooks on @neetquestionpaper2020

574 Physi cs
E E
a 1 1 q2 1 q2
E2 p/3 = [2 - 1] = =F
4pe 0 r 2 4pe 0 r 2
P
42. (b) The dipole experiences a torque pE sin q tending to
bring itself back in the direction of field.
p/3
–q O
+q Therefore, on being released (i.e. rotated) the dipole
36. (d) 37. (b) 38. (d) oscillates about an axis through its centre of mass and
39. (a) For a soap bubble (electrified) perpendicular to the field. If I is the moment of inertia
of the dipole about the axis of rotation, then the
4T
excess pressure = P = Pi - P0 = equation of motion is I.d 2 q / dt 2 = -pE sin q
r
T = surface tension, r = radius. For small amplitude sin q » q
Force due to excess pressure balances surface tension.
Thus d 2 q / dt 2 = -(pE / I).q = -w 2 q
When bubble is charged
4T where w = ( pE / I) .
Pexcess = Pelectrosta tic +
r
This is a S.H.M., whose period of oscillation is
s2 1 q2
Pelectrostatic =- =- T = 2p / w = 2p ( I / pE) .
2e 0 2e 0 16p 2 r 4
43. (d) The dipole is placed in a non-uniform field, therefore a
4é q2 ù
Pexcess = ê T - 2 3
ú ....... ( A ) force as well as a couple acts on it. The force on the
r êë 128p e 0 r úû negative charge is more (F µ E) and is directed along
negative x-axis. Thus the dipole moves along negative
Surface tension decreases after electrification of
x-axis and rotates in an anticlockwise direction.
æ 1ö E 1q
bubble and therefore pressure decreases. ç P µ ÷ –q
è rø
a
means radius increases. In equation (A ), q 2 is not E

affected by positive or negative charge hence, whether


it is given a positive or a negative charge it always +q E2q
expands in radius. 44. (b) t = pE sin q , this is given by the second curve.
40. (c) The field due to infinite linear charge distribution
1 dq 1 q2
1 45. (c) In vacuum, F = …(i)
E=
4pe 0 rò ÞEµ
r
So hyperbola. 4pe0 r 2
41. (c) Initial force between the two spheres carrying charge Suppose, froce between the chrages is same when
(say q) is charges are r¢ distance apart in dielectric.
1 q2 1 q2
F= (r is the distance between them)
4pe 0 r 2 \ F' = …(ii)
4pe0 kr '2
Further when an uncharged sphere is kept in touch
with the sphere of charge q, the net charge on both From (i) and (ii), kr'2 = r2 or, r = kr '
In the given situation, force between the charges
q+0 q
become = . Force on the 3rd charge, when would be
2 2
placed at center of the 1st two 1 q2 4 q2 4F
F' = = =
r/2 r/2 4pe0 æ r rö
2 9 4pe0 r 2 9
ç + 4 ÷
è2 2ø
1 3 2 q
q q/2 q/2 L/2
2 q
æqö æqö
qç ÷ ç ÷ L/2
1 è2ø - 1 è2ø L
F3 = 46. (c)
4pe 0 æ r ö 2 4pe 0 æ r ö 2 L L
ç ÷ ç ÷
è2ø è2ø L
Free eBooks on @neetquestionpaper2020

Electric Charges and Fields 575


The given square of side L may be considered as one Q Torque on q about Q is zero hence angular momentum
of the faces of a cube with edge L. Then given charge about Q will be conserved.
q will be considered to be placed at the centre of the Þ mvrmin = mud ............. (2)
cube. Then according to Gauss's theorem, the
By eq. (2) in eq. (1)
magnitude of the electric flux through the faces (six)
of the cube is given by 2
1 1 æ ud ö kQq
f = q/e0 mu 2 = m ç ÷ +
2 2 è rmin ø rmin
Hence, electric flux through one face of the cube for
the given square will be
1 2
æ d 2 ö mu 2 d
mu ç 1 - =
1 q Þ 2 2 ÷ rmin {Q kQq = mu2d }
f' = f = è rmin ø
6 6e 0
2
47. (c) The charges make an electric dipole. A and B points Þ rmin - 2rmin d - d 2 = 0
lie on the equitorial plane of the dipole. There fore
potential at A = potential at B = 0 2d ± 4d 2 + 4d 2
Þ rmin = = d (1 ± 2)
W = Q (VA – VB) = q × 0 = 0 2
Q distance cannot be negative
Y
B (0, a, 0) –Z \ rmin = d (1 + 2)
–q
(0, 0,–a/2) 50. (a) Let us consider a spherical shell of radius x and
A X thickness dx. The volume of this shell is 4px2(dx). The
(–a, 0, 0) +q
(0, 0, a/2) charge enclosed in this spherical shell is

dx
48. (b) a x
O
–q q C

r
dq = (4px 2 )dx ´ kx a
Electric field at C due to electric dipole
\ dq = 4pkx 2 + a dx .
1 2p
= along OC For r = R :
4pe 0 r3
The total charge enclosed in the sphere of radius R is
Electric field at C due to induced charge must be equal
R
and opposite to electric field due to dipole as net field at R 3+ a
C is zero. ò0
Q = 4pk x 2 + a dx = 4pk
3+ a .
49. (b) The path of the particle will be as shown in figure. At
the point of minimum distance (D) the velocity of the \ The electric field at r = R is
particle will be ^ to its position vector w.r.t. +Q.
1 4pkR3+ a 1 4pk 1+ a
1 1 kQq E1 = = R
mu 2 + 0 = mv 2 + ............. (1) 4pe 0 (3 + a ) R 2 4 pe0 3 + a
2 2 rmin
For r = R/2 :
The total charge enclosed in the sphere of radius R/2 is
v D
R/2
4pk ( R / 2)3+ a
u
B Q' = ò0 4pk x 2 + a dx =
3+ a
r min
\ The electric field at r = R/2 is
d
1+ a
A 1 4pk ( R / 2)3+ a 1 4 pk æ R ö
+Q C E2 = = ç ÷
4 pe 0 3 + a ( R / 2) 2 4pe 0 3 + a è 2 ø
Free eBooks on @neetquestionpaper2020

576 Physi cs
56. (b) If +ve charge is displaced along x-axis, then net force
1
Given, E2 = E1 will always act in a direction opposite to that of
8 displacement and the test charge will always come
1+ a back to its original position.
1 4 pk æ R ö 1 1 4 pk 1+ a
\ ç ÷ = ´ R 57. (c) qd = e, md = 2mp = 2m
4pe0 (3 + a ) è 2 ø 2 3 4pe 0 3 + a
qa = 2e, ma = 4mp = 4m
Þ 1+a=3 Þ a=2 F1 = Fa = eE, F2 = Fa = 2eE ¹ F1
51. (a, b, c, d)
F1 eE
The potential shown is for charged spherical conductor. Further, a1 = =
2m 2m
52. (b) The charge at the inner surface, outer surface and inside
the conductor at P = (– Q, + Q, 0) as shown in the figure F2 2eE eE
and a 2 = = = = a1
2m 4m 2m
+Q +
+ 58. (c) Electric field at any point depends on presence of all
r +
P charges.
–Q
+ + 59. (d) No. of lines entering the surface = No. of lines leaving
+Q
the surface.
+ + 60. (a) The electric field due to disc is superposition of electric
+ field due to its constituent ring as given in statement-2.
Statement-1 is true, statement-2 is true, statement-2 is
53. (b) Charge on glass rod is positive, so charge on gold leaves
a correct explanation for statement-1.
will also be positive. Due to X-rays, more electrons from
leaves will be emitted, so leaves becomes more positive 61. (c) Statement-1 is correct. The induced field cancels the
and diverge further. external field. Statement-2 is false. When a current is set
up in a conductor, there exists an electric field inside it.
Charge
54. (a) Surface charge density (s) = 62. (a) 63. (d) 64. (c)
Surface area
65. (a) 66. (a) 67. (a)
– Q + 2Q = Q
-2Q
So sinner = –2Q Sq -5.9 ´ 10 -9
4pb 2 68. (a) f = = = – 670 Nm2/C
a b Î0 8.85 ´ 10 -12
Q c –
and sOuter = +2Q – +
4pc 2 69. (b) –– +
–– +
55. (c) Electric lines of force never intersect the conductor. Charged enclosed in Gaussian surface is +ve, so f is
They are perpendicular and slightly curved near the positive.
surface of conductor. 70. (c) Electric field at any point depends on all the charges.
Free eBooks on @neetquestionpaper2020

17
Electrostatic Potential
& Capacitance
ELECTROSTATIC POTENTIAL Electric Potential due to Continuous Charge
Electric potential at a point in an electric field is defined as the Distribution
amount of work done in bringing a unit positive test charge The potential due to a continuous charge distribution is the sum
from infinity to that point along any arbitrary path. (Infinity is of potentials of all the infinitesimal charge elements in which the
taken as point of zero potential). It is denoted by V ; distribution may be divided.
work done in bringing unit positive
W test charge from infinity to some point r
V= =
q0 unit positive test charge P
Its SI unit is JC–1 or volt. It is a scalar quantity. dq
Also, electric potential at any point in an electric field is defined R
r
as the negative line integral of the electric field vector E from a
dq dq
point infinitely away from all charges to that point i.e. V = ò dV = ò where dV =
4pe o r 4 pe or
r uur uur
i.e. V = - ò E .dr P +lC Potential due to a System of Charges
¥ The electric potential due to a system of charges q 1, q2, ...qn is
Potential due to a Point Charge V = V1 + V2 + ... + Vn
The electric potential due to a point charge q at separation r is 1 æ q1 q2 q ö 1 q
given by = . +
4pe çè r1 r2
+ ... + n ÷ =
rn ø 4pe
å ri
i
1 q where ri is the point from charge qi and e is the permitivity of
V= .
4pe r medium in which the charges are situated.
(Please note that we have to write q with its sign in this formula) Potential at any point P due to a point chage q at a distance
4F potential difference between two points is the work done in (r1 + r2) where r1 is the thickness of medium of dielectric constant
bringing unit positive charge from one point to another. x1 and r 2 is the thickness of the medium of dielectric constant k2
r1 r2
Q B A
rB
q K1 P
rA K2

B
-Q æ l l ö q 1
VAB = VB – VA = - ò E.dr = . ç - ÷ J/C V =k where k =
4peo è rB rA ø r1 K1 + r2 K2 4 p Î0
A
Free eBooks on @neetquestionpaper2020

578 Physi cs
Relation between electric field and potential A
The relation between electric field (E) and potential (V) is
1
dV 2
E=- dl
dr
For 3-D we can write
¶V ¶V ¶V
Ex = - , Ey = - and E z = - Q
¶x ¶y ¶z
B
So electric field is equal to negative potential gradient. Calculation of external work done against the field and a point charge
In this relation negative sign indicates that in the direction of Q in moving a test charge q from A to B. For a conservative field the
electric field, potential decreases. Consider two points A and B work done by any path is same. The sectional force is – qE.
situated in a uniform electric field at a distance d then, If A is at infinity then at infinity since potential is zero
we assume infinity as reference point,VA = 0
d B
A B E Þ VB = - q E.dl ò
¥
The potential difference between A and B is Potential energy of a system of two charges Q 1 and Q 2 is,
VAB = Ed Q1 Q2
Conservative nature of electric field r
The electric field is conservative in nature. In figure the work, 1 Q1 Q2
WAB has the same value whatever path is taken in moving the U=
4p Î0 r 2
test charge.
[Please note that in this formula we have to write charges with
Terminal sign]
Qo point Potential energy of a system of three charges Q1, Q2 and Q3
2 B
Vb Q1

Qo r3 r1
1

A
Starting
point Q3 Q2
r2
WAB 1 é Q1 Q 2 Q 2 Q 3 Q 3 Q1 ù
so, VAB = VB - VA = U= ê + + ú
Q 4p Î0 ë r1 r2 r3 û
has the same value for any path between A and B and V B and VA
Keep in Memory
are unique for the points A and B.
1. For an assembly of n charges [Total number of intersection
We cannot find the absolute value of potential
n n (n - 1)
therefore conventionally, we take infinity as the point of zero C2 = ] the potential energy is
potential. If need arises, we can assume any point to be the point 2
of zero potential and find the potential of other points on this é ù
1ê q iq j ú
n
basis.
U = êk å
2 ê i , j rij ú
ú
POTENTIAL ENERGY OF A SYSTEM OF CHARGES êë i ¹ j úû
Potential energy can be defined only for those forces, which are
2. For a system of two charges.
conservative, such as gravitational and electrostatic forces. The
potential energy of a charge between two points is defined as If Usystem = –ve, then there is net force of attraction between
the amount of work done in bringing the charge from one point the charges of the system.
to another. If Usystem = +ve, then there is net force of repulsion between
the charges of the system
B
Wext Usystem = max for unstable equilibrium
i.e. VB - VA =
q ò
= - E.dr
A
Usystem = min for stable equilibrium
dU
Also F = - =0
dx
Free eBooks on @neetquestionpaper2020

Electrostatic Potential & Capacitance 579


3. The energy required to take away the charges of a dipole at (iii) A circular disc of radius R with uniformly distributed charge
with surface charge density s
q2
infinite distance U = k Q
2l
4. The work done when a charge q is moved across a potential + +
+ R ++ s é 2
difference of V volt is given by W = qV
+ V= R + x2 - xù
5. When one electronic charge (1.6×10–19 coulomb i.e., charge + +++ x P 2e 0 êë úû
++
of electron) is moved across one volt the work done is
called one electron volt (eV). Thus
(iv) A finite length of charge with linear charge density l
1eV = (1 volt) × (1.6×10–19 coulomb) = 1.6×10–19 joule.
x
EQUIPOTENTIAL SURFACE + + + + + + + æ x + Lö
V = k l loge ç
It is that surface where the potential at any point of the surface L P è x ÷ø
has the same value. The electric lines of force and the
equipotential surface are mutually perpendicular to each other. (v) Due to a spherical shell of uniformly distributed charge
No work is done in moving a charge from one point to other on an with surface charge density s
equipotential surface. Work is done in moving a charge from one
equipotential surface to another. Q ++ + +
+ +
V +
+
V1 +
V1 > V2 r1 + +
V2 R
+ +
o +q +
x=R x +
r2 + + +

Q Q Q
Vin = k , Vsurface = k , Vout = k
Spherical equipotential surface R R x
for point charge (vi) Due to a solid sphere of uniformly distributed charge with
V = V2 volume charge density r.
V = V1
Q
V

R
x=R x
Plane equipotential surface for uniform field
3Q kQ(3R2 - r 2 )
· Equipotential surface do not cut each other. Vcentre = k , Vin =
· The density of the equipotential lines gives an idea of the
2R 2 R3
strength of electric field at that point. Higher the density, Q Q
larger is the field strength. Vsurface = k , Vout = k
R x
Potential Due to Various Charge Distribution
Potential due to Electric dipole
(i) Electric potential due to isolated point charge
(a) Along axial line :
x ( + q) x
V =k –q +q
+q x 2l P
P
(ii) A circular ring of radius R with uniformly distributed charge Q
Q p
++ + + Vaxial = k
+ x - l2
2
+ +
+ R + kQ kp
+ + x V= when x > > l Vaxial =
+ + P R2 + x2 x2
+ +
+ + (b) Along equatorial line :
++ +
· Potential V does not depend on the way of charge Veq = zero
distriubution on the ring (uniform / non-uniform).
Free eBooks on @neetquestionpaper2020

580 Physi cs
(c) At any point from the dipole : Example 1
P A uniform electric field pointing in positive y-direction
x exists in a region. Let A be the origin, B be the point on the
p cos q q x-axis at x = 2 cm and C be the point on the y-axis at y = 2
V =k 2 2 2 cm. Then the potential at the points A, B and C satisfy:
( x - l cos q) –q +q
(a) VA < VB (b) VA > VB
p (c) VA < VC (d) VA > VC
Keep in Memory Solution : (d)
As electric field represents the direction of motion of positive
1. Electric field inside a charged conductor is zero charge, which is from higher potential to lower potential,
+ + + therefore, from fig, we find VA = VB and VA > VC
+ + + + +
+ + + +
E = 0 + +
+ in + + Ein = 0 Y
+ + +
+ + + + + + +
E E

Spherical conductor Irregular shaped conductor C


But in both the cases the potential at all the points of the
surface will remain the same. But charges will have same X
r A B
distribution on spherical conductor and in case of E
irregularly shaped conductor the charge distribution will Z
be non-uniform. At sharp points, charge density has
greatest value.
Example 2
2. Electronic lines of force are always perpendicular to the
A uniform electric field pointing in positive x-direction
equipotential surfaces.
exists in a region. Let A be the origin, B be the point on the
3. The work done in moving a charge from a point to the
x-axis at x = + 1 cm and C be the point on the y-axis at
other on an equipotential surface is zero as the potential
y = + 1 cm. Then the potential at the points A, B and C
difference between the two points is zero.
satisfy
4. The electric potential at a point due to a point charge
decreases (or increases) by K-times if the distance between (a) VA < VB (b) VA > VB
the charge and the point increases (or decreases) by (c) VA < VC (d) VA > VC
K-times. Solution : (b)
5. A ring with a charge distribution behaves as a point charge uur
As E is directed along +ve direction of X-axis, therefore,
for the points very far from its centre.
6. The electric potential is constant inside a hollow charged VA > VB
sphere and it is also equal to its value on the surface but it C is vertically above A. Therefore, VA = VC.
varies inversely with the distance outside the sphere.
V
7. The electric potential at points inside a solid sphere has a
non-zero value and decreases as we go from the centre
C
outwards. It behaves as a point charge for the points E
outside the sphere.
8. The electric potential at a point due to a dipole varies
directly with the dipole moment. X
A B
COMMON DEFAULT
Incorrect : Where electric field is zero, electric potential is
Example 3
also zero.
Correct : It is not always correct, for example in a charged A charge Q is distributed over two concentric hollow
spheres of radii r and R (> r) such that the surface charge
conducting shell, electric field inside the shell E = 0 but
potential is not zero. densities are equal. Find the potential at the common
Incorrect : Where electric potential is zero, electric field is centre.
also zero. Solution :
Correct : It is not always correct. In the case of equitorial Let q1 and q 2 be charges on inner and outer spheres
plane of an electric dipole the electric potential is zero but respectively.
the electric field is non-zero.
Free eBooks on @neetquestionpaper2020

Electrostatic Potential & Capacitance 581


Example 4
q1 q2 q1 r2
As s1 = s 2 \ = or = Calculate the maximum voltage upto which a sphere of
4pr2 4pR2 q2 R2 radius 2 cm can be charged in air under normal conditions,
assuming that maximum electric intensity in air can be
q1 r2 q1 + q 2 r 2 + R 2 3 × 106 volt/m. Also, find the charge required to be given
\ 1+ = + 1 or =
q2 R 2 q2 R2 to the sphere.
Solution :
Q r2 + R2 We know that
Þ = (It is given that Q = q1 + q2)
q2 R2 Electric Potential (V)
Electric intensity (E) =
Distance (r)
Q R2 Þ V = Er …… (1)
\ q2 =
r2 + R2 Given, E = 3 × 106 volt/m, r = 2 cm = 2 × 10–2 m.
Substituting the above values in eq. (1), we get
Q r2 V = (3 × 106 volt/m) × 2 × 10–2 m = 6 × 104 volt.
Similarly, q1 =
r2 + R 2 Þ V = 60 kV
Potential at the common centre, Also, we know that the electric intensity on the surface of
a charged sphere is given by
q1 q2 1 é q1 q 2 ù 1 Q
V= + = +
4 pe o r 4 pe o R 4 pe o êë r R úû E= . ; Q = charge on the sphere
4p Î0 r 2
Q
1 é Qr QR ù Q (r + R ) Þ 3 ´ 10 6 = (9 ´ 10 9 ) ´
V= + ÞV = ( 2 ´ 10 - 2 ) 2
4 pe o êë r 2 + R 2 r 2 + R 2 úû 4 pe o ( r 2 + R 2 )
Þ Q = 1.33 ´ 10 -7 coulomb.

17.1
Solve following problems with the help of above text and 4. An equipotential surface is that surface
examples. (a) on which each and every point has the same potential
1. The electric potential at a point on the equatorial line of (b) which has negative potential
an electric dipole is (c) which has positive potential
(a) directly proportional to distance (d) which has zero potential
(b) inversely proportional to distance 5. A unit charge moves on an equipotential surface from a
(c) inversely proportional to square of the distance point A to point B, then
(d) None of these (a) VA – VB = + ve (b) VA – VB = 0
2. Which of the following quantities do not depend on the (c) VA – VB = – ve (d) it is stationary
choice of zero potential or zero potential energy? 6. If a unit charge is taken from one point to another over an
(a) Potential at a point equipotential surface, then
(b) Potential difference between two points (a) work is done on the charge
(c) Potential energy of a two-charge system (b) work is done by the charge
(d) Change in potential energy of a two-charge system (c) work done on the charge is constant
(d) no work is done
3. The electric field and the electric potential at a point are E
and V respectively. Which of the following is correct? 7. An uncharged insulated conductor A is brought near a
charged insulated conductor B, then
(a) If E = 0, V must be zero.
(a) the charge & potential of B, both remain constant
(b) If V = 0, E must be zero.
(b) both change
(c) If E ¹ 0, V cannot be zero.
(c) the charge remains constant, but potential decreases
(d) None of these
(d) the charge remains constant but potential increases

ANSWER KEY

1. (d) 2. (b, d) 3. (d) 4. (a) 5. (b) 6. (d) 7. (c)


Free eBooks on @neetquestionpaper2020

582 Physi cs

ELECTROSTATICS OF CONDUCTORS ENERGY STORED IN A CAPACITOR


Conductor is a substance that can be used to carry or conduct If Q is charge, V is p.d, C is the capacitance of the capacitor then
electric charges. Metals like silver. Copper, aluminium etc. are 1 1 Q2
good conductors of electricity. the energy stored is U = CV 2 = QV =
2 2 2C
Regarding electrostatics of conductors following points are worth
Sharing of Charges
noting.
When the two charged conductors of capacitances C1 and C2 at
(i) Inside a conductor, electric field is zero.
potentials V1 and V2 respectively, are connected by a conducting
(ii) The interior of a conductor can have no excess charge in
wire, the charge flows from higher to lower potential, until the
static situation.
potentials of the two conductors are equal.
(iii) Electric field at the surface of a charged conductor is
r s
E= nˆ V1 V2
Î0 C1 C2
where, s = surface charge density
Q1' Q2'
n̂ = unit vector normal to the surface in the outward
direction. The common potential after sharing of charges,
(iv) Electric field just outside a charged conductor is Net charge Q + Q 2 C1V1 + C 2 V2
V= = 1 =
perpendicular to the surface of the conductor at every Net capacitance C1 + C 2 C1 + C 2
point. The charges after sharing on two conductors will be
(v) Electrostatic potential is constant throughout the volume Q1 C1 Q1¢
of the conductor and has the same value as on its surface. Q1' = C1V and Q '2 = C 2 V i.e., = =
Q 2 C2 Q¢2
(vi) Surface density of charge is different at different points.
There is a loss of energy during sharing, converted to heat given
CAPACITORS AND CAPACITANCE by
A capacitor or condenser is a device that stores electrical
é1 2 1 2ù 1 2
energy. It generally consists of two conductors carrying equal DU = Uinitial - Ufinal = ê C1V1 + C 2 V2 ú - (C1 + C 2 )V
but opposite charges. ë2 2 û 2
The ability of a capacitor to hold a charge is measured by a C1C2 (V1 - V2 )2
quantity called the capacitance. Let us consider two uncharged or, Du =
2(C1 + C2 )
identical conductors X and Y and create a P.D. (Potential
Difference) V between them by connecting with battery B as PARALLEL PLATE CAPACITOR
shown in figure. It consists of two parallel metallic plates of any shape, each of
area A and at a distance d apart.
+Q –Q
Y
X The capacitance of the capacitor is given by C = e 0 A
A d
Effect of Dielectric on Capacitance
+qp
d When a dielectric slab is placed +
between the plates of a parallel plate – + –
+
B capacitor, the charge induced on its – E + –
+ 0
Fig- A capacitor consists of electrically insulated conductors carrying – + –
plates due to polarisation of dielectric +
equal positive and negative charge – Ep + –
+ –
After connection with the battery, the two conductors X and Y æ 1ö
is q p = Qç1 - ÷ + + –
have equal but opposite charges. Such a combination of charged è Kø + – E = E 0 – E p+ –

conductors is a device called a capacitor. The P.D. between X where K = dielectric constant. + Q – qp –Q
and Y is found to be proportional to the charge Q on capacitor.
When an electric field is applied across a dielectric, induced
The capacitance C, of a capacitor is defined as the ratio of the charges appear on the surface of dielectric which is shown in the
magnitude of the charge on either conductor to the magnitude above figure. These induced charges produce their own field
of P.D. between them. which acts in the opposite direction of the applied field. Hence,
Q
V µ Q Þ = constant = (Capacitance). total field is reduced, i.e., E 0 - E p = E , where E0 is the applied
V
Capacitance is always a positive quantity. field, Ep is the induced field and E is the resultant field.
The S.I. unit of capacitance is coulomb per volt or farad (F). E0
Further more, the value of capacitance depends on size, shape, E is given by , where K is the dielectric constant.
K
relative positions of plate, and the medium between the plates. If medium between the plates is having a dielectric of dielectric
The value of C does not depend on the charge of the plate or p.d. æ K eo A ö
between the plates. constant K then the capacitance is given C = ç ÷
è d ø
Free eBooks on @neetquestionpaper2020

Electrostatic Potential & Capacitance 583


If the space between the plates is partly filled with dielectric 7. The energy of a capacitor for a particular separation between
then the capacitance of the capacitor will be given by, the plates is the amount of work done in separating the two
t plates to that separation if they are made to touch to each
other.
P1 + – P2
+ – 8. The loss of energy when the two charged conductors are
+ K – connected by a wire doesn’t depend on the length of the
+ – wire.
A + – Example 5
+ – A parallel plate capacitor is maintained at a certain potential
+ – difference. When a 3 mm slab is introduced between the plates,
in order to maintain the same potential difference, the distance
d between the plates is increased by 2.4 mm. Find the dielectric
constant of the slab.
e0 A e0 A Solution :
C= = ,
t æ 1ö The capacity of a parallel plate capacitor in air is given by
d -t + d - t ç1 - ÷
K è Kø e A
where t is the thickness of the dielectric with dielectric constant K. C= 0 ... (1)
d
By introducing a slab of thickness t, the new capacitance
Keep in Memory
C´ becomes
1. The unit farad is quite a big unit for practical purposes. e0 A
C' = ... (2)
Even the capacitance of a huge body like earth is 711 mF. d'- t (1 - 1/K)
2. A capacitor is a device which stores charges and produces The charge (Q = CV) remains the same in both the cases.
electricity whenever required. Hence
3. If the two plates of a capacitor is connected with a eo A e oA æ 1ö
conducting wire, sparking takes place which shows that = or d = d'- t ç 1 - ÷
d d'- t (1 - 1/K) è Kø
electrical energy is converted into heat and light energy.
4. A capacitor allows A.C. but doesn’t allow D.C. to pass d ' = d + 2.4 ´ 10-3 m, t = 3 mm = 3 × 10–3 m.
through it. Substituting these values, we have
5. The capacitance of a capacitor increases with insertion of a æ 1ö
d = d + (2.4 ´10 -3 ) - 3 ´10 -3 ç1 - ÷
dielectric between its plates and decreases with increase in è Kø
the separation between the plates.
æ 1ö
6. The capacitance of a capacitor increases K times if a medium or ( 2.4 ´ 10 -3 ) = 3 ´ 10 -3 ç1 - ÷
of dielectric constant K is inserted between its plates. è K ø
Solving it, we get K = 5.

17.2
Solve following problems with the help of above text and
examples.
1. A condenser of capacity C1 is charged to a potential V0. 12 3 45
The electrostatic energy stored in it is U0. It is connected E
to another uncharged condenser of capacity C2 in parallel.
Find the energy dissipated in this process.
C2 C1
(a) U0 (b) U æ Î0 A ö æ -2 Î 0 A ö
C1 +C 2 C1 +C 2 0 (a) ç ÷ E and ç ÷E
è d ø è d ø
2
æ C1 - C2 ö C1C2 æ 2 Î0 A ö æ - Î0 A ö
(c) çç ÷÷ U 0 (d) 2( C +C ) U 0 (b) ç ÷ E and ç ÷E
1 2 è d ø è d ø
è C1 +C 2 ø
æ - Î0 A ö æ 2 Î0 A ö
2. Five identical capacitor plates, each of area A, are arranged (c) ç ÷ E and ç ÷E
such that the adjacent plates are at a distance d apart. The è d ø è d ø
plates are connected to a battery of e.m.f. E volt as shown. æ -2 Î 0 A ö æ Î0 A ö
The charges on plates 1 and 4 respectively are (d) ç ÷ E and ç ÷E
è d ø è d ø
Free eBooks on @neetquestionpaper2020

584 Physi cs

3. Consider the situation shown in the figure. The capacitor q


+ –
+ –
A has a charge q on it whereas B is uncharged. Find the + –
+ –
charge appearing on the capacitor B a long time after the + –
+ –
switch is closed. + –
+ –
+ –
+ –
+ –
+ –
+ –
+ A – B
(a) zero (b) q/2
(c) q (d) 2q

ANSWER KEY

1. (a) 2. (a) 3. (a)

SPHERICAL CAPACITOR B The capacitance of cylindrical condenser of length l


It consists of two concentric ++ 2p Î0 l
spherical conductors of radii R1 ++ ++ C=
æR ö
(without dielectric)
R2 A
++ ++

log e ç 2 ÷
++ ++

and R2 . The space between two è R1 ø


++

++

conductors is filled by a dielectric R1 O


of dielectric constant K.
++ ++ 2p Îo K l
++ C=
æR ö
log e ç 2 ÷ (with dielectric)
(a) When outer conductor is earthed, è R1 ø
Capacitance of spherical capacitor,
COMBINATION OF CAPACITORS
4p Î0 R1R2
C= (without dielectric) Series Combination
R2 - R1
(i) In this combination, the positive plate of one capacitor is
(4pe o K ) R1R2 connected to the negative plate of the other.
C= (with dielectric)
( R2 - R1 ) Q Q Q
(b) When inner sphere is earthed,
A B
4pe o K R1R2
C= + 4pe o R2 C1 C2 C3
R2 - R1
This is because the combination behaves as two capacitors V1 V2 V3
in parallel, one is a capacitor formed by two concentric V
spherical shells and the other is an isolated spherical shell (ii) The charges of individual capacitor are equal.
of radius R2. (iii) The potential difference is shared by the capacitors in the
CYLINDRICAL CAPACITOR inverse ratio of their capacities
It consists of two-coaxial cylindrical conductors of radii R1 and R2, i.e. Q = C1V1 = C2 V2 = C3 V3
the outer surface of outer conductor being earthed. The space Hence V = V1 + V2 + V3
between the two is filled with a dielectric of dielectric constant K. (iv) The equivalent capacitance (C) between A and B is
R1
1 1 1 1 1
= + + + ..... +
C C1 C 2 C 3 Cn
–+ +– Parallel Combination
–+ +– q1
–+ +– (i) In this arrangement, +ve plates + – c1
l A K B
–+ +– of all the condensers are +–
–+ +–
–+ +– connected to one point and q2
negative plates of all the A c
–+ +– +– 2 B
–+ +–
–+ +– condensers are connected to Q +–
the other point. q3
(ii) The Potential difference +– c
across the individual capacitor +– 3
R2
is same.
V
Free eBooks on @neetquestionpaper2020

Electrostatic Potential & Capacitance 585


(iii) The total charge shared by the individual capacitor is in RELATION BETWEEN THREE ELECTRIC VECTORS
direct ratio of their capacities ur ur r
D , P AND E
q q q
i.e. V = 1 = 2 = 3 If an electric field E is applied across a parallel plate capacitor
C1 C 2 C3
filled with a dielectric of dielectric constant K (or permittivity e),
Hence, Q = q1 + q2 + q3
then
(iv) The equivalent capacitance between A and B is
Polarisation P = induced charge per unit area (opposite to free
Ceq = C1 + C2 + C3 + ........+ Cn
q'
Keep in Memory charge) =
A
1. The capacitance of a parallel plate capacitor having a Electric displacement D = eE = eo E + P
number of slabs of thickness t1, t2, t3 .... and dielectric i.e. Polarisation P = (e – eo) E = (Keo – eo) E
constant K1, K2, K3 .... respectively between the plates is
Electric susceptibility, c e = P / E
eo A Relation between dielectric constant K and electric susceptibility
C=
æ t1 t t ö ce is
çç + 2 + 3 + .....÷÷
è K1 K 2 K 3 ø ce
K = 1+
t1 t2 e0
Effect of filling dielectric With battery connected
When there is no dielectric
+ K1 K2 Ae 0
– Capacitance C 0 =
d
Potential difference between the plates V
Charge on a plate Q = CV
2. When a number of dielectric slabs of same thickness (d) 1
and different areas of cross-section A1, A2, A3 ... having Energy E 0 = C0V 2
2
dielectric constants K1, K2, K3, .... respectively are placed
between the plates of a parallel plate capacitor then the V V
Electric field E 0 =
capacitance is given by d
+
Electric slabs which fill When dielectric is inserted
Area A1
Area A2
Area A3

the whole space of parallel


KA e 0
plate capacitor C= = KC 0
d
d – Q = K C0 V = KQ0 V
e (K A + K 2 A 2 + K 3 A 3 + ....)
C= o 1 1 1
d U= KC 0 V 2 = KE 0
3. When five capacitors are connected in wheatstone bridge 2

C1 C3 V
arrangement as shown, such that = , the bridge is E= = E0
C2 C4 d

balanced and C5 becomes ineffective. No charge is stored Effect of filling a dielectric in a capacitor after
on C5. Therefore C1, C2 and C3, C4 are in series. The two disconnection of battery
series combinations are in parallel between A and C. Hence +Q –Q
+Q –Q
equivalent capacitance can be calculated.
B V
C1 C2 Capacitance C0 C = K C0

Charge Q0 Q = Q0
A C5 C
Q0 Q 0 V0
P.D V0 = V= =
C0 C K
C3 C4
1 2 2
1 V U0
D Potential energy U0 = C0V0 U= KC 0 0 =
2 2 K K
Free eBooks on @neetquestionpaper2020

586 Physi cs
CHARGING AND DISCHARGING A CAPACITOR é -
1 ù
t
Charging a Capacitor (iii) As t ® ¥, q ® Q 0 = CV0 \ q = Q0 ê1 - e CR ú
êë úû
When an uncharged capacitor is connected across a source of
(iv) At t = CR [‘CR’ has dimensions of time]
constant potential difference such as a cell, it takes a finite time
to get fully charged, although this time interval may be small. é 1ù
q = Q 0 ê1 - ú = 0.631Q 0
This time-interval depends on the capacity of the capacitor and ë eû
the resistance in the circuit. This value of t = CR is called the ‘time constant’ of the (CR)
During the period of charging : circuit.
1. The charge on the capacitor increases from ‘zero’ to the Discharging of a Capacitor
final steady charge. If after charging the capacitor, the source of constant potential
2. The potential difference developed across the capacitor difference is disconnected and the charged capacitor is shorted
opposes the constant potential difference of the source. through a resistance ‘R’, then by Kirchhoff’s law, at time ‘t’ from
3. The charge on the capacitor ‘grows’ only as long as the the instant of shorting,
potential difference of source is greater than the potential q dq
difference across the capacitor. This transport of the charge +R =0
C dt
from the source to the capacitor constitutes a transient Putting,
current in the circuit. (i) the initial condition, q = Q0 at t = 0 and
4. As the charge on the capacitor increases, more energy is
(ii) the final condition, q = 0 at t ® ¥ ,
stored in the capacitor.
5. When the capacitor is fully charged, potential difference the solution to the above equation is
across the capacitor is equal to the potential difference of
the source and the transient current tends to zero. Q– 0
If V0 = constant potential difference of the source – 1 t
q = Q 0 e CR q
R = pure resistance in the circuit
C = capacity of the capacitor
Q0 = final charge on the capacitor, when fully charged t
q = charge on the capacitor at time ‘t’ from the starting of Keep in Memory
the charging
V = potential difference across the capacitor at time ‘t’ 1. If n small drops each having a charge q, capacity ‘C’ and
potential V coalesc to form a big drop, then
Source V0 Key (i) the charge on the big drop = nq
·
( )
(ii) capacity of big drop = n1/3 C
(iii) potential of big drop = n2/3 V
(iv) potential energy of big drop = n5/3 U
(v) surface density of charge on the big drop = n1/3 ×
R C surface density of charge on one small drop.
Q0 q 2. Charged soap bubble : Four types of pressure act on a
Then = =C charged soap bubble.
V0 V
(i) Pressure due to air outside the bubble PO, acting
dq
and i = current in the circuit at time ‘t’ = inwards.
dt
(ii) Pressure due to surface tension of soap solution PT,
At time ‘t’ by Kirchhoff’s law
acting inwards.
dq q dq 1
V0 - R - =0 i.e. = dt (iii) Pressure due to air inside the bubble, P i, acting
dt C CV 0 - q CR outwards.
Integrating and putting in the initial condition q = 0 at t = 0,
we get s2
(iv) Electric pressure due to charging, Pe = , acting
2e o
é 1 ù outwards.
– t
ê
q = CV0 1 – e CR ú
ê ú In equilibrium, Pi + Pe = PO + PT
ë û or, Pi – PO = PT – Pe
Special cases : or, Pexcess = PT – Pe
(i) At t = 0, q = 0. 4T s 2
\ Pexcess = -
(ii) When t increases, q increases. r 2e o
Free eBooks on @neetquestionpaper2020

Electrostatic Potential & Capacitance 587


Where T = surface tension of soap solution, where, A = area of the plates of capacitor and
s = surface charge density of bubble. K = dielectric constant of the medium filled between the
If Pi = PO then Pi – PO = PT – Pe = 0 or PT = Pe plates.
2 In terms of electric field, the force of attraction
4T s 2 1 æ q ö
= = ç ÷ 1
r 2e o 2e o çè 4 pr 2 ÷ø F = e 0 KE 2
Hence for maintaining the equilibrium of charged soap 2
4. Uses of capacitor :
8e o T
bubble, s = = In LC oscillators = As filter circuits
r = Tuner circuit in radio etc.
q = 8p r 2e o rT 5. The total energy stored in an array of capacitors
3. Force of attraction between the plates of a parallel plate (in series or in parallel) is the sum of the individual energies
q2 stored in each capacitor.
capacitor =
2e o AK

COMBINATION OF CAPACITOR : EQUIVALENT CAPACITANCE


Ae 0 Ae
C1 = K1 0
+

C1 = K1 1 1 1
K1 K1 d/2 d/2 = +
(i) d º Ae 0 Ae 0 Ceq C1 C 2
K2 K2 C2 = K 2 C2 = K 2
d/2 d/2
– –
é d d ù
êë d = 2 + 2 and area of each plate = A úû

+
+
+ æ AA öA æAö
KK ç1 æç1 æç ÷ö÷eeö÷00e 0
1K K 2 ç ÷e 0
2
(ii) d K1 K2 K2
Cº1C=C1 1== è è è2ø2ø ø C2 = è2ø
; C eq= C +1 C 2
ddd d
– ––
+
+ K 2 (A / 2)e 0
C2 =
K2 K (A / 2)e 0 d/ 2 C 2 C3
(iii) d K1 º C1 = 1 Ceq = C1 +
K3 d K 3 (A / 2)e 0 C 2 C3
C3 =
d/ 2
– –

+ Ae 0
C1 = K1
+ d/2
K1 1 1 1
d K 2e 0 (A / 2) K 3e 0 (A / 2) = +
(iv) K2 K3 = º C2 = C3 = Ceq C1 C 2 + C 3
d/2 d/2

C
1 2
1
+ 2 A+ 3C 2 –B
(v) ºA º

3 B
C
3 4
4
Ceq = C + C + C = 3C
Free eBooks on @neetquestionpaper2020

588 Physi cs

+– +– +– +– 1 C
1 2
2
– A + –
(vi) A Bº
= =º
B A B
C
3
4 4 3
+ + + +

Ceq = C + C = 2C
2 3
C
(+) A A + –
– – – –1 º B
A
+ + + + C C
+ + + +2
(vii) º 2 1 4 3
– – – –3
– – – – 1 1 3C
= C+ or, C eq =
+ + + + C eq 1 1 2
(–) B 4 +
B C C

SOME ME THODS OF FINDING EQUIVALENT C


C
C
2
CAPACITANCE
C C
Method 1 : Successive Reduction C
C C 2
This method is applicable only when the capacitor can be clearly X
A B
Y =
A B
identified as in series or in parallel.
C C C
2 mF 2 mF 2
A A D DC
2 mmF 2 mF 2 mF 2 mF 2
2 mF 2 mF 3C
2 mF Þ 2
=
2 mF 2 mF 4 mF A B
B B Method 3 : Wheatstone bridge
C
Þ

2 mF
C1 C2
A
2 mF 3 mF
A B Þ
A B Þ
26 13 mF 4 B
mF mF A C5
19 3 3
B

Method 2 : Using Symmetry C3 C4


C
D
C
C C1 C 2
C If C = C then the wheatstone bridge is balanced. In this case
X C E C Y 3 4
A B
there will be no charge accumulation in C5 when battery is
C attached across A and B. Therefore the equivalent circuit is the
C C
capacitance C1 and C2 are in series. Similarly C3 and C4 is in
A
D series. Therefore the equivalent capacitance occurs between A
and B is
The above circuit is symmetrical about XAEBY axis. This is
because the upper part of the circuit is mirror image of lower part. 1 1 1
= +
Therefore VC = VE = VD. The circuit can be redrawn as C eq C 1 +C 2 C 3 + C 4
Free eBooks on @neetquestionpaper2020

Electrostatic Potential & Capacitance 589


The other forms of wheatstone bridge are : The high voltage electrode is a hollow conductor mounted on an
C1 insulating medium. The belt is charged at A by means of corona
C2 B discharge between comb-like metallic needles and a grounded
grid. The needles are maintained at a positive potential of typically
A C3 C5
104 eV. The positive charge on the moving belt is transferred to
C4 the high voltage electrode by second comb of needles at B.
or Since the electric field inside the hollow conductor is negligible,
the positive charge on the belt easily transfers to the high- voltage
electrode, regardless of its potential. We can increase the potential
C5 C2 C1 of the high voltage electrode until electrical discharge occur
through the air . The “ breakdown” voltage of air is about 3 × 10 6 V/m.
C4
Example 6
B A
Obtain equivalent capacitance of the following network
C3 as shown in fig. For a 300 volt supply, determine the
Method 4 : If none of the above method works, then we can use charge and voltage across each capacitor.
the method of Kirchhoff’s laws - junction law and loop law. C1
SHARP POINT ACTION (CORONA DISCHARGE)
When the electric field (s / e 0 ) on a point on the surface of a C2 100 pF C3
conductor exceeds the electric strength of air, then the air becomes
conducting and the surface of conductor loses charge. This action 200 pF 200 pF +
C4 300 V
occurs usually at the sharp points of a conductor as here s is –
high, thus creating high electric field. This phenomenon is also
called corona discharge. 100 pF
Solution :
VAN DE GRAAF GENERATOR
As it is clear from fig, C2 and C3 are in series.
R.J. Van de Graff in 1931 designed an electrostatic generator
capableof generating very high potential of theorder of 5 × 106 V, 1 1 1 1 1
\ = + = +
which was then made use of an accelerating charged particles so Cs C2 C3 200 200
as to carry out nuclear reactions.
Principle : When a charged conductor is placed in contact with 2 1
= = Þ Cs = 100 pF
the inside of a hollow conductor, all of the charge of first 200 100
conductor is transferred to the hollow conductor. i.e., the charge Now, Cs and C1 are in parallel.
on hollow conductor or its potential can be increased by any Cp = Cs + C1 = 100 + 100 = 200 pF.
limit by repeating that processes. Again, Cp and C4 are in series. Their combined capacitance
The basic fact of Van de Graaf generator is described in fig. C is
(Charge is delivered continuously to a high voltage electrode on 1 1 1 1 1 3
a moving belt of insulating material). = + = + =
C C p C 4 200 100 200
High voltage electrode
200
B C= = 66.7 pF = 66.7 ´ 10 -12 F
+ 3
As Cp and C4 are in series.
\ Vp + V4 = 300 volt.
Belt Charge on C4,
200
q 4 = CV = ´ 10 -12 ´ 300 = 2 ´ 10 -8 C.
3
Insulator
Potential difference across C4:
A q4 2 ´10 -8
V4 = = = 200 volt.
C 4 100 ´10 -12
V p = 300 - V 4 = 300 - 200 = 100 volt .
Grounded grid Potential difference across C1 = V1 = Vp = 100 volt.
Schematic diagram of a Van de Graaf generator. Charge is transferred Charge on
to hollow conductor at the top by means of a rotating belt. The charge is C1, q1 = C1 V1 = 100 × 10–12 × 100 = 10–8 C
deposited on the belt at point A and is transferred to hollow conductor
at point B.
Potential difference across C2 and C3 in series = 100 volt
V2 = V3 = 50 volt
Free eBooks on @neetquestionpaper2020

590 Physi cs
Charge on Example 9
C2 = q2 = C2V2 = 200 × 10–12 × 50 = 10–8 C Consider a parallel plate capacitor of capacity 10 mF
Charge on with air filled in the gap between the plates. Now one half
of the space between the plates is filled with a dielectric of
C3 = q3 = C3V3 = 200 × 10–12 × 50 = 10–8 C dielectric constant K = 4 as shown in fig.
Example 7
Two isolated metallic solid spheres of radii R and 2 R are
charged such that both of these have same charge density K=4
s. The spheres are located far away from each other, and
connected by a thin conducting wire. Find the new charge
density on the bigger sphere. The capacity of the capacitor changes to
Solution : (a) 25 mF (b) 20 mF
Charge on smaller sphere, q1 = 4pR2s (c) 40 mF (d) 5 mF
Charge on bigger sphere, q2 = 4p(2R)2s = 1pR2s Solution : (a)
The arrangement is equivalent to three capacitors in parallel
\ Total charge, q = q1 + q2 = 20pR2s
e A / 4 10
Combined capacity of two spheres, C1 = o = = 2.5 mF ;
d 4
C = C1 + C2 = 4peoR + 4peo (2R) = 12peoR Ke o A / 2 10
After contact, charge is exchanged and a common potential C2 = = 4´ = 20 mF ;
d 2
V is reached.
e A / 4 10
C3 = o = = 2.5 mF
total ch arg e q 20 p R 2 s 5 Rs d 2
V= = = =
total capacity C 12 p e o R 3 eo \ Ceq = C1 + C2 + C3 = 2.5 + 20 + 2.5 = 25mF
Now, charge on bigger sphere, Example 10
A parallel plate capacitor is filled with dielectric as shown
5 Rs 40 p R 2 s in fig. Its capacitance has ratio with that and without of
q2' = C2 V = 4 p e o (2 R ) ´ =
3e o 3 dielectric as
\ Surface density of charge
+
q '2 40 p R 2 s 5
s '2 = = = s. K2
surface area 3 (4 p) (2 R ) 2 6 d
d
Example 8 2
K1
Two insulated metal spheres of radii 10 cm and 15 cm –
charged to a potential of 150 V and 100 V respectively,
are connected by means of a metallic wire. What is the æ K1 + K 2 ö
charge on the first sphere? (a) (K1 + K2) (b) çç K – K ÷÷
è 1 2 ø
Solution :
Here, r1 = 10 cm, r2 = 15cm æ 2K 1 K 2 ö æ K1 +K 2 ö
(c) çç K + K ÷÷ (d) çç K K ÷÷
V1 = 150 V, V2 = 100 V è 1 2 ø è 1 2 ø
Common potential Solution : (c)
eo A
C1V1 + C 2 V2 Without dielectric, C 0 =
V= d
C1 + C 2 With dielectric as shown,
4 p Î0 (r1 V1 + r2 V2 ) K e A K 2e o A
= C1 = 1 o = 2K1C 0 ; C2 = = 2K 2C 0 ;
4 p Î0 (r1 + r2 ) d/2 d/2
= 120 volt As C1, C2 are in series,
q1 = C1V 1 1 1 C + C1
\ = + = 2 ;
Cs C1 C 2 C1 C 2
= 4 p Î0 r1 V
C1 C 2 2 K1 C 0 2 K 0 C 0 2 K1 K 2 C 0
10-1 \ Cs = = =
= ´ 12C C1 + C 2 2 C 0 (K1 + K 2 ) K1 + K 2
9 ´ 109
Cs 2 K1 K 2
12 =
= 9
´ 3 ´ 10 esu = 4esu C 0 K1 + K 2
9 ´ 109
Free eBooks on @neetquestionpaper2020

Electrostatic Potential & Capacitance 591


Example 11 Example 13
A circuit is shown in fig. A parallel plate condenser with plate area A and
separation d is filled with dielectrics as shown in fig.
A B
2 mF
3 mF
d/2 K1
5 mF d
K2
d/2
4 mF
+ –
6V
The dielectric constants are K1 and K2 respectively. Find
Find the charge on the condenser having a capacity of the capacitance.
5 mF. Solution :
Solution : The combination is equivalent to two capacitors in series,
Potential difference between A and B = 6 volts. The each with plate area A and separation d/2.
condensers 2 mF and 5 mF are in parallel. Their effective
capacitance, C = 2 + 5 = 7 mF.
The capacitance between A and B is given by
K1 C1
C ´ 3 7 ´ 3 21
C´= = = mF
C + 3 7 + 3 10 K2 C2
21 63
Total charge Q = CV = ´6 = mC
10 5
Total potential difference across 3 mF is K1 e o A 2 K1 e o A K 2 e o A 2 K 2 eo A
C1 = = ; C2 = =
Q 63 1 21 ( d / 2) d (d / 2 ) d
V1 = = ´ = volts
3 5 3 5 1 1 1
The equivalent capacitance C is given by = +
Hence the common potential difference across the C C1 C2
condensers in parallel is 1 d d
\ = +
21 9 C 2 K1 e o A 2 K 2 e o A
V2 = 6 - = V
5 5 d é 1 1 ù d æ K1 + K 2 ö
So, the charge on 5 mF condenser is = ê + ú= ç ÷
2 e o A ë K1 K 2 û 2 e o A çè K1 K 2 ÷ø
9
Q = 5 ´ V2 = 5 ´ = 9 mC 2 e o A æ K1 K 2 ö
5 or C= ç ÷
d çè K1 + K 2 ÷ø
Example 12
A parallel plate air capacitor is connected to a battery. Example 14
The quantities charge, voltage, electric field and energy Two spherical conductors A and B of radii a and b (b > a)
associated with this capacitor are given by Q0, V0, E0 and are placed concentrically in air. B is given a charge + Q
U0, respectively. An electric slab is now introduced to fill and A is earthed. Find the equivalent capacitance of the
the space between the plates with battery still in system.
B
connection. The corresponding quantities now given by
Q, V, E and U are related of the previous ones as A
(a) Q > Q0 (b) V > V0 a
(c) E > E0 (d) U > U0
Solution : (d) b
In the presence of dielectric, the capacity
+Q
C = K C0 or C > C0
Solution :
We know that Q = CV µ C (Q V = constant) The charge Q given to outer sphere is distributed as Q1
\ Q > Q0 ...(1) outer surface and Q2 inner surface. The inside charge
1 1 Q2 induces a charge –Q2 on outer surface of inner sphere
Further U = C V 2 ; U 0 = C 0 V 2 and + Q2 on inner surface of inner sphere which is earthed.
2 2
The inner surface of outer sphere and outer surface of inner
\ U > U0 (Q C > C0) ...(2) sphere constitute a spherical condenser having capacitance
Free eBooks on @neetquestionpaper2020

592 Physi cs
[4 p e0 a b/(b – a)}] and the outer surface of the outer K 1e o adx K 1e o adx K 1e o a 2 dx
constitutes an isolated sphere of capacitance 4 p e0 b. dC 1 = = =
d-y d - xd/a d(a - x)
Now the effective capacitance C is given by
ì ab ü
C = 4p e o í ý + 4p e o b (due to parallel combination)
î (b - a ) þ dC1 K1 d – y
ì a ü dC 1 and dC 2 are capacitances
= 4p e o bí + 1ý
î (b - a ) þ of upper and lower strips
ìa + b - a ü ìï b 2 üï dC2 y
= 4p e o b í ý = 4p e o bí (b - a ) ý K2
î (b - a ) þ ïî ïþ

Example 15 dx
Three plates A, B, C each of area 50 cm2 have separation Since dC1 and dC2 are in series, we have
3 mm between A and B and 3 mm between B and C. Find 1 1 1
= + xd d(a - x)
the energy stored when the plates are fully charged. = +
dC eq dC 1 dC 2 K e a 2 dx K e a 2 dx
2 o 1 o
A 1 d ìï x a - x üï
– + Þ = 2 í + ý
B dC eq e o a dx îï K 2 K 1 þï
C 12 V
d ìï K 1x + K 2 a - K 2 x üï
= 2 í ý
Solution : e o a dx îï K 2K 1 þï
The two condensers are connected in parallel. a
e o a 2 K 1K 2 dx
e A e A 2e A Þ dC eq = Þ C eq = ò dC eq
Hence C = o + o = o d{K 2 a + (K 1 - K 2 )x}
d d d 0

1 1 æ 2e A ö eo a 2K 1K 2 a dx
\ Energy stored = C V 2 = ´ ç o ÷ V 2
2 2 è d ø
Þ C eq =
d ò0 K 2 a + (K 1 - K 2 )x
Substituting the given values, we get U = 1.6 mJ eo a 2K
Þ C eq = 1K 2 ln
K1
Example 16 (K 1 - K 2 )d K2
A capacitor is formed by two square metal-plate of edge a,
separated by a distance d. Dielectric of dielectric constant Example 17
K1 & K2 are filled in the gap as shown in the fig. Find the Find the capacitance of the capacitor as shown in fig. The
capacitance of the system. plate area is A and the separation between the plates is d
in each case. Different dielectric slabs in each part of all
figures are of the same thickness and the entire gap
K1
d between the plates is filled with dielectric slabs.
K2 K1
a Case 1 : d
Solution : K2
Let us take a strip at a distance x from the left of thickness
dx. From similar triangles, we have
dx K1
Case 2 : K2 d
K1 K3
K2 d
y

Case 3 : K1 K2 d
x
a
y d xd Solution :
= Þy=
x a a In the first case the two capacitors are in series of
K e adx K 2 e o a 2 dx thickness d/2 and plate area A
\ dC 2 = 2 o = ……(1) K e A 2K 1e o A K e A 2K 2 e o A
y xd C1 = 1 o = , C2 = 2 o =
d/2 d d/2 d
Free eBooks on @neetquestionpaper2020

Electrostatic Potential & Capacitance 593

1 1 1 Example 19
\ = + A particle having a charge of 1.6 × 10–19 C enters midway
C eq C 1 C 2
between the plates of a parallel plate condenser. The
1 d d d ì 1 1 ü initial velocity of the particle is parallel to the plates. A
= + = í + ý
C eq 2K1e o A 2K 2e o A 2e o A î K 1 K 2 þ potential difference of 300 V is applied to the capacitor
plates. If the length of capacitor plates is 10 cm and they
2e o AK1K 2 are separated by 2 cm, calculate the greatest initial
Þ C eq =
(K1 + K 2 )d velocity for which the particle will not be able to come out
Similarly in case (2) of the plates. The mass of the particle is 12 × 10–24 kg.
1 1 / K1e o A 1 / K 2e o A 1 / K 3e o A Solution :
= + + Here, q = 1.6 × 10–19 C
C eq d /3 d /3 d /3
V = 300 volt, d = 2 × 10–2 m, x = 10 cm
d d d m = 12 × 10–24 kg, u = ?
= + +
3K1e o A 3K 2 e o A 3K 3e o A + + + + + + + + +

d ì 1 1 1 ü u E
= í + + ý
3e o A î K1 K 2 K 3 þ
y = 1 cm
d ì K1K 2 + K 2 K 3 + K 3 K1 ü
= í ý – – – – – – – – –
3e o A î K1K 2 K 3 þ 10 cm
3e o AK1K 2 K 3 V 300
Þ C eq = Electric intensity, E = = = 15000 V / m
(K1K 2 + K 2 K 3 + K 3 K1 )d d 2 ´10 - 2
In case (3) As the particle does not come out, its maximum deflection,
Ke A K e A e A ( K1 + K 2 ) y = 1 cm = 10–2 m
C eq = 1 o + 2 o ; C eq = o
2d 2d 2d 1 2 1 q E æ xö
2

Example 18 As y = at = ç ÷
2 2 m è uø
Consider the situation shown in figure. The width of each
q E x 2 1.6 ´ 10-19 ´ 15000 ´ (10-1 )2
plate is b. The capacitor plates are rigidly clamped in the \ u2 = = = 108
laboratory and connected to a battery of e.m.f. E. 2my 2 ´ 12 ´ 10-24 ´ 10-2
All the surfaces are frictionless. Calculate the value of M \ u = 104 m/s.
for which the dielectric slab will stay in equilibrium. Example 20
l A capacitor of capacity 1 mF is connected in closed series
circuit with a resistance of 107 W, an open key and a cell
of 2 V with negligible internal resistance.
d E K (i) When the key is switched on at a time t = 0, find :
(a) the time constant for the circuit.
(b) the charge on the capacitor at steady state.
(c) time taken to deposit charge equalling half that
M at steady state.
Solution : (ii) If after fully charging the capacitor, the cell is shorted
Let us take a moment of time when the slab is at a distance by zero resistance at time t = 0, find the charge on the
x from the left end. capacitor at t = 50 s.
e xb Ke 0 (l - x)b e b Solution :
C eq = o + = o {x + K(l - x)}
d d d (i) (a) Time constant = RC = (107) (10–6) = 10 s.
Energy at this moment = ½ Ceq E2 (b) Q0 = charge on capacitor at steady state
1 e ob = V0C = 2 × 10–6 = 2 mC
U= {x + K(l - x)} E 2 1
2 d 1 - t
dU 1 e o b (c) q = Q0 = Q0 [1 - e CR ]
= {1 - K)} E 2 2
dx 2 d t

dU e b(K - 1)E 2 Þ e10 = 2 or, t = 10 ´ 2.306 ´ log10 2 = 6.94 s.


- =F= o = Mg 1
dx 2d - t
[Q For the condition of equilibrium Fext should (ii) q = Q0 e CR

be equal to conservative force] 50


-6
- æ 1ö
= (2 ´ 10 )(e 10 ) = (2 ´ 10-6 ) ç 5 ÷
e o b(K - 1)E 2 èe ø
M=
2gd = 1.348 × 10–8 C (Q e = 2.718)
Free eBooks on @neetquestionpaper2020

594 Physi cs

17.3
Solve following problems with the help of above text and 9. The capacity of parallel plate capacitor depends on
examples. (a) metal used to make plates
1. When a metal plate is introduced between the two plates (b) thickness of plate
of a charged capacitor and insulated from them, then (c) potential applied across the plate
(more than one alternative may be correct) (d) area of plate
(a) the metal plate divides the capacitor into two 10. We increase the charge on the plates of a capacitor, it means,
capacitors connected in parallel to each other (a) increasing the capacitance
(b) the metal plate divides the capacitors into two (b) increasing P.D. between plates
capacitors connected in series with each other (c) decreasing P.D. between plates
(c) the metal plate is equivalent to a dielectric of zero (d) no change in field between plates
dielectric constant 11. If in a parallel plate capacitor, which is connected to a
(d) the metal plate is equivalent to a dielectric of infinite battery, we fill dielectrics in whole space of its plates,
dielectric constant then which of the following increases?
2. A capacitor works in (a) Q and V (b) V and E
(a) A. C. circuits (b) D. C. circuits (c) E and C (d) Q and C
(c) both ‘a’ and ‘b’ (d) neither ‘a’ nor ‘b’ 12. In the circuit shown capacitor C, charged by V volt battery
3. In a charged capacitor, the energy is stored in only when, keys
(a) the negative charges K1 K2
(b) the positive charges R1 R2
(c) the field between the plates
(d) both ‘a’ and ‘b’
4. A parallel plate capacitor is charged and then isolated. V C
What is the effect of increasing the plate separation on
charge, potential, capacitance, respectively?
(a) Constant, decreases, decreases
(a) K1 and K2 are closed
(b) Increases, decreases, decreases
(b) both keys are open
(c) Constant, decreases, increases
(c) K1 open and K2 closed
(d) Constant, increases, decreases
(d) K1 closed and K2 open
5. When a dielectric is introduced between the plates of a
13. Eight drops of Hg (equal radii), which have equal charge
condenser, the capacity of condenser
constitute a bigger drop. The capacitance of bigger drop
(a) increases (b) decreases in comparison to small drop is
(c) remains same (d) None of these (a) two times (b) four times
6. We assume that earth is at zero potential because (c) eight times (d) sixteen times
capacitance of the earth is
14. If on combining two charged bodies, the current does not
(a) infinite (b) zero flow then
(c) cannot say (D) 106 farad (a) charge is equal on both
7. In a parallel plate capacitor, the distance between the plates (b) capacitance is equal on both
is d & potential difference across plates is V. Energy stored
(c) potential is equal on both
per unit volume between the plates of capacitor is
(d) resistance is equal on both
Q2 1 15. Two spherical conductors A1 and A2 of radii r1 and r 2
(a) 2 (b) eo V 2 / d 2
2V 2 (r 2 > r1) are placed concentrically in air. A1 is given a
charge +Q while A2 is earthed. Then the equivalent
1 V2 1 V2
(c) 2 (d) eo capacitance of the system is
2
eod 2 d
A2
8. For a given potential difference V how would you connect – –
two capacitors, to obtain greater stored charge + +
A1 r1
(a) series
(b) parallel + +
(c) series and parallel combined – r2 –
(d) cannot say
Free eBooks on @neetquestionpaper2020

Electrostatic Potential & Capacitance 595

4p Î0 r1r2
(a) (b) 4p Î0 (r1 + r2 )
r2 - r1
+ P Q –
(c) 4p Î0 r2 (d) 4p Î0 r1
16. Two spherical conductors A1 and A2 of radii r1 and r2
X
and carrying charges q1 and q2 are connected in air by a
copper wire as shown in the figure. Then the equivalent (a) Ep is reduced by the presence of X if X is metallic
capacitance of the system is (b) Eq is increased by the presence of X if X is dielectric
A2 (c) Eq is in the opposite sense to Ep if X is a dielectric
A1
(d) Eq is zero if X is metallic
O1 O2
20. A copper plate is introduced between the plates A and B
r1
of a parallel plate capacitor as shown in fig. The copper
r2
plate is not in contact with A or B. Which one of the
following statements is correct?
4p Î0 r1r2
(a) (b) 4p Î0 (r1 + r2 )
r2 - r1
(c) 4p Î0 r2 (d) 4p Î0 r1 A B
17. A foil of aluminium of negligible thickness is inserted in
between the space of a parallel plate condenser. If the foil Copper Insulation
Plate
is electrically insulated, the capacity of the condenser will
(a) A charge is not induced in copper plate
(b) Copper plate divides the capacitor into two capacitors
Foil connected in parallel
(c) Copper plate divides the capacitor into two capacitors
connected in series
(a) increase (b) decrease (d) The effective distance between the plates of the
(c) remain unchanged (d) become zero capacitor is reduced
18. An infinite number of identical capacitors, each of 21. A finite ladder is constructed by connecting several
capacitance 1 mF are connected as in the adjoining figure. sections of 2 mF, 4 mF capacitor combinations as shown in
Then the equivalent capacitance between A and B is the figure. It is terminated by a capacitor of capacitance C.
What value should be chosen for C, such that the
equivalent capacitance of the ladder between the point A
and B becomes independent of the number of sections in
between?
4mF 4mF 4mF
A
A B
(a) 1 mF (b) 2 mF 2mF 2mF 2mF C
(c) 1/2 mF (d) ¥
19. A slab X is placed between the two parallel isolated B
charged plates, as shown in the figure . If Ep and Eq denote (a) 4 mF (b) 2 mF
the intensity of electric field at P and Q then (c) 18 mF (d) 6 mF

ANSWER KEY

1. (b,d) 2. (a) 3. (c) 4. (d) 5. (a) 6. (a) 7. (b) 8. (b) 9. (d) 10. (b) 11. (d) 12. (d)
13. (a) 14. (c) 15. (a) 16. (b) 17. (c) 18. (b) 19. (c,d) 20. (c) 21. (a)
Free eBooks on @neetquestionpaper2020

596 Physi cs

Very Short / Short Answer Questions 11. A network of four capacitors, each of capacitance 30 µF,
is connected across a battery of 60 V, as shown in the
1. Write down the relation between electric field and potential figure. Find the net capacitance and the energy stored in
at a point. each capacitor. [Outside Delhi - 2010 COMPTT.].
2. What is the work done in moving a charge of 50 nC
between two points on an equipotential surface?
3. How much energy will be stored by a capacitor of 470 mF C2
C1 C3
when charged by a battery of 20 V?
4. An electric dipole is free to move in a uniform electric
field. Explain its motion when it is placed (i) parallel to the C4
field and (ii) perpendicular to the field.
5. Derive an expression for the potential at a point due to a 60 V
short dipole. Hence show what will be the potential at an
axial and an equatorial point. 12. Why must the electrostatic potential inside a hollow
6. Calculate the equivalent capacitance between points A charged conductor be the same at every point?
and B in the following combination. [Outside Delhi - 2012 COMPTT.].
Long Answer Questions
13. Describe the construction and working of a Van de Graaff
A
5mF 10mF 15mF 30mF B generator with the help of a labeled diagram.
14. In the arrangement of capacitors as shown in the diagram
7. A parallel-plate capacitor has the air as a medium between
its plates and its capacitance is C. A slab of the dielectric below, the energy stored in 6 mF capacitor is E. Find the
constant K is introduced so as to fill the left half of space following:–
of the capacitor, as shown. What is the new capacitance (a) Energy stored in the 12 mF capacitor..
of the capacitor?
(b) Energy stored in the 3 mF capacitor..
(c) Total energy drawn from the battery.
K d
E
C1 = 6 mF C3 = 3 mF
8. Equipotential surfaces are perpendicular to field lines. C2 = 12 mF
Why?
9. Determine the potential difference across the plates of
the capacitor 'C1' of the network shown in the figure. 15. A network of four capacitors, each of capacitance 24µF, is
[Assume E2 > E1] [Delhi Board - 2012 COMPTT.]. connected across a battery of 50 V, as shown in the figure.
Find the net capacitance and the net energy stored in the
C1 capacitors. [Outside Delhi - 2012 COMPTT.].
–q +q

C1 C2 C3
E1 E2
C4
+q –q
C2
50 V
10. Write two properties of equipotential surfaces. Depict 16. (a) A parallel plate capacitor is charged by a battery to a
equipotential surfaces due to an isolated point charge. potential. The battery is disconnected and a dielectric
Why do the equipotential surfaces get closer as the slab is inserted to completely fill the space between
distance between the equipotential surface and the source
the plates. How will (i) its capacitance, (ii) electric field
charge decreases? [Outside Delhi - 2011 COMPTT.].
between the plates and (iii) energy stored in the
Free eBooks on @neetquestionpaper2020

Electrostatic Potential & Capacitance 597

capacitor be affected? Justify your answer giving (a) larger magnitude than the field at the mid-point
necessary mathematical expressions for each case. between the plates and point towards– x̂ .
(b) Sketch the pattern of electric field lines due to (i) a (b) smaller magnitude than the field at the mid-point
conducting sphere having negative charge on it. (ii) between the plates and point towards+ x̂ .
an electric dipole. [Delhi Board - 2011 COMPTT.]. (c) larger magnitude than the field at the mid-point
17. (a) Deduce the expression for the energy stored in a between the plates and point towards+ x̂ .
charged capacitor. (d) smaller magnitude than the field at the mid-point
(b) Show that the effective capacitance, C, of a series between the plates and point towards– x̂ .
combination, of three capacitors, C1, C2 and C3 is given by
P
s V
C1C2C3 z
C= s
(C1C2 + C2 C3 + C3C1 )
y
[Outside Delhi - 2010 COMPTT.].
x
Multiple Choice Questions 22. Four points a, b, c and d are set at equal distance from thwe
centre of a dipole as shown the figure. The electrostatic
18. A parallel plate capacitor is charged to a certain voltage. potential Va, Vb, Vc, and Vd would satisfy the following
Now, if the dielectric material (with dielectric constant k) is relation: a
removed then the (a) Va > Vb > Vc > Vd +q
(a) capacitance increases by a factor of k (b) Va > Vb = Vd > Vc d b
(b) electric field reduces by a factor k (c) Va > Vc = Vb = Vd –q
(d) Vb = Vd > Va > Vc c
(c) voltage across the capacitor decreases by a factor k
23. Eight drops of mercury of equal radius and possessing equal
(d) None of these charge combine to form a big drop. The capacitance of bigger
19. Two identical conducting balls having positive charges q1 drop as compared to each small drop is
and q2 are separated by a distance r.If they are made to (a) 16 times (b) 8 times
touch each other and then separated to the same distance, (c) 4 times (d) 2 times
the force between them will be 24. The figure given below shows four arrangements of
(a) less than before (b) same as before charged particles, all at the same distance from the origin.
Rank the situation according to the net electric potential
(c) more than before (d) zero
(V1, V2, V3, V4) at the origin, most positive first :
20. A sphere of radius R has uniform volume charge density.
The electric potential at a points (r <R) is +2q –2q –2q –6q
–2q
(a) due to the charge inside a sphere of radius r only
V1 –9q –3q –2q +2q +q
(b) due to the entire charge of the sphere V2 V3 –3q –5q
V4
(c) due to the charge in the spherical sheel of inner and 1 2 4
3
outer radii r and R, only (a) V1 > V2 > V3 > V4 (b) V2 > V1 > V3 > V4
(d) independent of r (c) V2 > V1 > V4 > V3 (d) V4 > V1 > V3 > V2
21. Two large parallel plates move with a constant speed v in 25. The capacitance of a metallic sphere is 1mF , then it’s radius
the positive y-direction as shown in the figure. If both the is nearly
plates have a surface charge density s > 0, the magnetic (a) 1.11 m (b) 10 m
field at the point P just above the top plate will have. (c) 9 km (d) 1.11 cm
Free eBooks on @neetquestionpaper2020

598 Physi cs

1. A parallel plate condenser with oil between the plates q3


(dielectric constant of oil K = 2) has a capacitance C. If the C
oil is removed, then capacitance of the capacitor becomes
[CBSE PMT 1997, 99]
40 cm
C C
(a) 2 C (b) 2 C (c) (d)
2 2 q2
q1
2. What is the effective capacitance between points X and Y?
[CBSE PMT 1999] A 30 cm B D
C1 = 6m F (a) 8q1 (b) 6q1 (c) 8q2 (d) 6q2
(a) 24 mF
(b) 18 mF 7. A network of four capacitors of capacity equal to C1 = C,
C3 = 6m F C5 = 20m F C2 = 6m F
X Y C2 = 2C, C3 = 3C and C4 = 4C are conducted to a battery as
(c) 12 mF A C B D
C4 = 6m F shown in the figure. The ratio of the charges on C2 and C4 is
(d) 6 mF [CBSE PMT 2005]
3. In a parallel plate capacitor, the distance between the plates
is d and potential difference across plates is V. Energy
stored per unit volume between the plates of capacitor is
[CBSE PMT 2001]
Q2 1 V2
(a) (b) e0
2V 2 2 d2
1 V2 1 V2
(c) (d) e0
2 e 0d 2 2 d
(a) 4/7 (b) 3/22 (c) 7/4 (d) 22/3
4. A capacitor C1 is charged to a potential difference V. The
charging battery is then removed and the capacitor is 8. A series combination of n1 capacitors, each of value C1, is
connected to an uncharged capacitor C2. The potential charged by a source of potential difference 4 V. When
difference across the combination is [CBSE PMT 2002] another parallel combination of n2 capacitors, each of value
C2, is charged by a source of potential difference V, it has
VC1 æ C ö
(a) (b) Vçç1 + 2 ÷÷ the same (total) energy stored in it, as the first combination
(C1 + C 2 ) è C1 ø has. The value of C2 , in terms of C1, is then
æ C ö VC 2
(c) Vçç1 + 1 ÷÷ (d) [CBSE-PMT 2010]
è C2 ø (C1 + C 2 )
2C1 n n 16 C1
5. As per this diagram a point charge +q is placed at the origin (a) (b) 16 2 C1 (c) 2 2 C1 (d)
O. Work done in taking another point charge n1 n2 n1 n1 n1 n2
– Q from the point A [coordinates (0, a)] to another point B 9. A condenser of capacity C is charged to a potential
[coordinates (a, 0)] along the straight path AB is difference of V1. The plates of the condenser are then
[CBSE PMT 2005] connected to an ideal inductor of inductance L. The current
(a) zero through the inductor when the potential difference across
æ - qQ 1 ö the condenser reduces to V2 is [CBSE-PMT 2010]
(b) çç 4pe 2 ÷÷ 2a y
1/ 2 1/ 2
è 0 a ø æ C(V12 - V22 ) ö æ C (V1 - V2 ) 2 ö
A (a) çè ÷ø (b) çè ÷ø
æ qQ 1 ö a L L
ç ÷
(c) ç 4pe 2 ÷. 2
è 0 a ø C(V12 - V22 ) C (V1 - V2 )
(c) (d)
æ qQ 1 ö O x L L
(d) çç 4pe 2 ÷÷. 2a B
10. Two parallel metal plates having charges + Q and – Q face
è 0 a ø
each other ata certain distance between them. If the plaves
6. Two charges q1 and q2 are placed 30 cm apart, as shown in
are now dipped in kerosene oil tank, the electric field
the figure. A third charge q3 is moved along the arc of a
between the plates will [CBSE-PMT 2010]
circle of radius 40 cm from C to D. The change in the
q3 (a) remain same (b) become zero
potential energy of the system is k , , where k is
4p Î0 (c) increases (d) decrease
[CBSE PMT 2005]
Free eBooks on @neetquestionpaper2020

Electrostatic Potential & Capacitance 599


11. A parallel plate condenser has a uniform electric field E(V/ 17. If there are n capacitors in parallel connected to V volt
m) in the space between the plates. If the distance between source, then the energy stored is equal to [AIEEE 2002]
the plates is d(m) and area of each plate is A(m2) the energy 1 1
(joules) stored in the condenser is [CBSE-PMT 2011] (a) CV (b) n CV 2 (c) CV2 (d) CV 2
2 2n
1 18. A thin spherical conducting shell of radius R has a charge
(a) E2Ad/ Î0 (b) Î0 E 2
2 q. Another charge Q is placed at the centre of the shell. The
1
(c) Î0 EAd (d) Î0 E 2Ad R
2 electrostatic potential at a point P a distance from the
2
12. Four electric charges +q, +q, –q and – q are placed at the centre of the shell is [AIEEE 2003]
corners of a square of side 2L (see figure). The electric
potential at point A, midway between the two charges +q 2Q 2q 2Q 2q
(a) 4 - (b) +
and +q, is [CBSE-PMT 2011] p eo R 4 p eo R 4 p e o R 4 p eo R
1 2q +q –q (q + Q) 2 2Q
(a) (1 + 5) (c) 4 p e R + R (d)
4 p eo R
4p Î0 L o
19. The work done in placing a charge of 8 × 10–18 coulomb on
1 2q æ 1 ö A a condenser of capacity 100 micro-farad is [AIEEE 2003]
1+
4p Î0 L çè ÷
(b)
5ø (a) 3.1 × 10–26 joule (b) 4 × 10–10 joule
–32
(c) 32 × 10 joule (d) 16 × 10–232 joule
1 2q æ 1 ö –q
(c) ç 1- ÷ +q 20. A sheet of aluminium foil of negligible thickness is
4p Î0 L è 5ø introduced between the plates of a capacitor. The
(d) zero capacitance of the capacitor [AIEEE 2003]
13. Four point charges –Q, –q, 2q and 2Q are placed, one at (a) decreases (b) remains unchanged
each corner of the square. The relation between Q and q for (c) becomes infinite (d) increases
which the potential at the centre of the square is zero is 21. A parallel plate capacitor is made by stacking n equally
[CBSE-PMT 2012S] spaced plates connected alternatively. If the capacitance
between any two adjacent plates is ‘C’ then the resultant
1 capacitance is [AIEEE 2005]
(a) Q = – q (b) Q = –
q (a) (n + 1) C (b) (n – 1) C
(c) nC (d) C
1
(c) Q = q (d) Q = 22. A fully charged capacitor has a capacitance ‘C’. It is
q discharged through a small coil of resistance wire embedded
14. A parallel plate capacitor has a uniform electric field E in the in a thermally insulated block of specific heat capacity ‘s’
space between the plates. If the distance between the plates and mass ‘m’. If the temperature of the block is raised by
is d and area of each plate is A, the energy stored in the
‘ DT ’, the potential difference ‘V’ across the capacitance is
capacitor is : [CBSE-PMT 2012S] [AIEEE 2005]
1
(a) e0 E 2 (b) E2 Ad/e0 mCDT 2mC DT
2 (a) (b)
s s
1
(c) e0 E 2 Ad (d) e0EAd 2msDT msDT
2 (c) (d)
15. A, B and C are three points in a uniform electric field. The C C
electric potential is [NEET 2013] 23. Two point charges + 8q and – 2q are located at x = 0 and
x = L respectively. The location of a point on the x axis at
which the net electric field due to these two point charges
A is zero is [AIEEE 2005]
B ®
E L
(a) (b) 2 L (c) 4 L (d) 8 L
C 4
24. Two thin wire rings each having a radius R are placed at a
(a) maximum at B distance d apart with their axes coinciding. The charges on
(b) maximum at C the two rings are +q and -q. The potential difference between
(c) same at all the three points A, B and C the centres of the two rings is [AIEEE 2005]
(d) maximum at A q é1 1 ù qR
(a) ê - ú (b)
16. On moving a charge of 20 coulombs by 2 cm, 2 J of work is 2pÎ0 êë R R + d2
2
úû 4p Î0 d 2
done, then the potential difference between the points is
[AIEEE 2002] q é1 1 ù
(c) ê - ú (d) zero
(a) 0.1 V (b) 8 V (c) 2 V (d) 0.5 V 4pÎ0 ëê R R + d2
2
ûú
Free eBooks on @neetquestionpaper2020

600 Physi cs
25. Two spherical conductors A and B of radii 1 mm and 2 mm from rest, if a charge Q moves from the middle of side 1 to
are separated by a distance of 5 cm and are uniformly the centre of square, its kinetic energy at the centre of square
charged. If the spheres are connected by a conducting wire is [AIEEE 2011]
then in equilibrium condition, the ratio of the magnitude of 1 2 qQ æ 1 ö
the electric fields at the surfaces of spheres A and B is 1-
4pe 0 a çè ÷
(a) (b) zero

[AIEEE 2006]
(a) 4 : 1 (b) 1 : 2 (c) 2 : 1 (d) 1 : 4 1 2 qQ æ 1 ö 1 2 qQ æ 2 ö
(c) ç 1+ ÷ (d) ç 1- ÷
26. An electric charge 10–3 m C is placed at the origin (0, 0) of 4pe 0 a è 5ø 4pe 0 a è 5ø
X – Y co-ordinate system. Two points A and B are situated at 31. Combination of two identical capacitors, a resistor R and a

∋ (
DC voltage source of voltage 6 V is used in an experiment
2, 2 and (2, 0) respectively. The potential difference on C-R circuit. It is found that for a parallel combination of
between the points A and B will be [AIEEE 2007] the capacitor the time in which the voltage of the fully
(a) 4.5 volt (b) 9 volt charged combination reduces to half its original voltage is
(c) zero (d) 2 volt 10 s. For series combination the time needed for reducing
27. Charges are placed on the vertices of a square as shown. the voltage of the fully charged series combination by half
ur is [AIEEE 2011]
Let E be the electric field and V the potential at the centre. (a) 20 s (b) 10 s (c) 5 s (d) 2.5 s
If the charges on A and B are interchanged with those on D 32. The electrostatic potential inside a charged spherical ball is
and C respectively, then [AIEEE 2007]
q q given by f = ar 2 + b where r is the distance from the centre
a, b are constants. Then the charge density inside the ball
A B is [AIEEE 2011]
(a) – 6a e0 r (b) – 24p ae0
(c) – 6a e0 (d) – 24pa e0 r
33. Two capacitors C1 and C2 are charged to 120 V and 200 V
D C
respectively. It is found that by connecting them together
-q -q
ur the potential on each one can be made zero. Then
(a)E changes, V remains unchanged [JEE Main 2013]
ur
(b) E remains unchanged, V changes (a) 5C1 = 3C2 (b) 3C1 = 5C2
ur
(c) both E and V change (c) 3C1 + 5C2 = 0 (d) 9C1 = 4C2
ur 34. A charge Q is uniformly distributed over a long rod AB of
(d) E and V remain unchanged length L as shown in the figure. The electric potential at the
28. A parallel plate condenser with a dielectric of dielectric point O lying at distance L from the end A is
constant K between the plates has a capacity C and is [JEE Main 2013]
charged to a potential V volt. The dielectric slab is slowly
O A B
removed from between the plates and then reinserted. The
net work done by the system in this process is L L
Q 3Q
[AIEEE 2007] (a) 8pe L (b) 4pe L
1 0 0
(a) zero (b) (K - 1) CV 2 Q Q ln 2
2
(c) 4pe0 L ln 2 (d) 4pe0 L
2
(c) CV (K - 1) (d) (K - 1) CV 2
35. A long, hollow conducting cylinder is kept coaxially inside
K
29. A parallel plate capacitor with air between the plates has another long, hollow conducting cylinder of larger radius.
capacitance of 9 pF. The separation between its plates is Both the cylinders are initially electrically neutral. Then
‘d’. The space between the plates is now filled with two [IIT JEE 2007]
dielectrics. One of the dielectrics has dielectric constant (a) a potential difference appears between the two
cylinders when a charge density is given to the inner
d cylinder.
k1 = 3 and thickness while the other one has dielectric
3 (b) a potential difference appears between the two
cylinders when a charge density is given to the outer
2d
constant k2 = 6 and thickness . Capacitance of the cylinder.
3 (c) no potential difference appears between the two
capacitor is now [AIEEE 2008] cylinders when a uniform line charge is kept along the
(a) 1.8 pF (b) 45 pF (c) 40.5 pF (d) 20.25 pF axis of the cylinders
30. Two positive charges of magnitude q are placed at the ends (d) no potential difference appears between the two
of a side 1 of a square of side 2a. Two negative charges of cylinders when same charge density is given to both
the same magnitude are kept at the other corners. Starting the cylinders.
Free eBooks on @neetquestionpaper2020

Electrostatic Potential & Capacitance 601


36. A 2 mF capacitor is charged as shown in the figure. The 39. Six point charges are kept at the vertices of a regular
percentage of its stored energy dissipated after the switch hexagon of side L and centre O, as shown in the figure.
S is turned to position 2 is [IIT-JEE 2011 ] 1 q
1 2 Given that K = , which of the following
4pe 0 L2
S statement(s) is (are) correct? [IIT-JEE 2012 ]
V 2µF 8µF L E–q
+ qF
P
(a) 0% (b) 20%
(c) 75% (d) 80%
37. Consider a thin spherical shell of radius R with centre at the
A S T D
origin, carrying uniform positive surface charge density. The +2q O
–2q
r
variation of the magnitude of the electric field E(r ) and the
electric potential V(r) with the distance r from the centre, is R
best represented by which graph? [IIT-JEE 2012 ] B+ q
® – qC
|E(r)| V(r)
(a) The electric field at O is 6K along OD
(b) The potential at O is zero
(a) (c) The potential at all points on the line PR is same
(d) The potential at all points on the line ST is same
O R r 40. In the circuit shown in the figure, there are two parallel
® plate capacitors each of capacitance C. The switch S1 is
|E(r)| V(r) pressed first to fully charge the capacitor C1 and then
released. The switch S2 is then pressed to charge the
capacitor C2. After some time, S2 is released and then S3 is
(b) pressed. After some time [JEE Adv. 2013]

O R r S1 S2 S3
®
|E(r)| V(r)
2V0 C1 C2 C3

(c)

O R r
® (a) the charge on the upper plate of C1 is 2CV0
|E(r)| V(r)
(b) the charge on the upper plate of C1 is CV0
(c) the charge on the upper plate of C2 is 0
(d) (d) the charge on the upper plate of C2 is – CV0
41. Two non-conducting spheres of radii R1 and R2 and carrying
uniform volume charge densities +r and –r, respectively,
O R r
are placed such that they partially overlap, as shown in the
38. In the given circuit, a charge of +80 mC is given to the upper
figure. At all points in the overlapping region
plate of the 4 mF capacitor. Then in the steady state, the charge
on the upper plate of the 3 mF capacitor is [IIT-JEE 2012 ]
r –r

R1 R2

[JEE Adv. 2013]


(a) the electrostatic field is zero
(b) the electrostatic potential is constant
(c) the electrostatic field is constant in magnitude
(a) + 32 mC (b) + 40 mC
(c) + 48 mC (d) + 80 mC (d) the electrostatic field has same direction
Free eBooks on @neetquestionpaper2020

602 Physi cs

1. The positive terminal of 12 V battery is connected to the 12. The electric potential V is given as a function of distance ×
ground. Then the negative terminal will be at (metre) by
(a) – 6 V (b) + 12 V (c) zero (d) – 12 V V = (5 x2 + 10 x – 4) volt. Value of electric field at x = 1 m is
2. Three charges 2 q, – q and – q are located at the vertices of (a) – 23 V/m (b) 11 V/m
an equilateral triangle. At the centre of the triangle (c) 6 V/m (d) – 20 V/m
(a) the field is zero but potential is non-zero 13. Find the dipole moment of a system where the potential
(b) the field is non-zero, but potential is zero 2.0 × 10–5 V at a point P, 0.1m from the dipole is 3.0 × 104.
(c) both field and potential are zero (Use q = 30°).
(d) both field and potential are non-zero (a) 2.57 × 10–17 Cm (b) 1.285 × 10–15 Cm
3. A positive point charge q is carried from a point B to a point –17
(c) 1.285 × 10 Cm (d) 2.57 × 10–15 Cm
A in the electric field of a point charge + Q at O. If the 14. A battery of e.m.f. V volt, resistors R1 and R2, a condenser
permitivity of free space is e0, the work done in the process C and switches S1 and S2 are connected in a circuit shown.
is given by The condenser will get fully charged to V volt when
qQ æ1 1ö qQ æ1 1ö
(a) ç + ÷ (b) ç - ÷ R1 S1 S2 R2
4 p eo èa bø 4 p eo èa bø
qQ æ 1 1 ö qQ æ 1 1 ö
ç - ÷ ç + ÷
4 p e o çè a 2 b 2 ÷ø 4 p e o çè a 2 b 2 ÷ø
(c) (d)
V C
4. Two conducting spheres of radii r 1 and r 2 are equally
charged. The ratio of their potentials is
(a) r1 / r2 (b) r2 / r1 (c) r12 / r22 (d) r22 / r12 (a) S1 and S2 are both closed
5. The electric potential due to a small electric dipole at a large (b) S1 and S2 are both open
distance r from the centre of the dipole is proportional to (c) S1 is open and S2 is closed
(a) r (b) 1/r (c) 1/r2 (d) 1/r3 (d) S1 is closed and S2 is open
6. A cube of a metal is given a positive charge Q. For this 15. A and B are two points in an electric field. If the work done
system, which of the following statements is true? in carrying 4.0C of electric charge from A to B is 16.0 J, the
(a) Electric potential at the surface of the cube is zero potential difference between A and B is
(b) Electric potential within the cube is zero (a) zero (b) 2.0 V (c) 4.0 V (d) 16.0 V
(c) Electric field is normal to the surface of the cube 16. Two concentric, thin metallic spheres of radii R1 and R2
(d) Electric field varies within the cube (R1 > R2) bear charges Q1 and Q2 respectively. Then the
7. An electron of mass m and charge e is accelerated from rest potential at distance r between R 1 and R 2 will be
through a potential difference V in vacuum. Its final speed æ 1 ö
will be çè k = 4pe ÷ø
0
2e V eV
(a) (b) (c) e V/2m (d) e V/m æ Q + Q2 ö æQ Q ö
m m (a) kç 1 ÷ø (b) kç 1 + 2 ÷
8. A hollow metal sphere of radius 5 cm is charged such that è r è r R2 ø
the potential on its surface is 10 V. The potential at a distance æQ Q ö æ Q1 Q2 ö
of 2 cm from the centre of the sphere is (c) kç 2 + 1÷ (d) k ç R + R ÷
è r R 1ø è 1 2ø
(a) zero (b) 10 V (c) 4 V (d) 10/3 V 17. Three point charges +q , + 2q and – 4q where q = 0.1 mC, are
9. The electric potential at the surface of an atomic nucleus placed at the vertices of an equilateral triangle of side 10 cm
(Z = 50) of radius of 9 × 10–15 m is as shown in figure. The potential energy of the system is
(a) 80 V (b) 8 × 106 V (c) 9 V (d) 9 × 105 V – 4q
10. –8
A ball of mass 1 g carrying a charge 10 C moves from a
point A at potential 600 V to a point B at zero potential. The
10
cm

change in its K.E. is


cm
10

(a) – 6 × 10–6 erg (b) – 6 × 10–6 J


(c) 6 × 10 J–6 (d) 6 × 10–6 erg
11. An alpha particle is accelerated through a potential +q 10 cm +2q
difference of 106 volt. Its kinetic energy will be (a) 3 × 10–3 J (b) –3 × 10–3 J
(a) 1 MeV (b) 2 MeV (c) 4 MeV (d) 8 MeV (c) 9 × 10–3 J (d) –9 × 10–3 J
Free eBooks on @neetquestionpaper2020

Electrostatic Potential & Capacitance 603


18. A parallel plate condenser is immersed in an oil of dielectric 27. The capacitor, whose capacitance 6mF 6mF 3mF
constant 2. The field between the plates is is 6, 6 and 3mF respectively are
(a) increased, proportional to 2 connected in series with 20 volt
line. Find the charge on 3mF.
1
(b) decreased, proportional to (a) 30 mc
2 (b) 60 mF
(c) increased, proportional to – 2 (c) 15 mF 20m F
1 (d) 90 mF
(d) decreased, proportional to - 28. Four metallic plates each with a surface area of one side A,
2
19. Two capacitors of capacitances C1 and C2 are connected in are placed at a distance d from each other. The two outer
parallel across a battery. If Q1 and Q2 respectively be the plates are connected to one point A and the two other inner
plates to another point B as shown in the figure. Then the
Q
charges on the capacitors, then 1 will be equal to capacitance of the system is
Q2
2
C2 C1 C1 C 22 A B
(a) (b) (c) (d)
C1 C2 C 22 C12
20. A one microfarad capacitor of a TV is subjected to 4000 V e0A 2e 0 A 3e 0 A 4e 0 A
(a) (b) (c) (d)
potential difference. The energy stored in capacitor is d d d d
(a) 8 J (b) 16 J 29. Two spherical conductors A and B of radii a and b (b>a) are
(c) 4 × 10–3 J (d) 2 × 10–3 J placed concentrically in air. The two are connected by a
21. The four capacitors, each of 25 m F are connected as shown copper wire as shown in figure. Then the equivalent
in fig. The dc voltmeter reads 200 V. The charge on each capacitance of the system is
plate of capacitor is ab
(a) 4pe 0
V b-a b B
A
– + – +
(b) 4pe 0 (a + b) a

– + – + (c) 4pe 0 b
(d) 4pe 0a
30. Force between two plates of a capacitor is
(a) ± 2 ´ 10 -3 C (b) ± 5 ´ 10 -3 C
(c) ± 2 ´ 10 -2 C (d) ± 5 ´ 10 -2 C (a)
Q
(b) Q2
eoA 2 eo A
22. An air capacitor of capacity C = 10 mF is connected to a
constant voltage battery of 12 volt. Now the space between Q2
the plates is filled with a liquid of dielectric constant 5. The (c) (d) None of these
eoA
(additional) charge that flows now from battery to the
31. If the potential of a capacitor having capacity 6 mF is increased
capacitor is
from 10 V to 20 V, then increase in its energy will be
(a) 120 m C (b) 600 m C (c) 480 m C (d) 24 m C
(a) 4 × 10–4 J (b) 4 × 10–4 J
23. A capacitor is charged to store an energy U. The charging –4
(c) 9 × 10 J (d) 12 × 10–6 J
battery is disconnected. An identical capacitor is now
32. When air is replaced by a dielectric medium of force
connected to the first capacitor in parallel. The energy in
constant K, the maximum force of attraction between two
each of the capacitors is
charges, separated by a distance
(a) 3 U/2 (b) U (c) U/4 (d) U/2
(a) decreases K-times (b) increases K-times
24. Two capacitors when connected in series have a
1
capacitance of 3 mF, and when connected in parallel have a (c) remains unchanged (d) becomes times
capacitance of 16 mF. Their individual capacities are K2
(a) 1 mF, 2 mF (b) 6 mF, 2 mF 33. Two capacitors C1 and C2 in a circuit are joined as shown in
(c) 12 mF, 4 mF (d) 3 mF, 16 mF figure. The potentials of points A and B are V1 and V2
25. Capacitance (in F) of a spherical conductor with radius 1 m respectively; then the potential of point D will be
is A B
(a) 1.1 × 10–10 (b) 106 V1 D V2
C1 C2
(c) 9 × 10–9 (d) 10–3
26. The capacity of a parallel plate condenser is 10 mF, when (V1 + V2 ) C 2 V1 + C1V2
(a) (b)
the distance between its plates is 8 cm. If the distance 2 C1 + C 2
between the plates is reduced to 4 cm, then the capacity of C1V1 + C 2 V2 C 2 V1 + C1V2
this parallel plate condenser will be (c) (d)
C1 + C 2 C1 + C 2
(a) 5 mF (b) 10 mF (c) 20 mF (d) 40 mF
Free eBooks on @neetquestionpaper2020

604 Physi cs
34. A conductor carries a certain charge. When it is connected 43. An air capacitor C connected to a battery of e.m.f. V acquires
to another uncharged conductor of finite capacity, then the a charge q and energy E. The capacitor is disconnected
energy of the combined system is from the battery and a dielectric slab is placed between the
(a) more than that of the first conductor plates. Which of the following statements is correct ?
(b) less than that of the first conductor (a) V and q decrease but C and E increase
(c) equal to that of the first conductor (b) V remains unchange, but q, E and C increase
(d) uncertain (c) q remains unchanged, C increases, V and E decrease
35. The magnitude of the electric field E in the annular region (d) q and C increase but V and E decrease.
of a charged cylindrical capacitor 44. Two capacitors C1 and C2 = 2C1 are Q C1
(a) is same throughout connected in a circuit with a switch between
(b) is higher near the outer cylinder than near the inner them as shown in the figure. Initially the
R
cylinder switch is open and C1 holds charge Q. The
1 switch is closed. At steady state, the charge
(c) varies as , where r is the distance from the axis
r on each capacitor will be
1 Q 2Q
C2= 2C1
(d) varies as , where r is the distance from the axis
r2 (a) Q, 2Q (b) ,
3 3
36. A parallel plate capacitor with air between the plates is
3Q 2Q 4Q
charged to a potential difference of 500V and then insulated. (c) ,3Q (d) ,
A plastic plate is inserted between the plates filling the 2 3 3
45. Two capacitors of capacitance C are connected in series. If
whole gap. The potential difference between the plates now
one of them is filled with dielectric substance k, what is the
becomes 75V. The dielectric constant of plastic is
effective capacitance ?
(a) 10/3 (b) 5 (c) 20/3 (d) 10
37. The plates of a parallel plate capacitor have an area of kC
(a) 1 + k (b) C(k + 1)
90 cm2 each and are separated by 2.5 mm. The capacitor is ( )
charged by a 400 volt supply. How much electrostatic 2kC
energy is stored by the capacitor? (c) (d) None of these
1+ k
(a) 2.55 × 10–6 J (b) 1.55 × 10–6 J 46. The potential at a point x (measured in m m) due to some
(c) 8.15 × 10 J –6 (d) 5.5 × 10–6 J
charges situated on the x-axis is given by V(x) = 20/(x2 – 4)
38. From a supply of identical capacitors rated 8 mF, 250V, the
volt
minimum number of capacitors required to form a composite
The electric field E at x = 4 m m is given by
16 mF, 1000V is
(a) 2 (b) 4 (c) 16 (d) 32 (a) (10/9) volt/ m m and in the +ve x direction
39. Calculate the area of the plates of a one farad parallel plate (b) (5/3) volt/ m m and in the –ve x direction
capacitor if separation between plates is 1 mm and plates (c) (5/3) volt/ m m and in the +ve x direction
are in vacuum (d) (10/9) volt/ m m and in the –ve x direction
(a) 18 × 108 m2 (b) 0.3 × 108 m2
8 2 47. A large insulated sphere of radius r charged with Q units of
(c) 1.3 × 10 m (d) 1.13 × 108 m2
electricity is placed in contact with a small insulated
40. A system of two parallel plates, each of area A, are separated
uncharged sphere of radius r´ and is then separated. The
by distances d1 and d2. The space between them is filled
charge on smaller sphere will now be
with dielectrics of permittivities e1 and e2. The permittivity
(a) Q (r' + r) (b) Q (r + r')
of free space is e0. The equivalent capacitance of the system
is Q Qr'
(c) (d)
e1e2 A e1e 2 e0 A r '+ r r '+ r
(a) e d +e d (b) e d +e d 48. Two metal pieces having a potential difference of 800 V are
2 1 1 2 11 2 2
0.02 m apart horizontally. A particle of mass 1.96 × 10–15 kg
e0 A e0 A is suspended in equilibrium between the plates. If e is the
(c) e1d1+e2d2 (d) e d +e d elementary charge, then charge on the particle is
1 2 2 1
41. Two capacitors, C1 = 2mF and C2 = 8 mF are connected in (a) 8 (b) 6 (c) 0.1 (d) 3
series across a 300 V source. Then 49. Identical charges – q each are placed at 8 corners of a cube
(a) the charge on each capacitor is 4.8×10–4 C of each side b. Electrostatic potential energy of a charge
(b) the potential difference across C1 is 60 V + q which is placed at the centre of cube will be
(c) the potential difference across C2 is 240 V - 4 2 q2 - 8 2 q2
(d) the energy stroed in the system is 5.2 × 10–2 J (a)
p eo b
(b)
p eo b
42. If a capacitor 900 µF is charged to 100 V and its total energy
is transferred to a capacitor of capacitance 100 µF then its - 4 q2 - 8 2 q2
potential is (c) (d)
3 p eo b p eo b
(a) 200 V (b) 30 V (c) 300 V (d) 400 V
Free eBooks on @neetquestionpaper2020

Electrostatic Potential & Capacitance 605


50. A charge +q is fixed at each of the points x = x0, x = 3x0, 56. A circuit is connected as shown in the figure with the switch
x = 5x0, .... upto ¥ on X-axis and charge –q is fixed on each S open. When the switch is closed, the total amount of
of the points x = 2x0, x = 4x0, x = 6x0, .... upto ¥ . Here x0 is charge that flows from Y to X is
a positive constant. Take the potential at a point due to a 3mF 6mF
X
Q
charge Q at a distance r from it to be . Then the
4pe 0 r
potential at the origin due to above system of charges will be
S
q
(a) zero (b) 3W 6W
8pe 0 x 0 log e 2
Y
q log e 2
(c) infinity (d) 9V
4pe 0 x 0 (a) 0 (b) 54mC (c) 27mC (d) 81 mC
51. Two equally charged spheres of radii a and b are connected 57. If a slab of insulating material 4 × 10–5 m thick is introduced
together. What will be the ratio of electric field intensity on between the plates of a parallel plate capacitor, the distance
their surfaces? between the plates has to be increased by 3.5 × 10–5 m to
a a2 b b2 restore the capacity to original value. Then the dielectric
(a) (b) (c ) (d) constant of the material of slab is
b b2 a a2
(a) 8 (b) 6 (c) 12 (d) 10
52. In a hollow spherical shell, potential (V) changes with respect
58. Three capacitors each of capacity 4mF are to be connected
to distance (s) from centre as
in such a way that the effective capacitance is 6 mF. This can
(a) (b) be done by
(a) connecting two in parallel and one in series
V V (b) connecting all of them in series
(c) connecting them in parallel
S (d) connecting two in series and one in parallel
S
59. If we increase ‘d’ of a parallel plate condenser to ‘2d’ and fill
(c) (d)
wax to the whole empty space between its two plate, then
V V capacitance increase from 1pF to 2pF. What is the dielectric
constant of wax?
(a) 2 (b) 4 (c) 4 (d) 8
S S
60. Two spherical conductors A and B of radii a and b (b > a)
53. A parallel plate capacitor of capacitance C is connected to are placed concentrically in air. B is given charge +Q and A
a battery and is charged to a potential difference V. Another is earthed. The equivalent capacitance of the system is
capacitor of capacitance 2C is similary charged to a potential + + B
ab
+
+
+
difference 2V. The charging battery is now disconnected (a) 4 pe 0

+
b-a
+

and the capacitors are connected in parallel to each other in b

+ + + +
A–
+ + + +

such a way that the positive terminal of one is connected to (b) 4pe 0 (a + b) – –
– a O –
the negative terminal of the other. The final energy of the (c) 4pe 0 b
configuration is – –
æ b2 ö –
+

ç ÷
+

3 25 9 (d) 4pe 0 ç
+

CV 2 (c) CV 2 (d) CV 2 ÷
+

(a) zero (b) è b -a ø ++ +


2 6 2 + +
54. A solid conducting sphere having a charge Q is surrounding 61. The capacitance of a parallel plate capacitor is Ca (Fig. a). A
by an uncharged concentric conducting hollow spherical dielectric of dielectric constant K is inserted as shown in
shell. Let the potential difference between the surface of fig (b) and (c). If Cb and Cc denote the capacitances in fig
the solid sphere and that of the outer surface of the hollow (b) and (c), then
shell be V. If the shell is now given a charge of – 3Q, the new d/2
potential difference between the same two surfaces is d
K
(a) V (b) 2 V (c) 4 V (d) – 2 V
55. In the electric field of an point charge A Ca Cb
(a) (b)
q, a certain charge is carried from point
A to B, C, D and E. Then the work +
done is q d K (c)
(a) least along the path AB B E
(b) least along the path AD C D Cc
(c) zero along any one of the path AB, AC, AD andAE (a) both Cb, Cc > Ca (b) Cc > Ca while Cb > Ca
(d) least along AE (c) both Cb, Cc < Ca (d) Ca = Cb = Cc
Free eBooks on @neetquestionpaper2020

606 Physi cs
62. In the circuit shown, which of the following statements is (a) V (b) V
true if V1 (potential across C1) is 30 V and V2 (potential
across C2) is 20 V?

V1 = 30V V2 = 20V
S1 S3 S2 x x
(c) V (d) V
C1 = 2pF C 2 = 3pF

(a) With S1 closed, V1 = 15 V, V2 = 25 V x x


(b) With S3 closed, V1 = V2 = 25 V 66. Three capacitors C1, C2 and C3 are connected to a battery
(c) With S1 and S2 closed, V1 = V2 = 0 as shown. With symbols having their usual meanings, the
correct conditions are
(d) With S1 and S3 closed, V1 = 30 V, V2 = 20 V
V
63. A parallel plate capacitor is located horizontally such that Q2 2 C 2
one of the plates is submerged in a liquid while the other is V1
Q1 C1
above the liquid surface. When plates are charged the level
of liquid Q3 C3
(a) rises V3
(b) falls V
(c) remains unchanged
(d) may rise or fall depending Changed Liquid (a) Q1 = Q2 = Q3 and V1 = V2 = V
on the amount of charge (b) V1 = V2 = V3 = V
(c) Q1 = Q2 + Q3 and V = V1 = V2
64. Two small conductors A and B are given charges q 1 and
q2 respectively. Now they are placed inside a hollow metallic (d) Q2 = Q3 and V2 = V3
conductor C carrying a charge Q. If all the three 67. Figure (i) shows two capacitors connected in series and
conductors A, B and C are connected by a conducting wire connected by a battery. The graph (ii) shows the variation
as shown, the charges on A, B and C will be respectively of potential as one moves from left to right on the branch
AB containing the capacitors. Then
Q
C1 C2
A V
A B
q1 C

q2 B
E
q1 + q 2 q1 + q 2 (i) (ii)
(a) , ,Q
2 2 (a) C1 = C2
(b) C1 < C2
Q + q1 + q 3 Q + q1 + q 2 Q + q1 + q 2
(b) , , (c) C1 > C2
3 3 3 (d) C1 and C2 cannot be compared
q1 + q 2 + Q q 1 + q 2 + Q 68. Two vertical metallic plates carrying equal and opposite
(c) , ,0
2 2 charges are kept parallel to each other like a parallel plate
(d) 0, 0, Q + q1 + q2 capacitor. A small spherical metallic ball is suspended by a
long insulated thread such that it hangs freely in the centre
65. Between the plates of a parallel plate capacitor dielectric of the two metallic plates. The ball, which is uncharged, is
plate is introduced just to fill the space between the plates. taken slowly towards the positively charged plate and is
The capacitor is charged and later disconnected from the made to touch that plate. Then the ball will
battery. The dielectric plate is slowly drawn out of the (a) stick to the positively charged plate
capacitor parallel to plates. The plot of the potential
(b) come back to its original position and will remain
difference V across the plates and the length of the dielectric
there
plate drawn out is
(c) oscillate between the two plates touching each
plate in turn
(d) oscillate between the two plates without touch them
Free eBooks on @neetquestionpaper2020

Electrostatic Potential & Capacitance 607


69. Two parallel plate capacitors of capacitances C and 2C are 75. A battery is used to charge a parallel plate capacitor till the
connected in parallel and charged to a potential difference V. potential difference between the plates becomes equal to
The battery is then disconnected and the region between the the electromotive force of the battery. The ratio of the
plates of the capacitor C is completely filled with a material fo energy stored in the capacitor and the work done by the
dielectric constant K. The potential difference across the battery will be
capacitors now becomes (a) 1/2 (b) 1 (c) 2 (d) 1/4
3V V 3 76. Four point charges q, q, q and – 3q are placed at the vertices
(a) (b) KV (c) (d) of a regular tetrahedron of side L. The work done by electric
K+2 K KV
70. A parallel plate capacitor is connected to a battery. The force in taking all the charges to the centre of the tetrahedron
quantities charge, voltage, electric field and energy 1
associated with this capacitor are given by Q0, V0, E0, and is (where k = )
4pe 0
U0 respectively. A dielectric slab is now introduced to fill
the space between the plates with the battery still in 6kq 2 -6kq 2 12kq2
connection. The corresponding quantities now given by (a) (b) (c) (d) zero
L L L
Q, V, E and U are related to the previous ones as 77. Two identical particles each of mass m and having charges
(a) Q > Q0 (b) V > V0 (c) E > E0 (d) U < U0 – q and +q are revolving in a circle of radius r under the
71. The effective capacitance of combination of combination of influence of electric attraction. Kinetic energy of each
equal capacitors between points A and B shown in figure is
C C æ 1 ö
particle is ç k = ÷
è 4 pe 0ø
C C C (a) kq2/4r (b) kq2/2r (c) kq2/8r (d) kq2/r
78. Figure shows three circular arcs, each of radius R and total
C C
A B charge as indicated. The net electric potential at the centre
of curvature is
C C C +Q
Q
(a) 2pe R
C 0
(a) C (b) 2C (c) 3C (d) Q
2 45°
(b) 4pe 0 R
72. A parallel plate capacitor of plate area A and plate separation 30°
d is charged to potential difference V and then the battery –2Q
2Q
is disconnected. A slab of dielectric constant K is then (c) pe0 R
inserted between the plates of capacitor so as to fill the R
space between the plates. If Q, E and W denote respectively, Q +3Q
(d) pe R
the magnitude of charge on each plate electric field between 0
the plates (after the slab is inserted), and work done on the 79. The capacitance of the capacitor of plate areas A1 and A2
system, in question, in the process of inserting the slab, (A1 < A2) at a distance d, as shown in figure is
then which is wrong ? Î0 (A1 + A 2 ) A2
e 0 AV e 0 KAV (a) A1
2d
(a) Q = (b) Q =
d d Î0 A 2
(b)
V e 0 AV 2 æ 1ö d
(c) E= (d) W = ç 1- ÷
Kd 2d è K ø (c) Î0 A1A 2
73. In the circuit given below, the charge in mC, on the capacitor d
having 5 mF is Î0 A1
2mF (d) d
3mF d
e d 80. In the given circuit with steady current, the potential drop
across the capacitor must be
f 5mF
c A V R B
4mF
a + b
6V C
V
(a) 4.5 (b) 9 (c) 7 (d) 15
74. If a charge – 150 nC is given to a concentric spherical shell
2V 2R
and a charge +50 nC is placed at its centre then the charge
on inner and outer surface of the shell is V
2V V
(a) –50 nC, –100 nC (b) +50 nC, –200 nC (a) (b) (c) (d) V
3 3 2
(c) –50 nC, –200 nC (d) 50 nC, 100 nC
Free eBooks on @neetquestionpaper2020

608 Physi cs
81. A uniformly charged thin spherical shell of radius R carries
uniform surface charge density of s per unit area. It is made
of two hemispherical shells, held together by pressing them
with force F (see figure). F is proportional to

F F

1 2 2 1 2 Statement 2 : In a parallel plate capacitor both plates always


(a) s R (b) s R carry equal and opposite charge.
e0 e0
86. Statement 1 : Each of the plates of a parallel-plate capacitor
1 s2 1 s2 is given equal positive charge Q. The charges on the facing
(c) (d)
e0 R e0 R 2 surfaces will be same.
82. A dielectric slab of thickness d is inserted in a parallel plate Statement 2 : A negative charge (–Q) will be induced on
capacitor whose negative plate is at x = 0 and positive plate each of the facing surfaces.
is at x = 3d. The slab is equidistant from the plates. the 87. Statement 1 : Electric potential and electric potential energy
capacitor is given some charge. As one goes from 0 to 3d are different quantities.
Statement 2 : For a system of positive test charge and
(a) the magnitude of the electric field remains the same point charge electric potential energy = electric potential.
(b) the direction of the electric field remains the same 88. Statement I : Two equipotential surfaces cannot cut each
(c) the electric potential decreases continuously other.
(d) the electric potential increases at first, then decreases Statement II : Two equipotential surfaces are parallel to
and again increases each other.
83. In the given circuit if point C is connected to the earth and 89. Statement 1 : For a non-uniformly charged thin circular
a potential of +2000V is given to the point A, the potential ring with net charge is zero, the electric field at any point on
at B is axis of the ring is zero.
10mF 10mF Statement 2 : For a non-uniformly charged thin circular
ring with net charge zero, the electric potential at each point
5mF on axis of the ring is zero.
C
A 90. Statement-1 : For a charged particle moving from point P to
B
point Q, the net work done by an electrostatic field on the
10mF particle is independent of the path connecting point P to
point Q.
(a) 1500V (b) 1000 V (c) 500 V (d) 400 V Statement-2 : The net work done by a conservative force
84. A 4 mF capacitor, a resistance of 2.5 MW is in series with on an object moving along a closed loop is zero.
12V battery. Find the time after which the potential difference
Directions for Qs. (91 to 98) : Read the following passage(s)
across the capacitor is 3 times the potential diference across carefully and answer the questions that follows:
the resistor. [Given In (b) = 0.693]
PASSAGE 1
(a) 13.86s (b) 6.93 s (c) 7s (d) 14 s
Three large plates A, B and C are placed parallel to each other and
Directions for Qs. (85 to 90) : Each question contains charges are given as shown.
STATEMENT-1 and STATEMENT-2. Choose the correct answer
(ONLY ONE option is correct ) from the following-
(a) Statement -1 is false, Statement-2 is true
(b) Statement -1 is true, Statement-2 is true; Statement -2 is a
correct explanation for Statement-1
(c) Statement -1 is true, Statement-2 is true; Statement -2 is not
a correct explanation for Statement-1
(d) Statement -1 is true, Statement-2 is false 91. The charge that appears on the left surface of plate B is
(a) 5C (b) 6C (c) 3C (d) –3 C
85. Statement 1 : Charges are given to plates of two plane
parallel plate capacitors C1 and C2 (such that C2 = 2C1) as 92. The charge on inner surface of plate C, if plate B is earthed
shown in figure. Then the key K is pressed to complete the is
circuit. Finally the net charge on upper plate and net charge (a) 5C (b) 6C (c) 3C (d) –3 C
the circuit. Finally the net charge on upper plate and net 93. The charge on left surface of B, if B and C both are earthed
charge on lower plate of capacitor C1 is positive. is
(a) 5C (b) 6C (c) 3C (d) –3 C
Free eBooks on @neetquestionpaper2020

Electrostatic Potential & Capacitance 609


PASSAGE 2 PASSAGE 3
A non-conducting disc of radius ‘a’ and uniform surface charge A solid conducting sphere of radius a is surrounded by a thin
density s is placed on the ground, with its axis vertical. A particle uncharged concentric conducting shell of radius 2a. A point
of mass m and positive charge q is dropped, along the axis of the charge q is placed at a distance 4a from common centre of con-
disc, from a height H with zero initial velocity. The particle has q/ ducting sphere and shell. The inner sphere is then grounded.
m = 4pe0g/s.
94. Electrostatic potential at H is
s é 2
(a) (a + H 2 )1/2 - H ùû
e0 ë 2a
s é 2
(b) (a + H 2 )1/2 + H ùû a q
e0 ë
s é 2
(c) (a + H 2 )1/2 - H ùû
2e 0 ë
s é 2
(d) ë (a + H 2 )1/2 + H ùû 96. The charge on solid sphere is.
2e 0
(a) – q/2 (b) – q/4
95. Which of the following is the correct graph of the potential
(c) – q/8 (d) – q/16
energy of the particle as a function of its height?
97. Pick up the correct statement.
(a) Charge on surface of inner sphere is non-uniformly
distributed.
(b) Charge on inner surface of outer shell is non-uniformly
(a) (b) distributed.
(c) Charge on outer surface of outer shell is non-uniformly
distributed.
(d) All the above statements are false
98. The potential of outer shell is
q q
(c) (d) (a) 32pe a (b) 16pe a
0 0
q q
(c) 8pe a (d) 4pe a
0 0
Free eBooks on @neetquestionpaper2020

610 Physi cs

Exercise 17.1 Î0 AE
therefore the charge on it is + while the plate
d
1. (d) At a point on equatorial line of electric dipole, V=0. 4 is common to two capacitors (formed between plate
4. (a) On an equipotential surface the potential at any 3 and 4 and between 4 and 5), therefore it has a charge
point has the same value. 2 Î0 AE
5. (b) At. equipotential surface, the potential is same at any - .
point i.e., VA = VB as shown in figure. Hence no work d
is required to move unit change from one point to 3. (a) The charge on +vely charged plate of capacitor A
another i.e., cannot flow as it is isolated.
W Exercise 17.3
VA - VB = =0Þ W =0
unit ch arg e
1. (b,d) The capacitor is divided into two capacitors joined in
equipotential series. So (b) also as K for a metal is ¥ , therefore (d)
surface is also correct.
VA VB 2. (a) A capacitor works in a.c. circuits only as it blocks d.c.
3. (c) Electrostatic energy of a condenser lies in the field in
between the plates of the condenser.
4. (d) As the capacitor is isolated after charging, charge on
VAVB it remains constant. Plate separation increases d,
7. (c) By induction negative charge developes on suface of decreases C =Î0 A / d and hence increases potential
A close to B and positive charge developes on surface V = Q/C.
of A away from B, (when B carries positive charge). Ke o A
Therefore, overall potential of B decreases, charge of 5. (a) Increase, because C = .
B remains unchanged. d
6. (a) Because earth can store infinite charge so its
Exercise 17.2 capacitance is infinite, therefore it is at a zero potential.
æ qö
çC = ÷
è Vø
C1C 2 (V0 - 0) 2
1. (a) Loss of Energy = 7. (b) Energy stored per unit volume
2(C1 + C 2 )
1 1 æ V 2 ö÷ 1 V 2 æ Vö
C V 2C 1 U = eo E 2 = eo ç = eo ç\ E = ÷
DU = 1 0 2 but C1V02 = U 0 2 ç
2 èd ø 2 2 ÷ d 2
è dø
2(C1 + C 2 ) 2
8. (b) Since q = CV & capacitance of two capacitor is more in
C2 U 0 parallel combination as compared to series
\ DU = combinations i.e., CP > CS.
(C1 + C 2 )
2. (a) The arrangement is equivalent to four capacitors each Ke o A
9. (d) C=
Î A d
of capacity C = 0 . 10. (b) Q q µ V for q = CV
d Þ as charge on capacitor increases means P.D.
\ charge on each capacitor is between plates increases.
Î AE 11. (d) Since battery remains connected so P.D. between the
q = CV = CE = 0
d plates is constant. But as we introduce the dielectric,
Now, the plate 1 is common only to one condensor the capacitance increases and hence charge increases.
and is connected to the positive terminal of the battery,
Free eBooks on @neetquestionpaper2020

Electrostatic Potential & Capacitance 611

1 1 1 1 C 2 C
18. (b) Rows contain capacitors 1, , , ,..... mF
2 4 8 As C = C
3 4
(All in parallel). Therefore effective capacitance Hence no charge will flow through 20mF
1 1 1
C = 1+ + + + ..... C1 C2 C'
2 4 16
It is an infinite Geometrical progression with common
ratio 1/2. X Y ÞX Y

1
So, C = = 2mF C3 C4 C''
(1 - 1 / 2)
21. (a) From left hand side, the effective capacitance between C1 and C2 are in series, also C3 and C4 are in series.
the terminals is C and the ladder is infinite. Hence C' = 3 mF, C'' = 3 mF
The effective network is shown in figure. Now, the C' and C'' are in parallel hence net capacitance
capacitance between A and B is = C' + C'' = 3 + 3 = 6 mF
3. (c) Energy stored per unit volume
4C 6C + 8
C= +2= or C 2 + 4C - 6C - 8 = 0 2
C+4 C+4 1 1 æVö 1 V2 æ Vö
= e0 E 2 = e0 ç ÷ = e0 çQ E = ÷
solving C = 4 mF 2 2 èdø 2 d2 è dø
4. (a) Charge Q = C1V
Exercise 1 : NCERT Based Questions Total capacity of combination (parallel) C = C1+ C2
Q C1V
dV P.D. = =
1. E=– `2. Zero 3. 94 × 10–3 J. C C1 + C 2
dr
5. (a) We know that potential energy of two charge system
6. 15 mF. 14. (a) 2E (b) 18 E (c) 21 E
is given by U = 1 q1q 2
18. (d) 19. (c) 20. (a) 4 p Î0 r
21. (c) 22. (b) 23. (d) According to question,
24. (c) 25. (c) 1 ( +q )( -Q) 1 Qq
UA = 4 p Î =-
Exercise 2 : PAST Competition MCQs 0 a 4 pe 0 a

1. (d) When oil is placed between space of plates 1 ( + q )( -Q) 1 Qq


and UB = 4 p Î =-
a 4 pe 0 a
2 Ae 0 0
C= ... (1)
d DU = UB–UA = 0
When known that for conservative force,
é KAe 0 ù
êQ C = d , where K = 2ú W = –DU = 0
ë û 6. (c) We know that potential energy of discrete system of
Ae 0 charges is given by
When oil is removed C' = ............. (2)
d
on comparing both equation, weget C' = C/2 1 æ q1q 2 q2 q3 q3q1 ö
U= + +
C1 = 6mF 4p Î0 çè r12 r23 r31 ÷ø
2. (d)
According to question,
C3 C5 6mF
X A Y 1 æ q1q 2 q 2q3 q3q1 ö
6mF 20mF C2 Uinitial = + +
C4 4p Î0 çè 0.3 0.5 0.4 ÷ø
6mF
Equivalent circuit 1 æ q1q 2 q 2q3 q3q1 ö
Ufinal = + +
4p Î0 çè 0.3 0.1 0.4 ÷ø
6mF C2
C1
m
6F
6mF 1 æ q 2 q 3 q 2 q3 ö
Ufinal – Uinitial = -
X C5 20mF
m
6F Y
4p Î0 çè 0.1 0.5 ÷ø
C3 6mF 1 q3
6mF C4 = é10q q - 2q q ù = (8q 2 )
4p Î0 ë 2 3 2 3 û 4p Î0
Free eBooks on @neetquestionpaper2020

612 Physi cs

s Q
7. (b) 10. (d) Electric field E = =
e Ae
e of kerosine oil is more than that of air..
As e increases, E decreases.
1
11. (d) U = CV 2
2
1 æ A Î0 ö 2 1 2
U= çè ÷ø (Ed) = A Î0 E d
2 d 2
Equivalent capacitance for three capacitors
12. (c) Distance of point A from the two +q charges = L.
(C1, C2 & C3) in series is given by
Distance of point A from the two –q charges
1 1 1 1 C 2 C3 + C 3C1 + C1C 2
= + + =
C eq. C1 C 2 C 3 C1C 2 C3 = L2 + (2L)2 = 5L .

C1C 2 C 3 +q –q
Þ Ceq. =
C1C2 + C2 C 3 + C 3C1

C(2C)( 3C) 6 A 2L
Þ Ceq. = = C
C(2C) + (2C)(3C) + (3C) C 11
Þ Charge on capacitors (C1, C2 & C3) in series +q –q
6C
= C eq V = V 2Kq é 1 ù
11 æ Kq ö æ Kq ö
\ VA = ç ´ 2÷ - ç ´ 2÷ = ê1- ú
Charge on capacitor C4 = C4V = 4C V è L ø è 5L ø L ë 5û
6C
V 1 2q æ 1 ö
Charge on C2 11 6 1 3 = . 1-
= = ´ = 4p Î0 L çè 5 ÷ø
Charge on C4 4CV 11 4 22
13. (a) Let the side length of square be 'a' then potential at
C1 centre O is
8. (d) In series, Ceff =
n1 –Q –q
\ Energy stored,
O
1 1 C1 2 C1
ES = Ceff VS2 = 16V 2 = 8V
2 2 n1 n1 2Q 2q
In parallel, Ceff = n2 C2
k (-Q) k (-q) k (2q ) k (2Q)
1 V= + + + = 0 (Given)
\ Energy stored, Ep = n2C2V2 æ a ö a a a
2 çè ÷
2ø 2 2 2
8V 2 C1 1 16C1 = – Q – q + 2q + 2Q = 0 = Q + q = 0
\ = n2C2V 2 Þ C2 =
n1 2 n1n2 =Q=–q
14. (c) The energy stored by a capacitor
9. (a) q = CV1 cos wt
1
U= CV 2 ...(i)
dq 2
Þi = = –wCv1 sin wt
dt V is the p.d. between two plates of the capacitor.
1 The capacitance of the parallel plate capacitor
2
Also, w = and V = V1 cos w t V = E.d.
LC
At t = t1 , V = V2 and i = -wCV1 sin wt1 Ae 0
C=
d
V2
\ cos w t1 = V (–ve sign gives direction) Substituting the value of C in equation (i)
1
1 Ae 0 1 Ae0 E 2 d
1/ 2
æ C (V12
1/ 2 U= ( Ed )2 =
C æ V22 ö - V22 ö 2 d 2 d
Hence, i = V1 ç 1 - =ç ÷ø
L 2÷ è L
è V1 ø
Free eBooks on @neetquestionpaper2020

Electrostatic Potential & Capacitance 613

15. (a) Potential at B, VB is maximum At (1) using, potential (V1 ) = Vself + Vdue to ( 2)
VB > VC > VA
As in the direction of electric field potential decreases.
1 éq q ù
16. (a) Potential difference between two points in a electric Þ V1 = ê - ú
field is, 4pe 0 ê R 2 2 ú
R +d û
ë
W At (2) using potential (V2 ) = Vself + Vdue to (1)
VA - VB =
q0
where W is workdone by moving charge q0 from point 1 é -q q ù
Þ V = ê + ú
A to B. 2 4pe 0 ê R úû
ë R2 + d2
2
So, VA - VB = = 0. 1 V DV = V1 - V2
20

17. (b) V 1 éq q q q ù
= ê + - - ú
V 4pe 0 ê R R R2 + d2 R2 + d2 úû
ë
V
1 éq q ù
V = ê - ú
2pe 0 êë R 2 2ú
R +d û
V
+Q +Q
Energy stored in each capacitor = 1 CV 2
2 r1 r2
25. (c)
1
\ energy stored in n capacitors = nCV 2 A B
2
After connection, V1 = V2
18. (b)
Q1 Q Q Q
(8 ´ 10-18 )2 ÞK =K 2 Þ 1 = 2
1 q2 r1 r2 r1 r2
19. (c) Work done = = = 32 × 10–32 J
2 C 2 ´ 100 ´ 10-6 The ratio of electric fields
20. (b)
Q
K 1
21. (b) As n plates are joined, it means (n – 1) combination E1 r12 E Q r2
= Þ 1 = 1´ 2
joined in parallel. E2 Q E 2 r12 Q 2
K 2
\ Resultant capacitance = (n – 1) C r22
22. (c) Applying conservation of energy,
E1 r1 ´ r22 E r 2
1 Þ = Þ 1 = 2 =
CV 2 = m. s Dt ; E 2 r12 ´ r2 E 2 r1 1
2
Since the distance between the spheres is large as
2m. s. Dt compared to their diameters, the induced effects may
V= be ignored.
C
26. (c) Y
-K2q K8q 1 4
23. (b) + =0Þ =
(x - L) 2
x2 ( x - L) 2 x2
A(Ö2,Ö2)
®
1 2 r1
or = Þ x = 2x - 2L or x = 2L
x-L x
24. (a) q q O X
(0,0) ®
r2 B (2,0)
The distance of point A ( )
2, 2 from the origin,
ur 2 2
R R
OA = | r1 | = ( 2) + ( 2)
1 2
d = 4 = 2 units.
Free eBooks on @neetquestionpaper2020

614 Physi cs
The distance of point B(2, 0) from the origin, k1Î0 A 3k1Î0 A 3 ´ 3Î0 A 9Î0 A
ur 2 2
C1 = = = =
OB = | r2 | = (2) + (0) = 2 units. d/3 d d d
and
1 Q
Now, potential at A, VA = .
3k 2 Î0 A
4p Î0 (OA) k 2 Î0 A 3 ´6 Î0 A 9Î0 A
C1 = = = =
2d/3 2d 2d d
1 Q
Potential at B, VB = . The equivalent capacitance Ceq is
4p Î0 (OB)
\ Potential difference between the points A and B is 1 1 1 d d 2d
= + = + =
given by Ceq C1 C2 9 Î0 A 9 Î0 A 9 Î0 A
1 Q 1 Q 9 Î0 A 9
VA – VB = . - . \ Ceq = = ´ 9 pF = 40.5pF
4p Î0 OA 4p Î0 OB 2 d 2

Q æ 1 1 ö Q æ 1 1ö 2kqQ 2k ( - q)Q 1 2qQ é 1 ù


30. (a) U i = + = . ê1- ú
= çè - ÷ø = ç - ÷ a 5a 4pe0 a ë 5û
4p Î0 OA OB 4p Î0 è 2 2 ø
Uf =0 q 1Q q
10-3 ´10-6 By conservation of energy
= ´ 0 = 0.
4p Î0 Gain in KE = loss in PE

5a
2a
27. (a) As shown in the figure, the resultant electric fields 1 2 qQ é 1 ù
before and after interchanging the charges will have K= . ê1- ú
4 pe 0 a ë 5û
the same magnitude, but opposite directions.
–q 2a –q
Also, the potential will be same in both cases as it is a
31. (d) If Ce be the effective capacitance, then
scalar quantity.
1 C
q q -q -q VC = V0
2
A B A ® B
E q q C
= 0
Ce 2Ce
R
-t / RCeq
®
E Þ q0 (1 - e ) = 0 Þ t = RCe ln 2
2
D C D C
For parallel grouping
-q -q q q
2C
Ce =
28. (a) The potential energy of a charged capacitor is given 2
\ t2 = 2RC ln 2
Q2
by U = . For series grouping,
2C C C
C
If a dielectric slab is inserted between the plates, the Ce =
2
Q2 RC
energy is given by , where K is the dielectric R
2KC \ t1 = ln 2
2
constant. t2 1
Again, when the dielectric slab is removed slowly its \ t = 4 Þ t2 = 2.5s
1
energy increases to initial potential energy. Thus, work
done is zero. df
32. (c) Electric field, E = - = -2ar
29. (c) dt
q
By Gauss's theorem E (4pr 2 ) =
e0
Þ q = -8pe 0 ar 3
dq dq dr = (–24pe ar 2 ) æ 1 ö
r= = ´ 0 çè ÷
dV dr dV 4pr 2 ø
The given capacitance is equal to two capacitances
= –6e 0 a
connected in series where
Free eBooks on @neetquestionpaper2020

Electrostatic Potential & Capacitance 615

C1 C2 37. (d) For a thin uniformly positive charged spherical shell


+ – – + (i) Inside the shell at any point
33. (b)
120 V 200 V 1 q
E = O and V = = constt.
For potential to be made zero, after connection 4 p Î0 R
where q = charge on sphere
é qù
êëQ C = v úû
120 C1 = 200 C2 R = Radius of sphere
(ii) Outside the shell at any point at any distance r
Þ 3C1 = 5C2
1 1
from the centre E µ and V µ
L L 2 r
34. (d) r
O 38. (c) The total charge on plate A will be – 80 µC. If qB and qC
A dx B be the charges on plate B and C then
x
qB + qC = 80 µC ...(1)
Electric potential is given by,

æ qö +80 mC
dx
1 çè L ÷ø
2L 2L q
kdq = ln(2)
V= ò x
= ò 4 pe 0 x 4pe 0 L 4mF
L L –80 mC A
35. (a) When a charge density is given to the inner cylinder,
the potential developed at its surface is different from +qB B +qC C
that on the outer cylinder. This is because the potential 3mF
decreases with distance for a charged conducting 2mF –qC
–qB
cylinder when the point of consideration is outside
the cylinder. But when a charge density is given to the
outer cylinder, it will charge its potential by the same
amount as that of the inner cylinder. Therefore no
Also 2µF and 3µF capacitors are in parallel. Therefore,
potential difference will be produced between the
cylinders in this case. qB qC
=
36. (d) When S and 1 are connected 2 3
The 2mF capacitor gets charged. The potential 80 - qC qC
\ =
difference across its plates will be V. 2 3
The potential energy stored in 2 mF capacitor 240
\ 240 – 3qC = 2qC \ qC = = 48µC
1 1 5
U i = CV 2 = ´ 2 ´ V 2 = V 2 This charge will obviously be positive.
2 2 39. (a, b, c)
When S and 2 are connected EB + EE
The 8mF capacitor also gets charged. During this +q F E– q
charging process current flows in the wire and some
amount of energy is dissipated as heat. The energy + 2q –2q 60º
loss is A D O 60º EA ED P
1 C1 C2 2
DU = 2 C + C (V1 - V2 ) +q –q
1 2 EF + EC
B C
uuur uuur
Here, C1 = 2mF, C2 8 mF, V1= V, V2 = 0 EA uuur uuur ED uuur uur
1 2´8 4 Here = E B = EC = = EE = E F = K
\ DU = ´ (V - 0)2 = V 2 2 2
2 2+8 5 \ EO = EA + ED + (EF + EC) cos 60º + (EB + EC)
cos 60º
DU
The percentage of the energy dissipated = ´ 100 1 1
Ui = 2K + 2K + (K + K) × + (K + K) × = 6K
2 2
4 2 The electric potential at O is
V
= 5 ´ 100 = 80% 1
V2 VO = [2q + q + q – q – q – 2q]
4p e 0 L
=0
Free eBooks on @neetquestionpaper2020

616 Physi cs

PR is perpendicular bisector (the equatorial line) for the Q 1


electric dipoles AB, FE and BC. Therefore the electric 4. (b) As V = i.e. V µ
4 p e0r r
potential will be zero at any point on PR.
V1 r2
At any point ST, the electric field will be directed from S \ =
to T. The potential decreases along the electric field V2 r1
line. 1
5. (c) Due to small dipole, V µ 2 .
r
40. (b,d) 6. (d) Surface of metallic cube is an equipotential surface.
Step 1 : When S1 is pressed : The capacitor C1 gets Therefore, electric field is normal to the surface of the
charged such that its upper plate acquires a positive cube.
charge + 2 CV0 and lower plate – 2 CV0. 7. (a) K.E. = Work done = eV
Step 2 : When S2 is pressed (S1 open) : As C1 = C2 the
charge gets distributed equal. The upper plates of 1 2 eV
C1 and C2 now take charge + CV0 each and lower m n2 = eV \ n=
2 m
plate – CV0 each.
(b) and (d) are correct option. 8. (b) Potential at any point inside the sphere = potential at
41. (c, d) Let us consider a point P on the overlapping region. the surface of the sphere = 10V.
The electric field intensity at P due to positively
r V=
q
=
(Ze)
rr1 9. (b) 4 p e o r 4p Î0 r
charged sphere =
3 Î0
The electric field intensity at P due to negatively (50 ´ 1.6 ´ 10-19 )
r 9 ´ 109 = 8 ´ 10 6 V
rr 2 9 ´ 10-15
charged sphere = . The total electric field,
3 Î0 10. (c) As work is done by the field, K.E. of the body increases
ur ur by
ur rr1 rr2 r r r
E= + = ér1 + r 2 ù K.E. = W = q (VA - VB )
3 Î0 3 Î0 3 Î0 ë û
ur r r = 10 -8 (600 - 0) = 6 ´10 -6 J
E= r
3 Î0 11. (b) Charge on a particle, q = 2 e.
Therefore the electric field is same in magnitude and K.E. = work done = q × V = 2e × 106 V = 2 MeV.
direction option (c) and (d) are correct. 12. (d) V = 5 x2 + 10 x – 4
-dV
E= = - (10 x + 10).
dx
P At x = 1 m, E = –20 V/m.
r1 r2
+r
r
–r
13. (a)
Vr 2
= qd =
(2.0 ´10 -5
)
V ( 0.1m)
2

= 2.57 ´10 -17 Cm.


k cos q ì ü
ïï m ïï æ 3 ö
9
í9.0 ´10 ý
ï æ C ö ï çè 2 ÷ø
çè ÷ø
ïî V ïþ
Note that the units cancel to leave units appropriate
Exercise 3 : Conceptual & Applied MCQs
for a dipole moment.
1. (d) When negative terminal is grounded, positive terminal 14. (d) When S1 is closed and S2 is opened, the capacitor will
of battery is at +12 V. When positive terminal is get charged to a potential difference of V volts.
grounded, the negative terminal will be at –12 V. 15. (c) Since WA ® B = q(VB –VA)
2. (b) Potential at the centre of the triangle, 16
Þ VB – VA = = 4V
åq 2q -q - q 4
V= = =0
4 p e0 r 4 p e0 r
Obviously, E ¹ 0 Q2 Q1
3. (b) WBA = q (VA - VB ) 16. (c) R1 R2
r
é Q Q ù = qQ é1 1ù
=q ê - ú 4 p e0 êa - bú
ë 4 p e0 a 4 p e 0 b û ë û
Free eBooks on @neetquestionpaper2020

Electrostatic Potential & Capacitance 617


25. (a) Capacitance of spherical conductor = 4pe0a
Q2 Q1
Vr = + where a is radius of conductor.
4pe 0 r 4pe 0 R1
1 1
Therefore, C = ´1 = ´10 -9
1 æ Q 2 Q1 ö 9 ´ 10 9 9
Vr = ç + ÷
4pe0 è r R1 ø
= 0.11´10 -9 F = 1.1 ´ 10-10 F
26. (c) C = 10 mF ; d = 8 cm
1 æ 2q 2 8q 2 4q 2 ö
17. (d) U= ç - - ÷ C' = ? ; d' = 4 cm
4pe0 çè a a a ÷ø
A Î0 1
C= Þ Ca
d d
1 æ 10q 2 ö
Þ U = ç- ÷ If d is halved then C will be doubled.
4pe0 çè a ÷ø
Hence C' = 2C = 2 × 10 mF = 20 mF
1 1 1 1
9 ´ 109 ´ 10 ´ (0.1 ´10 -6 )2 27. (a) In series = + + and charge on each
U=- C C1 C 2 C3
Þ æ 10 ö
ç ÷ capacitor is same.
è 100 ø
28. (b) It consists of two capacitors in parallel, therefore, the
Þ U = – 9 × 10–3J
18. (b) In oil, C becomes twice, V becomes half. Therefore, 2 Î0 A
total capacitance is =
E = V/d becomes half. d
19. (b) In parallel, potential is same, say V
+ + + +
Q1 C1V C1 + + + +
= = – – – –
Q 2 C2V C2 – – – –
+A – – – – –B
– – – –
1 2 1 -6 2 + + + +
20. (a) E = CV = ´ 1 ´ 10 ´ (4000) = 8 J.
2 2 + + + +
21. (b) Charge on each plate of each capacitor (The plates of B, having negative charge do not
constitute a capacitor).
Q = ±CV = ±25 ´ 106 ´ 200 = ±5 ´ 10 -3 C 29. (c) All the charge given to inner sphere will pass on to
22. (c) q1 = C1V = 10 ´12 = 120 m C the outer one. So capacitance that of outer one is
4p Î0 b .
q 2 = C 2 V = KC1 ´ V = 5 ´10 ´ 12 = 600 m C
30. (b) The magnitude of electric field by any one plate is
Additional charge that flows
Q –Q
= q 2 - q1 = 600 - 120 = 480 m C.
s Q
23. (c) As battery is disconnected, total charge Q is shared or E
2e o 2Ae o
equally by two capacitors.
2 2
Energy of each capacitor = (Q / 2) = 1 Q = 1 U. Q2
2C 4 2C 4 Now force magnitude is Q E i.e. F =
2Ae o
C1 C 2 31. (c) Capacitance of capacitor (C) = 6 mF = 6 × 10–6 F;
24. (c) Cs = =3 Initial potential (V1) = 10 V and final potential
C1 + C 2
(V2) = 20 V. The increase in energy (DU)
C p = C1 + C 2 = 16 \ C1 C 2 = 48 1
1 -6 2 2
= C( V22 - V12 ) = ´ (6 ´ 10 ) ´ [( 20) - (10) ]
2 2
C1 - C 2 = (C1 + C 2 ) 2 - 4 C1 C 2
= (3 ´10 -6 ) ´ 300 = 9 ´10 - 4 J .
2
= 16 - 4 ´ 48 = 64 = 8
1 q1q 2
C1 + C2 = 16 mF 32. (a) In air Fair =
4pe 0 r 2
C1 - C2 = 8 mF 1 q1q 2
In medium Fm = 4pe 2
Þ 2C1 = 24mF Þ C1 = 12mF 0 Kr
Fm 1 F
48 \ = Þ Fm = air (decreases K-times)
\ C2 = = 4mF Fair K K
12
Free eBooks on @neetquestionpaper2020

618 Physi cs
33. (c) Consider the potential at D be ‘V’. 40. (a)
Potential drop across C1 is (V – V1 ) and C2 is
(V2 – V)
\ q 1 = C1(V - V1 ), q 2 = C2 (V2 - V)
A e1 e2 A
As q1 = q2 [capacitors are in series]
\ C1 (V - V1) = C2 (V2 - V)

C1V1 + C2 V2
v= d1 d2
C1 + C2
1 1 1
34. (b) Energy will be lost during transfer of charge (heating = +
effect). Cs C1 C2

l 1 1
Eµ 1 Þ =
e1A e 2A
35. (c) hence Eµ Cs +
2pe 0 r r d1 d 2
q 1 1 æ d1 d 2 ö
36. (c) V0 = Þ = +
C0 Cs A çè e1 e 2 ÷ø

q V C0 Ae1e 2
V= = Þ Cs =
Þ e 2 d1 + e1d 2
C V0 C
2´8
C 0 500 20 41. (a) Cs = = 1.6mF
Þ = = 2+8
C 75 3 Since, Q = Cs V = 1.6 × 10–6 × 300
20 Q = 4.8 × 10–4 C
By definition, C = kC0 Þ k =
3 4.8 ´ 10-4
V1 = = 240V
37. (a) Here, A = 90 cm² = 90 × 10–4 m2; 2 ´ 10-6
d = 2.5 mm = 2.5 × 10–3 m; V = 400 volt
4.8 ´ 10 -4
e A 8.854 ´ 10-12 ´ 90 ´ 10-4 V2 = = 60V
C= 0 = 8 ´ 10-6
d 2.5 ´ 10 -3
= 3.187 × 10–11 F Q2 Q2
U= +
1 1 2C1 2C2
CV 2 = ´ 3.187 ´ 10-11 ´ ( 400 )
2
W=
2 2
= 2.55 × 10–6 J
ÞU=
( 4.8 ´ 10 ) -4 2
æ 1 ö
38. (d) Let ‘n’ such capacitors are in series and such ‘m’ such 2 çè -6 ÷ø
1.6 ´ 10
branch are in parallel.
Þ U = 3 × 2.4 × 10–2 J
\ 250 × n = 1000 \ n = 4 … (i)
Þ U = 7.2 × 10–2 J
8 1 1
Also ´ m = 16 42. (c) C1V12 = C2 V22
n 2 2
16 ´ n because total energy is transferred (given).
m= =8 … (ii)
8 1 1
\ ´ 900 ´ 10 -6 ´ 1002 = ´ 100 ´ 10 -6 ´ V 2
\ No. of capacitor = 8 ´ 4 = 32 2 2
2
\ V = 90000 Þ V = 300 V..
e0 A
39. (d) For a parallel plate capacitor C = 43. (c) When a battery across the plates of capacitor is
d disconnected and dielectric slab is placed in between
the plates, then
Cd 1 ´ 10 -3
\ A= e = = 1.13 × 108 m2 (i) capacity C increases
0 8.85 ´ 10 -12 (ii) charge q remains unchanged
This corresponds to area of square of side 10.6 km (iii) potential V decreases
which shows that one farad is very large unit of (iv) energy E decreases
capacitance.
Free eBooks on @neetquestionpaper2020

Electrostatic Potential & Capacitance 619


44. (b) In steady state, both the capacitors are at the same
potential,
a b
Q1 Q2 Q Q 51. (c)
i.e., = or 1 = 2 or Q2 = 2Q1
C1 C2 C 2C
Also Q1 + Q2 = Q
Let charge on each sphere = q
Q 2Q when they are connected together their potential will
\ Q1 = , Q2 =
3 3 be equal .
45. (a)
Now let charge on a = q1 and on b = 2q - q1
20
46. (a) Here, V(x) = 2 volt 1 q1 1 2q - q1
x -4 Þ Va = Vb or =
4peo a 4pe o b
dV d æ 20 ö q1 a
We know that E = - =- ç ÷ Þ =
dx dx è x 2 - 4 ø 2q - q1 b
40x 1. q1
or, E= + 2 2
(x - 4) Ea 4peo a 2 æ q1 ö b 2
= =ç ÷ a b2 b
At x = 4 mm , Eb 1 q 2 è 2q - q1 ø a 2 = . 2 =
b a a
40 ´ 4 160 10 4peo b2
E= + =+= + volt / mm. = b:a
(42 - 4) 2 144 9
r 52. (b) In shell, q charge is uniformly distributed over its
Positive sign indicates that E is in +ve x-direction. surface, it behaves as a conductor.
+
Q 0 +
47. (d) Common potential, V = + + q
4 pe 0 r 4 pe 0 r
R +
Q+0 +
= +
4 pe 0 (r + r ') +
+
\ charge on smaller sphere of radius r' is
q
Q r¢ V= potential at surface = and inside
4 pe 0 r¢ ´ V = 4pe 0 R
r + r¢
q
V V=
48. (d) In equilibrium, F = q E = ( n e) = mg 4pe 0 R
d
Because of this it behaves as an equipotential surface.
mg d 1.96 ´ 10 -15 ´ 9.8 ´ 0.02 53. (b) Energy stored,
n= = =3
eV 1.6 ´ 10 -19 ´ 800 1 1 3
U = Caq V 2net = (3C)V 2 = CV 2
49. (c) Length of body diagnonal = 2 2 2
3b
+ C

\ Distance of centre of cube from each corner, V
3
r= b 2V
2 – +
2C
Total P.E. of charge + q at the centre 54. (a) V2
8 q ( -q ) - 8q 2 - 4 q2
= = = Q r1 V1
4 pe o r 4 pe o ( 3 b / 2) pe o 3 b
50. (d) Potential at origin r2

= (V1 + V3 + V5 + .....) – (V2 + V4 + V6 + .....) Situation 1 :


q é 1 1 1 ù Q Q é1 1 ù
Þ - + .....¥ú V1 - V2 = V = K - K = KQ ê - ú
ê r1 r2
4pe 0 ë x 0 2x 0 3x 0 û ë r1 r2 û
Situation 2 :
q é 1 1 1 ù
Þ ê1 - 2 + 3 - 4 .....¥ ú é KQ 3KQ ù é KQ 3KQ ù
4 pe 0 x 0 ë û V '1 - V '2 = V ' = ê - ú-ê - ú
ë r1 r2 û ë r2 r2 û
q q
Þ log e (1 + 1) Þ log e 2 é1 1 ù
4pe 0 x 0 4pe 0 x 0 = KQ ê - ú = V
ë r1 r2 û
Free eBooks on @neetquestionpaper2020

620 Physi cs
55. (c) Since, all A, B, C, D and E lie on an equipotential surface and the outside of the outer constitutes an isolated
so, sphere of capacitance 4p Î0 b .
W= 0 \ the effective capacitance is
56. (c) When steady state is reached, the current I coming
ab
from the battery is given by 4p Î0 + 4p Î0 b
b-a
3mF 6mF
X é a ù éa + b - a ù
+Q1 –Q1 +Q2 –Q2 = 4 p Î0 b ê + 1ú = 4 p Î0 b ê ú
ëb - a û ë b-a û
S
3W 6W b2
C = 4p Î0
b-a
Y Î0 A Î0 A 2 Î0 A (1 + K )
61. (a) Ca = and Cb = =
9V d d d d
+
9 = I (3 + 9) Þ I = 1A 2 2K
Þ potential difference across 3 W resistance = 3V A A
Î0 Î0 K
and potential difference across 6 W resistance 2 2 Î A
and C c = + = 0 (1 + K)
= 6V d d 2d
Þ p.d. across 3 mF capacitor = 3V Î0 A
and p.d. across 6mF capacitor = 6V or C b = 2(1 + K ) > C a
d
\ Charge on 3 mF capacitor, Q1 = 3 × 3 = 9 mC
Charge on 6mF capacitor, Q2 = 6 × 6 = 36 mC Î0 A 1 + K
or C c = > C a \ C b and C c > C a .
Þ Charge (–Q1) is shifted from the positive plate of d 2
6mF capacitor. The remaining charge on the positive 62. (d) When S1 and S3 is closed V1 = 30 V and potential
plate of 6 mF capacitor is shifted through the switch. drop across C2 becomes 20 V.
\ Charge passing through the switch 63. (a) The molecules of liquid will convert into induced
= 36 – 9 = 27 mC dipole, get oriented along the electric field produced
between the plates and rise due to force of attraction.
æ 1ö 64. (d) Charge given to a hollow conductor resides only on
57. (a) As x = t ç1 - ÷ , where x is the addition distance of
è Kø the outer surface.
plate, to restore the capacity of original value. (l - x )b Î0 Kxb Î0
65. (b) C = C1 + KC2 = +
æ 1ö d d
\ 3.5 ´ 10 -5 = 4 ´ 10 -5 ç1 - ÷ . b d
è Kø b Î0
Solving, we get, K = 8. = [l - x + Kx]
d
C1 ´ C 2 4´ 4 b Î0
58. (d) For series C' = = = 2mF C= [l + x(K - 1)] l
C1 + C 2 4 + 4 d
For parallel Ceq = C'+ C2 = 2 + 4 = 6mF q qd x K
V= =
C b Î0 [l + x(K - 1)]
A B as x decreases, V increases.
66. (c) C2 and C3 are parallel so V2 = V3
Ke o A
59. (b) C= C1 and combination of C2 & C3 is in series.
d
So, V = V2 + V1 or V = V3 + V1
1´ Î0 A
1 ´ 10-12 = .......... (1) and also Q1 = Q2 + Q3
d
67. (c) Since, potential difference across C2 is greater than C1.
K ´ Î0 A
2 ´ 10 -12 = .......... (2) é q ù
2d Þ C1 > C2 êQ V = and q is same in series ú
ë C û
(2) K 68. (c) The ball on touching plate A will
Þ = 2 or K = 4
(1) 2 get positively charged. It will be A B
60. (d) The charge Q given to outer sphere distributes as repelled by A and get attracted
+ –
Q1 outside and Q2 inside which induces charge – Q2 towards B. After touching B it + + +
+ + –
on outside of inner sphere, + Q2 on inside of inner will get negatively charged. It + + +

sphere which is earthed. will now be repelled by B and + –
The inside of outer and the inner sphere constitute a get attracted towards A. + –
Thus it will remain oscillating + –
ab
spherical condenser having capacitance 4p Î0 and at the extreme position touch the plates.
b-a
Free eBooks on @neetquestionpaper2020

Electrostatic Potential & Capacitance 621


69. (a) Initial charge on first capacitor is CV = Q1. 73. (b) Potential difference across the branch de is 6 V. Net
Initial charge on second capacitor is 2CV = Q2. capacitance of de branch is 2.1 µF
Final capacitance of first capacitor is KC So, q = CV
If V' is the common potential then Þ q = 2.1 × 6 µC
Þ q = 12.6 µ C
Q1 + Q 2 CV + 2CV 3V Potential across 3 µF capacitance is
V¢ = V¢ = =
C1¢ + C 2 Þ KC + 2C 2+ K 12.6
70. (a) Since battery is still in connection, so, V= = 4.2 volt
3
V = V0 Potential across 2 and 5 combination in parallel is
Þ Q0 = C0V0 and Q = kC0V0 6 – 4.2 = 1.8 V
Þ Q = kQ0 So, q' = (1.8) (5) = 9 µC
Since, k > 1 74. (a) Whenever a charge (+50 nC) is kept inside a hollow
Þ Q > Q0 metallic spherical shell, it induces an equal and
opposite charge on the inner surface and an equal
1 and same type of charges on the outer surface.
Also, U 0 = Q 0 V0 and
2 \ Inside, induced charge is – 50 nC and outside, +50
1 nC – 150 nC already present.
U= QV = kU0 {\ Q = kQ0 and V = V0 } 75. (a) Required ratio
2
Hence, U > U0 1
Energy stored in capacitor CV 2
= = 2 ,
C C Workdone by the battery Ce 2
where C = Capacitance of capacitor
71. (b) A C C B V = Potential difference,
e = emf of battery
C C
1 2
Ce
C C = 2 . (Q V = e)
Ce 2
The figure shows two independent balanced
1
wheatstone Brides connected in parallel each having =
a capacitance C. So, 2
Cnet = CAB = 2C é (-3q) q (q) ´ (q) ù 2
72. (b) Ket C0 be the capacitance initially and C be the 76. (b) Ui = k ê ´ 3+ ´ 3ú = -6kq
ë L L û L
e0A Uf = 0
capacitance finally. The C 0 =
d Work done by electric field = – Change in potential energy
e 0 AV -6kq 2
Since, Q = C0 V Þ Q = = Ui – Uf =
d L
V E V 77. (c)
Further, E 0 = and E = 0 Þ E = r
d K Kd
–q +q
1 2 O
Also, if Ui is the initial energy, then Ui = C 0 V
2
After the introduction of slab if Uf be the final energy,
then mv2 kq 2 kq 2
= ; mv 2
=
1 1 æ Vö
2 r (2r)2 4r
2
U f = CVslab = ( KC0 ) ç ÷ Kinetic energy of each particle
2 2 è Kø
1 C0 V 2 1 kq2
Þ Uf = = mv2 =
2 K 2 8r
Þ DU = U 2 - U1 1 Q 1 æ -2Q ö
78. (a) V = V1 + V2 + V3 = . + ç ÷
1 æ1 ö 4p Î0 R 4p Î0 è R ø
Þ DU = C0 V 2 ç - 1÷
2 èK ø 1 æ 3Q ö 1 æ 2Q ö
+ çè ÷ø = ç ÷
Since, work done = Decrease in Potential Energy 4p Î0 R 4p Î0 è R ø
Þ W = -DU Î0 A
2 79. (d) C =
1 e 0 AV æ 1ö d
ÞW= ç1 - ÷ A ® common area, Here A = A1
2 d è Kø
Free eBooks on @neetquestionpaper2020

622 Physi cs
80. (b) As the capacitors offer infinite resistance to steady t t
V – 1
current so, the equivalent circuit is 84. (a) VR = 0 = V0e RC Þ = e10
V R
4 4
A B t
t
Þ4 =e 10 Þ log e 4 = Þ t = 10log 4 = 13.86 s
V VC 10
2R A B (RC = 2.5 × 106 × 4 × 10–6 = 10)
85. (d) Charge distribution on each surface makes both
2V I capacitor of same potential difference hence charge
Using ohm’s law, current in circuit is will not flow.
V 86. (d) The charge on each of two facing surfaces will be
2V – V = I (2R + R) Þ I =
3R zero.
The voltage drop across 87. (d) Potential and potential energy are different quantities
V 4 and cannot be equated.
VAB = 2V - ´ 2R = V
3R 3 88. (d) Two equipotential surfaces are not necessarily parallel
4 to each other.
VAB = V = V + VC 89. (a) For a non-uniformly charged thin circular ring with net
3
zero charge, electric potential at each point on its axis is
1
Þ Voltage drop across C = V.. zero. Hence electric field at each point on its axis must
3 be perpendicular to the axis. Therefore statement-1 is
81. (a) The electrostatic pressure at a point on the surface of a
false and statement-2 is true.
s2 90. (b) Statement 1 is true. Statement 2 is true and is the correct
uniformly charged sphere =
2 Î0 explanation of statement 1.
s2 91. (b) 92. (a) 93. (c)
\ The force on a hemispherical shell = ´ pR 2 94. (c) 95. (a)
2 Î0
82. (c) Even after introduction of dielectric slab, direction of 96. (b) The inner sphere is grounded, hence its potential is
electric field will be perpendicular to the plates and zero. The net charge on isolated outer sphere is zero.
directed from positive plate to negative plate. Let the charge on inner sphere be q'.
s \ Potential at centre of inner sphere is
Further, magnitude of electric field in air =
e0 1 q¢ 1 q
= 4pe a + 0 + 4pe 4a = 0
s 0 0
Magnitude of electric field in dielectric =
K e0 q
\ q¢ = -
Similarly electric lines always flows from higher to lower 4
potential, therefore, electric potential inceases 97. (c) The region in between conducting sphere and shell is
continuously as we move from x = 0 to x = 3d. shielded from charges on and outside the outer surface
of shell. Hence charge distribution on surface of sphere
(–) (+) and inner surface of shell is uniform.
The distribution of induced charge on outer surface of
shell depends only on point charge q, hence is non-
uniform.
98. (a) The charge distribution on all surfaces, is as shown.
x =0 x = d x=2d x =3d
+q/4 (uniform)
83. (c) The given circuit can be redrawn as follows
5mF 15mF A b c
A C x q
B
– q/4 (uniform)
– q/4 (non-uniform)
1 q
2000V The electric field at B is = 4pe 2 towards left
0 4x
æ 15 ö
(VA – VB) = ç ÷ ´ 2000 Þ V A – VB = 1500V a
1 q 1 q
è 5 + 15 ø \ VC = - ò dx =
Þ 2000 – VB = 1500V Þ VB = 500V 4pe0 4x 2 32 pe0 a
2a
Free eBooks on @neetquestionpaper2020

18
Current Electricity

ELECTRIC CURRENT If area is not normal to current, then area normal to current is A' =
It is the rate of flow of charge through any cross section. A cos q (see the figure)
dq A'
i.e. I = A
dt
Conventionally, the direction of flow of positive charge is taken
as the direction of electric current. It is a scalar quantity and its q q
J
S.I. unit is ampere (A).
Keep in Memory I r r r rr
J= or I = J A cos q or I = J . A = ò J . A
(i) Current carriers in conductor are electrons, (valence e–s) A cos q
ions in electrolytes, electrons & holes in semiconductor Its SI unit is Am–2
and positive ions /electrons in gases. Current density can also be related to electric field as
(ii) Charge of electron = 1.6 × 10–19c r
r r E
(iii) 1 ampere = 6.25 × 1018 electrons/sec j = sE =
r
(iv) Though direction is associated with current (opposite to
the motion of electrons), but it is not a vector quantity as it where s is conductivity of the substance & r is specific resistance
does not follow rules of vector addition. of the substance.
r
(v) For a current to flow through a cross-section, there must be J is a vector quantity and its direction is same as that of E .
a net flow of charge through that cross-section. Dimensions of J are [M°L–2T°A]
In a metal like copper there are around 1028 free electrons
Keep in Memory
per m3 moving randomly in all direction with speeds of the
order of 106 m/s even in the absence of electric field. But (i) Electric current is a macroscopic physical quantity where
since the number of electrons passing through a cross- as current density is a microscopic physical quantity.
section from left to right is equal to the number of electrons (ii) For a given conductor current does not change with change
passing from right to left in a given time, therefore the net in cross-sectional area.
charge flow is zero and hence the electric current is zero. uuur
(vi) A conductor remains uncharged when current flows in it. DRIFT VELOCITY Vd ( )
i.e. Net charge in a current carrying conductor is zero.
When the ends of a conductor are connected to the two terminals
CURRENT DENSITY of a battery, an electric field is set up in the conductor from the
positive terminal to the negative terminal. The free electrons in
Current density at a point inside a conductor is defined as the the conductor experiences a force opposite to the direction of
amount of current flowing per unit cross sectional area around the electric field and hence get accelerated. However this process
that point of the conductor, provided the area is held in a of acceleration is soon interrupted by collision with ions of solid.
direction normal to the direction of current. The average time for which each electron is accelerated before
I suffering a collision is called the mean free time or mean
i.e. Current density, J = relaxation time.
A
Free eBooks on @neetquestionpaper2020

624 Physi cs
Thus, the free electrons within the metal, in addition to its random where N0 = Avogadro number
motion acquire a small velocity towards the positive end of d = density of the metal
conductor. This velocity is called drift velocity. It is given by M = molecular weight
ur
r eEt and x = number of free electrons per atom
vd = – ,
m 3. For steady current :
where e is the charge and m is the mass of electron.
ur I I I
E is the electric field established in conductor and t is the J= ; E= ; Vd =
A sA neA
average relaxation time.
r r This means that for a given material and steady current in
Negative sign is because the directions of E and vd (for electron) case of non-uniform cross-section of material
are opposite.
V 1 1 1
E= Jµ ; E µ ; Vd µ
l A A A
l 4. Variation of drift velocity :
+ E –
E 1 V
Vd e– I Vd = = ; Vd µ E
ne ne l
J
when length is doubled, vd becomes half and
when V is doubled, vd becomes twice.

+V – Example 1.
The current in a wire varies with time according to the
where V is the potential difference across ends of the conductor relation i = 4 + 2t2
of length l. The uniform current I, flowing through the conductor How many coulomb of charge pass a cross-section of wire
is given by
in time interval t = 5s to t = 10s?
I = n e A vd
Solution :
where n = number of free electrons per unit volume,
A = area of cross-section, vd = drift velocity dq
ur r i= Þ dq = i dt Þ dq = (4 + 2 t2 ) dt
In vector form, Þ J = - nev d dt
The negative sign is because the direction of drift velocity of 10
r 10
electron is opposite to J . é 3ù
t
On integrating q = ò (14 + 2t 2 )dt = ê14t + 2 ú
Mobility - Drift velocity per unit electric field is called mobility. êë 3 úû
It is denoted by µ. 5 5
r = 603.33 C
Vd Example 2.
m=
E r æ 4 A ö ˆ
Its S.I. unit is m2/volt-sec. The current density at a point is J = çè 2 ´ 10 2 ÷ø j . Find
m
Keep in Memory r
the rate of charge flow through a cross-sectional area S
1. Drift velocity is very small, it is of the order of 10–4m/s such that
which is negligible as compared to thermal speed of e–s at r
(i) S = (2cm 2 )ˆj ,
room temperature (;105 m/s)
r r
2. The drift velocity is given by (ii) S = (4cm 2 ) ˆi and (iii) S = (2iˆ + 3jˆ cm 2 )
J Solution :
vd = ,
ne The rate of charge flow = current
I r r
where, J = current density = i = JdS
A
e = electronic charge = 1.6 × 10–19 C (i) Current i = (2 ´ 104 A / m 2 ) ˆj.(2 ´ 10 -4 m 2 )ˆj = 2A
n = the number of free electrons per unit volume
3. The number of free electrons per unit volume (n) can be [using ˆj.jˆ = 1]
determined by the following relation :
(ii) Current i = (2 ´ 10 4 A / m 2 ) ˆj.(4 cm 2 ) iˆ = 0
– M
No. of free e per atom
n= and volume= . [using ˆj.iˆ = 0]
Volume N0 d
N d ˆ ´ 10 -4 m 2 = 6A
(iii) Current i = (2 ´ 104 A / m2 ) ˆj.(2iˆ + 3j)
n = 0 ´ x,
M
Free eBooks on @neetquestionpaper2020

Current Electricity 625


Example 3. density rm, Avogadro’s number NA, and the molar mass M.
A typical wire for laboratory experiments is made of copper For copper r = 8.93 g/cm3 and M = 63.5 g/mol.
and has a radius 0.815 mm. Calculate the drift velocity of
electrons in such a wire carrying a current of 1A, assuming rm N A (8.93 g / cm3 ) (6.02 ´ 1023 atoms / mol)
na = =
one free electron per atom. M 63.5 g / mol
Solution :
DQ n a = 8.47 ´ 10 22 atoms / cm3 = 8.47 ´ 1028 atoms / m3
Equation I = = qnAvd relates the drift velocity to the The magnitude of the charge is e, and the area is A
Dt
number density of charge carriers, which equals the number \q=e
density of copper atoms na. We can find n a from the mass Area is related to the radius r of the wire as A = pr2
density of copper, its molecular mass, and Avogadro’s Substituting numerical values yields
number. I I
The drift velocity is related to the current and number vd = =
nqA n a epr 2
r I æ J ö
density of charge carriers : v d = çv = ÷ 1C/s
nqA è nq ø Þ vd =
If there is one free electron per atom, the number density of (8.47 ´ 10 m ) (1.6 ´ 10 -19 C) p (0.000815m) 2
28 -3

free electrons equals the number density of atoms


n a : n = n a. = 3.54 ´ 10 -5 m / s
The number density of atoms na is related to the mass

18.1
Solve following problems with the help of above text and 7. The electric field intensity E, current density J and specific
examples. resistance k are related to each other through the relation
1. The current which is assumed to be flowing in a circuit (a) E = J/k (b) E = J k
from positive terminal to negative, is called
(a) direct current (c) E = k/J (d) k = J E
(b) pulsating current 8. For which of the following dependence of drift velocity vd
(c) conventional current on electric field E, is Ohm’s law obeyed?
(d) alternating current
2. Relation between drift velocity (vd) of electron and thermal (a) vd µ E 2 (b) vd = E1/2
velocity (vt) of electron at room temp is expressed as
(c) vd = constant (d) vd = E
(a) vd = vt (b) vd > vt
(c) vd < vt (d) vd = vt = 0 9. If N, e, t and m are representing electron density, charge,
3. The speed at which the current travels, in conductor, is relaxation time and mass of an electron respectively, then
nearly equal to theresistance of wireof length l and cross-sectional area
(a) 3 × 104 m/s (b) 3 × 105 m/s A is given by
6
(c) 4 × 10 m/s (d) 3 × 108 m/s
4. In the absence of an electric field, the mean velocity of ml 2mtA
free electrons in a conductor at absolute temperature (T) (a) (b)
2 2
Ne A t Ne 2 l
is
(a) zero (b) independent of T Ne 2 t A Ne 2 A
(c) proportional to T (d) proportional to T 2 (c) (d)
5. When a potential difference V is applied across a 2ml 2m tl
conductor at a temperature T, the drift velocity of electrons 10. The current density (number of free electrons per m 3) in
is proportional to metallic conductor is of the order of
(a) V (b) V (c) T (d) T (a) 1022 (b) 1024 (c) 1026 (d) 1028
6. An electronic charge e is revolving in a circular orbit of 11. A straight conductor of uniform cross-section carries a
radius r around a nucleus with speed v. The equivalent current I. If s is the specific charge of an electron, the
current is momentum of all the free electrons per unit length of the
(a) ev/2πr (b) 2ev/πr conductor, due to their drift velocity only is
(c) ev/4πr (d) None of these (a) I s (b) I/s (c) I/s (d) (I / s)2
Free eBooks on @neetquestionpaper2020

626 Physi cs

12. Electrons are emitted by a hot filament and are accelerated 13. When no current is passed through a conductor,
by an electric field as shown in figure. The two stops at (a) the free electrons do not move
the left ensure that the electron beam has a uniform cross- (b) the average speed of a free electron over a large
section. period of time is zero
E (c) the average velocity of a free electron over a large
period of time is zero
(d) the average of the velocities of all the free electrons
A B at an instant is zero
14. A current passes through a resistor. If K1 and K2 represent
which of the following statement(s) is/are correct? the average kinetic energy of the conduction electrons
(a) The speed of the electron is more at B than at A. and the metal ions respectively then
(a) K1 < K2
(b) The electric current is from left to right.
(b) K1 = K2
(c) The magnitude of the current is larger at B than at A. (c) K1 > K2
(d) The current density is more at B than at A. (d) any of these three may occur

ANSWER KEY
1. (c) 2. (c) 3. (d) 4. (a) 5. (b) 6. (a) 7. (b) 8. (d) 9. (a) 10. (d) 11. (c) 12. (a)
13. (c,d) 14. (c)

OHM’S LAW AND ELECTRICAL RESISTANCE Electrical Resistance


When a potential difference is applied across the ends of a On application of potential difference across the ends of a
conductor, a current I is set up in the conductor. conductor, the free e–s of the conductor starts drifting towards
According to Ohm’s law “Keeping the given physical conditions the positive end of the conductor. While drifting they make
such as temperature, mechanical strain etc. constant, the current collisions with the ions/atoms of the conductor & hence their
(I) produced in the conductor is directly proportional to the motion is obstructed. The net hindrance offered by a conductor
potential difference (V) applied across the conductor”. to the flow of free e–s or simply current is called electrical
i.e., I µ V or I = KV ... (1) resistance.
where K is a constant of proportionality called the conductance It depends upon the size, geometry, temperature and nature of
of the given conductor. the conductor.
Resistivity : For a given conductor of uniform cross-section A
Alternatively, V µ I or V = RI ... (2)
and length l, the electrical resistance R is directly proportional to
where the constant R is called the electrical resistance or simply length l and inversely proportional to cross-sectional area A
resistance of the given conductor.
From above two eqs. it is clear that R = 1/K. l rl RA
i.e., R µ or R = or ρ =
If a substance follows Ohm’s law, then a linear relationship exists A A l
between V & I as shown by figure 1. These substance are called r is called specific resistance or electrical resistivity.
Ohmic substance. Some substances do not follow Ohm’s law,
m
these are called non-ohmic substance (shown by figure 2) Also, r =
Diode valve, triode valve and electrolytes, thermistors are some ne 2t
examples of non-ohmic conductors. The SI unit of resistivity is ohm - m.
V V 1
Conductivity(s) : It is the reciprocal of resistivity i.e. s = .
r
The SI unit of conductivity is Ohm–1m–1 or mho/m.
I
Ohm’s law may also be expressed as, J = sE
q where J = current density and E = electric field strength
I
Ohmic conductor Non-linear conductor ne 2 t
Conductivity, s = where n is free electron density, t is
or linear conductor or non-ohmic conductor m
Fig. 1 Fig. 2 relaxation time and m is mass of electron.
Slope of V-I Curve of a conductor provides the resistance of the (i) The value of r is very low for conductor, very high for
conductor insulators & alloys, and in between those of conductors &
insulators for semiconductors.
V
slope = tan q = (ii) Resistance is the property of object while resistivity is the
I
property of material.
The SI unit of resistance R is volt/ampere = ohm (W)
Free eBooks on @neetquestionpaper2020

Current Electricity 627


Materials and their resistivity where a = temperature coefficient of resistivity and
Material Resistivity (r) h (at 0°c) where DT = t2 – t1 = change in temperature
(in W m) The value of a is positive for all metallic conductors.
\ r2 > r1
(i) Silver 1.6 × 10–8
In other words, with rise in temperature, the positive ions of the
(ii) Copper 1.7 × 10–8 metal vibrate with higher amplitude and these obstruct the path
(iii) Aluminium 2.8 × 10–8 of electrons more frequently. Due to this the mean path decreases
(iv) Tungsten 5.2 × 10–8 and the relaxation time also decreases. This leads to increase in
(v) Platinum 10.6 × 10–8 resistivity.
(vi) Manganin 42 × 10–8 1 -1
Please note that the value of a for most of the metals is K
(vii) Carbon 35 × 10–6 273
For alloys : In case of alloys, the rate at which the resistance
(viii) Germanium .46
changes with temperature is less as compared to pure metals.
(ix) Silicon 2300 For example, an alloy manganin has a resistance which is
(x) Glass ~ 1013 30-40 times that of copper for the same dimensions.
(xi) Mica ~ 2 × 1015 Also the value of a for manganin is very small » 0.00001°C–1.
COMMON DEFAULT Due to the above properties manganin is used in preparing wires
for standard resistance (heaters), resistance boxes etc.
l Please note that eureka and constantan are other alloys for which
Since R = r Þ Rµl
A r is high. These are used to detect small temperature, protect
It is incorrect to think that if the length of a resistor is doubled its picture tube/ windings of generators, transformers etc.
resistance will become twice. For semiconductors : The resistivity of semi-conductors
If you look by an eye of physicist you will find that when l decreases with rise in temperature. For semi conductor the value
change, A will also change. This is discussed in the following of a is negative.
article.
Case of Reshaping a Resistor m
r= 2
On reshaping, volume of a material is constant. ne t
i.e., Initial volume = final volume With rise in temperature, the value of n increases. Please note
or, Ai li = Af lf ... (i) that t decreases with rise in temperature. But the value of increase
where li, Ai are initial length and area of cross-section of resistor in n is dominating for the value of r in this case.
and lf, Af are final length and area of cross-section of resistor. For electrolytes : The resistivity decreases with rise in
If initial resistance before reshaping is Ri and final resistance temperature. This is because the viscosity of electrolyte decreases
after reshaping is Rf then with increase in temperature so that ions get more freedom to
move.
l
r i For insulators : The resistivity increases nearly exponentially
Ri Ai l A with decrease in temperature. Conductivity of insulators is almost
= = i ´ f ... (ii)
Rf lf l f Ai zero at 0 K.
r Superconductors : There are certain materials for which the
Af
2 resistance becomes zero below a certain temperature. This
R i æ li ö temperature is called the critical temperature. Below critical
From eqs. (i) and (ii), =ç ÷ Þ R µ l2
R f çè lf ÷ø temperature the material offers no resistance to the flow of e–s.
The material in this case is called a superconductor. The reason
This means that resistance is proportional to the square of the
for super conductivity is that the electrons in superconductors
length during reshaping of a resistor wire.
2
are not mutually independent but are mutually coherent. This
R i æ Af ö 1 coherent cloud of e–s makes no collision with the ions of super-
Also from eqs. (i) and (ii), = çç ÷÷ Þ R µ
conductor and hence no resistance is offered to the flow of e–s
R f è Ai ø A2
For example, R = 0 for Hg at 4.2 K and R = 0 for Pb at 7.2 K. These
This means that resistance is inversely proportional to the square
substances are called superconductors at that critical temperature.
of the area of cross-section during reshaping of resistor.
Super conductors ar e used (a) in making very strong
Since A = p r2 (for circular cross-section)
electromagnets, (b) to produce very high speed computers
1
\R µ 4 (c) in transmission of electric power (d) in the study of high
r energy particle physics and material science.
where r is radius of cross section. SERIES AND PARALLEL COMBINATION OF RESISTORS
Effect of Temperature on Resistance and Resistivity Resistances in Series
Resistance of a conductor is given by Rt = R0 (1 + aDt) When a number of resistances are R1 R2
Where a = temperature coefficient of resistance and Dt = change joined end to end so that same current
in temperature flows through each, resistor when some
For metallic conductors : If r1 and r2 be resistivity of a conductor potential difference is applied across I V1 V2
at temperature t1 and t2, then the combination, the conductor are V
r2 = r1 (1 + a D T) said to be connected in series.
Free eBooks on @neetquestionpaper2020

628 Physi cs
The equivalent resistance in series is given by The equivalent resistance is given by
(Req)s = R1 + R2 + ...+ Rn 1 1 1 1
Equivalent resistance of same R1 R2 = + + ... +
(R eq )p R1 R 2 Rn
resistances connected in series
V1 V2 The equivalent resistance in a parallel combination is always less
is always greater than the
than the value of the least individual resistance in the circuits.
greatest of individual resistance. Current division rule in parallel combination
Potential division rule in series IR 2 IR1
combination : I1 = ; I2 =
V R1 + R 2 R1 + R 2
VR1 VR 2 In a given combination of resistors, when you want to detect
V1 = ; V2 =
R1 + R 2 R1 + R 2 whether the resistances are in series or in parallel then see that if
Resistances in Parallel the same current flows through two resistors then these are in
series and if same potential difference is there across two resistors
Two or more resistors are said to be connected in parallel if the then these are in parallel potential diff across each resistor is the
same potential difference exits across all resistors. same & is equal to the applied potential difference.
I1 R1
I HOW TO FIND EQUIVALENT RESISTANCE ?
R2 Successive Reduction
I2
This method is applicable only when the resistors can be clearly
identified as in series or parallel. Let us take some example to find
resistance between ends A and B
V

3W 3W 33 W 3W

6W 3W
Ex. (i) 3W = 6W 6W 3W = 6W
6W
3W 33 W 3W
6W 66 W 6W

A 3W B AA 33 W B
B 3W B A 3W B
6W
3W 3W 2W
= =
A 3W B A 3W B
7
10 10
5
Ex. (ii) 3 5 5
5 5
10 = 10 = A B
= =
A B A B A B
10 A B 10
10 10

Ex. (iii) =
= =

Ex. (iv) = = =

Please note that all points on the circumference are at same potential as there is no resistance on circumference.
Free eBooks on @neetquestionpaper2020

Current Electricity 629


Ex. (vi) Infinite series : Ex.(ii) The circuit shown is symmetrical about axis XY. Therefore
r VB = VH ; VC = VI = VG; VD = VF
r
B C
to ¥ r r
r r A r
B r D
r r
r r
We observe that there is a repetitive unit extending to infinity EY
X A r r I r r
on left hand side. We assume that the equivalent resistance
of all the unit except one (shown dotted) is equal to X ohm. r F
The equivalent circuit will be as shown below. H r

B G
r
Therefore the circuit can be redrawn as
X r
r r
r
r r r
r A B C D
X H I
The equivalent resistance across A and B is r r G r F r Y
X´r r r
R AB = r + +r
X+r
Please note that RAB can be taken as X because if you add
one unit to the sum of infinite units, then it will be Ex.(iii) The circuit is asymmetric about the dotted line
approximately the same. \ IBG = IGC; IFG = IGE and IAG = IGB
X´r
\ X = 2r +
X+r B C
Solve the equation as a normal algebraic equation to find r
A r r r
r
X. D
Using Symmetry of the Circuit X r r r r Y
r G
Axis symmetry : E
F r
Ex.(i) The circuit shown in figure is symmetrical about XAEBY
axis. This is because the upper part of the axis is the mirror Therefore the equivalent circuit is
image of lower part (resistors and current direction both) B r C
C r
A r r r
r D
r r r
r X r r r Y
r r
X A E B Y
r F E
r
r
2. Shifted symmetry :
D
\ IAC = IAD;ICB = IDB ; B
IAE = IEB ( Q VC < VE < VD wheatstone R1 R2
2.5 W
bridge principle) I1 I2
I3 5 W 5W
Þ ICE = IED = 0
X I A E C Y
Therefore the circuit can be redrawn. It is now easier to find 2.5 W I1
resistance between X and Y. I2
R2 R1
C
r D
r
r r The diagram given above is symmetrical but the positions
of the resistances are shifted. Let I be the current in the
X A E B Y
circuit from A. The same leaves the circuit at C. Let current
r r in AB, AD and AE be I1 , I2 and I3 respectively. Since the
same current flows in AE and EC, the detached equivalent
D circuit can be drawn as
Free eBooks on @neetquestionpaper2020

630 Physi cs

B 4. Star -delta connection :


B
10 W
I1
5W
r1 B
2.5 W

I2 E D Y
Rb
X I A r2 =
2.5 W I1 A A Ra
5W 10 W
I3
r3 Rc
C
C
C
5W 5W r1r3 r1r2 r2 r3
Ra = ; Rb = ; Rc = + + ;
r1 + r2 + r3 r1 + r2 + r3 r1 r2 r3
Keep in Memory
B
B r
Equivalent resistance between A and B of the resistors connected Rb 1

as shown in the figure Ra


= A r2
A
R1 R2 Rc r3
C
C
A B R R + R b R c + R cR a
r1 = a b ;
R1 Rc
R2
R a R b + R b R c + R cR a
r2 = ;
Wheatstone bridge Ra

R 1 (R1 + 3R 2 ) R a R b + R b R c + R cR a
R AB = r3 = ;
R 2 + 3R 1 Rb
Using delta to star conversion
3. Path symmetry :
3W 2W 2W
All paths from one point to another which have the same
=
setting of resistances have the same amount of currents. 2W 2W 9W 9W
Example : 3W 3W 9W
B C 2W 2W
I1 I1 I X Y X Y
I1
If none of the above method works then we may use Kirchhoff’s
I A I1 D
method which will be discussed later
I – 2I1
COMMON DEFAULTS
I – 2I1
F
1. Resistors are not just in series or in parallel if they look so
G geometrically, e.g. the resistors in the diagram are not in
E H parallel but in series.
A
Twelve wires each having resistance r are joined to form a cube.
We have to find the equivalent resistance across A and B.
By path symmetry, IAB = IBC = IAD = IDC = I
\ IAE = I– 2I1 Þ IGC = I– 2I1,
Since current in AB = current in BC B
Þ IBF = 0 These resistors across A and B are in series, as same current
Also IAD = IDC Þ IDH = 0 passes through them.
The equivalent circuit will be as shown. The resistance now clearly 2. This is a common thinking that current which comes out
visible as in series and in parallel. from the positive terminal of a battery is used up till it reaches
the negative terminal. But infact the current remains the
r
I same in a branch. In fact a potential drop takes place across
r
r a resistor.
I r
r B 5W C
r
r
r 1 amp 1 amp
r
r
A +5V 0V D
Free eBooks on @neetquestionpaper2020

Current Electricity 631


IA = IB = IC = ID = 1 amp Grey 8 108
VA = VB = +5V White 9 109
VC = VD = 0V Gold – 0.1 ± 5%
This means that a potential drop of 5V takes place across Silver – 0.01 ± 10%
the resistor
No colour – – ± 20%
3. O Incorrect : If two resistances are not in series then it
is in parallel and vice-versa. To learn the above table of colour codes of resistors let us
P Correct : The above thinking is incorrect. We may learn this interesting sentence :
have resistances which are neither in series nor in BB ROY of Great Britain has a Very Good Wife.
parallel. In the above sentence the capital letters have the following
meaning :
B Þ Black B Þ Brown
Colour Coding for Carbon Resistor and their Standard
Values – R Þ Red O Þ Orange
(i) It is a system of colour coding used to indicate the values Y Þ Yellow G Þ Green
of resistors. V Þ Violet G Þ Grey
(ii) For the fixed, moulded composition resistor, four colour W Þ White
bands are printed on one end of the outer casing as shown Remember the colour in the above order and the
below. corresponding digits from 0 to 9 and also the multiplier with
the power to 10 from 0 to 9.
4
3 Commercial resistors are of two types
2
1 (a) Wire round resistor made by winding of wires of an
alloy manganins, constantan and nichrome.
(b) Carbon resistors have low cost and are compact.
Tolerence THERMISTOR
A thermistor is a heat sensitive resistor usually made up of
Multiplier semiconductor. The oxides of various metals such as mickel,
Significant iron, copper etc. temperature coefficient of thermistor is –ve but
digits is usually large, of the order of 0.04/ºC.
The V–I curve of thermistor is as shown.
(iii) The colour bands are always read left to right from the end
that has the bands closest to it.
(iv) (a) The first and second colour bands, represent the first
and second significant digits respectively, of the I
resistance value.
(b) The third colour band is for the number of zeros that
follow the second digit.
(c) In case the third band is gold or silver, it represents a
multiplying factor of 0.1 or 0.01. V
(d) The fourth band represents the manufacture’s
tolerance. It is a measure of the precision with which Thermistors are used for resistance thermometer in very low
the resistor was made. temperature measurement of the order of 10K and to safeguard
(e) If the fourth band is not present, the tolerance is electronic circuits against current jumps because initially
assumed to be ± 20%. thermistors has high resistance when cold and its resistance
drops appreciably when it heats up.
(v) Standard value of colour codes for carbon resistors
JOULE’S LAW OF HEATING
Colour Digit Multiplier Tolerance It states that the amount of heat produced in a conductor is directly
Black 0 100 = 1 proportional to the
Brown 1 101 = 10 (i) square of the current flowing through the conductor,
Red 2 102 (q, T – constt) i.e. H µ i2
(ii) resistance of the conductor (i, T – constt.)
Orange 3 103 i.e. HµR
Yellow 4 104 (iii) time for which the current is passed (i, R, – constt)
Green 5 105 i.e., Hµt
Blue 6 106 i 2 RT
Violet 7 107 Thus H = i2 RT joule = cal
4.2
Free eBooks on @neetquestionpaper2020

632 Physi cs
ELECTRIC POWER This shows that power consumed by n equal resistances in
It is defined as the rate at which work is done in maintaining parallel is n2 times that of power consumed in series if
the current in electric circuit. voltage remains same.
Electric power, P = VI = I2 R = V2 / R watt or joule/second. 4. In parallel grouping of bulbs across a given source of voltage,
Electric energy : The electric energy consumed in a circuit is the bulb of greater wattage will give more brightness and will
defined as the total work done in maintaining the current in an allow more current through it, but will have lesser resistance
electric circuit for a given time. and same potential difference across it.
Electric energy = VIt = Pt = I2 Rt = V2 t / R
The S.I. unit of electric energy is joule (denoted by J) 5. For a given voltage V, if resistance is changed from R to
where 1 joule = 1 watt × 1 second = 1 volt × 1 ampere × 1 sec. R/n, power consumed changes from P to nP.
In household circuits the electrical appliances are connected in P = V2 / R; when R´ = R/n,
parallel and the electrical energy consumed is measured in kWh then P´ = V2 / (R/n) = n V2 / R = nP.
(kilo watt hour). 6. Filament of lower wattage bulb is thinner than that of higher
1 kWh (1 B.O.T. unit) = 1000 Wh = 3.6 × 106 J wattage bulb i.e. filament of 60 watt bulb is thinner than
Keep in Memory that of 100 watt bulb.
7. If I is the current through the fuse wire of length l, radius r,
For Series Combination : specific resistance r and Q is the rate of loss of heat per
1. If resistances (or electrical appliances) are connected in unit area of a fuse wire, then at steady state,
series, the current through each resistance is same. Then
power of an electrical appliance I 2rl
I2R = QA or = Q ´ 2prl
pr 2
P µ R and V µ R (Q P = i Rt )
2

It means in series combination of resistances, the potential 2p 2 Q 3


or I2 = r Þ I a r 3/ 2
difference and power consumed will be more in larger r
resistance. Hence current capacity of a fuse is independent of its length
2. (i) When the appliances of power P 1, P2, P3 ... are and varies with its radius as r3/2.
connected in series, the effective power consumed 8. If t1 and t2 are the time taken by two different coils for
(P) is producing same heat with same supply, then
1 1 1 1 1 (i) If they are connected in series to produce same heat,
= + ...++ i.e., the effective power is time taken, t = t1 + t2
P P1 P2 P3 Pn
(ii) If they are connected in parallel to produce same heat,
less than the power of individual appliance. t1 t 2
(ii) If n appliances, each of equal resistance R, are time taken is, t =
t1 + t 2
connected in series with a voltage source V, the power
dissipated Ps will be 9. When a bulb glows the temperature of the filament is of the
order of 3000K.
V2 10. To avoid lengthy calculations use P = I2R in series
Ps = ... (i)
nR
3. When two lamps of different wattage are connected in series V2
combination and P = in parallel combination and
in a house the lamp of lower wattage glows more brightly. R
For Parallel Combination : P = VI when we want to find power of a device and V and I
1. If resistances (i.e. electrical appliances) are connected in are known.
parallel, the potential difference across each resistance is 11. The resistance of an appliance, can be formed by rated
same. Then P µ 1 / R and I µ 1 / R . 2
It means in parallel combination of resistances the current Vrated
voltage and power is R =
and power consumed will be more in smaller resistance. Prated
2. When the appliances of power P1, P2, P3.... are in parallel, Now this resistance does not change (remember resistance
the effective power consumed (P) is depends only on the parameters of resistors and not on the
P = P1 + P2 + P3 ... +Pn
voltage across it or the current flowing through it, if we
i.e. the effective power of various electrical appliances is
neglect the changes occurring due to change in
more than the power of individual appliance.
temperature).
3. If n appliances, each of resistance R, are connected in parallel
with a voltage source V, the power dissipated Pp will be 12. The maximum current that can be safely passed through an
Prated
V2 nV 2 appliance (a resistor) is I = .
PP = = ... (ii) Vrated
(R / n ) R
From eqns. (i) and (ii), 13. Bulbs get fused sometimes when switched on. This is
because with the rise in temperature the resistance increases
PP and power decreases (P = V2 / R). Therefore the bulb glows
= n 2 or PP = n2 PS
PS brighter in the beginning and get fused.
Free eBooks on @neetquestionpaper2020

Current Electricity 633

14. A lead-acid secondary cell is discharged if the relative Example 5.


density of electrolyte drop to 1.18 and e.m.f. of 1.8V. A cylindrical copper rod is reformed to twice its original
15. Hot wire galvanometer is based on heating effect of current. length with no change in volume. The resistance between
Its deflection q is directly proportional to the heating effect its ends before the change was R. Now its resistance will
be
(i.e. I2). It works on A.C. as well as D.C. (a) 8 R (b) 6 R
16. Two wires of same material and same length but having (c) 4 R (d) 2 R
different diameters connected in parallel produce more heat Solution : (c)
than when connected in series. i.e., Hparallel>Hseries.
r l1 ...(1)
17. If t1and t2 be the time taken by two heaters to boil a given R=
mass of a liquid, then the time taken to boil the same amount pr12
of the liquid, when both heaters are connected in parallel is Now the rod is reformed such that
given by l2 = 2l1 and pr12l 1 = pr22l 2 (no change in volume)
tt
tp < 1 2 . or (r12 / r22 ) = (l 2 / l 1 )
t1 ∗ t2
rl2
18. If t1and t2 be the time taken by two heaters to boil a given \ R2 = ...(2)
amount of a liquid, then the time taken to boil the same pr22
amount of the liquid, when both heaters are connected in From eqns. (1) and (2), we get
series is given by
R l r2 l l 1 1
ts = t1 + t2 . < 1 ≥ 22 < 1 ≥ 1 < ≥
19. If I be the current at which a fuse wire of radius R blows, R2 l 2 r1 l2 l2 2 2
\ R2 = 4 R
I2 Example 6.
then 3 constant. i.e., for two fuse wires of radii R1and R2
R A copper wire is stretched to make 0.1% longer. What is
the percentage change in its resistance?
and maximum bearing current I1, and I2, we have Solution :
I12 I22 rl
3
< . The resistance R of a wire is given by R = , where
R1 R23 A
r = specific resistance
20. If the two resistors R1and R2 are first connected in series Let d and m be the density and mass of the wire, respectively
and then parallel then the ratio of heat produced in the two then A l d = m or A = m/ld
cases (series to parallel) is given by
r l ´l d r l2 d æ r d ö 2
2 \ R= = =ç ÷´ l
Hs (R1 ∗ R2 ) m m èmø
<
Hp R1 R2 Taking log, of both sides, we get
21. If two resistances R1 and R2 are connected in parallel and a æ rd ö
loge R = log e ç ÷ + 2e loge l
current is passed in them such that heat produced in them è mø
is H1 and H2 respectively, then
dR 2 dl æ rd ö
H1 R Differentiating = çèQ = constant ÷
< 2. R l m ø
H2 R1
dR æ dl ö
´100 = 2ç ´100 ÷% = 2 ´ (0.1)
Example 4. R è l ø
The temperature coefficient of resistance of a wire is \ Percentage change in resistance = 2×(0.1) % = 0.2%
0.00125ºC–1.At 300 K its resistance is one ohm. At what So the resistance increases by 0.2%.
temperature the resistance of the wire will be 2 ohm?
Solution : Example 7.
From formula Rt = R0 (1 + µDt) Determine the equivalent resistance of the arrangement
of resistances shown in fig between the points A and B.
R 300 = R 0 (1 + a ´ 27) = 1 8W

R t = R 0 (1+ µ D ´ t) = 2 16W
16W 20W
1 + 27 a 1
\ = A B
1 + at 2
or 2 + 54a = 1 + at 9W
6W
Þ 2 + 54(0.00125) = 1 + (0.00125)t 18W
\ t = 854ºC = 1127 K
Free eBooks on @neetquestionpaper2020

634 Physi cs
Solution : \ R'' = R5 + R6 = 10 + 10 = 20 W
8W, 16W, and 16W resistances are connected in parallel. Further, R' and R'' are in parallel
Their equivalent resistance is given by R’
R ¢R ¢¢ 20 ´ 20
1 1 1 1 2 +1+ 1 4 \ R ¢¢¢ = = = 10W
= + + = = R ¢ + R ¢¢ 20 + 20
R ' 8 16 16 16 16
Resistance between A and D = R1 + R'''+ R4 =
\ R’ = 4W. 10 + 10 + 10 = 30W
This is in series with 20W. So upper part of AB has a resistance Example 10.
4 + 20 = 24 W ...(1) A and B are two points on a uniform ring of resistance R.
For lower part, 9W and 18W resistances are connected in The ÐAOB = θ , where O is the centre of the ring. Find
parallel. Their equivalent resistance R” is given by the equivalent resistance between A and B.
1 1 1 2 +1 3 Solution :
= + = = Þ R ¢¢ = 18 / 3 = 6W
R ¢¢ 9 18 18 18 Let r be the radius of the ring.
This is in series with 6W. So the resistance of lower part of B
AB is 6 + 6 = 12W ...(2)
The upper and lower parts of AB are in parallel. Hence the
equivalent resistance between A and B is given by C O q D
1 1 1 1+ 2 3
= + = = Þ R = 24/3 = 8W
R 24 12 24 24 A

Example 8. Resistance per unit length of ring r = R / 2 p r


Lengths of sections ADB and ACB are rq and r (2 p – q)
A wire has a resistance of 10W. It is stretched by one-tenth
of its original length. Then its resistance will be R Rq
Resistance of section ADB, R1 = r r q = rq =
(a) 10W (b) 12.1W 2pr 2p
(c) 9W (d) 11W Resistance of section ACB,
Solution : (b) R R (2 p - q)
Here volume remains constant. Thus R2 = r ( 2 p - q) =
2pr 2p
V1 = V2 Þ l1 ´ A1 = l 2 ´ A2 Here R1 and R2 are in parallel between A and B, their
equivalent resistance
102
(pr12 )
pr12 l = pr22 (11l / 10) or pr2 = R q R ( 2 p - q)
11 ´
{Q When wire is stretched by 1/10 of its original length, R1 R 2 2p 2p R q (2 p - q)
R eq. = = =
the new length of wire becomes (11l/10)} R1 + R 2 R q R ( 2 p - q ) 4 p2
+
l2 2p 2p
Let the new resistance be R2. Then R2 = r . Example 11.
A2
There are two concentric spheres of radius a and b
respectively. If the space between them is filled with
æ 11 ö
r.ç l ÷ medium of resistivity r, then find the resistance of the
10 ø (11 / 10)rl
R2 = è = intergap between the two spheres.
pr22 (10 / 11)pr12 Solution :
Consider a concentric spherical shell of radius x and
thickness dx as shown in fig. Its resistance, dR is
(11 / 10) é rl ù (11 / 10) 121
= (10 / 11) ê 2 ú = (10 / 11) ´10 = 10 = 12.1W θ dx
p
ëê 1 ûú
r dR <
4 ο x2
Example 9.
What will be the equivalent resistance between the two
dx
points A and D of fig?
10W 10W 10W
A C
R1 R2 x
ab
R6 10W R3 10 W
R5 R4
B D
10W 10W 10W
Solution :
Resistances R2 and R3 are in series. b
r dx r é 1 1 ù
4 p òa x 2 4 p êë a b úû
\ R' = R2 + R3 = 10 + 10 = 20 W \ Resistance of the intergap, R = = - .
Similarly, R5 and R6 are in series.
Free eBooks on @neetquestionpaper2020

Current Electricity 635


Example 12.
A wire 1 m long has a resistance of 1 W. If it is uniformly Example 15.
stretched, so that its length increases by 25%, then its A bulb has voltage rating of 220 V and power rating of
resistance will increase by 40 W. How can this bulb be made to glow with normal
(a) 25% (b) 50% brightness if a voltage source of e.m.f. 330 V is available?
(c) 56.25% (d) 77.33% Solution :
Here V = 220 V; P = 40 W
Solution : (c)
V 2 (220) 2
Resistance of the bulb, R = = = 1210 W
New length, l¢ = l + 25 l = 125 l ; P 40
100 100 330
Current in the circuit, I =
Let new area of cross-section = A ¢ . Then 1210 + S
Potential difference across the bulb,
A l d = A ¢ l¢ d or A¢ = A l / l¢ (Volume is constant)
æ 330 ö
IR = ç 1210
æ 125 ö 100 è 1210 + S ÷ø
or A¢ = A ´ l / ç ´ l÷ = A
è 100 ø 125 330 ´ 1210
As per question, 220 =
1210 + S
rl r l¢ On solving, we get S = 605 W
R= and R ¢ =
A A¢ Hence to glow bulb, a resistance of 605W should be
connected in series.
æ 125 ö Example 16.
rç ÷ l r l 125 2 If two bulbs of wattage 25 and 30 W, each rated at 220
è 100 ø æ ö volts are connected in series with a 440 volt supply, which
R'= = ç ÷ = 1.5625 R
æ 100 ö A è 100 ø bulb will fuse?
ç ÷ A Solution :
è 125 ø
(220) 2
% increase in resistance Resistance of 25 W bulb, R1 = W;
25
æ R¢ - R ö æ 1.5625 - 1 ö 25
=ç ÷ ´ 100 = ç ÷ ´ 100 = 56.25% Current I1 = A
è R ø è 1 ø 220
(220) 2
Resistance of 30 W bulb, R 2 = W,
Example 13 : 30
A wire has a resistance of 16.0 ohm. It is melted and drawn 30
Current I 2 = A
to a wire half its initial length. What will be the new 220
resistance of the wire ? When bulbs are connected in series, effective resistance is
Solution : é1 1ù 11
R = R 1 + R 2 = ( 220) 2 ê + ú = 220 ´ 220 ´ W
l¢ 1 ë 25 30 û 150
The factor by which the length is changed is n = = When supply voltage is 440 V, then current is
l 2
The new resistance R' is given by 440 440 ´ 150 27.27
I¢ = = = A
2
R 220 ´ 220 ´ 11 220
æ 1ö As I¢ > I1 but less than I 2 , hence the bulb of 25 watt will
R ¢ = R (n 2 ) = 16 W ç ÷ = 4W
è 2ø fuse.
Example 17.
What will happen when 40 W, 220 V lamp and 100 W,
Example 14 :
220 V lamp are connected in series across 440V lamp?
The resistivity of silver at 0°C is 1.6 × 10–8 W-m. If its Solution :
temperature coefficient of resistance is 4.1 × 10–3 °C–1, Resistance of first lamp,
find the resistivity of silver at 80°C. R1 = 220 × 220 / 40 = 1210 W.
Solution : Resistance of second lamp,
Here, r 0 = 1.6 × 10–8 W-m, a = 4.1 × 10–3 °C–1 and R2 = 220 × 220 /100 = 484 W.
Total resistance in series = 1210 + 484 = 1694 W.
Dt = 80°C Current in the circuit when supply voltage is 440 V will be,
Using r = r0 [1 + a (Dt)] , the resistivity r at a temperature
440
80°C will be I= = 0.26 A
1694
r = 1.6 ´ 10 -8 W - m [1 + (4.1 ´ 10 -3 °C -1 ) (80°C)] Voltage drop across R1 = 0.26 × 1210 = 314.6 V
Voltage drop across R2 = 0.26 × 484 = 112.84V
= 2.1 ´ 10 -8 W - m Therefore 40 W bulb will fuse because lamp can tolerate
only 220 V.
Free eBooks on @neetquestionpaper2020

636 Physi cs
Example 18. Example 20.
Fig. shows three similar lamps L1, L2 and L3 connected A copper kettle weighing 1 kg contains 900 g of water at
across a power supply. 20º C. It takes 12 minutes to raise the temperature to
100ºC. If electric energy is supplied at 210 V, calculate
the strength of current assuming that 10% of heat is wasted.
L1 Sp. heat of copper is 0.1 cal/g°C.
S E Solution :
Here, mass of copper m1 = 1 kg = 100 g,
L2 L3 mass of water, m2 = 900 g
Initial temperature, q2 = 20º C,
If the lamp L3 fuse, how will the light emitted by L1 and Time taken t = 12 mins = 12 × 60 s = 720 s
L2 change? Final temperature, q2 = 100º C,
(a) no change Voltage, V = 210 V,
(b) brilliance of L1 decreases and that of L2 increases Current I=?
(c) brilliance of both L1 and L2 increases Heat wasted = 20%;
(d) brilliance of both L1 and L2 decreases Sp. heat of copper, c = 0.1cal/g°C
Solution : (b) Heat required for raising the temperature of water in the
Let R be the resistance of each lamp. If E be the applied kettle,
e.m.f., then the current in the circuit, I1, is given by H1 = m1 c (q2 - q1 ) + m 2 1 (q 2 - q1 )
E 2E = 100 ´ 0.1 (100 - 200) + 900 (100 - 20)
I1 = =
R + ( R / 2) 3R = 8000 + 72000 = 80,000 cals.
Heat produced,
1 é 2E ù E
Current flowing through L2 or L3 = = 210 ´ I ´ 720
2 êë 3R úû 3R H 2 = VIt = 210 ´ I ´ 720 J = cals.
4. 2
When L3 is fused, the whole current flows through L1 As 10% of heat is wasted, therefore, useful heat produced
and L2. = 90%
E E
Thus I2 < < 90 80 210 ´ I ´ 720
R ∗ R 2R H 2 = H1 i.e. ´ = 80,000
100 100 4.2
So, current through L1 decreases and through L2 increases.
Hence option (b) is correct. 80,000 ´ 4.2 ´ 100
or I= = 2.78 A.
Example 19. 80 ´ 210 ´ 720
A 1000 W immersion heater is placed in a pot containing Example 21.
1 litre of water at 20ºC. How long will it take to heat the Two metallic wires of the same material and same length
water to boiling temperature, if 20% of the available have different diameters. If we connect them in series across
energy is lost to surroundings? a battery the heat produced is H1. If we connect them in
Solution : parallel to the same battery the heat produced during the
Mass of water, m = volume × density same time is H2. Then
= 1000 × 1 = 1000 g (a) H1 = H2
Heat required to heat the water, (b) H1 > H2
H = mc (q2 – q1) + mL (c) H1 < H2
= 1000 × 1 × (100 – 20) + 1000 × 80 (d) it will depend upon the voltage of battery.
= 2000 × 80 cals. Solution : (c)
Let t minutes be taken for heating the water. Let R1, R2 be the resistances of two wires of different
diameters. In series combination, the heat produced,
Electric energy spent in heater = Pt = 1000 t ´ 60 J
V2 t
1000 ´ 60 ´ t = H1 ...(1)
= cals. (R 1 + R 2 )
4.2
In parallel combination, the heat produced,
1000 ´ 60 ´ t 80
Useful heat produced = ´ cal.
4.2 100 V2 t
= H2 ...(2)
(R 1 R 2 ) / (R1 + R 2 )
1000 ´ 60 ´ t 80
Hence ´ = 2000 ´ 80 Dividing eqn. (2) by (1) we get,
4.2 100
H2 (R 1 + R 2 )
4.2 ´ 100 ´ 2 = > 1 or H 2 > H1
or t= = 14 minutes . H1 (R 1 R 2 )
60
Free eBooks on @neetquestionpaper2020

Current Electricity 637

Example 22.
A heater boils 1 kg of water in T1 seconds and another V2 ts V2 t H
(i) H= = 2 s
heater boils the same water in T2 seconds. If both the R 1 + R 2 V (t1 + t 2 )
heaters are connected in parallel, the combination will
boil the same water in time (in seconds) or ts = t1 + t2 = 4 + 6 = 10 minutes.

T1 T2 T1 T2 V 2 (R 1 + R 2 ) t p é 1 1 ù
(ii) H = = V2 t p ê + ú
(a) (b) R1 R 2
T1 - T2 T1 + T2 ë R 2 R1 û

é ù æ t ∗ t ö÷
T12 + T22 T12 - T22 ê H H ú ç 1÷
< V2 tp ê 2 ∗ 2 ú < H tp çç 2 ÷÷
(c) (d)
T1 - T2 ê V t1 V t2 ú ç
çè t1 t2 ø÷
T1 + T2 ë û
Solution : (b) t2 t1 4≥6 24
Let R1, R2 be the resistances of two heaters. Effective or tp < < < < 2.4 minutes .
t1 ∗ t2 4 ∗ 10 10
R1 R 2
resistance of two heaters in parallel = Example 24.
R1 + R 2
A fuse wire with a radius of 1 mm blows at 1.5 A. If the fuse
If V be the voltage supplied and Tp be the time taken for wire of the same material should blow at 3.0 A, the radius
boiling the water when heaters are in parallel, then of the wire must be
V 2 T1 V 2 T2 V 2 Tp (a) 41/3 mm (b) 2 mm
H= = = (c) 0.5 mm (d) 8.0 mm.
R1 R2 R 1 R 2 / (R 1 + R 2 )
Solution : (a)
V 2 T1 V 2 T2 The temperature of the wire increases to such a value at
so R1 = and R 2 = which, the heat produced per second equals heat lost per
H H
second due to radiation i.e.
V 2 Tp (R 1 + R 2 ) = V 2 T é 1 + 1 ù
Now H = pê ú æ rl ö
R1 R 2 ë R 2 R1 û I 2 çç 2 ÷÷ = H ´ 2 p r l ,
è pr ø
é H H ù where H is heat lost per second per unit area due to radiation.
= V 2 Tp ê 2 + 2 ú
ëê V T1 V T2 ûú I 12 r13
Hence, I 2 µ r 3 so =
é T + T2 ù é T1 T2 ù I 22 r 23
= H Tp ê 1 ú or Tp = ê ú.
ë T1 T2 û ë T1 + T2 û or r 2 = r1 (I 2 / I 1 ) 2/ 3 < 1≥(3.0/1.5)2/3 < 41/3 mm.
Example 23.
An electric tea kettle has two heating coils. When one of Example 25
coils is switched on, the kettle begins to boil in 4 minutes.
A fuse wire with a circular cross section and having
When the other is switched on, the boiling begins in 6
minutes. In what time will the boiling begins if both coils diameter of 0.4 mm blows with a current of 3 amp. The
are switched on simultaneously (i) in series (ii) in parallel? value of the current for which another fuse wire made of
Solution : the same material but having circular cross-section with
Here; t1 = 4 minutes; t2 = 6 minutes. Let R1 and R2 be the diameter of 0.06 mm will blow is
resistance of the two coils and V be the voltage supplied. (a) 3 amp. (b) 3 ´ (3/2) amp.
Let ts be the time taken when the coils are in series and t p be
the time taken when they are in parallel. (c) 3×(3/2) amp. (d) 3 ´ (3/2) 3/ 2 amp.
Effective resistance of two coils in series = R1 + R2 Solution : (d)
R1 R 2 For a fuse wire I 2 µ r 3
Effective resistance of two coils in parallel =
R1 + R 2
Let H be the heat required to begin boiling in kettle, which I12 r13
\ =
is the same for every case. Then I 22 r23
V2 t1 V2 t2 0.04 0.06
H = power × time < < where r1 = = 0.02 cm and r2 = = 0.03 cm.
R1 R2 2 2
2 2
V ts V tp (3) 2 æ 0.02 ö
3
æ 2ö
3
< < \ =ç ÷ =ç ÷
(R1 ∗ R2 ) (R1R2 /R1 ∗ R2 ) I 22 è 0. 03 ø è 3ø
V 2 t1 V2 t 2 3 3/ 2
or R1 = and R 2 = æ 3ö æ 3ö
H H or I 22 = (3) 2 ç ÷ or I 2 = 3 ´ ç ÷
Using these values, we have è 2ø è 2ø
Free eBooks on @neetquestionpaper2020

638 Physi cs

18.2
Solve following problems with the help of above text and 12. As the temperature of a conductor increases, its resistivity
examples. and conductivity change. The ratio of resistivity to
1. The unit of specific resistance is conductivity
(a) increases
(a) W m -1 (b) W -1 m -1 (b) decreases
(c) remains constant
(c) W -1 (d) 2.5Wm2 (d) may increase or decrease depending on the actual
2. If n is the number density of free electrons in a metallic wire, temperature.
then the resistance is proportional to 13. A current passes through a wire of nonuniform cross-
(a) n (b) n 2 section. Which of the following quantities are independent
(c) 1/n (d) 1/n2 of the cross-section?
3. The example of non-ohmic resistance is (a) The charge crossing in a given time interval
(a) diode (b) copper wire (b) Drift velocity
(c) filament lamp (d) carbon resistor (c) Current density
(d) Free-electron density
4. Constantan wire is used for making standard resistance, 14. A metallic resistor is connected across a battery. If the
because it has number of collisions of the free electrons with the lattice is
(a) high melting point somehow decreased in the resistor (for example, by cooling
(b) low specific resistance it), the current will
(c) high specific resistance (a) increase (b) decrease
(d) negligible temperature coefficient of resistance (c) remain constant (d) become zero
5. At temperature 0K, the germanium behaves as a / an 15. Two resistors A and B have resistances RA and RB
(a) conductor (b) insulator respectively with RA < RB. The resistivities of their
(c) super-conductor (d) ferromagnetic materials are rA and rB. Then
(a) rA > rB (b) rA = rB
6. If temperature is decreased, then relaxation time of (c) rA < rB
electrons in metals will (d) The information is not sufficient to find the relation
(a) increase (b) decrease between rA and rB.
(c) fluctuate (d) remains constant 16. Appropriate material to be used in the construction of
7. The current I and voltage V graphs for a given metallic resistance boxes out of the following is
wire at two different temperatures T1 and T2 are shown in
(a) copper (b) iron
the fig. It is concluded that
I
(c) manganin (d) aluminium
T2 17. Two resistors R and 2R are connected in parallel in an
electric circuit. The thermal energy developed in R and 2R
T1 are in the ratio of
(a) 1 : 2 (b) 2 : 1
V (c) 1 : 4 (d) 4 : 1
(a) T1 > T2 (b) T1 < T2 18. A certain piece of copper is to be shaped into a conductor
(c) T1 = T2 (d) T1 = 2 T1 of minimum resistance. Its length and diameter should be
8. A typical thermistor can easily measure a change in respectively
temperature of the order of (a) l, d (b) 2 l, d
(a) 10–3 ºC (b) 10–2 ºC (c) l/2, 2 d (d) 2 l, d/2
2
(c) 10 ºC (d) 103 ºC 19. From the graph between current (I) and voltage (V) shown
9. Which of the following is used for the formation of below, identify the portion corresponding to negative
thermistor? resistance.
(a) Copper oxide (b) Nickel oxide E
(c) Iron oxide (d) All of the above B C
10. When potential difference is applied across an electrolyte,
Current (I)

then Ohm’s law is obeyed at


(a) zero potential (b) very low potential D
(c) negative potential (d) high potential
11. Which of the following quantities do not change when a
resistor connected to a battery is heated due to the
current? A Voltage (V)
(a) Drift speed (b) Resistivity (a) AB (b) BC
(c) Resistance (d) Number of free electrons (c) CD (d) DE
Free eBooks on @neetquestionpaper2020

Current Electricity 639

20. The smallest resistance that can be obtained by the 29. A current I passes through a wire of length l, radius r and
combination of n resistors, each of resistance R is resistivity r. The rate of heat generated is
(a) n2 R (b) nR
I 2r l I 2 rl
(c) R/n2 (d) R/n (a) (b)
21. Which of the following can be used to express energy? r pr2
Symbols have their usual meanings for the units of physical
I 2rl
quantities. (c) (d) None of these
(i) W h (ii) VC pr
(iii) V As 2 (iv) A2 W s. 30. Two filaments of same length are connected first in series
(a) (i), (ii) (b) (ii), (iii), (iv) and then in parallel. For the same amount of main current
(c) (i), (ii), (iv) (d) (i), (ii), (iii), (iv) flowing the ratio of the (parallel to series) heat produced
22. What should be the characteristic of fuse wire? is
(a) High melting point, high specific resistance (a) 2 : 1 (b) 1 : 2
(b) Low melting point, low specific resistance (c) 4 : 1 (d) 1 : 4
(c) High melting point, low specific resistance
31. The resistance R1 and R2 are joined in parallel and a current
(d) Low melting point, high specific resistance
is passed so that the amount of heat liberated is H1 and
23. What is the suitable material for electric fuse?
H2 respectively. The ratio H1/H2 has the value
(a) Cu (b) Constantan
(a) R2/R1 (b) R1/R2
(c) Tin-lead alloy (d) Nichrome
24. The heating element of an electric heater should be made (c) R12 / R 22 (d) R 22 / R12
with a material, which should have
32. Two electric bulbs rated P1 watt V volts and P2 watt V
(a) high specific resistance and high melting point
volts are connected in parallel and applied across V volts.
(b) high specific resistance and low melting point
The total power (in watt) will be
(c) low specific resistance and low melting point
(d) low specific resistance and high melting point (a) P1 + P2 (b) P1 P2
25. Resistance of conductor is doubled keeping the potential
difference across it constant. The rate of generation of P1 P2 P1 + P2
heat will (c) P1 + P2
(d)
P1 P2
(a) become one fourth (b) be halved
33. Which of the following plots may represent the thermal
(c) be doubled (d) become four times
energy produced in a resistor in a given time as a function
26. You have the following appliances each of 500 watt of the electric current?
running on 220 volts a.c. (1) electric iron (2) an electric
U d
lamp (3) an electric room heater. The electric resistance is
(a) maximum for the heater
(b) maximum for the electric lamp c
(c) maximum for the electric iron
b
(d) same in all the three cases a
27. Of the two bulbs in a house, one glows brighter than the
other. Which of the two has a large resistance? I
(a) The bright bulb (a) a (b) b
(b) The dim bulb (c) c (d) d
(c) Both have the same resistance 34. The heat developed in a system is proportional to the
(d) The brightness does not depend upon the resistance. current through it. Then
28. Appliances based on heating effect of current work on (a) it cannot be Thomson heat
(b) it cannot be Peltier heat
(a) only a.c. (b) only d.c.
(c) it cannot be Joule heat
(c) both a.c. and d.c. (d) None of these
(d) it can be any of the three heats mentioned above

ANSWER KEY
1. (c) 2. (c) 3. (a) 4. (d) 5. (b) 6. (a) 7. (a) 8. (a) 9. (d) 10. (d) 11. (d) 12. (c)
13. (a,d) 14. (a) 15. (d) 16. (c) 17. (b) 18. (c) 19. (c) 20. (d) 21. (c) 22. (d) 23. (c) 24. (a)
25. (b) 26. (d) 27. (b) 28. (c) 29. (b) 30. (d) 31. (a) 32. (a) 33. (d) 34. (c)
Free eBooks on @neetquestionpaper2020

640 Physi cs

ELECTROMOTIVE FORCE AND INTERNAL RESISTANCE 2. Emf is the property of a cell but terminal potential difference
OF A CELL depends on the current drawn from the cell.
An emf (electromotive force) device has a positive terminal (at
high potential) and a negative terminal (at low potential). This Short Circuiting
device is responsible for moving positive charge within itself When the terminals of an emf device are connected with a
from negative terminal to positive terminal.
conducting path without any external resistance then

+ –
+
H.P L.P I

+ r
H.P L.P
E
For this to happen, work is done by some agency in the emf
device. The energy required to do this work is chemical energy E
(as in a battery), mechanical energy (as in electric generator), E = Ir Þ I=
r
temperature difference (as in a thermopile).
The emf is thus given by the formula Since internal resistance has a very small value, therefore a very
high current flows in the circuit producing a large amount of
dW
E= heat. This condition is called short circuiting.
dq
During short circuiting, the terminal potential
J
The S.I unit of emf is < volt(V) difference is zero.
C
COMBINATION OF CELLS
Keep in Memory
Series Combination of Cells
1. Electromotive force is not a force but a potential difference.
2. E.m.f. can be defined as the work done in moving a charge E1 E2 En
once around a closed circuit.
A r1 r2 rn B
Internal Resistance (r) Equivalent Emf EAB = E1 + E2 + ... + En
The potential difference across a real source of emf is not equal Equivalent internal resistance, RAB = r1 + r2 + ....... + rn
to its emf. The reason is that the charge which is moving inside Parallel Combination of Cells
the emf device also suffers resistance. This resistance is called
internal resistance of the emf device. E1
E
r1
+ r –
I
E2
A r2 B
En
I R
p.d V rn
E = IR + Ir = V + Ir Equivalent emf
Þ V = E – Ir
E1 E 2 E
+ + ........+ n
Keep in Memory r1 r2 rn
E AB =
1. For a cell 1 1 1
+ + .....
Circuit Mode Expression r1 r2 rn

(i) Discharging V = E – Ir \V <E Equivalent internal resistance


1 1 1 1
= + + .........+
(ii) No current V= E R AB r1 r2 rn
Mixed Grouping of Cells :
(iii) Charging V = E + Ir \V >E If the cells are connected as shown below then they are said to
I
be in mixed grouping.
Free eBooks on @neetquestionpaper2020

Current Electricity 641

1
FARADAY’S LAW OF ELECTROLYSIS
2 n
1 (i) 1st law : The mass of the substance liberated or deposited
E r E r E r
at an electrode during electrolysis is directly proportional
to the quantity of charge passed through the electrolyte.
2
i.e., mass m µ q = Zq = Z It,
A E r E r E r B where Z = electrochemical equivalent (E.C.E.) of substance.
(ii) 2nd law : When the same amount of charge is passed
through different electrolytes, the masses of the substance
m
liberated or deposited at the various electrodes are
E r E r E
r proportional to their chemical equivalents
Equivalent emf EAB = nE m1 E
i.e. = 1
nr m 2 E2
Equivalent resistance =
m where m1 and m2 are the masses of the substances liberated
Where n = no. of cells in a row. and or deposited on electrodes during electrolysis and E1 and
M = no. of rows E2 are their chemical equivalents.
If this equivalent cell is attached to an external resistance R Faraday's Constant
then Faraday constant is equal to the amount of charge required to
nr liberate the mass of a substance at an electrode during electrolysis,
m equal to its chemical equivalent in gram (i.e. one gram equivalent)
nE One faraday (I F) = 96500 C/gram equivalent.
Keep in Memory
1. If r is the density of the material deposited and A is the area
R of deposition, then the thickness (d) of the layer deposited
æ nr ö m ZIt
nE = I ç R + ÷ Þ I = nmE in electroplating process is d = = .
è mø mR + nr rA rA
2. The back e.m.f. for water voltameter is 1.67 V and it is 1.34 V
Keep in Memory
for CuCl2 electrolytes voltameter with platinum electrodes.
(i) The condition for maximum current through external resistance 3. 96500 C are required to liberate 1.008 g of hydrogen.
R 4. 2.016 g of hydrogen occupies 22.4 litres at N.T.P.
5. E.C.E. of a substance = E.C.E. of hydrogen × chemical
m r
nr = mR Þ = Þ R = nr/m equivalent of the substance.
n R
In other words, when external resistance is equal to total Example 26.
internal resistances of all the cells. Upon a six fold increase in the external resistance of a circuit,
nE mE the voltage across the terminals of the battery has increased
The maximum current Imax = or from 5 V to 10 V. Find the e.m.f. of battery.
2R 2r Solution :
(ii) Maximum power dissipation for the circuit shown in fig. V = E – Ir.
E r E
V =E- ´r
(R + r)
E
E- ´r =5 ...(1)
(R + r )
æ R ö
R or E ç ÷=5 ...(2)
2
èR+rø
2
Power P = I 2 R = æç
E ö E R
÷ R= E
èR+rø (R + r ) 2 E- ´ r = 10 ...(3)
( 6R + r )
dP æ 6R ö
For maximum power across the resistor, =0
dR or E ç ÷ = 10 ...(4)
è 6R + r ø
On solving, we get R = r
Dividing eqn. (2) by eqn. (4), we get 2r = 3R
This is the condition for maximum power dissipation.
(iii) If identical cells are connected in a loop in order, then emf E´ r
between any two points in the loop is zero. Putting values in eqn. (3), we get E - = 10
4r + r
(iv) If n identical cells are connected in series and m are wrongly
connected then Enet = nE – 2mE Solving we get E = 12.5 V
Free eBooks on @neetquestionpaper2020

642 Physi cs
Example 27. 1.5 + 1.5V
12 cells each having the same e.m.f. are connected in series \ i= = 2.0 A
and are kept in a closed box. Some of the cells are wrongly 1.0 + 0.5W
connected. This battery is connected in series with an The voltage across the cell P is
ammeter and two cells identical with the others. The Vp = E1 - ir1 = 1.5V - 1.5 (1.0)V = zero
current is 3 A when the cells and battery aid each other and across cell Q is
and 2 A when the cells and battery oppose each other.
How many cells in the battery are wrongly connected? VQ = E 2 - ir2 = 1.5V - 1.5 (0.5)V = 0.75V
Solution : Example 29.
Let x cells be connected correctly and y cells are connected If electrochemical equivalent of hydrogen is ZH kg/coul-
wrongly. According to the given problem equivalent and chemical equivalent of copper is W, then
x + y = 12 ... (1) determine the electrochemical equivalent of copper.
If E be the e.m.f. of one cell, then the net e.m.f. will be (x – y) E Solution :
Let R be the resistance of the circuit which remains constant. Z Cu W
(i) When the cells aid the battery, the net e.m.f. =
(x – y) E + 2E ZH WH
Net e.m.f . (x - y)E + 2E We know that, WH = 1
Current = = =3 ... (2) \ ZCu = W.ZH
Resistance R
(ii) When the cells oppose the battery, net e.m.f. Example 30.
= (x – y) E – 2E Find the mass of silver liberated in a silver voltameter
Net e.m.f. (x - y) E - 2E carrying a current of 1.5A, during 15 minutes. The electro
Current = = =2 ... (3) chemical equivalent of silver is 1.12× 10–6 kg/C.
Resistance R
Solution :
( x - y) E + 2 E 3 Here, m = ?, i = 1.5 A, Z = 1.12× 10–6 kg/C
Dividing equation (2) by (3), we get ( x - y) E - 2E = 2 and t = 15 × 30s
Using m = Zit
( x - y) + 2 3 Mass of silver liberated is
or = ... (4)
(x - y) - 2 2 æ kg ö æ 1.5C ö
From eqns. (1) and (4), we get x = 11 and y = 1 m = ç1.12 ´ 10-6 ÷ ç ÷ (450s)
è Cøè s ø
Hence only one cell is wrongly connected.
= 7.6 × 10–4 kg = 0.76 g
Example 28.
Two cells P and Q connected in series have each an emf of Example 31.
1.5 V and internal resistances 1.0 Wand 0.5W respectively. In a water voltameter, the act of passing a certain amount of
current for a certain time produces of 1.2 g H2 at STP. Find
Find the current through them and the voltages across
the amount of O2 liberated during that period.
their terminals.
Solution :
Solution :
Since the same current flows through both the electrodes of
For a single closed loop, consisting of cells and resistors
a water voltameter, so the amount of oxygen and hydrogen
the current i flowing through it is given by
liberated (for the same charge) will be directly proportional
E1=1.5V r1=1.0
to their respective equivalent weights.
P By Faradays’ second law of electrolysis,
mO 2 8
i.e., =
SE i i mH2 1
i=
Sr + Sri
Q or mO 2 = æç 8 ö÷ m H = 8 ´ 1.2g = 9.6g
è 1ø 2
E2=1.5V r2=0.5

18.3
Solve following problems with the help of above text and (c) always goes from the negative terminal to the
examples. positive terminal
1. In an electric circuit containing a battery, the charge (d) does not move.
(assumed positive) inside the battery 2. A resistor of resistance R is connected to an ideal battery.
(a) always goes from the positive terminal to the If the value of R is decreased, the power dissipated in the
negative terminal resistor will
(b) may go from the positive terminal to the negative (a) increase (b) decrease
terminal (c) remain unchanged (d) None of these
Free eBooks on @neetquestionpaper2020

Current Electricity 643

3. Two non ideal batteries are connected in series. Consider 10. Same current is being passed through a copper voltameter
the following statements and silver voltameter. The rate of increase in weight of the
(A) The equivalent emf is larger than either of the two cathodes in the two voltameters will be proportional to
emfs. (a) relative densities (b) atomic masses
(B) The equivalent internal resistance is smaller than
(c) atomic numbers (d) none of the above
either of the two internal resistances.
(a) Both A and B are correct 11. If E be chemical equivalent of an element and Z is its
(b) A is correct but B is wrong electrochemical equivalent, then E/Z is measured in
(c) B is correct but A is wrong (a) farad (b) newton
(d) Both A and B are wrong (c) coulomb (d) faraday
4. In a closed circuit the e.m.f. and internal resistance of the 12. A silver and a zinc voltameter are connected in series and
generator are E and r respectively. If the external resistance a current I is passed through them for a time t, liberating
in the circuit is R, then the Ohm’s law has the form W grams of zinc. The weight of silver deposited is nearly
E E (a) 1.7 W gram (b) 2.4 W gram
(a) Ι= (b) Ι =
(R + r ) R (c) 3.5 W gram (d) 1.2 W gram
13. What determines the e.m.f. between the two metals placed
E E
(c) Ι= (d) Ι = in an electrolyte?
r Rr (a) Relative position of metals in the electro chemical
5. To draw a maximum current from a combination of cells, series
how should the cells be grouped?
(b) Distance between them
(a) Parallel
(b) Series (c) Strength of electrolyte
(c) Mixed grouping (d) Nature of electrolyte
(d) Depends upon the relative values of internal and 14. For electroplating a spoon, it is placed in the voltameter at
external resistances. (a) the position of anode
6. Under what condition will the strength of current in a wire (b) the position of cathode
of resistance R be the same for connection is series and in (c) exactly in the middle of anode and cathode
parallel of n identical cells each of the internal resistance
(d) anywhere in the electrolyte
r? When
(a) R = n r (b) R = r/n 15. Electroplating does not help
(a) fine finish to the surface
(c) R = r (d) R ® ¥, r ® 0
(b) shining appearance
7. A cell of internal resistance r is connected to an external
(c) metals to become hard
resistance R. The current will be maximum in R, if
(a) R = r (b) R < r (d) protect metals against corrosion
(c) R > r (d) R = r/2 16. Faraday’s laws are consequence of conservation of
8. Faraday’s laws of electrolysis are related to (a) energy
(a) the atomic number of positive ion (b) energy and magnetic field
(b) the equivalent weight of electrolyte (c) charge
(c) the atomic number of negative ion (d) magnetic field
(d) the velocity of positive ion 17. According to Faraday’s law of electrolysis, the amount of
9. The graph represents the current-voltage behaviour of a deposition is proportional to
water voltameter. Spot the correct explanation.
(a) 1/time for which current passes
y (b) electrochemical equivalent of the substance
(c) 1/current
(d) 1/electrochemical equivalent
Current

18. The current inside a copper voltameter


(a) is half the outside value
E
(b) is the same as the outside value
x (c) is twice the outside value
Potential difference
(d) depends on the concentration of CuSO4.
(a) Ohm’s law is obeyed but only for potential difference
19. The relation between Faraday constant F, electron charge
larger than the back e.m.f.
e, and Avogadro number N is
(b) Electrolysis do not obey Ohm’s law
(c) Ohm’s law is not valid for low voltages (a) F = N/e (b) F = Ne
(d) Dissociation takes place at E and it obey’s Ohm’s (c) N = Fe2 (d) F = N2 e
law thereafter.
Free eBooks on @neetquestionpaper2020

644 Physi cs

20. In electrolysis, the amount of mass deposited or liberated 22. If a cell of constant electromotive force produces the same
at an electrode is directly proportional to amount of heat during the same time in two independent
(a) amount of charge resistances R1 and R2, when connected separately one
(b) square of current after the other, across the cell, then the internal resistance
(c) concentration of electrolyte of the cell is
(d) square of electric charge
21. Of the following, the one that does not make use of chemical (R 1 - R 2 )
effect of electric current is (a) (R1 + R2) (b)
2
(a) electroplating
(b) electrotyping R1 - R 2
(c) thermoelectric refrigerator (c) (d) R 1R 2
2
(d) voltaic cells
ANSWER KEY
1. (c) 2. (b) 3. (b) 4. (a) 5. (d) 6. (c) 7. (a) 8. (b) 9. (b) 10. (d) 11. (d) 12. (c)
13. (d) 14. (b) 15. (c) 16. (a) 17. (b) 18. (b) 19.(b) 20.(a) 21. (c) 22.(d)

SEEBECK/THERMOELECTRIC EFFECT
When an electric circuit is composed of two dissimilar metals
and the junctions are maintained at different temperature, then

Thermo emf
an emf is set up in the circuit. This effect is known as
thermoelectric or seebeck effect.
Thermocouple: It is a device in which heat energy is converted
into electrical energy. Its working is based on seebeck effect. It
has two junctions of two dissimilar metals. O To Tn Ti
Temp. difference
Cu
With temperature difference T between hot and cold junctions,
the thermo-e.m.f. is given by
T1 T2 E = aT + bT2
Fe where a and b are Seebeck co-efficients
At Tn, (dE/dT) = 0
Some of the elements forming thermo-electric series \ Tn = – a/2b and Ti = – a/b, when To = 0
Sb, Fe, Zn, Cu, Au, Ag, Pb, Al, Hg, Pt, Ni, Bi S = dE/dT is called thermo-electric power.
(i) Lead (Pb) is thermo-electrically neutral PELTIER EFFECT
(ii) At the cold junction, current flows from the element occuring It states that if current is passed through a junction of two
earlier into the element occuring later in the series. different metals the heat is either evolved or absorbed at that
For example: In Cu–Fe thermo–couple, current flows from junction. It is the reverse of seebeck effect.
Cu to Fe at hot junction. The quantity of heat evolved or absorbed at a junction due to
Peltier effect is proportional to the quantity of charge crossing
Neutral Temperature (Tn)
that junction.
It is that temperature of hot junction for which the thermo emf
Peltier Coefficient (p) :
produced in a thermocouple is maximum.
It is defined as the amount of heat energy evolved or absorbed
It depends upon the nature of the material of thermocouple but is
per second at a junction of two different metals when a unit
independent of temperature of cold junction.
current is passed through it.
Temperature of Inversion (Ti) The Peltier heat evolved or absorbed at a junction of a
It is that temperature of hot junction for which the thermo emf thermocouple = pI t
becomes zero and beyond this temperature, the thermo emf in a where I = current passing through the junction for time t.
thermocouple reverses its direction.
It depends upon the nature of the material of thermocouple and dE
\ p =T
temperature of cold junction dT
Let To, Tn, T i be the temperature of cold junction, neutral where T and (T + dT) are the temperature of cold and hot junctions
temperature and temperature of inversion then of a thermocouple and dE is the thermo emf produced.
Tn - T0 = Ti - Tn or Tn = (Ti + To ) / 2 p dE
\ = = S (Seebeck coefficient)
T dT
Free eBooks on @neetquestionpaper2020

Current Electricity 645

THOMSON EFFECT Solution :


If a metallic wire has a non uniform temperature and an electric Let Ti,Tn and Tc be the temperature of inversion, neutral
current is passed through it, heat may be absorbed or produced temperature and temperature of cold junction
respectively, then
in different sections of the wire. This heat is over and above the
Ti – Tn = Tn – Tc
joule heat I2Rt and is called Thomson heat. The effect is called \ 620 – 300 = 300 – Tc Þ 320 = 300 – Tc
Thomson effect. \ Tc = 300 – 320 = – 20ºC
If a charge DQ is passed through a small section of given Example 33.
wire having temperature difference DT between the ends, The temperature of cold junction of a thermocouple is 0ºC
Thomson heat, DH = s DQ DT and the temperature of hot junction is TºC. The thermo
where s is constant for a given metal at a given temperature. e.m.f. is given by E = 16 T – 0.04 T2 m volt
Thomson emf, s DT, is defined as s DT = DH/DQ. Find (a) the neutral temperature and (b) the
temperature of inversion
s is positive if heat is absorbed when a current is passed Solution :
Given that E = 16T – 0.04T2
from low temp. to high temperature. s is numerically equal to P.D.
dE
developed between two points of the conductor differing in temp. \ = 16 - 2 ´ 0.04T
by 1ºC. dT
(a) At neutral temperature, dE/dT = 0
Keep in Memory \ 16 – 2×0.04 Tn = 0 or 16 = 0.08 Tn
1. The actual emf developed in a thermocouple loop is the 16
or Tn = = 200º C
algebraic sum of the net Peltier emf and the net Thomson 0.08
emf developed in the loop. (b) At the temperature of inversion, E = 0
\ 16 Ti – 0.04 Ti2 = 0 or 16 – 0.04 Ti = 0
sA(T–T0) or 16 = 0.04 Ti
A 16
\ Ti = = 400º C
(pAB)T 0.040
Example 34.
B (pAB)T0 One junction of a certain thermocouple is at a fixed
temperature Tr and the other junction is at a temperature
T sB(T–T0) T0 T. The electromotive force for this is expressed by,

EAB = (pAB )T - (pAB )To + s A (T - To ) - s B (T - To ) é 1 ù


E = k (T - Tr ) êT0 - (T + Tr ) ú .
ë 2 û
2. If S, p and s are the Seebeck coefficient, Peltier coefficient,
At, temperature T = T0/2. Determine the thermoelectric
and Thomson coefficient respectively then it is found that
power.
dE p Solution :
(i) S= =
dT T
é T 2 T Tr T Tr Tr2 ù
E = k êT T0 - - - Tr T0 + - ú
d2E d æ dE ö TdS êë 2 2 2 2 úû
(ii) s = -T 2
= -T ç ÷=-
dT dT è dT ø dT
dE é T T ù
3. For Peltier effect or Thomson effect, the heat evolved or Hence = k êT0 - T - r + r ú = k ( T0 - T )
dT ë 2 2û
absorbed is directly proportional to current. But for Joule's
law of heating, the heat produced is directly proportional to At temperature T = T0 /2,
the square of the current flowing through it. Thermoelectric power = k (T0 - T0 / 2) = k T0 / 2.
4. Thermo-emf set up in a thermocouple when its junctions Example 35.
The thermoelectric powers of iron and copper w.r.t. lead
are maintained at temperature T1 and T3 (i.e. E TT3 ) is equal are + 10.5 micro volt and + 13.5 micro volt respectively
1
at 100ºC. Find the e.m.f of a copper iron couple with
to the sum of the emfs set up in a thermocouple when its
junctions at 50ºC and 150ºC.
junctions are maintained first at temperature T1 and T2 (i.e. Solution :
T T
E T2 ) and then at T2 & T3 (i.e. E T32 ) i.e. E T3 = E T2 + E T3 dE
1 T1 T1 T2 Thermoelectric power = = S or E = ST..
It is called law of intermediate temperature. dT
\ (E) Fe
Pb = 10.5 ´100 = 1050 m V and
Example 32.
The temperature of inversion of a thermocouple is 620ºC (E)Cu
Pb = 3.5 ´ 100 = 350 m V
and the neutral temperature is 300ºC. What is the Hence E Cu Cu Pb Pb Pb
Fe = E Pb + E Fe = E Fe - E Cu
temperature of cold junction ?
= 1050 – 350 = 700 mV.
Free eBooks on @neetquestionpaper2020

646 Physi cs
Example 36.
Example 37.
bT 2 The thermo emf of a copper constantan thermocouple varies
The emf is given by E = aT + and a = 10µV/ºC2,
2
bT 2
-1 as E = aT +
b= µV/ºC2. If temperature of cold junction is zero find 2
20
neutral temperature and temperature of inversion? where a = 40 µV/C and b = 40 × 10–3 µ V/C2.
Solution : If T = 100ºC and Tc = 0ºC then find thermo-emf ?
Solution :
dE –a –10
At T = Tn
dT
= 0 or Tn =
b
=
-1 / 20
= 200ºC bT 2 40 ´ 10-9 ´ (100)2
E = aT + = 40 × 10–6 × 100 +
Ti = 2Tn = 400 ºC 2 2
= 4.2 × 10–3 V

18.4
Solve following problems with the help of above text and 7. The temperature of the thermocouple, which is
examples. independent of the temperature of cold junction is called
1. In Seebeck series, antimony appears before bismuth. But (a) zero temperature
in Sb-Bi thermocouple, the current flows from (b) neutral temperature
(a) Bi to Sb through hot junction (c) temperature of inversion
(b) Bi to Sb through cold junction (d) None of these
(c) Sb to Bi through cold junction
8. Amount of heat energy absorbed or evolved when
(d) Sb to Bi through hot junction
1 A of current passes for 1 second through a portion of
2. Which of the following thermocouple produces the
metal kept at a temperature difference of 1ºC is called
maximum thermo e.m.f. for the same temperature difference
between hot and cold junctions? (a) thermo e.m.f. (b) Thomson co-efficient
(a) Copper-bismuth (b) Antimony-bismuth (c) thermoelectric power (d) Peltier coefficient
(c) Iron-nickel (d) Copper-iron 9. At neutral temperature, thermoelectric power is
3. When a number of thermocouples are joined in series, the (a) infinite (b) zero
thermo e.m.f. (c) minimum (d) maximum
(a) is decreased (b) is increased 10. Amount of energy absorbed or evolved when 1 A of
(c) becomes zero (d) remains same current passes for one second through a junction of two
4. If one junction of a thermocouple is heated and other metals is called
junction is cooled continuously in the same steps, the (a) Peltier coefficient (b) Thermo e.m.f.
e.m.f. will
(c) Thomson coefficient (d) Thermoelectric power
(a) first increase and then decrease
11. Peltier coefficient of the junction of a pair of metals is
(b) first increase and then decrease and finally reverse
proportional to
(c) not change at all
(a) absolute temperature of junction T
(d) change appreciably
5. Inversion temperature of a thermocouple is that (b) square of absolute temperature of junction
temperature of hot junction at which the emf is (c) 1/T (d) 1/T2
(a) maximum (b) minimum 12. Which of the following statement is correct?
(c) zero (d) None of these (a) Both Peltier and Joule effects are reversible
6. Thermocouple is an arrangement of two different metals (b) Both Peltier and Joule effects are irreversible
to convert (c) Joule effect is reversible, whereas Peltier effect is
(a) heat energy into electrical energy irreversible
(b) electrical energy into chemical energy (d) Joule effect is irreversible, whereas Peltier effect is
(c) electrical energy into heat energy reversible
(d) heat energy into chemical energy

ANSWER KEY
1. (c) 2. (b) 3. (b) 4. (b) 5. (c) 6. (a) 7. (b) 8. (b) 9. (b) 10. (a) 11. (a) 12. (d)
Free eBooks on @neetquestionpaper2020

Current Electricity 647


KIRCHOFF’S LAWS AND ELECTRICAL CIRCUIT
E2 a E1
Many practical combination of resistors cannot be reduced to b f
simple series, parallel combinations. For example the resistors in R2 I2
the figure are neither in series nor in parallel. loop
loop R1
n R3
e direction
dir
a R1 I1+ I2
R5 R1
f d c R4 I1 d I2 R e
2
E R2 Fig 1
b R2
I2 I1
c E2 a E1
I b f
I1 I2
The use of Ohm’s law is not sufficient to solve such problems.
Kirchoff’s laws are used in such cases. R3 R1
We will often use the term junction and loop, so let us first
understand the meaning of these words. A junction in a circuit I3
is a point where three or more conductors meet. A loop is a
c R4 I1 d R2 I2 e
closed conducting path. In the above figure e, f, d, c are junctions.
a, b, are not junctions. The various loops are efde, cdfc, eabcf Fig 2
and eabcde.
In the above circuits we arbitrarily assumed the direction
(i) Kirchoff’s junction law : (Based on conservation of
of current I1 in branch abcd as anti-clockwiswe and the
charge) At any junction, the sum of currents entering the
direction of current I2 in branch afed as clockwise.
junction must be equal to the sum of currents leaving it.
If this is not so, charges will accumulate at the junction. In figure 1 we have two unknown currents (I1, I2) whereas
This cannot happen as this would mean high/low potential figure 2 we have three unknown currents (I1, I2 and I3).
maintained at a point in a wire without external influence. The first figure is a better option for solving problems. In
When we apply this rule at junction c, we get I = I 1 + I2 figure 1 we used junction rule at d simultaneously while
(ii) Kirchoff’s loop law : (Based on energy conservation) labelling currents.
The algebraic sum of changes in potential around any (iv) In a branch containing a capacitor, the current is zero when
closed loop of a circuit must be zero. d.c is applied and steady state conditions are achieved.
Sign convention for using loop law. If we move a loop element (v) Now we need as many independent equations as there are
(resistor, emf device, capacitor, inductor etc.) in the direction of conditions unknowns. If we have to find a particular
increasing potential, we take the potential difference positive unknown, we should ensure that, the unknown appears in
and vice-versa. one of the equations made by us.
Travel (vi) For making equations choose the loop and travel the loop
– + p.d = +E
E completely. We may travel the loop in clockwise or anti-
clockwise direction. While using second law use sign
Travel
p.d = –E conventions properly.
+E – (vii) Solve the equations formed to find the unknown quantities.
H.P. L.P. If any value of current comes out to be negative then that
p.d = – IR
I R particular current is in the opposite direction to that
assumed.
Travel
L. P. H.P
Applications
p.d = +IR
Let us use second law in the loop abcda of figure 1 taking the
R I
loop in anti-clockwise direction starting from a.
Travel
+ E2 – I1R4 – (I1 + I2) R3 = 0
Problem Solving Tactic for Using Kirchoff’s Law
For loop afeda, moving the loop in clockwise direction we get –
(i) Draw a circuit diagram large enough to show all resistors,
emf device, capacitors, currents clearly. E1 – I2 R1 – I2R2 – (I1 + I2)R3 = 0
(ii) Take into account the resistance of voltmeter/ammeter/ Node method to apply Kirchoff’s law (Open loop method)
internal resistance of a cell (if given). Step 1 : We select a reference node and assume its potential
(iii) Assume the direction of current in all branches. It may be to be (zero/x)V
noted here that one branch has only one direction of Step 2 : We calculate the voltage of other selected points w.r.t.
current. the reference node
It is best to use junction law simultaneously while drawing Step 3: We find some independent node (whose voltage is
currents. This helps to reduce the number of unknown not known). We apply Kirchoff’s law to find the
quantities. relevant values.
Free eBooks on @neetquestionpaper2020

648 Physi cs

Example 38. P R P Q
Find current through branch BD = also =
Q S R S
A 1W B 2W Note that when battery and galvanometer of a Wheatstone bridge
C
is interchanged, the balance position remains undisturbed, while
sensitivity of the bridge changes.
5V
15V In the balanced condition, the resistance in the branch BD may
3W be neglected
Example : Resistance connected to BC may be neglected.

D 2W
2W
Solution : A
Let VD = 0V B 2W C 2W D;
\ VA = +5V and VC = + 15 V
Let the voltage of B = VB 2W
Applying Kirchhoff’s junction law at B
5 - VB 15 - VB 0 - VB 2W 2W
+ + = 0 Þ VB = 6.82 Volt
1 2 3 2W
2W A B
6.82 - 0
Current through BD = = 2.27 A
3 2W
Example 39.
Calculate the currents I1, I2 and I3 in the circuit shown in
figure. In a Wheatstone bridge, the deflection in a
galvanometer does not change, if the battery and the
galvanometer are interchanged
Measuring temperature with the help of Wheatstone bridge
P Q
At balancing =
R + DR S(1 + aDT )

P Q
Solution :
Junction rule at C yields G
I1 + I2 – I3 = 0 i.e., I1 + I2 = I3 .... (1)
while loop for meshes a and b yields respectively : R
S

–14 – 4I2 + 6I1 – 10 = 0 DR


i.e., 3I1 – 2I2 = 12 .... (2)
When P = Q then DR = S a DT [Q R » S]
and, 10 – 6I1 – 2I3 = 0
i.e., 3I1 + I3 = 5 .... (3) DR
Substituting I3 from equation (1) in (3) \ DT =
Sa
4I1 + I2 = 5
Solving equations (2) and (4) for I1 and I2, we find COMMON DEFAULT
I1 = 2A and I2 = –3A O Incorrect If the current flows in a wire, there has to be a
And hence equation (1) yields, I3 = –1A potential difference. The potential drop takes
The fact that I2 and I3 are negative implies that actual place only when current passes through a
direction of I2 and I3 are opposite to that shown in the resistor.
circuit. 1
WHEATSTONE BRIDGE D
A r B r C r
The condition for balanced wheatstone bridge
B 2

P Q
P Correct In the diagram, the three resistors are in parallel.
A G C The potential at A is equal to the potential at C.
Current flows in wire 1 but there is no potnetial
drop across A and C.
EAB = (pAB )T - (pAB )To + s A (T - To ) -Rs B (T - To ) S
O Incorrect If potential difference between the points is zero,
D there is zero current between the two points.
P Coorect There is no p.d. between A and C still current
E flows in segment 1.
Free eBooks on @neetquestionpaper2020

Current Electricity 649


Example 40. In order to calculate potential difference between points A
Calculate the effective resistance between A and B in the and B, see fig.
following network.
3W 2V
5W 5W 5W
2W E C
D A
A B
C 7W 6W C F D
5W 5W B 5W
Half of the current goes to each part i.e., current in each
Solution : 4W
part is (2/15) amp. Consider the loop AEFDCA, we have
The circuit can be redrawn as
2 2 2
VAB = - ´ 5 + ´ 5 + ´ 5 = VA - VB
C 15 15 15

P=2W Q=3W 2 2 2 4
= ´ 5 = volt or VA = 2 - ´ 5 = volt ,
15 3 15 3
A 7W B 2 2 2
VB = 2 - ´ 10 = volt so VAB = VA - VB = volt
15 3 3
R=4W S=6W Example 42.
In the diagram below each resistance is of 1W. Find the
D equivalent resistance between A and B.

P R 2 C 1W D
Here = = so bridge is balanced
Q S 3
1W 1W 1W 1W
So the resistance between c and d is non useful.
Equivalent resistance = (P + Q) (R + S)
A 1W E 1W B
(P + Q)(R + S) (2 + 3)(4 + 6)
R eq = = Solution :
P+Q+R +S 2+3+4+6
Let a cell of e.m.f. E 1 be connected between A and B. The
5 ´ 10 10 currents through the various arms will be as shown in the
= = W
15 3 fig.
Even if not able to observe balanced wheatstone bridge try
to observe symmetry in network and use plane cutting C (I1–I2) 1W D
method. I2 I1
Example 41. 1W 1W 1W
Find the potential difference between the points A and B 1W
in fig.
5W 5W
A
I1
I
2
B
B
I
I–I) 1W
( 1
1
E (I–I ) 1W
1
I

5W + –2V 5W
E1

A According to Kirchoff’s second law, in closed circuit ACEA,


Solution : 5W 5W
0 = I1 ´1 + I 2 ´1 - (I - I1 )1 or I = 2 I1 + I 2 ...(i)
The upper three resistances of the cell are in series. Their
equivalent resistance is 15W. Similarly lower three In closed circuit CEDC;
resistances are in series. Their equivalent resistance is also 0 = I 2 ´1 + I 2 ´1 - (I1 - I 2 )1 or
3 I 2 = I1 ...(ii)
15W. The upper and lower equivalent resistances are
connected in parallel. So, resultant resistance of the circuit Putting this value in eqn. (i), we get
is given by I < 2 I1 ∗ I1 / 3 or I1 < (3 / 7)I
1 1 1 2 15 In a closed circuit E1 AE BE1,
= + = or R = W
R 15 15 15 2
E1 < (I , I1 )1 ∗ (I , I1)1 < 2(I , I1 ) = 2æç I - ö÷ =
3I 8I
2 4 è 7 ø 7
Current from the cell, i = = amp.
(15 / 2) 15 If R is the effective resistance between A and B, then E1 = I R
So, I R = 8 I /7 or R = 8/7 W
Free eBooks on @neetquestionpaper2020

650 Physi cs

18.5
Solve following problems with the help of above text and 3. In a Wheatstone bridge all the four arms have equal
examples. resistance R. If the resistance of galvanometer arm is also
1. Consider the following two statements. R, the equivalent resistance of combination is
(A) Kirchoff’s junction law follows from conservation of (a) 2R (b) R/4 (c) R/2 (d) R
charge. 4. Why is the Wheatstone bridge better than the other
(B) Kirchoff’s loop law follows from conservative nature methods of measuring resistances?
of electric field. (a) It does not involve Ohm’s law
(a) Both A and B are correct (b) It is based on Kirchoff’s law
(b) A is correct but B is wrong (c) It has four resistor arms
(d) It is a null method
(c) B is correct but A is wrong
5. If in the experiment of Wheatstone’s bridge, the positions
(d) Both A and B are wrong of cells and galvanometer are interchanged, then balance
2. Kirchoff’s first law, i.e., Ρ i = 0 at a junction, deals with point will
the conservation of (a) change
(a) charge (b) remain unchanged
(b) energy (c) depend on the internal resistance of cell and
(c) momentum resistance of galvanometer
(d) angular momentum (d) None of these

ANSWER KEY
1. (b) 2. (a) 3. (d) 4. (d) 5. (b)

METER BRIDGE OR SLIDE WIRE BRIDGE A cell whose emf has to be measured is also connected to the
Principle: Based on balanced Wheatstone bridge principle. potential wire in such a way that the positive terminal is connected
Use : To find unknown resistance with P and negative terminal is connected to a galvanometer and
then to a jockey (J) which is free to slide along the wire
Working : Let P be the unknown resistance.
B K Rh
P Q
A

l 100– l A C
P
IB J Q
G
IE
E
There is a potential drop along PQ.
The potential drop per unit length along PQ is called potential
At balance point gradient.
P Q When the jockey is pressed on some point, current flows from E
= to P (Þ). Also current that comes from B after reaching P divides
l 100 - l into two parts. One part moves towards A and the other towards
Q is known and l can be calculated. E (®). Three cases may arise.
POTENTIOMETER : (a) IB > IE. This happens when VPC > E. One side deflection in
galvanometer
Principle : The p.d. across a resistance wire is directly (b) IB = IE. This happens when VPC = E, Zero deflection in
proportional to its length provided I, r and A are constant. galvanometer
l (c) IB < IE . This happens when VPA < E. Other side defection
V = IR = I r in galvanometer
A
Þ V a l [ I, r and A are constant]
• At null point since no current flows through E therefore it is
Working : PQ is the resistance wire of potentiometer generally said to be in the condition of open circuit.
made up of constantan or nichrome. One end P is connected to • More is the length of potentiometer, higher is the sensitivity of
the positive terminal of the battery B while negative terminal is potentiometer and smaller is the potential gradient.
connected to Q through a Rheostat (Rh) and key (K). This is the • Potentiometer will work only when B > E. Also the positive
main circuit. terminal of the batteries is connected at P. If any of the above
conditions is not followed, we do not get a null point.
Free eBooks on @neetquestionpaper2020

Current Electricity 651

E1 l1 Total resistance between A and C


Uses : (i) Comparison of emfs of cells E = l R 0 é RR 0 R ù
2 2 R AC = R AB + =ê + 0ú
2 ë 2R + R 0 2 û
ælö
(ii)To find internal resistance of a cell r = ç ÷ R The current through the potentiometer wire
è l' ø
emf can be measured by potentiometer and not voltmeter. V
I=
Example 43. R AC
A 10 m long wire AB of uniform area of cross-section and The potential difference between A and B
20W resistance is used as a potentiometer wire. This wire V V RR 0
is connected in series with a battery of 5V and a resistor VAB = I R AB = ´ R AB = ´
R AC RR R (2 R + R 0 )
of 480W. An unknown emf is balanced at 600 cm of the 0 + 0
wire as shown in the figure. R + R0 2
5V 480 W V RR 0 2V RR 0
= ´ =
2RR 0 + R 0 ( 2R + R 0 ) ( 2R + R 0 ) R 0 [2R + 2 R + R 0 ]
2( 2R + R 0 )
600 cm 2VR
A J B VAB =
R 0 + 4R
Example 45.
G A uniform potential gradient is established across a
E potentiometer wire. Two cells of emf E1 and E2 connected
Calculate :
(i) The potential gradient for the potentiometer wire to support and oppose each other are balanced over
(ii) The value of unknown emf E l1 = 6m and l2 = 2m. Find E1/E2.
Solution : Solution :
(i) V = 5V E1 + E2 = xl1 = 6x and E1 – E2 = 2x
480 W I
E1 + E 2 6 E1 2
= or =
I E1 - E 2 2 E2 1
Example 46.
A B In a practical wheatstone bridge circuit as shown, when
Applying Ohm’s law one more resistance of 100 W is connected is parallel with
V = I (RAB + 480) unknown resistance ‘x’, then ratio l1 / l 2 become ‘2’. l1
5 is balance length. AB is a uniform wire. Find the value of'
5 = I (20 + 480) [RAB = 20 W (given)] Þ I = = 0.01A
500 x'.
Potential gradient of potentiometer wire Copper strip
V 0.2
< AB < < 0.02Vm,1 100 x
l AB 10
(ii) With reference to the current given in question, the emf E
and the potential drop across AJ should be equal for the G Copper
balancing : E = (potential gradient of potentiometer wire) A B strip
× balancing length = 0.02 × 6 = 0.12 V 1 2
Example 44.
A resistance of RW is powered from a potentiometer of
resistance R 0 W . A voltage V is supplied to the r
E
potentiometer. Derive an expression for the voltage fed Sol. Q Wheatstone bridge is in balanced condition
into the circuit when the slide is in the middle of the
100
potentiometer.
Solution : V 100 x

RC
A
B C

1 2
R
Total resistance between A and B 100x
R0 100 100 + x l1
≥R so = and Q l = 2
RR0
RAB < 2 < l1 l2 2
R0 2R ∗ R0 Þ x = 100 W
R∗
2
Free eBooks on @neetquestionpaper2020

652 Physi cs

18.6
Solve following problems with the help of above text and 2. In the experiment of potentiometer, at balance point, there
examples. is no current in the
1. In potentiometer a balance point is obtained, when (a) main circuit
(a) the e.m.f. of the battery becomes equal to the e.m.f of (b) galvanometer circuit
the experimental cell (c) potentiometer circuit
(b) the p.d. of the wire between the +ve end of battery (d) both main and galvanometer circuits
to jockey becomes equal to the e.m.f. of the 3. Sensitivity of potentiometer can be increased by
experimental cell (a) increasing the e.m.f of the cell
(c) the p.d. of the wire between +ve point of cell and jockey (b) increasing the length of the potentiometer
becomes equal to the e.m.f. of the battery (c) decreasing the length of the potentiometer wire
(d) the p.d. across the potentiometer wire becomes equal (d) None of these
to the e.m.f. of the battery
ANSWER KEY
1. (b) 2. (b) 3. (b)

MEASURING INSTRUMENTS For this, we connect a small resistance S (called shunt) in


Galvanometer parallel with the galvanometer.
It is an instrument used to detect small currents in a circuit. Mathematically, Ig × G = (I – Ig) S
The current required for full scale deflection in the galvanometer where I is the maximum current which ammeter can measure. G is
is called full scale deflection current and is denoted by Ig. the resistance of galvanometer and Ig is the current of full scale
deflection in galvanometer. S is shunt.
Current sensitivity of a galvanometer.
The resistance of the ammeter will be
It is defined as the deflection produced in the galvanometer,
when unit current flows through it. 1 1 1 G´S
= + RA =
RA G S G+S
q NBA
Current sensitivity, IS = = and its unit is rad A–1
I C Since shunt is a small resistance. Therefore the resistance of
ammeter is very small.
Current sensitivity can be increased either by decreasing C i.e.
The above arrangement is made so that when we connect
restoring torque per unit twist or increasing B.
ammeter in series to measure current, it does not change the
Voltage sensitivity of a galvanometer original current to a large extent. The change is infact very small.
It is defined as the deflection produced in the galvanometer when Also since galvanometer is a sensitive device and cannot take
a unit voltage is applied across the two terminals of the large currents, this arrangement serves the purpose. Most of
galvanometer. the current entering the ammeter passes through the shunt as
current always prefer low resistance path.
q q NBA
Voltage sensitivity VS = = = , its unit is rad V–1 An ideal ammeter is one which has zero resistance.
V IR CR The range of ammeter can be increased but cannot be decreased
Ammeter below Ig.
Ammeter is used to measure current in a circuit. Ammeter is Voltmeter
always connected in series in the circuit as shown. Voltmeter is used to measure potential difference across a
resistor. Voltmeter is always connected in parallel across a
I
A resistor.
R ext Conversion of a galvanometer into a voltmeter
Conversion of a galvanometer into an ammeter
Voltmeter (V)
V
A G
Ig R
Ig
G
I I
I a Rext b
I – Ig S V
Free eBooks on @neetquestionpaper2020

Current Electricity 653


For this, we connect a large resistance R in series with the
galvanometer. A B
J'
The potential difference which has to be measured is across the
external resistance i.e. across points a and b.
M Q
Let it be V. Then P
2V 10 W
V = Ig (G + R)
IQ I
where V is the maximum potential difference that the voltmeter
can measure and R is the large resistance connected in series V
with the galvanometer R S
Rv
The resistance of the voltmeter will be RV = G + R The potential drop across PQ
Since R is a large resistance. Therefore resistance of VPQ = (potential gradient for potential wire) × lAJ'
voltmeter is very large.
An ideal voltmeter is one which has infinite resistance. 2.4
= ´ 4.9 = 1.96V
The range of voltmeter can be increased and decreased. 6
Thus the reading shown by voltmeter is 1.96 V.
When ammeter/voltmeter is connected in the circuit,
(iii) Since the potential drop across PQ is 1.96, therefore the
the current or voltage indicated by these is less than the actual
potential drop across MQ should be (2 – 1.96)
values in their absence.
= 0.04 volt.
Example 47. Current I through the resistor of 10ς
The length of a potentiometer wire is 600 cm and it carries VMQ 0.04
a current of 40 mA. For a cell of emf 2V and internal < < < 0.004A
R MQ 10
resistance 10 ς , the null point is found to be at 500 cm. If
a voltmeter is connected across the cell, the balancing Applying Ohm’s law to PQSRP,
length is decreased by 10 cm. Find : 2 = 0.004 ( 10 + RV) Þ RV = 490 W.
(i) The resistance of whole wire Example 48.
(ii) Reading of voltmeter A galvanometer of coil resistance 20 ohm, gives a full
(iii) Resistance of voltmeter scale deflection with a current of 5 mA. What arrangements
Solution : should be made in order to measure currents upto 1.0 A ?
(i) Potential gradient of the potentiometer wire Solution :
The upper limiting value of current to be measured is to be
VAB V increased by a factor
= =
lAB 6
1.0A
n= = 200
600 cm 5A
0.04A J
\ Resistance of the shunt required will be
A 500 cm B
G 20W
S= = » 0.1 W
2 V 10W n - 1 200 - 1

The balancing length of the cell is obtained at 500cm. Hence, a shunt of resistance 0.1 W should be connected
Therefore in parallel across the galvanometer coil.
VAJ = (Potential gradient of potentiometer wire) × l AJ Example 49.
A voltmeter having 100 W resistance can measure a
V 12
´5 = 2 Þ V = = 2.4 volt potential difference of 25V. What resistance R is required
6 5 to be connected in series, to make it read voltage upto
Applying Ohm’s law 250 V ?
V 2. 4 Solution :
VAB = I R AB Þ R AB = AB = = 60W The upper limiting value of voltage is to be increased by a
I 0.04
factor
(ii) When a voltmeter is connected across the cell then the
balancing point shifts 10 cm left. This is because a current 250V
n= = 10
now flows in the loop PRSQP and hence the potential drop 25V
across PQ decreases. Here IAP =IQJ'=0 \ Resistance, R = (n – 1) G = (10 – 1) 100 = 900 W
Free eBooks on @neetquestionpaper2020

654 Physi cs

Keep in Memory Solution :


1. In order to increase the range of a voltmeter n times, its æ S ö
(a) We know that i g = ç ÷i
total resistance should also be increased by n times. So èS+G ø
the resistance to be connected in series is Substituting the given values, we get
R = (n – 1) G.
2. In order to increase the range of an ammeter n times, the æ S ö
2 ´10 -3 = ç ÷ ´ 0.3
value of shunt resistance to be connected in parallel is è S + 30 ø
S = G / (n – 1) Solving it, we get S = (30/149) W
3. For galvanometers with their coil in uniform magnetic field B So, a resistance of (30/149) W must be connected in
Cq parallel with the galvanometer.
I=
NAB sin a æ V ö V
(b) In this case, i g = ç ÷ or R = - G
C = torsional rigidity of suspension wire. èR+Gø ig
For galvanometer with concave pole-pieces – radial
magnetic field is produced, a = 90° 0.2
\ R= - 30 = 100 - 30 = 70 W
I = (C / NAB)q or I = Kq 2 ´10 -3
All galvanometers used in practice have concave So, a resistance of 70 W must be connected in series
pole-pieces, for making the radial magnetic field. with the galvanometer. 654
Example 50. Example 53.
A galvanometer has a resistance G and a current Ig flowing A voltmeter has a resistance of G ohm and range V volt.
in it produces full scale deflection. S1 is the value of the What will be the value of resistance used in series to
shunt which converts it into an ammeter of range 0 to I convert it into voltmeter of range nV volt?
and S2 is the value of the shunt for the range 0 to 2I. Solution :
S V
Determine the ratio 1 . We know that, R = -G
S2 Ιg
Solution :
The voltmeter gives full scale deflection for potential
ΙgRg ΙgRg S æ 2Ι - Ι g ö
S1 < ; S2 < ; so 1 = ç ÷ difference V. Its resistance is G
Ι , Ιg 2Ι , Ιg S2 çè Ι - Ι g ÷ø
Hence Ιg = (V / G)
Example 51.
A galvanometer together with an unknown resistance in nV
series is connected across two identical batteries each of \ R= - G = n G - G = (n - 1) G
(V / G)
1.5 V. When the batteries are connected in series, the
galvanometer records a current of 1 A, and when the Example 54.
batteries are in parallel, the current is 0.6 A. What is the In the circuit shown in fig., Find the reading of voltmeter.
internal resistance of battery? + –
Solution :
2V,r = 0
Let R = combined resistance of galvanometer and unknown
80W
resistance of the circuit, and V
r = internal resistance of each battery
20W 80W
2E 3
In first case, current = =1 or =1 Solution :
R + 2r R + 2r
or, R + 2r = 3 .... (1) The equivalent resistance of the circuit is given by
E 2E 80 ´ 80
In second case, current = = R eq. = 20 + = 20 + 40 = 60W
R + r / 2 2R + r 80 ´ 80
3
or, 0.6 = or, 2R + r = 5 .... (2) V
2R + r Current through the circuit I = =(2/60) = (1/30) amp
Solving eqns. (1) and (2), we get r = (1/3) W R
Example 52. Reading of voltmeter
A galvanometer has a resistance of 30 W and current of
2 mA is needed to give full scale deflection. What is the = current × RT
resistance needed and how is it to be connected to convert
1 æ 80 ´ 80 ö 40
the galvanometer = ´ç ÷= = 1.33 V
(a) into an ammeter of 0.3 ampere range 30 è 80 + 80 ø 30
(b) into voltmeter of 0.2 volt range
Free eBooks on @neetquestionpaper2020

Current Electricity 655


Example 55. This is the voltage to be measured. When a voltmeter of
A battery of e.m.f E is connected with three resistances R, resistance 10 R is connected in parallel with 2 R, their
2R and 3 R in series as shown in fig. The voltage across 2R 2 R ´10 R 5
is measured with a voltmeter whose resistance is 10R. What effective resistance R p = 2 R + 10 R = 3 R.
is the percentage error?
Now, total resistance of the circuit
V 5 17
= R + R + 3R = R
3 3
E 3E
R 2R 3R Current in the circuit Ι1 = =
(17 / 3)R 17R
3E 5 5
E \ Voltage across Rp, V1 < I1, Rp < ≥ R< E
Solution : 17 R 3 17
Total resistance of the circuit before connecting the This will be the reading of voltmeter.
voltmeter = R + 2 R + 3R = 6 R. V - V1 æ V ö
Hence the current, I = E/6 R. Percentage error = ´ 100 = ç1 - 1 ÷ ´ 100
V è Vø
Voltage across the resistance 2R is,
é (5 / 17) E ù
E E = ê1 - ´ 100 = 11.76%
V = Ι´2R = ´2R = ë E / 3 úû
6R 3

18.7
Solve following problems with the help of above text and 4. To reduce the range of voltmeter, its resistance is to be
examples. reduced. A voltmeter has a resistance G and range V. What
1. Select the correct option(s). resistance connected in parallel will convert it into a
(a) An ammeter should have small resistance voltmeter of range V/n?
(b) An ammeter should have large resistance (a) nG (b) (n + 1) G
(c) A voltmeter should have small resistance (c) (n – 1) G (d) None of these
(d) A voltmeter should have large resistance 5. The sensitivity of a galvanometer does not depend upon
2. The net resistance of an ammeter should be small to ensure (a) a very strong magnetic field in the permanent magnet
that (b) the current it measures
(a) it does not get overheated (c) a very thin, weak suspension
(b) it does not draw excessive current (d) a large number of turns in the coil
(c) it can measure large currents 6. Of the following an ideal voltmeter is
(d) it does not appreciably change the current to be (a) moving coil voltmeter
measured (b) voltameter
3. The net resistance of a voltmeter should be large to ensure (c) an electrometer
that (d) hot wire voltmeter
(a) it does not get overheated 7. The sensitivity of a galvanometer will not increase if
(b) it does not draw excessive current (a) radius of coil is increased
(c) it can measure large potential differences (b) number of turns in coil is decreased
(d) it does not appreciably change the potential (c) number of turns in coil is increased
difference to be measured (d) a strong magnetic field is used

ANSWER KEY
1. (a,d) 2. (d) 3. (d) 4. (d) 5. (b) 6. (c) 7. (b)
Free eBooks on @neetquestionpaper2020

656 Physi cs

Very Short / Short Answer Questions 14. In the circuit diagram given E1 E2
below, the cells E1 and E2 have
1. A wire of resistivity r is stretched to double its length. e.m.f’s of 4V and 8V and
What will be its new resistivity? internal resistance 0.5 W and I 4.5 W I1 3 W
2. Why is a voltmeter always connected in parallel with a 1 W respectively. Calculate I2 6 W
circuit element across which voltage is to be measured? the current in each resistance.
3. What happens to the power dissipation if the value of 15. The network PQRS, shown in the circuit diagram, has the
electric current passing through a conductor of constant batteries of 8V and 4V and negligible internal resistance.
resistance is doubled? A milliammeter of 40W resistance is connected between P
4. What is the largest voltage that you can safely put across and R. Calculate the reading in the milliammeter.
[Outside Delhi - 2012 COMPTT.].
a resistor marked 196 W-1 W?
T1 100 W
5. V-I graph for a metallic wire at S R
I
two different temperature T1 and
T2
T2 is as shown in the following
8V A 80 W
figure. Which of the two m
W
temperature is higher and why? V 40
6. Two cells of e.m.f E1 and E2 (E1 > E2) are connected as
P Q
shown in the figure.
A B C 4V
16. In the given circuit, assuming point A to be at zero
E1 E2 potential, use Kirchhoff’s rules to determine the potential
at point B. [Outside Delhi - 2011 COMPTT.].
When a potentiometer is connected between A and B, the
balancing length of the potentionmeter wire is 300 cm. On 1A D 4A 6V
B
connecting the same potentiometer between A and C, the
balancing length is 100 cm. Calculate the ratio of E1 and R 2W R1
E2.
7. Derive an expression for the current obtained for n cells
connected in series. Hence derive the condition for A 3V C 4A
maximum current. 17. Calculate the electrical conductivity of the material of a
8. Derive the balanced condition of wheatstone’s bridge conductor of length 3m, area of cross section 0.02 mm2
principle. having a resistance of 20 ohm.
9. Show on a graph, the variation of resistivity with Multiple Choice Questions
temperature for a typical semiconductor.
[Delhi Board - 2012] 18. The emf developed by a thermocouple is measured with
the help of a potentiometer and not by a moving coil
10. A resistance R is connected across a cell of emf e
millivoltmeter because
and internal resistance r. A potentiometer now measures (a) the potentiometer is more accurate than the voltmeter
the potential difference between the terminal of the cell (b) the potentiometer is more sensitive than voltmeter
as V. Write the expression for ‘r’ in terms of e, V and R. (c) the potentiometer makes measurement without
[Delhi Board - 2011] drawing any current from the thermocouple
11. Define the term ‘wattless current’. [Delhi Board - 2011] (d) measurement using a potentiometer is simpler than
12. The sequence of coloured bands in two carbon resistors with a voltmeter
R1 and R2 is [Outside Delhi - 2010 COMPTT.] 19. You have been provided with four 100 ohm resistors each
(i) brown, green, blue and with a tolerance of 2%. The number of ways in which these
(ii) orange, black, green can be combined to have different equivalent resistances
Find the ratio of their resistances is
(a) seven different combinations and seven different
Long Answer Questions equivalents
(b) eight different combinations and seven different
13. What is the emf of a cell ? State the factors on which equivalents resistances
its value depends. Derive a relation between emf E, (c) nine different combinations and eight different
contact potential V, internal resistance r of a cell and resistances
external resistance R. Proove that emf is more than (d) ten different combinations and nine different
potential difference. resistances
Free eBooks on @neetquestionpaper2020

Current Electricity 657


20. The equivalent resistance between points A and B is 22. The resistance of a resistor with the following colour code
R R
(a) 2R A B
(b) (3/4) R R
R
(c) (4/3) R Red Violet Green Gold
is equal to
(d) (3/5) R (a) 26 × 104 W ± 5% (b) 25 × 104 W ± 10%
21. In the house of a person who is weak of hearing, a light (c) 35 × 105 W ± 5% (d) 27 × 105 W ± 5%
23. Two cells of the same emf E have different internal
bulb is also lit when somebody rings the door bell. The ring resistances r1 and r2. They are connected in series with an
can be operated both from the garden gate and from the external resistance R and the potential difference across
door of the house. Select the correct possible circuit the first cell is found to be zero. Therefore, the external
required. resistance R must be
(a) r1 – r2 (b) r + r2
(c) 2r1 – r 2 (d) r1 – 2r2
24. Three resistances R, 2R and 3R are connected in parallel to
(a) (b) a battery. Then
(a) the potential drop across 3R is maximum
(b) the current through each resistance is same
(c) the heat developed in 3R is maximum
(d) the heat developed in R is maximum.
25. A piece of copper and another of germanium are cooled
from room temperature to 80° K. The resistance of
(c) (d) (a) each of them increases
(b) each of them decreases
(c) copper increases and germanium decreases
(d) copper decreases and germanium increases

1. A heating coil is labelled 100 W, 220 V. The coil is cut in half (c) both bulbs will fuse
and the two pieces are joined in parallel to the same source. (d) None of these
The energy now liberated per second is 5. An electric kettle has two heating coils. When one of the
[CBSE PMT 1995] coils is connected to an a.c. source, the water in the kettle
(a) 25 J (b) 50 J (c) 200 J (d) 400 J boils in 10 minutes. When the other coil is used the water
2. Five resistances have been connected as shown in the boils in 40 minutes. If both the coils are connected in parallel,
figure. The effective resistance between A & B is the time taken by the same quantity of water to boil will be
[CBSE PMT 2000] [CBSE PMT 2003]
(a) 15 min (b) 8 min (c) 4 min (d) 25 min
3W 4W 6. Two 220 volt, 100 watt bulbs are connected first in series
and then in parallel. Each time the combination is connected
A 7W B
to a 220 volt a.c. supply line. The power drawn by the
6W 8W combination in each case respectively will be
(a) 14/3W (b) 20/3W [CBSE PMT 2003]
(a) 50 watt, 200 watt (b) 50 watt, 100 watt
(c) 14W (d) 21W
(c) 100 watt, 50 watt (d) 200 watt, 150 watt
3. If specific resistance of a potentiometer wire is 10–7Wm 7. The electric resistance of a certain wire of iron is R. If its
current flowing through it, is 0.1 amp and cross sectional length and radius are both doubled, then
area of wire is 10–6 m2, then potential gradient will be [CBSE PMT 2004]
[CBSE PMT 2001] (a) the resistance and the specific resistance, will both
(a) 10–2 volt/m (b) 10–4 volt/m remain unchanged
(c) 10–6 volt/m (d) 10–8 volt/m (b) the resistance will be doubled and the specific
4. If 25W, 220 V and 100 W, 220 V bulbs are connected in resistance will be halved
series across a 440 V line, then [CBSE PMT 2001] (c) the resistance will be halved and the specific
(a) only 25W bulb will fuse resistance will remain unchanged
(b) only 100W bulb will fuse (d) the resistance will be halved and the specific
resistance will be doubled
Free eBooks on @neetquestionpaper2020

658 Physi cs
8. Five equal resistances each of resistance R are connected 16. The power dissipated in the circuit shown in the figure is 30
as shown in the figure. A battery of V volts is connected Watts. The value of R is [CBSE-PMT 2012]
between A and B. The current flowing in AFCEB will be
R
[CBSE PMT 2004] (a) 20 W
C
2V 3V (b) 15 W
(a) (b) 5W
R R R (c) 10 W
R R
F
A (d) 30 W 10V
V V R
(c) (d) B 17. Cell having an emf e and internal resistance r is connected
R 2R D E
across a variable external resistance R. As the resistance R
R
9. A battery is charged at a potential of 15V for 8 hours when is increased, the plot of potential difference V across R is
the current flowing is 10A. The battery on discharge given by [CBSE-PMT 2012]
supplies a current of 5A for 15 hour. The mean terminal (a) (b)
voltage during discharge is 14V. The “Watt-hour” e e
efficiency of the battery is [CBSE PMT 2004] V V
(a) 87.5% (b) 82.5% (c) 80% (d) 90%
10. When three identical bulbs of 60 watt, 200 volt rating are
connected in series to a 200 volt supply, the power drawn 0 0
R R
by them will be [CBSE PMT 2004] (c) (d)
(a) 20 watt (b) 60 watt (c) 180 watt (d) 10 watt e V
11. In India electricity is supplied for domestic use at 220 V. It V
is supplied at 110 V in USA. If the resistance of a 60 W
bulb for use in India is R, the resistance of a 60 W bulb for
use in USA will be [CBSE PMT 2004] 0 0
(a) R/2 (b) R (c) 2R (d) R/4 R R
12. For the network shown in the figure the value of the current 18. A milli voltmeter of 25 milli volt range is to be converted
i is [CBSE PMT 2005] into an ammeter of 25 ampere range. The value (in ohm) of
9V
necessary shunt will be [CBSE-PMT 2012]
(a) (a) 0.001 (b) 0.01
35
(c) 1 (d) 0.05
18V
(b) 19. In the circuit shown the cells A and B have negligible
5
resistances. For VA = 12V, R1 = 500W and R = 100W the
5V galvanometer (G) shows no deflection. The value of VB is
(c)
9 [CBSE-PMT 2012]
5V
(d) R1
18
13. A 5–ampere fuse wire can withstand a maximum power G
of 1 watt in the circuit. The resistance of the fuse wire is
[CBSE PMT 2005]
(a) 0.04 W (b) 0.2 W (c) 5 W (d) 0.4 W VA R VB

14. In producing chlorine by electrolysis 100 kW power at 125


V is being consumed. How much chlorine per minute is
(a) 4 V (b) 2 V
liberated? (E.C.E. of chlorine is 0.367×10–6 kg / C)
[CBSE-PMT 2010] (c) 12 V (d) 6 V
(a) 1.76 × 10–3 kg (b) 9.67 × 10–3 kg 20. If voltage across a bulb rated 220 Volt-100 Watt drops by
(c) 17.61 × 10–3 kg (d) 3.67 × 10–3 kg 2.5% of its rated value, the percentage of the rated value by
15. In the circuit shown in the figure, if potential at point A is which the power would decrease is [CBSE-PMT 2012]
taken to be zero, the potential at point B is (a) 20% (b) 2.5%
[CBSE-PMT 2011M] (c) 5% (d) 10%
R1 D 2V 21. A wire of resistance 4 W is stretched to twice its original
1A B length. The resistance of stretched wire would be
[NEET 2013]
R2
2A (a) 4 W (b) 8 W
(c) 16 W (d) 2 W
22. The internal resistance of a 2.1 V cell which gives a current
A of 0.2 A through a resistance of 10 W is [NEET 2013]
1V 1A C 2A
(a) –1V (b) + 2V (a) 0.5 W (b) 0.8 W
(c) –2V (d) + 1V (c) 1.0 W (d) 0.2 W
Free eBooks on @neetquestionpaper2020

Current Electricity 659


23. The resistance of the four arms P, Q, R and S in a 30. The negative Zn pole of a Daniell cell, sending a constant
Wheatstone’s bridge are 10 ohm, 30 ohm, 30 ohm and 90 current through a circuit, decreases in mass by 0.13g in 30
ohm, respectively. The e.m.f. and internal resistance of the minutes. If the electeochemical equivalent of Zn and Cu
cell are 7 volt and 5 ohm respectively. If the galvanometer are 32.5 and 31.5 respectively, the increase in the mass of
resistance is 50 ohm, the current drawn from the cell will be the positive Cu pole in this time is [AIEEE 2003]
[NEET 2013] (a) 0.180 g (b) 0.141g (c) 0.126 g (d) 0.242 g
(a) 0.2 A (b) 0.1 A 31. A 220 volt, 1000 watt bulb is connected across a 110 volt
(c) 2. 0 A (d) 1. 0 A mains supply . The power consumed will be
24. A wire when connected to 220 V mains supply has power [AIEEE 2003]
dissipation P1. Now the wire is cut into two equal pieces (a) 750 watt (b) 500 watt
which are connected in parallel to the same supply. Power (c) 250 watt (d) 1000 watt
dissipation in this case is P2. Then P2 : P1 is [AIEEE 2002] 32. The total current supplied to the circuit by the battery is
(a) 1 (b) 4 (c) 2 (d) 3 [AIEEE 2004]
25. If in the circuit, power dissipation is 150 W, then R is
[AIEEE 2002] 2W
6V
R 6W 3W

2W 1.5W

15 V
(a) 2 W (b) 6 W (c) 5 W (d) 4 W (a) 4 A (b) 2 A (c) 1 A (d) 6 A
26. The mass of product liberated on anode in an 33. The resistance of the series combination of two resistances
electrochemical cell depends on [AIEEE 2002] is S. When they are joined in parallel the total resistance is
(a) (It)1/2 (b) It (c) I/t (d) I2 t P. If S = n P then the minimum possible value of n is
(where t is the time period for which the current is passed). [AIEEE 2004]
27. If qi, is the inversion temperature, qn is the neutral (a) 2 (b) 3 (c) 4 (d) 1
temperature, qc is the temperature of the cold junction, then 34. An electric current is passed through a circuit containing
[AIEEE 2002] two wires of the same material, connected in parallel. If the
(a) qi + qc = q n (b) qi - q c = 2q n 4 2
lengths and radii are in the ratio of and , then the
qi + qC 3 3
(c) = qn (d) qc - qi = 2q n ratio of the current passing through the wires will be
2
28. The length of a wire of a potentiometer is 100 cm, and the [AIEEE 2004]
e.m.f. of its standard cell is E volt. It is employed to (a) 8/9 (b) 1/3
measure the e.m.f. of a battery whose internal resistance (c) 3 (d) 2
is 0.5W. If the balance point is obtained at l = 30 cm from 35. In a meter bridge experiment null point is obtained at 20 cm,
the positive end, the e.m.f. of the battery is from one end of the wire when resistance X is balanced
against another resistance Y. If X < Y, then where will be
30 E the new position of the null point from the same end, if one
(a) [AIEEE 2003]
(100 - 0.5) decides to balance a resistance of 4 X against Y?
[AIEEE 2004]
30 ( E - 0.5 i ) (a) 40 cm (b) 80 cm (c) 50 cm (d) 70 cm
(b) , where i is the current in the
100 36. Time taken by a 836 W heater to heat one litre of water from
potentiometer wire. 10°C to 40°C is [AIEEE 2004]
(a) 150 s (b) 100 s (c) 50 s (d) 200 s
30 E 30 E
(c) (d) 37. The thermo emf of a thermocouple varies with the
100 100.5
29. A 3 volt battery with negligible internal resistance is temperature q of the hot junction as E = aθ + bθ 2 in volts
connected in a circuit as shown in the figure. The current I, where the ratio a/b is 700°C. If the cold junction is kept at
in the circuit will be [AIEEE 2003] 0°C, then the neutral temperature is [AIEEE 2004]
(a) 1400°C (b) 350°C (c) 700°C
(d) no neutral temperature is possible for this
3W
thermocouple.
3W 38. The electrochemical equivalent of a metal is 3.3 × 10–7 kg per
3V
coulomb. The mass of the metal liberated at the cathode
when a 3A current is passed for 2 seconds will be
[AIEEE 2004]
3W (a) 6.6×1057kg (b) 9.9×10–7 kg
(a) 1 A (b) 1.5 A (c) 2 A (d) 1/3 A (c) 19.8×10–7 kg (d) 1.1×10–7 kg
Free eBooks on @neetquestionpaper2020

660 Physi cs
39. Two sources of equal emf are connected to an external 47. The current I drawn from the 5 volt source will be
resistance R. The internal resistance of the two sources are [AIEEE 2006]
R1 and R2 (R2 > R1). If the potential difference across the
source having internal resistance R2 is zero, then 10W
[AIEEE 2005]
5W 10W 20W
(a) R= R 2 - R 1
(b) R= R 2 ´ (R1 + R 2 ) /(R 2 - R1 )
I 10W
(c) R= R 1R 2 /( R 2 - R 1 )
+–
(d) R= R 1R 2 /( R 1 - R 2 )
5 volt
40. An energy source will supply a constant current into the
(a) 0.33 A (b) 0.5 A (c) 0.67 A (d) 0.17 A
load if its internal resistance is [AIEEE 2005]
48. The resistance of a bulb filmanet is 100W at a temperature
(a) very large as compared to the load resistance
of 100°C. If its temperature coefficient of resistance be 0.005
(b) equal to the resistance of the load
per °C, its resistance will become 200 W at a temperature of
(c) non-zero but less than the resistance of the load
[AIEEE 2006]
(d) zero
(a) 300°C (b) 400°C (c) 500°C (d) 200°C
41. A heater coil is cut into two equal parts and only one part
49. In a Wheatstone's bridge, three resistances P, Q and R
is now used in the heater. The heat generated will now be
connected in the three arms and the fourth arm is formed by
[AIEEE 2005]
two resistances S1 and S2 connected in parallel. The
(a) four times (b) doubled
condition for the bridge to be balanced will be [AIEEE 2006]
(c) halved (d) one fourth
42. Two voltameters, one of copper and another of silver, are P 2R P R (S1 + S2 )
(a) = (b) =
joined in parallel. When a total charge q flows through the Q S1 + S2 Q S1S 2
voltameters, equal amount of metals are deposited. If the
electrochemical equivalents of copper and silver are z1 P R (S1 + S2 ) P R
(c) = (d) =
and z2 respectively the charge which flows through the Q 2S1S 2 Q S1 + S2
silver voltameter is [AIEEE 2005]
50. The resistance of a wire is 5 ohm at 50°C and 6 ohm at
q q z z 100°C. The resistance of the wire at 0°C will be
(a) (b) (c) q 2 (d) q 1
z z z1 z2 [AIEEE 2007]
1+ 2 1+ 1 (a) 3 ohm (b) 2 ohm
z1 z2
(c) 1 ohm (d) 4 ohm
43. The resistance of hot tungsten filament is about 10 times 51. Shown in the figure below is a meter-bridge set up with null
the cold resistance. What will be the resistance of 100 W deflection in the galvanometer. [AIEEE 2008]
and 200 V lamp when not in use ? [AIEEE 2005] 55W R
(a) 20W (b) 40W (c) 200W (d) 400W
44. A thermocouple is made from two metals, Antimony and
Bismuth. If one junction of the couple is kept hot and the G
other is kept cold, then, an electric current will
[AIEEE 2005] 20 cm
(a) flow from Antimony to Bismuth at the hot junction
(b) flow from Bismuth to Antimony at the cold junction
(c) now flow through the thermocouple
(d) flow from Antimony to Bismuth at the cold junction The value of the unknown resistor R is
45. The Kirchhoff's first law (Si = 0) and second law (a) 13.75W (b) 220 W
(SiR = SE), where the symbols have their usual meanings, (c) 110 W (d) 55 W
are respectively based on [AIEEE 2006] Directions for Qs. (52 to 54) : Read the following passage
(a) conservation of charge, conservation of momentum carefully and answer the questions that follows:
(b) conservation of energy, conservation of charge PASSAGE
(c) conservation of momentum, conservation of charge Consider a block of conducting material of resistivity ‘r’ shown
(d) conservation of charge, conservation of energy in the figure. Current ‘I’ enters at ‘A’ and leaves from ‘D’. We
46. A material 'B' has twice the specific resistance of 'A'. A apply superposition principle to find voltage ‘DV’ developed
circular wire made of 'B' has twice the diameter of a wire between ‘B’ and ‘C’. The calculation is done in the following
made of 'A'. then for the two wires to have the same steps:
resistance, the ratio lB/lA of their respective length must be (i) Take current ‘I’ entering from ‘A’ and assume it to spread
[AIEEE 2006] over a hemispherical surface in the block.
1 1 (ii) Calculate field E(r) at distance ‘r’ from A by using Ohm’s
(a) 1 (b) (c) (d) 2 law E = r j, where j is the current per unit area at ‘r’.
2 4
Free eBooks on @neetquestionpaper2020

Current Electricity 661


(iii) From the ‘r’ dependence of E(r), obtain the potential V(r) at 57. Two electric bulbs marked 25W – 220 V and 100W – 220V
r. are connected in series to a 440 V supply. Which of the
(iv) Repeat (i), (ii) and (iii) for current ‘I’ leaving ‘D’ and bulbs will fuse? [AIEEE 2012]
superpose results for ‘A’ and ‘D’. (a) Both (b) 100 W
I
(c) 25 W (d) Neither
I DV
58. The supply voltage to room is 120V. The resistance of the
lead wires is 6W. A 60 W bulb is already switched on. What
a b a is the decrease of voltage across the bulb, when a 240 W
A B C D heater is switched on in parallel to the bulb?
[JEE Main 2013]
(a) zero (b) 2.9 Volt
(c) 13.3 Volt (d) 10.04 Volt
59. This questions has Statement I and Statement II. Of the
52. DV measured between B and C is [AIEEE 2008]
four choices given after the Statements, choose the one
rI rI rI rI that best describes the two Statements. [JEE Main 2013]
(a) – (b) –
pa p(a + b) a (a + b) Statement-I : Higher the range, greater is the resistance of
ammeter.
rI rI rI
(c) – (d) 2p(a - b) Statement-II : To increase the range of ammeter, additional
2pa 2p(a + b) shunt needs to be used across it.
53. For current entering at A, the electric field at a distance ‘r’ (a) Statement-I is true, Statement-II is true, Statement-II
from A is [AIEEE 2008] is the correct explanation of Statement-I.
rI rI (b) Statement-I is true, Statement-II is true, Statement-II
(a) 2 (b) is not the correct explanation of Statement-I.
8pr r2
(c) Statement-I is true, Statement-II is false.
rI rI (d) Statement-I is false, Statement-II is true.
(c) 2 (d)
2pr 4pr 2 60. A constant voltage is applied between the two ends of a
54. A 5V battery with internal resistance 2W and a uniform metallic wire. Some heat is developed in it. The
2V battery with internal resistance 1W are connected to a heat developed is double if [IIT JEE 1980]
10W resistor as shown in the figure. [AIEEE 2008] (a) both the length and radius of wire are halved
P2
(b) both length and radius of wire are doubled
(c) the radius of wire is doubled
(d) the length of the wire is doubled
61. In the circuit shown in fig. the heat produced in the 5 ohm
resistor due to the current flowing through it is 10 calories
5V 2V
2W 10W 1W per second. The heat generated in the 4 ohms resistor is
[IIT JEE 1981]
4W 6W

The current in the 10W resistor is


(a) 0.27 A P2 to P1 (b) 0.03 A P1 to P2 5W
(c) 0.03 A P2 to P1 (d) 0.27 A P1 to P2 (a) 1 calorie / sec (b) 2 calories /sec
55. Two conductors have the same resistance at 0°C but their (c) 3 calories /sec (d) 4 calories /sec
temperature coefficients of resistance are a1 and a 2 . The 62. A battery of internal resistance 4W is connected to the
respective temperature coefficients of their series and parallel network of resistances as shown. In order that the maximum
combinations are nearly [AIEEE 2010] power can be delivered to the network, the value of R in W
should be [IIT JEE 1995S]
a1 + a 2 a1 + a 2
(a) , a1 + a 2 (b) a1 + a 2 ,
2 2 R R

a1a 2 a1 + a 2 a1 + a 2 E R 6R R
(c) a1 + a 2 , ,
(d)
a1 + a 2 2 2 4W 4R
R
56. If a wire is stretched to make it 0.1% longer, its resistance
will [AIEEE 2011]
(a) increase by 0.2% (b) decrease by 0.2% 4 8
(c) decrease by 0.05% (d) increase by 0.05% (a) (b) 2 (c) (d) 18
9 3
Free eBooks on @neetquestionpaper2020

662 Physi cs
63. In the circuit shown in figure, with steady current, the potential
drop across the capacitor must be [IIT JEE 2001 S]
5W 10 W
V 10 V 20 V
2W
R
V (a) zero (b) 2 A (c) 5 A (d) 4 A
||
C 69. If a steady current I is flowing through a cylindrical element
ABC. Choose the correct relationship
A 2r B
r C
2V 2R
I
V V 2V
(a) V (b) (c) (d) l/2
2 3 3
l/2
64. The effective resistance between points P and Q of the [IIT JEE 2006 S]
electrical circuit shown in the figure is [IIT JEE 2002 S]
(a) VAB = 2VBC
2Rr (b) Power across BC is 4 times the power across AB
(a) (c) Current densities in AB and BC are equal
R+r 2R 2R
(d) Electric field due to current inside AB and BC are equal
8R ( R + r ) 70. A resistance of 2W is connected across one gap of a metre-
(b) 2R
3R + r bridge (the length of the wire is 100 cm) and an unknown
P Q resistance, greater than 2W, is connected across the other
(c) 2r + 4R r r
2R 2R
2R gap. When these resistances are interchanged, the balance
point shifts by 20 cm. Neglecting any corrections, the
5R
(d) + 2r unknown resistance is [IIT JEE 2007]
2 (a) 3 W (b) 4 W
65. Express which of the following set ups can be used to verify (c) 5 W (d) 6 W
Ohm’s law? [IIT JEE 2003S] 71. When two identical batteries of internal resistance 1W each
are connected in series across a resistor R, the rate of heat
produced in R is J1. When the same batteries are connected
in parallel across R, the rate is J2. If J1 = 2.25 J2 then the
value of R in W is [IIT-JEE 2010]
(a) (b) (a) 4 W (b) 6 W
(c) 0.5 W (d) 8 W
72. A meter bridge is set up as shown, to determine an unknown
resistance ‘X’ using a standard 10 ohm resistor. The
galvanometer shows null point when tapping-key is at
52 cm mark. The end-corrections are 1 cm and 2 cm
(c) (d) respectively for the ends A and B. The determined value of
‘X’ is [IIT-JEE 2011]

66. In the shown arrangement of the experiment of the meter


bridge if AC corresponding to null deflection of
galvanometer is x, what would be its value if the radius of
the wire AB is doubled? [IIT JEE 2003 S]
(a) x
(b) x / 4 R1 R2 (a) 10.2 ohm (b) 10.6 ohm
(c) 10.8 ohm (d) 11.1 ohm
(c) 4 x G 73. For the resistance network shown in the figure, choose the
(d) 2 x correct option(s). [IIT-JEE 2012]
A x C B
67. Six identical resistors are connected as shown in the figure.
The equivalent resistance will be [IIT JEE 2004 S]
R
Q
(a) maximum between P and R P
(b) maximum between Q and R R R
R
R R
(c) maximum between P and Q

(d) all are equal (a) The current through PQ is zero.


R (b) I1 = 3A
68. Find out the value of current through 2W resistance for the (c) The potential at S is less than that at Q.
given circuit [IIT JEE 2005 S] (d) I2 = 2A
Free eBooks on @neetquestionpaper2020

Current Electricity 663

1. A given resistor has the following colour scheme of the 9. In the given circuit, as the sliding contact C is moved from
various strips on it : Brown, black, green and silver. Its A to B
value in ohm is V
(a) 1.0 ´ 10 4 ± 10% (b) 1.0 ´ 105 ± 10% C
6
(c) 1.0 ´ 10 ± 10% 7
(d) 1.0 ´ 10 ± 10% A B
2. If R1 and R2 are respectively the filament resistances of a
200 watt bulb and a 100 watt bulb designed to operate on A
the same voltage (a) the readings of both the ammeter and the
(a) R1 is two times R2 (b) R2 is two times R1 voltmeter remain constant
(c) R2 is four times R1 (d) R1 is four times R2 (b) the reading of both the ammeter and the voltmeter
3. Three copper wires of lengths and cross sectional areas are increase
(l, A), (2 l, A/2) and (l/2, 2A). Resistance is minimum in (c) the reading of the ammeter remains constant but
(a) wire of cross-sectional area A/2 that of the voltmeter increases
(b) wire of cross-sectional area A (d) the reading of the ammeter remains constant but
(c) wire of cross-sectional area 2A that of the voltmeter decreases
(d) same in all the three cases 10. The circuit shown in the figure contains two diodes, each
4. The current in the primary circuit of a potentiometer wire is with a forward resistance of 50 W and with infinite backward
0.5 A, r for the wire is 4 × 10–7 W–m and area of cross- resistance. If the battery voltage is 6 V, the current through
the 100-W resistance is
section of wire is 8 × 10–6 m2. The potential gradient in the
50W
wire would be
(a) 25 mV/meter (b) 2.5 mV/meter
(c) 25 V/meter (d) 10 V/meter 50W
5. A cell when balanced with potentiometer gave a balance
length of 50 cm. 4.5 W external resistance is introduced in 100W
the circuit, now it is balanced on 45 cm. The internal
6V
resistance of cell is (a) zero (b) 0.02 A
(a) 0.25 W (b) 0.5 W (c) 0.04 A (d) 0.036 A
(c) 1.0 W (d) 1.5 W 11. A 100-W bulb and a 25-W bulb are designed for the same
6. A primary cell has an e.m.f. of 1.5 volt. When short-circuited voltage. They have filaments of the same length and
it gives a current of 3 ampere. The internal resistance of the material. The ratio of the diameter of the 100-W bulb to that
cell is of the 25-W bulb is
(a) 4.5 ohm (b) 2 ohm (a) 4 : 1 (b) 2 : 1
(c) 0.5 ohm (d) (1/4.5) ohm (c) 2 :1 (d) 1 : 2
7. Two wires A and B of the same material, having radii in the 12. In an household electric circuit, which of the following is/
ratio 1 : 2 and carry currents in the ratio 4 : 1. The ratio of are correct?
drift speed of electrons in A and B is (A) All electric appliances drawing power are joined in
(a) 16 : 1 (b) 1 : 16 parallel
(c) 1 : 4 (d) 4 : 1 (B) A switch may be either in series or in parallel with the
8. A torch bulb rated as 4.5 W, 1.5 V is connected as shown in appliance which it controls
fig. The e.m.f. of the cell, needed to make the bulb glow at (C) If a switch is in parallel with an appliance, it will draw
full intensity is power when the switch is in the ‘off’ position (open)
4.5 W, (D) If a switch is in parallel with an appliance, the fuse will
1.5V blow (burn out) when the switch is put ‘on’ closed.
2E/9 (a) A, D (b) A, C, D
(c) B, C, D (d) A, B, D
13. Two identical fuses are rated at 10 A. If they are joined
E/9 (A) in parallel, the combination acts as a fuse of rating 20 A
0.33 W (B) in parallel, the combination acts as a fuse of rating 5 A
E/3 (C) in series, the combination acts as a fuse of rating 10 A
(D) in series, the combination acts as a fuse of rating 20 A
E, r = 2.67 W Select the correct options.
(a) 4.5 V (b) 1.5 V (a) A, B (b) A, C
(c) 2.67 V (d) 13.5 V (c) B, D (d) B, C, D
Free eBooks on @neetquestionpaper2020

664 Physi cs
14. A conductor carries a current of 50 m A. If the area of cross- 26. A potentiometer consists of a wire of length 4m and
section of the conductor is 50 mm2, then value of the current resistance 10W. It is connected to a cell of e.m.f. 3V. The
density in Am–2 is potential gradient of wire is
(a) 0.5 (b) 1 (a) 5V/m (b) 2V/m
(c) 10–3 (d) 10–6 (c) 5V/m (d) 10V/m
15. When a current I is set up in a wire of radius r, the drift 27. Potentiometer measures potential more accurately because
velocity is vd. If the same current is set up through a wire of (a) it measures potential in open circuit
radius 2 r, the drift velocity will be (b) it uses sensitive galvanometer for null deflection
(a) 4 vd (b) 2 vd (c) it uses high resistance potentiometer wire
(c) vd/2 (d) vd/4 (d) it measures potential in closed circuit
16. A flow of 107 electrons per second in a conducting wire 28. The four wires from a larger circuit intersect at junction A
constitutes a current of as shown. What is the magnitude and direction of the
(a) 1.6 ´ 10 -26 A (b) 1.6 ´ 10 26 A current between points A and B ?
4A
(c) 1.6 ´ 10 -12 A (d) 1.6 ´ 1012 A
A B
17. In a neon gas discharge tube Ne+ ions moving through a 5A
cross-section of the tube each second to the right is 2.9 × ]
1018, while 1.2 × 1018 electrons move towards left in the
same time; the electronic charge being 1.6 × 10–19 C, the 6A
(a) 2 A from A to B (b) 2A from B to A
net electric current is
(c) 3A from A to B (d) 2A from B to A
(a) 0.27 A to the right (b) 0.66 A to the right
29. Potentiometer wire of length 1 m is connected in series with
(c) 0.66 A to the left (d) zero
490W resistance and 2 V battery. If 0.2 mV/cm is the potential
18. A capacitor of 10m F has a pot. difference of 40 volts across
gradient, then resistance of the potentiometer wire is
it. If it is discharged in 0.2 second, the average current
(a) 4.9 W (b) 7.9 W (c) 5.9 W (d) 6.9 W
during discharge is
30. The deflection in a galvanometer decreases from 25 divisions
(a) 2 m A (b) 4 m A
to 5 divisions when a resistor of 20W is connected in series.
(c) 1 m A (d) 0.5 m A
Find resistance of galvanometer.
19. The amount of charge Q passed in time t through a cross-
(a) 4 W (b) 5 W (c) 6 W (d) 7 W
section of a wire is Q = 5 t2 + 3 t + 1.
31. A galvanometer of 50 ohm resistance has 25 divisions. A
The value of current at time t = 5 s is
current of 4 × 10–4 ampere gives a deflection of one division.
(a) 9 A (b) 49 A
To convert this galvanometer into a voltmeter having a range
(c) 53 A (d) None of these
of 25 volts, it should be connected with a resistance of
20. A wire X is half the diameter and half the length of a wire Y
(a) 2450 W in series. (b) 2500 W in series.
of similar material. The ratio of resistance of X to that of Y is
(c) 245 W in series. (d) 2550 W in series.
(a) 8 : 1 (b) 4 : 1 (c) 2 : 1 (d) 1 : 1
32. In the equation AB = C, A is the current density, C is the
21. 2, 4 and 6 S are the conductances of three conductors.
electric field, Then B is
When they are joined in series, their equivalent conductance
(a) resistivity (b) conductivity
will be
(c) potential difference (d) resistance
(a) 12 S (b) (1/12) S
33. The resistance of the coil of an ammeter is R. The shunt
(c) (12/11) S (d) (11/12) S
required to increase its range n-fold should have a
22. 2, 4 and 6 S are conductances of three conductors. When they
resistance
are joined in parallel, their equivalent conductance will be
(a) 12 S (b) (1/12) S R R
(a) (b)
(c) (12/11) S (d) (11/12) S n n -1
23. Two identical cells connected in series send 1.0A current
R
through a 5 W resistor. When they are connected in parallel, (c) (d) nR
they send 0.8 A current through the same resistor. What is n +1
the internal resistance of the cell? 34. A cell of internal resistance r is connected across an
(a) 0.5 W (b) 1.0 W (c) 1.5 W (d) 2.5 W external resistance nr. Then the ratio of the terminal voltage
24. If the resistance of a conductor is 5W at 50º C & 7W at to the emf of the cell is
100º C, then mean temperature coefficient of resistance (of 1 1 n n -1
material) is (a) (b) (c) (d)
n n +1 n +1 n
(a) 0.013/ ºC (b) 0.004/ ºC 35. Coils in the resistance boxes are made from doubled up
(c) 0.006/ ºC (d) 0.008/ ºC insulated wires
25. If negligibly small current is passed though a wire of length (a) to cancel the effect of self induction
15 m & resistance of 5W, having uniform cross section of (b) to nullify the heating effect
6 × 10–7 m2, then coefficient of resistivity of material is (c) to nullify the Peltier effect
(a) 1×10–7W–m (b) 2×10–7W–m (d) to reduce effective length of the wire
(c) 3×10–7W–m (d) 4×10–7W–m
Free eBooks on @neetquestionpaper2020

Current Electricity 665


36. For measuring voltage of any circuit, potentiometer is 46. Two heating wires of equal length are first connected in
preferred to voltmeter because series and then in parallel to a constant voltage source.
(a) the potentiometer is cheap and easy to handle. The rate of heat produced in two cases is (parallel to series)
(b) calibration in the voltmeter is sometimes wrong . (a) 1 : 4 (b) 4 : 1 (c) 1 : 2 (d) 2 : 1
(c) t h e pot en t i a l dr a ws n o cur r ent duri n g 47. Two identical batteries each of e.m.f. 2 V and internal
measurement. resistance 1 W are available to produce heat in an external
(d) range of the voltmeter is not as wide as that of resistance by passing a current through it. The maximum
the potentiometer. power that can be developed across R using these batteries
37. The infinity resistance plug in a post-office box has is
(a) an air gap only (a) 3.2 W (b) 2.0 W (c) 1.28 W (d) 8/9 W
(b) a resistance coil of infinite resistance 48. A dynamo develops 0.5 A at 6 V. The energy which is
(c) largest resistance available in box generated in one second is
(d) resistance of the coil 5000 W (a) 0.083 J (b) 3 J
38. An ammeter has a resistance of G ohm and a range of I amp. (c) 12 J (d) None of these
The value of resistance used in parallel to convert it into an 49. In an electroplating experiment, m g of silver is deposited
ammeter of range nI amp is when 4 A of current flows for 2 minutes. The amount in g of
(a) nG (b) (n – 1)G silver deposited by 6 A of current for 40 seconds will be
(c) G/n (d) G/(n – 1) (a) 4 m (b) 2 m (c) m/2 (d) m/4
39. What is the equivalent resistance between the points A 50. A battery of e.m.f. 10 V and internal resistance 0.5 W is
and D in given figure? connected across a variable resistance R. The value of R
for which the power delivered in it is maximum is given by
(a) 0.5 W (b) 1.0 W (c) 2.0 W (d) 0.25 W
51. A heater of 220 V heat a volume of water in 5 minutes time.
A heater of 110 V heats the same volume of water in
(a) 5 minutes (b) 8 minutes
(c) 10 minutes (d) 20 minutes
(a) 10 W (b) 20 W 52. The internal resistance of a primary cell is 4W. It generates
(c) 30 W (d) 40 W a current of 0.2 A in an external reistance of 21 W. The rate
40. Two wires of same metal have the same length but their of chemical energy consumed in providing the current is
cross-sections are in the ratio 3 : 1. They are joined in series. (a) 0.42 J s–1 (b) 0.84 J s–1
(c) 1 J s –1 (d) 5 J s–1
The resistance of the thicker wire is 10 W. The total
resistance of the combination is 53. Water boils in the electric kettle in 15 minutes after switching
(a) 5/2 W (b) 40/3 W on. If the length of heating wire is decreased to 2/3 of its
(c) 40 W (d) 100 W initial value, then the same amount of water will boil with
41. In a metre bridge, the balancing length from the left end the same supply voltage in
(standard resistance of one ohm is in the right gap) is found to (a) 8 minutes (b) 10 minutes
be 20 cm. The value of the unknown resistance is (c) 12 minutes (d) 15 minutes
(a) 0.8 W (b) 0.5 W 54. An electric lamp is marked 60 W, 220 V. The cost of kilo
(c) 0.4 W (d) 0.25 W watt hour of electricity is Rs. 1.25. The cost of using this
42. The resistance of a wire at room temperature 30°C is found lamp on 220 V for 8 hours is
to be 10 W. Now to increase the resistance by 10%, the (a) Re 0.25 (b) Re 0.60
temperature of the wire must be [ The temperature coefficient (c) Re 1.20 (d) Re 4.00
of resistance of the material of the wire is 0.002 per °C] 55. A steady current of 5 A is maintained for 45 minutes. During
(a) 36°C (b) 83°C (c) 63°C (d) 33°C this time it deposits 4.572 g of zinc at the cathode of voltameter.
43. A potentiometer wire, 10 m long, has a resistance of 40W. It E.C.E. of zinc is
is connected in series with a resistance box and a 2 V storage (a) 3.387 × 10–4 g/C (b) 3.387 × 10–4 kg/C
cell. If the potential gradient along the wire is 0.1 m V/cm, –4
(c) 3.384 × 10 kg/C (d) 3.384 × 10–3 kg/C
the resistance unplugged in the box is
56. The cold junction of a thermocouple is maintained at 10ºC.
(a) 260 W (b) 760 W
No thermo e.m.f. is developed when the hot junction is
(c) 960 W (d) 1060 W
maintained at 530ºC. The neutral temperature is
44. If R1 and R2 are the filament resistances of 200 W and a 100
(a) 260ºC (b) 265ºC (c) 270ºC (d) 520ºC
W bulb respectively both designed to run at the same
voltage, then 57. If current flowing in a conductor changes by 1% then power
(a) R2 is four times of R1 (b) R1 is four times of R2 consumed will change by
(c) R2 is two times of R1 (d) R1 is two times of R2 (a) 10% (b) 2% (c) 1% (d) 100%
45. An electric fan and a heater are marked as 100 W, 220 V and 58. Two electric bulbs whose resistance are in the ratio 1 : 2 are
1000 W, 220 V respectively. The resistance of heater is arranged in parallel to a constant voltage source. The
(a) equal to that of fan (b) lesser than that of fan powers dissipated in them have the ratio
(c) greater than that of fan (d) zero (a) 1 : 2 (b) 1 : 1 (c) 2 : 1 (d) 1 : 4
Free eBooks on @neetquestionpaper2020

666 Physi cs
59. If nealy 105 coulomb are liberated by 1gm equivalent of 70. Each of the resistance in the network shown in fig. is equal
aluminium, then amount of aluminium (equivalent weight 9) to R. The resistance between the terminals A and B is
deposited through electrolysis in 20 minutes by a current L
of 50 ampere will be :
(a) 0.6 gm. (b) 0.09 gm (c) 5.4 gm (d) 10.8 gm
R R R
60. Three equal resistors, connected across a source of e.m.f.
together dissipate 10 watt of power. What will be the power A

R
dissipated in watts if the same resistors are connected in R B
K M
parallel across the same source of e.m.f. (a) R (b) 5 R
(a) 10 (b) 10/3 (c) 30 (d) 90 (c) 3 R (d) 6 R
61. Who among the following scientists made the statement ? 71. Four resistors are connected as shown in fig. A 6 V battery
“Chemical change can produce electricity”. of negligible resistance is connected across terminal AC.
(a) Galvani (b) Faraday The potential difference across terminals B, D will be
(c) Coulomb (d) Thompson 6V
+ –
62. Which of the following is not reversible ?
(a) Joule effect (b) Peltier effect
B C D E
(c) Seebeck effect (d) Thomson effect A
5W 15 W 30 W 10 W
63. In which of the following the power dissipation is
proportional to the square of the current ?
(a) 0 V (b) 1.5 V
(a) Peltier effect (b) Joule's effect (c) 2 V (d) 3 V
(c) Thomson effect (d) None of the above 72. A non-conducting ring of radius R has charge Q distributed
64. A wire of radius r and another wire of radius 2r, both of unevenly over it. If it rotates with an angular velocity w, the
same material and length are connected in series to each equivalent current will be
other. The combination is connected across a battery. The (a) zero (b) Qw
ratio of the heats produced in the two wires will be w w
(a) 4.00 (b) 2.00 (c) 0.50 (d) 0.25 (c) Q (d) Q
2p 2 pR
65. The electrochemical equivalent of a metal is 3.3 × 10–7 kg 73. All the edges of a block with parallel faces are unequal. Its
per coulomb. The mass of the metal liberated at the longest edge is twice its shortest edge. The ratio of the
cathode when a 3 A current is passed for 2 seconds will maximum to minimum resistance between parallel faces is
be (a) 2 (b) 4
(a) 19.8 × 10–7 kg (b) 9.9 × 10–7 kg (c) 8
(c) 6.6 × 10 kg–7 (d) 1.1 × 10–7 kg (d) indeterminate unless the length of the third edge is
66. When current is passed through a junction of two dissimilar specified
metals, heat is evolved or absorbed at the junction. This
74. In the network shown below, the ring has zero resistance.
process is called The equivalent resistance between the point A and B is
(a) Seebeck effect (b) Joule effect
(c) Petlier effect (d) Thomson effect (a) 2R
67. An electrical cable of copper has just one wire of radius 9 3R
(b) 4R 3R A
mm. Its resistance is 5 ohm. This single copper wire of the B
cable is replaced by 6 different well insulated copper wires (c) 7R R
each of radius 3 mm. The total resistance of the cable will 3R
now be equal to (d) 10R
(a) 7.5 ohm (b) 45 ohm 75. The belt of an electrostatic generator is 50 cm wide and
(c) 90 ohm (d) 270 ohm travels at 30 cm/sec. The belt carries charge into the sphere
68. In an experiment to measure the internal resistance of a cell, at a rate corresponding to 10–4 ampere. What is the surface
by a potentiometer, it is found that the balance point is at a density of charge on the belt.
length of 2 m, when the cell is shunted by a 5 W resistance (a) 6.7 ´10 -5 C m -2 / s (b) 6.7 ´10 -4 C m -2 / s
and is at a length of 3 m when the cell is shunted by a 10 W
resistance. The internal resistance of the cell is then (c) 6.7 ´10 -7 C m -2 / s (d) 6.7 ´10 -8 C m -2 / s
(a) 1.5 W (b) 10 W 76. A 4 ohm resistance wire is bent through 180º at its mid point
(c) 15 W (d) 1 W and the two halves are twisted together. Then the resistance
69. To get maximum current in a resistance of 3 ohms, one can is
use n rows of m cells (connected in series) connected in (a) 1 W (b) 2 W (c) 5 W (d) 8 W
parallel. If the total number of cells is 24 and the internal 77. A wire has a resistance 12 W. It is bent in the form of a circle.
resistance of a cell is 0.5 ohms then The effective resistance between two points on any
(a) m = 12, n = 2 (b) m = 8, n = 3 diameter is
(c) m = 2, n = 12 (d) m = 6, n = 4 (a) 6 W (b) 3 W (c) 12 W (d) 24 W
Free eBooks on @neetquestionpaper2020

Current Electricity 667


78. A 4 m long wire of resistance 8 W is connected in series 85. Determine the current in 2W resistor.
with a battery of e.m.f. 2 V and a resistor of 7 W. The internal
resistance of the battery is 1 W. What is the potential gradient 2W
along the wire?
3W
(a) 1.00 V m–1 (b) 0.75 V m–1
(c) 0.50 V m–1 (d) 0.25 V m–1 1W
79. The length of a given cylindrical wire is increased by 100%.
Due to the consequent decrease in diameter the change in
the resistance of the wire will be 6V 2.8 W
(a) 100% (b) 50% (c) 300% (d) 200%
80. Two resistances R1 and R2 are made of different materials.
The temperature coefficient of the material of R1 is a and (a) 1 A (b) 1.5 A (c) 0.9 A (d) 0.6 A
that of material of R2 is – b . The resistance of the series 86. Twelve resistors each of resistance 16 W are connected in
combination of R1 and R2 will not change with temperature the circuit as shown. The net resistance between AB is
R1
if equal to
R2

a a +b a 2 + b2 b
(a) (b) (c) (d)
b a -b 2ab a
81. Current I1 in the following circuit is
30W

I1
(a) 1 W (b) 2 W (c) 3 W (d) 4 W
40W 40V
87. A battery of e.m.f E and internal resistance r is connected
I3 to a variable resistor R as shown. Which one of the
I2
following is true ?
E r
40W 80V
(a) 0.4A (b) – 0.4 A (c) 0.8 A (d) – 0.8 A
82. Null point with 1V cell comes out to be 55 cm and with R
R = 10 W it is 50 cm. What is the internal resistance of the
cell ?
2V (a) Potential difference across the terminals of the
100 cm battery is maximum where R = r
(b) Power delivered to resistor is maximum when
1V R= r
R (c) Current in the circuit is maximum when R = r
(d) Current in the circuit is maximum when R >> r
(a) 0.5W (b) 0.4W (c) 1W (d) 0.2W 88. There is an infinite wire grid with cells in the form of
83. Three resistances are connected to form a T-shape as shown equilateral triangles. The resistance of each wire between
in the figure. Then the current in the 4K resistor is: neighbouring joint connections is R0. The net resistance
+2V 2K 8K – 4V of the whole grid between the points A and B as shown is

4K A B

– 8V
(a) 0.93mA (b) 1.42mA
(c) 2.5mA (d) 1.57mA R0 R0 R0
(a) R0 (b) (c) (d)
84. The equivalent resistance between A and B is 2 3 4
A 89. Is it possible that any battery has some constant value of
R R e.m.f but the potential difference between the plates is
zero?
B R (a) No
(b) Yes, if another identical battery is joined in series.
R R (c) Yes, if another identical battery is joined in
opposition in series.
8R 5R 3R 7R
(a) (b) (c) (d) (d) Yes, if another similar battery is joined in parallel.
5 8 8 8
Free eBooks on @neetquestionpaper2020

668 Physi cs

90. Twelve indentical resistors each of value 1 W are connected 96. Seven resistances, each of value 20 W, are connected to a 2
as shown. Net resistance between C and D (R) is V battery as shown in the figure. The ammeter reading will
be
C
7
(a) R= W
6 G 2V
4
(b) R= W A E A
3 B
F (a) 1/10 A (b) 3/10 A
(c) R =1 W
H (c) 4/10 A (d) 7/10 A.
3 97. In the circuit shown below, if the resistance of voltmeter is
(d) R = W 4 kW, then the error in the reading of voltmeter will be
4 D
91. Which of the following in electricity is analogous to
momentum mv in dynamics ?
(a) IV (b) I L (c) QL (d) IQ
92. The numerical value of charge on either plate of capacitor C
shown in figure is

(a) CE
CER1 C R2
(b) ||
R1 + r (a) 50% (b) 68% (c) 17% (d) 33.3%
R1 98. In the circuit , the galvanometer G shows zero deflection. If
CER 2 the batteries A and B have negligible internal resistance,
(c)
R2 + r E the value of the resistor R will be
r
CER1 500 W
(d) G
R2 + r
2V
93. In the circuit shown, the internal resistance of the cell is 12V B R A
negligible. The steady state current in the 2W resistor is

(a) 0.6 A (a) 100 W (b) 200W


(b) 0.9 A (c) 1000 W (d) 500 W
99. A fuse wire with a radius of 1 mm blows at 1.5 A. If the fuse
(c) 1.2 A wire of the same material should blow at 3.0 A, the radius of
the wire must be
(d) 1.5 A
(a) 41/3 mm (b) 2 mm
94. In the network shown, each resistance is equal to R. The
equivalent resistance between adjacent corners A and D is (c) 0.5 mm (d) 8.0 mm
100. There are two electric bulbs of 40 W and 100 W. Which one
(a) R will be brighter when first they are connected in series and
then in parallel.
2
(b) R (a) 40 W in series and 100 watt in parallel
3 (b) 100 W in series and 40 watt in parallel
3 (c) 40 W both in series and parallel will be uniform
(c) R (d) 100 W both in series and parallel will be uniform
7
101. A 4 m F conductor is charged 50 volts and then its plates
8 are joined through a resistance of 1 k W. The heat produced
(d) R
15 in the resistance is
95. Resistances 1 W, 2 W and 3 W are connected to form a (a) 0.16 J (b) 1.28 J (c) 0.64 J (d) 0.32 J
triangle. If a 1.5 V cell of negligible internal resistance is 102. The thermo e.m.f. of a thermocouple is given by
connected across the 3 W resistor, the current flowing E = 2164 t – 6.2 t2. The neutral temperature and a temperature
through this resistor will be of inversion are
(a) 0.25 A (b) 0.5 A (a) 349, 174.5 (b) 174.5, 349
(c) 1.0 A (d) 1.5 A (c) 349, 698 (d) 698, 349
Free eBooks on @neetquestionpaper2020

Current Electricity 669


103. One junction of a certain thermocouple is at a fixed 110. Two different metals are joined end to end. One end is kept
temperature Tr and the other junction is at a temperature T. at constant temperature and other end is heated to a very
The electormotive force for this is expressed by, high temperature. The graph depicting the thermo e.m.f. is
é 1 ù
E = k (T - Tr ) êT0 - (T + Tr ) ú
ë 2 û
At temperature T = T0/2, the thermo electric power is (a) (b)
(a) k T0/2 (b) k T0
T T
k T02 1 2
(c) (d) k (T0 - Ti )
2 2
104. The e.m.f. developed in a thermo-couple is given by
1 (c) (d)
E = a T + bT2
2
where T is the temperature of hot junction, cold junction
being at 0ºC. The thermo electric power of the couple is 111. A current source drives a current in a coil of resistance R1
for a time t. The same source drives current in another coil
b
(a) a+ T (b) a +bT of resistance R2 for same time. If heat generated is same,
2 find internal resistance of source.
a T 2 b T3 R1R 2
(c) + (d) a / 2 b (a) (b) R1 + R 2
2 6 R1 + R 2
105. The thermo e.m.f. E in volts of a certain thermocouple is
found to vary with temperature T of hot junction while cold (c) zero (d) R 1R 2
junction is kept at 0ºC 112. Three equal resistors connected across a source of e.m.f.
2 together dissipate 10 watt of power. What will be the power
T
E = 40 T - dissipated in watts if the same resistors are connected in
20
The neutral temperature of the couple is parallel across the same source of e.m.f.?
(a) 100ºC (b) 200ºC (c) 400ºC (d) 800ºC 10
106. Two bulbs of 500 W and 200 W are manufactured to operate (a) 10 (b) (c) 30 (d) 90
3
on 220 V line. The ratio of heat produced in 500 W and 200
W, in two cases, when firstly they are connected in parallel 113. An electric heating element in vacuum is surrounded by a
and secondary in series will be cavity at constant temperature of 227ºC; it consumes 60W
of power to maintain a temperature of 727ºC. What is the
5 2 5 5 2 5 2 2 power consumed by the element to maintain a temperature
(a) : (b) : (c) : (d) :
2 5 2 2 5 2 5 5 of 1227º C?
107. A wire of resistance 20 W is covered with ice and a voltage (a) 101 W (b) 304 W (c) 90 W (d) 320 W
of 210 V is applied across the wire, then rate of melting the 114. Silver and copper voltameters are connected in parallel
ice is with a battery of e.m.f 12 V. In 30 minute 1 g of silver and
(a) 0.85 g/s (b) 1.92 g/s 1.8 g of copper are liberated. The energy supplied by the
(c) 6.56 g/s (d) All of these battery is
108. Two identical batteries, each of e.m.f. 2 volt and internal
[ ZAg = 11.2 × 10–4gc–1; ZCu = 6.6 × 10–4 gc–1]
resistance 1.0 ohm are available to produce heat in a
resistance R = 0.5 W, by passing a current through it. The (a) 720 J (b) 2.41 J
maximum power that can be developed across R using these (c) 24.12 J (d) 4.34 × 104 J
batteries is 115. You are given a resistance coil and a battery. In which of
1W 2V the following cases is largest amount of heat generated ?
(a) When the coil is connected to the battery directly
(b) When the coil is divided into two equal parts and
1W 2V both the parts are connected to the battery in parallel
(c) When the coil is divided into four equal parts and all
the four parts are connected to the battery in parallel
(d) When only half the coil is connected to the battery
0.5W 116. In the Seebeck series Bi occurs first followed by Cu and Fe
(a) 1.28 W (b) 2.0 W (c) 8/9 W (d) 3.2 W among other. The Sb is the last in the series. If z1 be the
109. The thermo e.m.f. of a thermocouple is 25mV/ºC at room thermo emf at the given temperature difference for Bi – Sb
temperature. A galvanometer of 40 ohm resistance, capable thermocouple and z, be that for Cu-Fe thermocouple, which
of detecting current as low as 10–5 A, is connected with the of the following is true?
thermocouple. The smallest temperature difference that can (a) z1 = z2 (b) z1 < z2 (c) z1 > z2
be detected by this system is (d) Data is not sufficient to predict it.
(a) 12ºC (b) 0ºC (c) 20ºC (d) 16ºC
Free eBooks on @neetquestionpaper2020

670 Physi cs

117. 50 electric bulbs are connected in series across a 220 V 125. Statement 1 : When current through a bulb decreases by
supply and the illumination produced is I1. 5 bulbs are 0.5%, the glow of bulb decreases by 1%.
fused. If the remaining 45 are again connected in series, Statement 2 : Glow (Power) which is directly proportional
the illumination produced is I2. Which of the following is to square of current.
true ? 126. Statement 1 : Long distance power transmission is done
at high voltage.
(a) I1 = I2 (b) I1 < I2 (c) I1 > I2
Statement 2 : At high voltage supply power losses are
(d) It will depend on the resistance of each bulb.
less.
118. A leclanche cell supplies a current of one ampere for ten
127. Statement 1 : Resistance of 50W bulb is greater than that
minutes. The electreochemical equivalent of hydrogen = of 100 W.
0.00001014 gram per coulomb. The mass of hydrogen Statement 2 : Resistance of bulb is inversely proportional
liberated is : to rated power.
(a) 0.00625 g (b) 0.01248 g 128. Statement 1 : 40 W tube light give more light in comparison
(c) 0.01872 g (d) 0.02496 g. to 40 w bulb.
119. Three equal resistors connected across a source of e.m.f. Statement 2 : Light produced is same from same power.
together dissipate 10 watt of power. What will be the power 129. Statement 1: In a simple battery circuit, the point of the
dissipated in watt if the same resistors are connected in parallel lowest potential is negative terminal of the battery.
across the same source of e.m.f.? Statement 2: The current flows towards the point of the
(a) 10/3 (b) 10 (c) 30 (d) 90 higher potential, as it does in such a circuit from the negative
120. Two 1000 W heaters when connected in parallel across to the positive terminal.
130. Statement 1 : The working of thermoelectric refrigerator is
220 V supply produced heat QP in time t. If they are
based on Peltier effect.
connected in series across the same power supply the heat
Statement 2 : When current is passed through a
produced in the same time is QS. What is QP/QS? thermocouple, heat is absorbed at one junction and is
(a) 4 (b) 2 (c) 0.5 (d) 0.25. evolved at the other junction, thus thermocouple produces
121. In the circuit shown in figure, the 5W resistance a cooling effect.
develops 20.00 cal/s due to the current flowing through it. r
131. Statement 1 : The current density J at any point in ohmic
The heat developed in 2 W resistance (in cal/s) is r
resistor is in direction of electric field E at that point.
Statement 2 : A point charge when released from rest in a
region having only electrostatic field always moves along
electric lines of force.
Directions for Qs. (132 to 144) : Read the following passage(s)
(a) 23.8 (b) 14.2 (c) 11.9 (d) 7.1 carefully and answer the questions that follows:
PASSAGE-1
Directions for Qs. (122 to 131) : Each question contains
A 12-volt battery is connected to two light bulbs, as drawn in
STATEMENT-1 and STATEMENT-2. Choose the correct answer figure 1 light bulb 1 has resistance 3 ohms, while light bulb 2 has
(ONLY ONE option is correct ) from the following- resistance 6 ohms. The battery has essentially no internal
(a) Statement -1 is true, Statement-2 is true; Statement -2 is a resistance, and all the wires are essentially resistanceless, too.
correct explanation for Statement-1 When a light bulb is unscrewed, no current
(b) Statement -1 is true, Statement-2 is true; Statement -2 is not flows through that branch of the circuit.
a correct explanation for Statement-1 For instance , if light bulb 2 is unscrewed,
(c) Statement -1 is true, Statement-2 is false current flows only around the lower loop bulb 2

(d) Statement -1 is false, Statement-2 is true of the circuit, which consists of the battery
122. Statement 1 : When a battery is short-circuited, the terminal and light bulb 1. The more current flows
voltage is zero. through a light bulb, their equivalent bulb 1
Statement 2 : In the situation of a short-circuit, the current resistance is R eq = R1 + R 2 . By contrast,
is zero when two reisistors are wired in parallel, + –
123. Statement 1 : A current flows in a conductor only when their net resisance is given by
there is an electric field within the conductor.
1 1 1 12V
Statement 2 : The drift velocity of electron in presence of = +
R eq R1 R 2
electric field decreases.
124. Statement 1 : Voltmeter is much better than a potentiometer 132. When bulb 1 is screwed in , but bulb 2 is unscrewed, the
for measuring emf of cell. power generated in bulb 1 is
Statement 2 : A potentiometer draws no current while (a) 4 watts (b) 12 watts
measuring emf of a cell. (c) 36 watts (d) 48 watt
Free eBooks on @neetquestionpaper2020

Current Electricity 671

133. Which of the following statement is false? PASSAGE-3


(a) Some of the energy produced by the light bulb takes A resistor circuit is constructed such that twelve resistors are
the form of heat. arranged to form a cube as shown in figure. Each resistor has a
(b) The battery is the source of all the electrons flowing resistance of 2 ohm.
around the circuit.
(c) The current entering the light bulb equals the current
leaving the light bulb.
(d) The potential in the wire to the left of the light bulb
differs from the potential in the wire to the right of
that bulb.
134. Bulb 2 is now screwed in as a result, bulb 1
(a) turns off (b) becomes dimmer
(c) stays about the same brightness The potential difference of 30 V is applied across two of the
opposing points as shown.
(d) becomes brighter
139. The points having the same potential are :
135. With both light bulbs screwed in, the current through the (i) B, D, E (ii) C, F, H (iii) C, E
battery is (a) only (i) is correct
(a) 1.2 ampere (b) 2 ampere (b) (i), (ii) and (iii) are correct
(c) 4 ampere (d) 6 ampere (c) only (ii) is correct
PASSAGE-2 (d) (i) and (ii) both are correct
In the circuit given below, both batteries are ideal. EMF E1 of 140. If we replace resistors between A and B and resistors
battery 1 has a fixed value, but emf E 2 of battery 2 can be varied between G and H by resistors with wires of zero resistance,
between 1.0V and 10.0V. The graph gives the currents through then the points having the same potential are
the two batteries as a function of E2, but are not marked as which (i) D, E, C, F (ii) A, B (iii) G, H
plot corresponds to which battery. But for both plots, current is (a) only (i) is correct
(b) only (ii) is correct
assumed to be negative when the direction of the current through
(c) only (iii) is correct
the battery is opposite the direction of that battery's emf.
(d) (i), (ii) and (iii) are correct
(Direction of emf is from negative to positive.) 141. In the above question, the potential difference between the
points C and G is
(a) 15 V (b) 10 V
(c) 20 V (d) 7.5 V
E2 + R1 R2 PASSAGE-4

Potentiometer is an ideal voltmeter as voltmeter draws some
+ current through the circuit while potentiometer needs no current
– E1 to work. Potentiometer works on the principle of e.m.f. comparison.
In working condition, a constant current flows through out the
wire of potentiometer using standard cell of e. m. f. e1. The wire of
0.4 potentiometer is made of uniform material and cross sectional
area and it has uniform resistance per unit length. The potential
gradient, depends upon the current in the wire.
A potentiometer with a cell of e. m. f. 2V and internal resistance
Current (A)

0.2 0.4 W is used across the wire AB. A standard cadmium cell of e.
m. f. 1.02 V gives a balance point at 66.3 cm length of wire. The
standard cell is then replaced by a cell of unknown e. m. f. e and
the balance point found similarly turns out to be 82.3 cm length
0 5 10 of the wire. The length of potentiometer wire AB is 1 m .
142. The value of e is
(a) 1.26 V (b) 2.63 V
(c) 1.83 V (d) None of these
–0.2
E2(V) 143. The reading of potentiometer if 4 V battery is used instead
of e, is
(a) 88.3 cm (b) 47.3 cm
136. The value of emf E1 is
(c) 95 cm (d) cannot be calculated
(a) 8 V (b) 6 V (c) 4 V (d) 2 V
144. If the resistance is connected across the cell e, the balancing
137. The resistance R1 has value
length will
(a) 10 W (b) 20 W (c) 30 W (d) 40 W
(a) be increase (b) be decrease
138. The resistance R2 is equal to
(c) remain same (d) None of these
(a) 10 W (b) 20 W (c) 30 W (d) 40 W
Free eBooks on @neetquestionpaper2020

672 Physi cs

Exercise 18.1 I
11. (c) I < n A e vd or vd <
1. (c) The current flowing from positive terminal to negative
nAe
terminal is due to flow of positive charge, which is the Total number of free electrons in the unit length of
conventional current. conductor, N = n A × 1.
2. (c) The value of drift velocity of electrons = 10–5 m/s and Total linear momentum of all the free electrons per
that of thermal velocity = 105 m/s. unit length
3. (d) The speed at which current travels through the Ι Ι Ι
< (N m)v d < n A m ≥ < <
conductor means the speed of electric effect travelling nAe (e / m) s
through a conductor which is at the speed of light.
4. (a) In the absence of electric field, the electron moves in 12. (a) The direction of electric field is from right to left. Hence
zig-zag direction as shown in figure. So net æ dV ö
displacement is zero, hence mean velocity of free the potential at B is higher than potential çè E x = ÷
dx ø
electrons in a conductor at absolute temperature is
at point A, so the speed of electron is more at B than
zero
at A.
Exercise 18.2

ml 1
net direction or 2. (c) R= i.e. R µ .
2 n
n e tA
direction of electron movement
(In the absence of E) (In presence of 5. (b) At 0K, there is no free charge carries in the germanium
electric field) and hence it behaves as an insulator.
I 6. (a) With decrease in temperature, resistance of metal
[vd = drift velocity = , where n is number of free conductor decreases. But R µ 1 / t , therefore time of
neA
electrons per unit volume, I is current that flows relaxation t increases.
through a cross section A & e is electronic charge (1.6 7. (a) Slope of the graph will give us reciprocal of resistance.
× 10–19 coulomb) ]. Here resistance at temperature T1 is greater than that
eE of T2. Since resistance of metallic wire is more at higher
e æVö
5. (b) Drift velocity vd < σ < σ çç ÷÷÷; so vd × V temperature than at lower temperature, hence T1 > T2.
m m èlø 11. (d) Only number of free electrons is constant, other factors
6. (a) Electric current, I = charge /time period of revolution are temperature dependent.
= e/(2 p r / v). 14. (a) When the number of collisions of free electrons with
7. (b) We know that the lattice is decreased in the resistor, then it means
I I that less hindrance occurs in the flow of electrons,
J< and E < ;
A ρA consequently the resistance is low.
16. (c) Magnanin is used in making the resistance coils as its
J
[ E< < θJ < kJ. temperature coefficient is least and there is no
ρ appreciable change in resistance with temperature.
1 eE
9. (a) Since average drift velocity = ´ (t) V2 P 2
2 m 17. (b) P = I2 R = , in parallel, voltage is same, so 1 =
Now I = NeA × (avg. drift velocity) R P2 1
Ne2 AE Ne2 AV rl rl 4rl
< ≥σ < ≥σ 18. (c) Since R = = =
2ml 2ml A p (d / 2 ) 2
pd 2
V 2ml If l becomes l /2 & d becomes 2d, the new resistance
R< < 2 , where N is electron density.
I Ne σA rl/2 rl R
10. (d) The current density of electrons in a metallic conductor is R ¢ = = =
2 2 8
p( 2d / 2) 2 ´ pd
is » 10 22 / cm 3 or 10 28 / m 3 .
Free eBooks on @neetquestionpaper2020

Current Electricity 673


19. (c) The portion of the graph in which the current decreases 3. (b) If we add two non ideal batteries in series then
when voltage increases shows the negative resistance.
Hence the portion CD shows the negative resistance. E1 E2 E1 E2
20. (d) For smallest resistance, the resistors are to be
connected in parallel. The effective resistance = R/n.
21. (c) Electric Energy = power × time total E.M.F. = E1+E2 total E.M.F. = E1–E2
= voltage × charge = (current)2 × resistance × time (a) (b)
22. (d) Fuse wire should be such that it melts immediatley
when strong current flows through the circuit. The so in case (a) the equivalent e.m.f may be larger than
same is possible if its melting point is low and resistivity either of two e.m.f, but in case (b) the equivalent e.m.f
is high. may be smaller than either of two e.m.f.
23. (c) Fuse wire is made of tin-lead alloy (63% tin + 37% In series total resistance is always greater than
lead) individual resistance, whether batteries are connected
24. (a) A heating wire should be such that it produces more in any way [i.e., either according to case (a) or case
heat when current is passed through it and also does (b)].
not melt. It will be so if it has high specific resistance nE
and high melting point. 7. (c) In series combination of cells, Current Ι =
25. (b) The rate of generation of heat, for a given potential nr + R ¢
difference is, P = V2/R E
In parallel combination of cells, Ι¢ =
26. (d) Resistance, R = V2/P = (220)2/P. Since P of each (r / n) + R
appliance is the same, hence R is same for all the three nE E nE
If Ι = Ι¢ then = =
appliances. nr + R (r / n ) + R r + nR
27. (b) Brightness of a bulb in a house fitting is directly It will be so if r = R
proportional to the power of bulb and E
resistance µ 1/(power). E =
8. (a) Ι = 2 2
2 R+r [( R ) + ( r ) - 2 2 r ] + 2 R r
28. (c) As H µ I so for heating effect both a.c. and d.c. can
be used. E
=
29. 2 2
(b) The rate of heat generation = I R = I (r l / p r ). 2 ( R - r )2 + 2 R r

R´R I will be maximum if ( R - r ) = 0 or R = r


30. (d) R s = R + R = 2 R and R p = = R / 2.
R +R
10. (d) The masses of the copper and silver deposited on
2
H1 I R s R s 2R 4 H 1 cathodes of copper voltameter and silver voltameter
= = = = or 2 = are in the ratio of their chemical equivalent of copper
2
H2 I R p R p R / 2 1 H1 4
and silver respectively.
31. (a) Heat produced, H = V2 t /R i.e. H µ 1/R
12. (c) m Ag/m Zn = EAg/E Zn = 108/31
so H1/H2 = R2 / R1.
32. (a) In parallel combination, total power P = P1 + P2.
or m Ag = m Zn × 108/31 = W × 108/31
34. (c) Joule heat in resistor Q = I2 R or for given resistor
Q µ I2, so it cannot be joule heat. 14. (b) In electroplating, the metallic ions are positive, which
are deposited on cathode.
Exercise 18.3
15. (c) Electroplating does not help in making the metals
1. (c) Inside the battery or cell, the positive charge move become hard.
from negative terminal (lower potential) to positive 16. (a) Faraday’s laws are based on the conversion of
terminal (higher potential) of the battery. While in electrical energy into mechanical energy; which is in
external circuit the positive charge move from positive accordance with the law of conservation of energy.
terminal to negative terminal of the battery. 17. (b) The amount of decomposition (i.e. mass of the
direction of +ive charge direction of +ive charge substance liberated during electrolysis) is
in external circuit inside the battery proportional to electro chemical equivalent of the
I substance.
18. (b) Current inside a copper voltameter is same as that of
+ –
outside.
1 M 1 E
19. (b) Z= × = E; so, F = = Ne.
Ne P Ne Z
2. (a) P=I2R, If we decrease R, then I increases hence P 20. (a) m = ZIt = Zq where I is the current & q is the amount
increases. of charge.
Free eBooks on @neetquestionpaper2020

674 Physi cs
22. (d) Current in the circuit, I = E/(R +r) and heat produced is 5. (b) When Wheatstone bridge is balanced, then
I2 R. P R P Q
2 2
= or = . If the galvanometer is replaced
E R1 E R2 Q S R S
So H1 = = H2 =
2 with cell in balanced Wheatstone bridge, then the
(R1 + r ) (R 2 + r) 2
P Q
R1 R2 condition for balanced bridge will be = , which
Þ 2
< R S
(R1 ∗ r) (R 2 ∗ r)2 is there: Hence balance point will remain unchanged,
where galvanometer shows no current.
On solving we shall get, r = R 1 R 2
Exercise 18.6
Exercise 18.4
2. (b) At the balance point, the potential difference (in the
1. (c) For a given difference in temperature of two junctions figure the balance point is J) between A & J is equal to
of a thermocouple, the thermo e.m.f. is more if the e.m.f. of experimental cell x. Hence is no current flow in
separation between two metals in the Seebeck series the circuit & there is no deflection in galvanometer.
used in the formation of thermocouple Sb and Bi are V K Rh
extreme metals of the series. B (. ) C
3. (b) When thermocouples are joined in series the thermo
e.m.fs set up in each thermocouple will help to increase
the current in the external load. Therefore, the total J1 J J2
thermo e.m.f. will be increased. A D
G
4. (b) The variation of thermo emf with temperature of hot I I G
junction follows parabolic path. G
7. (b) Here T i = 2T n, so T i is not a constant for given X(experimental cell)
thermocouple, but depends on the temperature of cold Potentiometer
junction. 3. (b) Sensitivity of potentiometer means the smallest
E.m.f
potential difference it can measure. It can be increased
by reducing the potential gradient. The same is
neutral possible by increasing the length of the potentiometer.
temperature
Exercise 18.7
(inversion . 4. (d) Connecting a resistance in parallel will reduce the
Ti temperature) effective resistance of combination but will not reduce
o T the range. In order to reduce the range, the resistance
Tn
9. (b) The relation between e.m.f & temperature T is in series with the galvanometer is to be decreased.
The same cannot be achieved by connecting a
dE resistance in parallel with the voltmeter.
E = a T + bT2, where a & b are constants. & =S
dT π nBA
is called thermoelectric power & it is zero at neutral 5. (b) Current sensitivity Ιs < < ; which is
i k
temperature (Tn = –a/2b) independent of current i.
11. (a) Peltier coefficient is directly proportional to absolute temp. 6. (c) An electrometer is used to measure the potential
T. difference very accurately at it does not draw any
12. (d) The joule effect is irreversible, i.e., the heat in a current.
conductor is always evolved whatever be the direction
of current. Peltier effect on the other hand is reversible. π nBA nBοr2
7. (b) Sensitivity, ΙS < < < .
If on passing the current in one direction, heat is i C C
evolved at a junction, then on reversing the current,
heat is absorbed at the same junction. Exercise 1 : NCERT Based Questions
1. Resistivity will not change
Exercise 18.5 3. It becomes 4 times.
3. (d) P R
R R 4. 14 Volt. 5. R2 > R1 6. 3:2 8. =
R Q S
R R 9.
Resistivity r(10 W m)
–8

0.4
I

0.2
Hint Þ since bridge is balanced hence equivalent
1
resistance is = (½R + ½R) Þ R' = R
R¢ O 50 100 150
Temperature T(K)
Free eBooks on @neetquestionpaper2020

Current Electricity 675

æl ö r l1
10. r = ç 1 –1÷ ´ R. 7. (c) R= , now l2 = 2l1
l
è 2 ø A1
12. (i) 15 × 106 W ; (ii) 5 : 1 A 2 = p(r2)2 = p (2r1)2 = 4p r12 = 4A1
14. Current through 4.5 W = 0.5 A .
r(2 l1 ) r l R
\ R2 = = =
1 1 4A1 2A 2
I1 = A and I2= A.
3 6
\ Resistance is halved, but specific resistance
17. 7.5 × 10–7 Wm remains the same.
18. (c) 19. (d) 20. (d) 21. (d)
8. (d) A balanced Wheatstone’s bridge exists between
22. (d) 23. (a) 24. (d) 25. (d)
A & B. Req = R
Exercise 2 : PAST Competition MCQs current through circuit is V/R
1. (d) Power of heating coil = 100 W and voltage current through AFCEB = V/2R
(V) = 220 volts. When the heating coil is cut into two 9. (a) Efficiency is given by
equal parts and these parts are joined in parallel, then
the resistance of the coil is reduced to one-fourth of output Power 5 ´ 15 ´ 14
h= = = 8.75 or 87.5 %
the previous value. Therefore energy liberated per input Power 10 ´ 8 ´ 15
second becomes 4 times i.e., 4 × 100 = 400 J.
2. (a) Hint : The wheatstone bridge is balanced, when
1 1 1 1 1 3
10. (a) = + + or = Þ Peq = 20 watt.
3 4 Peq P1 P2 P3 Peq 60
P/Q = R/S, In the this case = , so bridge is balanced
6 8
& 7W resistance is not effective) V2 V 2 (220) 2 4(110) 2
11. (d) P= ÞR = = = ;
VA - VB i ´ r 0.1 ´ 10-7 R P 60 60
3. (a) Potential gradient = = =
l A 10-6 (110) 2 R
R¢ = =
60 4
= 10 -2 V / m
4. (a) As for an electric appliance R = (Vs2 /P) , so for same 12. (d) It is balanced Wheatstone bridge. Hence bridge 4W
can be eliminated.
specified voltage Vs ,
6 ´ 9 18 V 5V
R 25 100
= = 4 i.e, R = 4R with R \ R eq = = \ I = R = 18
25 100 = R
6+9 5 eq
R100 25
Now, in series potential divides in proportion to 13. (a) P 1
R = 2 = = 0.04W
resistance. I 25

R1 4 P 100 ´103
So, V1 = V i.e., V25 = 5 ´ 440 = 352V 105
14. (c) I = = A= A
(R1 + R 2 ) V 125 60
E.C.E. = 0.367 × 10–6 kg C –1
R2 1
and, V2 = V i.e., V100 = ´ 440 = 88V Charge per minute
(R 1 + R 2 ) 5
105 ´ 60 6 ´106
From this it is clear that voltage across 100 W bulb = (I × 60) C = C= C
(= 88 V) is lesser than specified (220 V) while across 25 125 125
W bulb (=352 V) is greater than specified (220 V), so
6 ´ 106
25 W bulb will fuse. \ Mass liberated = ´ 0.367 ´10-6
125
10 ´ 40 400
5. (b) Time = = = 8 min 6 ´ 1000 ´ 0.367 ´10-3
10 + 40 50 =
125
1 = 17.616 × 10–3 kg
6. (a) Power µ
Resistance 15. (d) Current from D to C = 1A
In series combination, Resistance doubles \ VD – VC = 2 × 1 = 2V
Hence, Power will be halved. VA = 0 \ VC = 1V, \ VD – VC = 2
In parallel combination, resistance halved Þ VD – 1 = 2 \ VD = 3V
Hence, power will be double. \ VD – VB = 2 \ 3 – VB = 2 \ VB = 1V
Free eBooks on @neetquestionpaper2020

676 Physi cs
16. (c) The power dissipated in the circuit. 19. (b) R1
2 G
V
P= ...(i)
Req R VB
VA
v = 10 volt
1 1 1 5+ R
= + = Since deflection in galvanometer is zero so current
Req R 5 5R will flow as shown in the above diagram.
æ 5R ö VA 12 12
Req = çè ÷ current I = = =
5 + Rø R1 + R 500 + 100 600
P = 30 W
Substituting the values in equation (i) 12
So VB = IR = ´ 100 = 2V
600
(10) 2
30 = 20. (c) Resistance of bulb is constant
æ 5R ö
çè ÷ Dp 2DV DR
5 + Rø V2 = +
P= Þ
R p V R
15 R
= 10 Þ 15R = 50 + 10R
5+ R Dp
5R = 50 Þ R = 10 W p
= 2 × 2.5 + 0 = 5%
17. (c) The current through the resistance R
æ e ö
I =ç
è R + r ÷ø rl
21. (c) Resistance R =
The potential difference across R A
Q l¢ = 2 l
æ e ö
V = IR = ç R
è R + r ÷ø A
\ A¢ =
I e r 2
2l
\ R¢ r A = 4R = 4 × 4 W = 16 W
2
Therefore the resistance of new wire becomes 16 W
R 22. (a) Given : emf e = 2.1 V
I = 0.2 A, R = 10W
e e Internal resistance r = ?
V= V
æ rö From formula.
çè1 + ÷ø e – Ir = V = IR
R
0 2.1 – 0.2r = 0.2 × 10
R 2.1 – 0.2 r = 2 or 0.2 r = 0.1
when R = 0, V = 0,
R = ¥, v = e 0.1
Þ r= = 0.5 W
Thus V increases as R increases upto certain limit, but 0.2
it does not increase further. e 2.1
18. (a) Galvanometer is converted into ammeter, by connected ALTERNATE : i = Þ 0.2 =
a shunt, in parallel with it. r+R r + 10
1
G Þ 2.1 = 0.2 r + 2 Þ r = = 0.5 W
I 2
23. (a) Given : V = 7 V
r = 5W
P Q
S

GS VG 25 ´ 10-3
==
G+ S I 25
GS 5W
= 0.001W
G+S 7V
Here S << G so S = 0.001 W
Free eBooks on @neetquestionpaper2020

Current Electricity 677

40 ´ 120 nR1R 2
Req = W S = nP Þ R1 + R 2 =
40 + 120 (R1 + R 2 )

V 7 Þ (R 1 + R 2 ) 2 = nR1R 2 n max value is 4


I= =
R 40 ´ 120
5+
40 + 120 for that (R1 + R 2 ) 2 = 4R 1R 2 Þ (R1 - R 2 ) 2 = 0

7 1
= = = 0.2 A.
5 + 30 5

V2 V2 V2 34. (b)
24. (b) P1 = V2/R; P2 = + =4 = 4P1.
(R / 2) (R / 2) R V

V2 (15) 2 (15) 2
25. (b) W = ; 150 = + Þ R = 6 W. rl rl 1 r l2
R net R 2 R1 = =
2
; R2 = 2
A pr1 pr2
26. (b) From Faradays law of electrolysis, m µ It.
27. (c) l1 4 r 2
28. (c) Potential µ R Given : = ; 1 =
l2 3 r2 3
R µ length Þ Potential difference µ l
V V i1 V R
l¢ 30 = i1 , = i2 Þ i = = 2
E¢ = E= E. R1 R2 V R1
l 100 2 R1 ´
R2
29. (b)
3W 3W 2
6W R (l ) ´ pr12 æ l 2 ö æ r1 ö 3 4 1
2W Þ 2 = 2 = ´ = ´ =
3W 3W = R1 (l1 ) ´ pr22 çè l1 ÷ø çè r2 ÷ø 4 9 3
=
3V 3V
3V R1 l 1
35. (c) = where l2 = 100 l1
3 R2 l2
Þ I= = 1. 5 A
2
30. (c) X 20
in the first case =
Y 80
V2 (220) 2
31. (c) P= or R = , 4X l
R 1000 in the second case = Þ l = 50
Y 20 - l
V2
Power consumed = 36. (a) DQ = mC p ´ DT = 1´ 4180 ´ (40 - 10)
R
= 4180 × 30
110 ´ 110
= ´ 1000 = 250 watt ( \ DQ = heat supplied in time t for heating 1L water
220 ´ 220 from 10°C to 40°C)
6V 6V 4180 ´ 30
also DQ = 836 × t Þ t = = 150 s
836
3/2 3/2 3 37. (d) Neutral temperature is the temperature of a hot junction
32. (a)
at which E is maximum
3 3
dE
Þ = 0 or a + 2bq = 0
6 dq
hence Req = 3/2 \I = = 4A
3/2
-a d2E
R1 R2 Þq = = -350 = 2b
33. (c) 2b dq
R1R 2 hence no q is possible for E to be maximum, no neutral
S = R1 + R 2 ; P = temperature is possible.
R1 + R 2
Free eBooks on @neetquestionpaper2020

678 Physi cs
38. (c) The mass liberated m, electrochemical equivalent of a 44. (d) At cold junction, current flows from Antimony to
metal Z, are related as Bismuth (because current flows from metal occurring
m = Zit Þ m = 3.3 ´10 -7 ´ 3 ´ 2 = 19.8 ´ 10 -7 kg. later in the series to metal occurring earlier in the
thermoelectric series).
2e 45. (d) Kirchhoff's first law is based on conservation of charge
39. (c) I = R1 R2
R + R1 + R 2 I and Kirchhoff's second law is based on conservation
of energy.
r B l B rA l A
46. (d) rB = 2rA; dB = 2dA; RB = RA Þ A = A
B A

2
l rA d B r 4d 2
R \ B = ´ = A ´ A =2
Pot. difference across second cell = V = e - IR 2 = 0 lA rB d 2A 2rA d 2A
2e 47. (b) The network of resistors is a balanced wheatstone
e = .R2 = 0
R + R1 + R 2 bridge. The equivalent current is
30W
R + R1 + R 2 - 2R 2 = 0
R + R1 - R 2 = 0 \ R = R 2 - R1
15W
E
40. (d) I = , Internal resistance (r) is zero,
R+r
E
I= = constant. 5V
R
15 ´ 30 V 5
Req = = 10 W Þ I = = = 0.5 A
V2 t 15 + 30 R 10
41. (b) H =
R 48. (a) R2 = R1 (1 + aDT)
R Þ 200 = 100[1 + 0.005DT] Þ 1 = 0.005DT
Resistance of half the coil =
2
1 1000
\ As R reduces to half, ‘H’ will be doubled. Þ DT = = = 200°C
42. (a) Mass deposited 0.005 5
Þ Final temperature = 300°C
1
m = zq Þ z µ NOTE : We may use this expression as an
q approximation because the difference in the answers
z1 q 2 is appreciable. For accurate results one should use
Þ = ............ (i) R = R0eaDT
z 2 q1
P R S1S 2
Also q = q1 + q 2 ............ (ii) 49. (b) = where S = +
Q S S1 S 2
q q
Þ = 1 +1 (Dividing (ii) by q 2 ) 50. (d) We know that, Rt = R0 (1 + at ),
q2 q2
where Rt is the resistance of the wire at tºC,
q R0 is the resistance of the wire at 0ºC
Þ q2 = q
............ (iii)
and a is the temperature coefficient of resistance.
1+ 1
q2 Þ R50 = R0 (1 + 50 a ) ... (i)
R100 = R0 (1 + 100 a ) ... (ii)
q
From equations (i) and (iii), q 2 = From (i), R50 – R0 = 50 aR 0 ... (iii)
z
1+ 2
z1 From (ii), R100 – R0 = 100 aR 0 ... (iv)

V2 R 50 - R 0 1
43. (b) P = Vi = Dividing (iii) by (iv), we get, =
R R100 - R 0 2

V2 200´ 200 Here, R50 = 5W and R100 = 6W


R hot = = = 400W
P 100 5 - R0 1
\ = or 6 – R0 = 10 – 2 R0 or R0 = 4W
400 6 - R0 2
R cold = = 40 W
10
Free eBooks on @neetquestionpaper2020

Current Electricity 679

51. (b) According to the condition of balancing


1 1 1
55 R Þ = +
= Þ R = 220W R0 R0 (1 + a1Dt ) R0 (1 + a 2 Dt )
20 80 (1 + aeq Dt )
2
52. (a) Let j be the current density.
2(1 - a eq Dt ) = (1 - a1Dt )(1 - a 2 Dt )
I rI
2
Then j ´ 2pr = I Þ j = \ E = rj =
2pr 2 2pr 2 a1 + a 2
\ a eq =
2
'
a r uur a
rI
Now, DVBC =- ò E. dr = - ò 2
dr 56. (a) Resistance of wire
a+b a + b 2 pr
rl rl 2
a R= = (where Al = C)
rI é 1 ù rI rI A C
=- ê - ú = -
2p ë r û a + b 2pa 2p (a + b) \ Fractional change in resistance
On applying superposition as mentioned we get DR Dl
=2
rI rI R l
'
DVBC = 2 ´ DVBC = -
pa p(a + b) \ Resistance will increase by 0.2%
57. (c) The current upto which bulb of marked 25W -220V,
rI
53. (c) As shown in Answer 52. E = W1 25
2pr 2 will not fuse I1 = V = 220 Amp
1
54. (c) Applying kirchoff's loop law in AB P2P1A, we get
-2i + 5 - 10 i1 = 0 .....(i) W2 100
Similarly, I2 = V = 220 Amp
2
B i P2 i–i1 C
i1 The current flowing through the circuit

10W 2V
1W B1 B2
5V
2W
R1 r2
A P1 D

Again applying kirchoff's loop law in P2 CDP1P2, we 440V


get
10 i1 + 2 – i + i1= 0 .....(ii)
440
I=
é 5 - 10i1 ù Reff
From (i) and (ii) 11 i1 + 2 - ê ú =0
ë 2 û
Reff = R1 + R2
1
Þ i1 = A from P2 to P1 V12 (220)2
32 V 2 (220)2
R1 = = ; R2 = 2 =
P1 25 P 100
55. (d) R1 = R0 [1 + a1Dt ] ; R2 = R0 [1 + a2 D t ]
In Series, R = R1 + R2 440 440
I= 2 2
=
(220) (220) é 1 1 ù
+ (220)2 ê +
é æ a1 + a 2 ö ù ë 25 100 úû
= R0 [ 2 + (a1 + a 2 )Dt ] = 2 R0 ê1 + ç ÷ Dt ú 25 100
ë è 2 ø û
40
a + a2 I= Amp
\ a eq = 1 220
2
1 1 1 1 1 æ 25 ö æ 40 ö æ 100 ö
In Parallel , = + = + Q I1 ç = A÷ < I ç = A ÷ < I2 ç = A÷
R R1 R2 R0 [1+a1Dt ] R0 [1+a2Dt ] è 220 ø è 220 ø è 200 ø
Thus the bulb marked 25W-220 will fuse.
Free eBooks on @neetquestionpaper2020

680 Physi cs

58. (d) \ The potential difference remains the same between


C and D; and between A and B.
6W \ (I – I1) (4 + 6) = I1 × 5 on solving using I1 from (i) we
Bulb
get
(Lead) (I – 2.9) 10 = 2.9 × 5
\ I – 2.9 = 1.45
\ I = 4.35 A
Heat released/sec in 4W resistance will be
= (4.35 – 2.9)2 × 4
= 8.4 J/s = 2 cal/s
120 V
62. (b) For Max. external power
Power of bulb = 60 W (given)
120 ´ 120 é V2 ù 2R
Resistance of bulb = = 240W êQ P = ú R R
R
60 ë R û 6R
Power of heater = 240W (given) R 6R R
E 4W
4R
120 ´120 2R
Resistance of heater = = 60W R 4R
240
Voltage across bulb before heater is switched on, E 4W

240
V1 =
´ 120 = 117.73 volt R 2R
246 Balanced
Voltage across bulb after heater is switched on, Wheat store 2R 4R
Bridge
48
V2 = ´120 = 106.66 volt
54
Hence decrease in voltage E 4W
V1 – V2 = 117.073 – 106.66 = 10.04 Volt (approximately)
59. (d) Statements I is false and Statement II is true
IgG 3R
For ammeter, shunt resistance, S = 2R
I – Ig 6R
Therefore for I to increase, S should decrease, So
additional S can be connected across it.
rl E 4W
60. (b) R= 2 E
pr
Resistance = Internal Resistance
l r
When l is and radius is , 2R = 4 Þ R = 2
2 2
V R
63. (c)
rl4 2rl F E
\ R' = 2
= 2
I
p2r pr
So, R' = 2R. So, heat is doubled according to C
V
H = I2RT. A D
0V G
61. (b) Let I1 be the current flowing in 5W resistance and
I
(I – I1) in 4W and 6W resistance. The heat generated in
5W resister is 10 cal/s = 4.2 × 10 J/s B C
2V 2R
(I – I 1) 4 W 6W
A B
V
I I Applying Kirchhoff’s law in BCDEFAB we get, I =
3R
5W Let A be at 0 V. Then potential at G is V.
D C Applying Krichhoff’s law for AFED, we get
I1
V 4V
\ 4.2 ´ 10 = I12 R 0 + V + IR = VD Þ 0 + V + ´ R = VD Þ VD =
3R 3
4.2 ´ 10
\ I1 = = 8.4 = 2.9 amp. ...(i) 4V V
5 \ potential different across capacitor = -V =
3 3
Since AB and CD are in parallel.
Free eBooks on @neetquestionpaper2020

Current Electricity 681


64. (a) The circuit is symmetrical about axis POQ. Therefore [Since, IAB = IBC, wire is of same material]
the equivalent circuit is drawn Therefore, option (a) is wrong.
1 1 1 1 1 1 R+r l
\ = + + = + = r
R PQ 4R 4R 2r 2R 2r 2Rr PBC I R BC 2
2[p ´ 4r 2 ] 1
(b) = = = Þ PAB = 4PBC ;
PAB I 2 R AB l 4
2R A 2R r 2
2[pr ]
2R
r O r
2Rr Therefore, (b) is correct.
Þ R PQ = P Q
R+r 2R
I
2R B 2R 2
(c) AB = p ´ 4r = 1 ; Therefore, (c) is incorrect.
J
If a potential difference is applied across P and Q, J BC I 4
there will be no currents in arms AO and OB. So these
p´ r2
resistance will be ineffective.
2R 2R é VAB ù
E AB êë l / 2 úû 1
(d) = = ; Therefore, (d) is incorrect.
P r r Q E BC é VBC ù 4
ê l/2 ú
ë û
2R 2R
70. (a) Given X is greater than 2W when the bridge is balanced
65. (a) Ammeter is always connected in series and voltmeter
is always connected in parallel. R X
=
l 100 - l
R1 R 3 x
66. (a) At null point, R = R = 100 - x 200
2 4 Þ 100 R – Rl = lx Þ 200 – 2l = lx Þ l =
If radius of the wire is doubled, then the resistance of X+2
AC will change and also the resistance of CB will
change. But since R1/R2 does not change so, R3/R4
should also not change at null point. Therefore the
point C does not change.

R1 R2
When the resistances are interchanged the jockey shift
G
20 cm. Therefore
A x C R4 B
X 2
=
67. (c) For various combinations equivalent resistance is l + 20 80 - l
maximum between P and Q. 80X – lX – = 2l + 40
68. (a) The current in 2W resistor will be zero because it is not
Þ 80X = l (X + 2) + 40
a part of any closed loop.
200
Þ 80X = ( X + 2 ) + 40
5W 10 W
( + 2)
X
10 V 20 V
2W 240
Þ X= = 3W
A B 80
C
71. (a) Cells connected in series
69. (b) 2r r
R R
l/2
l/2
l
r I
VAB I AB R AB R AB 2[p ´ 4r 2 ] 1
(a) = = = =
VBC I BC R BC R BC l 4
r 2
2[pr ] r r
E E 2E 2r
Free eBooks on @neetquestionpaper2020

682 Physi cs

æ 2E ö
2
12 é 6 ´ 12 ù
J1 = I 2 R = ç .R I1 = êëQ Req = 6 + 12 úû
è 2r + R ÷ø
...(1) 4
Cells connected in parallel
= 3A
é 12 ù
R \ I2 = 3 ê = 2A
R
ë 6 + 12 úû
As P and Q are equipotential and potential at S is less
I than the potential at P (potential drops across a
E r
resistance as current passes through it), therefore
VS < VQ.
E r
E r 2 Exercise 3 : Conceptual & Applied MCQs

æ E ö
2 1. (c) Numbers attached for brown, black, green and silver
J2 = I 2R = ç ´R are 1, 0, 5, ± 10%. Therefore the resistance of given
r÷ ...(2)
çR+ ÷ resistor
è 2ø
Given J1 = 2.25 J2 = 10 ´ 10 5 W ± 10% = 1.0 ´ 10 6 W ± 10%.

(2 E )2 E2 V2 V2
. R = 2.25 .R 2. (b) R1 = and R 2 =
(2r + R )2 r P1 P2
( R + )2
2 R 2 P1 200
\ = = =2 ( Q V = constant)
4 2.25 R1 P2 100
\ =
(2r + R )2 æ rö
2
R 2 = 2 R1
çè R + ÷ø
2 l
3. (c) Rµ ;
2
\ 4[ R + 0.5] = 2.25[2 + R] 2
[Qr = 1W] A
\ 2 (R + 0.5) = 1.5 (2 + R) So, the resistance of the wire will be minimum when
\ R = 4W the area of cross-section is maximum and length is
72. (b) At Null point minimum.
V ærö
4. (a) Potential gradient of wire = = ç ÷´ I
X 10W l èAø
where l & A are the length and cross-section of wire
-7
so V = 4 ´ 10 ´ 0.5 = 25mV / meter
l 8 ´ 10-6
A B
52 cm 48 cm 5. (a)
6. (c) r = E / I = 1.5 / 3 = 0.5 ohm.
X 10 7. (a) Current flowing through the conductor,
=
l1 l 2 I = n e v A. Hence
2
Here l1 = 52 + End correction = 52 + 1 = 53 cm 4 nevd1 p(1) vd 4 ´ 1 16
= or 1 = = .
l 2 = 48 + End correction = 48 + 2 = 50 cm 1 nevd p(2) 2 vd 2 1 1
2
X 10 53
\ = \X = = 10.6W (1.5) 2
53 50 5 8. (d) Resistance of bulb R b = = 0.5 W
4.5
73. (a, b, c, d)
The given circuit is an extension of wheatstone bridge, E E
therefore points P and Q are at the same potential and Current drawn from battery = =
2.67 + 0.33 3
point S and T are also at the same potential. Therefore
no current passes through PQ and ST and the circuit 2 E 2E
Share of bulb = ´ =
reduces to as shown 3 3 9
6W 2
æ 2E ö
I2 \ ç ÷ ´ 0.5 = 4.5 or E = 13.5 V..
è 9 ø
I1 – I2 9. (c) As we more from A to B the potential difference across
12 W
AC increases and hence the reading of voltmeter also
I1 increases. But currnet flowing through ammeter
remains almost constant.
12V
Free eBooks on @neetquestionpaper2020

Current Electricity 683


10. (c) Current cannot flow through the 50-W resistor. It can 23. (d) Case (I) : E + E = (r + r + 5) 1 or 2 E = 2 r + 5 ...(i)
therefore be removed from the circuit.
æ r´r ö ær ö
(since it is reverse biased). Case (II) : E = ç + 5 ÷ ´ 0.8 or E = ç + 5 ÷ 0.8
è r + r ø è2 ø
50 W or E = 0.4 r + 4.0 ...(ii)
Multiplying (ii) by 2 and equating with (i), we get
3
2 r + 5 = 0.8 r + 8 or 1.2 r = 3 or r = = 2.5
1.2
6V 100 W 24. (a) [Hint Þ Rt = Ro (1 + a t)]
\ current through 100W resistor 5W = R0 (1 + a × 50) and 7W = R0 (1 + a × 100)
5 1 + 50a 2
11. (b) R1 V 2 / P1 P2 or = or a = = 0.0133/ °C
= = 7 1 + 100a 150
R 2 V 2 / P2 P1
25. (b) (Hint Þ r = R .A = Coefficient of resistivity)
rl / p(d1 / 2) 2
d 22 100W l
= = =
rl / p(d 2 / 2) 2
d12 25W Pot.Difference VA - VB
=
26. (a) Hint : Potential gradient =
length of wire l
d 2 10 2
Þ = = 27. (a)
d1 5 1 28. (c) Kirchhoff’s junction rule states that the algebraic sum
12. (b) of all currents into and out of any branch point is zero :
13. (b) SI = 0. By convention, the sign of current entering a
14. (b) Current density J = I/A junction is positive and current leaving a junction is
negative.
= 50 ´16 -6 / 50 ´10 -6 = 1 Am -2 4A + 5 A – 6A + IAB = 0, therefore IAB = – 3A. The wire
between points A and B carries a current of 3A away
15. (d) I = n A e vd or vd µ 1/ p r 2
from the junction.
16. (c) I = q / t = 107 ´1.6 ´ 10-19 / 1 = 1.6 ´ 10 -12 A 29. (a) Pot. gradient = 0.2mV/cm
0.2 ´10 - 3
17. (b) Current, I = (2.9 ´ 1018 + 1.2 ´1018 ) × 1.6 × 10–19 = = 2 ´ 10 - 2 V / m
10 - 2
towards right.
Emf of cell = 2×10–2×1m = 2 ´ 10 - 2 V = 0.02 V
q CV (10 ´ 106 ) ´ 40
18. (a) Current I = = = = 2 ´ 10 -3 A As per the condition of potentiometer
t t 0. 2 0.02 (R + 490) = 2 (R) or 1.98 R = 9.8
dQ 9.8
19. (c) I= = 10t + 3 Þ R= = 4.9 W
dt 1.98
At t = 5s, I = 10 × 5 + 3 = 53 A 30. (b) Case (I) : When resistor is not connected
rl l Using V = IR Þ V = 25 (R G ) .............. (i)
20. (c) R= or R µ . Case (II) : When resistor is connected
2
( p D / 4) D2
V = 5(20 + R G ) = 100 + 5 R G ............. (ii)
Rx lx D y2 ly / 2 D y2 2 From (i) and (ii), 20 R G = 100 Þ R G = 5 W
= ´ = ´ =
R y Dx 2 l y (D y / 2)2 l y 1 31. (a) Rg = 50W, Ig = 25 × 4 × 10–4W = 10–2 A
Range of V = 25 volts
1 1 1
21. (c) R 1 = W; R 2 = W; R 3 = W V = Ig(HR + Rg)
2 4 6
V
\ HR = - R g = 2450W
In series; R s = R 1 + R 2 + R 3 = 1 + 1 + 1 = 11 Ig
2 4 6 12 R
1 æ 12 ö A B
\ ss = =ç ÷
R s è 11 ø
Ig HR Rg
22. (a) R1 = 1 / 2 W, R 2 = 1 / 4 W; R 3 = 1 / 6 W
32. (a) J = sE Þ J r = E
1 1 1 1 J is current density, E is electric field
In parallel; = + + = 2 + 4 + 6 = 12
R p R1 R 2 R 3 so B = r = resistivity.
Ig R Ig
1 33. (b) S= ÞS= R
\ sp = = 12 S nIg - I g (n - 1)I g
Rp
Free eBooks on @neetquestionpaper2020

684 Physi cs
34. (c) Internal resistance = r, External resistance = nr. 2 1
Let terminal voltage = V or, = or R = 800 - 40 = 760 W
40 + R 400
Er 44. (c) As R= V2/P or R µ 1/P
then V = E - Ir Þ V = E -
(n + 1)r so R2/R1 = P1 /P2 = 200/100 = 2
or R2 = 2 R1.
nE V n 45. (b) As R µ V2/P or R µ 1/P, so resistance of heater is less
V= Þ =
n +1 E n +1 than that of fan.
35. (a) 36. (c) 46. (b) If R is the resistance of each wire, total resistance in
37. (a) series = R + R = 2 R; and total resistance in parallel
Ig I G R´R R
38. (d) S = ´G = ´G = = = .
I - Ig ( nI - I ) ( - 1)
n R +R 2
Heat produced per second (= V2/R) will be four times
39. (c) The effective circuit will be as shown in the figure. in parallel than in series.
47. (b) For maximum current, the two batteries should be
connected in series. The current will be maximum when
external resistance is equal to the total internal
resistance of cells i.e. 2 W. Hence power developed
across the resistance R will be
2
æ 2E ö 2´2 ö
= I 2 R = çç ÷÷ R = æç ÷ ´ 2 = 2 W.
Effective resistance of R2 and R4 in series, è R + 2r ø è2+2ø
R' = 10 + 10 = 20 W. 48. (b) Energy produced = V I t = 6 × 0.5 × 1 = 3.0 J.
Effective resistance of R3 and R5 in series, 49. (c) Mass of substance deposited
R'' = 10 + 10 = 20 W m = Z I t = Z ´ 4 ´ (2 ´ 60) = 480 Z and
Net total resistance of R' and R'' in parallel is
m ¢ = Z ´ 6 ´ 40 = 240 Z or m ¢ = m / 2.
20 ´ 20 50. (a) According to maximum power theorem, the power in
R1 = = 10W.
20 + 20 the circuit is maximum if the value of external resistance
\Total resistance between A and D is equal to the internal resistance of battery.
= 10 + 10 + 10 = 30 W.
V2t
40. (c) Resistance of a wire = rl / A. 51. (d) Heat produced, H = . When voltage is halved,
R
For the same length and same material, the heat produced becomes one fourth. Hence time
R 2 A1 3 taken to heat the water becomes four time.
= = or, R 2 = 3R1 52. (c) Chemical energy consumed per sec
R1 A 2 1
= heat energy produced per sec.
The resistance of thick wire, R1 = 10 W
The resistance of thin wire = 3R1 = 3 × 10 = 30 W. = Ι2 (R + r) = (0.2)2 (21 + 4) = 1Js -1 .
Total resistance = 10 + 30 = 40W. V2 V2
P l 53. (b) H= ´ 15 ´ 60 = ´t
= l 20 R (2/3)R
41. (d) or P = ´Q = ´ 1 = 0.25W.
Q (100 - l ) 100 - l 80 2
42. (b) Rt = R0 (1 + at) or t = ´ 15 ´ 60 = 600 s = 10 minutes.
3
Initially, R0 (1 + 30a) = 10 W 54. (b) Energy consumed per day = P × t = 60 × 8 = 480 watt
Finally, R0 (1 + at) = 11 W hour = 480/1000 = 0.48 kWh or unit of electricity.
11 1 + at Hence the cost = 0.48 × 1.25 = Re 0.60.
\ = 55. (a) Electrochemical equivalent,
10 1 + 30a
or, 10 + (10 × 0.002 × t) = 11 + 330 × 0.002 m 4.572
Z= = = 3.387 ´ 10-4 g/C.
Ι t 5 ´ 45 ´ 60
1.66
or, 0.02t = 1 + 0.66 = 1.066 or t = = 86°C. 56. (c) Neutral temperature,
0.02
43. (b) Potential gradient along wire qi + q0 530 + 10
qn = = = 270º C.
potential difference along wire 2 2
=
length of wire 57. (b) As P = Ι 2 R, so P1 = (1.01 Ι)2 R = 1.02 I 2 R = 1.02 P.
It means % increase in power
I ´ 40
or, 0.1 ´10 -3 = V / cm æP ö
= ç 1 - 1÷ ´ 100 = 2%.
1000 P
è ø
1 2 P R 2
or, Current in wire, I= A 58. (c) P = V / R or P µ 1 / R \ 1 = 2 = .
400 P2 R 1 1
Free eBooks on @neetquestionpaper2020

Current Electricity 685


59. (c) (Hint : m = Z it, where Z is electro chemical equivalent) will flow through the arm KL. Equivalent resistance
60. (d) (Hint : P = V2/R) between
61. (a) Galvani made the statement “Chemical change can AKM = R + R = 2 R
produce electricity”. Equivalent resistance between ALM = R + R = 2 R
62. (a) Joule effect H = I2RT The two resistances are in parallel. Hence equivalent
When current flows heat is produced. But, by heating resistance between A and B is given by
conduction current cannot produced. K
63. (b)
R R
l
64. (a) H = I2 Rt. Here R1 = r and
p r2 A M B
R
l H
R 2 =r That is, R1 = 4R2. Hence, 1 = 4
p ( 2r )
2 H2
R R
65. (a) m = ZIt.
66. (c) Knowledge based question. L
1 1 1 2 1
67. (a) Let the resistance of single copper wire be R1. If r is = + = =
the specific resistance of copper wire, then R¢ 2 R 2 R 2 R R
r´ l r´l i.e., R ¢ = R
R1 = = 2 ...(1) 71. (a) The given figure is a circuit of balanced Wheatstone’s
A1 p r1 bridge. Point B and D would be at the same potential
When the wire is replaced by six wires, let the i.e., potential difference between these points is zero.
resistance of each wire be R2. Then 72. (c) With each rotation, charge Q crosses any fixed point
r´l r´l P near the ring. Number of rotations per second
R2 = = 2 ...(2)
A2 p r2 = w/2p.
From eqs. (1) and (2), we get Qw
\ charge crossing P per second = current =
2p
R1 r2 2 5 (3 ´103 ) 2
= or = (b) Let the edges be 2l, a, and l, in decreasing order.
R 2 (9 ´10 -3 ) 2 ; R 2 = 45 W
73.
R 2 r12
These six wires are in parallel. Hence the resistance of 2l 2r l r
R max = r = , R min = r = ;
the combination would be R 2 = 7.5 W al a 2 la 2 a
68. (b) In case of internal resistance measurement by R max
potentiometer, = 4.
R min
V1 l 1 {E R1 /(R1 + r)} R 1 (R 2 + r)
= = = 74. (a) As the ring has no resistance, the three resistances of
V2 l 2 {E R 2 /(R 2 + r )} R 2 (R1 + r) 3R each are in parallel.
Here l 1 = 2 m, l 2 = 3 m, R1 = 5 W and R 2 = 10 W 1 1 1 1 1
Þ = + + = Þ R¢ = R
2 5 (10 + r ) R ¢ 3R 3R 3R R
\ = or 20 + 4 r = 30 + 3 r or r = 10 W
3 10 (5 + r ) \ between point A and B equivalent resistance
69. (a) Let, we connect 24 cells in n rows of m cells, then if I is = R+ R = 2R
the current in external circuit then
75. (b) J = I / A = 10-4 /(0.30 ´ 0.50).
mE
I= ...(1)
mr / n + R = 6.7 ´ 10-4 C m -2 / s = 6.7 ´10 -4 Am -2
For I to be maximum, (mr + nR) should be minimum.
r l ¢ r ( l / 2) 1
mr 76. (a) R = r l / A; R ¢ = = = R.
It is minimum for R = ...(2) A¢ 2A 4
n
So maximum current in external circuit is 77. (b) Resistance of the wire of a semicircle = 12/2 = 6W
mE For equivalent resistance between two points on any
I= ...(3) diameter, 6W and 6W are in parallel.
2R
or
m If a wire of resistance R is bent in the form of a circle,
here R =3, r = 0.5 so equation (2) become =6
n the effective resistance between the ends of a diameter
so n = 2, m = 12 = R/4.
70. (a) The equivalent circuit is shown in fig. Since the
Wheatstone’s bridge is balanced, therefore no current
Free eBooks on @neetquestionpaper2020

686 Physi cs

2 1 because in null point position, E = V


78. (d) Current in the potentiometer, I = = A.
8 + 7 +1 8 E1 l1 R1
\ = = ..............(1)
1 E 2 l2 R 2
Voltage drop across potentiometer wire = ´ 8 = 1 V
8 For internal resistance, E = E1, r = E 2
\ Potential gradient of potentiometer wire E = iR + ir
1 æE ö
= = 0.25 V / m
4 E - V = ir (Q i = V / R ) r = ç - 1÷ R
èV ø
2 2
R1 æ l1 ö R æ 1ö æl ö æ 55 ö æ 5 ö
79. (c) = Þ = ç ÷ Þ r = çç 1 - 1÷÷R . = 10ç - 1÷ = 10ç ÷ =1W
R 2 çè l 2 ÷ø R ¢ è 2ø è l2 ø è 50 ø è 50 ø
83. (d) Let potential of X be x.
Þ R ¢ = 4R or DR = 3R Now current from AX and BX will pass through XC.
\ % change in resistance of wire 8K –4V
+2V 2K X
3R
= ´100 = 300%, A B
R
80. (d) R1 + R2 = Constant, R1 will increase, R2 will decrease. 4K
R1aD T - RbDT = 0 Þ R1aDT = R 2bD T
–8V
R b C
\ 1 =
R2 a so 2 - x + -4 - x = x + 8
81. (b) 2 8 4
A 30W B -4 - x x x x
I1 40W 40V 2- x + = + 4 - x -1 - = + 2
C 4 2 4 2
F
I2 I3 5x x 7x 12
40W - - =3 - = 3 Þ or x = -
E D 4 2 4 7
80V 12
Applying 1st law of Kirchoff’s, I3 = I1 + I2 … (i) - +8
7 44 11
Applying 2nd law of Kirchhoff So current in 4K will be = = = 1.57 A
in Mesh ABCFA 4 28 7
84. (b) The equivalent circuit can be redrawn as
-30I1 + 40 - 40I3 = 0 … (ii)
R R
in Mesh FCDEF
R R
40I3 - 40 - 80 + 40I2 = 0 … (iii) A· · B º ·
R R 2R
putting I3 from (i) in (ii) and (iii) we get R R
R R
-30I1 + 40 - 40I1 - 40I 2 = 0 5R / 8
º 2R / 3 º º
or , - 70I1 - 40I 2 = -40
R 5R / 3
or, 7I1 + 4I2 = 4 … (iv) 85. (c) At steady state the capacitor will be fully charged and
and 40I1 + 40I 2 - 40 - 80 + 40I 2 = 0 thus there will be no current in the 1W resistance. So
or , 40I1 + 80I 2 = 120 or , 4I1 + 8I 2 = 12 the effective circuit becomes
2W
or , 2I1 + 4I 2 = 6..............(v )
(iv ) - (v )
gives 5I1 = -2 I1 = -0.4 A
A
I1
B
82. (c) Length of wire = 100cm. I2 3W
2V = voltage of battery, 1V = voltage of a cell
When current flows along wire electric potential falls I
continuously along wire.
V 2V 6V 2.8W
100cm Net current from the 6V battery,
B 6 6 3
A I= = = = 1.5A
æ 2 ´ 3 ö 2.8 1.2 + 2.8 2
E 1V ç ÷+
è 2+3ø 1
Between A and B, voltage is same in both resistances,
2I1 = 3I 2 where I1 + I 2 = I = 1.5
E
Potential gradient = fall in potential length, K = Þ 2I1 = 3(1.5 - I1 ) Þ I1 = 0.9A
l
Free eBooks on @neetquestionpaper2020

Current Electricity 687


86. (d) R
E
I= potential across capacitor is equal to
R+r
R potential across resistance R1.
E
V1 = R 1 So charge = CV = ECR1
R R1 + r 1 R+r
93. (b) Given Capacitance of the capacitor, C = 0.2 µF and
R R e.m.f. of cell, E = 6V.
R R R
R
Reactance of a capacitor for a cell, which is a DC source,
R is infinity. Therefore, no current flows in 4W resistance.
Resistances 2 W and 3 W (both in upper arm) are
connected in parallel. Therefore, their equivalent
R
B resistance ( R ¢ ) = 2 ´ 3 = 1.2W
A
2+3
¢
Now, R and 2.8 W are in series combination.
R/3 Therefore, equivalent resistance of the circuit,
R = R ¢ + 2.8 = 1.2 + 2.8 = 4 W
E 6
R/3 R/3 Current drawn in the circuit, I = = = 1.5A
R 4
Therefore, potential difference across 2 W resistance,
V = IR = 1.5 × 1.2 = 1.8 V
A R/3 B
V 1.8
3R R Thus, current in 2 W resistance ( I1 ) = = = 0.9A
´ 2 2
R2
Rnet between AB = 3 3 = = 4W 94. (d) The equivalent circuit is as shown in figure.
3R R 4R The resistance of arm AOD (= R + R) is in parallel to
+ the resistance R of arm AD.
3 3
E E
87. (b,c) I = =
R + r ( R - r )2 + 2 R r
I is maximum when R = r
P = I 2 R , when I is max, P is also max.
Pmax = I 2max R .
88. (c) By principle of symmetry and superposition,
I R
2´ ´ R 0 = I R eq. Þ R eq. = 0
6 3
I Their effective resistance R1 = 2R ´ R = 2 R
(Current in AB is due to division in current entering 2R ´ R 3
6
The resistance of arms AB, BC and CD is
I
at A and current is due to current returning from 2 8
6 R2 = R + R + R = R
3 3
infinity of grid). The resistance R1 and R2 are in parallel. The effective
89. (c) resistance between A and D is
90. (d) Current in arm AE and FB is zero.
So, 2 8
R´ R
A R1 ´ R 2 3 3 = 8 R.
R3 = =
R1 + R 2 2 8 15
R+ R
E 3 3
95. (b) Equivalent resistance between A and B = series
combination of 1 W and 2 W in parallel with 3 W resistor..
C G H D
F

B
91. (b) m @ L, v @ I
92. (b) Current in branch of capacitor is zero.
Rnet = R + r
Free eBooks on @neetquestionpaper2020

688 Physi cs

1 1 1 2 dE T
= + = or R = 1.5 W. = 0 or 40 - = 0 or T = 400 o C.
R 3 3 3 dT 10
\ Current in the circuit is I = V/R = 1.5/1.5 = 1A. H1 P1 t P1 500 5
Since the resistance in arm ACB = resistance in arm AB 106. (a) In parallel, = = = =
= 3 W, the current divides equally in the two arms. Hence H 2 P2 t P2 200 2
the current through the 3 W resistor = I/2 = 0.5 A. H1 I 2 R 1 t R 1 V 2 /P1
96. (c) 97. (d) In series, = = =
H 2 I 2 R 2 t R 2 V 2 /P2
98. (a) 500W
A P2 200 2
= = = .
i P1 500 5
2V
R
12V V2 t
107. (c) H = m L = V 2 t / RJ or m =
JRL

10 1 (210) 2 ´ 1
= = 6.56 g / s.
12 – 2 = (500W) i Þ i = = 4.2 ´ 20 ´ 80
500 50
1´1
12 1 108. (b) Total internal resistance of two cells = = 0.5 W .
Again, i = = 1+1
500 + R 50 Since internal resistance of coil is equal to external
Þ 500 + R = 600 Þ R = 100 W resistance (= 0.5 W), hence power developed is
99. (a) The temperature of the wire increases to such a value maximum by cells in circuit.
at which, the heat produced per second equals heat
2
lost per second due to radiation i.e. Current through R = = 2A
0.5 + 0.5
æ rl ö
I 2 çç 2 ÷÷ = H ´ 2 p r l , where H is heat lost per Power = (2) 2 ´ 0.5 = 2 W.
è pr ø Vg 4 ´10 -4
2 3 109. (d) Vg = R g I g = 40´10-5 V ; q = = = 16º C.
second per unit area due to radiation. Hence, I µ r V 25 ´10 - 6
110. (d) At the heighest point O
I12 r13
so = or r2 = r1 (I 2 / I1 )2 / 3
I 22 r23
= 1´ (3.0 / 1.5) 2 / 3 = 41/ 3 mm.

100. (a) Resistance R = V2/P; so resistance of 40 W bulb is
more than that of 100 W bulb. In series, same current
flows. The heat produced H = I2 R t i.e. H µ R; so 40 B A
W bulb will be brighter. In parallel, heat produced, temperature
H = V2 t / R i.e. H µ 1/R; so 100 W bulb will be brighter.. E = maximum and A shows temperature of inversion
1 at which emf changes in sign.
101. (d) The energy stored in the capacitor = CV 2 ; This 111. (d) Let internal resistance of source = R
2
energy will be converted into heat in the resistor. V
102. (b) At neutral temperature, dE/dt = 0; Current in coil of resistance R1 = I1 =
R + R1
so 2164 – 6.2 × 2 × tn = 0
V
or tn = 174.5ºC. Current in coil of resistance R 2 = I 2 =
At temperature of inversion, E = 0 R + R2
Further, as heat generated is same, so
Þ 2164 ti – 6.2 t i2 = 0 Hence ti = 349ºC.
I12 R1 t = I 2 2 R 2 t
é T 2 TTr TT T 2 ù
103. (a) E = k ê TT0 - - - Tr T0 + r + r ú 2 2
ë 2 2 2 2 û æ V ö æ V ö
or çç ÷÷ R1 = çç ÷÷ R 2
dE é T T ù è R + R1 ø è R + R2 ø
Hence = k êT0 - T - r + r ú = k (T0 - T )
dT ë 2 2û Þ R1 (R + R 2 ) 2 = R 2 (R + R1 ) 2
At temp.T = T0 /2,
thermoelectric power = k (T0 – T0 /2) = k T0 /2. Þ R 2 R1 + R1R 22 + 2RR 1R 2
dE = R 2 R 2 + R 12 R 2 + 2RR 1 R 2 ?
104. (b) Thermo-electric power S = = a + bT
dT
Þ R 2 ( R 1 - R 2 ) = R 1R 2 ( R 1 - R 2 )
T2
105. (c) Given E = 40T – . At neutral temperature, Þ R= R 1R 2
20
Free eBooks on @neetquestionpaper2020

Current Electricity 689


112. (d) In series, Equivalent resistance = 3R 120. (a) PP = P1 + P2 and PS = P1 P2 / (P1 + P2)
2 2 Heat produced = Pt.
V V
Power = Þ 10 = Þ V2 = 30R
3R 3R Q P Pp 2000 W
Hence, = = = 4.
1 1 1 1 3 QS PS 500 W
In parallel, = + + =
R¢ R R R R 121. (b) Let I1 be the current throug 5 W resistance, I2 through
R (6 + 9) W resistance. Then as per question,
\ Equivalent resistance R ¢ =
3 I12 ´ 5 = 20 or, I1 = 2A.
V 2 30R Potential difference across C and D = 2 × 5 = 10V
\ Power = = = 90 W
R¢ R / 3 10 2
Current I 2 = = A.
H 6+9 3
113. (d) P = = s (T14 - T2 4 )
t Heat produced per second in 2 W
T1 = 727 + 273 = 1000 K, T2 = 227 + 273 = 500 K, 2
P = 60 watt æ8ö
= I2 R ç ÷ ´ 2 = 14.2cal / s.
In the second case, T1' = 1227 + 273 = 1500 K, è 3ø
T2 ' = 500 K, P' = ? 122. (c) In the case of a short-circuited battery, the current
P¢ (1500)4 - (500) 4 (500)4 [3 4 - 1] E (emf of the battery)
= = 80 I= ¹0
P (1000)4 - (500)4 (500)4 [2 4 - 1] Þ 15 r (internal resistance)
Terminal voltage V = IR = I (i) = I (0) = 0
80 where R = external resistance = 0
Þ P' = 60 ´ = 320 watt.
15 123. (c) Before the presence of electric field, the free electrons
114. (d) mCu = ZCuICut move randomly in the conductor, so their drift velocity
mAg = ZAgIAgt is zero and therefore there is no current in the
m Cu m Ag conductor. In the presence of electric field, each
I = I Cu + I Ag = + electron in the conductor experience a force in a
Z Cu t Z Ag t direction opposite to the electric field. Now the free
electrons are accelerated from negative and to the
ém m Ag ù positive end of the conductor and hence a current
Pt = Energy = VIt = V ê Cu + ú
êë Z Cu Z Ag úû starts to flow from the conductor.
124. (d) Potentiometer is preferred for measurement of emf of
é 1.8 1 ù cell. Potentiometer draws no current from cell while
= 12 ê -4
+ -4 ú voltmeter draws some current, therefore emf measured
ë 6.6 ´ 10 11.2 ´ 10 û
by voltmeter is slightly less than actual value of emf
=12 × 104 [0.362] = 4.34 × 104J.
of cell. Further the potentiometer is used with a
rl galvanometer which is set to null reading when the
115. (c) R=
A experiment is performed. The method of null reading
When wire is cut into 4 pieces and connected in avoids many errors.
parallel. 125. (a) Glow = Power (P) = I2R
R dP æ dI ö
R eff. = Þ PC = 16P \ = 2ç ÷ = 2 ´ 0.5 = 1%
16 P è I ø
V 2 V2 V2 V2 æ Pö
2
PA : PB : PC : PD :: : : :
R R / 4 R / 16 R / 2 126. (a) Power loss = i²R = ç ÷ R
è Vø
116. (c)
117. (b) Resistance of 45 bulbs in series is less than that of 50 [P = Transmitted power]
bulbs. Since illumination is proportional to the heat V2 1
127. (a) P= ; Rµ (same rated voltage)
æ V2 t ö R P
produced çç R ÷÷ , therefore it will be more with 45 128. (c) In tube light majority portion of radiation comes under
è ø visible region while bulb radiation consists of visible,
bulbs. ultraviolet, infrared radiation giving less visible part.
118. (a) m = ZIt = 0.0000104 × 1 × 600 g 129. (c) Positive terminal of a battery is point of highest
R potential and current flows from highest to lowest
119. (d) Rs = 3R and R P = potential i.e. from +ve to –ve potential.
3 130. (a) Due to Peltier effect, cooling is produced at one of the
V 2 junctions.
PS = = 10W and r r r
3R 131. (c) From relation J = s E , the current density J at any
PP = V2 /(R / 3) = 3V2/R = 9PS = 90 W. point in ohmic resistor is in direction of electric field
Free eBooks on @neetquestionpaper2020

690 Physi cs
r where Req denotes the equivalent resistance of the
E at that point. In space having non-uniform electric
field, charges released from rest may not move along whole circuit. These two light bulbs are wired in
ELOF. Hence statement-1 is true while statement-2 is parallel; any given electron flowing around the circuit
false. passes through one bulb or the other, but not both
So, we must use the in-parallel formula
132. (d) Since the power generated in the bulb equals the 1 1 1 1 1 2 1 3 1
voltage across the bulb times the current through the = + = + = + = =
R eq R1 R 2 3W 6W 6W 6W 6 W 2W
bulb (P = IV), we need to know the current and voltage.
(Equivalently, you can use P = l2R or P = V2/R, in and hence, R eq = 2W . So,
which case you need to know the current or the
voltage.) V 12V
I= =
Because the lower circuit loop contains only the battery R eq 2W = 6 amperes.
and bulb 1, the bulb “ feels” the battery’s full voltage,
12volts. So, by Ohm’s law (V=IR), a current Intuitively, the equivalent resisance of two in parallel
resistors is less than either individual resistance,
V 12V because the current can “split” itself between two
I= = = 4A
R 3W paths. It’s like opening an extra lane on a highway;
flows through that bulb. Therefore, the bulb generates cars can flow by more easily, because they can spread
power P = IV = (4A)(12V) = 48W. themselves out among more paths.
133. (b) Most of the charges flowing around the circuit are 136. (b) 137. (b) 138. (d)
valence electrons stripped off the metal atoms in the
i2 = 0 i2 = 0.1A
wires and light bulbs. A battery doesn’t “supply” all
of the charges. It merely pushes around charges
already present in the circuit. E2 + R1 R2
Statements (c) and (d) are both true. All charges flowing –
4V
into the light bulb also flow back out; no current gets + 0.1A
– E1
“ used up” But inside the bulb, those charges lose
energy. This lost electrical energy converts into light i1 = 0.1A, E 2 = 4V, i 2 = 0
and heat. So, the current has lower “potential” after
flowing through the bulb. As, 0.1 R1 + 0.1 R 2 - E1 = 0
134. (c) If the battery had internal resistance, bulb 1 would 0.1 R 2 - 4V = 0 , R2 = 40W
dim. But here, screwing in bulb 2 doesn’t prevent bulb
1 from “feeling” the full 12 volts produced by the Now, i 2 = 0.3A, i1 = 0.1A, E 2 = 8A
battery. Both bulbs get a full “does” of that voltage. 0.2A
So, bulb 2 turns on without lessening the brightness
of bulb 1. 0.3A
If you picked (b), you probably thought that the battery 40 W
can “supply” only a certain amount of current. 8V 0.1A
According to this resoning, bulb 2 draws current at
the expense of bulb 1. But the battery doesn’t “supply”
all of the electrons flowing around the circuit. It merely 0.1 R1 + E1 - 8 = 0
pushes around electrons already present in the metal 0.1 + 4 – E1 = 0
wires and bulb filaments, as mentioned in question 2. 4
Screwing in bulb 2 allows the valence electrons in that 0.2R1 – 4 = 0 Þ R1 = = 20 W
branch of the circuit to “join the current.” 0.2
135. (d) The total circuit, i.e., the current through the battery, E1 = 2 + 4 = 6V
139 (d) 140. (c) 141. (a)
V 142. (a) 143. (d) 144. (c)
is simply I = ,
R eq
Free eBooks on @neetquestionpaper2020

19
Moving Charges and
Magnetism
OERSTED EXPERIMENT Comparison between electric field and magnetic field
In 1802 Gian Domenico Romagnosi, an Italian lawyer and judge, Electric field Magnetic field
found that a steady electric current flowing in wire affected a
1. Source is an electric 1. Source is a current element
magnetic needle placed near it. He published his observation in a uur
local newspaper (called Gazetta di Trentino). But nobody noticed charge (q). ( Idl ) .
this phenomenon. 2. Isolated charge exists 2. Isolated poles do not exist.
3. Electric field at a point 3. Magnetic field at a point due to
In 1820 Hans Christian Oersted (a Danish Physicist) rediscovered
due to a point charge is a current element is perpendicular
this phenomenon. He noted that a magnetic compass needle, in the plane containing to the plane containing the point
brought close to a straight wire carrying a steady electric current, the point and the charge. and the current element.
aligned
r itself perpendicular to the wire i.e., the direction of magnetic 4. It obeys inverse square 4. It also obeys inverse square law
field B is tangential to a circle which has the wire as centre, and law (a long range force). (a long range force).
which has its plane perpendicular to the wire (Fig 1-a). Oersted 5. It obeys principle of 5. It also obeys principle of
also noticed that on reversing the direction of current; the direction superposition as the field superposition.
of magnetic field is reversed. is linear related to charge.
6. Angle dependence is not 6. Angle dependence is present.
present.
7. Line of electric lines of force 7. Lines of magnetic lines of
i do not form closed loops. force form closed loops.
B 8. Electric field changes 8. Magnetic field does not
kinetic energy of a charged change kinetic energy of a
B particle. charged particle.
i 9. A charged particle whether 9. A charged particle at rest
at rest or in motion in an do not experience force
Clockwise Anticlockwise electric field experiences due to magnetic field.
Fig. 1(a) Fig. 1(b) a force due to electric field.

In first case when current is in upward direction, magnetic field MAGNETIC FIELD DUE TO CURRENT CARRYING
is clockwise (Fig 1-a) and when the current is downward, CONDUCTOR, BIOT-SAVART’S LAW
magnetic field is anticlockwise (Fig. 1-b). uuur
The magnetic induction dB at any point outside A
MAGNETIC FIELD the current path due to a small current element of
uur
It is a region of space around a magnet or current carrying length dl (in the direction of the current) is given i P
conductor or a moving charge in which its magnetic effect can by Biot-Savart's law q
be felt. uur ur dl r
uuur m0 I (d l ´ r )
The conductor carrying current is electrically neutral but a dB =
magnetic field is associated with it. 4p r3
m I dl sin q B
The SI unit of magnetic field induction is tesla (T) or weber/m2 or, | dB | = 0
and cgs unit is gauss. 1 gauss = 10– 4 T 4p r2
Free eBooks on @neetquestionpaper2020

692 Physi cs
ur ur
uur m q(v ´ r ) Magnetic field due to an infinitely long conductor :
Also, dB = 0 where v is the drift velocity of charge
4p r 3
where m0 = 4p × 10–7 TmA–1.
uuur
Direction of dB ur m I ˆ m 2I a
uur uur r B = 0 (-k) ÞB= 0 I
The direction of dB is perpendicular to both dl and r , governed (at P) 2pa 4p a P
uur r
by the right hand thumb rule of the cross-product of dl and r .
The magnetic fields going into the page and coming out of the
Elucidation
page are represented as follows :
Magnetic field in the case of infinitely long wire
p/2
m0I
dB = ò 4pa
cos qdq I
-p / 2
Magnetic field Magnetic field
going into the page coming out of the page. r m I( - kˆ) a
ÞB= 0
Magnetic Field due to Various Current Carrying 2 pa
Conductors Magnetic field near the end of a long conductor :
Magnetic field due to finite sized conductor :

m0I
^
j B = I
I (at P) 4pa
a q1 ^
i
a
O q2 P P
Elucidation
^ r = a cosec q, l = a cot q, A
k 2
dl = a cosec q dq
ur q
r m I r m 0 dl ´ r
B(at P) = 0 (sin q1 + sin q 2 ) ( - k)
ˆ
dB = I r
2 pa 4pr 3

m 0 2I m Ia cosec 2 qdqˆj
l 90°
ÞB= (sin q1 + sin q 2 ) = 0 3 ´ (aj – a cot qˆj)
4p a O P
4pa cosec2 q a
Elucidation m I cos ec q uur
= 0 dq( -k) ˆ Þ dB = m 0 I sin qdq ( -k) ˆ
ur m I r r
( ) 4pa cos ec q 4pa
3
dB = 0 3 d l ´ r r = a sec q, l = a tanq
4pr uur m 0 I
r B = ˆ = æ m0 I - 0ö ( -k)
[- cos q]pp /2 ( - k) ˆ
Þ dl = a sec2 q dq Þ d l = a sec2 qdqˆj çè ÷ø
4 pa 4 pa
ur
r = -a tan qˆj + aiˆ m0I
ÞB = (- k̂ )
ur m Ia sec qdq 2 4pa
Þ dB = 0 3 3 ˆj ´ ( -a tan qˆj + ai)
ˆ
Magnetic field due to a current carrying coil :
4pa sec q
(i) Magnetic field at a point on the axis of symmetry of a
m 0 I sec 2 qdq m 0 I cos qdq
= (- k̂) = (- k̂) circular coil, at a distance “x” from its centre :
4pa sec q 3 4pa B = m0NI a2/2(a2 + x2)3/2
q2 q2
m 0I m I mo 2pNIa2 a
\B = ò
- q1
4 pa
cos qdq = 0
4pa ò cos qdq
( - q1 )
or, B =
4p (a2 + x2 )3/ 2 I
x
m I N = total number of turns
Þ B = 0 [sin q 2 + sin q1 ] (Pointing into the plane of paper)
4 pa a = coil radius
Magnetic field near the end of a finite sized conductor : r
The direction of B is given by Right hand screw rule.
Right hand screw rule : If direction of rotation of right handed
ur mI screw-head is the directon of current in a circular conductor then
B = 0 sin q I the direction of its advance is the direction of magnetic field. This
(at P) 4pa
a q is applicable even if the current, magnetic field are interchanged, as
P
in case of current flowing through a straight conductor.
Free eBooks on @neetquestionpaper2020

Moving Charges and Magnetism 693


r
Elucidation In this case the direction of magnetic field B is into the
y page.
dl Magnetic field inside a current carrying solenoid
r (i) Finite size solenoid
a
q q2
q x m0 nI q1
x B = (cos q1 - cos q2 )
( at P ) 2 P

(ii) Near the end of a finite solenoid


z
m0 nI
Let for a particular angle, position of small length element dl is B= cos q ; (q1 = q & q 2 = p / 2)
2
given by its coordinates as
(iii) In the middle of a very long solenoid, B = m0 n I
z = -a cos q, y = a sin q .
(iv) Near the end of a very long solenoid
r
Now, a = -a cos qkˆ + a sin qˆj ,
r r m0 n I q
r = - a + xiˆ = xiˆ - a sin qˆj + a cos qkˆ B= P
2
Also we have l = rq Þ dl = rdq .
n is the number of turns per unit length of solenoid.
Now d l ^ a at any instant (v) Magnetic field in the endless solenoid (toroid) is same
uur m0I ur r throughout and is m0nI.
\ d l = dl(sin qk̂ + cos qˆj) , dB =
4pr3
dl ´ r ( ) (vi) Magnetic field outside a solenoid or toroid is zero.
uur m Irdq AMPERE’S CIRCUITAL LAW
Þ dB = 0 3 (sin qkˆ + cos qˆj) ´ (xiˆ - a sin qˆj + a cos qk) ˆ
The line integral of magnetic field across a closed loop is equal
4pr
to 40 times the net correct inside the loop
m0 Irdq
= (x sin qˆj + a sin 2 qˆi + x cos qkˆ + a cos 2 qˆi) uur uur
2
4p(x + a ) 2 3/2
i.e., Ñò B .dl = m0 I
m0 Irdq where I is the net current inside the loop.
= [x(sin qˆj + cos qk)ˆ + ai]ˆ
4p(x 2 + a 2 )3/2 (1) The direction of the magnetic field at a point on one side
uur uur of a conductor of any shape is equal in magnitude but
Þ B = ò dB
opposite in direction of the field at an equidistant point
m 0 Ia 2p on the other side of the conductor.
= 2 2 3/2
[x | - cos qˆj + sin qkˆ |0 +a(2p - 0)i]
ˆ
4p(x + a ) (2) If the magnetic field at a point due to a conductor of any
uur ˆ shape is Bo if it is placed in vacuum then the magnetic
m I2pa(i)
0
B = field at the same point in a medium of relative permeability
4p(x 2 + a 2 )3/2 mr is given by B = mr Bo .
If number of turns of coil are N, then (3) If the distance between the point and an infinitely long
r m0 I2pNa (i) ˆ ˆ
m 0 INa(i) conductor is decreased (or increased) by K-times then
|B|= 2 2 3/2
= 2 2 3/2 the magnetic field at the point increases (or decreases)
4p(x + a ) 2(x + a )
by K-times.
m 0 2pNIa
Þ B= (4) The magnetic field at the centre of a circular coil of radius
4p (x 2 + a 2 )3/2 smaller than other similar coil with greater radius is more
m0 2pNI than that of the latter.
(ii) At the centre of a circular coil, B = m0NI / 2a = (5) For two circular coils of radii R1 and R2 having same
4p a
current and same number of turns, we have
(iii) Magnetic field at the centre of a circular arc carrying
current B1 R2
= , where B1 and B2 are the magnetic fields at their
B2 R1
ur m I q m Iq q centres.
B = 0 ´ (- kˆ) Þ B = 0
( at P ) 2a 360 4p a P (6) The magnetic field at a point outside a thick straight wire
a
carrying current is inversely proportional to the distance
where q is in radian. but magnetic field at a point inside the wire is directly
proportional to the distance.
Free eBooks on @neetquestionpaper2020

694 Physi cs
Keep in Memory m 0 2 Ix
B inside = for x < R
4p R 2
1. If in a coil the current is clockwise, it acts as a South-pole. If R
m 0 2I
the current is anticlockwise, it acts as North-pole. B surface = for x = R
4p R
x P
m 0 2I
B outside = for x > R
4p x
ur
4. Graph of magnetic field B versus x
2. No magnetic field occurs at point P, Q and R due to a thin
current element Idl . B

Idl
R P Q
x
3. Magnetic field intensity in a thick current carrying
conductor at any point x is 5. Magnetic field is zero at all points inside a current carrying
hollow conductor.

Magnitude and direction of magnetic field due to different configuration of current carrying conductor.

Configuration of Point of observation Magnetic field


S.No. current carrying conductor Magnitude Direction
1. Two long linear and parallel At P, the mid point between B=0 Normal to the plane of
current carrying conductors the two wires. paper, inwards.
Wire 1 Wire 2 The distance of P from each B = m 0 2Ié 1 + 1 ù
wire is r/2. 4p êë x r + x úû
At P', distant x from wire 2 as
I I P' shown. m0 é 1 1ù Normal to the plane of
P P'' At P'', distant x' from wire 1 B = 2Iê - ú paper, outwards.
x‘ x
as shown. 4p ë r - x' x ' û
r
2. Square loop At the centre. é m0 I ù Normal to the plane of
I
ê
B = 4 4p a / 2 ú paper, inwards.
ê ú
ëê (sin 45° + sin 45° )ûú
O I
I
45°45°
a/2
aI
3. Two concentric circular coils At the centre. m0 én n ù n1 n 2
B= 2p I ê 1 - 2 ú If >
having turns n1 and n2 4p ëa bû a b
n2 Perpendicular to the
plane of paper
b inwards.
Oa
n1

4. Straight wire and loop At the centre.


B=0

I O I
Free eBooks on @neetquestionpaper2020

Moving Charges and Magnetism 695

5. Straight wire & semi-circular loop At the centre. m 0 pI Normal to the plane of paper,
B= inwards.
4p a
a
I O I
6. Circular loop At the centre of loop. m 0 é 2p I 2I ù Normal to the plane of
B= - ú
I 4p êë a aû paper, inwards.

O
a
I I
7. Two concentric circular arcs At the common centre. m0 é1 1ù Normal to the plane of paper,
B= Iq -
4p êë a b úû outwards.

b
a
q
O

8. Semi-circular area and straight At the centre of the semi- m 0 pI m 0 I Normal to the plane of paper,
B= +
conductors circle. 4p r 4p r outwards.

r
I O

I
9. Two concentric coils mutually At the common centre. According to law of vectors
B = B12 + B 22
normal to each other. addition.
B1 where
I2 m 2p n1I
B B1 = 0
4p a

a B2 m 0 2p n 2 I
O B2 =
4p b
n1 I1
b

FORCE ON A CONDUCTOR t = BINA


The force on a conductor is given by where N is the number of turns in the coil, A is the area and I is the
F = BIl sin a current.
where l is the length of the conductor in meter; B is the flux
density of field in tesla (Wb/m2); I is the current in ampere and If the plane of the coil makes an angle a with the direction of the
a is the angle which the conductor makes with the direction of the field, then
field. t = BINA cos a.
Special case : Example 1.
If a = 90°, then F = BIl
The direction of the force is given by Fleming's left hand rule. The field normal to the plane of a wire of n turns and radius
r which carries a current i is measured on the axis of the
TORQUE ON A COIL
coil at a small distance h from the centre of the coil. By
The torque acting on a rectangular coil placed with its plane parallel
what fraction this is smaller than the field at the centre?
to a uniform magnetic field of flux density B is given by
Free eBooks on @neetquestionpaper2020

696 Physi cs
Solution : Solution :
The magnetic field on the axis of a current i carrying coil of Magnetic induction at O due to coil Y is given by
turns n, radius r and at a distance h from the centre of the m0 2p I ( 2 r ) 2
coil BY = ´ ...(1)
4p [(2r ) 2 + (d) 2 ]3 / 2
m0 2pnir 2 Similarly, the magnetic induction at O due to coil X is given
B= ´ .....(1)
4p ( r 2 + h 2 ) 3 / 2 by
The field at the centre is given by m0 2p I ( r ) 2
BX = ´ ...(2)
m 2pi ´ n 4 p [(r) 2 + (d / 2) 2 ]3 / 2
B centre = 0 ´ ....(2) (Q at centre h = 0)
4p r BY 1
From eqs. (1) and (2), =
B r3 BX 2
= 2
B centre (r + h 2 ) 3/ 2 Example 4.
A cell is connected between two points of a uniformly thick
r3 1
circular conductor. i1 and i2 are the currents flowing in
= 3/2 =
é h2 ù æ 3 h2 ö two parts of the circular conductor of radius a. What will
r 3 ê1 + 2 ú ç1 + ÷
r û ç 2 r2 ÷ be the magnetic field at the centre of the loop?
ë è ø
Solution :
æ 3 h 2 ö÷ Let l1, l2 be the lengths of the two parts PRQ and PSQ of the
or B ç1 + = Bcentre conductor and r be the resistance per unit length of the
ç 2 r 2 ÷ø
è conductor. The resistance of the portion PRQ will be R1 = l1r
3 h2 i2
\ (Bcentre - B) / B =
2 r2
Example 2. S O Q
In fig., there are two semi-circles of radii r1 and r2 in which
a current i is flowing. Find the magnetic induction at P i1
R
centre O.

r1 i The resistance of the portion PSQ will be R2 = l2r


Pot. diff. across P and Q = i1R1 = i2R2
O
r2 or i1l1r = i2l2r or i1l1 = i2l2 …… (i)
Solution : Magnetic field induction at the centre O due to currents
through circular conductors PRQ and PSQ will be
m0 p i m 0 p i
B = B1 + B2 = ´ + ´ m0 i1l1 sin 90º m 0 i 2 l 2 sin 90º
4p r1 4p r2 B1 – B2 = - =0
4p r2 4p r2
m 0 i é 1 1 ù m 0 i é r1 + r2 ù Example 5.
= ê + ú= ê ú A current passing through a circular coil of two turns
4 ë r1 r2 û 4 ë r1r2 û
produces magnetic field B at its centre. The coil is then
Example 3. rewound so as to have four turns and the same current is
Two circular coils X and Y having equal number of turns and passed through it.The magnetic field at its centre now is
carry equal currents in the same sense and subtend same B B
solid angle at point O. If the smaller coil X is midway between (a) 2 B (b) (c) (d) 4 B
O and Y, then if we represent the magnetic induction due to 2 4
Solution : (d)
bigger coil Y at O as BY and due to smaller coil X at O as BX
m 0 2 pnI n
B B= i.e. B µ ;
then find Y . 4p r r
BX
Given, L = 2pr1 × 2 = 2pr2 ×4 ; r1/r2 = 4/2 = 2
d B2 n 2 r1 4
d/2
= ´ = ´2 = 4 or B2 = 4B1 = 4B.
B1 n1 r2 2
2r
r Example 6.
O A coil having N turns is wound tightly in the form of a spiral
Y X
with inner and outer radii a and b respectively. When a
current I passes through the coil, find the magnetic field at
the centre.
Free eBooks on @neetquestionpaper2020

Moving Charges and Magnetism 697


Solution : Solution :
N ´ dx P
Refer to fig., no. of turns in dx, n = .
(b - a )

A Q B
dx X X X X X X X
b O x
a
D R
From symmetry, B at P and R should be same. Using
Ampere’s law,
The magnetic field induction at the centre O due to current L
r ur
I through the entire spiral ò B . dl = m0i = m0 (kL – kL)
b b 0
m 0 2 pnI m æ N ö dx
B=ò dx = 0 2p ò ç ÷ I r r
a
4p x 4p a è b - a ø x B.L = 0 Þ B = 0 hence at P and R, B = 0
m 0 NI m NI b Now, let us apply Ampere’s law between Q and R.
= (log e x) ab = 0 log e .
4p (b - a) 2 (b - a) a L
r uur
Example 7. Here, ò B . dl = m0 kL
A long straight wire along z-axis carries a current I rin the 0

negative z-direction. Find the magnetic field vector B at a Þ BL + 0 × L = m0kL (Q B is zero at R)


point having coordinates (x,y) in the z = 0 plane. Þ B = m0k towards right.
Solution : Example 9.
Compute the flux density in air at a point of 9 cm from
Refer fig., r = x 2 + y 2 the long straight wire carrying a current of 6A.
Y Solution :
Given : a = 9 cm = 9 × 10–2 m, I = 6A
P(x,y)
m o I 4p ´ 10 -7 ´ 6
B= = = 1.33 × 10–5 T
2pa 2p ´ 9 ´ 10-2
B
O Example 10.
X Calculate the flux density at a distance of 1 cm from a
I
very long straight wire carrying a current of 5A. At what
distance from the wire will the field flux density
Z
neutralize that due to the earth's horizontal component
flux density 2 × 10–5 T ? (m0 = 4p × 10–7 Hm–1)
Magnetic field induction at P, Solution :
r m 2I m I
B= 0 = 0 m o I 4p ´ 10-7 ´ 5
4p r 2p 2 2 B= = = 10-4 T
x +y 2pa 2p ´ 1 ´ 10-2
Unit vector perpendicular to the position vector For the second part,

yî - xˆj m o I 4p ´ 10-7 ´ 5


= a= = = 5 ´ 10 -2 = 5 cm
2pB 2p ´ 2 ´ 10 -5
x2 + y2
r r m0I Example 11.
\ B = B ´ unit vector = ( yî - xˆj) A wire 28m long is bent into N turns of circular coil of
2p( x 2 + y 2 ) diameter 14 cm forming a solenoid of length 60 cm.
Example 8. Calculate the flux density inside it when a current of 5 amp
Two large metal sheets carry surface current as shown in passed through it. (m0 = 12.57 × 10–7 m–1)
fig. The current through a strip of width dl is kdl where k is Solution :
a constant. Find the magnetic field at the point P, Q & R. Given : d = 14cm = 0.14m l = 60cm = 0.6 m
.P By the question, N × pd = 28 m.
N × p × 0.14 = 28
.Q 28
\ N= = 63.66 turns.
X X X X 0.14 ´ p
.R
Free eBooks on @neetquestionpaper2020

698 Physi cs

N 63.66 Solution :
B < λ o nI < λ o I < 12.57 ≥ 10,7 ≥ ≥5 l = 5cm = 5 × 10–2m, I wire = 2A, n = 1000 m–1, F = 10–4 N
l 0.6
= 6.67 ×10–4 T If I be the current through the solenoid, then
Example 12. B = m0 nI
A vertical conductor X carries a downward current of 5A.
(a) What is the flux density due to the current alone at a Force = BIwire × l or 10–4 = m 0 NI ´ I wire ´ l
point P 10 cm due east of X?
(b) If the earth's horizontal magnetic flux density has a or 10 -4 = 4 p ´ 10 -7 ´ 1000 ´ I ´ 2 ´ 5 ´ 10 -2 = 4p ´ 10 -5 I
value 4 × 10–5 T, calculate the resultant flux density 10-4 10
at P. \ I= = = 0.8A
4p ´ 10-5 4p
Is the resultant flux density at a point 10cm due north
of X greater or less than at P? Explain your answer. Example 14.
Two long parallel conductors carry currents of 12A and 8A
respectively in the same direction. If the wires are 10cm
N
apart, find where the third parallel wire also carrying a
current must be placed so that the force experienced by it
shall be zero.
Be W E
Solution :
B For the force on the third conductor to be zero, the direction of
S
the flux density due to the current flowing in the two wires
I must be opposite in the position of the wire.
Solution : \ Third wire must be placed between the wire. Let the third
(a) I = 5A, a = 10cm = 0.1 m wire placed at a distance x m from the wire carrying 12A,
m o I 4p ´ 10-7 ´ 5 then, B1 = B2
B= = = 1 ´ 10-5 T
2pa 2p ´ 0.1
At P, the earth's horizontal magnetic flux density, 12A 8A
X
Be = 4 × 10–5T (from South to North)
The direction of B is from north to south.
\ Resultant intensity at 0.lm
P = 4 × 10–5 – 1 × 10–5 T
= 3 × 10–5 T (From south to north) m o I1 m o I2
For a point 10 cm , north of X the flux density due to the = .
2px 2p(0.1 - x)
current in X = 1 ×10–5T (due east)
(b) The flux density due to the horizontal component of 12 8 3 2
the earth's field = 4× 10–3T (due north) or = or =
x 0.1 - x x 0.1 - x
0.3
or 0.3 = 5x = = 0.06 m
5
Be BR
Example 15.
The figure shows a simple form of current carrying the long
solenoids, which has 2000 turns per metre, is in series with
B the horizontal rectangular copper loop ABCDEF with
\ Resultant Intensity BC = 10cm and CD = 3cm the loop, which is freely pivoted
on the axis. AF goes well inside the solenoid, and CD is
BR = B2e + B 2 . = (4 ´ 10-5 )2 + (1 ´ 10-5 )2
perpendicular of this axis of the solenoid, when the current
is switched on, a rider of mass 0.2g placed 5cm from the axis
–5
= 17 ´ 10-5 = 4.1 × 10 T needed to restore equilibrium. Calculate the value of the
which is greater than the flux density at P. current I.
Example 13. Rider is on non-conducting support.
A horizontal wire, of lenth 5 cm and carrying a current of
2A placed in the middle of a long solenoid and right angles C B A
I
to its axis. The solenoid has 1000 turns per metre and carries D E
a steady current I. Calculate I if the force on the wire is
s

F
axi

vertically downwards and equal to 10–4 N. F


Free eBooks on @neetquestionpaper2020

Moving Charges and Magnetism 699


Solution : I

Torque acting on the copper loop X P Q

= BIA = m o nI ´ I ´ l ´ b 2 cm
10 cm

= 4p ´ 10-7 ´ 2000I2 ´ 10 ´ 3 ´ 10-4


S R
= 24 ο × 10–7 I2
I 3 cm
Torque due to the weight of the rider
Solution :
= mg × 5 × 10–2 = 0.2 × 10–3 × 9.8 × 5× 10–2 The force on the side PS due to current X,
\ 24 ο × 10–7I2 = 2 × 98 × 5 × 10–7 F = Bil
2 ´ 98 ´ 5 m0 i x 4p ´ 10-7 5
\I = 3.6A = ´ il = ´ ´ 0.10
24p 2 pr r 2p 2 ´ 10-2
Example 16. = 10 × 10–6N along + ve x-axis
Two identical wires R and S lie parallel in the horizontal The force on the side QR due to current through X,
plane, their axis being 0.10m apart. A current of 30A in S F = Bil
Neglecting the effect of the earth's magnetic flux density at a m o ix 4p ´ 10-7 5
point P in the plane of the wires. If P is (i) mid way between = ´ il = ´ ´ 2 ´ 0.1
2p r 2p 5 ´ 10-2
R and S (ii) 0.05 m from R and 0.15m from S. [permeability
= 4 ×10–6 N along – ve x-axis
of free space m0 = 4p × 10–7 Hm–1]
The force on PQ and SR due to the current carrying
Solution :
conductor are equal and opposite.
(i) Midway between R and S 10A 30A Hence the net force = 10 ×10–6 – 4 × 10–6
= 6 × 10–6 N along +ve x-axis
mo I Example 18.
Field due to R, BR = R P.
2 pr S Two long straight parallel wires in which there are
moI currents of 2.0A and 3.0A respectively in the same direction
Field due to S, Bs =
2 pr are 0.1m apart. Calculate the magnetic field at a point
0.10m mid-way between them.
4p ´ 10-7 ´ 30
= = 12 ´ 10-5 T Solution :
2p ´ 0.05
Magnetic field at a point mid-way between the wires due to
Q Direction of B due to two wires is the same, total flux
the Ist wire,
density,
B = BR + BS -7
B1 = m o I = 4p ´ 10 ´ 2 = 8.0×10–6 Wb/m2
= 4 × 10–5+12 × 10–5 = 16 × 10–5 = 1.6 × 10–4 T 2pr 2p ´ 0.05
(ii) If P is 0.05 m from R and 0.15 m from S
I1 I2
-7 10A
m o I 4p ´ 10
BR = =
2pr 2p ´ 0.05
S
= 4× 10–5 T .P R
0.05
m 0 I 4p ´ 10-7 ´ 30
BS = = 0.15m
2pr 2p ´ 0.15 (Inwards the plane of the paper)
Q BR and BS are in opposite direction Magnetic field at the point mid-way between the wires, due
\ Resultant field = BS – BR = 4 × 10–5 – 4 × 10–5 = 0 to the second wire,
Example 17. m o I 4p ´ 10 - 7 ´ 3
In the figure X is a very long straight wire. Rectangle PQRS B2 = =
2pr 2p ´ 0.05
is suspended with PS 2cm from x as shown the dimensions of = 12× 10 Wb/m2 (outwards the plane of the paper)
–6
PQRS are 10cm by 3cm, and current of 2A flows in the Hence, the resulting field at the point
coil. Calculate the resultant force on PQRS in magnitude = B2 – B1 = 12× 10– 6 – 8×10– 6
and direction. = 4 × 10– 6 Wb/m2 (outwards the plane of the paper)
Free eBooks on @neetquestionpaper2020

700 Physi cs

19.1
Solve following problems with the help of above text and radius. The magnetic field at the centre caused by the same
examples : current is
1. An electric charge in uniform motion produces (a) a quarter of its first value
(a) an electric field only (b) unaltered
(b) a magnetic field only (c) four times of its first value
(c) both electric and magnetic fields (d) a half of its first value
(d) no such field at all 8. If a copper rod carries a direct current, the magnetic field
2. The magnetic field at a point due to a current carrying associated with the current will be
conductor is directly proportional to (a) only inside the rod
(b) only outside the rod
(a) resistance of the conductor
(c) both inside and outside the rod
(b) thickness of the conductor
(d) neither inside nor outside the rod
(c) current flowing through the conductor
9. If a long hollow copper pipe carries a direct current, the
(d) distance from the conductor
magnetic field associated with the current will be
3. A current is passed through a straight wire. The magnetic
(a) only inside the pipe
field established around it has its lines of force (b) only outside the pipe
(a) circular and endless (c) neither inside nor outside the pipe
(b) oval in shape and endless (d) both inside and outside the pipe
(c) straight 10. A current i ampere flows along an infinitely long straight
(d) All of the above thin walled tube, then the magnetic induction at any point
4. The strength of the magnetic field around a straight inside the tube is
conductor (a) ¥ (b) zero
(a) is same everywhere around the conductor
(b) obeys inverse square law m 0 2i 2i
(c) . tesla (d) tesla
(c) is directly proportional to the square of the distance 4p r r
from the conductor 11. A steady current I flows down a hollow cylindrical tube of
(d) None of these radius a and is uniformly distributed around the tube. Let
5. Which of the following graphs, shows the variation of r be the distance from the axis of symmetry of the tube to a
magnetic induction B with distance r from a long wire given point. What is the magnitiude of the magnetic field
carrying a current? B at a point inside the tube?
(a) 0 (b) 2 I/r c
(c) 2 I r/a2c (d) 4 p (r – a) I/r2
B B
12. A current i ampere flows in a circular arc of wire which
(a) (b) subtends an angle (3 p/2) radians at its centre, whose radius
is R. The magnetic induction B at the centre is
r r
(a) m0 i/R (b) m0 i/2R
B B (c) 2 m0 i/R (d) 3 m0 i/8R
13. A current is flowing north along a power line. The direction
(c) (d) of the magnetic field above it, neglecting the earth’s field
r r is towards
6. A long straight conductor carrying a current lies along the (a) north (b) east
axis of a ring. The conductor will exert a force on the ring (c) south (d) west
if the ring 14. Field inside a solenoid is
(a) carries a current (a) directly proportional to its length
(b) has uniformly distributed charge (b) directly proportional to current
(c) has nonuniformly distributed charge (c) inversely proportional to number of turns
(d) None of these (d) inversely proportional to current
7. A length of wire carries a steady current. It is bent first to 15. Energy in a current carrying coil is stored in the form of
form a circular plane coil of one turn. The same length is (a) electric field (b) magnetic field
now bent more sharply to give a double loop of smaller (c) dielectric strength (d) heat

ANSWER KEY
1. (c) 2. (c) 3. (a) 4. (d) 5. (c) 6. (d) 7. (c) 8. (c) 9. (b) 10. (b) 11. (a) 12. (d)
13. (b) 14. (b) 15. (b)
Free eBooks on @neetquestionpaper2020

Moving Charges and Magnetism 701


FORCE ACTING ON A CHARGED PARTICLE ur
Direction of force F :We can use the rule of cross product. The
MOVING IN A UNIFORM MAGNETIC FIELD
The force acting on a particle having a charge q and moving with direction of F is perpendicular to the plane containing v and B
r r and can be found by right hand thumb rule. It is important to note
velocity v in a uniform magnetic field B is given by
ur ur ur that if q is positive, we will get the correct direction of F by right
F = q(v ´ B) Þ F = qvB sin q ,
r r hand thumb rule. But if q is negative, we have to reverse the
where q is the angle between v and B direction of force.
Case (i) If q = 0, F = 0. Also if q = 180°, F = 0 Flemings left hand rule : It states that if the fore finger, the central
If a charged particle enters a uniform magnetic field in the direction finger and the thumb of the left hand are stretched at right angles
of magnetic field or in the opposite direction of magnetic field, the to each other then if the central finger represents the direction of
force acting on the charged particle is zero. current and fore finger represents field, the thumb will represent
Case (ii) If q = 90°, F = qvB the direction of motion or force experienced by the current
carrying conductor.
In this case the force acting on the v × × × ×
particle is maximum and this force acts × × × × B FORCE BETWEEN TWO PARALLEL CURRENTS
as centripetal force which makes the × × v× × When a current flows in a conductor, the free charges (electrons
charged particle move in a circular path. F
× × × × in case of a metal wire) move. Each free charge movement generates
a force which adds up to give the force on the conductor.
mv2 × × × ×
Force between infinitely long conductors placed parallel to each
\ F = qvB = q × × × ×
r v other at distance d.
mv × × × ×
Þ r=
qB B1
where r is the radius of the circular path.
It is important to note that this force cannot change the speed of F
I2
the charged particle and hence its kinetic energy. But it changes I1
the velocity of charged particle (due to change in direction) and d
hence also causes a change in momentum.
F m 0 2 I1 I 2
Also the work done by the force is zero as the force is acting Force per unit length = =
perpendicular to the direction of motion. l 4p d
where l is the length of wire.
Case (iii) If q = a is any other angle then the path taken is If currents are pointing in same direction, the force is of attractive
helical. The velocity of the charged particle can be split into two nature and if currents are oppositely directed the force is of
parts for better understnading. repulsive nature.
B Lorentz Force Equation
For a charged particle q moving in a region of simultaneously
v uur uur
v sin a a applied electric field E and magnetic field B , the force
vcosa
experienced by it is given by
uur uur ur uur
F = q [ E + (v ´ B )]
(a) v cos a : The force on charged particle due to this component uur
Torque on a current loop in uniform magnetic field B is given
is zero. This component is responsible in moving the charged ur uuur uur uur
by t = (M ´ B ) where M is the magnetic moment of coil.
particle uniformly in the direction of B . uuur r
(b) v sin a : The force acting on charged particle due to this M = N I Anˆ where n is the unit vector normal to the plane of
the loop.
component is q(vsin a )Bsin 90° = qvBsin a . This acts
as the centripetal force and moves the particle in a circular Keep in Memory
path. 1. No force acts on a charged particle if it enters a magnetic
The combined effect of these two is a helical path. field in a direction parallel or antiparallel to the field.
A charged particle entering a uniform magnetic field at an angle 2. A finite force acts on a charged particle if it enters a uniform
executes helical path. magnetic field in a direction with finite angle with the field.
3. If two charged particles of masses m1and m2 and charges
mvsin a
Radius of the helix, R = q1 and q2 are projected in a uniform magnetic field with
qB same constant velocity in a direction perpendicular to the
Angular frequency of rotation, w = (2p / T ) = qB / m field then the ratio of their radii (R1: R2) is given by
2pmv cos a R1 m1 q 2
Pitch of the helix = (v cos a ) T = . = ´
qB R 2 m 2 q1
Free eBooks on @neetquestionpaper2020

702 Physi cs
4. The force on a conductor carrying current in a magnetic It essentially consists of a rectangular coil PQRS or a cylindrical
field is directly proportional to the current, the length of coil of large number of turns of fine insulated wire wound over a
conductor and the magnetic field. non-conducting frame of ivory or bamboo. This coil is suspended
5. If the distance between the two parallel conductors is by means of phosphor bronze wire between the pole pieces of a
decreased (or increased) by k-times then the force between powerful horse shoe magnet NS. The poles of the magnet are
them increases (or decreases) k-times.
curved to make the field radial. The lower end of the coil, is attached
6. The momentum of the charged particle moving along the
to a spring of phosphor-bronze wire. The spring and the free ends
direction of magnetic field does not change, since the force
acting on it due to magnetic field is zero. of phosphor bronze wire are joined to two terminals T 2 and T1
7. Lorentz force between two charges q1 and q2 moving with respectively on the top of the case of the instrument. L is a soft
velocity v1, v2 separated by distance r is given by: iron core. A small mirror M is attached on the suspension wire.
m (q v ) (q v ) Using lamp and scale arrangement, the deflection of the coil can
Fm = 0 . 1 1 2 2 2
4p r be recorded. The whole arrangement is enclosed in a non-metallic
8. If the charges move, the electric as well as magnetic fields case.
are produced. In case the charges move with speeds Theory : Let the coil be suspended freely in the magnetic field.
comparable to the speed of light, magnetic and electric force Suppose, n = number of turns in the coil
between them would become comparable. A = area of the coil
9. A current carrying coil is in stable equilibrium if the magnetic B = magnetic field induction of radial magnetic field in which
r r
dipole moment M , is parallel to B and is in unstable the coil is suspended.
r r Here, the magnetic field is radial, i.e., the plane of the coil always
equilibrium when M is antiparallel to B .
10. Magnetic moment is independent of the shape of the loop. remains parallel to the direction of magnetic field, and hence the
It depends on the area of the loop. torque acting on the coil
11. A straight conductor and a conductor of any shape in the t = niAB … (1)
same plane and between the same two end points carrying Due to this torque, the coil rotates. As a result, the suspension
equal current in the same direction, when placed in the same wire gets twisted. Now a restoring torque is developed in the
magnetic field experience the same force. suspension wire. The coil will rotate till the deflecting torque acting
12. There is net repulsion between two similar charges moving on the coil due to flow of current through it is balanced by the
parallel to each other inspite of attractive magnetic force restoring torque developed in the suspension wire due to twisting.
between them. This is because of electric force of repulsion Let C be the restoring couple for unit twist in the suspension wire
which is much more stronger than the magnetic force. and q be the angle through which the coil has turned. The couple
13. The speed of the charged particle can only be changed by for this twist q is Cq.
an electric force. In equilibrium, deflecting couple = restoring couple
MOVING COIL GALVANOMETER \ ni AB = Cq or i = Cq/ (nAB)
or i = Kq (where C/nAB = K) … (2)
The moving coil galvanometer was first devised by Kelvin and
K is a constant for galvanometer and is known as
later on modified by D’Arsonaval. This is used for detection and
galvanometer constant.
measurement of small electric current.
The principle of a moving coil galvanometer is based on the fact Hence i µ q
that when a current carrying coil is placed in a magnetic field, it Therefore, the deflection produced in the galvanometer is directly
experiences a torque. proportional to the current flowing through it.
Construction: A moving coil ballistic galvanometer is shown in Current sensitivity of the galvanometer : The current sensitivity
figure. of a galvanometer is defined as the deflection produced in the
galvanometer when a unit current is passed through it.
T1
Torsional We know that, niAB = Cq
head
Phosphor q nAB
bronze wire \ Current sensitivity, is = =
i C
M Mirror where C = restoring couple per unit twist
Soft iron core The SI unit of current sensitivity is radian per ampere or deflection
P Q
per ampere.
Voltage sensitivity of the galvanometer : The voltage sensitivity
L N S
N S of the galvanometer is defined as the deflection produced in the
S R galvanometer when a unit voltage is applied across the terminals
of the galvanometer.
Spring Showing radial
field q
T2 \ Voltage sensitivity, Vs =
V
Free eBooks on @neetquestionpaper2020

Moving Charges and Magnetism 703


If R be the resistance of the galvanometer and a current is passed Now there may be two cases:
through it, then Case I : If the charge particles are negatively charged, then these
V = iR negative charges will accumulate at the point P2 and therefore P2
q nAB will be at lower potential than P1.
\ Voltage sensitivity, Vs = i R = CR
Case II : If the charged particles are positively charged, then the
The SI. unit of voltage sensitivity is radian per ampere or deflection point P2 will be at higher potential than P1.
per ampere. Magnitude of hall voltage :
Conditions for sensitivity : A galvanometer is said to be more Let w be width and A be the cross-sectional area of the conductor.
sensitive if it shows a large deflection even for a small value of If e is magnitude of charge or the current carrier (electron or hole).
current. The force on the current carrier due to magnetic field B,
nAB Fm = Bevd
We know that, q= i
C Here, vd is drift velocity of the current carries.
For a given value of i, q will be large if (i) n is large, (ii) A is large, Due to the force Fm, the opposite charges build up at the points
(iii) B is large, and (iv) C is small. P1 and P2 of the conductor.
Regarding above factors, n and A cannot be increased beyond a If VH is Hall voltage developed across the two faces, then the
certain limit. By increasing n, the resistance of the galvanometer strength of electric field due to Hall voltage is given by
will increase and by increasing A, the size of the galvanometer will VH
increase. So, the sensitivity will decrease. Therefore, B is increased. EH = .
w
The value of B can be increased by using strong horse shoe Here w = P1P2.
magnet. Further, the value of C can be decreased. The value of C This electric field exerts an electric force on the current carries in
for quartz and phosphor-bronze is very small. So, the suspension a direction opposite to that of magnetic force. The magnitude of
wire of quartz or phosphor-bronze is used. The value of C is further V
decreased if the wire is hammered into a flat strip. this force is Fe = E H e = H e
w
HALL EFFECT In equilibrium condition, Fe = Fm,
When a current passes through a slab of material in the presence VH
of a transverse magnetic field, a small potential difference is or e = B e vd, or VH = B vd w
w
established in a direction perpendicular to both, the current j
flow and the magnetic field. This effect is called Hall effect Now, drift velocity of current carrier is given by, vd =
ne
The voltage thus developed is called Hall voltage.
where n is the number of current carries per unit volume of the
strip.
Z
Hall resistance, RH = H = æç
V V Bwj ö 1
P1
I è ne ÷ø I

Fm X \ Hall voltage VH = Bwj


P2 ne
B
Y Keep in Memory

1. Hall effect can determine nature of current (charge) carriers in


Hall effect enables us to :
the material. i.e. whether the charge is +ve or –ve.
(i) Determine the sign of charge carriers inside the conductor.
(ii) Calculate the number of charge carriers per unit volume. bBI
2. Hall voltage VH =
Explanation : Let us consider a conductor carrying current in +X neA
direction. The magnetic field is applied along +Y direction. where n is the density of charge carriers
Consider two points P1 and P2 on the conductor and connect a
Z
voltmeter between these points. If no magnetic field is applied
across the conductor, then the points P1 and P2 will be at same
potential and there will be no deflection in the galvanometer.
I
However, if a magnetic field is applied as shown in the figure, then b I O
VH Y
the Lorentz force acts on electrons as shown in the figure.
® ®
X B
The Lorentz force on electrons Fm = –e (vd ´ B) acts in the
b = thickness of plate, B = magnetic field, I = current flowing
downwards direction. through plate, A = area of cross-section of plates
Free eBooks on @neetquestionpaper2020

704 Physi cs
Example 19. Taking the motion of charged particle at right angles to the
A long straight wire carries a current of 4 amp. A proton initial direction of motion i.e. motion along the direction of
travels with a velocity of 4×104 m/s parallel to the wire electric field for the displacement y in the electric field.
0.2 m from it and in a direction opposite to the current.
1
What is the force which the magnetic field of current exerts u = 0, a = Eq/m, t = t, s = y. As, s = ut + at 2
on the moving proton ? 2
Solution : 1 Eq 2 1 Eq 2
The force on proton, F = q v B sin q \ y = 0´ t + t = t ....(1)
2 m 2 m
m0 i 4 p ´ 10 - 7 4 If x is the length of region of electric field, then t = x/v
Here B = ´ = ´ = 4 ´ 10 - 6 tesla
2p r 2p 0 .2 1 Eq x 2 1 Eqx 2
\ F = (1.6×10–4) (4×104) (4×10–6) sin 90º From eq. (1), y = ´ =
2 m v 2 2 2E k
= 2.56×10–20 newton
\ Both the trajectories will be equally curved
Example 20.
Example 22.
Two particles X and Y having equal charges, after being
A tightly wound, long solenoid has n turns per unit length,
accelerated throught the same potential difference, enter
a radius r carries a current i. A particle having charge q
a region of uniform magnetic field and describe circular
and mass m is projected from a point on the axis in a
paths of radii R1 and R2 respectively. Find the ratio of
direction perpendiculat to the axis. What can be the
mass of X to that of Y.
maximum speed for which particle does not strike the
Solution :
solenoid?
1 Solution :
mv 2 = qV
2 B = m 0 ni ......(1)
\v = (2qV / m) 1/ 2 Now at maximum speed the charge will move in a circle with
one end as axis and other the solenoid wire.
(where V is the potential difference)
r
\ Radius =
mv2 2
Centripetal force < qvB
R mv2
Using equation, qvB = ......(2)
æ qB ö ærö
\v = ç ÷R ç ÷
èmø è2ø
1/ 2 1/ 2
æ 2qV ö æ qB ö æ 2mV ö 1 Putting the value of B from equation (1),
Hence ç ÷ =ç ÷ R or R = çç ÷÷ ´
è m ø è m ø è q ø B 2mv 2 qm 0 nir
qvm 0 ni = or v =
r 2m
Here V, q and B are constant. Hence R µ m
Example 23.
2 A rectangular loop of sides 25 cm and 10 cm carrying a
m1 æ R 1 ö
\ =ç ÷ current of 15 A is placed with its longer side parallel to a
m 2 çè R 2 ÷ø long straight conductor 2.0 cm apart carrying a current of
Example 21. 25 A. What is the net force on the loop?
The mass of a proton is 1847 times that of electron. If an Solution :
electron and a proton are injected in a uniform electric Consider a rectangular loop PQRS placed near a long straight
field at right angles to the direction of the field, with the conductor AB as shown in Fig. Due to the interaction of
same kinetic energy, then currents, the arm PQ of the loop will get attracted while arm
(a) the proton trajectory will be less curved than that of RS will get repelled. Forces on the arms QR and SP will be
electron equal and opposite and hence cancel out.
(b) both the trajectories will be straight Here, PQ = 25 cm = 25 × 10–2 m,
(c) both the trajectories will be equally curved B
(d) the electron trajectory will be less curved than that of
proton Q R
I2 = 15A

I1 = 15A

Solution : (c)
I1 = 25A

1
Kinetic energy, E k = mv 2 or mv2 = 2E k
2
Force on the charged particle in electric field F = Eq. r1
Acceleration of the charged particle in the direction of P S
r2
electric field, a = Eq/m, where q is the charge of particle. A
Free eBooks on @neetquestionpaper2020

Moving Charges and Magnetism 705


PS = 10 cm = 10 × m 10–2 Example 25.
Distance of PQ from AB, Consider a nonconducting plate of radius r and mass m
r1 = 2.0 cm = 2.0 × 10–2 m which has a charge distributed uniformly over it . The plate
Distance of RS from AB, is rotated about its axis with an angular speed w. Show
r2 = 2.0 + 10 = 12.0 cm = 12.0 × 10–2 m that the magnetic moment m and the angular momentum L
Current through long wire AB, I1 = 25 A of the plate are related as m = q/2m.
Current through rectangular loop, I2 = 15 A Solution :
m 2I I Consider a small strip of thickness dx at a distance x from
\ Force on the arm PQ, F1 = 0 1 2 ´ length PQ
4p r1 the centre .
q
10 -1 ´ 2 ´ 25 ´15 ´ 25 ´10 -2 Charge per unit area =
F1 = pr 2
2.0 ´ 10 -2
\ charge in this strip is
= 9.375 × 10–4 (towards AB)
Force on the arm RS, q
q' = (2px)dx [Q dA = (2px )dx ]
m 2I I pr 2
F2 = 0 1 2 ´ length RS
4 p r2 2p
Time period of one rotation, T =
w
10-7 ´ 2 ´ 25 ´ 15 ´ 25 ´ 10-2
= q ' q ( 2px )dx
12 ´ 10-2 \ current i = =
T 2p
pr 2 .
w
= 1.563×10-4 (away from AB)
(2qxdx)w
\ Effective force on the loop, \i =
2pr 2
F = F1 - F2 = 9.375´10-4 - 1.563´10-4 = 7.812 ´104 N
æ 2qwxdx ö wq
(towards AB) dm = i ´ A = ç ÷ (px 2 ) = 2 x 3dx
Example 24. è 2pr ø
2
r
A rigid wire consists of a semicircular portion of radius R r
wq wq æ r 4 ö wqr 2
and two straight sections. The wire is partially immersed m=ò (x 3 )dx = ç ÷= 4
in a perpendicular magnetic field B as shown in the figure. 0 r2 r2 è 4 ø
Find the magnetic force on the wire if it carries a current i.
wqr 2
× × × × × × \m = ........(1)
4
B R C
× × × × × × Now to find angular momentum of disc, consider a circular
strip of radius x and thickness dx.
× × × × × ×
m
× × × × × × Mass of strip, m' = (2 px )dx
A D pr 2
Solution : Speed of each point of strip is wx.
FAB = ilB to the left, FCD = ilB to the right 2mw
Or dL = 2 x 3 dx
\ FAB + FCD = 0 r
Net force is FBC r
2 mwx 3
× × × × × \L = ò
0
r2
dx

× dl × × × mwr 2
d L= …… (2)
× × × × × 2
dl = Rdq, dF = idlB = iRBdq q
From eqs. (1) & (2), m = L
p 2m

ò
Fx = dFx = 0 from symmetry
0
Example 26.
A vertical straight conductor X of length 0.5 m is situated
p p p in a uniform horizontal magnetic field, of 0.1T (i) calculate
F y = ò dF y = ò iRBdq sin q = iRBò sin qdq = -iRB cos q the force on X when a current of 4A is passed into it (ii)
0 0 0 through what angle must X be turned in a vertical plane so
\ Fy = 2iRB in upward direction. that the force on X is halved ?
Free eBooks on @neetquestionpaper2020

706 Physi cs
Solution . Example 29.
(i) Here L = 0.5 m, B = 0.1T, I = 4A, F = ? A rectangular coil of 50 turns hungs vertically in a uniform
F = BIL = 0.1× 4 × 0.5 = 0.2N magnetic field of magnitude 10–2 T, so that the plane of the
coil is parallel to the field, the mean height of the coil is
F 5cm and its mean width is 2cm. Calculate the strength of
the current the must pass through the coil in order to
2
= deflect it 30° if the torsional constant of the suspension is
a
q = 90° – a
B 10–9 Nm per degree.
Solution :
N = 50 turns, B = 10–2T, π = 30°
C = 10–9 Nm per degree,
A = 5 × 2 cm2 = 10 × 10–4 m2 = 10–3 m2
(ii) Let a be the required angle. Torque = BINA cos q = Cq
The force on the conductor
Cq 10 -9 ´ 30
\ I= = -2 = 6.9 × 10-5
1 0.2 BNA cos q 10 ´ 50 ´ 10-3 cos 30o
F' = F = 0.1N
2 2 A = 69mA
F' = BIL sinq Example 30.
A copper wire has 1.0 × 1029 free electrons per cubic metre,
1 a cross sectional area of 2.0 mm2 and carries a current of
or BIL = BIL sin(90° - a )
2 5.0A. Calculate the force acting on each electron if the
wire is now placed in a magnetic field of flux density 0.15
1 æ 1ö
or cos a or a = cos -1 ç ÷ = 60° T which is perpendicular to the wire (e=1.6 × 1019C)
2 è 2ø Solution :
Example 27. n = 10 × 1029 m–3, A = 2.0 mm2 = 2 × 10–6 m2
An electron beam moving with a velocity of 106 ms–1 I = 5.0A, B = 0.15 T, F = ?.
through a uniform magnetic field of 0.1T, which is I
I = nevA or v =
perpendicular to the direction of the beam. Calculate the nev
force on an electron if the electron charge is 1.6 ×10–19C. Be ´ I BI 0.15 ´ 5
F = Bev = = =
Solution : neA nA 1.0 ´ 10 29 ´ 2 ´ 10 -6
–24
v=10–6ms –1, B = 0.IT, q = 1.6 × 10 – 19C = 3.75×10 N
F = Bqv = 0.1 × 106 × 1019 = 1.6 × 10–14 N Example 31.
A horizontal rop PQ of mass 10g and length 0.10m is placed
Example 28.
on a smooth plane incident at 60o to the horizontal. A
A narrow vertical rectangular coil is suspended from the uniform vertical magnetic field of value B is applied in the
middle of its upper side with its plane parallel to a uniform region of PQ. Calculate B if the rod remains stationary on
horizontal magnetic field of 0.02 T. The coil has 10 turns the plane when a current of 1.73 A flow in the rod. What is
and the lengths of its vertical and horizontal sides are 0.1 the direction of the current in the rod.
m and 0.05 m respectively. Calculate the torque on the coil P Q F cos 60°
when a current of 5A is passed into it.
° 60

B
What would be the new value of the torque if the plane of 60°
mg sin

the vertical coil was initially at 60o to the magnetic field F


and a current of 5A was passed into the coil.
Solution : 60°
60°
B = 0.02 T, N = 10 turns
Solution :
A = 1 × b = 0.1 × 0.05 = 0.005 m2
Force on the rod along the horizontal direction, F = BIl
I = 5A Force up the plane = BIl cos 60°.
Torque = BINA = 0.02 × 5 × 10 × 0.005 Component of the weight of the rod down the plane
= 0.005 Nm = 5 × 10–3Nm = mg sin 60°
For the rod to be stationary.
New value of the torque when the plane of the vertical coil
BIl cos 60° = mg sin 60°
was at 60° to the magnetic field.
= BINA cos q = 5 × 10–3 cos 60° mg 10 ´ 10-3 ´ 9.8 3
or B = tan 60° = = 0.98T
Il 1.73 ´ 0.1
1 As the force is to the right of PQ, applying Fleming's left
= 5× 10–3 × = 2.5 × 10–3 Nm.
2 hand rule, the direction of current should be from P to Q.
Free eBooks on @neetquestionpaper2020

Moving Charges and Magnetism 707


Example 32. Example 34.
If the coil of a moving coil galvanometer having 10 turns A circular coil of 50 turns and area 1.25 ×103m2 is pivoted
and of resistance 4W is removed and is replaced by a second a vertical diameter in a uniform horizontal magnetic field
coil having 100 turns and of resistance 160W. Calculate
and carries a current of 2A. When the coil held with its
(a) the factor by which the current sensitivity changes and
plane in a north-south direction, it experiences a couple
(b) the factor by which the voltage sensitivity changes.
of 0.04 N. When its plane is east-west the corresponding
Solution :
couple is 0.03 Nm. Calculate the magnetic flux density.
Given : N1 = 10 turns, R = 4W
Solution :
N2 = 100 turns, R2 = 160W
Let the plane of the coil make an angle π with the magnetic
æ q ö N AB
(a) Current sensitivity with the Ist coil, ç ÷ = 1 field when it is in the north-south direction.
è I ø1 C
At this position
æ qö N AB
Current sensitivity with the 2nd coil ç ÷ = 2 Torque T1 = BINA cos π
è Iø2 C
When the plane of the coil is in the east-west direction
æ qö Torque T2 = BINA sinq
çè ÷ø
I 2 N 2 100
\ = = = 10 T12 + T22 = B2 I2 N 2 A 2 (cos 2 q + sin 2 q)
æ qö N1 10
çè ÷ø
I 1
2 T12 + T22 ( 0.03) 2 + ( 0.04) 2
æ q ö N AB \ B = =
(b) Voltage sensitivity with the Ist coil ç ÷ = 1 I2 N 2 A 2 (2 ´ 50 ´ 1.25 ´ 10 –3 ) 2
è V ø 1 CR1
0.0025
æ qö N 2 AB =
Voltage sensitivity with the 2nd coil ç ÷ = (2 ´ 50 ´ 1.25 ´ 10 –3 )
è Vø 2 CR 2
æ qö 0.05
\ B= = 0.4T
çè ÷ø 2 ´ 50 ´ 1.25 ´ 10 –3
v 2 N R 100 4 1
\ = 2 1= ´ =
æ qö N1 R 2 10 160 4 Example 35.
çè ÷ø A proton having a speed of 5.0 × 10–6 m/s in a magnetic
v 1
Example 33. field feels a force of 8.0 × 1014 N towards the west when it
Two galvanometers, which are otherwise identical are fitted moves vertically upward. When moving horizontally in a
with different coils. One has a coil of 50 turns and resistance northern direction it feels no force. What is the magnitude
10W while the other has 500 turns and a resistance of and direction of the magnetic field in the region?
600W . What is the ratio of the deflection when each is N
connected in turns to a cell of e.m.f 2.5V and internal
resistance 50W ?
Solution :
N1 = 50, R1 =10W, N 2 = 500, R 2 = 600W E W

V = 2.5V, q1 : q2 = ?
C E
I1 q1 = …… (i)
N1BA R1 + r S
C E Solution :
I2 q2 = …… (ii) A charged particle feels no force if it moves along the
N 2 BA R2 + r
direction of the field. As the proton feels no force when
From eqs. (i) and (ii),
moving north, the field must be in the north south direction.
N 2 q1 R 2 + r
= If the left hand rule applied to field, the direction of the field
N1 q 2 R1 + r B points towards the north.
q1 R 2 + r N1 [ F < Bqv
or =
q 2 R1 + r N 2
600 + 50 50 650 F 8.0 ´ 10 –14
= = B= = = 0.1T
10 + 50 500 60 ´ 10 qv 1.6 ´ 10-19 ´ 5.0 ´ 106
= 13 : 12. towards the north.
Free eBooks on @neetquestionpaper2020

708 Physi cs

19.2
Solve following problems with the help of above text and 9. A charged particle is whirled in a horizontal circle on a
examples : frictionless table by attaching it to a string fixed at one
1. A charge q is moving with a velocity v parallel to a point. If a magnetic field is switched on in the vertical
magnetic field B. Force on the charge due to magnetic direction, the tension in the string
field is (a) will increase
(a) q v B (b) q B/v (b) will decrease
(c) zero (d) B v/q (c) will remain the same
2. A proton moving with a constant velocity passes through (d) may increase or decrease
a region of space without any change in its velocity. If E 10. If a charged particle projected in a gravity-free room
and B represent the electric and magnetic fields deflects, then
respectively, this region of space may not have (a) there must be an electric field
(a) E = 0, B = 0 (b) E = 0, B ¹ 0 (b) there must be a magnetic field
(c) E ¹ 0, B = 0 (d) E ¹ 0, B ¹ 0 (c) both fields cannot be zero
3. A proton (mass m and charge + e) and an alpha particle (d) None of these
(mass 4 m and charge + 2e) are projected with the same 11. A charged particle goes undeflected in a region containing
kinetic energy at right angles to a uniform magnetic field. electric and magnetic fields. It is possible that [more than
Which one of the following statements will be true? one may be correct]
r r r r
(a) The alpha-particle will be bent in a circular path with (a) E ||B, v || E
a smaller radius than that of the proton r r
(b) The radius of the path of the alpha-particle will be (b) E is not parallel to B
greater than that of the proton r r r r
(c) The alpha-particle and the proton will be bent in a (c) v || B but E is not parallel to B
circular path with the same radius r r r r
(d) E || B but v is not parallel to E
(d) The alpha-particle and the proton will go through the
field in a straight line. 12. If a charged particle goes unaccelerated in a region
4. An electron having a charge e moves with a velocity v in containing electric and magnetic fields, then
X-direction. An electric field acts on it in Y-direction? [more than one may be correct]
r r
The force on the electron acts in (a) E must be perpendicular to B
(a) positive direction of Y-axis r r
(b) negative direction of Y-axis (b) v must be perpendicular to B
r r
(c) positive direction of Z-axis (c) v must be perpendicular to E
(d) negative direction of Z-axis r r
5. A charged particle enters into a magnetic field with a (d) E must be perpendicular to v B
velocity vector making an angle of 30º with respect to 13. Two free parallel wires carrying currents in the opposite
the direction of magnetic field. The path of the particle is directions
(a) circular (b) helical (a) attract each other
(c) elliptical (d) straight line (b) repel each other
6. If electron velocity is 2 i + 3 j and it is subjected to (c) do not affect each other
magnetic field of 4 k, then its (d) get rotated to be perpendicular to each other
(a) speed will change (b) path will change 14. Two parallel wires carrying currents in the same direction
(c) both (a) and (b) (c) None of these attract each other because of
7. A positively charged particle moving due east enters a (a) potential difference between them
region of uniform magnetic field directed vertically (b) mutual inductance between them
upwards. This particle will (c) electric forces between them
(a) get deflected in vertically upward direction (d) magnetic forces between them
(b) move in circular path with an increased speed 15. A wire is placed parallel to the lines of force in a magnetic
(c) move in a circular path with decreased speed field and a current flows in the wire. Then
(d) move in a circular path with uniform speed (a) the wire will experience a force in the direction of the
8. The work done by a magnetic field, on a moving charge is magnetic field
r r (b) the wire will not experience any force at all
(a) zero because F acts parallel to v
r r (c) the wire will experience a force in a direction opposite
(b) positive because F acts perpendicular to v to the field
r r
(c) zero because F acts perpendicular to v (d) it experiences a force in a direction perpendicular
r r to lines of force
(d) negative because F acts parallel to v
Free eBooks on @neetquestionpaper2020

Moving Charges and Magnetism 709

16. A conducting circular loop of radius r carries a constant 21. A coil carrying a heavy current and having large number
current i. It is placed in a uniform magnetic field B such of turns is mounted in a N-S vertical plane. A current
that B is perpendicular to the plane of the loop. The
flows in the clockwise direction. A small magnetic needle
magnetic force acting on the loop is
(a) i r B (b) 2 p r i B at its centre will have its north pole in
(c) zero (d) p r i B (a) east-north direction (b) west-north direction
17. A straight steel wire of length l has a magnetic moment M. (c) east-south direction (d) west-south direction
When it is bent in the form of a semi-circle its magnetic 22. If E and B denote electric and magnetic fields respectivley,
moment will be which of the following is dimensionless?
(a) M (b) M/p
(c) 2 M/p (d) Mp E E
(a) m 0e 0 (b) m 0 e 0
18. A current carrying coil is subjected to a uniform magnetic B B
field. The coil will orient so that its plane becomes
(a) inclined at 45º to the magneic field 2
E m0
æBö
(b) inclined at any arbitrary angle to the magnetic field (c) m 0e 0 ç ÷ (d) ´
(c) parallel to the magnetic field èEø e0 B
(d) perpendicular to magnetic field
23. ABCD is a square loop made of a uniform conducting
19. Two long wires are hanging freely. They are joined first in
parallel and then in series and then are connected with a wire. A current enters the loop at A and leaves at D. The
battery. In both cases which type of force acts between magnetic field is
the two wires?
(a) Attraction force when in parallel and repulsion force B C
when in series
(b) Repulsion force when in parallel and attraction force
when in series
(c) Repulsion force in both cases
(d) Attraction force in both cases A D
20. The magnetic dipole moment of current loop is independent
of
(a) number of turns (a) zero only at the centre of the loop
(b) area of loop (b) maximum at the centre of the loop
(c) current in the loop (c) zero at all points outside the loop
(d) magnetic field in which it is lying (d) zero at all points inside the loop

ANSWER KEY
1. (c) 2. (c) 3. (c) 4. (b) 5. (b) 6. (b) 7. (d) 8. (c) 9. (d) 10. (c) 11. (a) 12. (a,b)
13. (b) 14. (d) 15. (b) 16. (c) 17. (c) 18. (d) 19. (a) 20. (d) 21. (b) 22. (a) 23. (a)
Free eBooks on @neetquestionpaper2020

710 Physi cs

Very Short / Short Answer Questions 11. Two identical circular wires P and Q Q
each of radius R and carrying current I
1. Under what condition, an electron moving through a magnetic
‘I’ are kept in perpendicular planes P
field experiences maximum force?
such that they have a common centre I
2. What is the nature of the magnetic field in a moving coil
as shown in the figure. Find the
galvanometer? magnitude and direction of the net
3. Using Ampere’s circuital law, derive the magnetic field due magnetic field at the common centre of the two coils.
to a current carrying solenoid.
4. Write expressions using Biot-Savart’s law for the magnetic [Delhi Board - 2012].
field at a point due to a current element, in C.G.S. and S.I. 12. Obtain, with the help of necessary diagram, the expression
system. for the magnetic field in the interior of a toroid carrying
5. Write the expression in vector form, for the magnetic force current. [Outside Delhi - 2011 COMPTT.].
r r
F acting on a charged particle moving with velocity V Long Answer Questions
r
in the presence of a magnetic field B.
13. Explain with the help of a labelled diagram with underlying
[Outside Delhi - 2012 COMPTT.].
principle, construction and working of a cyclotron.
6. Depict the direction of the magnetic field lines due to a
circular current carrying loop. 14. Describe with a labelled diagram the principle, construction
[Delhi Board - 2010 COMPTT.]. and working of a moving coil galvanometer.
7. A proton and a deuteron, each moving with velocity vr 15. A coil of 200 turns has a cross-sectional area 900 mm2. It
r carries a current of 2A. The plane of the coil is perpendicular
enter simultaneously in the region of magnetic field B to a uniform magnetic field of 0.5 T. Calculate (i) the magnetic
acting normal to the direction of velocity. Trace their moment of the coil and (ii) the torque acting on the coil.
trajectories establishing the relationship between the two.
16. (a) State Biot-Savart law. Deduce the expression for the
[Delhi Board - 2010 COMPTT.].
magnetic field due to a circular current carrying loop at a
8. A circular coil of ‘N’ turns and diameter ‘d’ carries a current
point lying on its axis.
‘I’. It is unwound and rewound to make another coil of
diameter ‘2d’, current ‘I’ remaining the same. Calculate the (b) Two long parallel wires carry currents I1 and I2 flowing
ratio of the magnetic moments of the new coil and the in the same direction. When a third current carrying wire
original coil. is placed parallel and coplanar in between the two, find
[Outside Delhi - 2012]. the condition when the third wire would experience no
9. A circular coil of closely wound N turns and radius r carries force due to these two wires.
a current I. [Outside Delhi - 2012]. [Outside Delhi - 2012 COMPTT.].
Write the expressions for the following : r
(i) The magnetic field at its centre 17. (a) Write the expression for the force, F , acting on a charged
(ii) The magnetic moment of this coil particle of charge ‘q’, moving with a velocity vr in the
10. A rectangular loop of wire of r r
presence of both electric field E and magnetic field B .
size 4 cm × 10 cm carries a I = 5A
steady current of 2 A. A 4 cm Obtain the condition under which the particle moves
straight long wire carrying 5 A undeflected through the fields.
current is kept near the loop as (b) A rectangular 100p of size l × b carrying a steady current
2A 10 cm 2A r
shown. If the loop and the wire I is placed in a uniform magnetic field B . Prove that the
are coplanar, find r r r,
1cm torque t acting on the loop is given by rt = m ´B
(i) the torque acting on the loop and r
where m is the magnetic moment of the loop.
(ii) the magnitude and direction of the force. [Outside Delhi - 2012].
[Delhi Board - 2012].
Free eBooks on @neetquestionpaper2020

Moving Charges and Magnetism 711


Multiple Choice Questions 22. A non-planar loop of conducting wire carrying a current I is
placed as shown in the figure. Each of the straight sections
18. P, Q and R are long straight wires in air, carrying currents as of the loop is of length 2a. The magnetic field due to this
shown. The force on Q is directed loop at the point P (a, 0, a) points in the direction
P Q R
(a) to the left
z
(b) to the right
20A 40A 60A
(c) ^ to the plane of the diagram y
I
(d) along the current in Q
19. The negatively and uniformly charged nonconducting disc
as shown is rotated clockwise.
The direction of themagnetic field at point A in the plane of x
the disc is
2a
(a) into the page
(b) out of the page 1 1
A (a) (- ˆj + k)
ˆ
(b) (-ˆj + kˆ + ˆi)
(c) up the page 2 3

(d) down the page 1 ˆ ˆ ˆ 1 ˆ ˆ


(c) (i + j + k) (d) (i + k)
20. A beam of electron passes undeflected through mutually 3 2
perpendicular electric and magnetic fields. If the electric field 23. A small current element of length dl and carrying current is
is switched off, and the same magnetic field is maintained, the placed at (1, 1, 0) and is carrying current in ‘+z’ direction. If
electrons move r r
(a) in a circular orbit (b) along a parabolic path magnetic field at origin be B1 and at point (2, 2, 0) be B2
(c) along a straight line (d) in an elliptical orbit then
21. A current carrying loop is placed in a uniform magnetic field r r r r
with four different orientations X, Y, Z and W is shown in (a) B1 = B2 (b) | B1 | = | 2B2 |
the figure. The correct decreasing order of potential energy r r r r
(c) B1 = -B2 (d) B1 = -2B2
is
24. A current carrying coil is subjected to a uniform magnetic
B field. The coil will orient so that its plane becomes
(a) inclined at 45° to the magnetic field
B (b) inclined at any arbitrary angle to the magnetic field

(c) parallel to the magnetic field
n^ (d) perpendicular to the magnetic field
X Y
25. An electron enters a region where magnetic field (B) and
B n^ electric field (E) are mutually perpendicular, then
(a) it will always move in the direction of B
B (b) it will always move in the direction of E
(c) it always possesses circular motion
n^ (d) it can go undeflected also
Z W
(a) X > Y > Z > W (b) Z > W> X > Y
(c) X > W > Y> Z (d) X> Y > W > Z
Free eBooks on @neetquestionpaper2020

712 Physi cs

1. A straight wire of diameter 0.5 mm, carrying a current of 8. A galvanometer acting as a voltmeter will have a
1A is replaced by another wire of 1mm diameter carrying [CBSE PMT 2004]
the same current. The strength of magnetic field far away (a) low resistance in series with its coil.
is (b) high resistance in parallel with its coil
[CBSE PMT 1995, 97, 99] (c) high resistance in series with its coil
(a) unchanged (d) low resistance in parallel with its coil
(b) quarter of its earlier value 9. A galvanometer of 50 ohm resistance has 25 divisions. A
(c) half of the earlier value current of 4 × 10–4 ampere gives a deflection of on division.
(d) twice the earlier value To convert this galvanometer into a voltmeter having a range
2. Two equal electric currents are flowing perpendicular to each of 25 volts, it should be connected with a resistance of
other as shown in figure. AB and CD are perpendicular to [CBSE PMT 2004]
each other and symmetrically placed with respect to the (a) 2450 W in series (b) 2500 W in series
currents. Where do we expect the resultant magnetic field (c) 245 W in series (d) 2550 W in series
to be zero? [CBSE PMT 1996]
10. An electron moves in a circular orbit with a uniform speed
(a) on AB C I B
v. It produces a magnetic field B at the centre of the
(b) on CD circle. The radius of the circle is proportional to
I
O [CBSE PMT 2005]
(c) on both AB & CD
A D B B v v
(d) on both OD & BO (a) (b) (c) (d)
3. The magnetic field (dB) due to small element (dl) at a distance v v B B
r 11. A very long straight wire carries a current I. At the instant
( r ) from element carrying current i, is [CBSE PMT 1996] ®
when a charge + Q at point P has velocity v , as shown,
æ ® rö æ ® rö the force on the charge is [CBSE PMT 2005]
m0 ç dl´ r ÷ m 0 2 ç dl´ r ÷
(a) dB = i (b) dB = i
4p è r ø 4p è r 2 ø y
æ ® rö æ ® rö
m 0 2 ç dl´ r ÷ m0 ç dl´ r ÷ Q
(c) dB = i (d) dB = i
4p è r 2 ø 4p è r 3 ø P®
v
4. A 10eV electron is circulating in a plane at right angles to a I o x
uniform field at a magnetic induction 10–4 Wb/m2 (= 1.0
gauss). The orbital radius of the electron is
[CBSE PMT 1996] (a) along oy (b) opposite to oy
(a) 12cm (b) 16cm (c) 11 cm (d) 18 cm (c) along ox (d) opposite to ox
5. A coil of one turn is made of a wire of certain length and 12. A square current carrying loop is suspended in a uniform
then from the same length a coil of two turns is made. If the magnetic field acting in ther plane of the loop. If the force on
same current is passed in both the cases, then the ratio of one arm of the loop is F , the net force on the remaining
the magnetic inductions at their centres will be
three arms of the loop is [CBSE-PMT 2010]
[CBSE PMT 1998] r r
(a) 2 : 1 (b) 1.4 (c) 4 : 1 (d) 1 : 2 (a) 3 F (b) – F
6. A charged particle moves through a magnetic field in a r r
direction perpendicular to it. Then the [CBSE PMT 2003] (c) – 3 F (d) F
(a) velocity remains unchanged 13. A current loop consists of two identical semicircular parts
(b) speed of the particle remains unchanged each of radius R, one lying in the x-y plane and the other in
(c) direction of the particle remains unchanged x-z plane. If the current in the loop is i., the resultant magnetic
(d) acceleration remains unchanged field due to the two semicircular parts at their common centre
7. A long solenoid carrying a current produces a magnetic is [CBSE-PMT 2010]
field B along its axis. If the current is double and the number m0 i m 0i
(a) (b)
of turns per cm is halved, the new value of the magnetic 2R 2 2R
field is [CBSE PMT 2003] m 0i m 0i
(a) 4 B (b) B/2 (c) B (d) 2 B (c) (d)
2R 4R
Free eBooks on @neetquestionpaper2020

Moving Charges and Magnetism 713


14. A current carrying loop in the form of a right angle isosceles mn and K = 2mp2n2R2
triangle ABC is placed in a uniform magnetic field acting (a) B=
e
along AB. If the magnetic force on the arm BC is F, what is
2pm n
the force on the arm AC? [CBSE-PMT2011] (b) B = and K = m2pnR2
r A e
(a) - 2F
r (c) B = 2pmn and K = 2mp2n2R2
(b) -F e
r mn
(c) F (d) B = and K = m2pnR2
r B C e
(d) 2F 20. A proton carrying 1 MeV kinetic energy is moving in a
15. A uniform electric field and uniform magnetic field are acting circular path of radius R in uniform magnetic field. What
along the same direction in a certain region. If an electron is should be the energy of an a-particle to describe a circle of
projected in the region such that its velocity is pointed along same radius in the same field? [CBSE-PMT 2012M]
the direction of fields, then the electron [CBSE-PMT2011] (a) 2 MeV (b) 1 MeV
(a) will turn towards right of direction of motion (c) 0.5 MeV (d) 4 MeV
(b) speed will decrease 21. A magnetic needle suspended parallel to a magnetic field
(c) speed will increase
requires 3 J of work to turn it through 60°. The torque
(d) will turn towards left direction of motion
16. A square loop, carrying a steady current I, is placed in a needed to maintain the needle in this position will be
horizontal plane near a long straight conductor carrying a [CBSE-PMT 2012M]
steady current I1 at a distance d from the conductor as 3
(a) 2 3J (b) 3J (c) 3J (d) J
shown in figure. The loop will experience 2
[CBSE-PMT 2011M] 22. A current loop in a magnetic field [NEET 2013]
I1 (a) can be in equilibrium in one orientation
(b) can be in equilibrium in two orientations, both the
d I equilibrium states are unstable
(c) can be in equilibrium in two orientations, one stable
while the other is unstable
(d) experiences a torque whether the field is uniform or
non-uniform in all orientations
I 23. Wires 1 and 2 carrying currents i1 and i2 respectively are
(a) a net repulsive force away from the conductor
inclined at an angle q to each other. What is the force on a
(b) a net torque acting upward perpendicular to the
small element dl of wire 2 at a distance of r from wire 1 (as
horizontal plane
shown in figure) due to the magnetic field of wire 1?
(c) a net torque acting downward normal to the horizontal
[AIEEE 2002]
plane
m0
(d) a net attractive force towards the conductor (a) i1i 2 dl tan q
2 pr 1 2
17. Charge q is uniformly spread on a thin ring of radius R. The
ring rotates about its axis with a uniform frequency f Hz. m0
(b) i1i 2 dl sin q
The magnitude of magnetic induction at the centre of the 2pr i1 i2
r
ring is [CBSE-PMT 2011M] m0
(c) i i
1 2 dl cos q
m 0 qf m0 q m0 q m 0 qf 2 pr q dl
(a) (b) (c) 2pf R (d) m0
2R 2f R 2 pR (d) i1i 2 dl sin q
4pr
18. Two similar coils of radius R are lying concentrically with
24. The time period of a charged particle undergoing a circular
their planes at right angles to each other. The currents
motion in a uniform magnetic field is independent of its
flowing in them are I and 2 I, respectively. The resultant
[AIEEE 2002]
magnetic field induction at the centre will be
(a) speed (b) mass
[CBSE-PMT 2012S] (c) charge (d) magnetic induction
25. A particle of mass M and charge Q moving with velocity vr
5m 0 I 3m0 I m0 I m0 I
(a) (b) (c) (d) describe a circular path of radius R when subjected to a
2R 2R 2R R
uniform transverse magnetic field of induction B. The work
19. An alternating electric field, of frequency v, is applied across done by the field when the particle completes one full circle
the dees (radius = R) of a cyclotron that is being used to is [AIEEE 2003]
accelerate protons (mass = m). The operating magnetic field
æ Mv 2 ö
(B) used in the cyclotron and the kinetic energy (K) of the (a) ç ÷ 2 pR (b) zero
proton beam, produced by it, are given by è R ø
[CBSE-PMT 2012S] (c) BQ2pR (d) BQv2pR
Free eBooks on @neetquestionpaper2020

714 Physi cs
26. A particle of charge 16 × 10–18 coulomb moving with 34. A uniform electric field and a uniform magnetic field are
velocity 10 ms–1 along the x-axis enters a region where a acting along the same direction in a certain region. If an
magnetic field of induction B is along the y-axis, and an electron is projected along the direction of the fields with a
electric field of magnitude 104V/m is along the negative certain velocity then [AIEEE 2005]
z-axis. If the charged particle continues moving along the (a) its velocity will increase
x-axis, the magnitude of B is [AIEEE 2003] (b) its velocity will decrease
(a) 10 3 Wb / m 2 (b) 10 5 Wb / m 2 (c) it will turn towards left of direction of motion
(d) it will turn towards right of direction of motion
(c) 1016 Wb / m 2 (d) 10 -3 Wb / m 2
35. In a region, steady and uniform electric and magnetic fields
27. A current i ampere flows along an infinitely long straight are present. These two fields are parallel to each other. A
thin walled tube, then the magnetic induction at any point charged particle is released from rest in this region. The
inside the tube is [AIEEE 2004] path of the particle will be a [AIEEE 2006]
μ 0 2i (a) helix (b) straight line
(a) . tesla (b) zero
4π r (c) ellipse (d) circle
2i 36. A long solenoid has 200 turns per cm and carries a current i.
(c) infinite (d) tesla
r The magnetic field at its centre is 6.28 × 10–2 Weber/m2.
28. A long wire carries a steady current. It is bent into a circle of Another long solenoid has 100 turns per cm and it carries a
one turn and the magnetic field at the centre of the coil is B. i
current . The value of the magnetic field at its centre is
It is then bent into a circular loop of n turns. The magnetic 3
field at the centre of the coil will be [AIEEE 2004] [AIEEE 2006]
(a) 2n B (b) n2 B (c) n B (d) 2 n2 B (a) 1.05 × 10–2 weber/m2 (b) 1.05 × 10–5 weber/m2
29. The magnetic field due to a current carrying circular loop of (c) 1.05 × 10–3 weber/m2 (d) 1.05 × 10–4 weber/m2
radius 3 cm at a point on the axis at a distance of 4 cm from 37. A long straight wire of radius a carries a steady current i.
the centre is 54 mT. What will be its value at the centre of The current is uniformly distributed across its cross section.
loop ? [AIEEE 2004] The ratio of the magnetic field at a/2 and 2a is
(a) 125 mT (b) 150 mT (c) 250 mT (d) 75 mT [AIEEE 2007]
30. Two long conductors, separated by a distance d carry current (a) 1/2 (b) 1/4 (c) 4 (d) 1
I1 and I2 in the same direction. They exert a force F on each 38. A current I flows along the length of an infinitely long,
other. Now the current in one of them is increased to two straight, thin walled pipe. Then [AIEEE 2007]
times and its direction is reversed. The distance is also (a) the magnetic field at all points inside the pipe is the
increased to 3d. The new value of the force between them is same, but not zero
[AIEEE 2004] (b) the magnetic field is zero only on the axis of the pipe
2F F F (c) the magnetic field is different at different points inside
(a) - (b) (c) –2 F (d) -
3 3 3 the pipe
31. Two concentric coils each of radius equal to 2 p cm are placed (d) the magnetic field at any point inside the pipe is zero
at right angles to each other. 3 ampere and 4 ampere are the 39. A charged particle with charge q enters a region of constant,
currents flowing in each coil respectively. The magnetic ur ur
uniform and mutually orthogonal fields E and B with a
induction in weber/m2 at the centre of the coils will be r ur ur
velocity v perpendicular to both E and B , and comes out
(m0 = 4p ´ 10-7 Wb / A.m) [AIEEE 2005] r
without any change in magnitude or direction of v . Then
(a) 10–5 (b) 12×10 –5 (c) 7 × 10–5 (d) 5 × 10–5 [AIEEE 2007]
32. A charged particle of mass m and charge q travels on a r ur ur 2 r ur ur 2
circular path of radius r that is perpendicular to a magnetic (a) v = B ´ E / E (b) v = E ´ B / B
field B. The time taken by the particle to complete one r ur ur 2 r ur ur
revolution is [AIEEE 2005] (c) v = B ´ E / B (d) v = E ´ B / E 2
40. A charged particle moves through a magnetic field
2pq 2 B 2pmq 2pm 2pqB perpendicular to its direction. Then [AIEEE 2007]
(a) (b) (c) (d)
m B qB m (a) kinetic energy changes but the momentum is constant
33. A magnetic needle is kept in a non-uniform magnetic field. It (b) the momentum changes but the kinetic energy is
experiences [AIEEE 2005] constant
(a) neither a force nor a torque (c) both momentum and kinetic energy of the particle are
(b) a torque but not a force not constant
(c) a force but not a torque (d) both momentum and kinetic energy of the particle are
(d) a force and a torque constant
Free eBooks on @neetquestionpaper2020

Moving Charges and Magnetism 715


41. Two identical conducting wires AOB and COD are placed
at right angles to each other. The wire AOB carries an
electric current I1 and COD carries a current I2 . The
magnetic field on a point lying at a distance d from O, in a B B
direction perpendicular to the plane of the wires AOB and (a) (b)
COD, will be given by [AIEEE 2007]
1 R R
m0 m0 æ I1 + I2 ö 2
(a) (I12 + I 2 2 ) (b) ç ÷
2 pd 2p è d ø
1
(c)
m0
2pd
( I12 + I 22 2) (d)
m0
2pd
( I1 + I2 )
(c)
B
(d)
B

DIRECTIONS : Qs 42 and 43 are based on the following


paragraph. R R
A current loop ABCD is held fixed on the B 45. A circular loop of radius 0.3 cm lies parallel to a much bigger
plane of the paper as shown in the figure. a A circular loop of radius 20 cm. The centre of the smaller loop
The arcs BC (radius = b) and DA (radius is on the axis of the bigger loop. The distance between their
= a) of the loop are joined by two straight I 30° centres is 15 cm. If a current of 2.0 A flows through the
1 I
wires AB and CD. A steady current I is O smaller loop, then the flux linked with bigger loop is
flowing in the loop. Angle made by AB [JEE Main 2013]
D
and CD at the origin O is 30°. Another
b C (a) 9.1 × 10–11 weber (b) 6 × 10–11 weber
straight thin wire with steady current I1
(c) 3.3 × 10–11 weber (d) 6.6 × 10–9 weber
flowing out of the plane of the paper is kept at the origin.
46. A conducting circular loop of radius r carries a constant
42. The magnitude of the magnetic field (B) due to the loop current I. It is placed in a uniform magnetic field B such that
ABCD at the origin (O) is [ AIEEE 2009] B is perpendicular to the plane of the loop. The magnetic
m o I (b - a ) force acting on the loop is [IIT JEE 1983]
(a) (a) BIR (b) 2 p (BIR)
24 ab
(c) Zero (d) p (BIR)
mo I é b - a ù 47. A battery is connected between two points A and B on the
(b) circumference of a uniform conducting ring of radius r and
4p êë ab úû
resistance R. One of the arcs AB of the ring subtends an
mo I angle q at the centre.The value of the magnetic induction at
(c) [2(b - a) + p 3 ( a + b)] the centre due to the current in the ring is
4p
[IIT JEE 1995 S]
(d) zero (a) proportional to 2 (180° – q)
43. Due to the presence of the current I1 at the origin (b) inversely proportional to r
(a) the forces on AD and BC are zero. [ AIEEE 2009] (c) zero, only if q = 180°
(b) the magnitude of the net force on the loop is given by (d) zero for all values of q
I1I 48. Two long parallel wires are at a distance 2d apart. They
m o [2(b - a ) + p (a + b] . carry steady equal currents flowing out of the plane of the
4p 3
paper, as shown. The variation of the magnetic field B along
(c) the magnitude of the net force on the loop is given by the line X'X is given by [IIT JEE 2005 AIEEE 2010]
m o II1
(b - a). B B
24ab
(d) the forces on AB and DC are zero. (a) x x' (b) x'
44. A charge Q is uniformly distributed over the surface of non-
conducting disc of radius R. The disc rotates about an axis d d
d d
perpendicular to its plane and passing through its centre
with an angular velocity w. As a result of this rotation a
B B
magnetic field of induction B is obtained at the centre of the
disc. If we keep both the amount of charge placed on the
disc and its angular velocity to be constant and vary the (c) x x' (d) x x'

radius of the disc then the variation of the magnetic induction


at the centre of the disc will be represented by the figure d d d d
[AIEEE 2012]
Free eBooks on @neetquestionpaper2020

716 Physi cs
ur If the magnetic moment of the loop is Nm0I0sin(300 t),
49. A magnetic field B = B0 Jˆ , exists in the region a < x < 2a,
uv then ‘N’ is [ IIT-JEE 2011]
and B = -B ˆj , in the region 2a < x < 3a, where B0 is a
0
positive constant. A positive point charge moving with a
r
velocity v = v ˆi , where v is a positive constant, enters
0 0
the magnetic field at x = a. The trajectory of the charge in
this region can be like [IIT JEE 2007]
B0
(a) 6 (b) 8 (c) 7 (d) 4
0 x 52. A circular wire loop of radius R is placed in the x-y plane
a 2a 3a
centered at the origin O. A square loop of side a(a<<R)
- B0 having two turns is placed with its centre at z = 3R along
z the axis of the circular wire loop, as shown in figure. The
plane of the square loop makes an angle of 45° with respect to
the z-axis. If the mutual inductance between the loops is given
(a) a 2a 3a
x m0 a 2
by p/2 , then the value of p is [ IIT-JEE 2012]
2 R
z
45°
z
a
3R
(b) x
a 2a 3a R y
O
x
(a) 6 (b) 7 (c) 5 (d) 8
z
53. A loop carrying current I lies in the x-y plane as shown in
the figure. The unit vector k̂ is coming out of the plane of
(c) x the paper. The magnetic moment of the current loop is
a 2a 3a
2 ˆ
(a) a I k [IIT-JEE 2012]
æp ö 2
z (b) çè + 1÷ø a I kˆ
2
æp ö 2
(c) - çè + 1÷ø a I kˆ
a 2a 3a 2
(d) x
(d) (2p + 1)a2 I kˆ
54. A particle of mass M and positive charge Q, moving with a
r
constant velocity u1 = 4iˆ ms-1 , enters a region of uniform
50. An electron and a proton are moving on straight parallel static magnetic field, normal to the x-y plane. The region
paths with same velocity. They enter a semi infinite region
of the magnetic field extends from x = 0 to x = L for all
of uniform magnetic field perpendicular to the velocity.
values of y. After passing through this region, the particle
Which of the following statement(s) is / are true?
emerges on the other side after 10 milliseconds with a
[IIT-JEE 2011]
r
(a) They will never come out of the magnetic field region. velocity u2 = 2( 3iˆ + ˆj ) ms -1 . The correct statement(s) is
(b) They will come out travelling along parallel paths. (are) [JEE Adv. 2013]
(c) They will come out at the same time. (a) The direction of the magnetic field is –z direction
(d) They will come out at different times. (b) The direction of the magnetic field is +z direction
51. A long circular tube of length 10 m and radius 0.3 m carries 50 pM
a current I along its curved surface as shown. A wire-loop of (c) The magnitude of the magnetic field units
3Q
resistance 0.005 ohm and of radius 0.1 m is placed inside the
100pM
tube with its axis coinciding with the axis of the tube. The (d) The magnitude of the magnetic field is units
current varies as I = I0cos(300 t) where I0 is constant. 3Q
Free eBooks on @neetquestionpaper2020

Moving Charges and Magnetism 717


55. A steady current I flows along an infinitely long hollow (a) In the region 0 < r < R, the magnetic field is non-zero
cylindrical conductor of radius R. This cylinder is placed (b) In the region R < r < 2R, the magnetic field is along
coaxially inside an infinite solenoid of radius 2R. The the common axis
solenoid has n turns per unit length and carries a steady (c) In the region R < r < 2R, the magnetic field is
current I. Consider a point P at a distance r from the tangential to the circle of radius r, centered on the
common axis. The correct statement(s) is (are) axis
[JEE Adv. 2013] (d) In the region r > 2R, the magnetic field is non-zero

1. A portion of a conductive wire is bent in the form of a (a) 4.5 × 10–5 T (b) 1.4 × 10–5 T
semicircle of radius r as shown below in fig. At the centre (c) 5.5 × 10–5 T (d) 2.6 × 10–5 T
of semicircle, the magnetic induction will be 8. An electron moving with kinetic energy 6×10–16 joules
enters a field of magnetic induction 6 × 10–3 weber/m2 at
i
r right angle to its motion. The radius of its path is
i (a) 3.42 cm (b) 4.23 cm
O (c) 5.17 cm (d) 7.7 cm
(a) zero (b) infinite 9. A cathode ray beam is bent in a circle of radius 2 cm by a
μ0 π i μ0 πi magnetic induction 4.5 × 10–3 weber/m2. The velocity of
(c) . gauss (d) . tesla electron is
4π r 4π r
(a) 3.43 × 107 m/s (b) 5.37 × 107 m/s
2. A current carrying conductor placed in a magnetic field
(c) 1.23 × 107 m/s (d) 1.58 × 107 m/s
experiences maximum force when angle between current
10. A proton, deutron and an a-particle enter a magnetic field
and magnetic field is
perpendicular to field with same velocity. What is the ratio of
(a) 3 p/4 (b) p/2 (c) p/4 (d) zero
the radii of circular paths?
3. A helium nucleus makes a full rotation in a circle of radius
0.8 meter in 2 sec. The value of the magnetic field induction (a) 1 : 2 : 2 (b) 2:1:1
B in tesla at the centre of circle will be (c) 1 : 1 : 2 (d) 1:2:1
11. A circular loop of area 0.02 m2 carrying a current of 10A,
(a) 2 ´ 10 -19 m 0 (b) 10 -19 / m 0 is held with its plane perpendicular to a magnetic field
induction 0.2 T. The torque acting on the loop is
(c) 10 -19 m 0 (d) 2 ´ 10 -20 / m 0
(a) 0.01 Nm (b) 0.001 Nm
4. A solenoid of length 1.5 m and 4 cm diameter possesses (c) zero (d) 0.8 Nm
10 turns per cm. A current of 5A is flowing through it, the 12. An electron moves in a circular arc of radius 10 m at a
magnetic induction at axis inside the solenoid is contant speed of 2 × 107 ms–1 with its plane of motion
(m 0 = 4 p ´ 10 - 7 weber amp -1 m - 1 ) normal to a magnetic flux density of 10–5 T. What will be
the value of specific charge of the electron?
(a) 4 p ´ 10 - 5 gauss (b) 2 p ´ 10 -5 gauss (a) 2 × 104 C kg–1 (b) 2 × 105 C kg–1
6
(c) 5 × 10 C kg –1 (d) 2 × 1011 C kg–1
(c) 4 p ´ 10 -5 tesla (d) 2 p ´ 10 -5 tesla 13. A current of 3 A is flowing in a linear conductor having a
5. Two concentric circular coils of ten turns each are situated length of 40 cm. The conductor is placed in a magnetic
in the same plane. Their radii are 20 and 40 cm and they field of strength 500 gauss and makes an angle of 30º with
carry respectively 0.2 and 0.4 ampere current in opposite the direction of the field. It experiences a force of
direction. The magnetic field in weber/m2 at the centre is magnitude
(a) m0/80 (b) 7m0 /80 (c) (5/4) m0 (d) zero (a) 3 × 10–4 N (b) 3 × 10–2 N
6. A wire of length L metre carrying a current I ampere is (c) 3 × 10 N 2 (d) 3 × 104 N
bent in the form of a circle. Its magnitude of magnetic 14. A milliammeter of resistance 5 ohms gives a full scale
moment will be deflection for a current of 15 mA. If the milliammeter is
(a) IL/4p (b) I2L2/4p (c) IL2 /4p (d) IL2 /8p to be used to measure currents upto 1.5 A the size of the
7. An electron (mass = 9 × 10 kg, charge = 1.6 × 10–19 C)
–31
resistance that must be attached to the milliammeter is
moving with a velocity of 106 m/s enters a magnetic field. If
(a) 0.0505 ohm (b) 0.505 ohm
it describes a circle of radius 0.1m, then strength of
(c) 5.05 ohm (d) 505 ohm
magnetic field must be
Free eBooks on @neetquestionpaper2020

718 Physi cs
15. A galvanometer of resistance 5 ohms gives a full scale 24. 1 Wbm–2 is equal to
deflection for a potential difference of 10 mV. To convert the (a) 104 G (b) 102 G (c) 10– 2 G (d) 10–4 G
galvanometer into a voltmeter giving a full scale deflection 25. If an ammeter is to be used in place of a voltmeter, then we
for a potential difference of 1V, the size of the resistance that must connect with the ammeter a
must be attached to the voltmeter is (a) low resistance in parallel
(a) 0.495 ohm (b) 49.5 ohm (b) high resistance in parallel
(c) 495 ohm (d) 4950 ohm (c) high resistance in series
16. A galvanometer of resistance 100 W gives a full scale (d) low resistance in series
deflection for a current of 10–5 A. To convert it into a ammeter 26. If in a circular coil A of radius R, current I is flowing and in
capable of measuring upto 1 A, we should connect a another coil B of radius 2R a current 2I is flowing, then the
resistance of ratio of the magnetic fields BA and BB, produced by them
(a) 1 W in parallel (b) 10–3 W in parallel will be
5
(c) 10 W in series (d) 100 W in series (a) 1 (b) 2 (c) 1/2 (d) 4
17. Two straight long conductors AOB and COD are
27. If an electron and a proton having same momenta enter
perpendicular to each other and carry currents I1 and I2.
perpendicular to a magnetic field, then
The magnitude of the magnetic induction at a point P at a
(a) curved path of electron and proton will be same
distance a from the point O in a direction perpendicular to
(ignoring the sense of revolution)
the plane ABCD is
(b) they will move undeflected
m0 m0 (c) curved path of electron is more curved than that of the
(a) (I1 + I 2 ) (b) (I1 - I 2 )
2pa 2pa proton
(d) path of proton is more curved
1
m0 2 2 2 m 0 I1 I 2 28. An ammeter reads upto 1 ampere. Its internal resistance is
(c) (I1 + I 2 ) (d) 0.81ohm. To increase the range to 10 A the value of the
2pa 2 p a I1 + I 2
required shunt is
18. Two long parallel wires P and Q are held perpendicular to
(a) 0.03 W (b) 0.3 W
the plane of paper with distance of 5 m between them. If P
and Q carry current of 2.5 amp. and 5 amp. respectively in (c) 0.9 W (d) 0.09 W
the same direction, then the magnetic field at a point half- 29. The radius of motion of a charged particle oscillating in a
way between the wires is magnetic field is

(a) m 0 / 17 (b) 3 m0 / 2 p mB mv
(a) (b)
qv qB
(c) m 0 / 2 p (d) 3 m 0 / 2 p
19. A charged particle with velocity 2 × 103 m/s passes mq qv
undeflected through electric and magnetic field. Magnetic (c ) (d)
vB mB
field is 1.5 tesla. The electric field intensity would be
30. Magnetic effect of current was discovered by
(a) 2 × 103 N/C (b) 1.5 × 103 N/C
3 (a) Faraday (b) Oersted
(c) 3 × 10 N/C (d) 4/3 × 10–3 N/C
(c) Kirchhoff (d) Joule
20. In cyclotron the gyro radius is
31. If we double the radius of a coil keeping the current through
(a) proportional to momentum
it unchanged, then the magnetic field at any point at a large
(b) proportional to energy
distance from the centre becomes approximately
(c) inversely proportional to momentum
(d) inversely proportional to energy (a) double (b) three times
21. A galvanometer having a resistance of 80 ohms is shunted (c) four times (d) one-fourth
by a wire of resistance 2 ohms. If the total current is 1amp., 32. In cyclotron the charged particle may be accelerated upto
the part of it passing through the shunt will be energies
(a) 0.25 amp (b) 0.8 amp (a) Several eV (b) MeV
(c) 0.02 amp (d) 0.5 amp (c) BeV (d) Kev
22. The current sensitivity of a moving coil galvanometer 33. In cyclotron the resonance condition is
depends on (a) the frequency of revolution of charged particle is
(a) the number of turns in the coil equal to the frequency of A.C. voltage sources
(b) moment of inertia of the coil (b) the frequency of revolution of charged particle is
(c) current sent through galvanometer equal to the frequency of applied magnetic field
(d) eddy current in Al frame (c) the frequency of revolution of charged particle is
23. Current i is flowing in a coil of area A & number of turns N, equal to the frequency of rotation of earth
then magnetic moment of the coil is M is equal to (d) the frequency of revolution of charged particle,
frequency of A.C. source and frequency of magnetic
(a) NiA (b) Ni/A (c) Ni / A (d) N2Ai
field are equal
Free eBooks on @neetquestionpaper2020

Moving Charges and Magnetism 719


34. The magnetic flux density B at a distance r from a long 39. A square coil of side a carries a current I. The magnetic field
straight wire carrying a steady current varies with r as at the centre of the coil is
(a) B (b) B

a O

r r

(c) B (d) B m0 I 2m 0 1
(a) (b)
ap ap
m 01 2 2 m0 I
(c) (d)
2 ap ap
r r 40. Protons and a-particles of equal momenta enter a uniform
magnetic field normally. The radii of their orbits will have
35. A current of I ampere flows in a wire forming a circular arc of the ratio.
radius r metres subtending an angle q at the centre as shown. (a) 1 (b) 2 (c) 0.5 (d) 4
The magnetic field at the centre O in tesla is 41. Under the influence of a uniform magnetic field a charged
particle is moving in a circle of radius R with constant speed
I
v. The time period of the motion
q (a) depends on both R and v
(b) is independent of both R and v
O (c) depends on R and not v
(d) depends on v and not on R
42. What is cyclotron frequency of an electron with an energy
m 0 Iq m 0 Iq m0 Iq m0 Iq of 100 e V in the earth's magnetic field of 1 × 10–4 weber / m2
(a) (b) (c) (d)
4 pr 2 pr 2r 4r if its velocity is perpendicular to magnetic field?
36. A uniform electric field and a uniform magnetic field exist in (a) 0.7 MHz (b) 2.8 MHz
a region in the same direction. An electron is projected with (c) 1.4 MHz (d) 2.1 MHz
velocity pointed in the same direction. The electron will 43. A circular loop of area 0.02 m2 carrying a current of 10A, is
(a) turn to its right held with its plane perpendicular to a magnetic field
(b) turn to its left induction 0.2 T. The torque acting on the loop is
(c) keep moving in the same direction but its speed will (a) 0.01 Nm (b) 0.001 Nm
increase (c) zero (d) 0.8 Nm
(d) keep moving in the same direction but its speed will 44. Two parallel wires carrying currents in the same direction
decrease attract each other because of
37. A current of I ampere flows along an infinitely long straight (a) mutual inductance between them
thin walled hollow metallic cylinder of radius r. The magnetic (b) potential difference between them
field at any point inside the cylinder at a distance x from the (c) electric forces between them
axis of the cylinder is (d) magnetic forces between them
m0 I m0 I 45. Two thin, long, parallel wires, separated by a distance ‘d’
(a) ¥ (b) (c) (d) zero carry a current of ‘i’ A in the same direction. They will
2p r 2p x
38. Two particles X and Y having equal charge, after being (a) repel each other with a force of m 0 i 2 /(2pd)
accelerated through the same potential difference enter a (b) attract each other with a force of m 0 i 2 /(2pd)
region of uniform magnetic field and describe circular paths
of radii R1 and R2 respectively. The ratio of the mass of X to (c) repel each other with a force of m 0 i 2 /(2pd 2 )
that of Y is (d) attract each other with a force of m 0 i 2 /(2pd 2 )
2 46. A horizontal overhead powerline is at height of 4m from
R1 æ R2 ö
(a) (b) ç ÷ the ground and carries a current of 100A from east to west.
R2 è R1 ø The magnetic field directly below it on the ground is
(m0 = 4p × 10–7 Tm A–1)
2
æ R1 ö R2 (a) 2.5×10–7 T southward (b) 5 × 10–6 T northward
(c) ç ÷ (d) R1 (c) 5 × 10–6 T southward (d) 2.5 × 10–7 T northward
è R2 ø
Free eBooks on @neetquestionpaper2020

720 Physi cs
47. If an electron describes half a revolution in a circle of 56. The field B at the centre of a circular coil of radius r is p
radius r in a magnetic field B, the energy acquired by it is times that due to a long straight wire at a distance r from it,
1 for equal currents. Fig. shows three cases:
(a) zero (b) mv 2
2
1 P
(c) mv 2 (d) p r ´ Bev P P
4
48. The orbital speed of electron orbiting around a nucleus in a (a) (b) (c)
circular orbit of radius 50 pm is 2.2 × 106 ms–1. Then the in all cases the circular part has radius r and straight ones are
magnetic dipole moment of an electron is infinitely long. For same current the field B at the centre P in
(a) 1.6 × 10–19 Am2 (b) 5.3 × 10–21 Am2 cases 1, 2, 3 has the ratio
(c) 8.8 × 10 Am –24 2 (d) 8.8 × 10–26 Am2 æ p ö p æ 3p 1 ö
49. A deutron of kinetic energy 50 keV is describing a circular (a) ç- ÷: :ç - ÷
orbit of radius 0.5m, in a plane perpendicular to magnetic è 2ø 2 è 4 2ø
r
field B . The kinetic energy of a proton that discribes a æ p ö æ p ö æ 3p 1 ö
(b) ç - + 1÷ : ç + 1÷ : ç + ÷
circular orbit of radius 0.5m in the same plane with the same è 2 ø è2 ø è 4 2ø
r
magnetic field B is p p 3p
(c) - : :
(a) 200 keV (b) 50 keV (c) 100 keV (d) 25 keV 2 2 4
50. A proton and an a-particle enter a uniform magnetic field æ p ö æ p 1 ö æ 3p 1 ö
perpendicularly with the same speed. If proton takes 25 m (d) çè - - 1÷ø : çè + ÷ø : çè + ÷ø
2 4 4 4 2
second to make 5 revolutions, then the time period for the
57. An infinite straight conductor carrying current 2 I is split
a-particle would be
into a loop of radius r as shown in fig. The magnetic field at
(a) 50 m sec (b) 25 m sec
the centre of the coil is
(c) 10 m sec (d) 5 m sec
51. Through two parallel wires A and B, 10A and 2A of currents m 0 2 (p + 1) I
(a)
are passed respectively in opposite directions. If the wire A 4p r
is infinitely long and the length of the wire B is 2m, then m 0 2 ( p - 1) 2I 2I
force on the conductor B, which is situated at 10 cm distance (b) O
4p r
from A, will be
(a) 8 × 10–7 N (b) 8 × 10–5 N m 0 ( p + 1)
–7 (c)
(c) 4 × 10 N (d) 4 × 10–5 N 4p r I
52. The deflection in a galvanometer falls from 50 division to 20 (d) zero
when a 12 ohm shunt is applied. The galvanometer resistance 58. A long wire is bent into shape ABCDE as shown in fig., with
is BCD being a semicircle with centre O and radius r metre. A
(a) 18 ohm (b) 36 ohm (c) 24 ohm (d) 30 ohm current of I amp. flows through it in the direction
53. A moving coil galvanometer has a resistance of 900 W. In A ® B ® C ® D ® E. Then the magnetic induction at the
order to send only 10% of the main current through this point O of the figure in vacuum is
galvanometer, the resistance of the required shunt is
é I I ù
(a) 0.9 W (b) 100 W (c) 405 W (d) 90 W (a) m0 ê + ú B
54. A cell is connected between two points of a uniformly thick ë2p r 4rû A
I
I
circular conductor and i1 and i2 are the currents flowing in é I I ù B × O r C
two parts of the circular conductor of radius a. The magnetic (b) m 0 ê - ú
field at the centre of the loop will be ë2p r 4rû I
E
m0 (c) m 0 I / 4 r D
(a) zero (b) (I1 - I 2 )
4p (d) m 0 I / p r
m0 m0
(c) ( I1 + I 2 ) (d) (I1 + I 2 ) 59. If a current is passed through a spring then the spring will
2a a (a) expand (b) compress
55. In fig, what is the magnetic field induction at point O (c) remains same (d) None of these.
m0 i 60. Three wires are situated at the same distance. A current of
(a) 4 p r i
1A, 2A, 3A flows through these wires in the same direction.
m0 i m0 i What is ratio of F1/F2, where F1 is force on 1 and F2 on 2?
(b) 4 r + 2 p r
(a) 7/8
m0 i m0 i
(c) 4 r + 4 p r r
(b) 1
m0 i m0 i O (c) 9/8
(d) 4 r - 4 p r
(d) None of these 1A 2A 3A
Free eBooks on @neetquestionpaper2020

Moving Charges and Magnetism 721


61. A conducting circular loop of radius r carries a constant 66. The wire loop PQRS formed by joining two semi circular
r wires of radii R1 and R2 carries a current I as shown. The
current i. It is placed in a uniform magnetic field B0 such
magnitude of magnetic induction at the centre O is
r
that B0 is perpendicular to the plane of the loop. The
magnetic force acting on the loop is
(a) ir B0 (b) 2p ir B0 R2
R1
(c) zero (d) p ir B0
62. An electron traveling with a speed u along the positive R S O P Q
x-axis enters into a region of magnetic field where B = –B0 (a) (µ0 / 4) I (R1 / R2 – 1 / R1)
k̂ ( x > 0). It comes out of the region with speed v then (b) (µ0 / 4) I (1 / R2 – 1 / R2)
(c) µ0 I (1/ R2 – 1 / R1)
×B (d) µ0 I (1 / R1)
y
67. Net magnetic field at the centre of the circle O due to a
e– u current through a loop as shown in fig. is (q < 180°)

(a) v = u at y > 0 (b) v = u at y < 0


(c) v > u at y > 0 (d) v > u at y < 0
63. A wire ABCDEF is bent in the form as shown in figure. The
wire carries a current I and is placed in a uniform magnetic
field of induction B parallel to positive Y-axis. If each side is (a) zero
of length L, the force experienced by the wire will be (b) perpendicular to paper inwards
Z (c) perpendicular to paper outwards
D (d) perpendicular to the paper inwards if q £ 90° and
perpendicular to paper outwards if 90° £ q < 180°
L
B 68. Two wires PQ and QR, carry equal currents I as shown in
C L fig. One end of both the wires extends to infinity. ÐPQR = q.
The magnitude of the magnetic field at O on the bisector
I angle of these two wires at a distance r from point Q is
L E F
L
Y
O
B L A
X
(a) IBL along the positive Z-direction
(b) IBL along the negative Z-direction
(c) 2IBL along the positive Z-direction
m0 I æ q ö m0 I
(d) 2IBL along the negative Z-direction (a) sin ç ÷ (b) cot ( q / 2 )
64. Four wires, each of length 2.0 m, are bent into four loops P, 4p r è2ø 4p r
Q, R and S and then suspended in a uniform magnetic field. m0 I æqö m0 I (1 + cos q / 2 )
(c) tan ç ÷ (d) 4p r sin q / 2
If the same current is passed in each, then the torque will be 4p r è2ø ( )
maximum on the loop
69. One conducting U tube can slide inside another as shown
in figure, maintaining electrical contacts between the tubes.
The magnetic field B is perpendicular to the plane of the
Q S figure . If each tube moves towards the other at a constant
P R speed v, then the emf induced in the circuit in terms of B, l
and v where l is the width of each tube, will be
X A X X X X
(a) P (b) Q B
(c) R (d) S
X v X X
65. A charged particle (charge q) is moving in a circle of radius v
R with uniform speed v. The associated magnetic moment µ X X X
X
is given by X C
X X
(a) qvR2 (b) qvR2/2 (c) qvR (d) qvR/2 (a) – Blv (b) Blv (c) 2 Blv (d) zero
Free eBooks on @neetquestionpaper2020

722 Physi cs
70. A moving coil galvanometer has 150 equal divisions. Its Since the principle was first successfully applied to the
current sensitivity is 10-divisions per milliampere and voltage acceleration of electrons (or b particles) in a device called the
sensitivity is 2 divisions per millivolt. In order that each betatron, this method of acceleration is often given that name.
division reads 1 volt, the resistance in ohms needed to be The general idea involved is shown in figure.
connected in series with the coil will be An electromagnet is used to produce a changing flux through a
(a) 105 (b) 103 (c) 9995 (d) 99995 circular loop defined by the doughnut shaped vacuum chamber.
Directions for Qs. (71 to 75) : Each question contains We see that there will be an electric field E along the circular
STATEMENT-1 and STATEMENT-2. Choose the correct length of the doughnut, i.e. circling the magnet poles, given by , 2
answer (ONLY ONE option is correct ) from the following. df
(a) Statement -1 is false, Statement-2 is true paE = , where 'a' is the radius of the doughnut. Any charged
dt
(b) Statement -1 is true, Statement-2 is true; Statement -2 is
particle inside the vacuum chamber will experience a force qE and
a correct explanation for Statement-1
will accelerate. Ordinarily, the charged particle would shoot out of
(c) Statement -1 is true, Statement-2 is true; Statement -2 is
the vacuum chamber and becomes lost.
not a correct explanation for Statement-1
However, if the magnetic field at the position of the doughnut is
(d) Statement -1 is true, Statement-2 is false
just proper to satisfy the relation,
71. Statement 1 : If the current in a solenoid is reversed in
direction while keeping the same magnitude, the magnetic m v2
Centripetal force = magnetic force or = q vB
field energy stored in the solenoid decreases. a
Statement 2 : Magnetic field energy density is proportional then the charge will travel in a circle within the doughnut. By
to square of current. proper shaping of the magnet pole pieces, this relation can be
72. Statement 1 : If a charged particle is released from rest in a satisfied. As a result, the charge will move at high speed along the
region of uniform electric and magnetic fields parallel to loop within the doughnut. Each time it goes around the loop, it
each other, it will move in a straight line. has, in effect, fallen through a potential difference equal to the
Statement 2 : The electric field exerts no force on the particle induced emf, namely
but the magnetic field does. df
73. Statement 1 : A cyclotron cannot accelerate neutrons. e= . Its energy after 'n' trips around the loop will be q(ne).
dt
Statement 2 : Neutrons are neutral. 76. Working of betatron is not based upon which of the following
74. Statement 1 : The magnetic field at the centre of the circular theories?
coil in the following figure due to the currents I1 and I2 is (a) Changing magnetic flux induces electric field
zero. (b) Charged particles at rest can be accelerated only by
I1 electric fields
I I (c) Magnetic fields can apply a force on moving charges
q which is perpendicular to both magnetic field and
motion of the particle
(d) b particles are emitted in radioactive decay process.
77. Variable magnetic flux
(a) can change sinusoidally
I2
(b) should either increase or decrease all the time
Statement 2 : I1 = I2 implies that the fields due to the current
I1 and I2 will be balanced. (c) must becomes zero when induced field is maximum
75. Statement 1 : Free electrons always keep on moving in a (d) None of these
conductor even then no magnetic force act on them in 78. Magnetic field which keeps the particles in circular path
magnetic field unless a current is passed through it. must
Statement 2 : The average velocity of free electron is zero. (a) remain a constant everywhere
Directions for Qs. (76 to 84) : Read the following passage(s) (b) increase gradually with a rate proportional to kinetic
carefully and answer the questions that follows: energy of the particle
PASSAGE 1 (c) increase gradually with a rate proportional to speed of
The fact that a changing magnetic flux produces an electric field the particle
is basic to the operation of many high energy particle accelerators. (d) None of these

PASSAGE 2
Curves in the graph shown give, as functions of radial distance r,
the magnitude B of the magnetic field inside and outside four long
wires a, b, c and d, carrying currents that are uniformly distributed
across the cross-sections of the wires. Overlapping portions of
the plots are indicated by double labels.
Free eBooks on @neetquestionpaper2020

Moving Charges and Magnetism 723

a
b a,c
a,b c
b,d 82. In figure, the magnitude of the magnetic torque exerted
c,d on the loop is closest to
r
(a) 0.55 N-m (b) 0.15 N-m
79. Which wire has the greatest radius? (c) 0.45 N-m (d) 0.35 N-m
(a) a (b) b (c) c (d) d 83. In figure the loop is released from rest. The initial motion of
80. Which wire has the greatest magnitude of the magnetic field the loop is described by
on the surface? (a) point a moves out of the plane, point c moves into the
(a) a (b) b (c) c (d) d plane
81. The current density in wire a is – (b) points a, b, c and d move counterclockwise
(a) greater than in wire c (c) point a, b, c and d move clockwise
(b) less than in wire c (d) point c moves out of the plane, point a moves into the
(c) equal to that in wire c plane
(d) not comparable to that of in wire c due to lack of 84. In figure, an external torque changes the orientation of loop
information from one of lowest potential energy to one of highest
PASSAGE 3 potential energy. The work done by the external torque is
A rigid circular loop has a radius of 0.20 m and is in the x-y plane. closest to
A clockwise current I is carried by the loop, as shown. The (a) 0.5 J (b) 0.2 J
magnitude of the magnetic moment of the loop is 0.75 A-m2. A (c) 0.3 J (d) 0.4 J
uniform external magnetic field, B = 0.20 T in the positive
x-direction, is present
Free eBooks on @neetquestionpaper2020

724 Ph y si cs

In the given problem, q = 3 p/2 and r = R


Exercise 19.1
m 3 p i 3 m0 i
\ B= 0 ´ ´ =
3. (a) 4p 2 R 8R
i 13. (b) Apply Right hand thumb rule.

lines of Exercise 19.2


force r P
r r
1. (c) Fn = q(v ´ B)
= qvB sinq = 0 (because q = 0º )
2. (c) (i) When no field is present E = 0, B = 0, the proton
m i experiences no force. Thus it moves with a
4. (d) B = o , where r is distance of any point P from the
2 pr constant velocity.
straight conductor, at which we want to calculate the (ii) When E = 0 and B ¹ 0, then there will be a
magnetic field. probability that proton may move parallel to
m i magnetic field. In this situation, there will be no
5. (c) B = 0 or B µ 1 / r.
2pr force acting on proton.
7. (c) The magnetic field at the centre of a circular coil is (iii) When both fields are present
m pi ´ n E ¹ 0, B ¹ 0, then let E, B and v may be mutually
Bcentre = o perpendicular to each other. In this case, the electric
2 pr
where 2pr = l = length of wire & n is number of turns and magnetic forces acting on the proton may be
It is given that 2pr1 = l = 2(2pr2) Þ r2 = r1/2 equal and opposite. Thus, there will be no resultant
m o pi ´ 1
force on the proton.
So B1 at centre =
2pr1 3. (c) Given that K p = K a
m o pi ´ 1 m v (2 m K)1/ 2
B2 at centre = We know that r = =
2pr1 qB qB
B2 2 ´ 2
so = Þ B 2 = 4B1 ( 2 m p K p )1 / 2 (2 m a K a )1 / 2
B1 1 \ rp = and ra =
qp B qa B
8. (c) In a metallic rod, charge carriers flow through whole
of the cross-section, the magnetic field exists both æ mp ö 2e
rp æ mp ö qa = ç ÷´ =1
inside as well as outside. Now = çç ÷´
÷ ç 4 mp ÷ e
r r ra è ma ø q p è ø
9. (b) According to Ampere’s law òÑ B • dl = λ 0 i
r r
To any closed path inside pipe, there will be no current 4. (b) Fe = -qE , where –ive sign comes due to electron,
enclosed. So B inside pipe is zero. Only, there would which has negative charge. It is clear from expression
be field outside the pipe. that force on electron is opposite to electric field i.e.,
10. (b) Magnetic induction inside a thin walled tube is zero. in negative direction.
(According to Ampere's Law) 5. (b) Here velocity vector have two components
11. (a) See Ampere’s Law (i) v cos q, parallel to magnetic field
12. (d) For a loop, magnetic induction at centre (ii) v sin q, perpendicular to magnetic field. Due to
m 2 pi component v cos q, the particle will have a linear
B= 0 ´
4p r motion but due to v sin q, the particle will have
When loop subtends angle q at centre, then simultaneously a circular motion. The resultant of the
two is a helical path.
m qi
B= 0 ´ (Here angle is q instead of 2 p) 7. (d) When a positively charged particle enters in a region
4p r
of uniform magnetic field directed vertically upwards,
Free eBooks on @neetquestionpaper2020

Moving Charges and Magnetism 725


it experiences a centripetal force which moves the Since the magnetic needle will be in the direction of
particle in circular path with a uniform speed (in resultant magnetic field, which will be in the west-
clockwise direction). north direction.
8. (c) Force on moving charge while moving in magnetic E
r r r r
22. (a) < c (velocity light if we consider free space) &
field is; F < q (vr ´ B) where F is perpendicular to v . B
rr 1 E
r = c so m o e o is dimensionless.
Work done/sec = F.v = Fv cos 90º = 0. m o eo B
9. (d) 23. (a) The resistance of AD is one-third of the resistance
Direction Direction along the parallel path AB + BC + CD. Hence if current
of change of change
3i i
i enters at A, will flow along AD and , along
B (Z) 4 4
AB. The magnetic field at the centre due to AD is equal
string B and opposite to the combined effects of AB, BC and
CD.
In this case at every In this case at every
point tension decreases point tension increases Exercise 1 : NCERT Based Questions 
Y 1. When the charged particle will move perpendicular to the
field, the force on it is maximum.
2. Radial magnetic field.
x r r
ur i(d l ´ r)
z 4. In C.G.S. system, dB =
r3
(right handed coordinate system) r r
15. (b) ur m i(d l ´ r)
lines of wire In S.I. system, dB = 0
force 4p r3
r r r
5. Fm = q(v ´ B)
i 8. 4:1
B

m 0 Ni m0 I
9. (i) At centre BC = (ii) B =
2R 2R
10. (i) zero
r r r
Fm < I( l ´ B) = 0 because the directions of current l 11. B = B12 + B22 + 2B1 B2 cos q
r
and magnetic field B are parallel. m0 nl
16. (c) The force acting on various small current carrying The direction of magnetic field B = (1 + cos q) .
2a
elements of the circumference of the loop will be
15. (i) 36 × 10–2 Am2 (ii) 0
distributed randomly in all possible directions. The
vector addition of such randomly distributed forces 18. (a) 19. (a) 20. (a) 21. (c)
will be zero. 22. (d) 23. (c) 24. (d) 25. (d)
r r r r
18. (d) t = (M ´ B) , where | M | = i.A Exercise 2 : PAST Competition MCQs
= MB sin q
r m 0i
where q is angle between Magnetic moment & B . 1. (a) [Hint Þ B = , where r is distance of point from
r 2pr
For q = 0 t = 0 & coil is in stable equilibrium. Hence
the wire, where we want to calculate the magnetic field.
plane of coil must be perpendicular to magnetic field.
It is clear from expression that B is independent of
M B thickness of wire.]
2. (a) The direction of the magnetic field due to a current is
i
given by right hand curled fingers rule. Therefore at
AB axis, the components of magnetic field will cancel
each other and the resultant magnetic field will be zero
20. (d) Magnetic dipole moment of current loop M = niA, it
on AB.
is independent of B.
3. (d)
21. (b) When the plane of coil is in N-S vertical plane and
current is passed in clockwise direction, the magnetic mv 2
4. (b) [Hint Þ = qvB ].
field at the centre of coil due to current is directed r
East to West. Earth’s magnetic field is south to North.
Free eBooks on @neetquestionpaper2020

726 Ph y si cs
5. (b) Let l be length of wire 14. (b) Let a current i be flowing in the loop ABC in the
l direction shown in the figure. If the length of each of
Ist case : l = 2pr Þ r = the sides AB and BC be x then
2p r
m 0I m0I | F| = i x B
B= =
2 pr l
l A
2nd Case : l = 2(2pr ¢) Þ r¢ =
4p
m 0 In 2m I
B¢ = = 0 (where n = 2)
l l
2p
4p 2
æ m0I ö
on putting the value of B Þ B¢ = 4ç ÷ = 4B
è l ø
6. (b) Magnetic force acts perpendicular to the velocity. B C
Hence speed remains constant. Direction of
æN ö magnetic field
7. (c) B = m0 N0i ; B1 = (m0 ) ç 0 ÷ (2 i) = m0 N 0 i = B
è 2 ø
where B is the magnitude of the magnetic force.
Þ B1 = B r
The direction of F will be in the dir ection
8. (c) A high resistance is connected in series so that less
perpendicular to the plane of the paper and going into
current passes through voltmeter.
it.
9. (a) Rg = 50W, Ig = 25 × 4 × 10–4W = 10–2 A
By Pythagorus theorem,
Range of V = 25 volts R
V = Ig(HR + Rg) A B AC = x 2 + x 2 = 2x
V \ Magnitude of force on AC
\ HR = - R g = 2450 W Ig
Ig HR Rg =i 2 x B sin 45°
mv v
10. (d) r = qB Þ r µ = i 2 xB´
1
B
2
Ù r
11. (a) The direction of B is along ( - k )
= ixB = | F |
\ The magnetic force The direction of the force on AC is perpendicular to
F = Q( v ´ B) = Q( v î ) ´ B(- k̂ ) = QvBĵ the plane of the paper and going out of it. Hence, force
r
Þ along OY.. on AC = - F
12. (b) The force on the two arms parallel to the field is zero. r r
15. (b) v and B are in same direction so that magnetic force
<
on electron becomes zero, only electric force acts.
But force on electron due to electric field is opposite
<

<

F
to the direction of velocity.
B
–F I1
16. (d)
F1
I
<
\ Force on remaining arms = – F z
13. (b) Magnetic fields due to the two i
parts at their common centre are F3 F4
respectively,
m i m i i
B y = 0 and Bz = 0 y
4R 4R F2
i
Resultant field = B y2 + Bz2 1
F1 > F2 as F µ , and F3 and F4 are equal and opposite.
2 d
2
æ m iö æ m iö Hence, the net attraction force will be towards the
= ç 0 ÷ +ç 0 ÷ conductor.
è 4R ø è 4R ø
17. (a) Magnetic field at the centre of the ring is
m i m i m 0 qf
= 2. 0 = 0
4R 2 2 R 2R
Free eBooks on @neetquestionpaper2020

Moving Charges and Magnetism 727


18. (a) 22. (c) A current loop in a magnetic field is in equilibrium in
two orientations one is stable and another unstable.
B2 r uur ur
Q t = M ´ B = M B sin q
B1 If q = 0° Þ t = 0 (stable)
If q = p Þ t = 0 (unstable)

The magnetic field, due the coil, carrying current I


Ampere
m I
B1 = 0
2R Do not experience a torque in some orientations
The magnetic field due to the coil, carrying current 2I
Hence option (c) is correct.
Ampere
23. (c)
m (2I ) 24. (a) T = 2 p m/Bq.
B2 = 0
2R 25. (b) Force is ^r to displacement Þ the work done is zero
The resultant B 26. (a) Since there is no deviation in the path of the charged
Bnet = B12 + B22 + 2B1B2 cos q, q = 90° particle, so net force due to presence of electric and
magnetic field must be zero
m 0 (2 I )
Bnet = B12 + B22 = 1+ 4 E 104
2R Þ qv B = qE Þ B = = = 103 Wb / m 2
v 10
5 m0 I 27. (b) Using Ampere’s law at a distance r from axis, B is
=
2R same from symmetry.
19. (c) Time period of cyclotron is
1 2pm
T= = ; B=
2pm
u; R =
mu p
=
ò B.dl = mi
u eB e eB eB B ´ 2pr = m0i
2pmu Here i is zero, for r < R, whereas R is radius
Þ P = eBR = e ´ R = 2pmuR
e \B = 0
2 2
p (2pmuR ) m 0i
K.E. = = = 2p2mu2R2 28. (b) B = , n ´ (2pr ' ) = 2pR Þ nr' = R ....(1)
2m 2m 2R
20. (b) According to the principal of circular motion in a n.m0 i
magnetic field B' = .....(2)
2r '
2
mv nm 0i.n
Fc = Fm = qVB
Þ From (1) and (2), B ' = = n2B
R 2R
mv P 2 m.k m 0i a 2
Þ R= = = B=
qB qB qB 29. (c) 3
2(x 2 + a 2 ) 2
2(4 m) K '
Ra = æ 2 2 2ö
3
2qB m i m 0i a 2 ç (x + a ) ÷
B' = 0 = 3 ç
R K 2a a2 ÷
= 2a(x 2 + a 2 ) 2 è ø
Ra K'
but R = Ra (given) B.(x 2 + a 2 )3/2
B' =
Thus K = K¢ = 1 MeV a3
21. (b) According to work energy theorem 54(53 )
W =Ufinal – Uinitial = MB (cos 0 – cos 60°) Put x = 4 & a = 3 Þ B' = = 250mT
3´ 3´ 3
MB æm I ö
W= = 3J ...(i) 30. (a) F = ç 0 1 ÷ lI2 When I, = – 2I, and r =3r, then
2 è 2 pr ø
r r æ ö
t = M ´ B = MB sin 60° = ç MB 3 ÷ m 0 - 2I1lI2
è 2 ø ...(ii) F' = –
2p.3r
From equation (i) and (ii)
æ m 0 I1I2 l ö æ 2 ö 2
=ç - =- F
2 3´ 3 è 2pr ÷ø çè 3 ÷ø 3
t= = 3J
2
Free eBooks on @neetquestionpaper2020

728 Ph y si cs
31. (d) (1) m0 I m I
(B2) = . = 0 .
2p (2a) 4pa
B m Ir 4pa
\ Required ratio = 1 = 0 1 ´
(2) B 2 2 pa 2 m 0 I
a
2 r1 2 ´ 2
= = = 1.
The magnetic field due to circular coil, a a
38. (d) There is no current inside the pipe. Therefore
µ0i1 m 0i1 m 0 ´ 3 ´ 10 2 ur uur
B1 =
2r
=
2 (2p ´ 10-2 )
=
4p Ñò B.dt = µo I
I=0 \ B=0
m 0i2 m 0 ´ 4 ´ 102 r r
B2 = = 39. (b) Here, E and B are perpendicular to each other and
2(2p ´ 10 -2 ) 4p r
the velocity v does not change; therefore
m0
B= B12 + B 22 = . 5 × 102 E
4p qE = qvB Þ v =
B
Þ B = 10 - 7 ´ 5 ´ 10 2 Þ B = 5 × 10–5 Wb / m2 r ur ur
r
If velocity is ^ to both E and B ,
v
32. (c) Equating magnetic force to centripetal force, r r
E´B E Bsin q E B sin 90° E r
mv 2 Also, = = = = |v| = v
2 2 2 B
= qvB sin 90º B B B
r 40. (b,d) When a charged particle enters a magnetic field at a
Time to complete one revolution,
direction perpendicular to the direction of motion, the
2pr 2pm path of the motion is circular. In circular motion the
T= =
v qB direction of velocity changes at every point (the
33. (d) A magnetic needle kept in non uniform magnetic field magnitude remains constant). Therefore, the tangential
experience a force and torque due to unequal forces momentum will change at every point. But kinetic
acting on poles. 1
34. (b) Due to electric field, it experiences force and energy will remain constant as it is given by mv2
2
accelerates i.e. its velocity decreases. and v2 is the square of the magnitude of velocity which
35. (b) The charged particle will move along the lines of does not change.
electric field (and magnetic field). Magnetic field will In this question, we could not infer that whether the
exert no force. The force by electric field will be along momentum mentioned here is tangential or angular.
the lines of uniform electric field. Hence the particle If it is angular momentum, then (d) is the correct option.
will move in a straight line. 41. (c) Clearly, the magnetic fields at a point P, equidistant
i from AOB and COD will have directions perpendicular
B2 m 0 n 2 i 2 100 ´ to each other, as they are placed normal to each other.
= B 2 3
36. (a) Þ =
B1 m0 n1i1 A D
6.28 ´ 10 -2 200 ´ i B1 B2
6.28 ´10 -2
Þ B2 = = 1.05 ´ 10- 2 Wb / m 2 I1 d I2
6
37. (d) Here, current is uniformly distributed across the cross- O
section of the wire, therefore, current enclosed in the
æ aö
amperean path formed at a distance r1 ç = ÷ C B
è 2ø
æ pr ö
2 \ Resultant field, B = B12 + B22
= ç 12 ÷ ´ I , where I is total current
è pa ø m I m I
But B1 = 0 1 and B2 = 0 2
\ Magnetic field at P1 is B1 2 pd 2 pd
m ´ current enclosed
( )
2 m 0 2 2 1/ 2
= 0
Path
æ m ö
\ B = ç 0 ÷ I12 + I22 or,, B =
è 2pd ø ( 2pd )
I1 + I2
a/2
æ pr ö
2
P1 P2 42. (a) The magnetic field at O due to current in DA is
m0 ´ ç 12 ÷ ´ I
ç pa ÷ m ´ I r1 m I p
B1 = è ø = 0 B1 = o ´ (directed vertically upwards)
2p r1 2p a 2 4p a 6
Now, magnetic field at point P2, The magnetic field at O due to current in BC is
m I p
B2 = o ´ (directed vertically downwards)
4p b 6
Free eBooks on @neetquestionpaper2020

Moving Charges and Magnetism 729


The magnetic field due to current AB and CD at O is 48. (b) Magnetic field induction due to a straight long
zero. conductor at a distance r away from it is given by,
Therefore the net magnetic field is m I
B = o . Its direction is given by Right hand thumb
B = B1 - B2 (directed vertically upwards) 2p r
rule.
mo I p m o I p 49. (a) For a < x < 2a
= - ´ r
4 p a 6 4p b 6 B = B0ˆj
m I æ 1 1ö m I r
= o ç - ÷ = o (b - a) Initial velocty is v = v 0 iˆ
24 è a b ø 24ab
r r r From the diagram it is clear that the force on the particle
43. (d) F = I ( l ´ B) is towards positive z-axis (apply Fleming's left hand
The force on AD and BC due to current I1 is zero. rule) at x = a, which shifts the particle as shown in the
uur
This is because the directions of current element I d l X–Z plane.
r For 2a < x < 3a
and magnetic field B are parallel. Z'
44. (a) The magnetic field due a disc is given as Y'
m wQ 1 F
B= 0 i.e., B µ B0
2 pR R
45. (a) As we know, Magnetic flux, f = B. A
X
m0 (2)(20 ´ 10 -2 ) 2 x=a V0
´ p(0.3 ´ 10 ) -2 2 r
2[(0.2)2 + (0.15) 2 ] B = - B0 ˆj
on solving The direction of velocoty is shown at x = 2a. Agian
= 9.216 × 10–11 using Fleming left hand rule, we get the direction of
» 9.2 × 10–11 weber force. This changes the profile of the particle as shown
46. (c) A closed current carrying loop when placed in a by the dotted line.
Alternatively use the vector form of B and V is the
uniform magnetic field always experiences no net r r r
force. ( )
formulae F = q V ´ B to get the instantaneous
47. (d) D direction of force at x = a and x = 2a
Z"
I2 2 –q Y"

A C
B x = 2a
X
I1 B0
F
1 –Y"
E r
Magnetic field at the centre due to current ABC arc is 50. (b,d) Figure shows that the megnetic field B is present on
m 2 pI1 q the right hand side of AB. The electron (e) and proton
B1 = 0 (Directed upwards) (p) moving on straight parallel paths with the same
4 p r 2p
Magnetic field at the centre due to current in ADC velocity enter the region of uniform magnetic field.
The entry and exit of electron & proton in the magnetic
arc is
field makes the same angle with AB as shown.
m 2 pI 2 ( 2p - q ) Therefore both will come out travelling in parallel
B2 = 0 (Directed downwards)
4p r 2p paths.
Therefore net magnetic field at the centre A
m 2pI1 q m 0 2pI2 (2p - q) q × × × × ×
B= 0 -
4p r 2 p 4 p r 2p
E E EA B
2q × × × × ×
Also, I1 = = = and
R1 rl1 /A rrq
× × × × ×
E E EA q
I2 = = = p
R 2 rl 2 / A rr(2p - q) e –q
×2 q × × × ×
q
m 0 é EA q EA ( 2p - q ) ù
\ B= ê ´ - ´ ú =0 × × × × ×
4p ë rrq r rr (2r - q) r û
B
Free eBooks on @neetquestionpaper2020

730 Ph y si cs
The time taken by proton 53. (b) The magnetic moment of a current carrying loop is
r r
distance arc angle × radius 2q ´ R p given by M = NIA
tp = = = = 2
speed speed speed v æ aö é pù
Here N = 1, A = a2+ 2p ç ÷ = a 2 ê1 + ú , the
2q æ m p v ö 2q m p è2ø ë 2û
= v ´ çç eB ÷÷ = eB direction is towards positive z-axis.
è ø r p
The time taken by electron is \ M = Ia 2 é1 + ù kˆ
êë 2 úû
(2p - 2q) R e (2p - 2q) æ me v ö (2p - 2q) me
te = = çè ÷= 54. (a,c) The magnetic field should be in the –z direction
v v eB ø eB (Fleming’s left hand rule)
clearly te is not equal to tp as mp >> me
vy 2 p
\ (b), (d) are correct options tan q = = \q =
51. (a) Let us consider an amperian loop ABCD which is a vx 2 3 6
rectangle as shown in the figure. y
Applying ampere’s circuital law we get O z x u2 = v
q
× × × × v× ×
y=2
D C × × × × q × ×
L
l × × × × vx = ×2 3 ×
× × × × × ×
A B
Q,M×+ × ×ˆ
ui = u = 4i
× × ×
× × × × × ×
r uur
Ñò B.d l = mo × (current passing through the loop) × ×
arc
× ×
speed ´ time
× ×
r uur æIö angle = =
\ Ñò
B.d l = mo ç ÷ ´ l
è Lø
radius
-3
radius
p 4 ´ 10 ´ 10 é Mv ù
I \ = êQ radius = QB ú
\ B × l = mo ´ l 6 M ´ 4 / QB ë û
L
50p M
mo I mo \B =
\ B= = Io cos (300 t) 3Q
L L
The magnetic moment of the loop (a) and (c) are the correct options
= (current in the loop) × pr2 55. (a, d) In the region O < r < R, the magnetic field is present
due to current in solenoid.
1 æ dfö 2
= çè - ÷ø ´ pr
R dt
1 éd 2 ù 2 p 2 r 4 dB
= - ( B ´ pr ) ´ pr = -
R êë dt ú
û R dt
é p2 r 4 mo ù In the region r > 2 R, the magnetic field is present due
= ê R ´ L I o sin(300 t ) ú ´ 300 to the current in the cylinder.
êë úû
For the region R < r < 2R, the magnetic field is neither
Comparing it with the expression given in the
along the common axis, nor tangential to the circle of
question we get
radius r. (a) and (d) are correct options.
300p2 r 4 1 300(3.14)2 ´ (0.1)4
N= ´ = =6 Exercise 3 : Conceptual & Applied MCQs
R L 0.005 ´10
52. (b) The magnetic field due to current carrying wire at the
1. (d) The straight part will not contribute magnetic field at
location of square loop is
the centre of the semicircle because every element of
µ0 2piR2 µ i the straight part will be 0º or 180º with the line joining
B= 2 2 3/2
= 0
4p (R + 3R ) 16R the centre and the element
The mutual induction 1 m0i m 0 i
Due to circular portion, the field is =
N f 2 é µ0 i 2 ù 2 2r 4r
M= = ê ´ a cos 45º ú
i i ë 16R û m0 i
µ0 a2 Hence total field at O = tesla
\ M= 4r
7
2. (b) F = iB l sin q. This is maximum when sin q = 1
22 R
or q = p/2.
Free eBooks on @neetquestionpaper2020

Moving Charges and Magnetism 731


m 0 2pi 19. (c) E = vB = 2 ´ 103 ´ 1.5 = 3 ´ 103 V / m.
3. (c) B = where
4p r
mv2 Þ r = mv
2e 2 ´ 1.6 ´10 -19 20. (a) Bqv =
i= = = 1.6 ´ 10 -19 A r Bq
t 2 21. (c) [Hint Þ S × (I – Ig) = Rg × Ig ]
m 0 i m 0 ´ 1.6 ´ 10 -19 nBA
\ B= = = m 0 ´ 10 -19 T 22. (a) Current sensitivity =
2r 2 ´ 0.8 K
where K is constant of torsional rigidity.
4. (d) B = m 0 nI = 4p ´10 -7 ´ 10 ´ 5 = 2p ´ 10 -5 T.
23. (a) 24. (a)
m 0 2pni1 m 0 2pni 2 m é ni ni ù 25. (c) A voltmeter is a high resistance galvanometer and is
5. (d) B = 4p . r - 4p . = 0 ê 1- 2ú
1 r2 2 ë r1 r2 û connected in parallel to circuit and ammeter is a low
6. (c) If r is the radius of the circle, resitance galvanometer so if we connect high resistance
in series with ammeter its resistance will be much high.
L
then L = 2pr or, r = m 2pI I
2p 26. (a) In coil A, B = 0 . \B µ ;
4p R R
Area = pr 2 = pL2 / 4p 2 = L2 / 4p
B1 I1 R 2 2
7. (c) Bqv =
m v2
or B =
mv
=
(9 ´ 10 -31 ) ´ 10 6 Hence, B = R × I = 2 = 1
r rq 0.1 ´ (1.6 ´ 10 -19 ) 2 1 2
27. (a) r = mv/Bq is same for both.
= 5.5 ´10 -5 T 28. (d)
1 29. (b) Magnetic force always acts on a charge particle in
8. (a) Ek = mv 2 or mv = 2 E k m and direction perpendicular to its motion .
2
Thus it does not change its speed but changes direction
mv 2 Ek m
r= = of motion, so particle moves in circular path and
Bq Bq centripetal force is provided by magnetic force.
Bqr 4.5 ´10 -3 ´ 1.6 ´10 -19 ´ 2 ´10 -2 Let radius of circle = R
9. (d) v= = then magnetic force = centripetal force
m 9.1´10 -31
= 1.58 ´107 m / s mv 2 R = mv
qvB =
R qB
mv m
10. (a) r= or, r µ for the same value of v and B. 30. (b)
Bq q
11. (c) Torque on loop t = nIAB cos q; Here q = 90º æ m 0 NI ö 2
31. (c) Baxis = çç ÷÷R
\ t = 0. è 2x 3 ø
12. (d) Bqv = mv2 /r or q/m = v /rB.
B µ R2
13. (b) F = Il B sin q = 3 ×0.40 × (500 × 10–4) × sin 30º So, when radius is doubled, magnetic field becomes
= 3 × 10–2 N. four times.
14. (a) Rg = 5 W; Ig = 15 × 10–3 A; 32. (b) In cyclotron, energy with which aceleration takes place
I = 1.5 A; S = Ig Rg/ (I – Ig); is in term of MeV.
15. (c) Ig = 10 × 10–3/5 = 2 × 10–3 A;
33. (a) 34. (c)
V 1 µ0 I q m 0 Iq
R= - Rg = - 5 = 495 W. 35. (a) B = ´ =
Ig 2 ´ 10 -3 2r 2p 4pr
36. (d) No magnetic force acts on the electron and force due
16. (b) Here, Rg = 100 W; Ig = 10–5 A; I =1A; S = ? to electric field will act opposite to its initial direction
of motion. Hence its velocity decreases in magnitude.
Ιg R g 10-5 ´ 100
S= = = 10-3 W in parallel 37. (d) Since no current is enclosed inside the hollow
I - Ig 1 - 10-5 conductor. Hence Binside = 0.
17. (c) The point P is lying symmetrically w.r.t. the two long 1 2mV
straight current carrying conductors. The magnetic 38. (c) r=
B q
fields at P due to these current carrying conductors are
mutually perpendicular. R1 mx
=
m 2i2 m 2 i1 m 4 R2 my
18. (c) B = 0 - 0 = 0 (i 2 - i1 )
4 p (r / 2) 4 m (r / 2) 4 p r 2
m x æ R1 ö
m 4 m Þ =ç ÷
= 0 (5 - 2.5) = 0 . my è R2 ø
4p 5 2p
Free eBooks on @neetquestionpaper2020

732 Ph y si cs
39. (d) Btotal = 4Bside
2m d E d 2m p E p
m0 I é p pù 49. (c) So rdeutron = ; rproton =
Btotal = 4 ê sin + sin ú Bq Bq
æa ö 4 4û
2p ç ÷ ë For same radius, B and q
è ø
2
m 2
2 2m0 I mp Ep = mdEd Þ E p = d E d = ´ 50 = 100keV
Btotal = mp 1
ap
50. (c) Time taken by proton to make one revolution
mv p rp q a 2e 2
40. (b) r = qB = qB Þ r = q = e = 1 =
25
= 5 m sec .
a p
5
41. (b) In a uniform magnetic field, a charged particle is moving 2 pm T m q
in a circle of radius R with constant speed v. As T = ; so 2 = 2 ´ 1
qB T1 m1 q 2
mv2 mv m 2 q1 5 ´ 4 m1 q
\ = Bqv or, R = .....(1) or T2 = T1 = ´ = 10 m sec.
R Bq m1 q 2 m1 2q
2pR 2pmv 2pm m0 2I1I2 10-7 ´ 2 ´10 ´ 2
Time period, T = = = .....(2) ´l = ´ 2 = 8 ´10-5 N
v Bqv Bq 51. (b) F =
4p r 0.1
Time period T does not depend on both R and v because
52. (a) I = 50 k; Ig = 20k, where k is the figure of merit of
when v is changed, R is also changed proportionately
and for period, it is R/v that is taken. 20 k . R g
galvanometer; S = Ig Rg(I – Ig); so 12 =
42. (b) 43. (c) 44. (d) (50 k - 20 k )
45. (b) On solving we get Rg = 18 ohms.
i i
F 53. (b) Ig = 0.1I, Is = 0.9 I ; S = Ig R g / Is
= 0.1 ´ 900 / 0.9 = 100 W.
54. (a) Let l1, l2 be the lengths of the two parts PRQ and PSQ
of the conductor and r be the resistance per unit length
of the conductor. The resistance of the portion PRQ will
F m 0 i1 i 2 m 0i 2
= = be R1 = l1 r
l 2pd 2 pd
I2
(attractive as current is in the same direction)
46. (c) The magnetic field is
m 2I 2 ´ 100 S O Q
B= 0 = 10-7 ´ = 5 × 10–6 T
4p r 4 R
W N P I1
100A
The resistance of the portion PSQ will be R2 = l2 r
Pot. diff. across P and Q = I1 R1 = I2 R2
4m
or I1 l1 r = I2 l2 r or I1 l1 = I2 l2 ...(1)
S E
Ground Magnetic field induction at the centre O due to
currents through circular conductors PRQ and PSQ
B
According to right hand palm rule, the magnetic field will be
is directed towards south. m 0 I1l1 sin 90º m 0 I 2 l 2 sin 90º
47. (a) Since magnetic force is always perpendicular to the = B1 – B2 = - = 0.
4p r2 4p r2
velocity of electron, so it can only change the direction
55. (c) B at O will be due to the following portions
of velocity of electron, but it (the magnetic force)
(i) Vertical straight portion. This is zero.
cannot accelerate or deaccelerate the electron.
(ii) Circular portion. This is given by
48. (c) Magnetic dipole moment
1 m0 i m0 i
e e erv Bcircular = =
m = iA = ´ pr 2 = ´ pr 2 = . 2 2r 4r
T (2 pr / v) 2 (iii) Straight horizontal portion. This is given by
1.6 ´ 10-19 ´ 50 ´ 10-12 ´ 2.2 ´ 106 m i
= Bstraight = 0
2 4pr
m i m i
= 8.8 ´ 10-24 Am2 . \ BTotal = 0 + 0
4r 4pr
Free eBooks on @neetquestionpaper2020

Moving Charges and Magnetism 733


56. (a) For case (a) magnetic field due to straight portions On element CD, the direction of force will be towards
is cancelled & the magnetic field due to semi circular right on the plane of the papper and the magnitude
arc of radius r at P is will be dF = IdlB.
m o i ´ p æ m oi ö 62. (b) From Lorentz equation
Ba = =ç ÷´ p
4p r è 4 pr ø F = -eu î ´ B0 (-k̂ ) = -euB 0 ˆj
It is in upward direction & we take upward direction
r æm iö hence it will complete a semicircular arc and comes
negative, So B a = -ç o ÷.p out of the region at a position y, such that y < 0
è 4 pr ø
63. (a)
For case (b) Due to straight portion the magnetic field
is zero so the magnetic field due to semi circular arc is 64. (d) For a given perimeter the area of circle is maximum.
r So magnetic moment of (S) is greatest.
æm iö
B b = ç o ÷ ´ p (in down wards direction so +ive sign) 65. (d) Magnetic moment µ = IA
è 4 pr ø
For case (c) Magnetic field due to straight portion is 2pR
Since T =
v
m i
= - o (upward direction)
4p r q qv
Also, I = =
Magnetic field due to circular arc which substand an T 2pR
angle 3p/2 at centre is
æ m i ö 3p
= ç o ÷´ (down ward direction) \ m=ç
æ qv ö
( )
÷ pR =
è 2pR ø
2 qvR
2
.
è 4pr ø 2
r 66. (b) 67. (b) 68. (d)
æ m i öæ 3p ö
so B c = ç o ÷ç - 1÷ 69. (c) Relative velocity = v + v = 2v
è 4pr øè 2 ø \ emf. = B.l (2v)
r r r æ 3p ö 70. (c) Resistance of Galvanometer,
so Ba : B b : B c = -p : p : ç - 1÷
è 2 ø Current sensitivity 10
G= Þ G= = 5W
æ -p ö p æ 3p 1 ö Voltage sensitivity 2
= ç ÷ : :ç - ÷
è 2 ø 2 è 4 2ø
Here i g = Full scale deflection current
57. (d) Here, the wire does not produce any magnetic field at
O because the conductor lies on the line of O. Also, the 150
loop does not produce magnetic field at O. = = 15 mA
10
m é Ι pI I ù m 0 é 2 I p I ù V = voltage to be measured = 150 volts
58. (a) B = 0 ê + + ú ê + r ú
4p ër r rû 4p ë r û (such that each division reads 1 volt)
é I I ù Þ R=
150
= m0 ê + ú - 5 = 9995W
ë 2 p r 4 rû 15 ´ 10 -3
r 71. (a) Reversing the direction of the current reverses the
(The direction of B is into the page.)
direction of the magnetic field. However, it has no
59. (b) It will compress due to the force of attraction between
effect on the magnetic-field energy density, which is
two adjacent coils carrying current in the same
proportional to the square of the magnitude of the
direction.
magnetic field.
60. (a) Due to flow of current in same direction at adjacent r r
side, an attractive magnetic force will be produced. 72. (c) Due to electric field, the force is F = qE in the
61. (c) The magnetic field is perpendicular to the plane of the r r r
paper. Let us consider two diametrically opposite direction of E . Since E is parallel to B , the particle
r
elements. By Fleming's Left hand rule on element AB velocity vr (acquired due to force F ) is parallel to
the direction of force will be Leftwards and the r r r r
magnitude will be B . Hence B will not exert any force since v ´ B = 0
r
dF = Idl B sin 90° = IdlB and the motion of the particle is not affected by B .
x x x x x x x 73. (b) Neutrons are neutral.
I1 2p - q
x xB x x Cxdl x x 74. (d)
I2
=
q
Þ I1q = I2 (2p - q) ........... (1)
dF dl dF
A D
x x x x Ix x x B1 =
q m 0 I1
. and B2 =
2p - q m 0 I 2
.
2p 2R 2p 2R
x x x x x x x Using (1), we get B1 = B2.
Free eBooks on @neetquestionpaper2020

734 Ph y si cs
75. (b) In the absence of the electric current, the free Outside the cylinder
electrons in a conductor are in a state of random B.2pr = m 0 I
motion, like molecule in a gas. Their average velocity
is zero. i.e. they do not have any net velocity in a m0I
\ B= ...... (2)
direction. As a result, there is no net magnetic force 2 pr
on the free electrons in the magnetic field. On passing 1
the current, the free electrons acquire drift velocity Inside the cylinder B µ r and outside B µ
r
in a definite direction, hence magnetic force acts on
So from surface nature of magnetic field changes.
them, unless the field has no perpendicular
component. Hence it is clear from the graph that wire ‘c’ has
76. (d) 77. (d) 78. (c) greatest radius.
79. (c) Inside the cylinder 80. (a) Magnitude of magnetic field is maximum at the surface
I of wire ‘a’.
B.2pr = m0 pr 2
pR 2 m0 I dB m 0 I
B= . r = .
2p R 2 Þ dr 2p R 2
81. (a)
m I
Þ B = 0 2 .r ....... (1)
2pR I
i.e. slope µ µ current density
pR 2
It can be seen that slope of curve for wire a is greater
R than wire c.
I
82. (b) 83. (a) 84. (c)

r
Free eBooks on @neetquestionpaper2020

20
Magnetism and
Matter
NATURAL MAGNET MAGNETIC FIELD
A natural magnet is an ore of iron (Fe3 O4) which The space around a magnet within which its influence can be
(i) attracts small pieces of iron, cobalt and nickel towards it. experienced is called its magnetic field.
(ii) when suspendeded freely, comes to rest along north-south Uniform magnetic field : A uniform magnetic field is one where the
direction. strength of the magnetic field is the same at all points of the field. In
The magnets which are obtained artificially are called artificial a uniform field, all the magnetic lines of force are parallel to one
magnets, e.g. a bar magnet, a magnetic needle, horse shoe magnet another. But in non-uniform magnetic field the strength of
magnetic field is not same at all points of the field and also the
etc.
magnetic lines of force are not parallel.
BAR MAGNET Atom as a Magnetic Dipole
A bar magnet consists of two equal and opposite magnetic poles Every atom of a magnetic material behaves as a magnetic dipole,
separated by a small distance. Poles are not exactly at the ends. because electrons in the atom revolve round the nucleus. The
The shortest distance between two poles is called effective length magnetic moment M associated with an atomic dipole as
(Le) and is less than its geometric length (Lg). neh
For bar magnet Le = 2l and Le =(5/6) Lg. M = = nm B
4pm
For semicircular magnet Lg = pR and Le = 2R. eh
where n = 1, 2, 3 ..... denotes the no. of orbits and m B = .
–24 2
4pm
Least value of dipole moment of atom = 9.27 × 10 Am .
S N
m B is called Bohr magneton.
Le =2l Most of the magnetic moment is produced due to electron spin,
Lg the contribution of the orbital revolution is very small.
Bar magnet MAGNETIC LINES OF FORCE
Properties of Magnets Magnetic line of force is an imaginary curve the tangent to which
(i) Attractive property : A magnet attracts small pieces of iron, at a point gives the direction of magnetic field at that point or
cobalt, nickel, etc. and other magnetic subsances. the magnetic field line is the imaginary path along which an
(ii) Directive property : A freely suspended magnet aligns itself isolated north pole will tend to move if it is free to do so.
nearly in the geographical north-south direction. Properties of Magnetic Lines of Force
(iii) Law of magnetic poles : Like magnetic poles repel, and unlike (i) Magnetic lines of force are hypothetical lines use to depict
magnetic poles attract each other. magnetic field in a region and understand certain
According to Gauss’s theorem in magnetism, surface phenomenon in magnetism.
integral of magnetic field intensity over a surface (closed or Direction of magnetic
uur ur uur ur lines outside the body
open) is always zero i.e. Ñò B .d s (or ò B .d s ) = 0. of magnet (from north
This theorem establishes that the poles always exist in equal pole to south pole)
and unlike pairs.
(iv) Magnetic poles exist in pairs : Isolated magnetic poles do N S Direction of magnetic
not exist. If we break a magnet into two pieces, we get two lines inside the body
smaller dipole magnets. Magnetic field lines of magnet (from south
(v) Repulsion is a sure test of magnetism. in a bar magnet pole to north pole)
Free eBooks on @neetquestionpaper2020

736 Physi cs
(ii) Tangent to field line at a point gives us the direction of Magnetic length : The shortest distance between the two poles
uur
magnetic field intensity B at that point. No two magnetic of a magnet is called its magnetic length. It is less than the
lines of force can intersect each other because magnetic geometrical length of the magnet. This magnetic length is also
field will have two directions at the point of intersection. called an effective length.
(iii) Magnetic lines of force are continuous curve from north to
south, outside the body of the magnet and from south to Geometrical length
north inside the body of the magnet.
Magnetic length
(iv) The number of lines originating or terminating on a pole is = 0.84 S N
Geometrical length
proportional to its pole strength.
Magnetic flux = number of magnetic lines of force Magnetic length
= µ0 × m
Where µ0 is number of lines associated with unit pole. MAGNETIC MOMENT
(v) The number of lines of force per unit area at a point gives The magnetic moment of a magnet in magnitude is equal to the
magnitude of field at that point. The crowded lines show a product of pole strength with effective length (i.e. magnetic
strong field while distant lines represent a weak field.
length). Its direction is along the axis of magnet from south pole
(vi) The magnetic lines of force have a tendency to contract
to north pole.
longitudinally like a stretched elastic string producing
r r r
attraction between opposite pole. M = m ´ 2 l ´ (n) Þ | M |= 2ml
If the same bar magnet is bent in a semicircle then

N S 2l
pr = 2l Þ r =
p

Longitudnal contraction (attraction)


(vii) The magnetic lines of force have a tendency to repel each r
other laterally resulting in repulsion between similar poles. m m

Net magnetic moment

2l 4l 2 M
M ¢ = m ´ 2r = m ´ 2 ´ = m´ =
S N X N p p p
r
Where m is pole strength, 2l is effective length and n is unit
vector having a direction from S-pole to N-pole.
n (attraction) Lateral expansion (repulsion)
r
The SI unit of M is A m2, which is equivalent to J/T..
(viii) The region of space with no magnetic field has no lines of
Circular current loop as a magnet : A small plane loop of current
force. At neutral point where resultant field is zero there
cannot be any line of force. behaves as a magnet with a definite dipole moment given by
uuur
SOME TERMS RELATED TO MAGNETISM M = I Anˆ
Magnetic poles : These are the regions of apparently concentrated
where A is the area of the loop, I the current in the loop and n̂ is
magnetic strength where the magnetic attraction is maximum.
It means that pole of a magnet is located not at a point but over a a unit vector perpendicular to the plane of the loop, and its direction
region. Magnetic poles exist in pairs. An isolated magnetic pole is decided by the sense of flow of current I using the Fleming’s
(north or south) does not exist. If a magnet is cut into two pieces, right hand rule.
then instead of obtaining separate N-pole and S-pole, each of the Relation between magnetic moment and angular momentum
two parts are found to behave as complete magnets.
uuur q uur
Magnetic axis : The line passing through the poles of a magnet is M = L
called its magnetic axis. 2m
Magnetic equator : The line passing through the centre of the Where q is the total charge on a body of mass m rotaing about a
magnet and at right angles ot the magnetic axis is called the fixed axis.
magnetic equator of the magnet.
Free eBooks on @neetquestionpaper2020

Magnetism and Matter 737


Magnetic moment, pole strength and effective length when a magnet is cut
Magnet placed after Pole strength Effective length Magnetic moment
cutting after cutting

N S Breadth
m 2l M = 2ml
Length

N S m M
m/2 2l M1 = .2l =
X X´ 2 2

M
S N m l M 2 = ml =
2

N S ml M
m/2 l M3 = =
X X´ 2 4

2l
m 2l M = m ´ 2l
m m

COULOMB’S LAW OF MAGNETIC FORCE A magnet of dipole moment M suspended freely in a magnetic
It states that : field B experiences a torque t given by
(i) The force of attraction or repulsion between two magnetic ur uuur uur
t = M ´ B ; t = MB sin q
poles is directly proportional to the product of their pole uur ur
strengths. where q is the angle between M and B
r
(ii) The force of attraction or repulsion between two magnetic It is clear from the expression that | tmax |= MB
poles is inversely proportional to the square of the distance i.e., when dipole is perpendicular to field the torque is maximum
between them. This law is also known as inverse square law. and when they are parallel, the torque is minimum
1 r
i.e., F µ m1m2 and F µ r (for q = 0 or 180º ® | t | = 0 ).
The net force acting on a bar magnet placed
m m m ® in a uniform magnetic field is zero
or, F = 0 . 1 2
4p r 2 ® in a non-uniform magnetic field is non-zero
where m1 and m2 are the pole strengths of the two magnetic Let the length of a bar magnet be 2l and pole strength be m,
poles, r is the distance between them and m0 is the the magnetic field is B, and the angle between B and bar
magnet is q. Force on north pole is mB along the field and
permeability of free space.
that on south pole is mB opposite to the field.
Unit magnetic pole : A unit magnetic pole may be defined as the
B mB
pole which when placed in vacuum at a distance of one metre from
an identical pole, repels it with a force of 10–7 newton.
q N
TORQUE ON A MAGNET IN A MAGNETIC FIELD
m(North pole)
N mB O
B lsinq
q Bar magnet at S
an angle q. with
S the magnetic field B
2l
mB m (South pole)
mB
Free eBooks on @neetquestionpaper2020

738 Physi cs
The torque of these two forces about O is Work done in Rotating a Uniform Magnetic Dipole
t = 2mBlsinq = MB sinq in a Magnetic Field
uur uur Work done in deflecting the dipole through an angle q is,
t =M ´B W = MB (1 – cos q)
where M is the magnetic moment of the magnet. If q = 0, cos q = 1 then W = MB (1 – 1) = 0
(Q M = 2ml) If q = 90°, cos q = 0 then W = MB
This torque tries to align the magnet with the field. If q = 180°, cos q = –1, then W = 2MB

Work Done by External Agent in Rotating the Magnet GAUSS'S LAW IN MAGNETISM
r
If an external agent rotates the magnet slowly, the agent has to The surface integral of magnetic field B over a closed surface S
exert a torque MBsinq opposite to that exerted by the field. is always zero.
® ®
Work done by the agent in changing the angle from q to q + dq is
Mathematically Ñò B . da = 0
dW = (MBsinq) dq
S
q (1) Isolated magnetic poles do not exist is a direct consequence
of gauss's law in magnetism.
Wext = ò (MB sin q)dq (2) The total magnetic flux linked with a closed surface is always
q0
zero.
Wext = MB(cos q 0 - cos q) (3) If a number of magnetic field lines are leaving a closed
surface, an equal number of field lines must also be entering
Wext is stored as potential energy of the field-magnet system. the surface.
Thus
Magnetic Flux
U(q) - U(q0 ) = MB(cos q0 - cos q)
The magnetic flux through a given area may be defined as the
If we take U(q 0 ) = 0 for q 0 = 90o , then total number of magnetic lines of force passing through this
Potential energy area. It is equal to the product of the normal components of the
uur uur magnetic field B and the area over which it is uniform. In general,
U (q) = U ( q) - U (90°) = - MB cos q = – M . B uur uuur
Magnetic flux, f = ò B .dA = ò BdA cos q , where q is angle
(i) When q = 0, U = – MB (minimum PE)
A A
(ii) When q = 90º , U = 0 between normal to the area dA with magnetic field B.
(iii) When q = 180º , U = MB (maximum PE)
Magnetic flux linked with a closed surface is zero i.e., ò B .dA = 0
s
M B
The S.I. Unit of magnetic flux is weber (Wb) : If a magnetic field of
N B 1 tesla passes normally through a surface of area 1 square metre,
then the magnetic flux linked with this surface is said to be 1
q = 90° S N weber.
S M Oscillations of a Bar Magnet in a Magnetic Field
A freely suspended magnet of magnetic moment M and of moment
U = – MB (min) of inertia I oscillates simple harmonically in a magnetic field B with
U=0 q = 0° frequency
q = 90° Stable equilibrium

B
+m
N mB
q
B
N M S mB S
–m
Freely suspended bar
U = + MB (max) magnet, at an angle q
with the magnetic field B
q = 180°
Unstable equilibrium 1 MB I
n= , \ Time period , T = 2 p
2p I MB
Free eBooks on @neetquestionpaper2020

Magnetism and Matter 739


MAGNETIC FIELD DUE TO A BAR MAGNET m 2ml m M
| B |= 0 = 0
(i) Magnetic field intensity B1 due to a bar magnet at any point 4 p ( d 2 + l 2 ) 3 / 2 4p ( d 2 + l 2 ) 3 / 2
m0 2 Md
on the axial line of the magnet is B1 = uur m M
4 p (d 2 - l 2 ) 2 0
If d > > l, | B |= 4p 3
d

S N B1 The magnetic field at any point having polar coordinates (r, q)


P relative to centre of magnet or loop
2l m0 M
B= (1 + 3 cos 2 q) and direction is given by
d 4p r 3
where d = distance of the point from the centre of the magnet. 1
tan a = tan q
The direction of B1 is along SN. 2
m m MAGNETIC POTENTIAL
BN = 0
4 p (d - l ) 2 where, m is pole strength. The magnetic potential at a point is defined as the work done in
BN is magnetic field due to north pole, it is directed away from the carrying a unit N-pole from infinity to that point against the
magnet. field. It may also be defined as the quantity whose space rate of
variation in any direction gives the intensity of the magnetic field
m m
BS = 0
4p (d + l) 2 it is directed towards the magnet. dV B
i.e., B=-
dx
m0m é 1 1 ù
\ B = BN - BS Þ B = ê - ú (i) Magnetic potential due to a point dipole, at a distance r
4 p êë (d - l) 2
(d + l) 2 úû
from the pole of strength m is given by
m 0 m (4ld ) m0 2 Md m m
= = V B = 0 . (joule/Wb)
4p ( d 2 - l 2 ) 2 [Q M = m(2L)] 4p r
4p ( d 2 - l 2 ) 2
B due to a pole of Pole strength m at a distance r is given by
0 m 2M
If d > > l, B = 4p 3 m0 m
d B=
4p r 2
(ii) Magnetic field intensity (B 2) due to A bar magnet at any
point on the equatorial line of the bar magnet is Now V0 at A is the work done is bringing a unit pole from
infinity to A.
m M
B2 = 0
4 p ( d 2 + l 2 )3 / 2 r
m 0m
The direction of B2 is along a line parallel to NS. \ VA - V¥ = ò a dr Þ V A = m 0m
2
¥
4pr 4 pr
P (ii) Potential due to a magnetic dipole at a point in end-side on
B
m0 æ m ö
d
position is VB = .ç ÷ , where M = 2ml.
4p è r 2 - l 2 ø
S N
S O
O N P

2l
r
m 0 m NP m m PS
BN = , BS = 0
4p NP 3 4p PS3
m0 m
Now, NP = PS = (d2 + l2 )3/ 2 If l2 < < r2 then VB =
4p r 2
uur uuur uuur
Resultant field at P is, B = BN + BS (iii) Potential due to a magnetic pole at a point in the broad side-
on position. Net potential at P = 0. The potential at any point
m m m m
ÞB = 0. ( NP + PS) = 0 . ( NS) lying on the magnetic equator of a magnet is zero in CGS
4p ( d + l )
2 2 3 / 2 4 p (d + l 2 ) 3 / 2
2
and MKS system.
Free eBooks on @neetquestionpaper2020

740 Physi cs
P
æ1 ö 1 1
= - MB ç - 1÷ = MB = W1
è2 ø 2 2
r
As W1 = n W2 ; \ n =2
Example 4.
N S
In a hydrogen atom, an electron revolves with a frequency
of 6.8 × 109 megahertz in an orbit of diameter 1.06 Å. What
2l
will be the equivalent magnetic moment?
(iv) The magnetic potential at a point lying on a line passing Solution :
through the centre and making angle q with the axis n = 6.8 × 109 MHz = 6.8 × 1015 Hz,

m 0 M cos q 1 .06
VB = and for small dipole, r >> l r= = 0 .53 Å = 0.53 × 10–10 m
4p (r 2 - l 2 ) 2

m 0 M cos q æ e ö 2 2
M = IA = ç ÷pr = enpr
VB = è 1 / n ø
4p r2
22
Example 1. = (1.6 ´ 10 -19 ) (6.80 ´ 1015 ) ´ (0.53 ´10 -10 ) 2
Two identical thin bar magnets each of length l and pole 7
strength m are placed at right angles to each other, with = 9.7 ´10 - 24 A m 2
north pole of one touching south pole of the other, then Example 5.
find the magnetic moment of the system. The time period of oscillation of a magnet in a vibration
N1 magnetometer is 1.5 sec. What will be the time period of
oscillation of another magnet similar in size, shape and
mass but having 1/4 magnetic moment than that of the 1st
magnet oscillating at the same place?
2 Solution :

T2 M1 M1
= = =2; \ T2 = 2 T1 = 3 s .
S1 T1 M2 1
M1
N2 S2 4
Solution : Example 6.
Initial magnetic moment of each magnet = m × l. The time period of oscillation of a magnet is 2 sec. When it
As is clear from fig., S1 and N2 neutralize each other. is remagnetised so that its pole strength is 4 times, what
Effective distance between will be its period?
Solution :
N1 and S2 = l 2 + l2 = l 2 \ M ¢ = ml 2 .
Example 2. T2 M1 m1 ´ 2 l T2 m1 1
= = ; = = ; T = 1 sec.
A steel wire of length l has a magnetic moment M. It is then T1 M2 m2 ´ 2l T1 4 m1 2 2
bent into a semi-circular arc. Find the new magnetic
moment. Example 7.
Solution : A thin rectangular magnet suspended freely has a period
Let d be the diameter of semi-circle. of oscillation of 4s. If it is broken into two equal halves,
\ l = (p d/2) or d = (2 l/p) what will be the period of oscillation of each half ?
New magnetic moment = m × d = m × (2 l/p) Solution :
= 2m l/p = (2 M/p) For each half, mass is half and length is half;
Example 3.
Work done in turning a magnet of magnetic moment M by an m l2
As M.I. = \ M.I. becomes 1/8th.
angle 90º from the magnetic meridian is n times the 12
corresponding work done to turn through an angle of 60º, Also M becomes 1/2
where n is As T = 2 p M.I. / MB
(a) 1/2 (b) 2
(c) 1/4 (d) 1 1/ 8 1
Solution : (b) \ T becomes times = times
1/ 2 2
W1 = – MB (cos 90º – cos 0º) = MB
W2 = – MB (cos 60º – cos 0º) 1
New time period = ´ 4s = 2s
2
Free eBooks on @neetquestionpaper2020

Magnetism and Matter 741

20.1
Solve following problems with the help of above text and 7. Which of these is correct in regard to a magnet?
examples : (a) Geometric length = 0.8 times the magnetic length
1. The magnetic lines of force inside a bar magnet (b) Magnetic length = 0.8 times the geometric length
(a) are from N-pole to S-pole of magnet (c) Magnetic length = Geometric length
(b) do not exist (d) Geometric length = 10/9 of magnetic length
(c) depend upon the area of cross section of bar magnet 8. Magnetic dipole moment is a vector quantity directed from
(d) are from S-pole to N-pole of magnet (a) south pole to north pole
2. A bar magnet of magnetic moment M, is placed in magnetic (b) north pole to south pole
field of induction B. The torque exerted on it is (c) east to west
r r r r (d) west to east
(a) M.B (b) -M.B 9. The ratio of magnetic fields due to a smaller bar magnet in
r r r r the end on position to broad side on position is
(c) M´ B (d) -B ´ M
3. The dimensional formula for magnetic flux is (a) 1/4 (b) 1/2
(a) [ML2T–2 A–1 ] (b) [ML3T–2 A–2 ] (c) 1 (d) 2
0 –2
(c) [M L T A ] 2 –2 (d) [ML2T –1A2 ] 10. A circular loop carrying a current is replaced by an
4. Current I is flowing in a coil of area A and number of turns equivalent magnetic dipole. A point on the axis of the loop
is N, then magnetic moment of the coil is M equal to is in
(a) NIA (b) NI/A (a) end-on position (b) broadside-on position
(c) both (d) None of these
(c) NI/ A (d) N2 AI 11. When a current in a circular loop is equivalently replaced
5. A bar magnet is cut into two equal halves by a plane parallel by a magnetic dipole
to the magnetic axis. Of the following physical quantities (a) the pole strength m of each pole is fixed
the one which remains unchanged is (b) the distance d between the poles is fixed
(a) pole strength (c) the product md is fixed
(b) magnetic moment (d) None of these
(c) intensity of magnetisation 12. Let r be the distance of a point on the axis of a bar magnet
(d) moment of inertia from its centre. The magentic field at such a point is
6. A magnetic needle suspended freely orientates itself proportional to
(a) in a definite direction
(b) in no direction 1 1
(a) (b)
(c) upward r r2
(d) downward
1
(c) (d) None of these
r3
ANSWER KEY
1. (d) 2. (c) 3. (a) 4. (a) 5. (c) 6. (a) 7. (b) 8. (a) 9. (d) 10. (a) 11. (c) 12. (d)

EARTH’S MAGNETISM Geographic meridian : The geographic meridian at a place is the


Magnetic field of earth extends nearly upto five times the radius vertical plane passing through the geographic north & south at
of earth i.e., 3.2 × 104 km. that place.
The magnetic field of earth is fairly uniform and can be represented Magnetic meridian : The magnetic meridian at a place is the vertical
by equidistant parallel lines.
plane passing through the magnetic axis of a freely suspended
NG small magnet. The earth’s magnetic field acts in the plane of
Magnetic axis Sm
Magnetic equator magnetic meridian.
Magnetic meridian
Geographic equator
Earth's magnetic N
Imaginary S
lines of force N Vertical plane
bar magnet in S
the core of Earth passing through
the magnetic axis
to depict earth's
magnetic field
SG Nm

Geographic axis
Free eBooks on @neetquestionpaper2020

742 Physi cs
Elements of Earth’s Magnetic Field Keep in Memory
The earth’s magnetic field at a place can be completely described
by three independent parameters called elements of earth’s 1. Dip circle is an instrument used to measure angle of dip at a
magnetic field. These are : place.
Geographic 2. At poles total magnetic intensity is 0.66 oersted and at
BH
meridian a d
equator it is 0.33 oersted
–d The total magnetic intensity at a particular latitude is
Bcos d d 90
°

Magnetic BI Bv
meridian B I = I 0 1 + 3 sin 2 l
Bv
where l is the angle of latitude
Magnetic
meridian 90°
d=
Magnetic
meridian Magnetic equator

S Horizontal Magnetic axis



line d=
d
Axis of a freely
N suspended magnet 90°
d=
(i) Magnetic declination (q) : The angle between the The angle of dip d = 0° at magnetic equator
geographic meridian and the magnetic meridian at a place and d = 90° at magnetic poles.
is called the magnetic declination at that place. 3. A spectacular effect due to earth's magnetism is observed
(ii) Angle of dip (d) : The angle made by the earth’s total near the magnetic poles of earth. This effect is called aurora-
uur borealis in the north and aurora - austorlis in south. It is
magnetic field B with the horizontal is called angle of
shown by patterns of coloured lights.
dip at any place.
B Magnetic Maps
tan d = H
BV Magnetic surveys all over the earth have been carried out and
True and apparent dip magnetic maps have been prepared which show the values of
When the plane of the dip circle is in the magnetic meridian, then magnetic element throughout the world. Lines can be drawn to
the needle stops in the actual direction of the Earth's magnetic join places having the same value of a particular magnetic element.
field. The angle made by the needle with the horizontal in this (i) Isogonic lines : These join places of equal declination. A
condition is called true dip. line joining places of zero declination is called agonic line.
In case, the plane of the dip circle is not in the magnetic meridian (ii) Isoclinic lines : These join places of equal dip.
then the angle made by the needle with the horizontal is called A line joining the places of zero-dip is called aclinic line.
apparent dip. In this case the vertical component of earth's magnetic
(iii) Isodynamic lines : These join places of equal horizontal
field remains the same but the effective horizontal component
component.
B'H = BHcos q
B'H Shielding from magnetic fields : For shielding a certain region of
Plane in q space from magnetic field, we surround the region by soft iron
which the dip rings. Magnetic field lines will be drawn into the rings and the
BH space enclosed will be free of magnetic field.
circle is
present
Neutral Points
Neutral points are the points where the net field intensity due to
Magnetic the field of the bar magnet and field of earth is zero. When magnet
meridian
is placed with its north pole towards geographic north, neutral
BV¢ BV points lie on equatorial line of the magnet. At each neutral point,
tan d¢ = =
BH¢ BH cos q m0 M
B2 = =H
where d¢ = apparent dip 4p (d 2 + l 2 )3/ 2
where H = horizontal component of earth’s magnetic field.
d = true dip
When the bar magnet is placed with its north pole towards
If the dip circle is rotated by 90°, the new apparent dip d¢¢ and d¢ geographic south, the neutral points lie on the axial line of the
and d are related as cot 2 d = cot 2 d¢ + cot 2 d¢¢ magnet. At each neutral point,
(iii) Horizontal component of earth’s magnetic field (B H) : It is m M
uur B1 = 0
4p (d 2 - l 2 ) 2
=H
the component of the earth’s total magnetic field B in the
horizontal diection and is given by, BH = B cos d
Free eBooks on @neetquestionpaper2020

Magnetism and Matter 743


Relation between the units of quantities associated with magnetic Example 9.
field : A uniform magnetic needle is suspended from its centre
1A = 1JT–1 m–2 = 1J Wb–1 by a thread. Its upper end is now loaded with a mass
1T = 1JA–1 m–2 = 1Wb m–2 of 50 milligram, when the needle becomes horizontal.
1Wb = 1JA–1 = 1Tm2 If the strength of each pole is 98.1 ab amp-cm and g = 981
[B] = NA–1 m–1 = T = Wb m–2 cm/s2, then find the vertical component of earth’s magnetic
[M] = A m2 = JT–1 = J m2 Wb–1 induction.
[m0] = NA–2 = T2 m2 N–1 = Wb2 J–1 m–1 Solution :
Example 8. Here, mass M = 50 milligram = 50 × 10–3 gram
A circular coil of radius 0.157 m has 50 turns. It is placed
mV
such that its axis is in magnetic meridian. A dip needle is
supported at the centre of the coil with its axis of rotation
horizontal and in the plane of the coil. The angle of dip is O
30º, when a current flows through the coil. The angle of
dip becomes 60º on reversing the current. Find the current mV
Mg
in the coil assuming that magnetic field due to the coil is
2
smaller than the horizontal component of earth’s field. Take
H = 3 × 10–5 T.
strength of each pole, m =98.1 ab amp-cm, g = 981 cm/s2, V = ?
Solution :
In equilibrium, mV× 2l = Mg × l
If H is horizontal component and V is vertical component at
the place, then true value of dip (d) at the place is given by, Mg 50 ´ 10-3 ´ 981
BV = or BV = = 0.25 gauss
V 2m 2 ´ 98.1
tan d =
H Example 10.
If B is magnetic field intensity at the centre of coil due to current
If q1 and q2 are angles of dip in two vertical planes at
V right angle to each other and q is true dip then prove
I in circular coil and B is along H, then tan 30º = .
H+B cot2q = cot2q1 + cot2q2.
On reversing the direction of current, the direction of B is Solution :
reversed. If the vertical plane in which dip is q1 subtends an angle a
V with meridian than other vertical plane in which dip is q2 and
\ tan 60º =
H-B is perpendicular to first will make an angle of 90° – a with
magnetic meridian. If q1 and q2 are apparent dips then
tan 60º H + B 3
Dividing, we get = = =3 BV BV
tan 30º H - B 1 / 3 BV
tan q1 = ; tan q 2 = =
BH cos a B H cos(90 - a ) B H sin a
3 ´ 10 -5
\ B = H/2 Þ B = = 1.5 ´ 10 -5 T 1 1
2 cot 2 q1 + cot 2 q 2 = +
2
m nI (tan q1 ) (tan q 2 ) 2
For a circular coil, B = o
2r
2
-7 B2H cos 2 a + B2H sin 2 a B2H æ B cos q ö
\ 1.5 × 10–5 = (4 p ´ 10 ) ´ 50 ´ I = = =ç = cot 2 q
B2V B2V è B sin q ÷ø
2 ´ 0.157
-5 s o cot2q1 + cot2q2 = cot2q
\ I = 1.5 ´ 10 ´ 2 ´ 0.157 = 0.75 A .
-7
4p ´ 10 ´ 50

20.2
Solve following problems with the help of above text and 2. At the magnetic north pole of the earth, the value of the
examples : horizontal component of earth’s magnetic field and angle
1. The lines of force due to earth’s horizontal magnetic field of dip are respectively
are (a) zero, maximum (b) maximum, minimum
(a) parallel and straight (b) concentric circles (c) maximum, maximum (d) minimum, minimum
(c) elliptical (d) curved lines
Free eBooks on @neetquestionpaper2020

744 Physi cs

3. The magnetic compass is not useful for navigation near 6. At magnetic poles, the angle of dip is
the magnetic poles, since (a) 45º (b) 30º
(a) R = 0 (b) V = 0 (c) zero (d) 90º
(c) H = 0 (d) q = 0º 7. A compass needle which is allowed to move in a horizontal
4. The earth’s magnetic field always has a vertical component plane is taken to a geomagnetic pole. It
except at the (a) will stay in north-south direction only
(a) magnetic equator (b) will stay in east-west direction only
(b) magnetic poles (c) will become rigid showing no movement
(c) geographic north pole (d) will stay in any position
(d) latitude 45º 8. A dip circle is taken to geomagnetic equator. The needle is
5. A magnetic needle suspended horizontally by an unspun silk allowed to move in a vertical plane perpendicular to the
fibre oscillates in the horizontal plane, because of the restoring mangetic meridian. the needle will stay in
force originating mainly from (a) horizontal direction only
(a) the torsion of the silk fibre (b) vertical direction only
(b) the horizontal component of earth’s magnetic field (c) any direction except vertical and horizontal
(c) the force of gravity (d) any direction it is released
(d) all of the above
ANSWER KEY
1. (b) 2. (a) 3. (c) 4. (a) 5. (b) 6. (d) 7. (d) 8. (d)

TERMS RELATED TO MAGNETISM Also B = B0 + Bm


where B0 = magnetic induction in vacuum produced by
Magnetic intensity (H ) : When a magnetic material is placed in magnetising field
a magnetic field, it becomes magnetised. The capability of the Bm = magnetic induction due to magnetisation of material.
magnetic field to magnetise a material is expressed by means of But B0 = m0H and Bm = m0 I m Þ B = m 0 [H + I m ]
uur
a magnetic vector H , called the `magnetic intensity’ of the field. Im
\ B = m 0 H [1 + ] = B0 [1 + c ] ; \ B / B0 = [1 + c]
The relation between magnetic induction B and magnetising field H
B mH
H H = , m being permeability of medium. \ B / B0 = = m / m0 = µr, the relative magnetic permeability
m m0 H
Intensity of magnetisation (I) : When a material is placed in a
magnetising field, it acquires magnetic moment M. The intensity \ mr =1+ c
of magnetisation is defined as the magnetic moment per unit
This is required relation.
M
volume i.e., I m =
V CLASSIFICATION OF MATERIALS
V being volume of mateiral. If the material is in the form of a bar of According to the behaviour of substances in magnetic field, they
cross-sectional area A, length 2l and pole strength m, then are classified into three categories:
1. Diamagnetic Substances : These are the substances which
M m.2l m
M = m × 2l; V = A × 2l \
\ Im = = = when placed in a strong magnetic field acquire a feeble
V A.2 l A magnetism opposite to the direction of magnetising field.
Magnetic susceptibility (c) : The magnetic susceptibility is defined The examples are copper, gold, antimony, bismuth, alcohol,
as the intensity of magnetisation per unit magentising field. water, quartz, hydrogen, etc.
Im
i.e., c = N S
H
Magnetic permeability (m) : The magnetic permeability of a material N S
is the measure of degree to which the material can be permited by
a magnetic field and is defined as the ratio of magnetic induction
N S
B Bo
(B) in the material to the magnetising field i.e. m =
H Diamagnetic
B<B0
Relation between Magnetic Susceptibility and
Permeability
r
We have magnetic induction in mateiral, B = mH Behaviour of diamagnetic substance in an external magnetic field Bo
Free eBooks on @neetquestionpaper2020

Magnetism and Matter 745


The characteristics of diamagnetic substances are This is called Curie law. At a temperature called curie
(a) They are feebly repelled by a strong magnet temperature, ferromagnetic substance becomes
(b) Their susceptibility is negative (i.e. c < 0) paramagnetic. The curie temperatures for Ni, Fe and
Co are 360ºC, 740ºC and 1100ºC respectively.
(c) Their relative permeability is less than 1. (i.e. mr < 1) (f) They are found in those material which have domains
(d) Their susceptibility is independent of magnetising field and can be converted into strong magnets
and temperature (except for Bismuth at low
Keep in Memory
temperature)
2. Paramagnetic Substances : These are the materials which 1. Diamagnetism is universal. It is present in all materials. But
when placed in a strong magnetic field acquire a feeble it is weak and hard to detect if substance is para or
magnetism in the same sense as the applied magnetic field. feromagnetic.
The examples are platinum, aluminium, chromium, 2. I – H curve for different materials
manganese, CuSO4, O2, air, etc. Ferromagnetic
I
Paramagnetic
N S
N S D iama
g netic H
3. Curve for magnetic susceptibility and temperature for a
paramagnetic and ferromagnetic material.
N S
X
Bo Paramagnetic
B > B0 1

T
r
Behaviour of paramagnetic substance in an external field Bo T

The characteristics of paramagnetic substances are HYSTERESIS


(a) They are attracted by a strong magnet When a bar of ferromagnetic material is magnetised by a varying
(b) Their susceptibility is positive but very small ( c > 0) magnetic field and the intensity of magnetisation I m induced is
(c) Their relative permeability is slightly greater than unity. measured for different values of magnetising field H, the graph of
(m > 1) I versus H is as shown in fig
(d) Their susceptibility and permeability do not change
with the variation of magnetising field.
Im
(e) Their susceptibility is inversely proportional to
B A
temperature, æç i.e. c a ö÷ .
1
è Tø
(f) They are found in those material which have atoms C O G H
containing odd number of electrons
3. Ferromagnetic Substances : These are the substances which F
D
are strongly magnetised by relatively weak magnetising
The graph shows :
field in the same sense as the magnetising field. The (i) When magnetising field is increased from O the intensity of
examples are Ni, Co, iron and their alloys. magnetisation I m increases and becomes maximum (i.e.
The characteristics of ferromagnetic substances are point A). This maximum value is called the saturation value.
(a) They are attracted even by a weak magnet. (ii) When H is reduced, I m reduces but is not zero when H = 0.
(b) The susceptibility is very large and positive. The remainder value OB of magnetisation when H = 0 is
( c >> 0) called the residual magnetism or retentivity. OB is retentivity.
(c) The relative permeability is very high (of the order of (iii) When magnetic field H is reversed, the magnetisaiton
hundreds and thousands). (m >> 1) decreases and for a particular value of H, it becomes zero
i.e., for H = OC, I = 0. This value of H is called the coercivity.
(d) The intensity of magnetisation is proportional to the
(iv) When field H is further increased in reverse direction, the
magnetising field H for smaller values, varies rapidly intensity of magnetisation attains saturation value in reverse
for moderate values and attains a constant value for direction (i.e., point D).
larger values of H. (v) When H is decreased to zero and changed direction in steps,
(e) The susceptibility of a ferromagnetic substance is we get the part DFGA.
inversely proportional to temperature i.e., c µ1/ T
C
Þ c = ; C = curie constant .
T
Free eBooks on @neetquestionpaper2020

746 Physi cs
Properties of Soft Iron and Steel
For soft iron, the susceptibility, permeability and retentivity are f 2.4 ´ 10-5
B= = = 1.2 weber / m 2 ;
greater while coercivity and hysteresis loss per cycle are smaller A 0.2 ´ 10-4
than those of steel.
B 1. 2
m= = = 7.5 ´ 10 - 4 TA -1m ;
B B H 1600
[Q As m = m 0 (1 + c m ) ]
Hysteresis
curve
m 7.5 ´10 -4
\ cm = -1 = -1 ;
Hysteresis H H m0 4p ´10 -7
curve Soft magnetic Hard magnetic
material
material 7.5 ´103
cm = - 1 = 597.1 - 1 = 596.1
4p
PERMANENT MAGNETS AND ELECTROMAGNETS Example 12.
Permanent magnets are made of steel and cobalt while A sample of paramagnetic salt contains 2 × 1024 atomic
electromagnets are made of soft iron. dipoles, each of moment 1.5 × 10–23 JT–1. The sample is
An electromagnet is made by inserting a soft iron core into the placed under a homogeneous magnetic field of 0.84 T and
interior of a solenoid. Soft iron does not retain a significant cooled to a temperature of 4.2 K. The degree of magnetic
permanent magnetization when the solenoid’s field is turned off– saturation achieved is equal to 15%. What is the total
soft iron does not make a good permanent magnet. When current dipole moment of the sample for a magnetic field of 0.98 T
flows in the solenoid, magnetic dipoles in the iron tend to line up and a temperature of 2.8 K. Assume Curie’s law.
with the field due to the solenoid. The net effect is that the field Solution :
inside the iron is intensified by a factor known as the relative
Here, number of dipoles n = 2 × 1024
permeability. The relative permeability is analogous in magnetism
Dipole moment of each dipole, M' = 1.5 × 10–23 JT–1
to the dielectric constant in electricity. However, the dielectric
constant is the factor by which the electric field is weakened, Total dipole moment of sample
while the relative permeability is the factor by which the magnetic = n × M' = 2 × 1024 × 1.5 ×10–23 = 30
field is strengthened. The reactive permeability of a ferromagnet As saturation achieved is 15%, therefore, effective dipole
can be in the hundreds or even thousands–the intensification of moment
the magnetic field is significant. Not only that, but in an
15
electromagnet the strength and even direction of the magnetic M1 = ´ 30 = 4.5 JT -1 ; B1 = 0.84 T, T1 = 4.2 K
field can be changed by changing the current in the solenoid. 100
M2 = ? B2 = 0.98 T, T2 = 2.8 K.
Keep in Memory
C Ιm CH
1. By alloying soft-iron with 4% silicon ‘transformer steel’ is According to Curie’s law, c m = = or Ι m =
T H T
produced. It has a higher relative permeability and is an ideal
material for cores of transformers. Alloys of iron and nickel As and H µ B
Ιm µ M
called ‘permalloys’, also have very large permeabilities. CB M 2 B 2 T1
\ Mµ , = .
2. Energy spent per unit volume of specimen is complete cycle T M1 B1 T2
of magnetisation is numerically equal to area of I – H loop
B 2 T1M1 0.98 ´ 4.2 ´ 4.5
or M 2 = = , M2 = 7.875 JT–1.
Perme- Suscep- Intensity of Reten- Coerc- Hysteresis T2 B1 2.8 ´ 0.84
ability tibility magnetisation tivity ivity loss
Example 13.
Soft iron high high high high low low
A solenoid of 500 turns/m is carrying a current of 3A. Its
Steel low low low low high high core is made of iron which has a relative permeability of
Þ Steel is most suitable for making parmanent magnet 5000. Determine the magnitude of magnetic intensity,
Þ Soft iron is most suitable for making core of an magnetisation and magnetic field inside the core.
electromagnet. Solution :
Magnetic intensity
Example 11.
H = ni = 500 × 3 = 1500 A/m
A magnetising field of 1600 Am–1 produces a magnetic
µr = 1 + cm so cm = µr – 1 = 4999 » 5000
flux of 2.4 × 10–5 weber in a bar of iron of cross section 0.2
Intensity of magnetisation
cm2. Calculate permeability and susceptibility of the bar.
Solution : I = cH = 5000 × 1500 = 7.5 × 106 A/m
Here, H = 1600 Am–1, f = 2.4 ×10–5 Wb. Magnetic field B = µr µ0 H = 5000 × 4p × 10–7 × 1500
A = 0.2 cm2 = 0.2 × 10–4 m2, m = ? cm = ? = 9.4 tesla.
Free eBooks on @neetquestionpaper2020

Magnetism and Matter 747

20.3
Solve following problems with the help of above text and 10. Susceptibility is positive and large for a
examples : (a) paramagnetic substance
1. Which of the following is not a magnetic substance? (b) ferromagnetic substance
(a) Brass (b) Iron (c) diamagnetic substance
(c) Cobalt (d) Nickel (d) non magnetic substance
2. Metals getting magnetised by orientation of atomic 11. A ferromagnetic material is heated above its curie
magnetic moments in external magnetic field are called temperature. Which one is a correct statement?
(a) diamagnetics (b) paramagnetics (a) Ferromagnetic domains are perfectly arranged
(c) ferromagnetics (d) antimagnetics (b) Ferromagnetic domains become random
3. The magnetic susceptibility for diamagnetic materials is (c) Ferromagnetic domains are not influenced
(a) small and negative (b) small and positive (d) Ferromagnetic material changes into diamagnetic
(c) large and positive (d) large and negative material
4. The B – H curve (i) and (ii) shown in fig associated with 12. The direction of magnetic lines of force of a bar magnet is
(a) from south to north pole
(b) from north to south pole
(i) (c) across the bar magnet
(ii) (d) from south to north pole inside the magnet and from
H north to south pole outside the magnet
13. Which magnetic materials have negative susceptibility?
(a) Diamagnetic materials
(b) Paramagnetic materials
(a) (i) diamagnetic and (ii) paramagnetic substance (c) Ferromagnetic materials
(b) (i) paramagnetic and (ii) ferromagnetic substance (d) All the above
(c) (i) soft iron and (ii) steel 14. A bar magnet of magnetic moment M, is placed in a magnetic
(d) (i) steel and (ii) soft iron field of induction B. The torque exerted on it is
5. The ratio of intensity of magnetisation and magnetising r r r r
field is called (a) M . B (b) - M . B
(a) permeability (b) magnetic intensity r r r r
(c) M ´ B (d) - B. M
(c) magnetic intensity (d) magnetic susceptibility
6. The meniscus of a liquid contained in one of the limbs of a 15. If a diamagnetic substance is brought near north or south
narrow U-tube is held in an electromagnet with the meniscus pole of a bar magnet, it is
in line with the field. The liquid is seen to rise. This indicates (a) attracted by poles
that the liquid is (b) repelled by poles
(a) ferromagnetic (b) paramagnetic (c) replaced by north pole and attracted by south pole
(c) diamagnetic (d) non-magnetic (d) attracted by north pole and repelled by south pole
7. Which of the following instruments is used to measure 16. A material is placed in a magnetic field and it is thrown out
magnetic field? of it. Then the material is
(a) Thermometer (b) Pyrometer (a) paramagnetic (b) diamagnetic
(c) Hygrometer (d) Flux meter (c) ferromagnetic (d) non-magnetic
8. If a diamagnetic solution is poured into a U-tube and one 17. Among which the magnetic susceptibility does not depend
arm of this U-tube is placed between the poles of a strong on the temperature?
magnet, with the meniscus in line with the field, then the (a) Dia-magnetism (b) Para-magnetism
level of solution will (c) Ferro-magnetism (d) Ferrite
(a) rise (b) fall 18. If m0 is absolute permeability of vacuum and mr is relative
(c) oscillate slowly (d) remain as such magnetic permeability of another medium, then permeability
9. A magnetic needle suspended by a silk thread is vibrating m of the medium is
in the earth’s magnetic field. if the temperature of the needle (a) m0 mr (b) m0/mr
is increased by 700ºC, then (c) mr/m0 (d) 1/m0 mr
(a) time period decreases 19. The relation between B, H and I in S.I. units is
(b) time period increases (a) B = m0 (H + I) (b) B = H + 4 p I
(c) time period remains unchanged
(c) H = m 0 (B + I) (d) None of these
(d) the needle stops vibrating
Free eBooks on @neetquestionpaper2020

748 Physi cs

20. A temporary magnet is made of 25. Of dia, para and ferromagnetism, the universal property of
(a) cast iron (b) steel all substances is
(c) soft iron (d) stainless steel (a) diamagnetism (b) paramagnetism
21. Hysterisis is the phenomenon of lagging of (c) ferromagnetism (d) all of the above
(a) I behind B (b) B behind I 26. The hysteresis cycle for the material of permanent magnet is
(c) I and B behind H (d) H behind I (a) short and wide (b) tall and narrow
22. The narrowest hysterisis loop is for (c) tall and wide (d) short and narrow
(a) cobalt steel (b) alnico 27. A diamagnetic material in a magnetic field moves
(c) perm alloy (d) stainless steel (a) perpendicular to field
23. The hysterisis curve is studied generally for (b) from stronger to weaker parts of field
(a) ferromagnetic materials (c) from weaker to stronger parts of the field
(b) paramagnetic materials (d) None of these
(c) diamagnetic materials 28. Curie temperature is the temperature above which
(d) all of the above (a) a ferromagnetic material becomes paramagenetic
24. In a cassette player, material used for coating magnetic (b) a paramagnetic material becomes diamagnetic
tapes is (c) a ferromagnetic material becomes diamagnetic
(a) cobalt (b) CoFe2 O4 (d) a paramagnetic meterial becomes ferromagnetic
(c) NiFe2O4 (d) Nickel

ANSWER KEY
1. (a) 2. (b) 3. (a) 4. (c) 5. (d) 6. (b) 7. (d) 8. (b) 9. (d) 10. (b) 11. (b) 12. (d)
13. (a) 14. (c) 15. (b) 16.(b) 17. (a) 18. (a) 19.(a) 20. (a) 21. (c) 22. (c) 23. (a) 24. (c)
25. (a) 26. (c) 27. (b) 28. (a)

TANGENT GALVANOMETER Magnetic field of earth extends nearly upto five times the
It is an instrument used for measuring small current. It is based on radius of earth i.e., 3.2 × 104 km.
tangent law. It is a moving magnet and fixed coil type galvanometer. The magnetic field of earth is fairly uniform and can be
Tangent Law : If a small magnetic needle is under the influence represented by equidistant parallel lines.
of two crossed magnetic fields (B) and (H) and suffers a deflection VIBRATION MAGNETOMETER
q from field H, then by tangent law, B = H tan q. It is an instrument for comparing the magnetic moments of two
Formula for current : If a current passing through the coil of n magnets and for comparing their magnetic fields.
turns and mean radius r of a tangent galvanometer placed in The time period of a bar magnet vibrating in the vibration
magnetic meridian causes a deflection q in the magnetic needle magnetometer kept in magnetic meridian is given by
kept at the centre of the coil, then
I 4 p2 I
T = 2p \ M =
æ 2rH ö 2rH MH T 2H
I=ç tan q ; I = K tanq where K = and is called the
è m 0 n ÷ø m0 n
æ m(l 2 + b2 ) ö
reduction factor. where I = ç ÷ is the moment of inertia of the vibrating
è 12 ø
DEFLECTION MAGNETOMETER
magnet, m = mass of magnet, l = length of magnet, b = breadth of
It's working is based on the principle of tangent law. magnet.
(a) Tan A Position : In this position the magneto- meter is set Example 14.
perpendicular to magnetic meridian so that, magnetic field
A vibration magnetometer consists of two identical bar
due to magnet is in axial position and perpendicular to earth’s
magnets, placed one over the other, such that they are
field and hence
mutually perpendicular and bisect each other. The time
m0 2Md period of oscillation in a horizontal magnetic field is 4
= H tan q
4p (d 2 - l2 )2 second. If one of the magnets is taken away, find the period
where d = distance of needle from centre of magnet and of oscillation of the other in the same field.
2l = length of magnet. Sol. For a vibration magnetometer, we know that T = 2p I / MH
(b) TanB position : The arms of magnetometer are set in Let M be the magnetic moment and M.I, moment of inertia
magnetic meridian, so that the field is at equatorial position of each magnet,
m0 M
and hence, H tan q = \ M¢ = M2 + M2 = M 2
4p ( d 2 - l 2 )3 / 2
Free eBooks on @neetquestionpaper2020

Magnetism and Matter 749


and net M.Ι ¢ = M.Ι + M.Ι = 2M.Ι Example 17.
A small magnet oscillating in earth’s magnetic field has a
2 M.I. 2 M.I. time period of 4 seconds. When another magnet is brought
\ T ¢ = 2p = 2´ ....(1)
M 2H MH near it, it performs 50 oscillations in 160 seconds. Compare
the magnetic fields due to the magnet and the horizontal
When one of the magnets is taken away, component of earth’s field when (a) both the fields are in
M² = M, MI² = MI, the same direction and (b) both the fields are in opposite
M.Ι directions.
\ T¢¢ = 2p ...(2)
MH Sol. When the magnet oscillates in earth’s horizontal field in
Divide eqn. (2) by (1), vibration magnetometer, we have

T ¢¢ 1 T¢ 4 æ Ι ö
= or T ¢¢ = = = 3 .34 s T = 4 = 2p ç ...(1)
T ¢ ( 2)1 / 4 1/ 4 1/ 4
( 2) 2 è M BH ÷ø
Example 15.
On bringing another magnet near it, one more field B acts.
Two bar magnets of the same mass, same length and breadth
Now,
but having magnetic moments M and 2M are joined
(a) when both the fields are in the same direction, the
together pole for pole and suspended by a string. The time
resultant field will be (BH + B). Then time period is
period of assembly in a magnetic field of strength H is 3
seconds. If now the polarity of one of the magnets is reversed given by
and the combination is again made to oscillate in the same
160 é Ι ù
field, find the time of oscillation. = 2p ê ú ....(2)
Sol. For pole combination, 50 ë M ( BH + B ) û
M1 = 2 M + M = 3 M; T1 = 3 s. Dividing eqn. (1) by (2), we get
When polarity of one is reversed,
M2 = 2 M – M = M; T1 = ? 4´ 5 æ B + Bö
= çç H ÷
÷ .....(3)
16 è BH ø
T2 M1 3M
= = = 3 Þ T2 = 3 T1 = 3 3 s
T1 M2 M 25 B
Squarring, we have =1+
Examples 16. 16 BH
The magnetic needle of an oscillation magnetometer makes
10 oscillations per minute under the action of earth’s 9 B
or = .....(4)
magnetic field alone. When a bar magnet is placed at some 16 B H
distance along the axis of the needle, it makes 14 \ B : BH = 9 : 16
oscillations per minute. If the bar magnet is turned so that
(b) when the fields are in opposite directions, the resultant
its poles interchange their positions, then what will be the
field will be (B – BH). Now we have
new frequency of oscillation of the needle?

1 MH 160 é Ι ù
Sol. 10 = ...(i) = 2p ê ú ...(5)
2p I 50 ë M(B - B )
H û

Dividing eqn. (1) by (5), we get


1 M (H + F)
14 = ...(ii) 5 (B - BH )
2p I = 25 B - B H B
4 BH or = = -1
16 BH BH
1 M (H - F) ...(iii)
n= 41 B
2p I or = \ B : BH = 41 : 16
16 BH
14 H+F F 7
Divide eqs. (ii) by (i), = = 1+ = Example 18.
10 H H 5
The period of oscillation of a magnet in a vibration
F 24 magnetometer is 2 sec. What will be the period of oscillation
\ =
H 25 of a magnet whose magnetic moment is four times that of
n H-F F 1 the first magnet?
Divide eqs. (iii) by (i), = = 1- =
10 H H 5
æ Ι ö
10 Sol. T = 2p ç
or, n= = 2 vibs /minute. è M BH ÷ø
5
Free eBooks on @neetquestionpaper2020

750 Physi cs
Example 20.
æ Ι ö 1é Ι ù
T ' = 2p ç ÷ = ê2p ú In a vibration magnetometer, the time period of a bar magnet
è 4M BH ø 2 ë (M BH ) û
oscillating in horizontal component of earth’s magnetic
1 field is 2 sec. When the magnet is brought near and parallel
= ´ 2 = 1 second. to it, the time period reduces to 1 sec. Find the ratio of F/H
2
Example 19. of the horizontal component H and the field F due to
A magnet is suspended in such a way that it oscillates in magnet.
the horizontal plane. It makes 20 oscillations per minute
at a place where dip angle is 30º and 15 oscillations per æ Ι ö
minute at a place where dip angle is 60º. What will be the Sol. We know that, T = 2p ç
ratio of the total earth’s magnetic field at the two places? è M BH ÷ø
Sol. Let the total magnetic fields due to earth at the two places
be B1 and B2. If horizontal components be (BH)1 and (BH)2 1/ 2
é Ι ù
respectively, then Here, T = 2 = 2p ê ú where BH = H
(BH)1 = B1 cos 30º and (BH)2 = B2 cos 60º ëM Hû
Here T1 = 3 sec. and T2 = 4 sec. When the magnet is brought near and parallel to it, the time
period reduces to 1 sec. i.e., time period decreases. Hence
æ Ι ö
T1 = 3 = 2p ç the field is now (H + F). Hence
è M B1 cos 30 ÷ø
1/ 2
é Ι ù
æ Ι ö T = 1 = 2p ê ú
and T2 = 4 = 2p ç ë M (H + F) û
è M B2 cos 60º ÷ø
1/ 2
1/ 2 2 éH + Fù 4 æH +Fö
3 æ B cos 60 ö B1 16 cos 60 \ = or =ç ÷
\ = çç 2 ÷ or = ´ 1 êë H úû 1 è H ø
4 è B1 cos 30 ÷ø B2 9 cos 30

B1 16 1 2 16 F F
or = ´ ´ = or B1 : B2 = 16 : 9 3 or 4 = 1 + or =3
B2 9 2 3 9 3 H H

20.4
Solve following problems with the help of above text and 3. A tangent galvanometer is connected directly to an ideal
examples : battery. If the number of turns in the coil is doubled, the
deflection will
1. If the current is doubled, the deflection is also doubled in
(a) increase
(a) a tangent galvanometer
(b) decrease
(b) a moving-coil galvanometer (c) remain unchanged
(c) both (d) either increase or decrease
(d) None of these 4. The formation of a dipole is due to two equal & dissimilar
2. To measure the magnetic moment of a bar magnet, one point charges placed at a
may use (a) short distance (b) long distance
(a) a deflection glavanometer if the earth’s horizontal field (c) above each other (d) None of these
is known 5. Two magnets of magnetic moments M & 2M are placed in
a vibration magnetometer, with the identical poles in same
(b) an oscillation magnetometer if the earth’s horizontal
direction. The time period of vibration is T1. If the magnets
field is known are placed with opposite pole together and vibrate with
(c) both deflection and oscillation magnetometer if the time period T2 then
earth’s horizontal field is not known. (a) T2 is infinte (b) T2 = T1
(d) all of the above (c) T2 > T1 (d) T2< T1

ANSWER KEY
1. (b) 2. (d) 3. (c) 4. (a) 5. (c)
Free eBooks on @neetquestionpaper2020

Magnetism and Matter 751

Very Short / Short Answer Questions 16. A magnetized needle is placed 30° with the direction of
uniform magnetic field of intensity 3 × 10–2 T. The torque
1. Name the elements of earth’s magnetic field.
acting onthe needle is 7.2 × 10–4J. Calculate the magnetic
2. The angle of dip at two places onthe surface of the earth are
moment of the needle.
respectively 0° and 90°. Where are these places located?
17. A short bar magnet placed with its axis at 30° to a uniform
3. Write mathematical form of tangent law in magnerism.
magnetic field of 0.2 T experiences a torque of 0.060 Nm. (i)
4. Write the relation between relative permeability and
Calculate magnetic moment of the magnet and (ii) find out
susceptibility.
what orientation of the magnet corresponds to its stable
5. What are the probable causes of the earth’s magnetism?
equilibrium in the magnetic field.
6. How will you distinguish between a diamagnetic and a
18. A magnetic compass needle of magnetic moment 60 Am2 is
paramagnetic material with respect to their behaviour in a
uniform and non-uniform field? placed at a place. The needle points towards geographical
north. Using the data given below, find hte value of
7. A magnetic needle free to rotate in a horizontal position
declination at that place. Horizontal component of earth’s
orients itself with its axis vertical at a certain place on the
magnetic field = 40 × 10–6 Wb m2 and torque experienced by
earth. What are the values of (i) angle of dip and (ii) horizontal
the needle = 1.2 × 10–3 Nm.
component of the earth’s field at the place? Where will be
this place on the earth? Multiple Choice Questions
8. (a) How does a diamagnetic material behave when it is
19. The main difference between electric lines of force and
cooled to very low temperature?
magnetic lines of force is
(b) Why does a paramagnetic sample display greater
(a) electric lines of force are closed curves whereas magnetic
magnetisation when cooled? Explain.
lines of force are open curves
[Delhi Board - 2012 COMPTT.]
(b) electric lines of force are open curves whereas magnetic
9. A bar magnet is moved in the direction indicated by the
lines of force are closed curves
arrow between tow coils PQ and CD. Predict the direction of
induced current in each coil. [Outside Delhi - 2012] (c) magnetic lines of force cut each other whereas electric
lines of force do not cut
P Q C D (d) electric lines of force cut each other whereas magnetic
N S lines of force do not cut
A A 20. The ratio of intensities of magnetic field, at distance x and 2x
from the center of magnet of length 2cm on its axis, will be
10. The horizontal component of the earth’s magnetic field at a
place is B and angle of dip is 60°. What is the value of (a) 4 : 1 (b) 4 : 1 approx
vertical component of earth’s magnetic field at equator? (c) 8 : 1 (d) 8 : 1 approx
[Delhi Board - 2010] 21. A bar magnet is oscillating in the earth’s magnetic field with
11. If the horizontal and vertical components of the Earth’s a period T. What happens to its period of motion, if its mass
magnetic field are equal at a certain place, what would be is quadrupled ?
the angle of dip at that place? (a) Motion remains simple harmonic with new period = T/2
[Delhi Board - 2011 COMPTT.] (b) Motion remains simple harmonic with new period = 2 T
12. Where on the surface of Earth is the angle of dip zero?
(c) Motion remains simple harmonic with new period = 4T
[Outside Delhi - 2011]
(d) Motion remains simple harmonic and the period stays
13. Where on the surface of Earth is the angle of dip 90°?
nearly constant
[Outside Delhi - 2011]
22. The work done in turning a magnet of magnetic moment M
Long Answer Questions by an angle of 90° from the meridian, is n times the
14. Distinguish between dia, para and ferromagnetic materials corresponding work done to turn it through an angle of 60°.
according to their different properties. The value of n is given by
15. What are the magnetic elements of earth? Explain them briefly. (a) 2 (b) 1 (c) 0.5 (d) 0.25
Free eBooks on @neetquestionpaper2020

752 Physi cs
23. Current i is flowing in a coil of area A and number of turns N, 25. The line on the earth surface joining the point where the field
then magnetic moment of the coil, M is is horizontal, is called
(a) magnetic equator (b) magnetic line
Ni Ni
(a) NiA (b) (c) (d) N2Ai (c) magnetic axis (d) magnetic inertia
A A
26. When a ferromagnetic material is heated to temperature above
24. Nickel shows ferromagnetic property at room temperature. If its Curie temperature, the material
the temperature is increased beyond Curie temperature, then (a) is permanently magnetized
it will show (b) remains ferromagnetic
(a) anti ferromagnetism (b) no magnetic property (c) behaves like a diamagnetic material
(c) diamagnetism (d) paramagnetism (d) behaves like a paramagnetic material

1. A bar magnet is oscillating in the Earth’s magnetic field 7. Two magnets of magnetic moments M and 2M are placed
with a period T. What happens to its period and motion if in a vibration magnetometer, with the identical poles in the
its mass is quadrupled? [CBSE PMT 1994, 2003] same direction. The time period of vibration is T1. If the
(a) Motion remains S.H. and period remains nearly magnets are placed with opposite poles together and vibrate
constant with time period T2, then [CBSE PMT 2002]
T (a) T2 is infinite (b) T2 = T1
(b) Motion remains S.H. with time period =
2 (c) T2 > T1 (d) T2 < T1
(c) Motion remains S.H. with time period = 2T
8. A diamagnetic material in a magnetic field moves
(d) Motion remains S.H. with time period = 4T
[CBSE PMT 2003]
2. The work done in turning a magnet of magnetic moment M
by an angle of 90° from the meridian, is n times the (a) perpendicular to the field
corresponding work done to turn it through an angle of (b) from stronger to the weaker parts of the field
60°. The value of n is given by [CBSE PMT 1995] (c) from weaker to the stronger parts of the field
(a) 2 (b) 1 (c) 0.5 (d) 0.25 (d) None of these
3. For protecting a sensitive equipment from the external 9. According to Curie’s law, the magnetic susceptibility of a
electric arc, it should be [CBSE PMT 1998] substance at an absolute temperature T is proportional to
(a) wrapped with insulation around it when passing [CBSE PMT 2003]
current through it
1 1
(b) placed inside an iron can (a) T 2 (b) (c) T (d)
T T2
(c) surrounded with fine copper sheet
(d) placed inside an aluminium can 10. A coil in the shape of an equilateral triangle of side l is
suspended between the pole pieces of a permanent magnet
4. If a diamagnetic substance is brought near north or south ®
pole of a bar magnet, it is [CBSE PMT 1999] such that B is in plane of the coil. If due to a current i in
(a) attracted by the poles the triangle a torque t acts on it, the side l of the triangle is
(b) repelled by the poles [CBSE PMT 2005]
(c) repelled by north pole and attracted by the south pole 1
1
(d) attracted by the north pole and repelled by the south 2 æ t ö2 æ t ö2
pole (a) ç ÷ (b) 2çç ÷
÷
3 è B.i ø è 3B.i ø
5. A bar magnet, of magnetic moment M , is placed in a
magnetic field of induction B . The torque exerted on it is 2 æ t ö 1 t
(c) ç ÷ (d)
[CBSE PMT 1999] 3 è B.i ø 3 .i
B
(a) M.B (b) – M . B (c) M ´ B (d) B´M 11. If the magnetic dipole moment of an atom of diamagnetic
material, paramagnetic material and ferromagnetic material
6. Current i is flowing in a coil of area A and number of turns are denoted by md, mp and mf respectively, then
N, then magnetic moment of the coil M is
[CBSE PMT 2005]
[CBSE PMT 2001]
(a) md = 0 and mp ¹ 0 (b) md ¹ 0 and mp = 0
Ni Ni
(a) NiA (b) (c) (d) N2Ai (c) mp = 0 and mf ¹ 0 (d) md ¹ 0 and mf ¹ 0
A A
Free eBooks on @neetquestionpaper2020

Magnetism and Matter 753


12. A vibration magnetometer placed in magnetic meridian has
a small bar magnet. The magnet executes oscillations with a
time period of 2 sec in earth's horizontal magnetic field of 24
microtesla. When a horizontal field of 18 microtesla is
produced opposite to the earth's field by placing a current
carrying wire, the new time period of magnet will be
[CBSE-PMT 2010]
(a) 1 s (b) 2 s (c) 3 s (d) 4 s
13. Two identical bar magnets are fixed with their centres at a
3 2 M
distance d apart. A stationary charge Q is placed at P in (a) M (b) M (c) (d) M
between the gap of the two magnets at a distance D from p p 2
the centre O as shown in the figure. [CBSE-PMT 2010] 18. A thin rectangular magnet suspended freely has a period of
oscillation equal to T. Now it is broken into two equal halves
P (each having half of the original (length) and one piece is
D made to oscillate freely in the same field. If its period of
T'
oscillation is T¢, the ratio is [AIEEE 2003]
T

O 1 1 1
S N N S (a) (b) (c) 2 (d)
2 2 2 4
19. A magnetic needle lying parallel to a magnetic field requiers
d W units of work to turn it through 60°. The torque needed
to maintain the needle in this position will be [AIEEE 2003]
The force on the charge Q is
(a) directed perpendicular to the plane of paper (a) (b) W (c) 3
3W W (d) 2W
(b) zero 2
(c) directed along OP 20. The magnetic lines of force inside a bar magnet
(d) directed along PO [AIEEE 2003]
14. The magnetic moment of a diamagnetic atom is (a) are from north-pole to south-pole of the magnet
(a) equal to zero [CBSE-PMT 2010] (b) do not exist
(b) much greater than one (c) depend upon the area of cross-section of the bar magnet
(c) unity (d) are from south-pole to north-pole of the magnet
(d) between zero and one 21. Curie temperature is the temperature above which
15. There are four light-weight-rod samples A,B,C,D separately
[AIEEE 2003]
suspended by threads. A bar magnet is slowly brought near
(a) a ferromagnetic material becomes paramagnetic
each sample and the following observations are noted
(i) A is feebly repelled [CBSE-PMT 2011] (b) a paramagnetic material becomes diamagnetic
(ii) B is feebly attracted (c) a ferromagnetic material becomes diamagnetic
(iii) C is strongly attracted (d) a paramagnetic material becomes ferromagnetic
(iv) D remains unaffected 22. The length of a magnet is large compared to its width and
Which one of the following is true ? breadth. The time period of its oscillation in a vibration
(a) B is of a paramagnetic material magnetometer is 2s. The magnet is cut along its length into
three equal parts and these parts are then placed on each
(b) C is of a diamagnetic material
other with their like poles together. The time period of this
(c) D is of a ferromagnetic material
combination will be [AIEEE 2004]
(d) A is of a non-magnetic material
16. A short bar magnet of magnetic moment 0.4J T–1 is placed in 2 2
(a) 2 3 s (b) s (c) 2 s (d) s
a uniform magnetic field of 0.16 T. The magnet is in stable 3 3
equilibrium when the potential energy is
[CBSE-PMT 2011M] 23. The materials suitable for making electromagnets should
have [AIEEE 2004]
(a) – 0.64 J (b) zero
(c) – 0.082 J (d) 0.064 J (a) high retentivity and low coercivity
17. A bar magnet of length ‘l’ and magnetic dipole moment ‘M’ (b) low retentivity and low coercivity
is bent in the form of an arc as shown in figure. The new (c) high retentivity and high coercivity
magnetic dipole moment will be [NEET 2013] (d) low retentivity and high coercivity
Free eBooks on @neetquestionpaper2020

754 Physi cs
24. A boat is moving due east in a region where the earth's placed on a horizontal table parallel to each other with their
magnetic field is 5.0 × 10–5 NA–1 m–1 due north and horizontal. N poles pointing towards the South. They have a common
The boat carries a vertical aerial 2 m long. If the speed of the magnetic equator and are separated by a distance of
boat is 1.50 ms–1, the magnitude of the induced emf in the 20.0 cm. The value of the resultand horizontal magnetic
wire of aerial is [AIEEE 2011] induction at the mid-point O of the line joining their centres
(a) 0.75 mV (b) 0.50 mV is close to (Horizontal component of earth’s magnetic
(c) 0.15 mV (d) 1mV induction is 3.6× 10–5 Wb/m2) [JEE Main 2013]
25. Two short bar magnets of length 1 cm each have magnetic (a) 3.6 × 10–5 Wb/m2 (b) 2.56 × 10–4 Wb/m2
moments 1.20 Am2 and 1.00 Am2 respectively. They are (c) 3.50 × 10–4 Wb/m2 (d) 5.80 × 10–4 Wb/m2

1. Two isolated point poles of strength 30 A-m and 60 A-m are 9. A bar magnet 8 cms long is placed in the magnetic merdian
placed at a distance of 0.3m. The force of repulsion is with the N-pole pointing towards geographical north. Two
netural points separated by a distance of 6 cms are obtained
(a) 2 × 10–3 N (b) 2 × 10–4 N
on the equatorial axis of the magnet. If horizontal component
(c) 2 × 105 N (d) 2 × 10–5 N of earth’s field = 3.2 × 10–5 T, then pole strength of magnet
2. The magnet of pole strength m and magnetic moment M is is
cut into two pieces along its axis. Its pole strength and (a) 5 ab-amp × c (b) 10 ab-amp × cm
magnetic moment now becomes (c) 2.5 ab-amp × cm (d) 20 ab-amp × cm
10. At a certain place, the angle of dip is 30º and the horizontal
m M M m component of earth’s magnetic field is 0.50 oerested. The
(a) , (b) m, (c) ,M (d) m, M
2 2 2 2 earth’s total magnetic field (in oerested) is
3. The magnetic moment of a magnet is 0.1 amp × m2. It is 1 1
(a) 3 (b) 1 (c) (d)
suspended in a magnetic field of intensity 3 × 10–4 weber/m2. 3 2
The couple acting upon it when deflected by 30º from the 11. Two tangent galvanometers having coils of the same radius
magnetic field is are connected in series. A current flowing in them produces
(a) 1 × 10–5 N m (b) 1.5 × 10–5 N m deflections of 60º and 45º respectively. The ratio of the number
of turns in the coils is
(c) 2 × 10–5 N m (d) 2.5 × 10–5 N m
3 +1 3 +1 3
4. At a certain place, horizontal component is 3 times the (a) 4/3 (b) (c) (d)
1 3 -1 1
vertical component. The angle of dip at this place is 12. Two identical magnetic dipoles of magnetic moments
(a) 0 (b) p/3 1.0 A-m2 each, placed at a separation of 2 m with their axis
perpendicular to each other. The resultant magnetic field at
(c) p/6 (d) None of these point midway between the dipole is
5. In end on and broadside on position of a deflection (a) 5 × 10–7 T (b) 5 × 10–7 T
magnetometer, if q1 and q2 are the deflections produced by
(c) 10–7 T (d) 2 × 10–7 T
short magnets at equal distances, then tan q1/ tanq2 is
13. The force between two short bar magnets with magnetic
(a) 2 : 1 (b) 1 : 2 moments M1 and M2 whose centres are r metres apart is 8 N
(c) 1 : 1 (d) None of these when their axes are in same line. if the separation is increased
6. If relative permeability of iron is 2000. Its absolute to 2 r, the force between them in reduced to
permeability in S.I. units is (a) 4 N (b) 2 N (c) 1 N (d) 0.5 N
(a) 8p × 10–4 (b) 8p × 10–3 14. A bar magnet of magnetic moment M and length L is cut
(c) 800/p (d) 8p × 109/p into two equal parts each of length L/2. The magnetic moment
of each part will be
7. The relative permeability of a medium is 0.075. What is its
magnetic susceptibility? (a) M (b) M/4 (c) 2M (d) M/2
(a) 0.925 (b) – 0.925 (c) 1.075 (d) –1.075 15. When 2 ampere current is passed through a tangent
8. The moment of a magnet (15 cm × 2 cm × 1 cm) is 1.2 A-m2. galvanometer, it gives a deflection of 30º. For 60º deflection,
What is its intensity of magnetisation? the current must be
(a) 4 × 104 A m–1 (b) 2 × 104 A m–1 (a) 1 amp. (b) 2 3 amp.
4
(c) 10 A m –1 (d) None of these
(c) 4 amp. (d) 6 amp.
Free eBooks on @neetquestionpaper2020

Magnetism and Matter 755


16. If horizontal and vertical components of earths magnetic 27. The variation of magnetic susceptibility (x) with temperature
field are equal, then angle of dip is for a diamagnetic substance is best represented by
(a) 60° (b) 45° (c) 30° (d) 90° (a) (b)
17. If a magnetic substance is kept in a magnetic field then
which of the following substance is thrown out?
(a) Paramagnetic (b) Ferromagnetic O T O T
(c) Diamagnetic (d) Antiferromagnetic
18. Which of the following is diamagnetism?
(a) Aluminium (b) Quartz (c) (d)
(c) Nickel (d) Bismuth
19. A frog can jump higher than normal in a magnetic field
because the tissues of a frog are
(a) paramagnetic (b) diamagnetic
(c) ferro magnetic (d) antiferro magnetic T O T
O
20. In the hysterisis cycle, the value of H needed to make the 28. The major contribution of magnetism in substances is due
intensity of magnetisation zero is called to
(a) Retentivity (b) Coercive force (a) orbital motion of electrons
(b) spin motion of electrons
(c) Lorentz force (d) None of the above
(c) equally due to orbital and spin motions of electrons
21. Isogonic lines on magnetic map represent lines of (d) hidden magnets
(a) zero declination (b) equal declination 29. Iron is ferromagnetic
(c) equal dip (d) equal horizontal field (a) at all temperatures (b) at NTP only
22. The isoclinic lines on magnetic map represent lines of (c) above 770°C (d) below 770°C
(a) zero dip (b) equal declination 30. A clinic lines on magnetic map represent the lines of
(c) equal dip (d) equal horizontal field (a) zero declination (b) equal declination
23. Magnetic field is measured by a (c) zero dip (d) equal dip
(a) thermopile (b) fluxmeter 31. Magnetic susceptibility of a diamagnetic substance
(c) pyrometer (d) thermometer (a) decreases with temperature
24. A curve between magnetic moment and temperature of (b) is not affected by temperature
magnet is (c) increases with temperature
(d) first increases then decreases with temperature
(a) M (b) M 32. Magnetic force required to demagnetize the material
(a) retainity (b) coercivity
(c) energy loss (d) hysterisis
33. At a temperatur of 30°C, the susceptibility of a ferromagnetic
material is found to be c . Its susceptibility at 333°C is
O T O T (a) c (b) 0.5 c (c) 2 c (d) 11.1 c
34. Of the following fig., the lines of magnetic induction due to
(c) M (d) M a magnet SN, are given by
N N

O T T
O
25. Magnetic lines of force due to a bar magnet do not intersect
S S
because
(1) (2)
(a) a point always has a single net magnetic field
(b) the lines have similar charges and so repel each other N N

(c) the lines always diverge from a single force


(d) the lines need magnetic lenses to be made to interest
26. For which of the following substances, the magnetic
susceptibility is independent of temperature?
(a) diamagnetics only
(b) paramagnetics only S S
(c) ferromagnetics only (3) (4)
(d) diamagnetics and paramagnetics both (a) 1 (b) 2 (c) 3 (d) 4
Free eBooks on @neetquestionpaper2020

756 Physi cs
35. The B – H curve (i) and (ii) shown in fig. associated with 43. A compass needle placed at a distance r from a short magnet
B in Tan A position shows a deflection of 60º. If the distance is
(i) increased to r (3)1/3, then deflection of compass needle is
(ii) 1
(a) 30º (b) 60´ 3 3
2 3
H (c) 60´ 3 3 (d) 60´ 3 3
44. Two short magnets have equal pole strengths but one is
twice as long as the other. The shorter magnet is placed 20
cms in tan A position from the compass needle. The longer
(a) (i) diamagnetic and (ii) paramagnetic substance
magnet must be placed on the other side of the magnetometer
(b) (i) paramagnetic and (ii) ferromagnetic substance for no deflection at a distance equal to
(c) (i) soft iron and (ii) steel respectively
(a) 20 cms (b) 20 (2)1/3 cms
(d) (i) steel and (ii) soft iron respectively
(c) 20 (2)2/3 cms (d) 20 (2)3/3 cms
36. The ratio of intensity of magnetisation and magnetising field
45. A dip needle lies initially in the magnetic meridian when it
is called
shows an angle of dip q at a place. The dip circle is rotated
(a) permeability
through an angle x in the horizontal plane and then it shows
(b) magnetic induction an angle of dip q'.
(c) magnetic intensity tan q '
(d) magnetic susceptibility Then is
tan q
37. The magnetic materials having negative magnetic
1 1 1
susceptibility are (a) (b) (c) (d) cos x
cos x sin x tan x
(a) non-magnetic (b) para magnetic
46. A dip circle is so set that its needle moves freely in the
(c) dia magnetic (d) ferro magnetic
magnetic meridian. In this position, the angle of dip is 40º.
38. A thin bar magnet of length 2 l and breadth 2 b pole strength Now the dip circle is rotated so that the plane in which the
m and magnetic moment M is divided into four equal parts
needle moves makes an angle of 30º with the magnetic
with length and breadth of each part being half of original
meridian. In this position, the needle will dip by an angle
magnet. Then the pole strength of each part is
(a) 40º (b) 30º
(a) m (b) m/2 (c) 2 m (d) m/4
(c) more than 40º (d) less than 40º
39. In the above question, magnetic moment of each part is
47. Work done in turning a magnet of magnetic moment M by
(a) M/4 (b) M (c) M/2 (d) 2 M an angle of 90º from the mgnetic meridian is n times the
40. Two points A and B are situated at a distance x and 2x corresponding work done to turn through an angle of 60º,
respectively from the nearer pole of a magnet 2 cm long. The where n is
ratio of magnetic field at A and B is
(a) 1/2 (b) 2 (c) 1/4 (d) 1
(a) 4 : 1 exactly (b) 4 : 1 approximately 48. Two magnets are held together in a vibration magnetometer
(c) 8 : 1 approximately (d) 1 : 1 approximately and are allowed to oscillate in the earth’s magnetic field with
41. If a magnet is suspended at angle 30º to the magnetic meridian, like poles together. 12 oscillations per minute are made but
the dip needle makes an angle of 45º with the horizontal. for unlike poles together only 4 oscillations per minute are
The real dip is executed. The ratio of their magnetic moments is
(a) tan -1 ( 3 / 2 ) (b) tan -1 ( 3 ) (a) 3 : 1 (b) 1 : 3 (c) 3 : 5 (d) 5 : 4
49. A superconductor exhibits perfect :
(c) tan -1 ( 3 / 2) (d) tan -1 (2 / 3 ) (a) ferrimagnetism (b) ferromagnetism
42. Two bar magnets of the same mass, same length and breadth (c) paramagnetism (d) diamagnetism
but having magnetic moments M and 2M are joined together 50. A freely suspended magnet oscillates with period T in earth’s
pole for pole and suspended by a string. The time period of horizontal magnetic field. When a bar magnet is brought
assembly in a magnetic field of strength H is 3 seconds. If near it, such that the magnetic field created by bar magnet is
now the polarity of one of the magnets is reversed and
in same direction as earth’s horizontal magnetic field, the
combination is again made to oscillate in the same field, the
T
time of oscillation is period decreases to . The ratio of the field of the magnet
2
F to the earth’s magnetic field (H) is
(a) 3 sec (b) 3 3 sec
(a) 1 : 3 (b) 1: 1 (c) 3 : 1 (d) 9 : 1
(c) 3 sec (d) 6 sec
Free eBooks on @neetquestionpaper2020

Magnetism and Matter 757


51. A watch glass containing some powdered substance is 59. The time period of oscillation of a magnet in a vibration
placed between the pole pieces of a magnet. Deep concavity magnetometer is 1.5 sec. The time period of oscillation of
is observed at the centre. The substance in the watch glass
another magnet similar in size, shape and mass but having
is
(a) iron (b) chromium 1/4 magnetic moment than that of the 1st magnet oscillating
(c) carbon (d) wood at the same place will be
52. A steel wire of length l has a magnetic moment M. It is then (a) 0.75 sec (b) 1.5 sec
bent into a semicircular arc. The new magnetic moment is (c) 3.0 sec (d) 6.0 sec
M 2M 3M 4M 60. Time periods of vibation of two bar magnets in sum and
(a) (b) (c) (d)
p p p p difference positions are 4 sec and 6 sec respectively. The
53. The magnetic moment of atomic neon is equal to ratio of their magnetic moments M1 / M2 is
1 3 (a) 6 : 4 (b) 30 : 16
(a) zero (b) µB (c) µB (d) µB
2 2 (c) 2 . 6 : 1 (d) 1 . 5 : 1
54. Torques t1 and t 2 are required for a magnetic needle to 61. Horizontal component of earth's field at a height of 1 m from
remain perpendicular to the magnetic fields at two different the surface of earth is H. Its value at a height of 10 m from
places. The magnetic fields at those places are B1 and B2 surface of earth is
B1 (a) H/10 (b) H/9
respectively; then ratio is (c) H/100 (d) H
B2
62. If a toroid uses bismuth for its core, the field in the core
t2 t1 compared to that in empty core will be slightly
(a) (b)
t1 t2 (a) greater (b) smaller
t1 + t 2 t1 - t 2 (c) equal (d) None of these
(c) (d)
t1 - t 2 t1 + t 2 63. Needles N1, N2 and N3 are made of a ferromagnetic, a
55. The net magnetic moment of two identical magnets each of paramagnetic and a diamagnetic substance respectively. A
magnetic moment M0, inclined at 60° with each other is magnet when brought close to them will
N (a) attract N1 and N2 strongly but repel N3
(a) M 0 (b) attract N1 strongly, N2 weakly and repel N3 weakly
(b) 2 M0 (c) attract N1 strongly, but repel N2 and N3 weakly
(c) 3 M0 (d) attract all three of them
60°
(d) 2M0 S N 64. Relative permittivity and permeability of a material e r and
56. A magnetic needle vibrates in a vertical plane parallel to the
magnetic meridian about a horizontal axis passing through m r , respectively. Which of the following values of these
its centre. Its frequency is n. If the plane of oscillation is quantities are allowed for a diamagnetic material?
turned about a vertical axis by 90°C, the frequency of its
oscillation in vertical plane will be (a) e r = 0.5, m r = 1.5 (b) e r = 1.5, m r = 0.5
(a) n (b) zero
(c) less than n (d) more than n (c) e r = 0.5, m r = 0.5 (d) e r = 1.5, m r = 1.5
57. A thin rectangular magnet suspended freely has a period of DIRECTIONS (Qs. 65 to 68) : Each question contains
oscillation of 4 s. If it is broken into two halves (each having
STATEMENT-1 and STATEMENT-2. Choose the correct answer
half the original length) and one of the pieces is suspended
similarly. The period of its oscillation will be (ONLY ONE option is correct ) from the following-
(a) 4 s (b) 2 s (a) Statement -1 is false, Statement-2 is true
(c) 0.5 s (d) 0.25 s (b) Statement -1 is true, Statement-2 is true; Statement -2 is a
58. A steel wire of length l has a magnetic moment M. It is bent correct explanation for Statement-1
in L-shape (Figure). The new magnetic moment is (c) Statement -1 is true, Statement-2 is true; Statement -2 is not
(a) M a correct explanation for Statement-1
M (d) Statement -1 is true, Statement-2 is false
(b) l
2 2 65. Statement-1 : The ferromagnetic substance do not obey
Curie’s law.
M
(c) l Statement-2 : At Curie point a ferromagnetic substance start
2 2 behaving as a paramagnetic substance.
(d) 2M
Free eBooks on @neetquestionpaper2020

758 Physi cs
66. Statement-1 : Magnetism is relativistic. guide way which repels the strong magnets placed in the train's
Statement-2 : When we move along with the charge so that under carriage. This helps train move over the guideway , a technic
there is no motion relative to us, we find no magnetic field called electro-dynamic suspension. When current passes in the
associated with the charge. coils of guideway , a typical magnetic field is set up between the
67. Statement-1 : A paramagnetic sample display greater undercarriage of train and guideway which pushes and pull the
magnetisation (for the same magnetic field) when cooled. train along the guideway depending on the requirement.
Statement-2 : The magnetisation does not depend on The lack of friction and its aerodynamic style allows the train to
temperature. more at very high speed.
68. Statement-1 : Electromagnetic are made of soft iron. 69. The levitation of the train is due to
Statement-2 : Coercivity of soft iron is small. (a) mechanical force
(b) electrostatic attraction
DIRECTIONS (Qs. 69 to 71) : Read the following passage carefully (c) electrostatic repulsion
and answer the questions that follows: (d) magnetic repulsion
PASSAGE 70. The disadvantage of maglev trains is that
Advanced countries are making use of powerful electromagnets (a) more friction (b) less pollution
to move trains at very high speed. These trains are called maglev
(c) less wear and tear (d) high initial cost
trains (abbreviated from magnetic levitation). These trains float
71. The force which makes maglev move
on a guideway and do not run on steel rail tracks.
(a) gravitational field (b) magnetic field
Instead of using a engine based on fossil fuels, they make use of
(c) nuclear forces (d) air drag
magnetic field forces. The magnetized coils are arranged in the
Free eBooks on @neetquestionpaper2020

Magnetism and Matter 759

8. (b) A diamagnetic liquid moves from stronger parts of


Exercise 20.1
magnetic field to weaker parts.
5. (c) For each half M = m × 2 l becomes half and volume 9. (d) On increasing the temperature by 700ºC, the magnetic
V = a × 2 l also becomes half therefore, I = M/V, remains needle is demagnetised. Therefore, the needle stops
constant. vibrating.
6. (a) A magnetic needle suspended freely orientates itself 10. (b) For ferromagnetic substances, cm is large and
in a definite direction.
positive.
7. (b) Magnetic length of a magnet is roughly 0.8 times the
11. (b) Beyond curie temperature, ferormagnetic material turns
geometric length.
into paramagnetic material, as if ferromagnetic domains
8. (a) Magnetic dipole moment is directed from south pole to
become random.
north pole of magnetic dipole.
12. (d) The direction of magnetic lines of force of a bar magnet
9. (d) End on position corresponds to axial line and Broadside
is from north to south pole outside the magnet and
on position corresponds to equatorial line. Therefore,
from south pole to north pole inside the magnet.
B1 / B2 = 2.
r r
14. (c) t = M B sin q = M ´ B
Exercise 20.2
15. (b) Diagmagnetic substances are repelled by the poles of
1. (b) Due to horizontal component of earth’s field, the a bar magnet.
magnetic lines of force are parallel and straight. 16. (b) Diagmagnetic materials are repelled by the magnetic
2. (a) At magnetic north pole of earth, H = 0 and d = 90º, field.
maximum. 17. (a) In diamagnetism, susceptibility does not depend on
3. (c) Near the magnetic poles, H = 0, therefore, magnetic temperature.
compass will not work.
18. (a) m = m r m 0 , as m r = m / m 0 .
4. (a) At magnetic equator, R = H; V = 0.
19. (a) In S.I. units, we have B = m0 (H + I)
5. (b) The restoring torque is due ot horizontal component
of earth’s field. Exercise 1 : NCERT Based Questions 
6. (d) At poles, d = 90º.
1. (a) Angle of inclination, (b) angle of declination and
Exercise 20.3 (c) horizontal component of earth’s magnetic field.
2. The angle of dip is 0° at equator and that is 90° at poles.
1. (a) Brass is non magnetic material. 3. F= Htan q
2. (b) In paramagnetic metals, magnetism is acquired by
4. m r = 1 + cm
orientation of atomic dipoles.
16. M = 4.8 × 10–2 J/T.
3. (a) For diamagnetic material, cm is low and negative.
17. 0.6 J/T, q = 0°
4. (c) The loop (i) is for soft iron and the loop (ii) is for steel.
18. 0.5 or q = 30°
5. (d) Im / H = cm . 19. (b) 20. (a)
6. (b) Paramagnetic liquid tends to flow from region of weaker 21. (b) 22. (a)
magnetic fields to stronger magnetic fields. 23. (a) 24. (d)
7. (d) A flux meter is used for measuring a magnetic field. 25. (a) 26. (d)
Free eBooks on @neetquestionpaper2020

760 Physi cs
Exercise 2 : PAST Competition MCQs 10. (b) t = MB sinq, t = iAB sin90º
A
1. (c) T µ I ; I a M Þ Ta M

T1 M1
=
T2 M 2 Þ T2 = 2T1 = 2T
D
2. (a) Magnetic moment = M; Initial angle through which B C
magnet is turned (q1) = 90º and final angle which l
magnet is turned (q2)= 60º. Work done in turning the t
\ A= = 1/2 (BC) (AD)
magnet through iB
90º(W1) = MB (cos 0º – cos 90º)= MB (1–0) = MB.
2
Similarly, W2 = MB (cos 0º – cos 60º) 1 1 ælö 3 2
But (BC)(AD) = (l ) l 2 - ç ÷ = l
2 2 è 2ø 4
æ 1 ö MB
= MBç1 - ÷ = .
è 2ø 2
3 2 t
Þ (l ) =
\ W1 = 2W2 or n = 2. 4 Bi
3. (b) The iron can produces a magnetic screening for the
1
equipment as lines of magnetic force can not enter iron æ t ö2
enclosure. \ l = 2ç ÷
è 3 B.i ø
4. (b) Diamagnetic substances do not have any unpaired
electron. and they magnetised in direction opposite to 11. (a) The magnetic dipole moment of diamagnetic material is
that of magnetic field. Hence when they are brought to zero as each of its pair of electrons have opposite spins,
North or South pole of Bar magnet, they are repelled i.e., md = 0.
by poles. Paramagnetic substances have dipole moment > 0,
5. (c) We know that when a bar magnet is placed in the i.e. mp ¹ 0, because of excess of electrons in its molecules
magnetic field at an angle q, then torque acting on the spinning in the same direction.
bar magnet Ferro-magnetic substances are very strong magnets
and they also have permanent magnetic moment,
(t) = MB sin q = M ´ B i.e. mf ¹ 0.
Note : This torque t has a tendency to make the axis of 12. (d) Time period of a vibration magnetometer,
the magnet parallel to the direction of the magnetic
1 T1 B2
field. Tµ Þ =
B T2 B1
6. (a) Magnetic moment linked with one turn = iA
Magnetic moment linked with N turn
B1
= iNA amp-m2. Þ T2 = T1
B2
Here A = Area of current loop.

K1 + K 2 K 24 ´10-6
T1 = 2p = 2p =2 = 4s
7. (c) ( M + 2 M) H 3MH 6 ´ 10-6
13. (b) Force on a charged particle is given by F = qvB. Here
K K v = 0 and also resultant B is zero.
T2 = 2p = 2p
( 2 M - M) H MH \ Force = 0
14. (a) The magnetic moment of a diamagnetic atom is equal
Obviously T2 > T1 to zero.
8. (b) A diamagnetic material in a magnetic field moves, from 15. (a) A ® diamagnetic B ® paramagnetic
stronger to the weaker parts of the field. C ® Ferromagnetic D ® Non-magnetic
1 16. (a) For stable equilibrium
9. (b) According to Curie’s law, cm µ
T U = –MB = – (0.4) (0.16) = – 0.064 J
Free eBooks on @neetquestionpaper2020

Magnetism and Matter 761


17. (a) Magnetic dipole moment 24. (c) Induced emf = vBH l = 1.5 × 5 × 10–5 × 2 = 15 × 10–5
M=m×l M' = m × r = 0.15 mV
From figure 25. (b) Given : N
BH
l M 1 = 1.20Am 2
B1
S B2 S
M 2 = 1.00Am 2
r sin 30º r sin 30º O
N N
20 r r
r 30º 30º r= cm = 0.1m
60º 2
From figure Bnet = B1 + B2 + BH S

pr 3l m 0 ( M1 + M 2 )
l= or r= Bnet = + BH
3 p 4p r3

m ´ 3l 10 -7 (1.2 + 1)
so, M' = m × r =
3
= M = + 3.6 ´ 10 -5 = 2.56 ´ 10 -4 wb/m2
p p (0.1)3
Exercise 3 : Conceptual & Applied MCQs
T1 l1
18. (b) Tµ l; = ; I µ L2 m 0 m1 m 2 30 ´ 60
T2 l2 1. (a) F= = 10 -7 ´ = 2 ´ 10 -3 N.
4p r2 (0.3) 2

é I1 L21 (2L 2 ) 2 ù 2. (a) When cut along the axis, area of cross-section
êÞ =
2
=
2
= 4ú becomes half. Therefore, pole strength is halved and
êë I 2 L 2 L2 úû
M = m (2 l), is also halved.
T1 3. (b) t = MB sin q = 0.1 × 3 × 10–4 sin 30º
Þ = 4 =2
or t = 1.5 × 10–5 N–m.
T2
V V 1
T2 1 4. (c) tan d = = =
T1 = H 3V 3
Þ T2 = Þ T1 2
2
\ d = 30º = p / 6 radian
19. (a) t = (H) tan 60º = W. 3
tan q1 2
5. (a) =
20. (d) 21. (a) tan q 2 1

T = 2p
I
= 2p
I 6. (a) m = m0 mr = (4 p´ 10-7 ) ´ 2000 = 8 p´10 -4 S.I. units
22. (b)
M´B MB
7. (b) From m r = 1 + c m ;
l
When the magnet is cut into three pieces l ' = Magnetic suscaptibility, c m = m r - 1
3
cm = 0.075 - 1 = - 0.925.
M ' l '2 Ml 2 I I
\ I' = = = \ I' tot = ´3 8. (a) Intensity of magnetisation
12 108 9 9
M 1.2
Ιm = = = 4 ´ 104 A m -1
æ l ö ml M V (15 ´ 2 ´ 1)10-6
and M ' = m ´ (l' ) = m ´ ç ÷ = = ,
è 3ø 3 3 6
9. (a) Here, 2l = 8 cm , l = 4 cm , d = = 3 cm .
2
M' tot = 3M / 3
At neutral point,
I' tot T 2 m0 M
\ T ' = 2p = = H =B=
M ' tot ´ B 3 3 4 p (d + l 2 )3 / 2
2

23. (b) Electro magnet should be amenable to magnetisation M M


and demagnetization = 10 - 7 =
-2 3 1250
(5 ´ 10 )
\ retentivity should be low and coercivity should be
low. \ M = 1250 H = 1250 ´ 3.2 ´ 10 -5 Am 2
Free eBooks on @neetquestionpaper2020

762 Physi cs
29. (d) 30. (c)
M 1250 ´ 3.2 ´ 10 -5
m= = A m. 31. (a) 32. (b)
2l 8 ´ 10 -2
m0C
33. (b) According to Curie's law, cm =
1 T
= 0.5 Am = 0.5 ´ ab amp ´ 100 cm
10 where C is Curie constant, T = temperature
= 5 ab-amp cm. 1
\ cma
H 0.50 0.50 ´ 2 T
10. (c) B= = = = 1/ 3
cos q cos 30º 3
cm1 T2 273 + 333 606
11. (d) In series, same current flows through two tangent = = = =2
c m2 T1 273 + 30 303
galvanometers.
12. (b) As the axes are perpendicular, mid point lies on axial
line of one magnet and on equatorial line of other (
\ c m2 = c m1 / 2 = 0.5c m = 0.5 c. Q c m1 = c
1 )
magnet.
34. (a) As lines of magnetic induction B are continuous curves,
they run continuously through the bar and outside, as
m 0 2 M 10-7 ´ 2 ´ 1
\ B1 = = = 2 ´ 10-7 shown in Fig. (1).
4 p d3 13
35. (c) The loop (i) is for soft iron and the loop (ii) is for steel
m0 M in Fig.
and B2 = = 10-7
4 p d3 36. (d) IH = cm
37. (c) cm is negative for diamagnetic materials.
\ Resultant field = B12 + B 22 = 5 ´ 10 -7 T 38. (b) As breadth of each part is half the original breadth,
therefore, pole strength becomes half (i.e. m/2).
1 39. (a) As length of each part also becomes half, therefore
13. (d) As F µ 4 and r becomes twice, therefore, F becomes
r magnetic moment M = pole strength × length
1 1 1 1 1
4
= times Þ ´ = th i.e. M/4.
2 16 2 2 4
40. (c) Taking distances from the centre of the magnet,
1
\ ´ 8 = 0 .5 N .
16 B1 æ x 2 ö
3 3
æ 2 x +1ö
= çç ÷÷ = ç ÷ = 8 : 1, approximately.
14. (d) As magnetic moment = pole strength x length and B 2 è x1 ø è x +1 ø
length is halved without affecting pole strength,
therefore, magnetic moment becomes half. 41. (d) Angle of dip, d = 45°

tan 45 1 2
2 i 2 tan q \ tan d ¢ = tan d = = =
15. (d) As i = tan q cos q cos 30º 3/2 3
1 1

B \ Real dip d ¢ = tan -1 (2 / 3)


16. (b) tan q = V = 1, BV = BH
BH
T22 2M + M
q = tan -1 (1) = 45° 42. (b) =
2M - M
=3 \ T2 = T1 3 = 3 3 s.
T12
17. (c) If a magnetic substance is placed in a magnetic field
then substances are repelled by magnet. This shows tan q 2 d13 r3 1
43. (c) = 3 = =
diamagnetic property. tan q1 d 2 [r (3)1/ 3 ]3 3
18. (d) Bi(Bismuth) is diamagnetic.
19. (b) 20. (b) 1 tan 60 3 1
tan q 2 = tan q1 = = = \ q 2 = 30º
3 3 3 3
21. (b) 22. (c)
23. (b) 24. (c) 44. (b) Here, d1 = 20 cm, M2 = 2 M1, d2 = ?
25. (a) 26. (a)
M 2 d 32 1/ 3
27. (b) 28. (b) = = 2 Þ d2 = 2 d1 = 20 (2)1/ 3 cm
M1 d13
Free eBooks on @neetquestionpaper2020

Magnetism and Matter 763

V V tan q¢ 1
45. (a) tan q = , tan q¢ = ; = 55. (c) Mnet = M 20 + M02 + 2M 02 cos 60 °
H H cos x tan q cos x

46. (d) d1 =40º , d 2 =30º , d =? = 3M 20 = 3M0

cot d = cot 2 d1 + cot 2 d 2 1 MB


56. (c) n=
2p I
= cot 2 40º + cot 2 30º When it is turned by an angle 90° the effective field is
vertical = V and B > V
cot d = 1.192 + 3 =2.1
So, new frequency < n.
\ d =25º i.e. d < 40º.
I
47. (b) W1 = – MB (cos 90º – cos 0º) = MB 57. (b) T = 2p
MB
W2 = – MB (cos 60º – cos 0º)
2
æ1 ö 1 1
= - MBç - 1÷ = MB = W1
mælö I
I= ç ÷ Þ I' =
è2 ø 2 2 2 è2ø 8
As W1 = n W2 \ n =2
M
M' =
60 60 2
48. (d) Here, T1 = = 5 s, T2 = = 15 s
12 4
I T
So, T ' = 2p Þ T' = = 2 sec .
M1 T22 + T12 152 + 5 2 250 5 4MB 2
= = = =
M 2 T22 - T12 15 2 - 52 200 4 58. (b) Magnetic moment, M = ml
49. (d) A superconductor exhibits perfect diamagnetism. M
= m , where m is the polestrength.
l
50. (c) I ,
T1 = 2p Therefore distance between poles
HM
l
T2 = 2p
I = ( l 2 ) 2 + ( l 2 )2 =
2
(H + F)M
ml M
So, M' = =
T12 H+F T 2
4 2 2
= = = Þ H + F = 4H
T22 H 2
T /4 1
T2 M1 M1
Þ 3H = F 59. (c) = = =2
T1 M2 1
M
51. (a) Iron is ferromagnetic. 4 1
52. (b) Let pole strength = m
\ T2 = 2T1 = 3 s
So, M = ml
When wire is in form of arc, then the distance between M1 T22 + T12 62 + 42 52
60. (c) = = = = ( 2.6 ) :1
2l M 2 T22 - T12 62 - 42 20
poles =
p
61. (d) The value of H is fairly uniform.
m 2l 2M 62. (b) Field in the core with Bismuth will be smaller because
So, M' = = bismuth is diamagnetic.
p p
63. (b) Ferromagnetic substance has magnetic domains
53. (a) Magnetic moment is cancelled and mnet = 0.
whereas paramagnetic substances have magnetic
54. (c) t = MB sin q (q = 90°) dipoles which get attracted to a magnetic field.
Diamagnetic substances do not have magnetic dipole
B1 t1
t = MB Þ = (since magnetic moment is same) but in the presence of external magnetic field due to
B2 t2 their orbital motion these substance are repelled.
Free eBooks on @neetquestionpaper2020

764 Physi cs
64. (b) For a diamagnetic material, the value of µr is less than tendency to disrupt the alignment of dipoles (due to
one. For any material, the value of Îr is always greater magnetising field) decreases on account of reduced
random thermal motion.
than 1.
68. (b) Electromagnets are magnets, which can be turnd on
65. (c) The susceptibility of ferromagnetic substan ce
and off by switching the current on and off.
decreases with the rise of temperature in a complicated
manner. After Curies point in the susceptibility of As the material in electromagnets is subjected to cyclic
ferromagnetic substance varies inversely with its changes (magnification and demangetisation), the
absolute tempearture. Ferromagnetic substance obey’s hysteresis loss of the material must be small. The
Curie’s law only above its Curie point. material should attain high value of I and B with low
value of magnetising field intensity H. As soft iron has
66. (b) A magnetic field is produced by the motion of electric
small coercivity, so it is a best choice for this purpose.
charge. Since motion is relative, the magnetic field is
also relative. 69. (d) The magnetised coils running along the track repel large
magnets on the train's under carriage.
67. (d) A paramagnetic sample display greater magnetisation
when cooled, this is because at lower temperature, the 70. (d) Initial cost will be more.
71. (b) The magnetic force will pull the vehicle.
Free eBooks on @neetquestionpaper2020

21
Electromagnetic
Induction
THE EXPERIMENTS OF FARADAY AND HENRY Current is induced in coil C1 due to motion of the current carrying
The discovery and understanding of electromagnetic induction coil C2
are based on a long series of experiments carried out by Faraday
and Henry. These experiments are illustrated by the following
figures.
When the bar magnet is pushed towards the coil, the pointer in
the galvanometer G deflects.

S. No. Experiment Observation


1. Place a magnet near a conducting No current flows through the galvanometer.
loop with a galvanometer in the
circuit.

2. Move the magnet towards the The galvanometer register a current.


loop.

3. Reverse the direction of motion The galvanometer deflection reverses.


of the magnet.
Free eBooks on @neetquestionpaper2020

766 Ph y si cs

4. Reverse the polarity of the The galvanometer deflection reverses.


magnet and move the magnet
towards the loop.

5. Keep magnet fixed and move The galvanometer register a


the coil towards the magnet. current.

6. Increases the speed of the magnet. The deflection in the galvanometer increases.
7. Increase the strength of the magnet. The deflection in the galvanometer increases.
8. Increase the diameter of the coil. The deflection in the galvanometer increases.
9. Fix the speed of the magnet but The deflection in the galvanometer increases.
repeat the experiment with the
magnet closer to the coil.
10. Move the magnet at an angle to Deflection decreases, it is maximum when the magnet moves perpendicular to the
the plane of the coil. plane of the coil and is zero when the magnet moves parallel to the plane of the coil.
11. Increase the number of turns of Magnitude of current increases.
the coil.

MAGNETIC FLUX - df
The number of magnetic lines of force crossing a surface is called i.e. e=
dt
magnetic flux linked with the surface.
The -ve sign shows that the induced emf opposes the change in
It is represented by f . magnetic flux (Lenz’s law).
B LENZ’S LAW
The direction of induced e.m.f. is given by Lenz’s law. According
to this law, the direction of induced e.m.f. in a circuit is always
q such that it opposes the every cause which produces it.
^n
-d f
Thus, e=
dt
Lenz’s law is in accordance with the principle of conservation of
ur ur energy. Infact, work done in moving the magnet w.r.t. the coil
Magnetic flux f = B. A = BA cos q changes into electric energy producing induced current.
where B is strength of magnetic field, A is area of the surface There is also another law for finding the direction of induced
and q is the angle which normal to the area (unit area vector) current. This is Fleming’s right hand rule. According to this
makes with the direction of magnetic field. rule, if we stretch the right-hand thumb and two nearby fingers
The S.I. unit of magnetic flux is weber which is the amount of perpendicular to one another such that the first finger points in
magnetic flux over an area of 1 m2 held normal to a uniform the direction of magnetic field and the thumb in the direction of
magnetic field of one tesla. motion of the conductor, then the middle finger will point in the
The c.g.s. unit of f is maxwell. direction of the induced current.
1 weber = 108 maxwell.
N
FARADAY’S LAW OF ELECTROMAGNETIC
Direction of induced
INDUCTION current inwards
Whenever the number of magnetic lines of force (flux) linked Direction of motion
Thumb of the conductor
with any closed circuit change, an induced current flows through (motion)
the circuit which lasts only so long as the change lasts. An
First finger
increase in the number of lines of force produces an inverse (field)
current, while a decrease of such lines produces a direct current. S B
Central finger
The induced emf is equal to the negative rate of change of (current) Application of Fleming’s
A right-hand rule
magnetic flux.
(a) (b)
Free eBooks on @neetquestionpaper2020

Electromagnetic Induction 767


Total flow of charge due to change of flux (Df): Case (iv) In three coils arranged coaxially : Three coils P, Q
and R are arranged coaxially as shown in figure. Equal
(No. of turns ´ change in magnetic flux)
Q = NDf / R = currents are flowing in coils P and R . Coils Q and R
Resistance are fixed. Coil P is moved towards Q. The induced
METHODS OF INDUCING E.M.F. current in Q will be in anti-clockwise direction so that
As is known, e.m.f. is induced in a circuit only when amount of it may oppose the approach of P according to Lenz’s
magnetic flux linked with the circuit changes. As f = BA cos q, law. As the face of P towards Q is a south pole hence
therefore three methods of producing induced e.m.f. : plane of Q towards P will also be a south pole.
(i) By changing B, (ii) By changing A and, (iii) By changing q Induced current
(orientation of the coil). When a conductor of length l moves P Q R Axis
with a velocity v in a magnetic field of strength B so that magnetic
Observer
flux linked with the circuit changes, the e.m.f. induced (e) is
given by coils
e = B l v. As there is no relative motion between Q and R, hence
no current is induced in Q due to R.
Induced e.m.f. and its direction
Case (v) Current increases in straight conductor : When
Case (i) In conducting rod: The induced e.m.f. is generated
current in the straight conductor is increased then
because of rotation of a conducting rod in a
perpendicular magnetic field (a) the direction of induced current in the loop will
be clockwise so that it may oppose the increase
Bl 2w of magnetic flux in the loop in downward
e =- also, e = – BAf
2 direction.
where f = frequency of rotation and P
A = pr2, where r is the radius of circle in which this
rod moves, hence r = l. w = angular velocity, l = length
of conducting rod. O
Case (ii) In disc: Induced e.m.f generated in a disc rotating
with a constant angular velocity in a perpendicular
magnetic field
A B
Br 2 w (b) the direction of induced current in the loop will
e = –BAf = -Bpr 2 f = - be anti-clockwise so that it may oppose the
2
where A = area of disc = pr2, r = radius of disc, increase of magnetic flux in the loop in upward
w = angular velocity of disc. direction.
Case (iii) In two coils: When two coils are arranged as shown A B
in the figure P
P
Q O

+ -
K
Case (vi) Magnet dropped freely in long vertical copper
(a) if key K is closed then current in P will flow in tube: The resistance of copper tube is quite negligible
clockwise direction and consequently induced and hence maximum induced current are generated
current in Q will flow in anticlockwise direction. in it due to the motion of the magnet. Due to these
(see fig. a) induced current the motion of magnet is opposed to
(b) when key K is opened then current in P falls from maximum. Consequently the acceleration of the
maximum to zero and consequently induced magnet will be zero (a = g – g = 0).
current in Q will flow in clockwise direction. (see S
fig. b)
P P
Q Q N
O g
O g
a=g–g=0

(a) (b)
Free eBooks on @neetquestionpaper2020

768 Ph y si cs
Case (vii) Magnet dropped freely into a long solenoid of
æ Blv ö B2 l 2 v
copper wire: The resistance of copper solenoid is F2 = BI l = B ç ÷ l=
much higher than that of copper tube. Hence the è R ø R
induced current in it, due to motion of magnet, will Power required to move the loop out
be much less than that in the tube. Consequently the
B2 l 2 v 2
opposition to the motion of magnet will be less and P= F2 × v =
the magnet will fall with an acceleration (a) less than R
g. (i.e. a < g). Case (xi) The magnet is stationary and the loop is moving
towards the magnet.
S

I
S N v
N
Stationary magnet
a <g
Moving loop
The induced emf or current I is shown which is in
Case (viii) Motional EMF: Induced emf in a conducting rod accordance to Lenz's law. In this case the magnetic
moving perpendicular through a uniform magnetic force causes the charge to move. We know that if a
field as shown charged particle is in motion in a field it experiences
a magnetic force. This is because when charged
× ++× × × particle moves it creates its own magnetic field which
interacts with the existing magnetic field.
×l × × B
× Case (xii) The magnet is moving towards the loop which is
stationary.
v

× ––× × ×
The induced emf produced across the rod v
I
l
r r
e = Blv = ò(v ´ B).dl S N
0

This is also called motional emf and it develops when Moving magnet
a metal rod cuts magnetic lines of force.
Special case : If the rod moves in the magnetic field making an
Stationary loop
angle q with it, then induced emf e = Bn vl = Bvl sin q .
The induced emf or current I is shown which is in
COMMON DEFAULT accordance to Lenz’s law. Here the varying magnetic field
O Incorrect. When there is no change in magnetic flux no at the location of loop (due to the movement of magnet)
induced current is produced. creates an electric field.
P Correct. Consider the case (viii) discussed above. There is We should remember certain points regarding the induced
no change in the magnetic flux throgh the rod, still induced electric field produced due to changing magnetic field.
emf is produced.
· Induced electric field lines form closed loops (different
Case (ix) A straight conductor (slider) moving with velocity v
from the electric field lines used to depict electric field
on a U shaped wire placed in a uniform magnetic field.
produced due to charges)
Blv · Induced electric field is non-conservative in nature
The induced current produced is I =
R (again a difference from the electric field produced
Case (x) When a rectangular loop perpendicular to the by electric charges)
magnetic field is pulled out, then forces F1 and F3
df
being equal and opposite cancel out. Mathematically, e = ò E .dl = - ¹0
dt
B
I
>
F1 1. An emf is induced in a circuit where the magnetic flux is
F2 l F v
3 changing even if the circuit is open. But obviously no
current will flow. If we close the circuit, the current will
start flowing.
Free eBooks on @neetquestionpaper2020

Electromagnetic Induction 769


2. In a loop moving in a uniform magnetic field, when the As resistance /length = 1W/m
loop remains in the field, the net emf induced is zero. therefore, equivalent circuit is as shown in fig.
A E B
v B
l Bl v Bl v i1 i i2
1.5V
3W 2W

Example 1.
A copper rod of length l is rotated about one end D F C
perpendicular to the uniform magnietic field B with 1 1 1 5 6
= + = ; R p = ohm
constant angular velocity w. What will be the induced e.m.f. Rp 3 2 6 5
between two ends ?
1.5
Solution : Current through EF = i = = 1.25 A
6/5
Consider a small element of the rod of length dx at a
This is total current which is divided in the inverse ratio of
distance x from the centre O.
resistances.
w 1.25 ´ 2
dx = 0.5 A
\ Current through AD or AE =
(3 + 2 )
x
l 1.25 ´ 3
O Current through BE or BC = = 0.75 A.
(3 + 2)
Example 3.
The following figure shows a conducting disc rotating about
its axis in a perpendicular magnetic field B. The resistor
Let v be the linear velocity of the element at right angles to of resistance R is connected between the centre and the
the magnetic field B. The e.m.f. developed across the rim. Calculate the current in the resistor. Does it enter the
element is d e = B v dx = B (w x) dx (Q v = w x)
disc or leave it at the centre ? The radius of the disc is 5.0
The e.m.f. across the entire rod of length l is given by cm, angular speed w = 10 radian, B = 0.40 T and R = 10W.
l R
l é x2 ù × × × × × × ×
ò ò
e = de = B w x dx = B w ê ú × × × × × ×w ×
ëê 2 ûú 0
0 × × × × × × ×
× × × × × × ×
× × × × × × ×
2 1 × ×x × × × × ×
= B w (l / 2) = B wl2 × × × × × × ×
2 × × × × × × ×
× × × × × × ×
Example 2. × × × × × × ×
A rectangular frame ABCD made of uniform metal wire Solution :
has a straight connection between E and F, made of the 1
Emf induced across circumference and centre is Bw a 2 .
same wire as shown in fig. AEFD is a square of side 1m 2
and EB = FC = 0.5 m. The entire circuit is placed in a Resistance = R.
steadily increasing uniform magnetic field directed into e 1 Bwa 2
the plane of the paper and normal to it. The rate of change \i = =
R 2 R
of magnetic field is 1 T/s. The resistance per unit length of Putting the values of B, w, a & R, i = 0.5mA
the wire is 1 W/m. Find the magnitude and direction of
According to Lenz’s law, the current is leaving the disc at
current in segments AE, BE and EF.
the centre.
A 1m E 0.5m B
Example 4.
X X X X X Fig. shows a straight, long wire carrying a current i and a
X X B X X X
rod of length l coplanar with the wire and perpendicular
to it. The rod moves with a constant velocity v in a direction
X X X X X parallel to the wire. The distance of the wire from the centre
of the rod is x. Find the motional emf induced in the rod.
D F C
Solution : v
df d A dB
Induced e.m.f. Î= = (BA ) = (because area A i
dt dt dt
is constant with respect to time) l
e.m.f. induced in AEFD = 1 × 1×1 = 1 V
e.m.f. induced in EBCF = 0.5 ×1×1 = 0.5 V
Total induced e.m.f. = 1.0 + 0.5 = 1.5 V x
Free eBooks on @neetquestionpaper2020

770 Ph y si cs
Solution : Solution :
l l The flux linked with the coil when the plane of the coil is
OA = x - ; OB = x + perpendicular to the magnetic field is
2 2
f = nAB cos q = nAB.
Change in flux on rotating the coil by 180° is
v
df = nAB – (–nAB) = 2nAB
OA B df
i \ induced charge =
l R
x 2nAB 2 ´ 100 ´ 0.001 ´ 1
= =
Take a small element of length dr at a distance r from the dt 10
wire. = 0.01 coulomb
Emf induced across dr is given by, de = Br vdr , Example 8.
whereas Br is magnetic field at a distance r from wire. Predict the direction of induced current in the situations
m i described by the following fig. (1) to (5).
Br = 0 ( Ampere' s law)
2 pr
vm i dr
\ de = 0
2p r a b N S
x +l / 2
m 0 iv dr m 0 iv æ x + l / 2 ö Fig -1
\e = ò 2p r
= ln ç
2p è x - l / 2 ø
÷
a b N S c d
x -l / 2
m 0 iv æ 2 x + l ö
\e = ln ç ÷
2p è 2x - l ø Fig.2
Example 5. b
A conductor of length 10 cm is moved parallel to itself with
a speed of 10 m/s at right angles to a uniform magnetic
induction 10–4 Wb/m². What is the induced e.m.f. in it?
Solution :
Given : l = 10 cm = 0.1 m, v = 10 m/s c a
B = 10–4 Wb/m2
e.m.f. induced in conductor Fig. 3
e = B l V = 10–4 × 0.1 × 10 = 10–4 V
Example 6.
A metal rod of length 1 m is rotated about one of its ends in
a plane right angles to a field of inductance 2.5 × 10–3 Wb/
m². If it makes 1800 revolutions/min. Calculate induced a b
e.m.f. between its ends. Fig. 4
Solution :
Given : l = 1m, B = 5 × 10–3 Wb/m2
1800
f= = 30 rotations/sec
60
In one rotation, the moving rod of the metal traces a circle
of radius r = l Decreasing at
\ Area swept in one rotation = pr2 I a stead rate
df d dA Bpr 2
= ( BA ) = B.
Fig.5
= = B f p r2
dt dt dt T Solution :
= (5 × 10–3) × 3.14 × 30 × 1 = 0.471 V Applying Lenz’s law
\ e.m.f. induced in a metal rod = 0.471 V Fig. (1) along a ® b
Example 7.
Fig. (2) along b ® a
A coil having 100 turns and area 0.001 metre2 is free to
rotate about an axis. The coil is placed perpendicular to a Fig. (3) along c ® a
magnetic field of 1.0 weber/metre2. If the coil is rotate Fig. (4) along a ® b
rapidly through an angle of 180°, how much charge will Fig. (5) no induced current since field lines lie in the plane
flow through the coil? The resistance of the coil is 10 ohm. of the loop.
Free eBooks on @neetquestionpaper2020

Electromagnetic Induction 771

21.1
Solve following problems with the help of above examples. 8. A cylindrical bar magnet is kept along the axis of a circular
1. Two identical coaxial circular loops carry a current i each coil. On rotating the magnet about its axis, the coil will
circulating in the same direction. If the loops approach have induced in it
each other, you will observe that the current in (a) a current
(a) each increases (b) no current
(b) each decreases
(c) only an e.m.f.
(c) each remains the same
(d) both an e.m.f. and a current
(d) one increases whereas that in the other decreases
2. Two coils, A and B, are lined such that emf e is induced 9. A moving conductor coil produces an induced e.m.f. This
in B when the current in A is changing at the rate I. If i is in accordance with
current is now made to flow in B, the flux linked with A (a) Lenz’s law (b) Faraday’s law
will be (c) Coulomb’s law (d) Ampere’s law
(a) ( e /I)i (b) e i I 10. A coil of insulated wire is connected to a battery. If it is
(c) ( e I)i (d) i I/e taken to galvanometer, its pointer is deflected, because
3. An induced e.m.f. is produced when a magnet is plunged (a) the induced current is produced
into a coil. The strength of the induced e.m.f. is
(b) the coil acts like a magnet
independent of
(a) the strength of the magnet (c) the number of turns in the coil of the galvanometer
(b) number of turns of coil are changed
(c) the resistivity of the wire of the coil (d) None of these
(d) speed with which the magnet is moved 11. A conducting rod AB moves parallel to X-axis in a
4. Whenever the magnetic flux linked with a coil changes, uniform magnetic field, pointing in the positive
an induced e.m.f.is produced in the circuit. The e.m.f. X-direction. The end A of the rod gets
lasts
Y
(a) for a short time
(b) for a long time
B
(c) for ever
(d) so long as the change in flux takes place
5. A magnet is moved towards a coil (i) quickly (ii) slowly, V
then the induced e.m.f. is
(a) larger in case (i) A
O X
(b) smaller in case (i)
(c) equal to both the cases (a) positively charged
(d) larger or smaller depending upon the radius of the (b) negatively charged
coil (c) neutral
6. Lenz’s law is a consequence of the law of conservation (d) first positively charged and then negatively charged
of
12. A metal ring is held horizontally and bar magnet is
(a) charge (b) mass
dropped through the ring with its length along the axis of
(c) energy (d) momentum
the ring. The acceleration of the falling magnet
7. The laws of electromagnetic induction have been used in
(a) is equal to g
the construction of a
(a) galvanometer (b) voltmeter (b) is less than g
(c) electric motor (d) generator (c) is more than g
(d) depends on the diameter of ring and length of magnet

ANSWER KEY
1. (b) 2. (a) 3. (c) 4. (d) 5. (a) 6. (c) 7. (d) 8. (b) 9. (b) 10. (a) 11. (a) 12. (b)
Free eBooks on @neetquestionpaper2020

772 Ph y si cs
EDDY CURRENTS The value of L depends on geometry of the coil and is given by
The induced circulating currents produced in a metal itself due m N2A
to change in magnetic flux linked with the metal are called eddy L= 0 .
l
currents. These currents were discovered by Foucault, so they where l is length of the coil (solenoid), N is total number of
are also known as Foucault Currents. turns of solenoid and A is area of cross section of the solenoid.
The direction of eddy currents is given by Lenz’s law. The S.I. unit of L is henry. Coefficient of self induction of a coil
Eddy currents produced in a metallic block moving in a is said to be one henry when a current change at the rate of 1
non-uniform magnetic field is shown in fig. ampere/sec. in the coil induces an e.m.f. of one volt in the coil.
Keep in Memory

1 2
1. Energy stored in a coil (inductor) = Li
2
where L is the self-inductance and i current flowing through
the inductor.
The energy stored in the magnetic field of the coil.
2
Applications of Eddy Current 1 2 1 æ B ö
E= Li = (m 0 n 2 Al) ç
Like friction, eddy currents are helpful in some fields and have 2 2 è m 0 n ÷ø
to be increased, while in some other fields they are undesirable æ B2 ö æ B2 ö
and have to be minimised. =ç ÷ Al = ç ÷ ´ volume
(1) Dead beat galvanometer. (2) Energy meter. è 2m0 ø è 2m0 ø
(3) Speedometer. (4) Electric brakes. 2. The self inductance is a measure of the coil to oppose the
(5) Single phase AC motor. (6) Induction furnace. flow of current through it. The role of self-inductance in
(7) Diathermy an electrical circuit is the same as that of the inertia in
mechanics. Therefore it is called electrical inertia.
In a moving coil galvanometer, damping is necessary 3. The magnetic energy density (energy stored per unit
to avoid oscillation of display needle. This is brought into practice B2
with the help of eddy currents. The winding of the coil of volume) in a solenoid =
2m 0
galvanometer is done on a metallic frame. When the coil rotates
the magnetic flux linked with the metallic frame changes due to Mutual Inductance
which eddy currents are developed which oppose the rotation of Mutual induction is the property of two coils by virtue of which
the coil. This is called dead beat galvanometer. each opposes any change in the strength of current flowing
SELF INDUCTANCE AND MUTUAL INDUCTANCE through the other by developing an induced e.m.f.

Self Inductance Second


First coil coil
The property of a coil by virtue of which the coil opposes any
change in the strength of the current flowing through it, by
Mutual
inducing an e.m.f. in itself is called self inductance.
Coil induction

Direction of N1 N2
I1
induced e.m.f.
(e) Coefficient of mutual inductance (M) of two coils is said to be
K
one henry, when a current change at the rate of 1 ampere/sec. in
When a current I flows through a coil, the magnetic flux f one coil induces an e.m.f. of one volt in the other coil. The value
linked with the coil is f = LI, where L is coefficient of self of M depends on geometry of two coils, distance between two
inductance of the coil. coils, relative placement of two coils etc.
On differentiating, we get The coefficient of mutual inductance of two long co-axial
solenoids, each of length l, area of across section A, wound on
df dI
=L = -e m N N A
dt dt an air core is M = 0 1 2 ] … (1)
If dI / dt = 1; L = – e. l
Hence coefficient of self inductance of a coil is equal to e.m.f. where N1 and N2 are total number of turns of the two solenoids.
induced in the coil when rate of change of current through the The mutual inductance M is defined by the equation
same coil is unity. Coefficient of self induction of a coil is also N2f2 = MI1
defined as the magnetic flux linked with a coil when 1 ampere where I1 is the current in coil 1, due to which flux f2 is linked
current flows through the same coil. with each turn of secondary coil.
Free eBooks on @neetquestionpaper2020

Electromagnetic Induction 773


Now we can calculate, e.m.f. e2 induced in secondary by a AC GENERATOR/DYNAMO/ALTERNATOR
changing current in first coil. From Faraday‘s law An electrical machine used to convert mechanical energy into
d dI electrical energy is known as AC generator/alternator or
e2 = - ( N 2 f2 ) = - M 1 dynamo.
dt dt
Principle : It works on the principle of electromagnetic induction,
dI1 i.e., when a coil is rotated in uniform magnetic field, an induced
If = 1 Þ e 2 = -M ...(2) emf is produced in it.
dt
Working :
The two definitions for M defined by equations (1) and (2) are
B C
equivalent. We can express these two equations in words as :
(i) M is numerically equal to the flux-linkage in one circuit,
when unit current flows through the other. (we use this
definition to calculate M) N A D S
(ii) M is numerically equal to the e.m.f. induced in one circuit, B1
when the current changes in the other at the rate of one R1
ampere in each second. (it is used to describe the mutual RL Output
behavior of two circuits).
For a pair of coils, M12 = M21 = m0 N1 N2 A/l, when wound on R2
B2
one another.
When the armature coil ABCD rotates in the magnetic field
Keep in Memory provided by the strong field magnet, it cuts the magnetic lines of
force. Thus the magnetic flux linked with the coil changes and
1. Coefficient of self inductance of two coils in series :
hence induced emf is set up in the coil. The direction of the
induced emf or the current in the coil is determined by the
L1 L2 Ls = L1 + L2
Fleming’s right hand rule.
The effective self inductance is Ls = L1 + L2
The current flows out through the brush B1 in one direction of
If M is the coefficient of mutual inductance between the
half of the revolution and through the brush B2 in the next half
two coils when they have flux linkage in the same
revolution in the reverse direction. This process is repeated.
sense,then L = L1 + L2 + 2M
Therefore, emf produced is of alternating nature.
L1
Ndf
e=- = NBAw sin wt = e 0 sin wt , where e0 = NBAw
dt
L1 L2 e e
L2 I = = 0 sin wt = I0 sin wt , R ® resistance of the circuit
And for flux linkage in opposite direction R R
L = L1 + L2 – 2M DC MOTOR
2. Coefficient of self inductance of two coils in parallel : A D.C. motor converts direct current energy from a battery into
L1 mechanical energy of rotation.
Lp Principle : It is based on the fact that when a coil carrying current
is held in a magnetic field, it experiences a torque, which rotates
the coil.
L2 Working :
1 1 1
= + B C C B
L p L1 L 2
(i) The coefficient of coupling between two coils having N S N S
self inductance L1 & L2 and coefficient of mutual
inductance M is A D D A
±M B1 B2 B1 B2
K=
L1 L 2 R1 R2 R2 R1

(ii) Generally the value of K is less than 1.


V V
(iii) If K is 1, then the coupling of two coils is tight while
if K < 1, then coupling is loose. The battery sends current through the armature coil in the
· Inductance is pure geometrical factor,and is direction shown in fig. Applying Fleming’s left hand rule, CD
independent of current or applied e.m.f. experiences a force directed inwards and perpendicular to the
plane of the coil. Similarly, AB experiences a force directed
· If the angle between the axis of two closely
outwards and perpendicular to the plane of the coil. These two
placed coil is q then M µ cos q .
Free eBooks on @neetquestionpaper2020

774 Ph y si cs
forces being equal, unlike and parallel form a couple. The couple Solution :
rotates the armature coil in the anticlockwise direction. After Magnetic flux at the centre of solenoid B1 = m 0 ( N1 / L)i1
the coil has rotated through 180°, the direction of the current in Magnetic flux through each turn of the coil of area A,
AB and CD is reversed, fig. Now CD experiences an outward
m Ni
force and AB experiences an inward force. The armature coil f1 = B1A = 0 1 1 ´ A
thus continues rotating in the same i.e., anticlockwise direction. L
Magnetic flux linked with th e coil of turns N 2 ,
Efficiency of the d.c. motor : Since the current I is being
m N N i A
supplied to the armature coil by the external source of e.m.f. V, f 2 = f1 ´ N 2 = 0 1 2 1
L
therefore,
Input electric power = VI According to the definition of mutual inductance f 2 = Mi1
According to Joule’s law of heating, m 0 N1 N 2 i1 m N N A
\ Mi1 = A or M = 0 1 2
Power lost in the form of heat in the coil = I2 R L L
If we assume that there is no other loss of power, then Example 11.
Power converted into external work Two pure inductors, each of self inductance L are connected
in parallel but are well separated from each other, then
i.e., Output mechanical power = VI – I2 R = (V – IR) I = EI
find the total inductance.
\ Efficiency of the d.c. motor Solution :
Output mechanical power When the coils are connected in parallel, let the currents in
h= the two coils be i1 and i2 respectively. Total induced current
Input electric power

EI E Back e.m. f . di di1 di 2


h= = = i = i1 + i 2 or = + ...(1)
or dt dt dt
VI V Applied e.m. f .
æ di ö æ di ö
Uses of D.C Motor Now e = - L1 ç 1 ÷ = - L 2 ç 2 ÷ (Q in parallel, induced
è dt ø è dt ø
1. The D.C. motors are used in D.C. fans (exhaust, ceiling or e.m.f. across each coil will be same)
table) for cooling and ventilation.
di e di e
2. They are used for pumping water. Hence 1 = - and 2 = - ...(2)
dt L1 dt L2
3. Big D.C. motors are used for running tram-cars and even
trains. Let L’ be the equivalent inductance
di di e
Example 9. \ e = – L' or =- ...(3)
dt dt L'
Two coils are wound on the same iron rod so that the flux
e e e
generated by one also passes through the other. The From eqs. (1), (2) and (3), we get - = - -
primary has 100 loops and secondary has 200 loops. When L' L1 L 2
a current of 2 A flows through the primary the flux in it is 1 1 1
25 x 10–4 Wb. Determine value of M between the coils. or = +
L' L1 L 2
Solution :
L1L 2
df di p \ L' = here L1 = L2 = L
e s = N s s and e s = M ; L1 + L 2
dt dt
L´L L
dfs di p \ L' = =
\ Ns =M or L+L 2
dt dt Example 12.
df 200 (2.5 ´10 -4 - 0) A small coil of radius r is placed at the centre of a large
M = Ns s =
di p ( 2 - 0) coil of radius R, where R >>r. The two coils are coplanar.
The mutual induction between the coils is proportional to
= 2.5×10–2 = 25 mH
Example 10. (a) r/R (b) r2/R
2
(c) r /R 2 (d) r/R2
A long solenoid of length L, cross section A having N1 turns
has wound about its centre is small coil of N2 turns as Solution : (b)
shown in fig. Then find the mutual inductance of two Let I be the current flows in the large coil.
circuits. m0I
L \ Mag. field at the centre of coil B =
2R
1 N2 N1 Mag. flux linked with smaller coil
æm Iö
f = pr 2 B = pr 2 ç 0 ÷
2 è 2R ø
Free eBooks on @neetquestionpaper2020

Electromagnetic Induction 775

f pr 2m0 Example 14.


But f = MI \ M= = or M µ r 2 / R Find the self inductance of a coil in which an e.m.f. of 10 V
I 2R
is induced when the current in the circuit changes uniformly
Example 13. from 1 A to 0.5 A in 0.2 sec.
The mutual inductance of a pair of coils is 0.75 H. If current Solution :
in the primary coil changes from 0.5 A to zero in 0.01 s find
average induced e.m.f. in secondary coil. dI 1 - 0.5 0.5
Given : e = 10 V and = = = 2.5 A / s
Solution : dt 0.2 0.2
e 10
dI 0.5 - 0 Self inductance of coil L = = =4H
Given : M = 0.75 H =
and = 50 A / s dI / dt 2.5
dt 0.01
\ Average induced e.m.f. in secondary coil, é dI ù
êQ e = L dt (Considering Magnitude only) ú
dI ë û
e=M = 0.75 ´ 50 = 37.5 V
dt

21.2
Solve following problems with the help of above text and 7. An inductor may store energy in
examples. (a) its electric field
1. Whenever, current is changed in a coil, an induced e.m.f.
(b) its coils
is produced in the same coil. This property of the coil is
due to (c) its magnetic field
(a) mutual induction (b) self induction (d) both in electric and magnetic fields
(c) eddy currents (d) hysteresis 8. If the number of turns per unit length of a coil of a solenoid
2. When current i passes through an inductor of self is doubled, the self inductance of the solenoid will
inductance L, energy stored in it is 1/2. L i2. This is stored (a) remain unchanged (b) be halved
in the (c) be doubled (d) become four times
(a) current (b) voltage 9. If N is the number of turns in a coil, the value of self
(c) magnetic field (d) electric field inductance varies as
3. A series combination of L and R is connected to a battery (a) N0 (b) N (c) N2 (d) N–2
of E.M.F., E having negligible internal resistance. The 10. Two coils of inductances L1 and L2 are linked such that
final value of current depends upon their mutual inducatance is M. Then
(a) L and R (b) E and L (a) M = L1 + L2
(c) E and R (d) L, R and E
4. Two pure inductors, each of self inductance L are 1
(b) M= (L1 + L 2 )
connected in parallel but are well separated from each 2
other, then the total inductance is (c) the maximum value of M is (L1 + L2)
(a) L (b) 2 L (c) L/2 (d) L/4
(d) the minimum value of M is L1 L 2
5. Two coils of self inductances L1 and L2 are placed so
close together that effective flux in one coil is completely 11. The SI unit of inductance, the henry can be written as
linked with the other. If M is the mutual inductance (a) weber/ampere (b) volt second/ampere
between them, then (c) joule/ampere2 (d) all of the above
(a) M = L1L2 (b) M = L1/L2 12. Two coils are placed close to each other. The mutual
inductance of the pair of coils depends upon
(c) M = (L1L2)2 (d) M = (L1 L 2 )
(a) relative position and orientation of the two coils
6. A coil of resistance R and inductance L is connected to a (b) the materials of the wires of the coils
battery of e.m.f. E volt. The final current in the coil is (c) the currents in the two coils
(a) E/R (b) E/L (d) the rates at which currents are changing in the two
(c) [ E /( R 2 + L2 )] (d) [ E L /( R 2 + L2 )] coils

ANSWER KEY
1. (b) 2. (c) 3. (c) 4. (c) 5. (d) 6. (a) 7. (c) 8. (d) 9. (c) 10. (d) 11. (d) 12. (a)
Free eBooks on @neetquestionpaper2020

776 Ph y si cs

Very Short / Short Answer Questions (ii) Why is the negative peak longer than the positive
peak?
1. How can the self-inductance of a given coil having ‘N’
number of turns, area of cross-section ‘A’ and length ‘l’ be P.d. (in mV)
increased? [Outside Delhi - 2012 COMPTT.] Magnet
2. (i) When primary coil P is moved towards secondary coil Time (in ms)
S (as shown in the figure below) the galvanometer
shows momentary deflection. What can be done to Coil
have larger deflection in the galvanometer with the V R
C
same battery?
(ii) State the related law. [Delhi Board - 2010] (a) (b)
S P
Long Answer Questions

14. Define self-inductance of a coil. Show that magnetic energy


– + required to build up the current I in a coil of self inductance
G V
3. What are eddy currents? Write their two applications. 1 2
L is given by LI . [Delhi Board - 2012 ]
[Outside Delhi - 2009] 2
4. A large circular coil, of radius R, and a small circular coil, 15. State the working of a.c. generator with the help of a
of radius r, are put in vicinity of each other. If the coefficient labelled diagram. [Outside Delhi - 2011]
of mutual induction, for this pair, equals 1 mH, what would The coil of an a.c. generator having N turns, each of area
be the flux hinked with the larger coil when a current of A, is rotated with a constant angular velocity w. Deduce
0.5 A flows through the smaller coil? the expression for the alternating e.m.f. generated in the
When the current in the smaller coil falls to zero would be coil.
it effect in the larger coil? [Delhi Board - 2008 COMPTT.] What is the source of energy generation in this device?
5. The flux, linked with a large circular coil, of radius R, is 16. State Faraday’s law of electromagnetic induction.
0.5 × 10–3 Wb when a current of 0.5 A flows through a Figure shows a rectangular conductor PQRS in which the
small neighbouring coil of radius ‘r’. Calculate the conductor PQ is free to move in a uniform magnetic field
coefficient of mutual inductance for the given pair of coils. B perpendicular to the plane of the paper. The field extends
If the current through the small coil suddenly falls to zero, from x = 0 to x = b and is zero for x > b. Assume that only
what would by its effect in the larger coil? the arm PQ possesses resistance r. When the arm PQ is
[Delhi Board - 2008 COMPTT.] pulled outward from x = 0 with constant speed v, obtain
6. Define the term self inductance. Give its unit. the expressions for the flux and the induced emf. Sketch
7. How does the self inductance of an air core coil change, the variations of these quantities with distance 0 £ x £ 2b.
when (i) the number of turns in the coil is decreased and [Outside Delhi - 2010]
(ii) an iron rod is introduced in the coil.
8. Write an expression for the energy stored in an inductor of
inductance L, when a steady current is passed through it.
S
Is the energy electric or the magnetic? P
9. What is the basic cause of induced emf?
10. State Lenz’s law.
11. Define the term mutual inductance. Write its S.I. unit. Give
Q
two factors on which the coefficient of mutual inductance
R
between a pair of coil depends.
12. Derive the formula for the self inductance of a long
x=0 x=b x = 2b
solenoid.
13. A bar magnet M is dropped so that it falls vertically through 17. Explain with the help of a diagram, the principle and
the coil C. The graph obtained for voltage produced across working of an a.c. generator. Write the expression for the
emf generated in the coil in terms of speed of rotation. Can
the coil versus time is shown in the figure.
the current produced by an a.c. generator be measured with
(i) Explain the shape of the graph. a moving coil galvanometer.
Free eBooks on @neetquestionpaper2020

Electromagnetic Induction 777


18. (a) A toroidal solenoid with an air core has an average 23. A current i = 2 sin (pt/3) amp is flowing in an inductor of 2
radius of 15 cm, area of cross section 12 cm 2 and 1200 henry. The amount of work done in increasing the current
turns. Obtain the self inductance of toroid. Ignore field from 1.0 amp to 2.0 amp is
variation across the cross-section of the toroid. (a) 1 J (b) 2 J (c) 3 J (d) 4 J
(b) A second coil of 300 turns is wound closely on the 24. An electron moves along the line PQ which lies in the same
above toroid. If the current in the primary coil is plane as a circular loop of conducting wire as shown in figure.
increased from 0 to 2 A in 0.05s, obtain the induced What will be the direction of the induced current in the loop ?
emf in the second coil. (a) Anticlockwise loop
19. The magnetic flux through a coil perpendicular to its plane (b) Clockwise
and directed into paper is varying according to the relation (c) Alternating P Q
f = (5t2 + 10t + 5) milliweber. Calculate the e.m.f induced (d) No current will be induced
in the loop at t = 5s. 25. Induced emf in the coil depends upon
(a) conductivity of coil
Multiple Choice Questions
(b) amount of flux
20. Eddy currents are produced when (c) rate of change of linked flux
(a) a metal is kept in varying magnetic field (d) resistance of coil
(b) a metal is kept in steady magnetic field 26. Two identical coaxial circular loops carry current i each
(c) a circular coil is placed in a magnetic field circulating in the clockwise direction. If the loops are
(d) through a circular coil, current is passed approaching each other, then
21. An inductor may store energy in (a) current in each loop increases
(a) its electric field (b) current in each loop remains the same
(b) its coils (c) current in each loop decreases
(c) its magnetic field (d) current in one-loop increases and in the other it decreases
(d) both in electric and magnetic fields 3
27. The current in a LR circuit builds up to th of its steady
22. If N is the number of turns in a coil, the value of self 4
inductance varies as state value in 4s. The time constant of this circuit is
(a) N0 (b) N 1 2 3 4
(a) ln 2 s (b) s (c) s (d) s
(c) N2 (d) N–2 ln 2 ln 2 ln 2

1. A conductor of length 0.4 m is moving with a speed of 5. As a result of change in the magnetic flux linked to the
7 m/s perpendicular to a magnetic field of intensity closed loop shown in the figure, an e.m.f. V volt is
0.9 Wb/m2. The induced e.m.f. across the conductor is induced in the loop.
[CBSE PMT 1995]
(a) 1.26 V (b) 2.52 V (c) 5.04 V (d) 25.2 V
2. A varying current in a coil change from 10A to zero in 0.5
sec. If the average e.m.f induced in the coil is 220V, the
self-inductance of the coil is [CBSE PMT 1995]
(a) 5 H (b) 6 H (c) 11 H (d) 12 H
3. In an inductor of self-inductance L = 2 mH, current changes The work done (in joule) in taking a charge Q coulomb
with time according to relation i = t2e–t. At what time emf once along the loop is [CBSE PMT 2005]
is zero? [CBSE PMT 2001]
(a) QV (b) 2QV (c) QV/2 (d) zero
(a) 4s (b) 3s (c) 2s (d) 1s
4. The magnetic flux through a circuit of resistance R changes 6. A conducting circular loop is placed in a uniform magnetic
by an amount Df in a time Dt. Then the total quantity of field, B = 0.025 T with its plane perpendicular to the loop.
electric charge Q that passes any point in the circuit during The radius of the loop is made to shrink at a constant rate
the time Dt is represented by [CBSE PMT 2004] of 1 mm s–1. The induced e.m.f. when the radius is 2 cm, is
[CBSE PMT 2010]
Df 1 Df
(a) Q = R . (b) Q = . (a) 2pm V (b) pm V
Dt R Dt
Df Df p
(c) Q = (d) Q = (c) mV (d) 2 mV
R Dt 2
Free eBooks on @neetquestionpaper2020

778 Ph y si cs
7. The current i in a coil varies with time as shown in the (a) zero
figure. The variation of induced emf with time would be
(b) RvB
[CBSE PMT 2011] L v
(c) vBL/R
i
(d) vBL
13. The inductance between A and D is [AIEEE 2002]
(a) 3.66 H
(b) 9 H
0 t
T/4 T/2 3T/4 T (c) 0.66 H A 3H 3H 3H D
emf emf (d) 1 H
14. Two coils are placed close to each other. The mutual
T/4 inductance of the pair of coils depends upon [AIEEE 2003]
(a) 0 t (b) 0 t
T/2 3T/4 T T/4 T/2 3T/4 T (a) the rates at which currents are changing in the two coils
(b) the relative position and orientation of the two coils
emf
emf (c) the materials of the wires of the coils
(d) the currents in the two coils
T/4 T/2 3T/4 T T/2 3T/4 T t 15. When the current changes from + 2 A to – 2A in 0.05
(c) 0 t (d) 0
second, an e.m.f. of 8 V is induced in a coil. The coefficient
T/4
of self-induction of the coil is [AIEEE 2003]
8. In a coil of resistance 10 W, the i(amp) (a) 0.2 H (b) 0.4 H (c) 0.8 H (d) 0.1 H
induced current developed by 16. A coil having n turns and resistance R W is connected with
changing magnetic flux 4 a galvanometer of resistance 4R W. This combination is
through it, is shown in figure moved in time t seconds from a magnetic field W1 weber
as a function of time. The to W2 weber. The induced current in the circuit is
magnitude of change in flux [AIEEE 2004]
t(s)
through the coil in weber is 0 0.1 n ( W2 - W1 )
( W1 - W2 )
[CBSE PMT 2012] (a) - (b) - 5 Rt
Rnt
(a) 8 (b) 2 (c) 6 (d) 4
( W2 - W1 ) n ( W2 - W1 )
9. A coil of resistance 400W is placed in a magnetic field. If (c) - (d) -
the magnetic flux f (wb) linked with the coil varies with 5 Rnt Rt
time t (sec) as f = 50t2 + 4. The current in the coil at t = 2 17. In a uniform magnetic field of induction B, a wire in the
sec is [CBSE PMT 2012] form of a semicircle of radius r rotates about the diameter
(a) 0.5 A (b) 0.1 A (c) 2 A (d) 1 A of the circle with an angular frequency w. The axis of
10. A coil of self-inductance L is connected in series with a rotation is perpendicular to the field. If the total resistance
bulb B and an AC source. Brightness of the bulb decreases of the circuit is R, the mean power generated per period of
rotation is [AIEEE 2004]
when [NEET 2013]
(a) number of turns in the coil is reduced (Bprw) 2 (Bpr 2 w) 2
(a) (b)
(b) a capacitance of reactance XC = XL is included in 2R 2R
the same circuit 2
Bpr w (Bprw2 ) 2
(c) (d)
(c) an iron rod is inserted in the coil 2R 8R
(d) frequency of the AC source is decreased 18. A metallic rod of length ‘l’ is tied to a string of length 2l
and made to rotate with angular speed w on a horizontal
11. A wire loop is rotated in a magnetic field. The frequency
table with one end of the string fixed. If there is a vertical
of change of direction of the induced e.m.f. is magnetic field ‘B’ in the region, the e.m.f. induced across
(a) twice per revolution [NEET 2013] the ends of the rod is [JEE Main 2013]
(b) four times per revolution 2 Bwl 2
(c) six times per revolution (a)
2
(d) once per revolution 3Bwl 2
12. A conducting square loop of side L and resistance R moves (b)
2
in its plane with a uniform velocity v perpendicular to one
of its sides. A magnetic induction B constant in time and 4 Bwl 2
(c)
space, pointing perpendicular and into the plane at the loop 2
exists everywhere with half the loop outside the field, as 5Bwl 2
shown in figure. The induced emf is [AIEEE 2002] (d)
2
Free eBooks on @neetquestionpaper2020

Electromagnetic Induction 779


19. A small bar magnet is being slowly inserted with constant (c) The emf induced in the loop is zero if the current
velocity inside a solenoid as shown in figure. Which graph decreases at a steady rate.
best represents the relationship between emf induced with (d) The emf induced in the loop is infinite if the current
time [IIT-JEE 2004 S] decreases at a steady rate.
Paragraph for Questions 22 and 23
A point charge Q is moving in a circular orbit of radius R in the
x-y plane with an angular velocity w. This can be considered as
Qw
equivalent to a loop carrying a steady current . A uniform
(a) emf (b) emf 2p
magnetic field along the positive z-axis is now switched on,
which increases at a constant rate from 0 to B in one second.
Assume that the radius of the orbit remains constant. The
Time Time
application of the magnetic field induces an emf in the orbit.
The induced emf is defined as the work done by an induced
electric field in moving a unit positive charge around a closed
(c) emf (d) emf loop. It is known that, for an orbiting charge, the magnetic dipole
moment is proportional to the angular momentum with a
Time Time
proportionality constant g.
20. An infinitely long cylinder is kept parallel to an uniform 22. The change in the magnetic dipole moment associated with
magnetic field B directed along positive z axis. The the orbit, at the end of the time interval of the magnetic
direction of induced current as seen from the z axis will field change, is [JEE Adv. 2013]
be [IIT-JEE 2005 S]
(a) zero BQR 2
(a) –gBQR2 (b) - g
(b) anticlockwise of the +ve z axis 2
(c) clockwise of the +ve z axis BQR 2
(d) along the magnetic field (c) g (d) g BQR 2
2
21. A current carrying infinitely long wire is kept along the 23. The magnitude of the induced electric field in the orbit at
diameter of a circular wire loop, without touching it, the any instant of time during the time interval of the magnetic
correct statement(s) is(are) [IIT-JEE 2012] field change is [JEE Adv. 2013]
(a) The emf induced in the loop is zero if the current is BR BR
constant. (a) (b)
4 2
(b) The emf induced in the loop is finite if the current is (c) B R (d) 2BR
constant.

1. A coil having an area A0 is placed in a magnetic field which The value of the magnetic flux through the area would be
changes from B0 to 4 B0 in time interval t. The e.m.f. equal to
induced in the coil will be
(a) 2.0 weber (b) 3 weber
(a) 3 A 0 B0 / t (b) 4 A 0 B0 / t
(c) 3 / 2 weber (d) 0.5 weber
(c) 3 B0 / A 0 t (d) 4A 0 / B0 t 3. In a coil of area 10 cm2 and 10 turns with magnetic field
2. Fig shown below represents an area A = 0.5 m 2 situated in directed perpendicular to the plane and is changing at the
a uniform magnetic field B = 2.0 weber/m2 and making an rate of 108 Gauss/second. The resistance of the coil is 20W.
angle of 60º with respect to magnetic field. The current in the coil will be
(a) 0.5 A (b) 5 A
(c) 50 A (d) 5 × 108 A
B 4. If a current increases from zero to one ampere in 0.1 second
60
in a coil of 5 mH, then the magnitude of the induced e.m.f.
will be
(a) 0.005 volt (b) 0.5 volt
(c) 0.05 volt (d) 5 volt
Free eBooks on @neetquestionpaper2020

780 Ph y si cs
5. A generator has an e.m.f. of 440 Volt and internal resistance 14. According to Faraday’s law of electromagnetic induction
of 4000 hm. Its terminals are connected to a load of 4000 (a) electric field is produced by time varying magnetic
ohm. The voltage across the load is flux.
(a) 220 volt (b) 440 volt (b) magnetic field is produced by time varying electric
flux.
(c) 200 volt (d) 400 volt
(c) magnetic field is associated with a moving charge.
6. When the current in a coil changes from 2 amp. to 4 amp. (d) None of these
in 0.05 sec., an e.m.f. of 8 volt is induced in the coil. The 15. A coil having 500 square loops each of side 10 cm is placed
coefficient of self inductance of the coil is normal to a magnetic field which increases at the rate of
(a) 0.1 henry (b) 0.2 henry 1 Wb/m2. The induced e.m.f. is
(c) 0.4 henry (d) 0.8 henry (a) 0.1 V (b) 5.0 V (c) 0.5 V (d) 1.0 V
7. A copper disc of radius 0.1 m rotated about its centre with 16. A circular coil and a bar magnet placed nearby are made to
10 revolutions per second in a uniform magnetic field of move in the same direction. If the coil covers a distance of
0.1 tesla with its plane perpendicular to the field. The e.m.f. 1 m in 0.5. sec and the magnet a distance of 2 m in 1 sec,
the induced e.m.f. produced in the coil is
induced across the radius of disc is
(a) zero (b) 0.5 V (c) 1 V (d) 2 V.
p 2p 17. Two solenoids of same cross-sectional area have their
(a) volt (b) volt
10 10 lengths and number of turns in ratio of 1 : 2. The ratio of
self-inductance of two solenoids is
(c) p ´10 -2 volt (d) 2p ´ 10 -2 volt
(a) 1 : 1 (b) 1 : 2 (c) 2 : 1 (d) 1 : 4
8. A 100 millihenry coil carries a current of 1 ampere. Energy 18. The back e.m.f. in a d.c. motor is maximum, when
stored in its magnetic field is (a) the motor has picked up max speed
(a) 0.5 J (b) 1 J (c) 0.05 J (d) 0.1 J (b) the motor has just started moving
9. A coil has 200 turns and area of 70 cm2. The magnetic field (c) the speed of motor is still on the increase
perpendicular to the plane of the coil is 0.3 Wb/m2 and take (d) the motor has just been switched off
0.1 sec to rotate through 180º.The value of the induced e.m.f. 19. Magnetic flux f in weber in a closed circuit of resistance
will be 10W varies with time f (sec) as f = 6t2 – 5t + 1. The
magnitude of induced current at t = 0.25s is
(a) 8.4 V (b) 84 V (c) 42 V (d)4.2 V
(a) 0.2 A (b) 0.6 A (c) 1.2 A (d) 0.8 A
10. The mutual inductance of a pair of coils, each of N turns, 20. The mutual inductance between two coils depends on
is M henry. If a current of I ampere in one of the coils is (a) medium between the coils
brought to zero in t second, the emf induced per turn in (b) separation between the two coils
the other coil, in volt, will be (c) orientation of the two coils
MI NMI MN MI (d) All of the above
(a) (b) (c) (d) 21. If coefficient of self induction of a coil is 1 H, an e.m.f. of
t t It Nt 1V is induced, if
11. A rectangular coil of single turn, having area A, rotates in (a) current flowing is 1A
a uniform magnetic field B with an angular velocity w (b) current variation rate is 1 As–1
about an axis perpendicular to the field. If initially the (c) current of 1A flows for one sec.
plane of the coil is perpendicular to the field, then the (d) None of these
average induced emf when it has rotated through 90° is 22. The current in a coil of L = 40 mH is to be increased
wBA wBA wBA 2wBA uniformly from 1A to 11A in 4 milli sec. The induced e.m.f.
(a) (b) (c) (d) will be
p 2p 4p p
(a) 100 V (b) 0.4 V (c) 440 V (d) 40 V
12. The armature of a dc motor has 20W resistance. It draws 23. The self inductance of the motor of an electric fan is 10 H.
a current of 1.5 A when run by a 220 V dc supply. The In order to impart maximum power at 50 Hz, it should be
value of the back emf induced in it is connected to a capacitance of
(a) 150 V (b) 170 V (c) 180 V (d) 190 V (a) 8 mF (b) 4 mF (c) 2 mF (d) 1 mF
13. In the figure the flux through the loop perpendicular to
the plane of the coil and directed into the paper is varying ML2
24. Which of the following units denotes the dimension ,
according to the relation f = 6t 2 + 7t + 1 where f is in Q2
milliweber and t is in second. The magnitude of the emf where Q denotes the electric charge?
(a) Wb/m2 (b) henry (H)
induced in the loop at t = 2 s and the direction of induce
Ä Ä Ä Ä Ä
(c) H/m2 (d) weber (Wb)
current through R are
Ä Ä Ä Ä Ä 25. In an AC generator, a coil with N turns, all of the same
(a) 39 mV; right to left Ä Ä Ä Ä Ä area A and total resistance R, rotates with frequency w in a
(b) 39 mV; left to right Ä Ä Ä Ä Ä magnetic field B. The maximum value of emf generated in
Ä Ä Ä Ä Ä the coil is
(c) 31 mV; right to left (a) N.A.B.R.w (b) N.A.B.
(d) 31 mV; left to right R (c) N.A.B.R. (d) N.A.B.w
Free eBooks on @neetquestionpaper2020

Electromagnetic Induction 781


26. The flux linked with a coil at any instant 't' is given by such that the total resistance of the circuit is R. When the
f = 10t2 – 50t + 250. The induced emf at t = 3s is ring is suddenly squeezed to zero area, the charge flowing
(a) –190 V (b) –10 V (c) 10 V (d) 190 V through the galvanometer is
27. A conducting square loop of side L and resistance R moves
in its plane with a uniform velocity v perpendicular to one BR AB B2 A
(a) (b) (c) ABR (d)
of its side. A magnetic induction B constant in time and A R R2
space, pointing perpendicular and into the plane of the loop 35. Consider the situation shown. The wire AB is sliding on
exists everywhere. fixed rails with a constant velocity. If the wire AB is
x x x x x x replaced by semi-circular wire, the magnitude of induced
x x x x x x x x e.m.f. will
x x x x x vx B
x x x x x Ä Ä Ä A Ä Ä
x x x x x x
Ä Ä Ä Ä Ä
The current induced in the loop is
Ä Ä vÄ Ä Ä R
Bl v Bl v
(a) clockwise (b) anticlockwise Ä Ä Ä Ä Ä
R R
Ä Ä Ä B Ä Ä
2Blv
(c) anticlockw ise (d) zero (a) increase
R
(b) decrease
28. A horizontal telegraph wire 0.5 km long running
(c) remain the same
east and west in a part of a circuit whose resistance is
(d) increase or decrease depending on whether the semi-
2.5 W. The wire falls to g = 10.0 m/s2 and B = 2 × 10–5
weber/m 2 , then the current induced in the circuit is circle buldges towards the resistance or away from it.
(a) 0.7 amp (b) 0.04 amp 36. A coil is wound on a frame of rectangular cross-section.
(c) 0.02 amp (d) 0.01 amp If all the linear dimensions of the frame are increased by a
29. The two rails of a railway track, insulated from each other factor 2 and the number of turns per unit length of the coil
and the ground, are connected to millivoltmeter. What is remains the same, self-inductance of the coil increases by
the reading of the millivoltmeter when a train passes at a a factor of
speed of 180 km/hr along the track, given that the vertical (a) 4 (b) 8 (c) 12 (d) 16
component of earth’s magnetic field is 0.2 × 10–4 wb/m2 37. A metal rod moves at a constant velocity in a direction
and rails are separated by 1 metre perpendicular to its length. A constant uniform magnetic
(a) 10–2 volt (b) 10 mV (c) 1 volt (d) 1 mV field exists in space in a direction perpendicular to the
30. A long solenoid having 200 turns per cm carries a current of rod as well its velocity. Select correct statements (s) from
1.5 amp. At the centre of it is placed a coil of 100 turns of the following.
cross-sectional area 3.14 × 10–4 m2 having its axis parallel (a) The entire rod is at the same potential
to the field produced by the solenoid. When the direction of (b) There is an electric field in the rod
current in the solenoid is reversed within 0.05 sec, the (c) The electric potential is highest at the centre
induced e.m.f. in the coil is (d) The electric potential is lowest at its centre and
(a) 0.48 V (b) 0.048 V increases towards its ends
(c) 0.0048 V (d) 48 V
38. A small square loop of wire of side l is placed inside a
31. Two coils have a mutual inductance 0.005H. The current
changes in first coil according to equation I = I0 sin wt large square loop of side L (L >> l ). The loop are coplanar
where I0 = 10A and w = 100p radian/sec. The max. value and their centres coincide. The mutual inductance of the
of e.m.f. in second coil is system is proportional is
(a) 2p (b) 5p (c) p (d) 4p
32. A metal conductor of length 1 m rotates vertically about one l l2 L L2
(a) (b) (c) (d)
of its ends at angular velocity 5 radians per second. If the L L l l
horizontal component of earth’s magnetic field is 39. A wire loop is rotated in a uniform magnetic field about
0.2 × 10–4T, then the e.m.f. developed between the two ends an axis perpendicular to the field. The direction of the
of the conductor is current induced in the loop reverses once each
(a) 5 mV (b) 50 mV (c) 5 mV (d) 50 mV (a) quarter revolution (b) half revolution
33. Two identical induction coils each of inductance L are (c) full revolution (d) two revolutions
jointed in series are placed very close to each other such 40. A square metal loop of side 10 cm and resistance 1 W is
that the winding direction of one is exactly opposite to moved with a constant velocity partly inside a uniform
that of the other, what is the net inductance? magnetic field of 2 Wbm–2, directed into the paper, as
(a) L2 (b) 2 L (c) L /2 (d) zero shown in the figure. The loop is connected to a network
34. A thin circular ring of area A is held perpendicular to a of five resistors each of value 3W. If a steady current of 1
uniform magnetic field of induction B. A small cut is made mA flows in the loop, then the speed of the loop is
in the ring and a galvanometer is connected across the ends
Free eBooks on @neetquestionpaper2020

782 Ph y si cs
the other 400 turns, their mutual inductance is
v (m0 = 4p × 10 –7 Tm A–1)
Ä Ä Ä Ä Ä (a) 2.4p × 10–5 H (b) 4.8p × 10–4 H
Ä Ä Ä Ä Ä (c) 4.8p × 10–5 H (d) 2.4p × 10–4 H
Ä Ä Ä Ä Ä DIRECTIONS for Qs. (47 to 51) : Each question contains
STATEMENT-1 and STATEMENT-2. Choose the correct
Ä Ä Ä Ä Ä
answer (ONLY ONE option is correct ) from the following.
Ä Ä Ä Ä Ä (a) Statement -1 is false, Statement-2 is true
(b) Statement -2 is true, Statement-2 is true; Statement -2 is a
(a) 0.5 cms–1 (b) 1 cms–1 correct explanation for Statement-1
(c) 2 cms–1 (d) 4 cms–1 (c) Statement -1 is true, Statement-2 is true; Statement -2 is
41. Two identical circular loops of metal wire are lying on a not a correct explanation for Statement-1
table without touching each other. Loop A carries a current (d) Statement -2 is true, Statement-2 is false
which increases with time. In response the loop B 47. Statement 1 : An induced emf appears in any coil in which
(a) remains stationary the current is changing.
(b) is attracted by loop A Statement 2 : Self induction phenomenon obeys Faraday's
(c) is repelled by loop A law of induction.
(d) rotates about is CM with CM fixed 48. Statement 1 : Lenz's law violates the principle of
42. A square loop of side a is rotating about its diagonal with
r conservation of energy.
angular velocity w in a perpendicular magnetic field B . It Statement 2 : Induced emf always opposes the change in
has 10 turns. The emf induced is magnetic flux responsible for its production.
49. Statement 1 : When number of turns in a coil is doubled,
coefficient of self-inductance of the coil becomes 4 times.
Statement 2 : This is because L µ N2.
× × × ×B
50. Statement 1 : An induced current has a direction such that
the magnetic field due to the current opposes the change
in the magnetic flux that induces the current.
Statement 2 : Above statement is in accordance with
a
conservation of energy.
(a) B a2 sin wt (b) B a2 cos wt 51. Statement 1 : The coil in the resistance boxes are made by
doubling the wire.
(c) 5 2 B a2 (d) 10 B a2 sin wt Statement 2 : Thick wire is required in resistance box.
43. In fig., final value of current in 10W resistor, when plug of DIRECTIONS for Qs. (52 to 55) : Read the following passage
key K is inserted is carefully and answer the questions that follows:
1H
3 PASSAGE
(a) A An inductor is device which works on Faraday’s law and direction
10
of emf developed is given by Lenz/s law. The mathematical
3 10 W
A di
(b) equation can be given by e = –L . (In this e is emf developed,
20 30 W dt
di
3 L is inductance and is rate of change of current). The
(c) A dt
11 3V K
(d) zero Faraday’s law and Lenz’s law mathematically can be represented
44. In a circuit given in figure 1 and 2 are ammeters. Just after df df
key K is pressed to complete the circuit, the reading by e = - (in this is rate of change of flux).
C R1 is dt dt
(a) zero in both 1 and 2 1 52. Lenz’s law is based on conservation of
(b) maximum in both 1 and 2 L R2 (a) charge (b) mass
(c) zero in 1 and maximum in 2 2 (c) energy (d) momentum
(d) maximum in 1 and zero in 2 K 53. If N is the number of turns in a coil, the value of self
– inductance varies as
45. A solenoid has 2000 turns wound over a+ length of (a) N2 (b) N–2 (c) N (d) N–3
0.3 m. Its cross-sectional area is 1.2 × 10–3 m2. Around its 54. If L and R denote inductances and resistance respectively
central section a coil of 300 turns is wound. If an initial then dimension of LR is
current of 2 A flowing in the solenoid is reversed in 0.25 s, (a) [MLTA] (b) [ML2T–2A–2]
the emf induced in the coil will be (c) [M0L0T2A0] (d) [M0L0T–2A0]
(a) 2.4 × 10–4 V (b) 2.4 × 10–2 V 55. If inductance per unit length for a solenoid near its centre
(c) 4.8 × 10–4 V (d) 4.8 × 10–2 V and near the end is denoted by LM and Le
46. Two coaxial solenoids are made by winding thin insulated (a) LM < Le (b) LM > Le
wire over a pipe of cross-sectional area A = 10 cm2 and (c) LM = Le (d) LM > 1 > Le
length = 20 cm. If one of the solenoid has 300 turns and
Free eBooks on @neetquestionpaper2020

Electromagnetic Induction 783

Here L1 = L2 = L
Exercise 21.1
L´ L L
\ L¢ = =
2. (a) Let M = mutual inductance between A and B. L+L 2
8. 2
(d) L ¥ n , where n is number of turns per unit length.
di .
e B = M A or e B = M I or M = e / I 9. (c) L = (m 0 N 2 p r 2 / 2) \ L µ N2
dt B

where I = diA/dt Exercise 1 : NCERT Based Questions


fA = Mi B = (e B / I)i B 6. The self inductance of a coil is defined as the magnetic
flux linked with the coil when unit current flows through
\ fA = (e / I ) i ( i = iB ) it. S.I. unit is henry.
Ndf df 7. (i) If number of turns (N) of one coil is decreased, self
3. (c) e = – Þ e µ N and e µ inductance (L) also decreases.
dt dt
(ii) Self inductance also increases.
If so the speed of magnet is fast then correspondingly
rate of change of flux is fast & e is maximum. It does 1 2
8. Energy stored in an inductor = LI .
not depend on the resistance of coil. 2
Ndf df 9. The basic cause of induced emf is the change in flux linked
5. (a) Since e = – if is fast, so e is large.
ge. with the circuit.
dt dt
9. (b) According to Faraday’s law, the changing magnetic flux -df
10. e =
causes induced e.m.f. dt
11. (a) According to right hand palm rule, the Lorentz force 11. Its S.I. unit is Henry. It depends on size, shape, number of
on free electrons in the conductor will be directed turns and nature of material of two coils.
towards end B. Hence, the end A gets positively charged.
12. (b) Induced e.m.f. in the ring opposes the motion of the m0 N2 A
12. L =
magnet. l
18. (a) 2.3 mH; (b) 0.023 volt.
Exercise 21.2
19. 0.06 volt.
4. (c) When the coils are connected in parallel, let the currents 20. (a) 21. (c) 22. (c) 23. (c)
in the two coils be i1 and i2 respectively. Total induced 24. (a) 25. (c) 26. (c) 27. (b)
current
Exercise 2 : PAST Competition MCQs
di di1 di 2
i = i1 + i 2 or = + …… (1)
1. (b) Length of conductor (l) = 0.4 m; Speed (v) = 7 m/s and
dt dt dt
magnetic field (B) = 0.9 Wb/ m 2 . Induced e.m.f.
æ di ö æ di ö (e) = Blv cos q = 0.9 × 0.4 × 7 × cos 0º = 2.52 V.
Now e = - L1 ç 1 ÷ = - L 2 ç 2 ÷ 2. (c) Initial current (I1) = 10 A; Final current (I2)= 0; Time
è dt ø è dt ø
(t) = 0.5 sec
(Q In parallel, induced e.m.f. across each coil will be and induced e.m.f. (e) = 220 V.
same)
dI (I - I ) (0 - 10)
di1 e di e -L = -L 2 1 = -L = 20L
Hence =- and 2 = - …… (2) dt t 0.5
dt L1 dt L2
Let L' be the equivalent inductance. 220
or L= =11 H
di di e 20
Then e = - L¢ or = - …… (3) (where L = Self inductance of coil)
dt dt L¢
3. (c) L = 2mH, i = t2e–t
From eqs. (1), (2) and (3), we get
di
e e e 1 1 1 E = -L = - L[ - t 2 e - t + 2 te - t ]
- =- - or = + dt
L¢ L1 L 2 L¢ L1 L 2
when E = 0
L1 L 2 –e–t t2 + 2te–t = 0
\ L¢ = 2t e–t = e–t t2
L1 + L 2 t = 2 sec.
Free eBooks on @neetquestionpaper2020

784 Ph y si cs

Df 13. (d) The given circuit clearly shows that the inductors are in
Df
4. (c) = e = iR Þ Df = (iDt )R = QR Þ Q = 1 1 1 1
Dt R parallel we have, = + + or L= 1H
L 3 3 3
W
5. (a) V = Q Þ W = Q V 14. (b)
dI 2 - ( -2)
6. (b) Magnetic flux linked with the loop is f = Bpr 2 15. (d) e = - L. Þ 8 = (L ) Þ L = 0.1 H
dt 0.05
df dr df (W2 - W1 )
| e |= = Bp × 2 r 16. (b) = R tot = (R + 4R)W = 5R W
dt dt dt t
dr ndj - n(W2 - W1 )
When r = 2 cm, = 1 mm s–1 i= = .
dt R tot dt 5Rt
e = 0.025× p ×2 ×2 ×10–2×10–3 (Q W2 & W1 are magnetic flux)
= 0.100 × p × 10–5 = p × 10–6 V = pmV r r
17. (b) j = B.A f = BA cos wt
di
7. (a) e = -L df wBA
dt e=- = wBA sin wt ; i = sin wt
dt R
T di
During 0 to , = const. 2
4 dt æ wBA ö
Pinst = i 2 R = ç 2
÷ ´ R sin wt
\ e = – ve è R ø
T T di T T
During to , = 0
ò ò sin
4 2 dt 2
Pinst ´ dt wtdt
\e=0 2
0 (wBA) 0
Pavg = =
T 3T di T R T
During to , = const.
2 4 dt ò dt ò dt
\ e = +ve 0 0
Thus graph given in option (a) represents the variation
(wBA) 2 æ T ö
of induced emf with time. = çè ÷.
8. (b) The charge through the coil = area of current-time R 2.T ø
(i – t) graph 2 2
(ωBπr )
1 \ Pavg =
q= ´ 0.1 ´ 4 = 0.2 C 2R
2 18. (d) Here, induced e.m.f. w l
2l
Df 3l
q= Q Change in flux (Df) = q × R dx
R e= ò (wx) Bdx x
2l
Df
q = 0.2 = [(3l) 2 – (2l)2 ] 5Bl 2 w
10 = Bw =
Df = 2 weber 2 2
9. (a) According, to Faraday’s law of induction 19. (c) Initially fB increases as magnet approaches the solenoid
\ e = –ve and increasing in magnitude. When magnet
df is moving inside the solenoid, increases in fB slows
Induced e.m.f. e = - = - (100t )
dt down and finally fB starts decreasing.
Induced current i at t = 2 sec. \ e.m.f. is positive and increasing.
Only graph (c) shows these charateristic.
e 100 ´ 2
= =+ = + 0.5Amp 20. (a) For a current to induce in the cylindrical conducting rod.
R 400 (a) The cylindrical rod should B
10. (c) By inserting iron rod in the coil, cut magnetic lines of force
L ­ z ­ I ¯ so brightness ¯ which will happen only
11. (a) This is the case of periodic EMI when the cylindrical
conducting rod is moving. –
e v=0
E
Since conducting rod is at
rest, no current will be
t induced.
(b) The magnitude / direction of the magnetic field
changes. A changing magnetic field will create
an electric field which can apply force on the
From graph, it is clear that direction is changing once free electrons of the conducting rod and a current
will get induced.
1
in cycle. But since the magnetic field is constant, no
2 current will be induced.
12. (d)
Free eBooks on @neetquestionpaper2020

Electromagnetic Induction 785


21. (a) Emf will be induced in the circular wire loop when 1 1
flux through it changes with time. 7. (c) e.m.f. induced = B R 2 w = BR 2 (2 p n)
2 2
Df I 1
e=-
Dt = ´ (0.1) ´ (0.1) 2 ´ 2 p ´ 10 = (0.1)2 p volts
2
when the current is 8. (c) Energy stored U is given by
constant, the flux changing ®
B 1 1
through it will be zero. U = L i 2 = ´ (100 ´10 -3 ) (1) 2 = 0.05 J.
When the current is ® 2 2
B 9. (a) Change in flux = 2 B A N
decreasing at a steady rate
then the change in the flux Circular 2 ´ 0.3 ´ 200 ´ 70 ´10 -4
(decreasing inwards) on loop
\ Induced e.m.f. =
0.1
the right half of the wire is equal to the change in flux
d dI NM I
(decreasing outwards) on the left half of the wire such 10. (a) E = ( NMI ) Þ E = NM Þ E =
that Df through the circular loop is zero. dt dt t
22. (b) Given M = g L E MI
\ M = g mwR2 emf induced per unit turn = =
N t
\ M = g m (Dw) R2 ...(1)
11. (d) Initially flux, f = BA cos 0 = BA
Q×B After rotating through an angle 90°.
But Dw = ...(2) Flux through the coil is zero.
2m
2 So, Df = BA
æ QB ö 2 - gBQR
From (1) and (2) DM = -g m ç ÷ R = 2p
è 2m ø 2 Angular speed = w, so, time period = =T
The negative sign shows that change is opposite to w
the direction of B. T
(b) is the correct option. is time taken to rotate 90°.
4
® ® - df d dB Δf BA 2BAω
23. (b) ò E .dl = = - (BpR 2 ) = -pR 2 So, = =
dt dt dt Δt T/4 π
2 12. (d)
= - pR B
df
\ E × 2pR = – pR2B 13. (d) f = 6t 2 + 7 t + 1 Þ = 12t + 7
dt
- BR At time, t = 2 sec.
\E=
2 df
= 24 + 7 = 31 volt
Exercise 3 : Conceptual & Applied MCQs dt
Direction of current is from left to right according to
df dB A dB Flemmings right hand rule.
1. (a) Induced e.m.f. e = = = A0 14. (a) Farady's law states that time varying magnetic flux can
dt dt dt induce an e.m.f.
æ 4 B 0 - B0 ö df d dB
= A0 ç ÷ = 3 A 0 B0 / t 15. (b) e = = ( NBA ) = NA = 500 ×10–2 × 1 = 5.0 V
è t ø dt dt dt
2.0 ´ 0.5 1
2. (d) f = BA cos q = 2.0 ´ 0.5 ´ cos 60º = = 0.5 weber. 16. (a) Vel. of coil = = 2m / s
2 0.5
df dB 2
3. (b) e = = nA velocity of magnet = = 2m / s.
dt dt 1
As they are made to move in the same direction, their
8 4 relative velocity is zero. Therefore, induced e.m.f. = 0.
\ e = 10 ´ (10 ´ 10-4 ) (104 ) (10 Gauss/sec=10 T/s)
= 100 V. m0 N 2A N2
17. (b) From L = a
Ι = (e/R) = (100/20) = 5amp. l l
L1 (1 / 2)
2
4. (c) e = (5 ´10 -3 ) (1 / 0.1) = 0.05 V . 1
we get, = =
5. (d) Total resistance of the circuit = 4000 + 400 = 4400 W L2 1/ 2 2
V 440 18. (a) The back e.m.f. in a motor is induced e.m.f., which is
Current flowing i = = = 0.1 amp. maximum, when speed of rotation of the coil is
R 4400 maximum.
Voltage across load = R i = 4000 × 0.1 = 400 volt.

6.
di
(b) e = M or 8 = M ê
é ( 4 - 2) ù
ú
19. (a) e =
- df -d 2
dt
=
dt
( )
6t - 5t + 1 = -12t + 5
dt ë 0.05 û e = – 12 (0.25) + 5 = 2 volt
8 ´ 0.05 e 2
\ M= = 0.2 henry i= = = 0.2A.
2 R 10
Free eBooks on @neetquestionpaper2020

786 Ph y si cs
20. (d) Mutual inductance between two coils depends on all the three 33. (d) When two inductance coil are joined in series, such that the
factors given here. winding of one is exactly opposite to each other the emf produced
e 1 in the two coils are out of phase such that they cancel out.
21. (b) = = 1As -1
From e = LdI / dt, dI / dt =
L 1 - df
34. (b) The individual emf produced in the coil e =
-3
LdI 40 ´10 (11 - 1) dt
22. (a) e = = = 100V |e| 1 df
dt 4 ´10-3 \ The current induced will be i = Þi=
R R dt
23. (d) For maximum power, X L = X C , which yields
1 1 dq dq 1 df 1 BA
C= = But i = Þ = Þ ò dq = ò df Þ q =
2 2 dt dt R dt R R
( 2pn ) L 4 p ´ 50 ´ 50 ´ 10 35. (c) E.m.f. will remain same because change in area per unit time
\ C = 0.1´ 10 -5 F = 1mF will be same in both cases.
2 2
f BA 36. (b) Self inductance = m 0 n AL = m 0 n (l ´ b ) ´ L
24. (b) Mutual inductance = =
I I n = Total number of turns/length
-1 -1 2 L = Length of inductor
[MT Q L ]
[Henry] = = ML2 Q - 2 l = Length of rectangular cross section
-1 b = breadth of rectangular cross-section
[QT ]
ur ur So, when all linear dimensions are increased by a factor of 2.
df d(NB.A) The new self inductance becomes L' = 8L.
25. (d) e = - =-
37. (b) Due to shifting of electrons, one end of the rod becomes
dt dt
positive and the other end negative. This developes a electric
d
= - N (BA cos wt ) = NBAw sin wt field in the rod.
dt 38. (b)
Þ e max = NBAw 39. (b) It is because after every 1/2 revolution the current becomes
zero and mode of change in flux changes thereafter (If before
26. (b) f = 10t 2 - 50t + 250 the current becomes zero, the mode of flux change was from
left to right then after the current becomes zero the mode of
df flux change becomes right to left).
e=- = - (20 t - 50)
dt
40. (c) e = Blv = 2 ´ 10-1 ´ v = 0.2 v
e t =3 = -10 V
e 0.2v
27. (d) Since the magnetic field is uniform the flux f through the I= = 10-3 Þ = 10-3
square loop at any time t is constant, because R 4
f = B × A = B × L2 = constant
6´6
df [Since effective resistance R of bridge is R = = 3W
\ e=- = zero 6+6
dt so total resistance = 1 + 3 = 4W]
e 1 df Þ v = 2 cm s–1
28. (c) i = = 41. (c) An opposite current induced in B in accordance to Lenz's
R R dt law. So the two loops repel each other.
Here df = B × A = (2 ´ 10-5 ) ´ (0.5 ´ 10+3 ´ 5) 42. (d) f = n BA cos q = 10 B a2 cos wt
dt = time taken by the wire to fall at ground
= (2 h / g) 1/ 2
= (10 /10) 1/ 2
= 1sec.
e=-
df
dt
=-
d
dt
( )
10 B a 2 cos wt = 10 B a 2 sin wt ( w ) .
43. (d) As resistance of 1 H coil is zero, the entire current flows
1 é (2 ´ 10 -5 ) ´ (0.5 ´ 103 ´ 5) ù through the coil. Current through 10W resistance is zero.
\ i= ê ú = 0.02 amp. 44. (c) Capacitor is a dc blocking element and hence no current flow
2.5 êë 1 úû in (1).
29. (d) ε=B l v = (0.2 ´10 -4 ) (1) (180 ´ 5 / 18) = 10 -3 V = 1 mV An inductor offers a zero resistance path to flow of dc and
hence maximum current flows through (2).
30. (b) B = m 0 n i = ( 4 p ´ 10 -7 ) ( 200 ´ 10 -2 ) ´ 1.5 N 2000 20000
45. (b) n= = =
= 3.8 × 10–2 Wb / m2 l 0.3 3
Magnetic flux through each turn of the coil
d dB
f = BA = (3.8 × 10–2) (3.14 × 10–4) = 1.2 × 10–5 weber x= ( NBA ) = NA
When the current in the solenoid is reversed, the dt dt
change in magnetic flux Since B = µ0nI
= 2 ´ (1.2 ´ 10-5 ) = 2.4 ´10 -5 weber Þ x = ( mNAn )
dt
Þ x = 0.024V
dt
df 2.4 ´ 10 -5
Induced e.m.f. = N = 100 ´ = 0.048 V.
dt 0.05 m N N A 4p´ 10-7 ´ 300 ´ 400 ´ 100 ´ 10-4
46. (d) M= 0 1 2 =
M l 0.2
31. (b) ε= dI=0.005×I 0 cos ωt×ω
dt = 2.4p ´ 10 -4 H
and e max = 0.005 × I0 × w = 5 p 47. (b)
48. (a) Lenz's law (that the direction of induced emf is always such
32. (b) l = 1m, w = 5 rad/s, B = 0.2 ´ 10 -4 T as to oppose the change that cause it) is direct consequence
-4 of the law of conservation of energy.
Bωl 0.2 ´ 10 ´ 5 ´1 49. (b) 50. (b) 51. (d) 52. (c)
e= = = 50mV
2 2 53. (a) 54. (b) 55. (b)
Free eBooks on @neetquestionpaper2020

22
Alternating Current

ALTERNATING AND DIRECT CURRENT AVERAGE AND RMS VALUE OF ALTERNATING


An alternating current (A.C.) is one which periodically changes CURRENT
in magnitude and direction. It increases from zero to a maximum The average value of AC over one full cycle is zero since there
value, then decreases to zero and reverses in direction, increases are equal positive and negative half cycles.
to a maximum in this direction and then decreases to zero. The average current for half cycle is 2I0 /p where I0 is the peak
The source of alternating emf may be a dynamo or an electronic value of current.
oscillator. The root mean square (rms) value of AC is
The alternating emf E at any instant may be expressed as
E = E0 sinwt I0
where w is the angular frequency of alternating emf and E0 is I rms =
2
the peak value of emf.
where I0 is the peak or maximum value of alternating current.
I I0 The rms value of alternating current can also be defined as the
(A.C.) direct current which produces the same heating effect in a given
resistor in a given time as is produced by the given A.C. flowing
through same resistor for the same time. Due to this reason the
T=0 T T 3T t rms value of current is also known as effective or virtual value
— –—
2 2 of current.

Direct current (D.C.) is that current which may or may not I0


\ Ieffective = I virtual = Irms =
change in magnitude but it does not change its direction. 2
Similarly the rms value of alternating voltgae is called the effective
or virtual value of alternating voltage (or emf).
I
(D.C.) E0
\ E effective = E virtual = Erms =
2
t Keep in Memory
Advantages of A.C. over D.C. (1) Time period : The time taken by A.C. to go through one
(i) The generation of A.C. is cheaper than that of D.C. 2p
cycle of changes is called its period. It is given as T =
(ii) Alternating voltage can be easily stepped up or stepped w
down by using a transformer. (2) Phase : It is that property of wave motion which tells us
(iii) A.C. can be easily converted into D.C. by rectifier. D.C. is the position of the particle at any instant as well as its
converted to A.C. by an inverter. direction of motion. It is measured either by the angle
(iv) A.C. can be transmitted to a long distance without which the particle makes with the mean position or by
appreciable loss. fraction of time period.
Free eBooks on @neetquestionpaper2020

788 Ph y si cs
(3) Phase angle : Angle associated with the wave motion (sine Then current I in the circuit is
or cosine) is called phase angle. E E 0 sin wt
(4) Lead : Out of the current and emf the one having greater I= = = I0 sin wt , where I0 = E0 / R
R R
phase angle will lead the other e.g., in equation
Comparing this with standard equation, we get that
æ pö
i = i0 sin ç wt + ÷ and e = e0 sin wt, impedance of circuit, Z = R and phase difference between
è 2ø
current & emf = 0.
p
the current leads the emf by an angle . Hence we conclude that in a purely resistive ac circuit the current
2 and voltage are in same phase and impedance of circuit is equal
(5) Lag : Out of current and emf the one having smaller phase
to the ohmic resistance.
angle will lag the other. In the above equations, the emf
Phasor diagram : X
p I E
lags the current by .
2 Graph of emf or current versus wt :
RESISTANCE OFFERED BY VARIOUS ELEMENTS
(INDUCTOR, RESISTOR AND CAPACITOR) TO A.C.
Alternating current in a circuit may be controlled by resistance, E
inductance and capacitance, while the direct current is controlled or E
only by resistance. I I
(i) Impedance (Z) : In alternating current circuit, the ratio
of emf applied and consequent current produced is called wt
the impedance and is denoted by Z,
E E Circuit Contianing only Inductor (L)
i.e., Z = E = 0 = rms
I I0 I rms Consider a pure inductor (zero ohmic resistance) of inductance
Physically impedance of ac circuit is the hindrance offered L connected to an alternating source of emf E = E0 sin wt.
by resistance along with either inductance or capacitance or L
both in the circuit to the flow of ac through it. Its unit is ohm.
(ii) Reactance (X) : The hindrance offered by inductance or
capacitance or both to the flow of ac in an ac circuit is
called reactance and is denoted by X. Thus when there is E = E0 sin w t
no ohmic resitance in the cirucit, the reactance is equal to
Then current I in the circuit is
impedance. The reactance due to inductance alone is called
inductive reactance and is denoted by X L, while the æ pö E0
I = I 0 sin ç wt - ÷ where I 0 =
reactance due to capacitance alone is called the capacitive è 2ø wL
reactance and is denoted by XC. Its unit is also ohm. Comparing this with standard equation, we get
(iii) Admittance (Y) : The inverse of impedance is called the Z = w L and phase difference f = p/2.
1 Hence we conclude that in a purely inductive circuit the current
admittance and is denoted by Y, i.e., Y =
Z lags behind the applied voltage by an angle p/2 and the
Its SI unit is ohm–1. impedance to the circuit is wL and this is called as inductive
IMPEDANCES AND PHASES OF AC CIRCUIT reactance.
CONTAINING DIFFERENT ELEMENTS Graph of emf or current versus wt
As already pointed out that in an ac circuit the current and applied
emfs are not necessarily in same phase. The applied emf (E) and E
current produced (I) may be expresed as E I
E = E0 sin wt and I = I0 sin (wt + f) with I0 = E0 / Z or
where E0 and I 0 are peak values of alternating emf and current. I wt
Circuit Containing only Resistor (R)
Consider a pure ohmic resistor (zero inductance) of resistance R Phasor diagram Graph between XL and f
connected to an alternating source of emf E = E0 sinwt. O E
R X XL
90º

I
f
E = E0 sin w t Y
Free eBooks on @neetquestionpaper2020

Alternating Current 789


Circuit Containing only Capacitor Phasor diagram
Consider a capacitor of capacitance C connected to an alternating E
source of emf, E = E0 sin wt. E= V 2R + V 2L
C VL

VC f
I
VR
From phasor diagrom, resultant voltage is given by,
E = E0 sin w t
Then the current through capacitor is given by, 2 2 2 2
E= (VR + VL ) = (RI ) + (wL.I )

pö Graph of emf or current versus wt


æ
I = I0 sin ç wt + ÷ emf
è 2ø Current
Comparing this with standard equation, we find that capacitive E
or
reactance XC = 1/wC and phase difference f = + p/2 I
Phasor diagram Graph between XC and f wt
IC
p/2 XC E
\ = é R 2 + (wL )2 ù
I ë û
\ Impedance of R – L circuit,
E
VC f Z= = ( R 2 + X L2 ) where XL = wL
Hence we conclude that in a purely capacitive circuit the current I
It is obvious that the current lags behind the emf by angle f
leads the applied emf by an angle p/2 and the impedance of the
given by,
circuit is 1/ wC and this is known as capacitive reactance

Z = XC =
1
wC
. f = tan
–1 æ VL ö
è VR ø
= tan –1 ( )
XL
R
–1 æ wL ö
= tan ç ÷
è Rø

Graph of emf or current versus wt Circuit Containing Resistance and Capacitance in


Series (C–R Series Circuit)
Consider a circuit containing resistance R and capacitance C in
E E series having an alternating emf E = E0 sin wt.
I
or R C
I 3p
p/2 2p wt VR VC

Circuit Containing Resistance and Inductance in


Series (LR Series Circuit)
E = E0 sin w t
Consider a circuit containing resistance R and inductance L in
series having an alternating emf E = E0 sin wt. Let I be the current flowing in the circuit, VR the potential
R L difference across resistance and VC the potential difference across
capacitance.
Phasor diagram
VR VL
VR
I
f
E = E0 sin w t
Let I be the current flowing in the circuit and VR (= IR) the
potential difference across resistance and VL (= wL.I) the
E
potential difference across inductance. VC
The current I and the potential difference VR are always in phase
but the potential difference VL across inductance leads the current From phasor diagram the resultant emf is given by
I by an anlgle p/2. E = ( VR2 + VC2 ) = ( RI ) 2 + ( X C I ) 2 ]
Free eBooks on @neetquestionpaper2020

790 Ph y si cs
At certain frequency the impedance of the circuit is minimum
\ Impedance, Z = E / I = ( R 2 + X 2C ) , where
and the current is maximum.
æ 1 ö This frequency is called the resonant frequency.
XC = ç ÷
è wC ø Circuit Containing Resistance, Inductance and
The potential difference VR and current I are in same phase and Capacitance in Series (Series LCR Circuit)
the potential difference VC lags behind the current I (and hence Consider a circuit containing a resistance R, inductance L and
VR) by angle p/2 capacitance C in series having an alternating emf
The current leads the applied emf by an angle f given by
E = E0 sin wt.
V X I X
tan j = C = C = C
VR RI R R L C
æX ö æ 1/ wC ö æ 1 ö
or tan f = ç C ÷ Þ f = tan -1 ç = tan -1 ç VC
è R ÷ø è wCR ÷ø VR VL
è R ø
Graph of emf or current versus wt
emf
E = E0 sin w t
E
or Let I be the current flowing in circuit. VR, VL and VC are
I respective potential differences across resistance R, inductance
wt L and capacitance C.
Current Phasor diagram :
Circuit Containing Inductance and Capacitance in
VL
Series (Series LC Circuit)
Consider a circuit containing inductance L and capacitance C in VR
f I
series having an alternating emf E VC – VL (if VC > VL)
VC
E = E0 sin wt.
L C

VL VC
The p.d. VR is in phase with current I . The p.d. VC lags behind
the current by angle p/2. The p.d. VL leads the current by angle
p/2.
E = E0 sin w t
\ Resultant applied emf, E = [VR2 + ( VC - VL ) 2 ]
Let I be the current flowing in circuit, VL the potential difference
across inductance L and VC the p.d. across capacitance C.
Phasor diagram : i.e., E= { (R I) 2
+ (I X C - I X L ) 2 }
Y O X
{ }
VL
E
VL > VC VC – VL
p/2
\ Impedance, Z = = R2 + ( X C - X L ) 2
E E=VC–VL I I
p/2
The phase leads of current over applied emf is given by
VL – VC VC > VL
O X Y' VC VC - VL I X C - I X L X C - X L
The p.d. VC lags behind the current by angle p/2 and the p.d. VL tan j = = =
VR RI R
leads the current by angle p/2.
\ Resultant applied emf, E = VC – VL = XCI – XLI æ X - XL ö
i.e., j = tan -1 ç C ÷ø
\ Reactance of circuit, è R
æ 1 ö It is concluded that :
X = E / I = XC - X L = ç - wL÷
è wC ø (a) If XC > XL, the value of f is positive, i.e., current leads the
The current leads applied emf by f = p / 2 . applied emf.
1 1 (b) If XC < XL, the value of f is negative, i.e., current lags
In case of XC = XL, Z = 0, then = wL or w = behind the applied emf.
wC ( LC )
(c) If XC = XL, the value of f is zero, i.e., current and emf are
1 in same phase. This is called the case of resonance and
\ Frequency f = w / 2p ==
2p ( LC ) resonant frequency for condition XC = XL, is given by :
Free eBooks on @neetquestionpaper2020

Alternating Current 791


PARALLEL RESONANT CIRCUIT
Z
E/R A parallel resonant circuit consists of an inductance L and a
Current capacitance C in parallel as shown in fig.
E R L
2R
C
R
f
fo O fo
Frequency
1 1 E=E0 sin w t
= wL i.e., w = The condition of resonance is again that the current and applied
wC LC
emf must be in same phase. The condition gives angular resonant
1 frequency.
\ fo = w / 2p = 2p ( LC ) .
1 R2
Thus the resonant frequency depends on the product of L wr = -
and C and is independent of R. LC L2
At resonance, impedance is minimum, Zmin = R and current
wr 1 1 R2
E E \ Resonant frequency f r = = -
is maximum I max = = 2 p 2p LC L2
Z min R
Z R 2 + w 2 L2 L
The impedance at resonance, Z = =
R RC
In parallel resonant circuit the impedance is maximum and the
current is minimum.
Zmin = R 1
R If R ® 0 , then f r = and Z ® ¥ .
2p (LC)
w Q - FACTOR
w =w 0=2p f0
Resonance frequency The sharpness of tuning at resonance is measured by
Circuit impedence in series RLC circuit Q-factor or quality factor of the circuit and is given by
Rapid fall of current 1 L
in A in comparison Q=
to B curve R C
I Higher the value of Q-factor, sharper is the resonance i.e. more
Imax
Small R higher rapid is the fall of current from maximum value (I0) with slight
A Q i.e., sharper resonance change in frequency from the resonance value.
B It is clear from the figure that at low value of q, the resonance is
High R, small Q i.e., poor. However the bandwidth increases
no sharp resonance
I High Q
w
w =w0=2pf0
Resonance frequency
Current amplitude in series RLC circuit
It is interesting to note that before resonance the current Low Q
leads the applied emf, at resonance it is in phase, and after
resonance it lags behind the emf. LCR series circuit is also
called as acceptor circuit and parallel LCR circuit is called w0 w
rejector circuit. The figure given below explains the concept of bandwidth and
cut-off frequency.
COMMON DEFAULT
Imax
O Incorrect. Adding impedances / reactances /resistors
algebrically.
0.707 Imax
P Correct. For these physical quantities, vector additon must
be done Lower cut off
Band width 0.707 Imax
Upper cut -off frequency
O Incorrect. Kirchoff's laws are applicable in D.C. circuit
only
frequency

P Correct. Kirchoff's laws are applicable in A.C. circuit also


(which may include inductor and capacitor). w0 w
Free eBooks on @neetquestionpaper2020

792 Ph y si cs
It is clear from the figure that for smaller R, value of Q0 is high
when w < w0 XC < XL
(Q0 is Quality factor of circuit) & hence sharper resonance i.e.
when w > w0 XL > XC greater rate of fall of average power maximum average power
when w = w0 XC = X L Pav changes with slight change in frequency from resonant
Bandwidth : It is the band of allowed frequencies and is defined frequency.
as the difference between upper and lower cut-off frequencies, w0
the frequency at which power becomes half of maximum value The Quality factor, Q0 is defined as, Q0 =
Dw
and current becomes Imax / 2 . Where Dw = w2 – w1 and w2 & w1 are half power points.
POWER IN AN A.C. CIRCUIT w0 L
R
The power is defined as the rate at which work is being done in Now, since Dw » ; so Q 0 »
L R
the circuit. In ac circuit, the current and emf are not necessarily
in the same phase, therefore we write wo w
E = E0 sin wt & I = I0 sin (wt + f). Whereas w1 = w0 - ; w2 = w0 + 0
Q0 Q0
The instaneous power, P = EI
= E0 sin wt I0 sin (wt + f), æ 1 ö
The average power Pav = Erms Irms cos f In concise term, we can write as, w = w r çç1 ± ÷
÷
è Q0 ø
E0 I 0
\ Pav = cos f Keep in Memory
2 2
In this expression cos f is known as power factor. The value of 1. Unless mentioned otherwise, all a.c. currents and voltages
cos f depends on the nature of the circuit. For L, C and are r.m.s. values.
L-C circuit, the power factor is zero ( Q f = 90º); for R-circuit 2. For resonance to occur, the presence of both L and C
cos f = 1 (Q f = 0) and for all other circuit cos f = R/Z, where elements in the circuit is a must.
Z = impedance.
3. In series resonant circuit, current is maximum at
If R = 0, cos f = 0 and Pav = 0 i.e., in a circuit with no resistance,
resonance. In a parallel resonant circuit, current is
the power loss is zero. Such a circuit is called the wattless circuit
minimum (or zero) at resonance but p.d across the
and the current flowing is called the wattless current.
combination is maximum.
Power is of two types
4. To depict oscillatory motion mathematically we may use
(i) Reactive power Preactive = Vrms Irms sin f sines, cosines or their linear combination. This is because
This is also called wattless power. changing the zero position transforms one into another.
It is not read by energy meter 5. While adding voltage across different elements in an a.c.
circuit we should take care of their phases.
(ii) Active power Pactive = Vrms I rms cos f
6. The average current over a complete cycle in an a.c circuit
It is read by energy meter
is zero but the average power is not zero.
Half Power Points 7. An inductor offers negligibly low resistance path to d.c.
The values of w at which the average power is half of its and a resistive path for a.c.
maximum value (at resonant frequency) are called half power 8. A capacitor acts as a block for d.c and a low resistance
points. path to a.c.

9. Inductive reactance Capacitive reactance


Pav
X L = wL = 2pfL 1 1
XC = =
Small R Þ XL µ f wC 2pfC
Dw higher Q 1
Þ XC µ
f
Large R, Current through pure Current through pure
low Q inductor lags behind capacitor leads
w emf by 90° emf by 90°
w 1 w 0= w r w 2
For d.c f = 0 \ X L = 0 For d.c f = 0 Þ X C = ¥
Plot of average power versus frequency for a series RLC circuit. For a.c as f increases For a.c as f increases
The upper curve is for a small R & lower broad curve is for large X L increases XC decreases
value of R.
Free eBooks on @neetquestionpaper2020

Alternating Current 793

10. Series Resonant circuit Parallel resosant circuit dq


= wq 0 cos wt or I 0 = w q 0 (maximum current)
1 1 dt
XL = XC = 1 1
X L XC w= = = (109 )1/ 2
LC -9
10
1 1 1 R2
nr = nr = - I o = (109 )1/2 ´ 10-6 = 1000 mA
2p LC 2p LC L2
2 = 3.16 ´ 10-2 A = 31.62mA
2 2 1 1 æ 1 ö
Z = R + (X L - X C ) = + Cw -
2 ç ÷ Example 4.
Z R è Lwø
When 100 volt D.C. is applied across a solenoid a current
11. The principle of electric meter is heating effect of current. of 1.0 amp flows in it. When 100 volt A.C. is applied across
These meters give the reading of Irms. It is important to the same coil, the current drops to 0.5 amp. If the frequency
note that these meters can measure D.C. as well as A.C. of the A.C. source is 50 Hz, then determine the impedance
12. D.C. flows through the cross-section of the conductor and inductance of the solenoid.
whereas A.C. flows mainly along the surface of the Solution :
conductor. This is also known as Skin Effect. The skin In case of D.C., w = 0 and hence Z = R
effect is directly proportional to the frequency. E 100
\ Z=R= = = 100Ω
I 1
Example 1. 1/2
For A.C., Z = éëR 2 + (2 p n L) 2 ùû
Calculate the r.m.s. value of e.m.f. given by
E = 8 sin w t + 6 sin 2 w t volts. 2 2 1/ 2
= [(100) + (100 p L) ]
Solution : (where w = 2pn & n is frequency of AC source)
The mean square value is given by E = E 2 \ 200 = [(100) 2 + (100 p L) 2 ]1 / 2
\ E 2 = (8 sin w t + 6 sin 2 w t ) 2 ì 100 ü
íQ Z = = 200 W ý Solving we get L = 0.55 henry
î 0.5 þ
= 64sin 2 w t + 96sin w t .sin 2 w t + 36sin 2 2 w t Example 5.
1 1 A series LCR circuit containing a resistance of 120 ohm
We know that sin 2 w t = , sin 2 2 w t = , and has angular resonance frequency 4 × 10 5 rad/s. At
2 2
resonance, the voltage across resistance and inductance
sin w t . sin 2 w t = 0 are 60 V and 40 V respectively. Find the values of L and C.
1 1 At what frequency, the current in the circuit lags behind
\ E 2 = 64 ´ + 96 ´ 0 + 36 ´ = 32 + 18 = 50 the voltage by 45º?
2 2
Solution :
or E r.m.s. = (E 2 ) = 50 = 7.07 volt Here, R = 120 W, w0 = 4 × 105 rad/s VR = 60 V, VL = 40
Example 2. V, L = ? C = ? n = ?
If a current i given by i = i0 sin(wt – p/2) flows in an A.C. VR 60
Current through R at resonance = = = 0. 5 A
circuit across which an A.C. potential E = E0 sin wt has R 120
been applied, then power consumption P in the circuit will This is also the current through L.
be \ VL = I (XL) = I ( w 0L)
VL 40
(a) P = E0 Ι0 / 2 (b) P = E Ι / 2 L= = = 2 ´10 - 4 H
(c) P = E0 I0 / 2 (d) zero w0 I 4 ´10 ´ 0.5
5

Solution : (d) 1 1 1
From w0 = ,C= =
1 LC w02 L (4 ´ 10 ) ´ 2 ´ 10- 4
5 2
P = Vr.m.s. ´ Ι r.m.s. ´ cos f = E 0 ´ I 0 cos p / 2 = 0
2 1 1
Example 3. = ´ 10 - 6 F = mF
32 32
A capacitor of capacitance 1 mF is charged to a potential X - XC
of 1 V. It is connected in parallel to an inductor of From tan f = L
inductance 10–3 H. Find the maximum current that will R
flow in the circuit. XL - XC
Þ tan 45º = = 1 Þ X – X = 120
L C
Solution : 120
Charge on capacitor q0 = V × C = 1 × 10–6 coulomb 1
wL - = 120 ...(i)
Here q = q0 sinwt wC
Free eBooks on @neetquestionpaper2020

794 Ph y si cs
1 1 (c) the current through the a.c. source is maximum
As w0 = \ = w0 2 L (d) currents through L and R are equal
LC C
Solution :
w20 2 2 In the circuit shown, L and C are in series. Therefore, it is
Put in (i) w L - L = 120 or w L - w0 L = 120 w
w a series resonance circuit. Hence current through a.c. source
Putting the value of L and w0, we get
is maximum.
w 2 ´ 2 ´10 -4 - (4 ´ 105 ) 2 ´ 2 ´ 10-4 = 120 w Example 9.
or w 2 - 60 w - 16 ´ 1010 = 0 An a.c. circuit consists of only an inductor of inductance 2H.
As current lags behind the voltage by 45º, therefore If the current is represented by a sine wave of amplitude 0.25
XL > XC amp. and frequency 60 Hz, calculate the effective potential
Taking positive sign and solving the quadratic equation, difference across the inductor.
we get w = 8 × 105 rad/sec Solution :
Example 6. The effective potential difference across the inductor is
A coil has an inductance of 0.7 henry and is joined in series given by
with a resistance of 220 W. When an alternating e.m.f. of I0
220 V at 50 cycles per second, is applied to it, then what Veff = Ieff. XL = .2 p f L; Veff = Vrms
2
will be the wattless component of current in the circuit? Given that I0 = 0.25 amp, f = 60 Hz, L = 2H
Solution :
0.25
Here, XL = wL = 2 p n L = 2 p × 50 × 0.7 × = 220 W \ Veff = × 2 × 3.14 × 60 × 2 = 133.2 Volt
R = 220 W 2
Example 10.
Z = R 2 + X 2L = 220 2 + 220 2 = 220 2 ohm. If a domestic appliance draws 2.5 A from a 220-V, 60- Hz
\ wattless component of current is A.C. power supply, then find
EO 220 1 (a) the average current
I= = = = 0.707A (b) the average of the square of the current
Z 220 2 2
Example 7. (c) the current amplitude
A 60 volt-10 watt bulb is operated at 100 volt-60 Hz a.c. (d) the supply voltage amplitude.
The inductance required is Solution :
(a) 2.56 H (b) 0.32 H (a) The average of sinusoidal AC values over any whole
(c) 0.64 H (d) 1.28 H number of cycles is zero.
Solution : (d) (b) RMS value of current = Irms = 2.5 A
P 10 1 V 2 60 ´ 60 \ (I2 )av = (I rms )2 = 6.25 A 2
I= = = A; R= = = 360 W ;
V 60 6 P 10 Im
(c) Irms =
V 100 2
Z= = = 600W
Ι 1/ 6 \ Current amplitude = 2Irms = 2(2.5A) = 3.5 A
X 2L = Z 2 - R 2 = 6002 - 360 2 = (600 + 360) (600 - 360) Vm
(d) Vrms = 220V =
X L = 960 ´ 240 = 240 ´ 2 = 480 W 2
wL = 2pnL = X L = 480 \ Supply voltage amplitude
X 480 Vm = 2(Vrms ) = 2(220V ) = 311 V..
L= L = = 1.28 H
2pn 120 p Example 11.
Example 8. A 100 mF capacitor in series with a 40 W resistance is
In the circuit shown in the fig., at resonance connected to a 110 V, 60 Hz supply.
L C (a) What is the maximum current in the circuit?
(b) What is the time lag between current maximum and
voltage maximum?
Solution :
R (a) Here, C = 100 mF = 100 × 10–6 F, R = 40 W,
Vrms = 110 V, f = 60 Hz
Peak voltage, V0 = 2 . Vrms = 100 2 = 155.54 V
(a) the power factor is zero 1
Circuit impedance, Z = R2 +
(b) the current through the a.c. source is minimum w C2 2
Free eBooks on @neetquestionpaper2020

Alternating Current 795

1 Now, I = I0 sin wt = 2 Irms sin 2 p f t


= 40 2 + -6 2
(2 ´ p ´ 60 ´ 100 ´ 10 ) 1
= 2 × 5.0 sin 2 p × 50 × = 6.124 amp
= 1600 + 703.60 = 2303.60 = 48 W 300
Hence, maximum current in coil, Example 13.
V 155.54 30.0 µF capacitor is connected to a 220 V, 50 Hz source.
I0 = 0 = = 3.24 A Find the capacitive reactance and the current (rms and
Z 48
(b) Phase lead angle (for current), peak) in the circuit. If the frequency is doubled, what
1 1 happens to the capacitive reactance and the current?
q = tan -1 = tan -1 Solution :
wCR 2 ´ 3.14 ´ 60 ´ 100 ´ 10-6 ´ 40
–1
= tan 0.66315 = 33° 33’ (taken 33.5°) 1
The capacitive reactance is XC = = 106W
q q 33.5 2pfC
Time lead, t = = = = 0.001551 sec
w 2 p n 360 ´ 60 Vrms
= 1.551 × 10–3 sec The rms current is Irms = = 2.08A
XC
Voltage will lag current by = 1.551 ms.
Example 12. The peak current is Im = 2Irms = 2.96A
The effective value of current in a 50 cycle a.c. circuit is This current oscillates between 2.96A and – 2.96A and is
5.0 amp. What is the value of the current 1/300 sec after it ahead of the voltage by 90º.
is zero? If the frequency is doubled, the capacitive reactance is
Solution : halved and consequently, the current is doubled.
Here, Irms = 5.0 amp, f = 50 cps

22.1
Solve following problems with the help of above text and 5. An inductor-coil having some resistance is connected to
examples an AC source. Which of the following quantities have zero
1. A capacitor acts as an infinite resistance for average value over a cycle?
(a) DC (b) AC [more than one option may be correct]
(c) DC as well as AC (d) neither AC nor DC (a) Current
(b) Induced emf in the inductor
2. An inductor, a resistor and a capacitor are joined in series
(c) Joule heat
with an AC source. As the frequency of the source is
slightly increased from a very low value, the reactance of (d) Magnetic energy stored in the inductor
the 6. An A.C. source is connected to a resistive circuit. Which
of the following is true?
(a) inductor increases
(a) Current leads ahead of voltage in phase
(b) resistor increases
(b) Current lags behind voltage in phase
(c) capacitor increases (c) Current and voltage are in same phase
(d) circuit increases (d) Any of the above may be true depending upon the
3. The reactance of a circuit is zero. It is possible that the value of resistance.
circuit contains [more than one option may be correct] 7. The capacitive reactance in an A.C. circuit is
(a) an inductor and a capacitor (a) effective resistance due to capacity
(b) an inductor but no capacitor (b) effective wattage
(c) a capacitor but no inductor (c) effective voltage
(d) neither an inductor nor a capacitor (d) None of these
8. The power factor in a circuit connected to an A.C. The
4. In an AC series circuit, the instantaneous current is zero
when the instantaneous voltage is maximum.Connected value of power factor is
to the source may be a (a) unity when the circuit contians an ideal inductance
only
[more than one option may be correct]
(b) unity when the circuit contians an ideal resistance
(a) pure inductor only
(b) pure capacitor (c) zero when the circuit contains an ideal resistance only
(c) pure resistor (d) unity when the circuit contains an ideal capacitance
(d) combination of an inductor and a capacitor only
Free eBooks on @neetquestionpaper2020

796 Ph y si cs
9. Current in a circuit is wattless if 16. In an L.C.R. series a.c. circuit, the current
(a) inductance in the circuit is zero (a) is always in phase with the voltage
(b) resistance in the circuit is zero (b) always lags the generator voltage
(c) current is alternating (c) always leads the generator voltage
(d) resistance and inductance both are zero (d) None of these
10. Power factor is one for 17. The root mean square value of alternating current is equal
(a) pure inductor to
(b) pure capacitor (a) twice the peak value
(c) pure resistor (b) half the peak value
(d) either an inductor or a capacitor.
(c) 1 / 2 times the peak value
11. Of the following about capacitive reactance which is
correct? (d) peak value
(a) The reactance of the capacitor is directly proportional 18. Same current is flowing in two alternating circuits. The
to its ability to store charge first circuit contains only inductance and the other contains
(b) Capacitive reactance is inversely proportional to the only a capacitor. If the frequency of the e.m.f. of a.c. is
frequency of the current increased, what will be the effect on the value of the
current?
(c) Capacitive reactance is mesured in farad
(a) Increases in the first circuit and decreases in the other
(d) The reactance of a capacitor in an A.C. circuit is
similar to the resistance of a capacitor in a D.C. circuit (b) Increases in both the circuits
12. With increase in frequency of an A.C. supply, the inductive (c) Decreases in both the circuits
reactance (d) Decreases in first circuit and increases in the other
(a) decreases 19. The average power dissipated in a pure inductance is
(b) increases directly with frequency 1 2
(a) LΙ (b) L Ι 2
(c) increases as square of frequency 2
(d) decreases inversely with frequency (c) L Ι 2 / 4 (d) zero
13. With increase in frequency of an A.C. supply, the
20. If a current I given by I = I0 sin (w t – p/2) flows in
impedance of an L-C-R series circuit
inductance in an A.C. circuit across which an A.C. potential
(a) remains constant E = E0 sin wt has been applied, then power consumption
(b) increases P in the circuit will be
(c) decreases
(d) decreases at first, becomes minimum and then (a) P = E0 Ι0 / 2 (b) P = E Ι / 2
increases.
(c) P = E 0 Ι 0 / 2 (d) zero
14. An alternating voltage of frequency w is induced in electric
circuit consisting of an inductance L and capacitance C, 21. In series L-C-R circuit, the voltages across R, L and C are
connected in parallel. Then across the inductance coil the VR, VL and VC respectively. Then the voltage of applied
(i) current is maximum when w2 = 1/(L C) a.c. source must be
(ii) current is minimum when w2 = 1/(L C) (a) VR + VL + VC
(iii) voltage is minimum when w2 = 1/(L C)
(b) [( VR ) 2 + ( VL - VC ) 2 ]
(iv) voltage is maximum when w2 = 1/(L C)
(a) (i) and (iii) are correct (c) VR + VC - VL
(b) (i) and (iv) are correct (d) [(VR + VL ) 2 + (VC ) 2 ]1 / 2
(c) (ii) and (iii) are correct 22. The heat produced in a given resistance in a given time by
(d) (ii) and (iv) are correct the sinusoidal current I0sin wt will be the same as that of
15. The current in resistance R at resonance is a steady current of magnitude nearly
R (a) 0.71 I0 (b) 1.412 I0
(c) I0 (d) I 0
23. The sinusoidal A.C. current flows through a resistor of
C resistance R. If the peak current is Ip, then power dissipated
L
is
~ (a) I p 2 R cos q (b)
1 2
Ip R
V = V0 sin wt 2
(a) zero (b) minimum but finite 4 2 1 2
(c) maximum but finite (d) infinite (c) Ip R (d) 2 I p R
p p
Free eBooks on @neetquestionpaper2020

Alternating Current 797

24. In a series combination of R, L and C to an A.C. source at 26. What will be the phase difference between virtual voltage
resonance, if R = 20 ohm, then impedance Z of the and virtual current, when the current in the circuit is
combination is wattless?
(a) 90º (b) 45º
(a) 20 ohm (b) Zero
(c) 180º (d) 60º
(c) 1 ohm (d) 400 ohm 27. In an a.c. circuit with phase voltage V and current I, the
25. If the frequency of an A.C. is made 4 times of its initial power dissipated is
value, the inductive reactance will VΙ VΙ
(a) be 4 times (b) be 2 times (a) (b)
2 2
(c) be half (d) remain the same (c) VΙ (d) VI cos q
ANSWER KEY
1. (a) 2. (a) 3. (a,d) 4. (a,b,d) 5. (a,b) 6. (c) 7. (a) 8. (b) 9. (b) 10. (c) 11.(b) 12.(b) 13.(d)
14.(d) 15.(c) 16. (d) 17.(c) 18.(d) 19.(d) 20.(d) 21. (b) 22.(a) 23.(b) 24.(a) 25.(a) 26.(a) 27. (d)

VARYING CURRENT Time Constant


When the key in a D.C. circuit (containing a D.C. source of emf, L
has dimensions of time. It is called inductive time constant
inductance coil, resistance and capacitor) is closed or opened, R
the current in the circuit varies. This is known as varying current of LR-circuit.
as it varies w.r.t. time and takes a final value after a short while. R L
L - . æ e - 1ö
At t = ; I = I0 (1 - e L R ) = I0 (1 – e–1) = I0 ç
Growth of Current R è e ÷ø
If K is closed at t = 0 so at t = 0, current in the circuit I = 0
After closing the key K at time t let current in the circuit = I æ 2.71 - 1ö
= I0 çè ÷ = 0.632 I0
and for small time in the circuit, current varies with time, 2.71 ø
E The inductive time constant of an LR-circuit is the time in which
the current grows from zero to 0.632 (or 63.2%) of its maximum
K value. When t ® ¥.
æ - .¥ ö
R

I è ø
(
I = I0 ç1 - e L ÷ = I0 I - e -¥ = I 0 (1 - 0 ))
a c Potential difference across resistance :
R b L æ - tö
R
dI E
dI VR = E ç1 - e L ÷ ; VL = L
so if rate of change of current with time = è ø dt
dt
then due to phenomenon of self induction, induced emf across R VL
- t
I = I0 - I0 e L ;
dI
inductance = - L - t æ Rö
R
t
dt dI
Potential difference across the resistance = IR = 0 – I0 e L ç - ÷
dt è Lø
During growth of current in L-R circuit, if we applying R
- t
Kirchhoff’s loop rule then VL = E e L
æ dI ö Initially, an inductor acts to oppose changes in the current through
E + ç - L ÷ = IR it. A long time later, it acts like ordinary connecting wire.
è dt ø
On solving it we get the value of current at any time t during DECAY OF CURRENT
æ - tö
R Let the current has reached its steady state value I0 through in-
growth of current in LR-circuit. I = I0 ç1 - e L ÷ ductor. Now switch K in the circuit shown in fig. has been closed.
çè ÷ø
Graph showing how current varies with time
I
I0= E/R

0.63 I0

Let this time is t = 0.


Let at t = 0 current in the circuit (which is maximum) = I0
t After time t current in the circuit = I
tL
Free eBooks on @neetquestionpaper2020

798 Ph y si cs
Applying Kirchhoff’s loop rule to this circuit dissipated as heat. Assume an idealized situation in which energy
æ dI ö is not radiated away from the circuit. With these idealizations-
0 + ç -L ÷ = IR (since there is no source of e.m.f.) zero resistance and no radiation, the oscillations in the circuit
è dt ø
persist indefinitely and the energy is transferred from the
dI dI R capacitor’s electric field to the inductor’s magnetic field back
or L = - IR or = - dt
dt I L and forth. The total energy associated with the circuit is constant.
The eqn. gives the value of current at any time t during decay of This is analogous to the transfer of energy in an oscillating
current in LR-circuit. mechanical system from potential energy to kinetic energy and
I back, with constant total energy.
I0 = E/R Let us now derive an equation for the oscillations in an L-C
circuit.

0.37 I0

tL t
L
Again, dimensions of are same as that of time Refer figure (a) : The capacitor is charged to a potential difference
R V such that charge on capacitor q0 = CV
The inductive time constant of the LR-circuit can also be defined
Here q0 is the maximum charge on the capacitor. At time t = 0, it
by using equation
is connected to an inductor through a switch S. At time t = 0,
L switch S is closed.
Setting t = in equation., we get
R Refer figure (b) : When the switch is closed, the capacitor starts
R L discharging. Let at time t charge on the capacitor is q (< q0) and
- . 1
I = I0 e L R = I 0 e -1 = I0 or I @ 0.37 I0. since, it is further decreasing there is a current i in the circuit in
e
As t ® ¥, I®0 the direction shown in figure.
VR = IR The potential difference across capacitor = potential difference
across inductor,
q æ di ö
E or Vb – Va = Vc – Vd \ = Lç ÷ ...(i)
C è dt ø
Now, as the charge is decreasing,
æ -dq ö di d 2q
i=ç =- 2
VR è dt ÷ø or
dt dt
di
Substituting this value of in equation (i), we get
0 t dt
R dI
- t R
q æ d2q ö d2q æ 1 ö
or V = e L VL = L I = I0 e - L t
= -L ç 2 ÷ or = -ç q
è LC ÷ø
R dt C ...(ii)
è dt ø dt 2
R
dI - t æ Rö R This is the standard equation of simple harmonic motion
= I0 e L ç - ÷ or V = - E e - L t
dt è Lø L æ d 2x ö
2
0 t ç 2 = -w x ÷
è dt ø
1 1
Here w = or f = ...(iii)
LC 2p LC
VL The general solution of equation (ii),
is q = q 0 cos(wt ± f)
In case f = 0 as q = q0 at t = 0.
Thus, we can say that charge in the circuit oscillates with angular
LC OSCILLATIONS frequency given by equation (iii). Thus,
If a charged capacitor C is short-circuited though an inductor L, di
In L-C oscillations, q, i and all oscillate harmonically with
the charge and current in the circuit start oscillating simple dt
harmonically. If the resistance of the circuit is zero, no energy is same angular frequency w. But the phase difference between q
Free eBooks on @neetquestionpaper2020

Alternating Current 799


di Power losses in a transformer :
and i or between i and is p/2. Their amplitudes are q0, q0w (a) Copper loss. This is due to resistance of the winding of
dt
and w2q0 respectively. So primary and secondary coil (I2 R)
q = q0coswt, then (b) Iron loss or Eddy current loss.
dq di (c) Loss due to leakage of magnetic flux.
i= = - q 0 w sin wt ; = -q 0 w 2 cos wt
dt dt (d) Hysteresis : Due to repeated magnetisation and
Similarly potential energy across capacitor (UC) and across demagnetisation of iron core.
inductor (UL) also oscillate with double the frequency 2w. (e) Humming loss : Due to vibration.
TRANSFORMER Inspite of all these losses, we have transformers with efficiency
A transformer is a device for converting high voltage into low of 70% – 90%.
voltage and vice versa, without change in power. Example 14.
There are two types of transformers. An ideal choke takes a current of 10 ampere when connected
(a) Step up transformer : It converts low voltage into high to an A.C. supply of 125 volt and 50 Hz. A pure resistor
voltage. under the same conditions takes a current of 12.5 ampere. If
(b) Step down transformer : It converts high voltage into the two are connected to an A.C. supply of 100 2 volt and
low voltage.
40 hertz, then find the current in a series combination of the
The principle of a transformer is based on mutual above resistor and inductor.
induction and a transformer always works on AC. The input
Solution :
is appleid across primary terminals and output is obtained
across secondary terminals. For series combination, Z = [R 2 + (X L ) 2 ]
The ratio of number of turns in secondary and primary is
called the turn ratio 125
R= = 10 W, w L = 2 p f L = V/I
nS 12.5
i.e., = turn ratio K.
nP \ 2 p×50×L = 125/10 = 12.5 or 2 p L = 0.25
If E P and E S are alternating voltages, IP and IS the For 40 Hz frequency, XL = 2 p L×f = 0.25 × 40 = 10 W
alternating currents across primary and secondary terminals
E n I Now Z = [(10) 2 + (10) 2 ] = 10 2 ;
respectively then, S = S = K = P .
E P nP IS
Efficiency of transformer, I 0 100 2
Current = = = 10 A
Output power Pout ES I S Z 10 2
h= = =
Input power Pin EP I P Example 15.
Comparative study of step-up transformer and step-down A low loss transformer has 230 V applied to primary and
transformer. gives 4.6 V in secondary. The secondary is connected to a
load which draws 5 A current. Find the current in primary.
Step - up transformer Step - down transformer
Solution :
1. E s > E P 1. Es < E p
Assuming no loss of power Ep Ip = EsIs
2. Ns > N p 2. Ns < N p
3. IS < Ip 3. Is > I p Es Is 5
\ Ip = = 4.6 ´ = 0.1 A
4. Zs < Zp 4. Zs < Z p Ep 230
5. k > 1 5. k < 1

22.2
1. For long distance transmission, the A.C.is stepped up 2. The transformer voltage induced in the secondary coil of
because of high voltage, the transmission is a transformer is mainly due to
(a) faster (a) a varying electric field
(b) economical (b) a varying magnetic field
(c) undamped (c) the vibrations of the primary coil
(d) less dangerous (d) the iron core of the transformer
Free eBooks on @neetquestionpaper2020

800 Ph y si cs

3. Eddy currents in the core of transformer can't be developed 7. Alternating current is converted to direct current by
by (a) rectifier
(a) increasing the number of turns in secondary coil (b) dynamo
(b) taking laminated transformer (c) transformer
(c) making step down transformer (d) motor
(d) using a weak a.c. at high potential 8. A transformer is employed to
4. The frequency of A.C. mains in India is (a) convert A.C. into D.C.
(a) 30 c/s (b) 50 c/s (b) convert D.C. into A.C.
(c) 60 c/s (d) 120 c/s (c) obtain a suitable A.C. voltage
5. Hot wire ammeters are used for measuring (d) obtain a suitable D.C. voltage
(a) A.C. only 9. A choke is preferred to a resistance for limiting current in
A.C. circuit because
(b) D.C. only
(a) choke is cheap
(c) both A.C. and D.C.
(b) there is no wastage of energy
(d) None of these
(c) current becomes wattless
6. Alternating current cannot be measured by D.C. ammeter
(d) current strength increases
because
10. Transformers are used
(a) A.C. cannot pass through D.C. ammeter
(b) average value of current for complete cycle is zero (a) in DC circuit only
(c) A.C. is virtual (b) in AC circuits only
(d) A.C. changes its direction (c) in both DC and AC circuits
(d) neither in DC nor in AC circuits

ANSWER KEY
1. (c) 2. (b) 3. (b) 4. (b) 5. (c) 6. (b) 7. (a) 8. (c) 9.(b) 10.(b)
Free eBooks on @neetquestionpaper2020

Alternating Current 801

Very Short / Short Answer Questions 11. State the principle of working of a transformer. Can a
transformer be used to step up or step down a d.c. voltage?
1. What is the frequency of domestic alternating current
Justify your answer. (Outside Delhi 2011)
supply ? How many times does it become zero in one
12. Show that in an a.c. circuit containing a pure conductor,
second?
the voltage is ahead of current by p/2 in phase.
2. What will be the effect on inductive reactance XL and
(Outside Delhi 2011)
capacitive reactance XC, if frequency of a.c. source is
13. An a.c. voltage, V = V0sin wt, is applied across a pure
increased?
capacitor C. Obtain an expression for the current I in the
3. What is the value of power factor at resonance in LCR
circuit and hence obtain the
circuit?
(i) capacitive reactance of the circuit, and
4. What is the cause of hysteresis loss in a transformer?
(ii) the ‘phase’ , of the current flowing, with respect to
5. In an L.C. circuit, name any physical quantity that
the applied voltage. (COMPTT. Outside Delhi 2010)
oscillates in (i) the inductor (ii) the capacitor.
6. Derive an expression for the root mean square value Long Answer Questions
(R.M.S.) of an alternating current.
14. What is impedance? Give its SI unit. Using the phasor
7. Derive an expression for the average power consumed in diagram or otherwise derive an expression for the
a pure resistive circuit over a complete cycle. impedance of an a.c. circuit containing L, C and R in
8. An inductor L of reactance XL is connected in series with series. Find the expression for resonant frequency.
a bulb B to an a.c. source as shown in the figure. 15. Explain how LC circuit produces electrical oscillations.
L B 0.05
16. A 12 ohm resistance and an inductance of H are
p
connected in series. Across the ends of this circuit an
~ alternating voltage of 130 V and frequency 50 cycles/s is
Explain briefly how does the brightness of the bulb connected. Calculate the current in the circuit and the
changes when potential difference across the inductance.
(i) number of turns of the inductor is reduced and L R
(ii) a capacitor of reactance XC = XL is introduced in the
circuit. 17.
9. An alternating voltage given by V = 140 sin 314 t is ~
connected across a pure resistor of 50 W. Find In the given circuit, the potential difference across the
(Outside Delhi -2012) inductor L and resistor R are 200 V and 150 V respectively
(i) the frequency of the source. and the r.m.s value of current is 5 A. Calculate the angle
(ii) the rms current through the resistor. between the voltage and the current.
10. The graph (i) and (ii) represent the variation of the 18. (a) A voltage V = V0 sin wt applied to a series LCR
opposition offered by the circuit element to the flow of circuit drive a current i = i0 sin wt in the circuit.
alternating current with frequency of the applied emf. Deduce the expression for the average power
Identify the circuit element corresponding to each graph. dissipated in the circuit.
(COMPTT. Outside Delhi 2011) (b) For circuits used for transporting electric power, a
(i) Opposition low power factor implies large power loss in
to current transmission. Explain
(c) Define the term ‘wattless current’.
(COMPTT. Delhi Board -2012)
19. The figure shows a series LCR circuit with L = 10.0 H,
O Frequency C = 40 µF, R = 60 W connected to a variable frequency
(ii) Opposition 240 V source.
to current R

C
O Frequency
L
Free eBooks on @neetquestionpaper2020

802 Ph y si cs
Calculate 23. The frequency of A.C. mains in India is
(i) the angular frequency of the soruce which drives (a) 30 c/s (b) 50 c/s
the circuit at resonance, (c) 60 c/s (d) 120 c/s
(ii) the current at the resonating frequency. 24. An LCR series circuit, connected to a source E, is at
(iii) the rms potential drop across the inductor at resonance. Then the voltage across
resonance. (Delhi Board -2012) (a) R is zero
20. (a) What do you understand by sharpness of resonance (b) R equals applied voltage
(c) C is zero
in a series LCR circuit? Derive an expression for
(d) L equals applied voltage
Q-factor of the circuit.
25. The impedance in a circuit containing a resistance of 1 W
(b) Three electrical circuits having a.c. source of variable
and an inductance of 0.1 H in series, for AC of 50 Hz, is
frequency are shown in the figures. Initially the
current flowing in each of these is same. If the (a) 100 10 W (b) 10 10 W
frequency of the applied a.c. source is increased, how
(c) 100W (d) 10W
will the current flowing in these circuits be affected?
Give reason for your answer. 26. Fleming's left and right hand rules are used in
R L C (a) DC motor and AC generator
(b) DC generator and AC motor
(c) DC motor and DC generator
(d) Both rules are same, any one can be used
E E E 27. An AC voltage source has an output of V = 200sin 2pft .
(Delhi Board -2011 COMPTT.)
This source is connected to a 100 W resistor. RMS current
Multiple Choice Questions in the resistance is
21. The time constant of C–R circuit is (a) 1.41 A (b) 2.41 A (c) 3.41 A (d) 0.71 A
(a) 1/CR (b) C/R (c) C R (d) R/C 28. A bulb and a capacitor are connected in series to a source
22. In an a.c. circuit, the r.m.s. value of current, Irms is related of alternating current. If its frequency is increased, while
to the peak current, I0 by the relation keeping the voltage of the source constant, then bulb will
(a) I rms = 2 I 0 (b) I rms = p I 0 (a) give more intense light
(b) give less intense light
1 1 (c) give light of same intensity before
(c) I rms = I0 (d) I rms = I0
p 2 (d) stop radiating light

1. In an experiment, 200 V A.C. is applied at the ends of an 4. An inductance L having a resistance R is connected to an
LCR circuit. The circuit consists of an inductive reactance alternating source of angular frequency w. The Quality factor
(XL ) = 50 W, capacitive reactance (XC ) = 50 W and ohmic Q of inductance is [CBSE PMT 2000]
resistance (R) = 10 W. The impedance of the circuit is (a) R/ wL (b) (wL/R)2 (c) (R /wL)½ (d) wL/R
[CBSE PMT 1996]
(a) 10W (b) 20W (c) 30W (d) 40W 5. A capacitor has capacitance C and reactance X, if
2. The primary winding of transformers has 500 turns whereas capacitance and frequency become double, then reactance
its secondary has 5000 turns. The primary is connected to will be [CBSE PMT 2001]
an A.C. supply of 20 V, 50 Hz. The secondary will have an (a) 4X (b) X/2 (c) X/4 (d) 2X
output of [CBSE PMT 1997] 6. In a series resonant circuit, having L,C and R as its elements,
(a) 2V, 5Hz (b) 200 V, 500 Hz the resonant current is i. The power dissipated in circuit at
resonance is [CBSE PMT 2002]
(c) 2V, 50 Hz (d) 200 V, 50Hz
3. A step up transformer operates on a 230 V line and supplies i2R
a current of 2 ampere. The ratio of primary and secondary (a) (b) zero
(wL -1/ wC)
winding is 1:25 . The current in primary is
[CBSE PMT 1998] (c) i2 wL (d) i2 R.
(a) 25 A (b) 50 A (c) 15 A (d) 12.5 A Whereas w is angular resonant frequency
Free eBooks on @neetquestionpaper2020

Alternating Current 803


7. In a circuit L, C and R are connected in series with an 15. In an electrical circuit R, L, C and an a.c. voltage source are
alternating voltage source of frequency f. The current leads all connected in series. When L is removed from the circuit,
the voltage by 45°. The value of C is [CBSE PMT 2005] the phase difference between the voltage the current in the
1 1 circuit is p/3. If instead, C is removed from the circuit, the
(a) (b) phase difference is again p/3. The power factor of the circuit
pf ( 2pfL - R ) 2pf (2pfL - R) is [CBSE -PMT 2012S]
1 1
(c) (d) 2pf (2pfL + R) (a) 1/2 (b) 1/ 2 (c) 1 (d) 3/2
pf (2pfL + R) 16. The current (I) in the inductance is varying with time
8. The core of any transformer is laminated so as to according to the plot shown in figure.
[CBSE PMT 2006]
(a) reduce the energy loss due to eddy currents I
(b) make it light weight
(c) make it robust and sturdy T/2
(d) increase secondary voltage t T
9. In the given circuit the reading of voltmeter Which one of the following is the correct variation of voltage
V1 and V2 are 300 volt each. The reading of the voltmeter with time in the coil? [CBSE -PMT 2012S]
V3 and ammeter A are respectively [CBSE-PMT 2010] V V
L C R = 100 W
(a) t (b)
T/2 T T/2 T
V1 V2 V3 t
A
V V
~ t
(c) (d)
220 V, 50 Hz T/2 t T T/2 T
(a) 150 V and 2.2 A (b) 220 V and 2.0 A
17. The instantaneous values of alternating current and voltages
(c) 220 V and 2.0 A (d) 100 V and 2.0 A
in a circuit are given as [CBSE -PMT 2012M]
10. A 220 volts input is supplied to a transformer. The output
circuit draws a current of 2.0 ampere at 440 volts. If the 1
i= sin(100pt ) A
efficiency of the transformer is 80%, the current drawn by 2
the primary windings of the transformer is 1
e= sin(100pt + p / 3) Volt
[CBSE-PMT 2010] 2
(a) 3.6 ampere (b) 2.8 ampere The average power in Watt consumed in the circuit is :
(c) 2.5 ampere (d) 5.0 ampere
1 3 1 1
11. An ac voltage is applied to a resistance R and an inductor L (a) (b) (c) (d)
in series. If R and the inductive reactance are both equal to 4 4 2 8
3W, the phase difference between the applied voltage and 18. The power factor of an AC circuit having resistance (R)
the current in the circuit is [CBSE-PMT 2011] and inductance (L) connected in series and an angular
velocity w is [AIEEE 2002]
(a) p/6 (b) p/4 (c) p/2 (d) zero
(a) R/wL (b) R/(R2 w2L2 )
12. In an ac circuit an alternating voltage e = 200 2 sin 100 (c) wL/R (d) R/(R2w2 L2)½
t volts is connected to a capacitor of capacity 1 mF. The
19. In a transformers, number of turns in primary coil are 140
r.m.s. value of the current in the circuit is and that in secondary coil are 280. If current in primary
[CBSE-PMT 2011] coil is 4A, then that in secondary coil is [AIEEE 2002]
(a) 10 mA (b) 100 mA (c) 200 mA (d) 20 mA (a) 4 A (b) 2 A (c) 6 A (d) 10 A
13. The r.m.s. value of potential difference V shown in the figure 20. In an LCR circuit the capacitance is changed from C to 2C.
is [CBSE-PMT 2011M] For the same resonant frequency, the inductance should be
V
changed from L to [AIEEE 2004]
V0 (a) L/2 (b) 2L (c) 4L (d) L/4
21. Alternating current can not be measured by D.C. ammeter
O t
T/2 T because [AIEEE 2004]
(a) V0 (b) V0 / 2 (c) V0/2 (d) V0 / 3 (a) average value of current for complete cycle is zero
(b) A.C. Changes direction
14. A coil has resistance 30 ohm and inductive reactance 20 ohm (c) A.C. can not pass through D.C. Ammeter
at 50 Hz frequency. If an ac source, of 200 volt, 100 Hz, is (d) D.C. Ammeter will get damaged
connected across the coil, the current in the coil will be 22. In an LCR series a.c. circuit, the voltage across each of the
[CBSE-PMT 2011M] components, L, C and R is 50V. The voltage across the LC
20 combination will be [AIEEE 2004]
(a) 4.0 A (b) 8.0 A (c) A (d) 2.0 A
13 (a) 100 V (b) 50 2 V (c) 50 V (d) 0 V
Free eBooks on @neetquestionpaper2020

804 Ph y si cs
23. The phase difference between the alternating current and VR1R2 V
p (a) at t = 0 and R at t = ¥
emf is . Which of the following cannot be the constituent R12 + R22 2
2
of the circuit? [AIEEE 2005] V V ( R1 + R2 )
(a) R, L (b) C alone (c) L alone (d) L, C (b) R at t = 0 and R1 R2 at t = ¥
2
24. A circuit has a resistance of 12 ohm and an impedance of
V VR1R2
15 ohm. The power factor of the circuit will be –
(c) R at t = 0 and at t = ¥
[AIEEE 2005] 2 R12 + R22
(a) 0.4 (b) 0.8 (c) 0.125 (d) 1.25 V ( R1 + R2 ) V
25. A coil of inductance 300 mH and resistance 2 W is (d) R1 R2 at t = 0 and R at t = ¥
2
connected to a source of voltage 2 V. The current reaches 31. In a series LCR circuit R = 200W and the voltage and the
half of its steady state value in [AIEEE 2005] frequency of the main supply is 220V an d 50 Hz
(a) 0.1 s (b) 0.05 s (c) 0.3 s (d) 0.15 s respectively. On taking out the capacitance from the circuit
26. In a series resonant LCR circuit, the voltage across R is the current lags behind the voltage by 30°. On taking out
100 volts and R = 1 kW with C = 2mF. The resonant the inductor from the circuit the current leads the voltage
frequency w is 200 rad/s. At resonance the voltage across L by 30°. The power dissipated in the LCR circuit is
is [AIEEE 2006] [AIEEE 2010]
(a) 305 W (b) 210 W (c) Zero W (d) 242 W
(a) 2.5 × 10–2 V (b) 40 V
32. A diode detector is used to detect an amplitude modulated
(c) 250 V (d) 4 × 10–3 V
wave of 60% modulation by using a condenser of capacity
27. An inductor (L = 100 mH), a resistor (R = 100 W) and a 250 picofarad in parallel with a load resistance 100 kilo
battery (E = 100 V) are initially connected in series as shown ohm. Find the maximum modulated frequency which could
in the figure. After a long time the battery is disconnected be detected by it. [JEE Main 2013]
after short circuiting the points A and B. The current in the D
(a) 10.62 MHz
circuit 1 ms after the short circuit is [AIEEE 2006]
L
(b) 10.62 kHz
(c) 5.31 MHz Signal C R
(d) 5.31 kHz
R
33. In an LCR circuit as shown below both switches S1 and S2
A
are open initially. Now switch S1 is closed, S2 kept open.
B
(q is charge on the capacitor and t = RC is capacitive time
E
constant). Which of the following statements is correct ?
(a) 1/eA (b) eA (c) 0.1 A (d) 1 A
28. In an a.c. circuit the voltage applied is E = E0 sin wt. The [JEE Main 2013]
æ (a) Work done by the battery V

resulting current in the circuit is I < I0 sin ççwt , ÷÷÷ . The is half of the energy
çè 2ø
dissipated in the resistor R
power consumption in the circuit is given by [AIEEE 2007] S1
(b) At t = t, q = CV/2
E 0 I0
(a) P = 2E0 I0 (b) P< (c) At t = 2t, q = CV (1 – e–2) C
2 S2
t –1
E 0 I0 (d) At t = , q = CV (1 – e ) L
(c) P = zero (d) P < 2
2 34. Find the time constant (in ms) for the given RC circuits in
29. An ideal coil of 10H is connected in series with a resistance the given order respectively. [IIT-JEE 2006]
of 5W and a battery of 5V. After 2 sec the connection is V V V
made, the current flowing in ampere in the circuit is R1 C1
[AIEEE 2007] R1 C1
–1 R2 C2 R1
(a) (1 – e ) (b) (1 – e) (c) e (d) e–1 C1
30. In the circuit shown below, the key K is closed at t = 0. The R2
.
current through the battery is [AIEEE 2010] R2 C2
V K
C2
R1 = 1W, R 2 = 2W, C1 = 4mF and C 2 = 2mF
L R1 8 8
(a) 18, 4, (b) 18, ,4
9 9
R2
8 8
(c) 4, 18, (d) 4, , 18
9 9
Free eBooks on @neetquestionpaper2020

Alternating Current 805


35. An AC voltage source of variable angular frequency w and (a) the bulb glows dimmer
fixed amplitude V0 is connected in series with a capacitance (b) the bulb glows brighter
C and an electric bulb of resistance R (inductance zero). (c) total impedance of the circuit is unchanged
When w is increased [IIT-JEE 2010] (d) total impedance of the circuit increases

The question contains statements given in two columns, which have to be matched. The statements in Column-I are labelled A, B, C
and D, while the statements in Column-II are labelled p, q, r, s and t. Any given statement in Column-I can have correct matching with
ONE OR MORE statement(s) in Column-II.
36. You are given many resistances, capacitors and inductors. These are connected to a variable DC voltage source (the first two
circuits) or an AC voltage source of 50 Hz frequency (the next three circuits) in different ways as shown in Column II. When
a current I (steady state for DC or rms for AC) flows through the circuit, the corresponding voltage V1and V2 , (indicated in
circuits) are related as shown in Column I Match the two.
[IIT-JEE 2010]
Column I Column II
V1 V2

6mH 3m F
(A) I ¹ 0, V1 is proportional to I (p)

V
V1 V2

6mH 2W
(B) I ¹ 0, V2 > V1 (q)

V
V1 V2

6mH 2W
(C) V1 = 0, V2 = V (r)

V
V1 V2

6mH 3m F
(D) I ¹ 0, V2 is proportional to I (s)

V
V1 V2

1kW 3m F
(t)

V
37. A series R– C circuit is connected to AC voltage source. (a) I RA > I RB
Consider two cases; (A) when C is without a dielectric
medium and (B) when C is filled with dielectric of constant (b) I RA < I RB
4. The current IR through the resistor and voltage VC across (c) VCA > VCB
the capacitor are compared in the two cases. Which of the
following is/are true? [IIT-JEE 2011] (d) VCA < VCB
Free eBooks on @neetquestionpaper2020

806 Ph y si cs
38. A series R-C combination is connected to an AC voltage supply power to the consumers at the specified lower voltage. It
of angular frequency w = 500 radian/s. If the impedance of is reasonable to assume that the power cable is purely resistive
the R-C circuit is R 1.25 , the time constant (in and the transformers are ideal with power factor unity. All the
millisecond) of the circuit is [IIT-JEE 2011] currents and voltages mentioned are rms values.
(a) 5 (b) 4 (c) 6 (d) 8 39. In the method using the transformers, assume that the ratio
Paragraph for Questions 39 and 40 of the number of turns in the primary to that in the secondary
A thermal power plant produces electric power of 600 kW at in the step-up transformer is 1 : 10. If the power to the
4000 V, which is to be transported to a place 20 km away from consumers has to be supplied at 200 V, the ratio of the
the power plant for consumers' usage. It can be transported either number of turns in the primary to that in the secondary in
directly with a cable of large current carrying capacity or by the step-down transformer is (JEE Adv. 2013)
using a combination of step-up and step-down transformers at (a) 200 : 1 (b) 150 : 1 (c) 100 : 1 (d) 50 : 1
the two ends. The drawback of the direct transmission is the 40. If the direct transmission method with a cable of resistance
large energy dissipation. In the method using transformers, the
0.4 W km–1 is used, the power dissipation| (in %) during
dissipation is much smaller. In this method , a step-up transformer
transmission is (JEE Adv. 2013)
is used at the plant side so that the current is reduced to a smaller
value. At the consumers' end, a step-down transformer is used to (a) 20 (b) 30 (c) 40 (d) 50

1. In an A.C. circuit, the current flowing in inductance is 7. The primary winding of a transformer has 100 turns and its
I = 5 sin (100 t – p/2) amperes and the potential difference secondary winding has 200 turns. The primary is connected
is V = 200 sin (100 t) volts. The power consumption is to an A.C. supply of 120 V and the current flowing in it is
equal to 10 A. The voltage and the current in the secondary are
(a) 1000 watt (b) 40 watt (a) 240 V, 5 A (b) 240 V, 10 A
(c) 20 watt (d) Zero (c) 60 V, 20 A (d) 120 V, 20 A
2. If resistance of 100W, and inductance of 0.5 henry and 8. A step down transformer is connected to 2400 volts line
capacitance of 10 × 106 farad are connected in series through and 80 amperes of current is found to flow in output load.
50 Hz A.C. supply, then impedance is The ratio of the turns in primary and secondary coil is 20 :
(a) 1.8765 W (b) 18.76 W 1. If transformer efficiency is 100%, then the current flowing
(c) 187.6 W (d) 101.3 W in the primary coil will be
3. Using an A.C. voltmeter the potential difference in the (a) 1600 amp (b) 20 amp
electrical line in a house is read to be 234 volt. If the line (c) 4 amp (d) 1.5 amp
frequency is known to be 50 cycles/second, the equation 9. In the circuit shown in fig, the resonant frequency is
for the line voltage is (a) 75 kc/s 5mF
(a) V = 165 sin (100 p t) (b) V = 331 sin (100 p t) (b) 750 kc/s
(c) V = 220 sin (100 p t) (d) V = 440 sin (100 p t)
(c) 7.5 kc/s 0.1H 5W
4. An inductance of negligible resistance whose reactance is
22 W at 200 Hz is connected to 200 volts, 50 Hz power line. (d) 75 mc/s
The value of inductance is 10. An alternating voltage E (in volts) = 200 2 sin 100 t is
(a) 0.0175 henry (b) 0.175 henry connected to one micro farad capacitor through an a.c.
(c) 1.75 henry (d) 17.5 henry ammeter. The reading of the ammeter shall be
5. An inductive circuit contains resistance of 10 ohms and an (a) 100 mA (b) 20 mA (c) 40 mA (d) 80 mA
inductance of 2 henry. If an A.C. voltage of 120 Volts and 11. The r.m.s value of an a.c. of 50 Hz is 10 amp. The time
frequency 60 Hz is applied to this circuit, the current would taken by the alternating current in reaching from zero to
be nearly maximum value and the peak value of current will be
(a) 0.32 Amp (b) 0.16 Amp (a) 2 × 10–2 sec and 14.14 amp
(c) 0.48 Amp (d) 0.80 Amp (b) 1 × 10–2 sec and 7.07 amp
6. In an a.c. circuit V and I are given by (c) 5 × 10–3 sec and 7.07 amp
V = 100 sin (100 t) volts (d) 5 × 10–3 sec and 14.14 amp
I = 100 sin (100 t + p/3) mA 12. The ratio of mean value over half cycle to r.m.s. value of
the power dissipated in the circuit is A.C. is
(a) 104 watt (b) 10 watt (c) 2.5 watt (d) 5.0 watt (a) 2 : p (b) 2 2 : p (c) 2 :p (d) 2 :1
Free eBooks on @neetquestionpaper2020

Alternating Current 807


13. In a RLC circuit capacitance is changed from C to 2 C. For 25. A.C. power is transmitted from a power house at a high
the resonant frequency to remain unchanged, the inductance voltage as
should be changed from L to (a) the rate of transmission is faster at high voltages
(a) 4 L (b) 2 L (c) L/2 (d) L/4 (b) it is more economical due to less power loss
14. A 12 W resistor and a 0.21 henry inductor are connected in (c) power cannot be transmitted at low voltages
series to an a.c. source operating at 20 volt, 50 cycle. The
phase angle between the current and source voltage is (d) a precaution against theft of transmission lines
(a) 30º (b) 40º (c) 80º (d) 90º 26. In the given circuit, the current drawn from the source is
15. A step down transformer reduces 220 V to 110 V. The

V = 100x sin(100pt )
primary draws 5 ampere of current and secondary supplies

X C = 20W
X L = 10W
R = 20W
9 ampere. The efficiency of transformer is
(a) 20% (b) 44% (c) 90% (d) 100%
~
16. An alternating current is given by
i = i1 coswt + i2 sinwt
The rms current is given by (a) 20 A (b) 10 A (c) 5 A
(d) 5 2 A
i1 + i 2 i1 + i 2 27. The time taken by the current to rise to 0.63 of its maximum
(a) (b) value in a d.c. circuit containing inductance (L) and
2 2
resistance (R) depends on
i12 + i 22 i12 + i 22 L
(c) (d) (a) L only (b) R only (d) LR (c)
2 2 R
17. In an A.C. circuit with phase voltage V and current I, the 28. In a pure capacitive A.C. circuit current and voltage differ
power dissipated is in phase by
(a) VI (b) V2I (c) VI2 (d) V2I2 (a) 0° (b) 45° (c) 90° (d) 180°
18. Which of the following will have the dimensions of time 29. An alternating voltage of 220 V, 50 Hz frequency is applied
(a) LC (b) R/L (c) L/R (d) C/L across a capacitor of capacitance 2 µF. The impedence of
19. In series combination of R, L and C with an A.C. source at the circuit is
resonance, if R = 20 ohm, then impedence Z of the p 1000 5000
combination is (a) (b) (c) 500 p (d)
5000 p p
(a) 20 ohm (b) zero (c) 10 ohm (d) 400 ohm
30. An alternating voltage V = V0 sin wt is applied across a
20. The power factor of an AC circuit having resistance (R) circuit. As a result, a current I = I0 sin (wt – p/2) flows in it.
and inductance (L) connected in series and an angular The power consumed per cycle is
velocity w is (a) zero (b) 0.5 V0I0
(a) R/wL (b) R/(R2 + w2L2)1/2 (c) 0.707 V0I0 (d) 1.414 V0I0
(c) wL/R (d) R/(R2 – w2L2)1/2 31. In an A.C. circuit, a resistance of R ohm is connected in
21. In a LCR circuit at resonance which of these will effect the series with an inductance L. If phase angle between voltage
current in circuit and current be 45°, the value of inductive reactance will be
(a) R only (b) L and R only (a) R/4
(c) R and C only (d) all L, C and R (b) R/2
22. The primary winding of transformer has 500 turns whereas (c) R
its secondary has 5000 turns. The primary is connected to (d) cannot be found with given data
an A.C. supply of 20 V, 50 Hz. The secondary will have an 32. In LCR circuit if resistance increases quality factor
output of
(a) increases finitely (b) decreases finitely
(a) 2 V, 5 Hz (b) 200 V, 500 Hz
(c) remains constant (d) None of these
(c) 2V, 50 Hz (d) 200 V, 50 Hz
33. For the circuit shown in the fig., the current through the
23. Determine the rms value of the emf given by inductor is 0.9 A while the current through the condenser is
E (in volt) = 8 sin ( w t) + 6sin (2 w t) 0.4 A. Then
C
(a) 5 2 V (b) 7 2 V (c) 10 V (d) 10 2 V (a) current drawn from
generator I = 1.13 A
24. A transformer is used to light a 140 W, 24 V bulb from a
(b) w = 1/(1.5 L C) L
240 V a.c. mains. The current in the main cable is 0.7 A.
The efficiency of the transformer is (c) I = 0.5 A
~
(a) 63.8 % (b) 83.3 % (c) 16.7 % (d) 36.2 % (d) I = 0.6 A V = V0 sin wt
Free eBooks on @neetquestionpaper2020

808 Ph y si cs
34. A transformer has an efficiency of 80%. It works at 4 kW 41. Two coils A and B are connected in series across a 240 V,
and 100 V. If secondary voltage is 240 V, the current in 50 Hz supply. The resistance of A is 5 W and the inductance
primary coil is
of B is 0.02 H. The power consumed is 3 kW and the power
(a) 0.4 A (b) 4 A (c) 10 A (d) 40 A
factor is 0.75. The impedance of the circuit is
35. In an oscillating LC circuit the max. charge on the capacitor
is Q. The charge on capacitor when the energy is stored (a) 0.144 W (b) 1.44 W (c) 14.4 W (d) 144 W
equally between electric and magnetic field is 42. In LCR series circuit fed by a DC source, how does the
(a) Q/2 (b) Q / 3 (c) (d) Q amplitude of charge oscillations vary with time during
Q/ 2
discharge ?
36. In an LR circuit f = 50 Hz, L=2H, E=5 volts, R=1 W then
q q
energy stored in inductor is
(a) 50 J (b) 25 J qo
(a) (b)
(c) 100 J (d) None of these
37. A capacitor in an ideal LC circuit is fully charged by a DC
source, then it is disconnected from DC source, the current
in the circuit O t O t
(a) becomes zero instantaneously
(b) grows , monotonically q q
(c) decays monotonically
qo
(d) oscillate infinitely (c) qo (d)
38. In a circuit inductance L and capacitance C are connected
as shown in figure. A1 and A2 are ammeters.
A1 C R1
t O t
O
L A2 43. A steady potential difference of 10 V produces heat at a
rate x in a resistor. The peak value of the alternating voltage
R2
K x
which will produce heat at a rate in the same resistor is
Battery 2
When key K is pressed to complete the circuit, then just (a) 5 V (b) 5 2 V (c) 10 V (d) 10 2 V
after closing key (K), the readings of A1 and A2 will be
(a) zero in both A1 and A2 44. In the circuit shown, when the switch is closed, the capacitor
(b) maximum in both A1 and A2 charges with a time constant
C R
(c) zero in A1 and maximum in A2 (a) RC
(d) maximum in A1 and zero in A2 (b) 2RC
39. The tuning circuit of a radio receiver has a resistance of
1
50 W , an inductor of 10 mH and a variable capacitor. A (c) RC
2 +
1 MHz radio wave produces a potential difference of
0.1 mV. The values of the capacitor to produce resonance (d) RC ln 2 B
is (Take p2 = 10) 45. In the question 86, if the switch is opened after the capacitor
(a) 2.5 pF (b) 5.0 pF (c) 25 pF (d) 50 pF has been charged, it will discharges with a time constant
40. Which one of the following curves represents the variation
1
of impedance (Z) with frequency f in series LCR circuit? (a) RC (b) 2RC (c) RC (d) RC ln 2
2
Z Z
46. Which of the following statement is incorrect ?
(a) (b) (a) In LCR series ac circuit, as the frequency of the source
increases, the impedence of the circuit first decreases
and then increases.
f f (b) If the net reactance of an LCR series ac circuit is same
Z Z as its resistance, then the current lags behind the voltage
(c) (d) by 45°.
(c) At resonance, the impedence of an ac circuit becomes
purely resistive.
(d) Below resonance, voltage leads the current while above
f f it, current leads the voltage.
Free eBooks on @neetquestionpaper2020

Alternating Current 809


47. An inductive coil has a resistance of 100 W. When an a.c. electric field energy stored in the capacitor and average magnetic
signal of requency 1000 Hz is fed to the coil, the applied energy stored in the coil are 25 mJ and 5 mJ respectively. The
voltage leads the current by 45°. What is the inductance of value of RMS current in the circuit is 0.1 A. Then find :
the coil ? 55. Capacitance (C) of the capacitor is
(a) 10 mH (b) 12 mH (c) 16 mH (d) 20mH. (a) 10mF (b) 15mF
48. The primary of a transformer has 400 turns while the (c) 20mF (d) None of these
secondary has 2000 turns. If the power output from the 56. Inductance (L) of inductor is
secondary at 1000 V is 12 kW, what is the primary voltage? (a) 0.25 henry (b) 0.5 henry
(a) 200 V (b) 300 V (c) 400 V (d) 500 V (c) 1 henry (d) 2 henry
49. An inductor of self inductance 100 mH and a resistor of 57. The sum of rms potential difference across each of the three
resistance 50W, are connected to a 2 V battery. The time elements is
required for the current to half its steady value is (a) 50 volt (b) 50 2 volt
(a) 2 milli second (b) 2 ln (0.5) milli second 50
(c) 2 ln (3) milli second (d) 2 ln (2) milli second (c) volt (d) None of these
2
50. A step-up transformer has transformation ratio of 3 : 2. What
PASSAGE 2
is the voltage in secondary, if voltage in primary is 30 V?
In an A. C. source, peak value of A.C. is the maximum value of
(a) 45 V (b) 15 V (c) 90 V (d) 300 V
current in either direction of the cycle. Root mean square (RMS)
51. The instantaneous voltage through a device of impedance is also defined as the direct current which produces the same
20 W is e = 80 sin 100 pt. The effective value of the current is heating effect in a resistor as the actual A.C.
(a) 3 A (b) 2.828 A (c) 1.732 A (d) 4 A 58. A. C. measuring instrument measures its
52. Resonance frequency of LCR series a.c. circuit is f0. Now (a) rms value (b) peak value
the capacitance is made 4 times, then the new resonance (c) average value (d) square of current
frequency will become 59. Current time graph of different source is given which one
(a) f0/4 (b) 2f0 (c) f0 (d) f0/2. will have R. M. S. value V0 :
Directions for Qs. (53 to 54) : Each question contains
STATEMENT-1 and STATEMENT-2. Choose the correct
answer (ONLY ONE option is correct ) from the following-
(a) Statement -1 is false, Statement-2 is true (a)
(b) Statement -1 is true, Statement-2 is true; Statement -2 is a
correct explanation for Statement-1
(c) Statement -1 is true, Statement-2 is true; Statement -2 is
not a correct explanation for Statement-1
(d) Statement -1 is true, Statement-2 is false (b)
53. Statement - 1 : A capacitor blocks direct current in the
steady state.
Statement - 2 : The capacitive reactance of the capacitor is
2V0
inversely proportional to frequency f of the source of emf.
(c)
54. Statement - 1 : In the purely resistive element of a series
LCR, AC circuit the maximum value of rms current V=2V0sin w r

increases with increase in the angular frequency of the


applied emf.
2
(d)
e æ 1 ö
Statement - 2 : Imax = max , z = R 2 + ç wL - ÷ ,
z è wC ø
60. Average voltage for the given source is :
where Imax is the peak current in a cycle.

Directions for Qs. (55 to 60) : Read the following passage(s)


carefully and answer the questions that follows:
PASSAGE 1
In a series LCR circuit with an ideal ac source of peak voltage
50
E0 = 50V, frequency v = Hz and R = 300W. The average (a) V0 (b) 2V0 (c) V0/2 (d) 3V0/2
p
Free eBooks on @neetquestionpaper2020

810 Ph y si cs

16. (d) VL
Exercise 22.1
VR

VC – VL
f I
1 E

I
1. (a) XC(reactance of capacitor) = for D.C., E(applied
wC voltage)
w = 0 Þ XC = ¥
VC - VL (if V > V )
2. (a) The reactance of inductor, XL = wL tan f = C L
VR
1 VL - VC
The reactance of capacitor, X C =
wC = (if VL> VC)
R
where w = 2pn & n is the frequency of A.C source. where f is angle between current & applied voltage.
3. (a,d) The reactance of circuit is zero in two cases E
(i) At resonance, XL = XC 18. (d) In first circuit I1 =
wL
So total reactance, X=(XL–XC) = 0 because they
æ E ö
are in opposite phase. In second circuit I 2 = ç ÷
(ii) It contain neither inductor nor capacitor è 1 / wC ø
6. (c) When resistance is connected to A.C source, then 19. (d) In case of pure inductance cos f = 0, so no power
current & voltage are in same phase. dissipates.
7. (a) Capacitive reactance in an A.C circuit is 1
20. (d) P = Vr.m.s. ´ Ι r.m.s. ´ cos f = E 0 ´ Ι 0 cos p / 2 = 0
1 2
XC = ohm , where C is the capacitance of capacitor I
wC 23. (b) The value of r.m.s current is I rms = P
& w = 2pn (n is the frequency of A.C source). 2
1 2
R so power dissipated is P = I 2rms R = IPR
8. (b) cos f = , where Z is the impedance & 2
Z 24. (a) At resonance inpedence Z = R
26. (a) For wattless current, cos f = 0 i.e. f = 90º.
Z = R 2 + (XL - XC )2 , if there is only resistance then
27. (d) P = V I cos q. So, power dissipation depends upon V
Z = R Þ cosf = 1 and I.
9. (b) If R = 0 Þ cos f = 0 Þ f = 90º so P =0, in this case
power loss is zero & current flowing in the circuit is called Exercise 22.2
wattless current.
1 1 1. (c) The transmission is undamped, because energy loss
11. (b) XC = Þ XC¥ for given C. becomes negligible.
wC w
4. (b) In India the frequency of A.C. current is 50 Hz.
12. (b) XL = wL Þ XL µ w
7. (a) I I
13. (d) Z

t t
Zmin = R D.C after
Alternating current (A.C) rectifying action

wo ® w Exercise 1 : NCERT Based Questions


15. (c) At resonance X L = X C Þ Z = R & curr ent is 1. 50 Hz.
E It becomes zero twice in a complete cycle. So in one
maximum but finite, which is I max = , where E is
R second it becomes zero 100 times.
applied voltage. 9. 50, 1.98 A
Free eBooks on @neetquestionpaper2020

Alternating Current 811


19. (i) 50 Hz. (ii) 5.64A (iii) 0 V2 I 2 10 440 ´ 2 ´ 10
21. (c) 22. (d) 23. (b) I1 = ´ = =5A
V1 8 220 ´ 8
24. (b) 25. (b) 26. (c)
11. (b) The phase difference f is given by
27. (a) 28. (a)
XL 3
Exercise 2 : PAST Competition MCQs tan f = = =1
R 3
1. (a) Given : Supply voltage (Vac) = 200 V p
Þf= .
Inductive reactance (XL) = 50 W 4
Capacitive reactance (XC) = 50 W 200 2
12. (d) Vrms = = 200V
Ohmic resistance (R) = 10 W. 2
We know that impedance of the LCR circuit 200
2 2 2 2 Vrms 1
(Z) = {(X L - X C ) + R } = {(50 - 50) + (10) } = 10 W Irms = =
X C 100 ´ 10-6
2. (d)
= 2 × 10–2 = 20mA
np Ep 1
3. (b) = = (T / 2)V0 2 + 0 V
ns Es 25 13. (b) Vrms = = 0 .
T 2
\ E s = 25E p 14. (a) If w = 50 × 2p then wL = 20W
If w¢ = 100 × 2p then w¢L = 40W
E S ´ IS
But Es Is = E p I p Þ I p = Þ I p = 50A Current flowing in the coil is
Ep
200 200 200
Potential drop across capacitor or inductor I= = =
4. (d) Q = Z 2
R + (w¢L) 2
(30)2 + (40) 2
Potential drop across R.
I = 4A.
wL
= 15. (c)
R
1 when L is removed from the circuit
5. (c) The reactance of capacitor X = where w is R
wC XC p
frequency and C is the capacitance of capacitor. = tan p/3
R 3
6. (d) At resonance wL= 1/wC Z1
p
and i = E/R , So power dissipated in circuit is P = i2R. X C = R tan ....(1) XC
3
7. (d) From figure, when C is remove from the circuit
1 XL p
- wL = tan Z2
tan 45º = wC
1
- wL
– R 3 XL
R w C
p p/3
1 X C = R tan ...(2)
Þ - wL = R 3 R
wC
45º net impedence Z = R 2 + ( X L - X C )2 = R
1 O R
Þ = R + wL
wC R
1 1 power factor cos f = =1
Z
ÞC = = 2 pf ( R + 2 pfL)
w ( R + w L)
16. (d) V = - L di
8. (a) dt
9. (b) As VL = VC = 300 V, resonance will take place di T
\ VR = 220 V Here is + ve for time and
dt 2
220 di T
Current, I = = 2.2 A is – ve for next time so
100 dt 2
\ reading of V3 = 220 V 17. (d) The average power in the circuit where cos f = power
and reading of A = 2.2 A factory
< P > = Vrms × Irms cos f
V2 I
10. (d) = 0.8 1 Þ V2 I 2 = 0.8 180
V1 I2 V1 I1 f = p/3 = phase difference = = 60
3
V1 = 220 V, I2 = 2.0 A, V2 = 440 V
Free eBooks on @neetquestionpaper2020

812 Ph y si cs
i t
1 di R di R
2 = 1 volt
Þ
i
= - dt Þ
L ò i ò
= - dt
L
Vrms = i0 0
2 2
R
1 i R - t
Þ log e =- t Þ i = i0 e L
2 = æ 1ö A i0 L
Irms = ç ÷
2 è 2ø R -100´10 -3
E - t 100 100´10-3 1
p 1 Þi= e L = e =
=
cos f = cos R 100 e
3 2
28. (c) We know that power consumed in a.c. circuit is given
1 1 1 1
<P>= ´ ´ = W by, P = Erms . Irms cosf
2 2 2 8
18. (b) 19. (b) Here, E = E0 sinwt

1 æ pö
20. (a) Resonant frequency w2 = . I = I0 sin ç wt - ÷
LC è 2ø
21. (a) D.C. ammeter measure average current in AC current, which implies that the phase difference,
average current is zero for complete cycle. Hence p
reading will be zero. f=
2
22. (d) Since the phase difference between L & C is p
p æ p ö
\ net voltage difference across LC = 50 - 50 = 0 \ P = E rms .Irms .cos
2
=0 çè Q cos = 0÷
ø
2
æ pö
23. (a) Phase difference for R–L circuit lies between ç 0, ÷ æ - tö
R
è 2ø 29. (a) We have, I = Io ç1 - e L ÷
R 12 4 çè ÷ø
24. (b) Power factor = cos j = = = = 0.8
Z 15 5 (When current is in growth in LR circuit)
25. (a) The charging of inductance given by,
æ - tö
R æ - ´2 ö
5
æ E 5
- ö÷
Rt
ç = ç 1- e L ÷ = ç 1 - e 10 ÷
i = i 0 ç1 - e L ÷ R çè ÷ø 5 çè ÷ø
ç ÷
è ø
= (1 – e–1)
Rt Rt
i0 - - 1
= i 0 (1 - e L )Þ
e = L
2 2 30. (c) At t = 0 , no current will flow through L and R1
Taking log on both the sides, V
\ Current through battery =
Rt R2
- = log 1 - log 2
L At t = ¥,
L 300 ´10 -3 R1R2
Þ t = log 2 = ´ 0.69 Þ t = 0.1 sec. effective resistance, Reff =
R 2 R1 + R2
V 100 V V ( R1 + R2 )
26. (c) Across resistor, I = = = 0.1 A
R 1000 \ Current through battery = =
Reff R1R2
At resonance,
31. (d) When capacitance is taken out, the circuit is LR.
1 1
X L = XC = = = 2500
wC 200 ´ 2 ´ 10-6 wL
\ tan f =
Voltage across L is R
I X L = 0.1 ´ 2500 = 250 V 1 200
Þ wL = R tan f = 200 ´ =
27. (a) Initially, when steady state is achieved, 3 3
Again , when inductor is taken out, the circuit is CR.
E
i= 1
R \ tan f =
Let E is short circuited at t = 0. Then wCR
E 1 1 200
At t = 0, i0 = Þ = R tan f = 200 ´ =
R wc 3 3
Let during decay of current at any time the current
2
Now, Z = R 2 + æç ö
di 1
flowing is - L - iR = 0 - wL ÷
dt è wC ø
Free eBooks on @neetquestionpaper2020

Alternating Current 813


32. (b) Given : Resistance R = 100 kilo ohm = 100 × W 103 2
æ 1 ö
–12
Capacitance C = 250 picofarad = 250 × 10 F 37. (a,c) We know that Z = R2 + ç
è WC ÷ø
t = RC = 100 × 103 × 250 × 10–12 sec
The capacitance in case B is four times the capacitance
= 2.5 × 107 × 10–12 sec in case A
= 2.5 × 10–5 sec \ Impedance in case B is less then that of case
The higher frequency which can be detected with A (ZB < ZA)
tolerable distortion is V
Now I =
1 1 Z
f= = Hz
2pm a RC 2p ´ 0.6 ´ 2.5 ´ 10 -5 \ I RA < I RB . option (a) is correct.

100 ´ 104 4 \ VRA < VRB .


= Hz = ´ 10 4 Hz = 10.61 KHz
25 ´ 1.2p 1.2p Þ VCA > VCB
This condition is obtained by applying the condition [Q If V is the applied potential difference access
that rate of decay of capacitor voltage must be equal
series R-C circuit then V = VR2 + VC2 ]
or less then the rate of decay modulated singnal
voltage for proper detection of modulated signal. \ (c) is the correct option.
33. (c) Charge on the capacitor at any time t is given by 38. (b) Time constant = RC
q = CV (1– et/t) æ 1 ö
2
Impedance = R2 + ç
at t = 2t è wC ÷ø
q = CV (1 – e–2) Given impedance = R 1.25
34. (b) Time constant of a R - C circuit is t = R eq C eq 2
2æ 1 ö
\ R 1.25 = R +ç
(i) t1 = (2 + 1)(2 + 4) = 18ms è wC ÷ø
2
æ 2 ´ 1 öæ 2 ´ 4 ö 8 2 æ 1 ö
(ii) t 2 = ç ÷ç ÷ = ms \ R (1.25) = R + ç
2
è 2 + 1 øè 2 + 4 ø 9 è wC ÷ø
2
æ 2 ´1 ö R2 æ 1 ö
(iii) t3 = ç ÷ ´ ( 4 + 2) = 4ms \ = ç
è wC ÷ø
è 2 +1ø 4
C R R 1
Vrms \ =
35. (b) I rms = 2 wC
2
æ 1 ö Irms 2 2
R2 + ç \ RC = = ´ 1000 ms
è wC ÷ø ~ w 500
As w increases, Irms increases. Therefore the bulb \ RC = 4 ms
glows brighter. N V 10 V
39. (a) For step up transformer s = s Þ = s
36. A-r,s,t; B-q,r,s,t; C-p,q; D-q,r,s,t N p Vp 1 4000
The following are the important concepts which are \ Vs = 40,000 V
applied in the given situation. Np Vp
For step down transformer =
(i) For DC circuit, in steady state, the current I through Ns Vs
the capacitor is zero. In case of L-C circuit, the
potential difference across the inductor is zero and 40,000 200
= =
that across the capacitor is equal to the applied 200 1
potential difference. In case of L-R circuit, the (a) is the correct option.
potential difference across inductor is zero across 40. (b) We know that P = V × I
resistor is equal to the applied voltage. P 600 ´ 1000
\ I= = = 150 A
(ii) For AC circuit in steady state, Irms current flows V 4000
through the capacitor, inductor and resistor. The Total resistance = 0.4 × 20 = 8 W
potential difference across resistor, inductor and \ Power dissipated as heat = I2R = (150)2 × 8
capacitor is proportional to I. = 180,000W = 180 kW
(iii) For DC circuit, for changing current, the potential 180
\ % loss = ´ 100 = 30%
difference across inductor, capacitor or resistor is 600
proportional to the current. (b) is the correct option.
Free eBooks on @neetquestionpaper2020

814 Ph y si cs
Exercise 3 : Conceptual & Applied MCQs
12. (b) We know that Ι r m s = Ι 0 / 2 and Ι m = 2 Ι 0 / p
1. (d) Power, P = Ι r.m.s ´ Vr.m.s ´ cos f
Ιm 2 2
In the given problem, the phase difference between \ =
voltage and current is p/2. Hence Ιrms p

P = Ι r.m.s ´ Vr.m.s ´ cos(p / 2) = 0. 1


13. (c) We know that f = ,
2 2 p (LC)
æ 1 ö
2. (c) Z = R 2 + çç w L - ÷ when C is doubled, L should be halved so that resonant
è w C ÷ø
frequency remains unchanged.
Here R = 100 W, L = 0.5 henry, C = 10 × 106 farad 14. (c) The phase angle is given by
w = 2 p p = 100 p. w L 2 p ´ 50 ´ 0.21
tan f = = = 5. 5
3. (b) V= V0 sin w t R 12
Voltage in r.m.s. value
f = tan -1 5.5 = 80º
V0 = 2 ´ 234 V = 331 volt Es I s 110 ´ 9
15. (c) h = \h= = 0.9 ´ 100% = 90%
and w t = 2 p n t = 2 p ´ 50 ´ t = 100 p t Ep I p 220 ´ 5

Thus, the equation of line voltage is given by 16. (c) 17. (a) 18. (c)
V = 331 sin (100 p t) 19. (a) 20. (b)
1
4. (a) X L = w L = 2 p n L 21. (a) At resonance, wL =
wC
XL 22 ´ 7 Hence the impedance of the circuit would be just equal
\ L= = H = 0.0175H to R (minimum). In other words, the LCR-series circuit
2 p n 2 ´ 22 ´ 200
will behave as a purely resistive circuit. Due to this the
5. (b) current is maximum. This condition is known as
resonance
1
6. (c) P = Vr.m.s ´ I r.m.s ´ cos f = V0 I 0 cos f
V
2 \ Z = R, Current =
R
1
= ´ 100 ´ (100 ´ 10 - 3 ) cos p / 3 = 2.5 W 22. (d) The transformer converts A.C. high voltage into A.C.
2 low voltage, but it does not cause any change in
frequency.
Es n æn ö
7. (a) = s or E s = E p ´ ç s ÷
Es Ns N 5000
Ep np ç np ÷ = Þ Es = s E p = ´ 20 = 200V
è ø Ep Np Np 500

æ 200 ö Thus output has voltage 200 V and frequency 50 Hz.


\ E s = 120 ´ ç ÷ = 240 V
è 100 ø 23. (a) E = 8 sin wt + 6 sin 2wt

Ιp ns æ np ö æ 100 ö Þ E peak = 82 + 6 2 = 10 V
= or Ι s = Ι p ç ÷ \ Is = 10 ç = 5 amp
Ιs np è ns ø è 200 ÷ø
10
np 80 20 E rms = =5 2 V
Ι 2
8. (c) s = ; = or Ι p = 4 amp.
Ιp ns Ιp 1
24. (b) Power of source = EI = 240 × 0.7 = 166
9. (a) 140
Þ Efficiency = Þ h = 83.3%
E æE ö 166
10. (b) Ι = = E w C = çç 0 ´ w C ÷÷
XC è 2 ø 25. (b) 26. (d) 27. (c) 28. (c)
29. (d) Impedence of a capacitor is XC = 1/wC
æ 200 ö
\ I = 120 ´ ç = 240V = 20 ´10 -3 amp.
è 100 ÷ø XC =
1
=
1
=
5000
.
2pfC 2p´ 50 ´ 2 ´ 10 - 6 p
11. (d)
Free eBooks on @neetquestionpaper2020

Alternating Current 815


30. (a) The phase angle between voltage V and current I is 1 q0 2
p/2. Therefore, power factor cos f = cos (p/2) = 0. Hence in L is zero and energy is stored in electric field.
2 C
the power consumed is zero.
Then capacitor begins to discharge through L causing
wL X L a current to flow and build up a magnetic field, around
31. (c) tan f = = L. Therefore, energy stored.
R R
Given f = 45°. Hence XL = R. 1 2
Now in L = LI0 when C is fully discharged, V
32. (b) 2
across the plate reduces to zero.
33. (c) The current drawn by inductor and capacitor will be in
\ Electric field energy is transferred to magnetic
opposite phase. Hence net current drawn from generator field and vice-versa.
= IL – IC = 0.9 – 0.4 = 0.5 amp. 38. (d) Initially there is no D.C. current in inductive circuit
and maximum D.C.current is in capacitive current.
Pi 4000
34. (d) As E p Ι p = Pi \ Ιp = = = 40 A. Hence, the current is zero in A2 and maximum in A1.
Ep 100
39. (a) L = 10 mHz = 10–2 Hz
35. (c) In the case of maximum charge on capacitor, the whole f = 1MHz = 106 Hz
energy is, stored in capacitor in the form of electric
1
field which is f=
2p LC
1 Q2
U= 1
2 C f2 = 2
4p LC
When energy in distributed equally between electric
and magnetic field , then energy stored in electric field 1 1 10 -12
ÞC= = = = 2.5 pF
2 2 -2 12 4
U æç 1 Q 2 ö÷ 1 4p f L 4 ´ 10 ´ 10 ´ 10
i.e. in capacitor is U1 = =
2 çè 2 C ÷ø 2 40. (c) Impedance at resonant frequency is minimum in series
LCR circuit.
At that time if charge on capacitor is Q1, then
2
2 æ 1 ö
1 Q12 U 1 Q 2 So, Z = R + ç 2pfL - ÷
U1 = = = Þ Q1 = Q / 2 è 2pfC ø
2 C 2 4 C
36. (d) L = 2H , E = 5 volts, R = 1W When frequency is increased or decreased, Z increases.

E 2v cos f
64 447
Z 4 4 48 41. (c) P =
Z
R L
(240) 2 (0.75)
P = 3000 = d Þ Z = 14.4W
Z
E
42. (c) 43. (c)
1 2 E 44. (a) The resistance in the middle plays no part in the charging
Energy in inductor = LI I=
process of C, as it does not alter either the potential
2 Z
difference across the RC combination or the current
5 5 through it.
I= =
R 2 + ( wL ) 1 + 4 p 2 ´ 50 2 ´ 4 45. (b) C discharges through both resistance in series.
2

46. (d) Option (d) is false because the reason why the voltage
5 5 1
= = leads the current is because > Lw and if the voltage
200p Cw
1 + ( 200p )
2
lags, the inductive reactance is greater than the
capacitive reactance.
1 5´5
Energy = ´2´ = 6.33 × 10–5 joules 47. (c)
2 200 ´ 200 p 2 48. (a) NP = 400, NS = 2000 and VS = 1000 V.
37. (a) In ideal condition of LC circuit R = 0 and LC oscillation
VP N P VS ´ N P 1000 ´ 400
continue indefinitely. Energy being shunted back and = of, VP = = = 200V.
forth between electric field of capacitor and magnetic VS NS NS 2000
field of inductor. As capacitor is fully charged current
Free eBooks on @neetquestionpaper2020

816 Ph y si cs
49. (d) The time constant of the circuit is 52.(d) In LCR series circuit, resonance frequency f 0 is given
by
L 100 ´10-3
t= = = 2 ´10-3 s= 2 milli second. 1 1 1
R 50 Lw = Þ w2 = \ w= = 2pf0
Cw LC LC
Current at time t is given by I = I0e–t/t
1 1
where I0 is the steady current. Therefore, time for I to \ f0 = or f 0a
fall to I0/2 is 2p LC C
When the capacitance of the circuit is made 4 times, its
1
e – t/ t = or, et/t = 2, t = tln(2) = 2ln(2) milli second. resonant frequency become f 0'
2
f0' C f0
N \ = or f 0' =
50. (a) Transformation ratio k = S f0 4C 2
NP
53. (b) 54. (c)
VS NS N æ1 2 ö
Since = \ VS = S ´ VP 55. (c) Av. electric field energy = çè CVrms ÷ø = 25 × 10–3 J
VP N P NP 2

3 1
VS = ´ 30 = 45V \ C × (Irms XC )
2 2
1 1
51. (b) Given equation, e = 80 sin 100pt …(i) \ × C.I2rms × –3
2 2 2 = 25 × 10 J
Standard equation of instantaneous voltage is given by 2 4p v c
e = em sinwt …(ii) \ C = 20mF
Compare (i) and (ii), we get em = 80 V FG 1 LI IJ
2

where em is the voltage amplitude.


56. (c) Av. magnetic energy H2 K rms

em 2 ´ 5 ´ 10 -3
Current amplitude Im = where Z = impendence \ L= Þ L =1 henry
Z (.10) 2
= 80/20 = 4 A. 57. (d) The sum for rms voltage across C, rms voltage across
R and rms voltage across L is not equal to rms voltage
4 4 2 across ideal ac source.
I r.m.s = = = 2 2 = 2.828 A. 58. (a) 59. (a) 60. (c)
2 2
Free eBooks on @neetquestionpaper2020

23
Electromagnetic
Waves
DISPLACEMENT CURRENT This law relates the electric field to charge distribution,
It is that current which comes into play in a region, where electric whereas electric field lines orignate from positive (+ive)
field (and hence the electric flux) is changing with time. charge and terminate on negative (–ive) charge. The
r r r
df differential form of this law is Ñ.E =
The displacement current is given by, I D = eo E
dt e0
where e o = absolute permittivity (or permitivity of free space) and where r is volume charge density and eo is the permittivity
dfE of free space.
= rate of change of electric flux. (ii) Gauss's law for magnetism : It states that the net magnetic
dt flux through a closed surface is zero. It means that number
In case of a steady electric flux linked with a region, the
of magnetic field lines that enter a closed volume must be
displacement current is maximum.
equal to number of field lines leaving that volume
The current in the electric circuit which arises due to flow of
r r
electrons in the connecting wires of the circuit, in a defined
closed path is called conduction current.
i.e., Ñò B.dA = 0
It means that magnetic mono pole cannot exists in nature.
MAXWELL’S MODIFICATION OF AMPERE’S CIRCUITAL r r
LAW The differential form of this law is Ñ.B = 0
(iii) Faraday’s law : This law states that the line integral of the
In 1864, Maxwell showed that Ampere’s circuital law is logically
electric field around any closed path (which equals the e.m.f.)
inconsistent for non-steady currents. He modified Ampere’s law equals the rate of change of magnetic flux through any
ur uuur df ö
Ñò
æ surface area bounded by that path.
as B.d l = m 0 ç I + e 0 E ÷ (Sum of conduction current and
è dt ø r r d fm
displacement current) i.e., Ñò E.d l = dt
The term I d = e 0 (df E / dt) is displacement current. It is that
The consequence of this law is that, if we keep any
current which comes into existence, in addition to the conduction conducting loop in a time varying magnetic field, then an
current, whenever the electric field and hence the electric flux induced current, flows in that conducting loop. The
changes with time. r r dB
Maxwell’s Equation differential form of this law is Ñ ´ E = -
dt
Maxwell found that all the basic principles of electromagnetism can This law decribes a relationship between an electric field
be formulated in terms of four fundamental equations, called and a changing magnetic flux.
Maxwell’s equation. These are : (iv) Modified Ampere-Maxwell law : It states that “ the line
(i) Gauss's law for electrostatics : According to Gauss's law integral of magnetic field around any closed path is
the total electric flux through any closed surface is equal to determined by the sum of the net conduction current through
the net charge inside that surface divided by eo that path and the rate of change of electric flux through any
surface bounded by that path
r uur
Ñò E.dA = eo
Q r r df
Ñò
i.e.,
i.e., B.dS = m0 I + e om0 e
dt
Free eBooks on @neetquestionpaper2020

818 Physi cs
It describes the relationship between magnetic and electric 2. The velocity of electromagnetic wave does not depend on
fields and electric current. The differential form of this law is amplitude of field vectors.
r r dE
3. The electric vector of an electromagnetic wave is responsible
Ñ ´ B = m0 J + m0 E0 for the optical effects & is called a light vector.
dt
4. In a plane electromagnetic wave, the average energy
ELECTROMAGNETIC WAVES densities of electric and magnetic fields are equal.
Maxwell-on the basis of four basic equations of electromagnetism In vacuum, the average electric energy density is given by,
theoretically predicted the existence of electromagnetic waves.
1 1 1
An electromagentic wave is the one constituted by oscillating uE = e o E 2 = e o ( E0 / 2) 2 = e o E02
electric and magnetic fields which oscillate in two mutually 2 2 4
perpendicular planes. The wave itself propagates in a direction The average magnetic energy density is given by
perpendicular to both of the directions of oscillations of electric
B 2 ( B0 / 2) 2 B2
and magnetic fields. uB = = = 0
2m 0 2m 0 4m 0
Properties of Electromagnetic Waves
(i) The direction of oscillations of E and B fields are 1 2 1 2 1 2 2
perpendicular to each other as well as to the direction of Also, u E = e o E0 = e o (c B0 ) = e oc B0
4 4 4
propagation.Electromagnetic waves are transverse in nature.
(ii) The electric and magnetic field oscillate in same phase. 1 e o B02 B2
= = 0 = uB
(iii) The electromagnetic waves travel through vacuum with the 4 e o m 0 4m 0
same speed of light
\ Total average energy density
1 8 -1
c= = 3 ´ 10 ms 1 B2
m0 eo = uE + uB = 2uE = 2uB = e o E02 = 0
2 2m 0
1
(iv) The energy density of electric field is e o E 2 and that of 5. Poynting vector: When an electromagnetic wave advances,
2
the electromagnetic energy flows in the direction E ´ B .
2
1B The total energy flowing perpendicularly per second per
magnetic field is , so the energy density of the
2 m0 unit area into the space in free space is called a poynting
ur
vector S where S = ce o ( E ´ B ) = (E ´ B ) / m 0 .
1é 2 B

electromagnetic wave is u = êeo E + ú uur
2 êë m 0 úû 1 EB
S =|S | = E B sin 90° =
m0 m0
where E and B are the instantaneous values of the electric
and magnetic field vectors. The SI unit of S is watt/m2
6. The intensity of a sinusoidal plane electromagnetic wave is
E 1
(v) The ratio =c= . defined as average value of poynting vector taken over one
B m0 eo cycle. Thus,
Production of Electromagnetic Waves 1 æ E0 ö æ B0 ö E0 B0 E0 2 c B0 2
I = Sav = = = =
An accelerated charge emits electromagnetic waves. An oscillating m 0 çè 2 ÷ø çè 2 ÷ø 2 m0 2 m 0c 2 m 0
charge, as in an LC-circuit has non-zero acceleration, it continues
to emit electromagnetic waves. The frequency of electromagnetic 7. Radiation pressure : The pressure exerted by electromagnetic
waves is the same as that of the oscillating charge. waves is called as radiation pressure (P). When an
r
Hertz’s experiment : In 1888, Hertz succeeded in experimentally electromagnetic wave with Poynting vector S is incident on a
confirming the existence of electromagnetic waves. By using perfectly absorbing surface, then radiation pressure on surface
oscillator LC-circuits, he not only produced and detected r
electromagnetic waves, but also demonstrated their properties of S
is P = .
reflection, refraction and interference and established beyond c
doubt that light radiation has wave nature.
ELECTROMAGNETIC SPECTRUM
Keep in Memory
All the known radiations form a big family of electromagnetic
1. The amplitudes of electric and magnetic fields in free space, waves according to frequency or wavelength. We call this family
E0 as the complete electromagnetic spectrum. It includes: Gamma
in electromagnetic waves are related by E0 = cB0 or C = rays, X-rays, ultraviolet light, visible light, infrared light,
B0
microwaves and radio waves.
Free eBooks on @neetquestionpaper2020

Electromagnetic Waves 819


Wavelength range Frequency range
S.No. Name Source Detection
(in m) (in Hz)
1 Gamma rays 6 × 10
–14
to 1 × 10
–11 22
5 × 10 to 3 × 10
19 Nuclear origin Photographic film,
Geiger tubes,
ionisation chamber
2 X-rays 1 × 10
–11 –8
to 3 × 10
19
3 × 10 to 1 × 10
16 Sudden deceleration of Photographic film,
high energy electron Geiger tubes,
(collision of electrons ionisation chamber
with target)
3 Ultra-violet –8
6 × 10 to 4 × 10
–7 17
5 × 10 to 8 × 10
14 Excitation of atom, Photocells,
spark and arc lamp photographic film

4 Visible light –7
4 × 10 to 7 × 10
–7 14
8 × 10 to 4 × 10
14 Excitation of valency The eye, photocells,
electron photographic film
5 Infrared –7
7 × 10 to 3 × 10
–5 14
4 × 10 to 1 × 10
13 Excitation of atoms and Thermopiles, bolometer
molecules
6 Heat radiations –5
10 to 10
–1 13
3 × 10 to 3 × 10
9 Hot bodies
7 Micro-waves –3
10 to 0.3
11
3 × 10 to 1 × 10
9 Oscillating current in Point contact diodes
special vacuum tube
8 Ultra high –1
1 × 10 to 1
9 8
3 × 10 to 3 × 10 Oscillating circuit
frequency
9 Very high radio 1 to 10 8 7
3 × 10 to 3 × 10 Oscillating circuit
frequency
10 Radio frequency 10 to 104 7 4
3 × 10 to 3 × 10 Oscillating circuit Receiver's aerial
11 Power 5 × 106 to 6 × 106 60 to 50 Weak radiations from
frequency a.c. circuit

Example 1. 2 pc 2p ´ 3 ´ 108
In a plane electromagnetic wave the electric field varies w = 2pn = = = p ´ 1011 rad / s.
l 6 ´ 10 -3
with time having an amplitude 1 Vm–1. The frequency of The maximum magnetic field,
wave is 0.5 × 1015 Hz. The wave is propagating along
E 33
Z-axis. What is the average energy density of (i) electric B0 = 0 = = 11´ 10 -8 T
c 8
field (ii) magnetic field (iii) total field. (iv) What is amplitude 3 ´ 10
The equation for the electric field, along y-axis in the
of magnetic field?
electromagnetic wave is
Solution :
(i) Average energy density of electric field æ xö
E y = E 0 sin wç t - ÷ = 33 sin p ´ 1011 ( t - x / c)
2
é E0 ù è cø
1 1 æ E0 ö
êQ E =
2 ç ÷
U E = eoE = eo ç ÷ ú The equation for the magnetic field along z-axis in the
2 2 è 2ø ë 2û electromagnetic wave is
1 1
= e o E 02 = ´ (8.854 ´10 -12 ) ´12 = 2.21´ 10 -12 Jm -3 . æ xö
4 4 B z = B 0 sin wç t - ÷ = 11´ 10 -8 sin p ´ 1011 ( t - x / c)
(ii) Average energy density of magnetic field, è cø
UB = Average energy density of electric field Example 3.
= 2.21 × 10–12 Jm–3 If ε o and m0 represent the permittivity and permeability of
(iii) Total average energy density U = UE + UB = 2 UE vacuum and ε and m represent the permittivity and
= 2 × 2.21 ×10–12 = 4.42 × 10–12 Jm–3 permeability of medium, then refractive index of the medium
(iv) Amplitude of magnetic field, B0 = E0/c is given by
= 1/(3 × 108) = 3.33 × 10–9 T μ 0ε o με ε μ 0ε o
Example 2. (a) (b) μ 0ε o (c) μ 0ε o (d)
με μ
A plane electromagnetic wave propagating in the
Solution : (b)
x-direction has a wavelength of 6.0 mm. The electric field
is in the y-direction and its maximum magnitude is 33 Vm–1. 1
Q Velocity of light in medium c =
Write suitable equations for the electric and magnetic fields me
as a function of x and t. c 0 1/ m0 eo me
Solution : \ Refractive index = = =
c 1 / me m 0e o
Here, l = 6.00 mm = 6 × 10–3 m; E0 = 33 V/m.
Free eBooks on @neetquestionpaper2020

820 Physi cs
Example 4. Example 5.
The magnetic field in a plane electromagnetic wave is given A plane electromagnetic wave of frequency 25 MHz travels
by B = 2 × 10–7 sin (0.5× 103 x +1.5 × 1011t) T. in free space along the x-direction. At a particular point in
(a) What is the wavelength and frequency of the wave? r r
(b) Write an expression for the electric field. space and time, E = 6.3 ˆj V / m . What is B at this point ?
Solution : Solution :
(a) Comparing the given equation with r
The magnitude of B is
é æ x t öù
B = B0 sin ê 2p ç + ÷ ú E 6.3 V / m
ë è l Tøû B= = = 2.1 ´ 10-8 T
2p c 3 ´ 108 m / s
we get, l = m = 1.26 cm
0.5 ´ 103 r
To find the direction, we note that E is along y-direction
1
and = n = (1.5 ´ 1011 ) / 2p = 23.9 GHz and the wave propagates along x-axis. Therefore, B should
T
(b) E0 = B0c = 2 × 10–7 T × 3 × 108 m/s = 6 × 101 V/m be in a direction perpendicular to both x- and y-axes. Using
r r
The electric field component is perpendicular to the vector algebra, E ´ B should be along x-direction.
direction of propagation and the direction of magnetic r r
Since, (+ ˆj) ´ (+ k)
ˆ = i.B
ˆ , B is along the z-direction.
field. Therefore, the electric field component along the z-
axis is obtained as r
Thus, B = 2.1 ´ 10 -8 kT
ˆ
Ez = 60 sin (0.5 × 103x + 1.5 × 1011 t) V/m

23.1
Solve following problems with the help of above text and 8. Displacement current is
examples. (a) continuous when electric field is changing in the circuit
1. The electromagnetic waves (b) continuous when magnetic field is changing in the
(a) travel with the speed of sound circuit
(b) travel with the same speed in all media (c) continuous in both types of fields
(d) continuous through wires and resistance only
(c) travel in free space with the speed of light
9. Microwaves are detected by
(d) do not travel through a medium (a) balometer (b) point contact diodes
2. The displacement current was first postulated by (c) thermopiles (d) the eye
(a) Maxwell (b) Marconi r r
(c) Ampere (d) Hertz 10. If E and B are the electirc and magnetic field vectors of
e.m. waves then the direction of propagation of e.m. wave
3. Which of the following type of radiations are radiated by
is along the direction of
an oscillating electric charge? r r
(a) Electric (b) Magnetic (a) E (b) B
r r
(c) Thermoelectric (d) Electromagnetic (c) E ´ B (d) None of these
4. The speed of electromagnetic wave is same for 11. Maxwell’s modified form of Ampere’s circuital law is
(a) odd frequencies (b) even frequencies ® ®
(c) all frequencies (d) all intensities (a) ò B . ds = 0
5. A plane electromagnetic wave is incident on a material s
® ®
surface. If the wave delivers momentum p and energy E,
then
(b) ò B . dl = m 0I
s
(a) p = 0, E = 0 (b) p ¹ 0, E ¹ 0 ® ® 1 dq
(c) p ¹ 0, E = 0 (d) p = 0, E ¹ 0 (c) ò B . dl = m 0 I + e odt
6. Ampere’s circuital law holds good for s
(a) conduction current (b) displacement current ® ® d fE
(c) both (a) and (b) (d) None of these (d) ò B . dl = m 0I + m 0 e o dt
7. Conduction current flows s
12. According to Maxwell’s equation the velocity of light in
(a) only through resistance
any medium is expressed as
(b) through wires and resistance
(c) only through capacitor 1 1 m0
(d) through wires, resistance and capacitor (a) m0 eo (b) m e (c) m / e (d)
e
Free eBooks on @neetquestionpaper2020

Electromagnetic Waves 821


13. The displacement current is 21. The frequencies of X-rays, g-rays and ultraviolet rays are
eo respectivley a, b and c. Then
(a) e o d f E / dt (b) d f E / dt (a) a < b, b > c (b) a > b, b > c
R
(c) a > b, b < c (d) a < b, b < c
(c) eo E / R (d) e o q C / R 22. In electromagnetic wave, the average energy density is
14. The electromagnetic waves do not transport associated
(a) energy (b) charge (a) with electric field only
(c) momentum (d) information (b) with magnetic field only
15. The amplitudes of electric and magnetic fields related to (c) equally with electric and magnetic fields
each other are (d) None of these
(a) E 0 = B0 (b) E 0 = c B0 23. The range of wavelength of visible light is
(a) 10 Å to 100 Å (b) 4000 Å to 8000 Å
B c (c) 8000 Å to 10,000 Å (d) 10,000 Å to 15,000 Å
(c) E 0 = 0 (d) E 0 =
c B0 24. Which of the following are not electromagnetic waves?
16. In an electromagnetic wave, the direction of the magnetic (a) Cosmic rays (b) Gamma rays
r (c) b-rays (d) X-rays
induction B is
r 25. If vs, vx and vm are the speed of soft gamma rays, X-rays
(a) parallel to the electric field E and microwaves respectively in vacuum, then
r
(b) perpendicular to the electric field E (a) vs > vx > vm (b) vs < vx < vm
r
(c) antiparallel to the Poynting vector S (c) vs > vx < vm (d) vs = vx = vm
(d) random 26. Which of the following electromagnetic radiations have
17. Which of the following electromagnetic waves has the the smallest wavelength?
longest wavelength? (a) Ultraviolet waves (b) X-rays
(a) Heat waves (c) g-rays (d) Microwaves
(b) Visible light 27. Which of the following rays has minimum frequency?
(b) Radio frequency waves (a) U.V. ways (b) X-rays
(d) Microwaves (c) g-rays (d) Infra-red rays
18. Which of the following is of shortest wavelength? 28. An accelerated electron would produce
(a) X-rays (b) g-rays (a) g-rays (b) b-rays
(c) Microwaves (d) Radio waves (c) a-rays (d) e.m. rays
19. Radio waves with frequencies higher than television 29. Which of the following is the infrared wavelength?
signals are (a) 10–4 cm (b) 10–5 cm (c) 10–6 cm (d) 10–7 cm
30. According to Maxwell’s hypothesis, a changing electric
(a) ultrasonic waves (b) sound waves
field gives rise to
(c) light waves (d) microwaves
(a) an e.m.f
20. Maxwell’s equation describe the fundamental laws of
(b) electric displacement current
(a) electricity (b) magnetism (c) magnetic field
(c) mechanics (d) both (a) and (b) (d) pressure gradient

ANSWER KEY
1. (c) 2. (a) 3. (d) 4. (d) 5. (b) 6. (c) 7. (d) 8. (a) 9. (b) 10. (c) 11.(d) 12.(b)
13.(a) 14.(b) 15.(b) 16.(b) 17.(b) 18.(b) 19.(d) 20.(d) 21.(a) 22.(c) 23.(b) 24.(a)
25.(d) 26.(c) 27.(c) 28.(d) 29.(a) 30.(c)

USES OF ELECTROMAGNETIC WAVES 4. Ultraviolet radiations are used in the detection of invisible
The following are some of the uses of electromagnetic waves writing, forged documents, finger prints in forensic
1. Radio waves are used in radio and T.V. communication laboratory and to preserve the food stuffs.
systems. 5. Thestudy of infrared, visible and ultraviolet radiations help
2. Microwaves are used in microwave oven. us to know through spectra, the structure of the molecules
3. Infrared radiations are used (a) in revealing the secret and arrangement of electrons in the external shells.
writings on the ancient walls (b) in green houses to keep the 6. X-rays can pass through flesh and blood but not through
plants warm (c) in warfare, for looking through haze, fog or bones. This property of X-rays is used in medical diagnosis,
mist as these radiations can pass through them. after X-rays photographs are made.
Free eBooks on @neetquestionpaper2020

822 Physi cs
The study of X-rays has revealed the atomic structure and transmission is possible only with waves of wavelengths
crystal structure. above 200 m or frequencies below 1500 k Hz.
7. The study of g-rays provides us valueable information (ii) Sky wave propagation : When a radiowave is directed
about the structure of the atomic nuclei towards the sky and is reflected by the ionosphere towards
8. Super high frequency electromagnetic waves (3000 to desired location on the earth, it is called sky wave. This
30,000 MHz) are used in radar and satellite communication. method is useful for the transmission of waves of
wavelengths less than 200 m or frequencies above 1500 k
9. Electromagnetic waves (frequency 50 to 60 Hz) are ued for
Hz upto 30 MHz.
lighting. These are weak waves having wavelength
(iii) Space wave propagation : For the transmission of television
5 × 106 to 6 × 106 m and can be produced from A.C. circuits.
signals (frequencies in the range 100-200 M Hz), space
Keep in Memory wave propagation method is used, in which the wave travels
directly from a high transmitting antenna to the receiving
1. Green house effect : It is the phenomenon which keeps the antenna. The relation between the height ‘h’ of the
earth’s surface warm at night. The earth absorbs solar radiation and transmitting antenna above the ground level and the distance
reflects back only infrared rays due to its low temperature. These ‘d’ upto which TV signal can be received is d = 2hR
rays are reflected back by the clouds and the gas molecules of the = range of a TV tower where R is the radius of the earth.
lower atmosphere. This keeps the earth’s surface warm at night. Example 6.
A TV tower has a height of 100 m. How much population
PROPAGATION OF RADIO WAVES THROUGH THE is covered by the TV broadcast if the average population
ATMOSPHERE density around the tower is 1000 km–2 ?(radius of the
It takes place in three ways : earth = 6.37 × 106 m)
(i) Ground wave propagation, Solution :
(ii) Sky wave propagation and Height of twoer h = 100 m
(iii) Space wave propagation. Radius of the earth R = 6.37 × 106m
(i) Ground wave propagation : When the radio wave travel
d = 2 hR = 2 ´ 100 ´ 6.37 ´ 106
directly from one point to another following the surface of Population covered = p d 2 × population density
the earth, it is called ground or surface wave. This type of Solving we get population covered = 40 lakh.

23.2
Solve following problems with the help of above text and 5. The wavelength of X-rays is of the order of
examples (a) 1 metre (b) 1 cm
1. Television signals pass only through the ground waves. (c) 1 micron (d) 1 angstrom
The range R related with the transmitter height h is 6. Which of the following is the longest wave?
(a) h (b) h 1/2
–1/2
(a) X-rays (b) g-rays
(c) h (d) h –1
(c) Microwaves (d) Radio waves
2. Transmission via sky waves is impossible for the
7. Radio waves of constant amplitude can be generated with
frequencies
(a) rectifier (b) filter
(a) less than 40 MHz (b) equal to 40 MHz
(c) more than 40 MHz (d) all of the above (c) F.E.T. (d) oscillator
3. It is necessary to use satellites for long distance TV 8. Radio waves do not penetrate in the band of
transmission because (a) ionosphere (b) mesosphere
(a) TV signal has short wavelength (c) troposphere (d) stratosphere
(b) TV signal has long wavelength 9. A radar sends the waves towards a distant object and
(c) TV signal has light signal receives the signal reflected by object. These waves are
(d) TV signals are not reflected by ionosphere (a) sound waves (b) light waves
4. The process of superimposing signal frequency (i.e. audio (c) radio waves (d) microwaves
wave) on the carrier wave is known as 10. What is the cause of “Green house effect”?
(a) transmission (b) reception (a) Infrared rays (b) Ultraviolet rays
(c) modulation (d) detection (c) X-rays (d) Radio waves

ANSWER KEY
1. (b) 2. (c) 3. (d) 4. (c) 5. (d) 6. (d) 7. (d) 8. (a) 9. (d) 10.(a, b)
Free eBooks on @neetquestionpaper2020

Electromagnetic Waves 823

Very Short / Short Answer Questions 16. The amplitude of the magnetic field of a harmonic
electromagnetic wave in vacuum is B0 = 510 nT. What is the
1. Write the frequency limit of the visible range of amplitude of the electric field part of the wave ?
electromagnetic spectrum in kHz.
17. (a) How does an oscillating charge produce electro-
2. What is the relationship between amplitudes of electric magnetic wave? Explain
and magnetic fields in free space?
[Delhi Board - 2012 COMPTT.].
3. What do you mean by conduction current?
(b) Draw a sketch showing the propagation of a plane e.m.
4. Can the displacement current act be a source of magnetic
wave along the Z-direction, clearly depicting the
field?
directions of oscillating electric and magnetic field
5. What do electromagnetic waves consist of ? Explain on vectors.
what factors does its velocity in vacuum depend.
Multiple Choice Questions
6. Give two characteristics of electromagnetic waves. Write
the expression for velocity of electromagnetic waves in 18. The electromagnetic radiation used in food processing
terms of permittivity and permeability of the medium. sterilizing agent is
7. Find the wavelength of electromagnetic waves of frequency (a) microwaves
5 × 109 Hz in free space. Give its two applications. (b) UV rays
8. Find the wavelength of electromagnetic waves of frequency (c) gamma rays
4 × 109 Hz in free space. Give its two applications.
(d) radio waves
9. The speed of an electromagnetic wave in a material medium
19. An electromagnetic wave is made up of joint electric (E) and
1 magnetic (B) fields. Its direction of propagation is
is given by v = , µ being the permeability of the (a) parallel to E
me
(b) perpendicular to E but parallel to B
medium and e its permittivity. How does its frequency
change? [Outside Delhi - 2010] (c) parallel to B
10. What are the directions of electric and magnetic field (d) perpendicular to both E and B
vectors relative to each other and relative to the direction 20. Intensity of electromagnetic wave will be
of propagation of electromagnetic waves? (a) I = cm 0 B02 / 2 (b) I = ce 0 B20 / 2
[Outside Delhi - 2010]
11. Name the physical quantity which remains same for (c) I = B20 / cm 0 (d) I = E 02 / 2ce 0
microwaves of wavelength 1 mm and UV radiations of 1600
21. The frequency of electromagnetic wave, which best suited
Å in vacuum. [Delhi Board - 2012]
to observe a particle of radii 3 × 10–4 cm is of the order of
12. Describe briefly how electromagnetic waves are produced
(a) 1015 (b) 1014
by oscillating charges. 13
(c) 10 (d) 1012
[Outside Delhi - 2011 COMPTT.].
13. Give one use of each of the following 22. If l = 10 Å then it corresponds to
(i) Microwaves [Delhi Board - 2011 COMPTT.]. (a) infrared (b) microwaves
(ii) Ultraviolet rays (c) ultraviolet (d) X-rays
(iii) Infrared rays 23. 10 cm is a wavelength corresponding to the spectrum of
(iv) Gamma rays (a) infrared rays (b) ultra-violet rays
14. What is the frequency of electromagnetic waves produced (c) microwaves (d) g -rays
by oscillating charge of frequency n? 24. In an electromagnetic wave
[Delhi Board - 2011 COMPTT.]. (a) power is transmitted along the magnetic field
Long Answer Questions (b) power is transmitted along the electric field
(c) power is equally transferred along the electric and
15. Describe Hertz’s experiment for the production and detection magnetic fields
of electromagnetic waves. (d) power is transmitted in a direction perpendicular to
both the fields
Free eBooks on @neetquestionpaper2020

824 Physi cs

r r (c) the ratio of magnetic permeability to the electric


1. If E and B represent electric and magnetic field vectors of
the electromagnetic waves, then the direction of susceptibility of vacuum
propagation of the waves will be along [CBSE PMT 2002] (d) unity
r r r r r r 9. The condition under which a microwave oven heats up a
(a) B ´ E (b) E (c) B (d) E ´ B food item containing water molecules most efficiently is
2. If l v , l r and l m represent the wavelength of visible light [NEET 2013]
x-rays and microwaves respectively, then (a) the frequency of the microwaves has no relation with
[CBSE-PMT 2005] natural frequency of water molecules.
(a) l m > l x > l v (b) l v > l m > l y (b) microwaves are heat waves, so always produce heating.
(c) infra-red waves produce heating in a microwave oven.
(c) l m > l v > l x (d) l v > l x > l m
(d) the frequency of the microwaves must match the
3. The electric and magnetic field of an electromagnetic wave resonant frequency of the water molecules.
are in [CBSE-PMT 2007] 10. Which of the following are not electromagnetic waves?
(a) phase and parallel to each other
(a) cosmic rays (b) g-rays [AIEEE 2002]
(b) opposite phase and perpendicular to each other
(c) b-rays (d) X-rays.
(c) opposite phase and parallel to each other
11. Which of the following radiations has the least wavelength ?
(d) phase and perpendicular to each other
4. Which of the following statement is false for the properties (a) g -rays (b) b -rays [AIEEE 2003]
of electromagnetic waves? [CBSE-PMT 2010] (c) a -rays (d) X -rays
(a) Both electric and magnetic field vectors attain the 12. An electromagnetic wave of frequency n = 3.0 MHz
maxima and minima at the same place and same time. passes from vacuum into a dielectric medium with
(b) The energy in electromagnetic wave is divided equally
permittivity e = 4.0 . Then [AIEEE 2004]
between electric and magnetic vectors
(c) Both electric and magnetic field vectors are parallel to (a) wave length is halved and frequency remains
each other and perpendicular to the direction of unchanged
propagation of wave (b) wave length is doubled and frequency becomes half
(d) These waves do not require any material medium for (c) wave length is doubled and the frequency remains
propagation. unchanged
5. The electric and the magnetic field associated with an E.M. (d) wave length and frequency both remain unchanged.
wave, propagating along the +z-axis, can be represented by 13. The intensity of gamma radiation from a given source is I.
I
[CBSE-PMT 2011] On passing through 36 mm of lead, it is reduced to . The
r r r r r 8
ˆ B = B ˆjù
(a) éë E = E 0 i, (b) éë E = E 0 k, B = B0ˆi ùû I
0 û
r r r r thickness of lead which will reduce the intensity to will
2
(c) éë E = E 0 ˆj, B = B0 iˆ ùû (d) éë E = E 0 ˆj,B = B0 kˆ ùû be [AIEEE 2005]
6. The decreasing order of wavelength of infrared, microwave, (a) 18mm (b) 12mm (c) 6mm (d) 9mm
ultraviolet and gamma rays is [CBSE-PMT 2011] 14. The rms value of the electric field of the light coming from
(a) microwave, infrared, ultraviolet, gamma rays the Sun is 720 N/C. The average total energy density of the
(b) gamma rays, ultraviolet, infrared, micro-waves electromagnetic wave is [AIEEE 2006]
(c) microwaves, gamma rays, infrared, ultraviolet (a) 4.58 × 10–6 J/m3 (b) 6.37 × 10–9 J/m3
(d) infrared, microwave, ultraviolet, gamma rays (c) 81.35 × 10–12 J/m3 (d) 3.3 × 10–3 J/m3
7. The electric field associated with an e.m. wave in vacuum is 15. An electromagnetic wave in vacuum has the electric and
r r r
given by E = iˆ 40 cos (kz – 6 × 108t), where E, z and t are in magnetic field E and B , which are always perpendicular to
r
volt/m, meter and seconds respectively. The value of wave each other. The direction of polarization is given by X and
r
vector k is [CBSE-PMT 2012S] that of wave propagation by k . Then [AIEEE 2012]
(a) 2 m–1 (b) 0.5 m–1 r r r r r r r r r r
(a) X || B and k || B ´ E (b) X || E and k || E ´ B
(c) 6 m–1 (d) 3 m–1 r r r r r r r r r r
8. The ratio of amplitude of magnetic field to the amplitude of (c) X || B and k || E ´ B (d) X || E and k || B ´ E
electric field for an electromagnetic wave propagating in 16. The magnetic field in a travelling electromagnetic wave has
vacuum is equal to [CBSE-PMT 2012M] a peak value of 20 nT. The peak value of electric field strength
(a) the speed of light in vacuum is [JEE Main 2013]
(b) reciprocal of speed of light in vacuum (a) 3 V/m (b) 6 V/m (c) 9 V/m (d) 12 V/m
Free eBooks on @neetquestionpaper2020

Electromagnetic Waves 825

1. If a source is transmitting electromagnetic wave of frequency 10. The ratio of electric field vector E and magnetic field vector
8.2 × 106 Hz, then wavelength of the electromagnetic waves æEö
transmitted from the source will be H i.e., çç H ÷÷ has the dimensions of
è ø
(a) 36.6 m (b) 40.5 m (a) resistance
(c) 42.3 m (d) 50.9 m (b) inductance
2. In an apparatus, the electric field was found to oscillate with (c) capacitance
an amplitude of 18 V/m. The magnitude of the oscillating
(d) product of inductance and capacitance
magnetic field will be
(a) 4 × 10–6 T (b) 6 × 10–8 T 11. The energy of electromagnetic wave in vacuum is given by
–9 the relation
(c) 9 × 10 T (d) 11 × 10–11 T
3. A TV tower has a height of 100m. How much population is E2 B2 1 1
(a) + (b) e 0E 2 + µ0B2
covered by the TV broadcast if the average population 2e 0 2µ 0 2 2
density around the tower is 100 km–2 (radius of the earth
= 6.37 × 106 m) E 2 + B2 1 B2
(c) (d) e0E 2 +
(a) 4 lakh (b) 4 billion c 2 2µ0
(c) 40,000 (d) 40 lakh 12. An electromagnetic wave is propagating along Y-axis. Then
4. In a plane electromagnetic wave propagating in space has (a) oscillating electric field is along X-axis and
an electric field of amplitude 9 × 103 V/m, then the amplitude oscillating magnetic field is along Y-axis
of the magnetic field is (b) oscillating electric field is along Z-axis and
(a) 2.7 × 1012 T (b) 9.0 × 10–3 T oscillating magnetic field is along X-axis
(c) 3.0 × 10 T –4 (d) 3.0 × 10–5 T (c) both oscillating electric and magnetic fields are
5. Electromagnetic waves are transverse in nature is evident along Y-axis, but phase difference between them
by is 90°
(a) polarization (b) interference (d) both oscillating electric and magnetic fields are
(c) reflection (d) diffraction mutually perpendicular in arbitrary direction
6. If l = 10 Å then it corresponds to 13. The frequency 2 MHz belongs to
(a) infrared (b) microwaves (a) visible light (b) X-rays
(c) ultraviolet (d) X-rays (c) microwaves (d) radiowaves
7. In an electromagnetic wave, the amplitudes of magnetic field 14. In an electromagnetic wave, the electric and magnetic fields
Ho and electric field Eo in free space are related as : are 100 V m–1 and 0.265 A m–1. The maximum energy flow is
(a) 26.5 W/m2 (b) 36.5 W/m2
Eo 2
(a) Ho = Eo (b) H= (c) 46.7 W/m (d) 765 W/m2
c 15. The frequency of electromagnetic wave, which best suited
to observe a particle of radii 3 × 10–4 cm is of the order of
eo
(c) H o = Eo m o e o (d) Ho = E o (a) 1015 (b) 1014
mo 13
(c) 10 (d) 1012
8. In an electromagnetic wave 16. 10 cm is a wavelength corresponding to the spectrum of
(a) power is transmitted along the magnetic field (a) infrared rays (b) ultra-violet rays
(b) power is transmitted along the electric field (c) microwaves (d) g -rays
(c) power is equally transferred along the electric and 17. If e0 and m0 are the electric permittivity and magnetic
magnetic fields permeability in vacuum, e and m are corresponding
(d) power is transmitted in a direction perpendicular to quantities in medium, then refractive index of the medium is
both the fields
9. The ionosphere e e 0m e 0m 0 em
(a) (b) (c) (d)
(a) reflects back radiowaves in the AM band e0 em 0 em e 0m 0
(b) reflects back radiowaves in the FM band 18. The ozone layer absorbs radiation of wavelengths
(c) absorbs radiowaves in the AM band (a) less than 3 × 10–7 m (b) more than 3 × 10–7 m
(d) absorbs radiowaves in the FM band (c) less than 3 × 10–5 m (d) more than 3 × 10–5 m
Free eBooks on @neetquestionpaper2020

826 Physi cs
19. A wave is propagating in a medium of electric dielectric 30. An electromagnetic wave, going through vacuum is described
constant 2 and relative magnetic permeability 50. The wave by E = E 0 sin(kx - wt) . Which of the following is
impedance of such a medium is
independent of wavelength ?
(a) 5 W (b) 376.6 W (c) 1883 W (d) 3776 W (a) k (b) w
20. The frequency modulated waves are (c) k/w (d) kw
(a) reflected by atmosphere 31. Which of the following shows green house effect ?
(b) absorbed by atmosphere (a) Ultraviolet rays (b) Infrared rays
(c) bend by atmosphere (c) X-rays (d) None of these
(d) radiowaves 32. Radio waves diffract around building although light waves
21. The relation between electric field E and magnetic field H in do not. The reason is that radio waves
an electromagnetic wave is (a) travel with speed larger than c
µ0 (b) have much larger wavelength than light
(a) E = H (b) E= H
e0 (c) are not electromagnetic waves
(d) None of these
µ0 e
(c) E= H (d) E = 0 H 33. Radio waves and visible light in vacuum have
e0 µ0 (a) same velocity but different wavelength
22. To double the covering range of a T.V. transmitter tower, its (b) continuous emission spectrum
height should be made (c) band absorption spectrum
(a) 2 times (b) 4 times (d) line emission spectrum
(c) 2 times (d) 8 times 34. In which one of the following regions of the electromagnetic
spectrum will the vibrational motion of molecules give rise to
23. The transmitting antenna of a radiostation is mounted
absorption ?
vertically. At a point 10 km due north of the transmitter the
peak electric field is 10–3 Vm–1. The magnitude of the radiated (a) Ultraviolet (b) Microwaves
magnetic field is (c) Infrared (d) Radio waves
(a) 3.33 × 10–10 T (b) 3.33 × 10–12 T 35. All components of the electromagnetic spectrum in vacuum
–3 have the same
(c) 10 T (d) 3 × 105 T
(a) energy (b) velocity
24. It is possible to take pictures of those objects which are not
fully visible to the eye using camera films sensitive to (c) wavelength (d) frequency
(a) ultraviolet rays (b) infrared rays 36. The oscillating electric and magnetic vectors of an
electromagnetic wave are oriented along
(c) microwaves (d) radiowaves
(a) the same direction but differ in phase by 90°
25. The wave impendance of free space is
(b) the same direction and are in phase
(a) zero (b) 376.6 W (c) 33.66 W (d) 3.76 W (c) mutually perpendicular directions and are in phase
26. The electromagnetic waves travel with a velocity (d) mutually perpendicular directions and differ in phase
(a) equal to velocity of sound by 90°
(b) equal to velocity of light 37. Approximate height of ozone layer above the ground is
(c) less than velocity of light (a) 60 to 70 km (b) 59 km to 80 km
(d) None of these (c) 70 km to 100 km (d) 100 km to 200 km
27. The speed of electromagnetic wave in vacuum depends 38. Biological importance of ozone layer is
upon the source of radiation. It (a) it stops ultraviolet rays
(a) increases as we move from g-rays to radio waves (b) ozone layer reduce green house effect
(b) decreases as we move from g-rays to radio waves (c) ozone layer reflects radio waves
(c) is same for all of them (d) ozone layer controls O2 / H2 radio in atmosphere
(d) None of these 39. The frequencies of X-rays, g-rays and ultraviolet rays are
28. Maxwell in his famous equation of electromagnetism respectively a, b and c then
introduced the concept of (a) a < b, b > c (b) a > b, b > c
(a) a.c. current (b) d.c. current (c) a > b, b < c (d) a < b, b < c
(c) displacement current (d) impedance 40. An electromagnetic wave going through vacuum is
29. A plane electromagnetic wave is incident on a material described by E = E0sin(kx – wt); B = B0 sin (kx – wt). Which
surface. If the wave delivers momentum p and energy E, then of the following equations is true?
(a) p = 0, E = 0 (b) p ¹ 0, E ¹ 0 (a) E0 k = B0w (b) E0 w = B0 k
(c) p ¹ 0, E = 0 (d) p = 0, E ¹ 0 (c) E 0 B0 = w k (d) None of these
Free eBooks on @neetquestionpaper2020

Electromagnetic Waves 827


41. A plane electromagnetic wave travels in free space along 48. A plane electromagnetic wave is incident on a plane surface
x-axis. At a particular point in space, the electric field along of area A, normally and is perfectly reflected. If energy E
y-axis is 9.3V m–1. The magnetic induction (B) along z-axis is strikes the surface in time t then average pressure exerted
(a) 3.1 × 10–8 T (b) 3 × 10–5 T on the surface is (c = speed of light)
(c) 3 × 10 T –6 (d) 9.3 × 10–6 T (a) zero (b) E/Atc
42. Which of the following has/have zero average value in a (c) 2E/Atc (d) E/c
plane electromagnetic wave ? DIRECTIONS for Qs. (49 to 55) : Each question contains
(a) Both magnetic and electric field STATEMENT-1 and STATEMENT-2. Choose the correct answer
(b) Electric field only (ONLY ONE option is correct ) from the following-
(c) Magnetic energy (a) Statement -1 is false, Statement-2 is true
(d) Electric energy (b) Statement -1 is true, Statement-2 is true; Statement -2 is a
43. For sky wave propagation of a 10 MHz signal, what should correct explanation for Statement-1
be the maximum electron density in ionosphere? (c) Statement -1 is true, Statement-2 is true; Statement -2 is not
(a) ~1.2 × 1012m–3 (b) ~106m–3 a correct explanation for Statement-1
(c) ~10 m 14 –3 (d) ~1022m–3 (d) Statement -1 is true, Statement-2 is false
44. A new system of unit is evolved in which the values of µ0 49. Statement 1 : When variable frequency a.c. source is
connected to a capacitor, displacement current increases
and Î0 are 2 and 8 respectively. Then the speed of light in with increase in frequency.
this system will be Statement 2 : As frequency increases conduction current
(a) 0.25 (b) 0.5 (c) 0.75 (d) 1 also increases.
45. Light wave is travelling along y-direction. If the 50. Statement 1 : Short wave bands are used for transmission
r
corresponding E vector at any time is along the x-axis, the of radio waves to a large distance.
r y Statement 2 : Short waves re reflected by ionosphere.
direction of B vector at that time is along
(a) y-axis 51. Statement 1 : Television signals are received through sky-
(b) x-axis x wave propagation.
(c) + z-axis Statement 2 : The ionosphere reflects electromagnetic
(d) – z-axis z waves of frequencies greater than a certain critical frequency.
46. An electromagnetic wave of frequency n = 3.0 MHz 52. Statement 1 : Ultraviolet radiations of higher frequency
passes from vacuum into a dielectric medium with waves are dangerous to human being.
permittivity Î = 4.0 . Then Statement 2 : Ultraviolet radiation are absorbed by the
(a) wavelength is halved and frequency remains atmosphere.
unchanged 53. Statement 1 : Environmental damage has increased the
(b) wavelength is doubled and frequency becomes half amount of ozone in the atmosphere.
(c) wavelength is doubled and the frequency remains Statement 2 : Increase of ozone increases the amount of
unchanged ultraviolet radiation on earth.
(d) wavelength and frequency both remain unchanged 54. Statement 1 : The earth without atmosphere would be
47. An electromagnetic wave passes through space and its
inhospitably cold.
equation is given by E = E0 sin (wt – kx) where E is electric
field. Energy density of electromagnetic wave in space is Statement 2 : All heat would escape in the absence of
atmosphere.
1 1
(a) e 0 E 20 (b) e 0 E 02 (c) e 0 E 20 (d) 2e 0 E 02 55. Statement 1 : Radio waves can be polarised.
2 4
Statement 2 : Sound waves in air are longitudinal in nature.

4. (d) The speed of electromagnetic wave in a region is same


Exercise 23.1
for all intensities but different for different frequencies.
5. (b) An electromagnetic wave has both energy and
1. (c) The electromagnetic waves of all wavelengths travel with momentum.
the same speed in space which is equal to velocity of
light. 6. (c) In the steady state Ñò B.dl = m o (I) , where I is conduction
current.
Free eBooks on @neetquestionpaper2020

828 Physi cs

[In non steady state Ñò


B.d l = (I + I d ) , where I is
d
Exercise 2 : PAST Competition MCQs
displacement current.] 1. (d) Direction of propagation of Electro-magnetic waves is
8. (a) Displacement current is set up in a region where the perpendicular to electric field and magnetic field. Hence
electric field is changing with time. direction is given by pointing
10. (c) The direction of propagation of electromagnetic wave is
r E´B
perpendicular to the variation of electric field E as well vector S = E ´ H = .
r m0
as to the magnetic field B 2. (c) l m > l v > l x
12. (b) Velocity of light in a medium,
3. (d)
1 1
c= = 4. (c) Electromagnetic waves are the combination of mutually
m0 eo mr er me perpendicular electric and magnetic fields. So, option
13. (a) Ι D = e o d fE / dt . (c) is false.
5. (a) E.M. wave always propagates in a direction
19. (d) The frequency of television signal is smaller than that of
perpendicular to both electric and magnetic fields. So,
microwave and visible waves.
electric and magnetic fields should be along + X and +
20. (d) Maxwell’s equations describe the fundamental laws of
Y- directions respectively. Therefore, option (a) is the
electricity and magnetism.
correct option.
21. (a) The frequency of r-rays is more than that of X-rays. 6. (a) The decreasing order of the wavelengths is as given
23. (b) The range of visible radiations is 4000Å to 8000 Å below :
24. (a) Cosmic rays are coming from outer space, having high microwave, infrared, ultraviolet, gamma rays.
energy charged particles, like a-particle, proton etc. b- 7. (a) On comparing the given equation to
rays are stream of high energy electrons, coming from r
the nucleus of radioactive atoms. E = a0iˆ cos (wt – kz)
29. (a) The wavelength of infrared region is 8 × 10–5 cm to w = 6 × 108z,
3 × 10–3 cm. So maximum wavelength of infrared region 2p w
k= =
= 8 × 10–5 cm » 10–4 cm. r c
30. (c) According to Maxwell, a changing electric field is a w 6 ´ 108
source of magnetic field. k= = = 2 m -1
c 3 ´ 108
8. (b) The average energy stored in the electric field
Exercise 23.2
1
U E = e0 E 2
2
1. (b) Range, R = 2hR where R is the radius of earth so The average energy stored in the magnetic field

R µ h1 / 2 . 1 B2
= UB = ,
2. (c) Ionosphere do not reflect the electromagnetic waves of 2 m0
frequencies more than 40 MHz. According to conservation of energy UE = UB
8. (a) Radiowaves are reflected by ionosphere. B2
9. (d) Now a days microwaves are used to locate the flying e 0m 0 =
objects by Radar. E2
B 1
Exercise 1 : NCERT Based Questions = e 0 m0 =
E C
1. 4 × 1011 kHz to 7.5 × 1011 kHz. 9. (d) Required condition : Frequency of microwaves =
Resonant frequency of water molecules.
Amplitudeof electric field E
2. = 0 = c (speed of light) 10. (c) b -rays are the beam of fast moving electrons.
Amplitude of magnetic field B0
3. conduction current. l decreasin g
11. (a) ¾¾¾¾¾¾
®
4. Yes, RMIVUXGC
5. Electromagnetic waves consist of time varying sinusoidal R ® Radio waves M ® Micro waves
electric and magnetic fields in perpendicular direction. I ® Infra red rays V ® Visible rays
1 U ® Ultraviolet rays X ® x rays
6. v=
me G ® g rays C ® Cosmic rays
7. 6 × 10–12 m Þ g rays has least wavelength
8. 7.5 cm 12. (a) Velocity of light,
16. 153 N/C 1 1 c
v = = =
18. (b) 19. (a) 20. (b) 21. (a) m0e0er m 0 e 0 .4 2
22. (d) 23. (c) 24. (d) v remains constant hence
Free eBooks on @neetquestionpaper2020

Electromagnetic Waves 829


7. (c) Relation between E0 and H0
n l0 c
l= = since v = Eo 1
v 2 2 = = m0 e 0
\ wavelength is halved and frequency remains Ho C
unchanged i.e. Ho = Eo eo m o
1 I
13. (b) I ' = I e -mx Þ x = loge 8. (d) 9. (a) 10. (a)
m I' 1
(where I = original intensity, I¢ = changed intensity) 11. (d) e 0 E 02 is electric energy density..
2
1 I 3 B2
36 = log e = loge 2 … (i) is magnetic energy density..
m I/8 m 2m 0
1 I 1 1 2 B 20
x = loge = loge 2 … (ii) So, total energy = e E
0 0 +
m I/2 m 2 2m 0
From equations (i) and (ii), x = 12 mm. 12. (b) 13. (d)
14. (a) Erms = 720 14. (a) Here, amplitude of electric field, E0 = 100 V/m; amplitude
The average total energy density of magnetic field, H0 = 0.265 A/m. We know that the
1 1 maximum rate of energy flow,
= Î0 E 20 = Î0 [ 2E rms ] =Î0 E rms
2 2
2 2 S = E0 × H0 = 100 × 0.265 = 26.5 W/m2.
15. (a) Size of particle = 2 ×(3 × 10–4) = 6 × 10–4 m. To observe a
= 8.85 ´10-12 ´ (720)2
particle, the wavelength of electromagnetic waves must
= 4.58 ´10-6 J / m3 be of the size of particle.
15. (b) Q The E.M. wave are transverse in nature i.e., 16. (c) Microwave region wavelength = 10–3 m to 1m
r r 17. (d) We know that velocity of electromagnetic wave in
k ´E r 1
= =H …(i) vacuum (v0 ) = and velocity of electromagnetic
mw m 0e0
r
r
where H = B wave in medium is (v) =
1
.
r r m me
k ´H r
and = -E … (ii) Therefore refractive index of the medium
we
r r r r (m ) =
vel. of E.M.wave in vacuum (v 0 )
k is ^ H and k is also ^ to E vel. of E.M. wave in medium (v)
r r r r r
or In other words X || E and k || E ´ B me
1/ m0 e0
16. (b) From question, = = m e
B0 = 20 nT = 20 × 10–9T 1/ me 0 0
r r r 18. (a) 19. (c) 20. (c) 21. (c)
E0 = B0 ´ C
r r r 22. (b) It is in accordance with relation = 2 hR
| E 0 |=| B0 | . | C |= 20 ´ 10-9 ´ 3 ´ 108 = 6 V/m. [where h is height of antenna]
(Q velocity of light in vacuum C = 3 × 108 ms–1) So, it must be made 4 times, to make coverage distance 2
times.
Exercise 3 : Conceptual & Applied MCQs
E0
23. (b) B0 =
c 3 ´ 10 8 c
1. (a) Here, l = = = 36.6 m. E0 - Electric field, c - speed of light, B0 - Magnetic Field.
n 8.2 ´ 10 6
2. (b) Here, E0 = 18 V/m; B0 = ? 10 -3
B0 = = 3.33 ´ 10 -12 T
E 18 8
B0 = 0 = = 6 ´ 10-8 T 3 ´ 10
c 3 ´108 24. (b) It is given by relation =
1
3. (d) d = 2 h R . m0e0
Population covered m0
25. (b) Wave impedance = Z = = 376.6 W
= p d2 × population density e0
E 0 9 ´103 26. (b) Velocity of EM waves
4. (d) B0 = = = 3 ´10 -5 T.
c 3 ´ 108 1
= = 3 ´ 108 m/s = velocity of light
5. (a) m 0 Î0
6. (d) l = 10Å = 10 ´ 10 –10 m = 10 –9 m 1
27. (c) Speed of EM waves in vacuum = = constant
X- ray wavelength is of the order of IÅ m 0 Î0
Free eBooks on @neetquestionpaper2020

830 Physi cs
28. (c)
l med vmed c / 2 1
29. (b) EM waves carry momentum and hence can exert = = =
l air vair c 2
pressure on surfaces. They also transfer energy to the
surface so p ¹ 0 and E ¹ 0. \ wavelength is halved and frequency remains
unchanged.
2p
30. (c) The angular wave number k = ; where l is the 47. (a) Energy density (EM waves)
l
2
wave length. The angular frequency is w = 2pn . 2 æ E0 ö 1
= e0 E rms = e 0 ç ÷ = e0 E02
è 2ø 2
k 2p / l 1 1
The ratio = = = = constant
w 2pn nl c E
31. (b) Infrared radiations reflected by low lying clouds and 48. (c) Incident momentum, p =
c
keeps the earth warm. For perfectly reflecting surface with normal incidence
32. (b)
33. (a) In vacuum velocity of all EM waves are same but their 2E
Dp = 2p =
wavelengths are different. c
34. (b) Molecular spectra due to vibrational motion lie in the Dp 2E
microwave region of EM-spectrum. Due to Kirchhoff’s F= =
Dt ct
law in spectroscopy the same will be absorbed.
35. (b) All components of electromagnetic spectrum travel in F 2E
P= =
vacuum with velocity 3 × 108 m/s. A ctA
r r
36. (c) E and B are mutually perpendicular to each other and 49. (b)
are in phase i.e., they become a zero and minimum at 50. (c) Short wave (wavelength 30 km to 30 cm). These waves
the same place and at the same time. are used for radio transmission and for general
37. (a) 38. (a) communication purpose to a longer distance from
ionosphere.
39. (a) Vg- rays > Vx -rays > VUV -rays 51. (a) In sky wave propagation , the radio waves having
frequency range 2 MHz to 30 MHz are reflected back
E0 2p
40. (a) = c . also k = and w = 2pv by the ionosphere. Radio waves having frequency
B0 l nearly greater than 30 MHz penetrates the ionosphere
and is not reflected back by the ionosphere. The TV
These relation gives E0 k = B0w
signal having frequency greater than 30 MHz therefore
41. (a) Velocity of light cannot be propagated through sky wave propagation.
E E 9.3 In case of sky wave propagation, critical frequency is
C= ÞB= = = 3.1 ´ 10-8 T defined as the highest frequency is returned to the
B C 3 ´ 108
earth by the considered layer of the ionosphere after
42. (a) Both magnetic and electric fields have zero average having sent straight to it. Above this frequency, a wave
value in a plane e.m. wave. will penetrate the ionosphere and not reflected by it.
43. (a) If maximum electron density of the ionosphere is Nmax 52. (c) The wavelength of these wave ranges between 4000 Å
per m 3 then the critical frequency fc is given by to 100 Å that is smaller wavelength and higher
fc = 9(Nmax)1/2 frequency. They are absorbed by atmosphere and
convert oxygen into ozone. They cause skin diseases
Þ 10 ´ 106 = 9(N)1/ 2 Þ N = 1.2 ´ 1012 m -3
and they are harmful to eye and cause permanent
1 1 1 blindness.
44. (a) The speed of light C = = = = 0.25 53. (a) Ozone layer in the stratosphere helps in protecting life
m0 e 0 2´8 4
of organism form ultraviolet radiation on earth. Ozone
layer is depleted due to of several factors like use of
45. (c) Light wave is an electromagnetic wave in which E chlorofluoro carbon (CFC) which is the cause of
environmental damages.
and B are at right angles to each other as well as at
54. (b) In the absence of atmosphere, all the heat will escape
right angles to the direction of wave propagation. from earth’s surface which will make earth in hospitably
46. (a) Frequency remains constant during refraction cold.
55. (c) Radio waves can be polarised because they are
1 c
vmed = = transverse in nature. Sound waves in air are longitudinal
µ0 Î0 ´4 2 in nature.
Free eBooks on @neetquestionpaper2020

24
Ray Optics and
Optical Instruments
REFLECTION OF LIGHT (ii) If an object moves towards (or away from) a plane mirror at
It is the turning back of light in the same medium. a speed v, the image will also approach (or recede) at the
same speed v, i.e. the speed of image relative to object will
Normal be v – (–v) = 2v.
(iii) If two plane mirrors are inclined to each other at 90º, the
emergent ray is always antiparallel to incident ray if it suffers
one reflection from each whatever be the angle of incidence.
Incident ray Reflected ray The same is found to hold good for three-plane mirrors
i r forming the corner of a cube if the incident light suffers one
reflection from each of them.
(iv) If there are two plane mirrors inclined to each other at an
Reflecting angle q, the no. of images of a point object formed are
Surface determined as follows :
i = angle of incidence
360°
r = angle of reflection (a) If is even integer (say m), no. of images formed
q
Laws of Reflection = (m – 1), for all positions of object.
(i) Angle of incidence (i) = angle of reflection (r) æ 360° ö
(ii) The incident ray, reflected ray and normal are always in (b) If çè ÷ is odd integer (say m), no. of images formed,
q ø
same plane.
n = m, if the object is not on the bisector of mirrors and
REFLECTION FROM PLANE SURFACE n = (m – 1), if the object is on the bisector of mirrors.
Plane mirror has infinitely large radius of curvature. It produces æ 360° ö
virtual image of same size but laterally inverted. Image is as much (c) If çè ÷ is a fraction, the no. of images formed will
q ø
behind the mirror as much is the object in front of it.
be equal to its integral part.
(i) If the direction of the incident ray is kept constant and the
mirror is rotated through an angle q about an axis in the A plane mirror always forms a virtual image if object is
plane mirror, then the reflected ray rotates through an real and forms a real image if the object is virtual.
angle 2q.
MIRROR FORMULA
1 1 1
q + =
v u f
90° – q where, u = distance of the object from the pole of mirror
M2 v = distance of the image from the pole of mirror
f = focal length of the mirror.
q
Mirror formula is valid only when the following
conditions are satisfied :
(a) Object is placed very near to the principal axis.
M1 (b) Object is placed far from the mirror.
Free eBooks on @neetquestionpaper2020

832 Physi cs
Magnification Image Formed by Convex Mirror

v v- f f I év ù Position of Position of
m= = = or m = = – ê ú Magnification Nature of image
u f u- f O ëu û object image
Infront of Between P
where, I = size of the image and O = size of the object and negative m < +1 Virual and erect
mirror and F
sign implies that image is inverted w.r.t the object. A t infinity At F m < < +1 Virtual and erect
The above formulae are applicable only for paraxial rays (the rays
which makes very small angle with the principal axis). Keep in Memory
Areal magnification : When a two dimensional object is placed 1. Rays retrace their path when their direction is reversed.
with its plane perpendicular to the principal axis, its magnification 2. Focal length of a mirror depends only on the curvature of
called superficial magnification or aerial magnification and is given æ Rö
by the mirror ç f = ÷ . It does not depend on the material of
è 2ø
2 the mirror or on the wavelength of incident light.
Area of image æ vö
Ms = =m2 =ç ÷ 3. Focal length of concave mirror is always negative.
Area of object è uø
Focal length of convex mirror is always positive.
Sign Conventions for Mirror and Lenses 1 1
4. The graph of versus for a concave mirror, if real
New cartesian sign conventions : v u
image is formed.
1. All the distances are measured from pole of spherical mirror
and optical centre in case of lenses. 1/v
2. The distances measured in a direction opposite to the direction
of incident light is taken as negative and vice-versa.
3. The heights measured upward and perpendicular to the 1/f
principal axis of mirror are taken as positive and vice -versa.
4. Angle measured from the normal in the anticlockwise
direction is positive and vice-versa. 1/f 1/u

5. The graph shows variation of v with change in u for a


mirror.
Incident ray v
– ve +ve +ve
Object For plane mirror
P – ve
(Pole) For
2f spherical mirror
f
f 2f u
Reflecting or refracting surface 6. A person needs a plane mirror of minimum half of his height
to see his full image.
IMAGE FORMED BY CONCAVE AND CONVEX MIRROR 7. A person standing in the middle of room can see complete
Image Formed by Concave Mirror 1
wall behind him if the mirror in front of him is rd of height
3
Pos ition of Pos ition of of wall.
Magnification Nature of image 8. A convex mirror is used as a rear view mirror (called driver
object image
Between P Behind the mirror).
+ve, m > 1 Virtual and erect 9. If two or more optical components produce magnification,
and F mirror
– ve, Highly then overall magnification (m) is the product of magnification
At F At infinity Real and inverted due to each component,
magnified
Between F i.e., m = m1 × m2× . . .
Beyond C – ve, M agnified Real and inverted • If m is negative, the image is inverted
and C
At C At C m = –1 Real and inverted
• If m is positive, the image is erect.
10. When an object moves with constant speed towards a
between F
Beyond C
and C
Dis mished Real and inverted concave mirror from infinity to focus, the image will move
away from the mirror slower in the beginning and with the
Highly
At infinity At F
diminshed
Real and inverted speed of the object when it is at centre of curvature C and
faster later on.
Free eBooks on @neetquestionpaper2020

Ray Optics and Optical Instruments 833


11. Concave mirrors are used as reflectors, as objective in Solution :
reflecting telescope and by doctors (ENT) to examine ears, v
nose and throat. It is also used as shaving mirrors. v = – 30, m = - =–2
u
12. The inability of a spherical mirror (or lens) of large aperture
to focus the paraxial rays and marginal rays to the same \ A ¢B ¢ = C ¢ D ¢ = 2 ´ 1 = 2mm
point on the principal axis is called spherical aberration.
B ¢C ¢ A ¢D ¢ v 2
Due to this defect the image formed is blurred. This defect Now = = 2 = 4 Þ B ¢ C ¢ = A ¢D ¢ = 4 mm
BC AD u
can be removed by using parabolic mirror.
13. Chromatic aberration is absent in mirrors but present in \ Perimeter = 2 + 2 + 4 + 4 = 12 mm
lenses. This is because the focal length of mirror is
æ Rö Example 3.
independent of wavelength of light ç f = ÷ but that of The sun (diameter D) subtends an angle q radian at the
è 2ø
pole of a concave mirror of focal length f. What is the
lens is dependent on wavelength.
diameter of the image of the sun formed by the mirror?
14. Different colour rays travel with different velocity in a
Solution :
medium but velocity of all coloured rays is same in vacuum Since the sun is very distant, u is very large and so 1/u is
(and air). practically zero
15. If a hole is formed in a mirror, then also we will get full image
with no hole in the image. The hole will only reduce the 1 1 1 1 1
+ = ; = - Þ v = –f
intensity of rays forming the image. v u f v f
The image of sun will be formed at the focus and will be real,
Newton’s Formula inverted and diminished
In case of spherical mirrors if object distance (x1) and image
distance (x 2 ) are measured from focus instead of pole,

u = (f + x1) and v = (f + x2), the mirror formula 1 + 1 = 1 B'


v u f F
A' q
1 1 1
reduces to + =
( f + x 2 ) ( f + x1 ) f P
which on simplification gives x1 x2 =f2
A'B' = height of image
Example 1.
Arc A 'B' d
The focal length of a concave mirror is 30 cm. Find the q= = Þq = Þd= fq
position of the object in front of the mirror, so that the image Radius FP f
is three times the size of the object. Therefore the diameter of the image = f q
Solution : Example 4.
Here image can be real or virtual. If the image is real A convex mirror has its radius of curvature 20 cm. Find
f = –30, u = ?, m = –3 the position of the image of object placed at a distance of
f -30 12 cm from the mirror.
m= Þ -3 = Þ u = –40 cm. Solution :
f -u -30 - u
If the image is virtual
f -30
m= Þ 3= Þ u = –20 cm.
f -u -30 - u
O I C
Example 2.
A square ABCD of side 1mm is kept at distance 15 cm infront
of the concave mirror as shown in the figure. The focal
The situation is shown in figure.
length of the mirror is 10 cm. Find the perimeter of its image. Here u = –12 cm and R = +20 cm, we have
/ / ///

1 1 2 1 2 1
+ = or = -
/ ///////

B C u v R v R u
///////////////////////

1 2 1 11
A D Þ = cm - cm = cm
v 20 -12 60
15cm.
60
v= cm
// / / /

11
Free eBooks on @neetquestionpaper2020

834 Physi cs
Example 5.
An object of length 2.5 cm is placed at a distance of 1.5 f dv fdx / dt(t - x) - ( -dx / dt)xf f 2 dx / dt
Þ = =
from a concave mirror where f is the magnitude of the dt (f - x) 2 (f - x) 2
focal length of the mirror. The length of the object is
At t = 0, x = 0
perpendicular to the principal axis. Find the length of the
image. Is the image erect or inverted? dv dx (M + m) v
\ = =
Solution : dt dt M
The given situation is shown in figure. \ image is moving in the +ve direction.
\ velocity of separation of image and object is
dv du dx æ dx ö 2dx 2( M + m ) v
f = - = - ç- ÷= =
dt dt dt è dt ø dt M
F
O
1.5f Example 7.
Consider the situation shown in fig. The elevator is going
The focal length f = –f and u = –1.5f, we have up with an acceleration of 2.00 m/s2 and the focal length
of the mirror is 12.0 cm. All the surfaces are smooth and
1 1 1 1 1 1
+ = or - + =- the pulley is light. The mass pulley system is released from
u v f 1.5f v f
rest (with respect to the elevator) at t = 0 when the distance
1 1 1 -1 of B from the mirror is 42.0 cm. Find the distance between
or = - = or v = –3f
v 1.5f f 3f the image of block B and the mirror at t = 0.200 s.
Take g = 10 m/s2
v 3f h
Now m = - = = -2 or 2 = -2
u - 1.5f h1

or h 2 = -2h1 = -5.0 cm m
A
The image is 5.0 cm long. The minus sign shows that it is
inverted. m B
2
a = 2.00 m/s
Example 6.
A gun of mass M fires a bullet of mass m with a horizontal
M
speed v. The gun is fitted with a concave mirror of focal
length f, facing towards the receding bullet. Find the speed
of separation of the bullet and the image just after the gun Solution :
was fired. With cabin as a reference frame, a psuedo force will act
Solution : such that ma + mg – T = ma1 ....(1)
From conservation of momentum mv = mu, u is the speed of or T = ma1 ....(2)
mirror in the opposite direction of v
whereas a1 = acceleration of m w.r.t. cabin
\ speed of separation of bullet from mirror is
\ adding eqs. (1) & (2),
mv (m + M)v
u+v= +v= g + a 12
M M a1 = = = 6m / s 2
2 2
Let h is the distance moved by block m in 0.2 sec.

1 1
\h = a 1 t 2 Þ h = ´ 6 ´ 0 .2 ´ 0 .2 = 0 .12 m
Bullet x 2 2
or h = 12 cm
\ At t = 0.2 sec distance of object from mirror is
Let at any instant the bullet is at a distance x w.r.t. mirror
42 – 12 = 30 cm.
\ u = –x, focal length = – f
Using equation,
1 1 1 1 1 1 1 1 1 f -x
+ = Þ - =- Þ = - = 1 1 1 1 1 1 fu
v u f v x f v x f xg + = , we have = - or v =
v u f v f u f -u
xf Putting values of f and u and solving, we get
Þv=
f -x v = 8.57 cm
Free eBooks on @neetquestionpaper2020

Ray Optics and Optical Instruments 835

24.1
Solve following problems with the help of above text and (b) They have zero average value
examples. (c) They are perpendicular to the direction of
1. Figure shows two rays A and B being reflected by a mirror propagation of light
and going as A' and B'. The mirror (d) They are mutually perpendicular
4. A point source of light is placed in front of a plane mirror.
Then
A (a) all the reflected rays meet at a point when produced
backward
(b) only the reflected rays close to the normal meet at a
B A point when produced backward.
(a) is plane (c) only the reflected rays making a small angle with the
(b) is convex B mirror, meet at a point when produced backward.
(c) is concave (d) light of different colours make different images.
(d) may be any spherical mirror 5. In image formation from spherical mirrors, only paraxial
2. The image formed by a concave mirror is rays are considered because they
(a) always real (a) are easy to handle geometrically
(b) always virtual (b) contain most of the intensity of the incident light
(c) certainly real if the object is virtual (c) form nearly a point image of a point source
(d) certainly virtual if the object is real (d) show minimum dispersion effect
3. When light propagates in vacuum there is an electric field 6. Which of the following (referred to a spherical mirror)
and a magnetic field. Which of the following is not true depends on whether the rays are paraxial or not?
about these fields? (a) Pole (b) Focus
(a) They are constant in time (c) Radius of curvature (d) Principal axis
ANSWER KEY
1. (a) 2. (c) 3. (a) 4. (a) 5. (c) 6. (b)

REFRACTION 1
2m or 1m2 = refractive index of 2nd medium w.r.t. the 1st medium.
Whenever a wave is bounced back into same medium at an
interface reflection is said to have occurred. Transmission of a m1 = refractive index of 1st medium w.r.t vacuum (or air)
wave into the second medium at an interface is called refraction. m2 = refractive index of 2nd medium w.r.t vacuum (or air)
When a ray of light is passing from denser to rarer medium, it (ii) The incident ray, the normal and the refracted ray at the
bends away from the normal and when passing from rarer to interface all lie in the same plane.
denser medium, it bends towards the normal. Refractive Index of the Medium
• When a ray of light passing from one medium to another (a) Refractive index of second medium w.r.t. first medium
medium frequency and phase do not change while
m 2 c / v2 v1
wavelength and velocity changes. 1m2 = = =
• Twinkling of stars, appearance of sun before actual sunrise m1 c / v1 v2
and after actual sunset etc. are due to atmospheric refraction.
Velocity of light in first medium
Laws of Refraction =
Velocity of light in second medium
(i) Snell's Law : When a light ray is incident on a surface
separating two transparent media, the ray bends at the time (b) Absolute refractive index of medium (n or µ)
of changing the medium. Velocity of light in vacuum c sin i
= = = n=µ
sin i v1 m 2 Velocity of light in medium v sin r
i.e. = = = 1m 2 ,
sin r v2 m1 Refractive index is the relative property of two media. If the
where i = angle of incidence first medium carrying the incident ray is a vacuum, then the
r = angle of refraction Sin i
ratio is called the 'absolute refractive index of the
Sin r
v1 = vel. of light in 1st medium second medium'. The relative refractive index of any two
v2 = vel. of light in 2nd medium media is equal to the ratio of their absolute refractive indices.
Free eBooks on @neetquestionpaper2020

836 Physi cs
Therefore, if the absolute refractive index of medium 1 and 2 (i) If second medium is denser, in comparison to first medium
be n1 and n2 respectively, then the refractive index of medium (i.e. m2 > m1), then from Snell’s law
2 with respect to medium 1 is
sin i m 2 v 2
= =
= n12
n
= 2 =
Sin i sin r m1 v1 here m2 >m1 so v1 > v 2
1 n2
n1 Sin r Þ k1 < k2 and l1 > l2.
n1 sin i = n2 sin r It means that if ray goes from rarer medium to denser
According to cauchy’s formula medium (i.e. from first medium to second medium), then
B wave number increases & wavelength decreases.
m = A+ 2 (ii) If second medium is rarer in comparison to first medium,
l
where, A and B are cauchy’s constant. then from Snell’s law
lred > lviolet sin i m 2 v1
= =
so, mred < mviolet sin r m1 v 2 here m2 < m1 so v1 < v 2
1 Þ k1 > k2 and l1 < l2.
(c) 1n2 = It means that when ray goes from denser to rarer medium,
2 n1
then wave number decreases & wavelength increases.
(d) For three mediums 1, 2 and 3 due to successive refraction. (iii) No change in wave number k occurs on reflection.
1n 2 × 2n 3 × 3n 1 = 1 Image due to refraction at a plane surface, Apparent shift
n 2 n 3 n1
´ ´ =1 O
n1 n 2 n3 (Denser)
(e) For two mediums, n1 and n2 are refractive indices with respect Medium 1
t i m1
I
to vacuum, the incident and emergent rays are parallel then r
n1 sinf1 = n2 sinf2. B
A
r m2 = 1
f1 Medium
1 Medium 2
f2 (Rarer)
Medium
2 (a) Here O = position of object
f2 I = position of image
f1 Medium Apparent depth 1 m 2 AI 1 t
1 = = i.e. = Þ AI =
f1 Real depth m m1 AO m m
Factors affecting refractive index : (b) The image shifts closer to eye by an amount
(i) Nature of the medium æ 1ö
(ii) Wavelength OI = AO – AI or Dt = ç 1 - ÷ t where t = thickness of
è mø
(iii) Temperature of the medium-with increase in temperature,
refractive index of medium decreases. medium over the object and Dt = apparent shift in its
position towards the observer.
Transmission of Wave
When an object in denser medium is seen through rarer
(i) The equation of the wave refracted or transmitted to the
medium, then apparent depth is less than real depth. But
(
next medium is given by : y = A´´ sin wt - k '0 x . This is ) when an aeroplane or bird flying is seen by an observers in
independent of the nature (rarer/denser) of the medium. denser medium, the apparent height is more by (m – 1)t
The wave is not inverted. Lateral shift by a slab of uniform thickness t, is
t
(ii) The amplitude (A´´) of the transmitted wave is less than x= sin(i - r )
that (A) of the incident wave. cos r
(iii) The angular frequency remains unchanged. However the
wave number changes. Note that the phase of the i
transmitted wave is ( wt - k '0 x ) and that of the incident
wave is (wt – kx).
(iv) The compression or rarefaction are transmitted as such and r t
same is the case with the crest or trough. m
The wave velocity (vp), the angular frequency (w) and the wave
number (k) are related as vp = w/k = nl. Let the wave velocity in x
the medium to which the wave is transmitted be v'p = w/k´ = nl'.
Free eBooks on @neetquestionpaper2020

Ray Optics and Optical Instruments 837


The apparent shift through a glass slab is in the direction of light
mv Denser medium
t
I Ov
If an object moves towards a rarer medium with a velocity v
æ 1ö then the image moves slower with a speed v/m as seen by
I I ¢ = S = t ç1 - ÷ the observer in rarer medium.
m I I¢ è mø
v/m Denser medium
O
I v
TOTAL INTERNAL REFLECTION (TIR)
4. Denser the medium, smaller is the wavelength.
When the object is placed in an optically denser medium and if
the incident angle is greater than the critical angle then the ray 5. When light travels from one medium to another the
of light gets reflected back to the originating medium. This wavelength and velocity changes proportionally but
phenomenon is called total internal reflection. frequency of rays remains the same
a
Critical angle (ic) : When a ray passes from an optically denser m2 b mc
6.
1
m= m =
medium to an optically rarer medium, the angle of refraction r is m1 and (‘a’ for air/vacuum)
2 c a
mb
greater than the corresponding angle of incidence i. From Snell’s law
7. When a parallel compound slab consists of two media of
r = 90° equal thickness and refractive indices m1 and m 2 then the
2m1m 2
ic equivalent refractive index m =
i i > ic m1 + m 2

COMMON DEFAULT
û Incorrect : If a mirror or a lens is painted black on one half,
sin i m 2 then half of image will be formed.
=
sin r m1 ü Correct : If half of the mirror or lens is blackened, we get
full image but with half the intensity.
Let m1 = m and m2 = 1 and let for i = ic, r = 90º then sin i c = 1/ m
1 REFRACTION AT A SPHERICAL SURFACE
\ ic = sin -1 ; i is called the critical angle For any curved spherical surfaces. Relation between u and v in
m c
This phenomenon takes place in shining of air bubble, sparkling terms of refractive indices of the mediums and the radius of
of diamond, mirage and looming, in optical communication using curvature of the curved spherical surface.
optical fibre.
m 2 m1 m 2 - m1
- =
Keep in Memory v u R
1. On travelling through a series of parallel layers, light follows P
the following formula i
m sin q = constant = m1 sin q1 = m 2 sin q 2 = m 3 sin q3
r
q1 b g
a
l1 O m1 P m2 C I (Image)
(Object)
q2
l2
q3
l3 Q
Spherical surface separating two media
(i) The lateral magnification in case of refraction from curved
• It is important to note that the above relationship is m æ vö
surfaces m = 1 ç ÷
valid only when boundaries undeviated. m2 è u ø
2. In case of refraction, if i = 0 then r = 0. This means that the m 2
(ii) Longitudinal magnification m ' = 2 m
ray which strikes to a boundary at 90° passes through the m1
boundary undeviated.
m1 is refractive index of medium 1 through which light
3. If an object moves towards a denser medium with a velocity
passes first before meeting the interface and m2 is the
v then the image moves faster with speed of mv as seen by refractive index of medium 2 to which light encounters after
the observer in denser medium. it passes through the interface.
Free eBooks on @neetquestionpaper2020

838 Physi cs
REFRACTION BY A LENS Focal Length by Displacement Method
The focus point of a lens is the point where image of an object
placed at infinity is formed. And its distance from optical centre D2 - d 2
f =
of the lens is called focal length. 4D
Focal length of convex lens is +ve, and of concave lens is –ve. where D = distance between an object and screen
(i) Lens formula or thin lens formula and d = distance between two positions of lens.
1 1 1
- = d
v u f
(ii) Lens maker's formula,
1 æ m2 ö æ 1 1ö é1 1 ù I1
=ç - 1÷ ç - ÷ = (m - 1) ê - ú
f è m1 ø è R1 R2 ø ë R1 R2 û
m2
where 12 m = D I2
m1
In the above formula m2 is refractive index of lens whereas m1 is Aperture of a lens : With reference to a lens, aperture means the
the refractive index of surrounding medium. effective diameter of its light transmitting area. So the brightness
R1 is the radius of curvature of the lens reached first by light and i.e. intensity of image formed by a lens which depends on the
R2 is the radius of curvature of the other surface. light passing through the lens will depend on the square of
aperture i.e. I µ (aperture)2
Magnification : m = v/u
This relation holds for both convex and concave lenses for real COMBINATION OF LENSES
as well as virtual images. (i) If a lens of focal length f is cut in two equal parts as shown in
Power of a lens, P = reciprocal of focal length expressed in metres. figure, each part will have focal length = f
1
i.e., P = . Its unit : dioptre(D). L L¢
f (in metre)
(ii) If the above parts of lens are put in contact as shown then
To solve numerical problems use sign conventions the resultant focal length will be,
while substituting values in above equations.
Equivalent focal length of two lenses separated by distance d 1 1 1 2 f
= + = i.e. F = L
d F f f f 2 L¢

1 1 1 d (iii) If the two parts are put as shown, then L will behave as
= + - convergent lens of focal length f while the other (L´)
F f1 f 2 f1 f 2
divergent of same focal length,
f1 f2 1 1 1
\ = + or F =¥
Equivalent focal length of lens - mirror combination : F +f -f
L L¢
In such a case, the ray of light suffers two refraction from the lens \ P=0
and one reflection from the mirror. The combination behaves as a
mirror whose focal length is given by (iv) If a lens of focal length f is divided into equal parts as shown,
then each part will have focal length f',

1 1 1
i.e. = + or f ´= 2 f
f f´ f´
1 2 1 i.e., each part will have focal length 2f.
= +
F fl fm (v) If these parts are put as shown, then the resultant focal
length of the combination will be
f l = focal length of lens , fm = focal length of mirror
It is important that in the above formula, we cannot apply the
sign conventions of cartesian system rather following sign or
conventions are followed.
Focal length of a converging lens / mirror is taken as positive 1 1 1
and focal length of diverging lens/mirror is taken as negative. = + or F = f i.e. initial value.
F 2f 2f
Free eBooks on @neetquestionpaper2020

Ray Optics and Optical Instruments 839


REFRACTION THROUGH A PRISM This shows that for a small angled prism, deviation is
Prism is a transparent medium whose refracting surfaces are independent of angle of incidence.
inclined to each other. 3. Angle of emergence of a prism is 90° (called grazing
(i) The angle of deviation is given by d = i + i¢ – A emergence) when angle of incidence
where A= angle of prism. For d to be minimum, i = i¢ and
r = r¢ i = sin -1[ (m 2 - 1) sin A - cos A]
dm = A(m – 1) 4. A single prism produces deviation and dispersion
A
simultaneously.
U 5. Dispersion without deviation : When white light is incident
T on a combination of two prisms of different materials and
of suitable angles placed opposite to each other, the
d
i i´ emergent light may have only dispersion without any
r r´ deviation ( of mean colour yellow).
Q R
P
A
S Flint
B C t v Y
e ligh
Whit R
æ A + dm ö
sin ç Crown
è 2 ÷ø A¢
Refractive index of prism, m =
æ Aö For this to happen the conditions is
sin ç ÷
è 2ø A (m¢ - 1)
=- ( For thin lenses)
where dm = minimum angle of deviation A¢ (m - 1)
If angle of prism A is small, than dm is also small.
The net angular dispersion produced
( A + dm ) / 2 q = (w - w¢)d ( For thin lenses)
\ m=
A/2 6. Deviation without dispersion
Plot of angle of deviation (d) versus angle of incidence (i)
for a triangular prism.
t R
d te ligh
Whi
v

A¢ -(m v - m r )
dm For this to happen A = (m ¢ - m ¢) .... (1)
v r

i i = i¢ i¢ i æ wö
Net deviation d net = dç1 - ÷
è w¢ ø
Dispersion Equation (1) is said to be the condition of achromatism for
It is the breaking up of white ray of light into its constituents combination of two prisms.
colours VIBGYOR. The band of seven constituents colours is 7. Variation of refractive index of a medium with wavelength
called spectrum. causing incident light to split into constituent colours is
Angular dispersion : It is defined as the difference of deviations dispersion.
suffered by the extreme colours. b c
Cauchy’s equation : m = a + + , where a, b and c are
i.e., q = dv - d r = (mv - m r ) A [For thin prism] l 2 l 4
constants.
Dispersive power : It is defined as the ratio of angular dispersion
8. Rayleigh scattering law explains blue colour of sky. Intensity
to the mean deviation produced by the prism.
of scattered light is proportional to 1/l4. Hence the red
d - d r mv - mr light having highest value of lR scatters less.
i.e., w = v = [For thin prism]
d m 9. Rainbow can be observed if light source is behind and the
droplets are in front of the observer, i.e. when the back of a
Keep in Memory person is towards the sun.
It is a consequence of dispersion of sunlight by water
1. A ray entering a prism of angle A will not emerge out of droplets due to a combinations of refraction and total internal
prism if A > 2qc where qc = critical angle reflection. If the rainbow is formed after one internal
2. Maximum deviation through a prism will occur when angle reflection in the droplets, it is called a primary rainbow. In
of incidence is 90°. this the violet ray emerges at an angle of 40.8º and red rays
For this prism d = (m - 1)A at an angle of 42.8º. If the rainbow is formed after two internal
Free eBooks on @neetquestionpaper2020

840 Physi cs
reflections, it is called a secondary rainbow. In this the violet Example 8.
rays emerge at 54º and red at 51º, i.e. the order of colours is Consider the situation shown in figure. Find maximum
reversed. The primary rainbow is brighter than the angle for which the light suffers total internal reflection
secondary. at the vertical surface.
10. When a point source of light is placed at a depth h below
the surface of water of refractive index m , then radius of
q
m = 1.0
bright circular patch on the surface of water is given by
h q’
R=
2
m -1 m = 1.25
q’’

h Solution :
The critical angle for this case is
B
1 4 4
q ' ' = sin -1 = sin -1 or sin q ' ' =
11. When a lens made up of glass is immersed in water, its 1 .25 5 5
focal length changes.
p 3
Since q '' = - q ' , we have sin q ' = cos q '' =
1
f
= ( a
g m -1 ) æ 1 1ö 1
çè R - R ÷ø f ¢ =
1 2
( w
)
g m -1
æ 1 1ö
çè R - R ÷ø
1 2
2
From Snell's law,
5

a sin q 3 3
f ¢ g m - 1 g m / m a - 1 ( g m - a m)m w = 1.25 or sin q = 1.25 ´ sin q' = 1.25 ´ =
= w = = sin q' 5 4
f g m -1 g m / m w -1 ( g m - w m)m a
3
12. For achromatic combination of these lenses in contact, the or q = sin -1
4
w1 w2 If q² is greater than the critical angle, q will be smaller than
necessary condition is f + f = 0 this value. Thus, the maximum value of q¢ for which total
1 2
13. For two lenses separated by distance d, spherical aberration internal reflection takes place at the vertical surface is
is minimum when d = f1 – f2. sin–1 (3/4).
14. A convex lens forms a real image when the object is placed Example 9.
beyond focus. When the object is placed between optical A point object O is placed in front of a transparent slab at
centre and focus, convex lens forms a virtual image. a distance x from its closer surface. It is seen from the
15. A concave lens always form a virtual image for a real object. other side of the slab by light incident nearly normally to
16. A lens is called thin when the thickness of the lens is small the slab. The thickness of slab is t and its refractive index
compared to the object distance, image distance, radii of is m. Show that the apparent shift in the position of the
curvatures of the lens. object is independent of x and find its value.
In the case of thick lens, the problem has to be solved
using formula for each interface one by one.
17. Real image (inverted) Virtual image (erect)
Real image is formed by Virtual image is formed by actual
O1 O IA B
intersection of rays. extending the rays in the m
back direction. x t
This image can be brought This image cannot be Solution :
on a screen. brought on screen.
The situation is shown in figure. Because of the refraction
at the first surface, the image of O is formed at O1. For this
COMMON DEFAULT
refraction, the real depth is AO = x and the apparent depth
û Incorrect : Using thin lens formula while the lens given in is AO1. Also, the first medium is air and the second is the
the numerical problem is thick. slab. Thus,
æ1 1 1ö
ü Correct : The lens formula ç - = ÷ and lens maker’ss x 1
èv u f ø =
é1 AO1 m or, AO1 = mx.
æ 1 1 öù
formula ê = (m - 1)çç - ÷ú are valid only for thin
÷ The point O1 acts as the object for the refraction at the
ëê f è R 1 R 2 øûú second surface. Due to this refraction, the image of O1 is
lenses. formed at I. Thus,
Free eBooks on @neetquestionpaper2020

Ray Optics and Optical Instruments 841

BO 1 AB + AO1 t + mx Example 12.


= m or, = m or =m An object is placed 21 cm in front of a concave mirror of
BI BI BI
radius of curvature 20 cm. A glass slab of thickness 3 cm
t and refractive index 1.5 is placed close to the mirror in
or BI = x +
m the space between the object and the mirror. Find the
The net shift is position of the final image formed. The distance of the
nearer surface of the slab from the mirror is 10 cm.
æ tö æ 1ö Solution :
OI = OB - BI = ( x + t ) - çç x + ÷÷ = t çç1 - ÷÷
è mø è mø
which is independent of x.

Example 10.
A tank is filled with water to a height of 12.5 cm. The P P1
apparent depth of a needle lying at the bottom of full tank
is measured by a microscope to be 9.4 cm. What is the
refractive index of water? If water is replaced by a liquid
of refractive index 1.63 upto the same height, by what The situation is shown in figure. Because of the refraction
distance would the microscope have to be moved to focus at the two surfaces of the slab, the image of the object P is
the needle again? formed at P1, shifted towards the mirror by a distance
Solution :
Here real depth =12.5 cm : apparent depth = 9.4 cm; m = ? æ 1ö æ 1 ö
t ç1 - ÷ = (3 cm) ç1 - ÷ = 1 cm.
è mø è 1.5 ø
real depth 12 . 5
m= = = 1 .33 Thus, the rays falling on the concave mirror are diverging
Apparent depth 9 .4 from P1 which is at
Now in second case, m = 1.63, real depth = 12.5 cm , apparent 21 cm – 1 cm = 20 cm from the mirror.
depth = ? But the radius of curvature is also 20 cm, hence P1 is at the
centre. The rays, therefore, fall normally on the mirror and
12 . 5 12 . 5
1 . 63 = or y = = 7 . 67 cm . hence, retrace their path. The final image is formed at P
y 1 .63 itself.
Distance through which microscope has to be moved up Example 13.
= 9.4 – 7.67 = 1.73 cm. Consider the situation shown in figure. Light from a point
Example 11. source S is made parallel by a convex lens L. The beam
A converging lens has a focal length of 20 cm in air. It is travels horizontally and falls on an 88°-88°-4° prism as
made of a material of refractive index 1.6. If the lens is shown in the figure. It passes through the prism
immersed in a liquid of refractive index 1.3. What will be symmetrically. The transmitted light falls on a vertical
the new focal length of the lens? mirror. Through what angle should the mirror be rotated
Solution : so that the image of S is formed on S itself?

1 2
= (m1 - 1) m1 = 1.6, f1 = 20 d
f1 R S
1 2
= (1.6 - 1) L 88°
f1 R 88°

1 0 .6 ´ 2 0 .6 ´ 2 ´ 20
or = \R = = 24 cm Solution :
20 R 10
The parallel beam after going through the prism will be
1 2 1 1. 6 2 deviated by an angle d. If the mirror is also rotated by this
= (m1 - 1) ´ \ =( - 1) = angle d, the rays will fall normally on it. The rays will be
f2 R f2 1. 3 R
reflected back along the same path and form the image of S
1 1 . 6 - 1 .3 2 0. 3 1 1 on itself.
=( ) = ´ = As the prism is thin, the angle d is given by
f2 1 .3 24 1. 3 12 52
d = (m - 1)A = (1.5 - 1) ´ 4° = 2°
f2 = 52 cm.
Thus, the mirror should be rotated by 2°.
Free eBooks on @neetquestionpaper2020

842 Physi cs
Example 14. Example 16.
Locate the image formed by refraction in the situation There is a small air bubble inside a glass sphere (m = 1.5)
shown in figure. The point C is the centre of curvature. of radius 10 cm. The bubble is 4.0 cm below the surface
and is viewed normally from the outside. Find the apparent
depth of the bubble.
m=1.5 Solution :
m=1.0
The observer sees the image formed due to refraction at
the spherical surface when the light from the bubble goes
C
from the glass to the air.
25 cm 20 cm

P
A
I
Solution :
O
m 2 m1 m 2 - m1
We have, - = ....(1) C
v u R m = 1.5
Here u = –25 cm, R = 20 cm, m1 = 1.0 and m2 = 1.5
Putting the values in (1),
Here u = – 4.0 cm, R = –10 cm, m1 = 1.5 and m2 = 1.
1 .5 1 .0 1 .5 - 1 .0 1. 5 1 1
+ = or, = - We have,
v 25 cm 20 cm v 40 cm 25 cm 1 1 .5 1 - 1 .5
m 2 m1 m 2 - m1
- = , or v - - 4. 0 cm = - 10 cm
or, v = –100 cm. v u R
As v is negative, the image is formed to the left of the 1 0 .5 1 .5
separating surface at a distance of 100 cm from it. or v = 10 cm - 4 .0 cm or v = –3.0 cm.
Example 15. Thus, the bubble will appear 3.0 cm below the surface.
One end of a horizontal cylindrical glass rod (m = 1.5) of
Example 17.
radius 5.0 cm is rounded in the shape of a hemisphere. An
Calculate the focal length of the thin lens shown in figure.
object 0.5 mm high is placed perpendicular to the axis of
The points C1 and C2 denote the centres of curvature.
the rod at a distance of 20.0 cm from the rounded edge.
Locate the image of the object and find its height.
Solution :
Taking the origin at the vertex, u = –20.0 cm and R = 5.0 cm. C1 C2

m 2 m1 m 2 - m1
We have, - =
v u R 10 cm
20 cm
1 .5 1 0 .5 1 Solution :
or v = - 20 .0 cm + 5.0 cm = 20 cm or v = 30 cm As is clear from the figure, both the radii of curvature are
positive. Thus, R1 = +10 cm and R2 = +20 cm. The focal
length is given by
1 æ 1 1 ö æ 1 1 ö
= (m - 1)çç - ÷ = (1.5 - 1)çç
÷ - ÷÷
f è R1 R 2 ø è 10 cm 20 cm ø

5.0cm 1 1
20.0cm 25 cm = 0.5 ´ =
20 cm 40 cm
The image is formed inside the rod at a distance of 30 cm or, f = 40 cm.
from the vertex.
Example 18.
m1 v 30 cm
The magnification is m = = = -1 A point source S is placed at a distance of 15 cm from a
m 2 u - 1.5 ´ 20 cm converging lens of focal length 10 cm on its principal
Thus, the image will be of same height (0.5 mm) as the axis. Where should a diverging mirror of focal length 12
object but it will be inverted. cm be placed so that a real image is formed on the source
itself ?
Free eBooks on @neetquestionpaper2020

Ray Optics and Optical Instruments 843


Solution : We have,
m 2 m1 m 2 - m1 1 .5 1 1 .5 - 1
- = or - 25 cm - u = 25 cm
v u R

S I1 or
1
u
=-
1 .5
-
0 .5
25 cm 25 cm
or u = –12.5 cm

Thus, the object should be placed at a distance of 12.5 cm


L M from the lens.
15 cm

1 1 1 Example 20.
The equation for the lens is - = ....(i)
v u f A concavo-convex lens made of glass (m = 1.5) has surfaces
Here u = –15 cm and f = +10 cm. of radii 20 cm and 60 cm. (a) Locate the image of an
object placed 80 cm to the left of the lens along the
Using equation (i),
principal axis. (b) A similar lens is placed coaxially at a
1 1 1 1 1 1 1 distance of 160 cm right to it. Locate the position of the
+ = Þ = - = image.
v 15 cm 10 cm v 10 cm 15 cm 30 cm
or, v = 30 cm.
The positive sign of v shows that the image is formed to
the right of the lens in the figure. the diverging mirror is to
be placed to the right in such a way that the light rays fall
on the mirror perpendicularly. Then only the rays will retrace
O I2 I1
their path and form the final image on the object. Thus, the
image I1 formed by the lens should be at the centre of
curvature of the mirror.
We have, LI1 = 30 cm, MI1 = R = 2f = 24 cm.
Hence, LM = LI – MI1 = 6 cm. Solution :
The focal length of the lens is given by
Thus, the mirror should be placed 6 cm to the right of the lens.
Example 19. 1 æ 1 1 ö
= (m - 1)çç - ÷
÷
A biconvex thin lens is prepared from glass (m = 1.5), the f R
è 1 R 2 ø
two bounding surfaces having equal radii of 25 cm each.
One of the surfaces is silvered from outside to make it æ 1 1 ö 1
= (1.5 - 1)çç - ÷÷ =
reflecting. Where should an object be placed before this è 20 cm 60 cm ø 60 cm
lens so that the image is formed on the object itself?
or, f = 60 cm.
Solution :
(a) For the image formed by the first lens, u = –80 cm so
that
A B
1 1 1 1 1 1
= + = + =
v u f - 80 cm 60 cm 240 cm

C2 O P C1 or, v = 240 cm.


The first image would form 240 cm to the right of the
25 cm 25 cm first lens.
(b) The second lens intercepts the converging beam as
Refer to figure. The object is placed at O. A ray OA starting suggested by the figure. The image I1 acts as a virtual
from O gets refracted into the glass at the first surface and source for the second lens. For the image formed by
hits the silvered surface along AB. To get the image at the this lens,
object, the rays should retrace their path after reflection u = 240 cm – 160 cm = +80 cm so that
from the silvered surface. This will happen only if AB falls
1 1 1 1 1 7
normally on the silvered surface. Thus, AB should appear = + = + =
v u f 80 cm 60 cm 240 cm
to come from the centre C2 of the second surface. Thus,
due to the refraction at the first surface, a virtual image of O or, v = 34.3 cm.
is formed at C2. For this case, The final image is formed 34.3 cm to the right of the
v = –25 cm, R = +25 cm, m1 = 1, m2 = 1.5. second lens.
Free eBooks on @neetquestionpaper2020

844 Physi cs
Example 21. f of this lens = 30 cm
A convex lens focuses a distant object on a screen placed
1 1 1 1 1 1 1 1 1 1
10 cm away from it. A glass plate (m = 1.5) of thickness 1.5 \ - = Þ = + Þ =- + =
v v'1 f v v '1 f v 40 30 120
cm is inserted between the lens and the screen. Where
should the object be placed so that its image is again or v = 120 cm.
focused on the screen. Final image is 120 cm right from second lens.
Solution : Example 23.
A lens which is cut into two equal halves are kept as shown,
the focal length of original lens is 20 cm. Find the position
of image of object, kept at 15 cm left from the system of
lenses.

The focal length of the lens is 10 cm. The situation with the
glass plate inserted is shown in figure. The object is placed Solution :
f
at O. The lens would form the image at I1 but the glass plate F of system of lenses as shown is = 10 cm
2
intercepts the rays and forms the image at I on the screen. u = – 15 cm, F = 10 cm
The shift 1 1 1
\ From the formula, - =
v u F
æ 1ö æ 1 ö
I1I = t çç1 - ÷÷ = (1 .5 cm ) ç1 - ÷ = 0 .5 cm . 1 1 1 1
è m ø è 1 .5 ø We have, =- + = Þ v = 30 cm right of
v 15 10 30
Thus, the lens forms the image at a distance of 9.5 cm from system.
itself. Using
Example 24.
1 1 1 1 1 1 1 1 Locate the image of the point P as seen by the eye through
- = Þ = - = -
v u f u v f 9.5 cm 10 cm the slabs of refrective index (m = 1.2, 1.3, 1.4 respectively
starting from the bottom) as shown in figure.
Þ u = 190 cm
Thus, the object should be placed at a distance of 190 cm
from the lens.
t = 0.4cm
Example 22. 1.0cm
A lens is cut into two equal pieces and they are placed as t = 0.3cm
shown in figure. An object is placed at a distance of 12 cm
left from the first half lens. The focal length of original 1.0cm
lens was 30 cm. Find the position of final image. t = 0.2cm
1.0cm
P
Solution :
æ 1ö
O The net shift from one slab is t ç 1 - ÷ where as t is the
è mø
thickness of slab, m is the refractive index of slab. Total
shift is
Solution :
æ 1ö æ 1 ö æ 1ö
Focal length of each lens is 30 cm t1 ç 1 - ÷ + t 2 ç1 - ÷ + t 3 ç1 - ÷
For first lens u = – 12, f = 30 cm è m1ø è m 2ø è m 3ø

1 1 1 1 1 1 1 1 æ 1 ö æ 1 ö æ 1 ö
- = Þ = + =- + = 0 . 2ç 1 - ÷ + 0 . 3ç 1 - ÷ + 0 .4 ç 1 - ÷
v1 u f v1 u f 12 30 è 1.2 ø è 1 .3 ø è 1 .4 ø
or, v1 = – 20 cm.
æ 0 .2 ö 0 .3 ´ 0 .3 0 .4 ´ 0 .4
For the second half lens image formed at v1 acts as object. = 0 .2 ç ÷+ +
è 1 .2 ø 1 .3 1 .4
Therefore object distance from second lens is = 20 + 20 = 40
cm or v1' = –40 cm = 0.0333 + 0.0692 + 0.114 = 0.2 cm above point P.
Free eBooks on @neetquestionpaper2020

Ray Optics and Optical Instruments 845


Example 25. Also image of fish, formed by plane mirror is H/2 below
Consider the situation in figure. The bottom of the plot is the mirror.
a reflecting plane mirror. S is a small fish and T is a human \ h real = H + H / 2 = 3H / 2
eye, refractive index of water is m (a) At what distance (s)
(Q hreal is distance of fish image formed by the mirror
from itself, will the fish see the image (s) of the eye (b) At
from the surface)
what distance (s) from itself will the eye see the image (s)
of the fish. h app 1 3H
Now = Þ h app =
h real. m 2m
H
æ 3Hö
\ Image formed is ç H + ÷ below the eye i.e,
è 2mø
æ 3ö
H H ç1 + ÷ below the eye.
S
H/2 è 2m ø
Example 26.
Solution : A lens has a power of +5 diopter in air. What will be its
(a) We have the formula power if completely immersed in water ?
h app m 3 4
= = h app = mH (from the surface of water) Given m g = ; mw =
h real 1 2 3
Now distance of fish from surface is H/2 Solution :
\ Image of eye as seen by fish is = H(m + ½) above the Let fa and fw be the focal lengths of the lens in air water
fish. respectively, then
also the apparent image of eye, again makes an image 1 mw
with the plane mirror, the apparent distance of eye is Pa = and Pw = f ; fa = 0.2 m = 20 cm
fa
mH + H from the plane mirror w
\ Now image formed is mH + H below the plane mirror. Using lens maker’s formula
Distance of fish from the mirror is H/2
1 é 1 1 ù
\ Distance of image from the fish is mH + H +H/2 = H Pa = = (mg - 1) ê - ú ...(i)
(m + 3/2) below the fish. fa ë R1 R 2 û
(b) Here we have to find the images of fish as seen by the
eye. 1 æ mg öé 1 1 ù
=ç - 1÷ ê - ú
Let happarent be the apparent distance of the fish from f w è m w ø ë R1 R 2 û
the surface
h app. mw é 1 1 ù
1 mH Þ Pw = f = (m g - m w ) ê R - R ú ...(ii)
\ = ; h app =
h real m 2m w ë 1 2û

\ Image formed is (H + H/2m) below the, eye, Dividing equation (ii) by equation (i), we get,

æ 1ö Pw (m g - m w ) 1 P + 5 10
i.e, H ç1 + ÷ below the eye. = = or Pw = a + = D
Pa (m g - 1) 3 3 3 3
è 2m ø

24.2
Solve following problems with the help of above text and (a) Wavelength (b) Frequency
examples. (c) Velocity (d) Amplitude
1. When light is refracted into a medium, 3. Wh ich of th e following sources gives best
(a) its wavelength and frequency both increase monochromatic light ?
(b) its wavelength increases but frequency remains (a) A candle (b) A bulb
unchanged (c) A mercury tube (d) A laser
(c) its wavelength decreases but frequency remains 4. Total internal reflection can take place only if
unchanged (a) light goes from optically rarer medium (smaller
(d) its wavelength and frequency both decrease refractive index) to optically denser medium
2. When light is refracted, which of the following does not (b) light goes from optically denser medium to rarer
change ? medium
Free eBooks on @neetquestionpaper2020

846 Physi cs

(c) the refractive indices of the two media are close to 12. Two thin lenses of focal lengths f1 and f2 are made of
different materials of dispersive powers w1 and w2 respectively.
(d) the refractive indices of the two media are widely The lenses are kept in contact. The combination will show
different no chromatic aberration if
5. A ray of light travelling inside a rectangular glass block of (a) w1 = w2 (b) (f1/f2) = –(w1/w2)
(c) f1 w1 = f2 w2 (d) f1 w1 + f2 w2 = 0
refractive index 2 is incident on the glass-air surface at 13. The focal length of a normal eye-lens is about
an angle of incidence of 45º. The refractive index of air is (a) 1 mm (b) 2 cm
one. Under these conditions the ray will (c) 25 cm (d) 1 m
(a) emerge into the air without any deviation 14. Which of the following is used in optical fibres ?
(b) be reflected back into the glass (a) Total internal reflection
(c) be absorbed (b) Scattering
(d) emerge into the air with an angle of refraction equal (c) Diffraction
to 90º (d) Refraction
6. The angular dispersion produced by a prism 15. A parallel beam of light is incident on a converging lens
parallel to its principal axis. As one moves away from the
(a) increases if the average refractive index increases
lens on the other side on its principal axis, the intensity of
(b) increases if the average refractive index decreases
light
(c) remains constant whether the average refractive (a) remains constant
index increases or decreases (b) continuously increases
(d) has no relation with average refractive index. (c) continuously decreases
7. If a glass prism is dipped in water, its dispersive power (d) first increases then decreases
(a) increases 16. The rays of different colours fail to converge at a point
(b) decreases after going through a converging lens. This defect is called
(c) does not change (a) spherical aberration
(d) may increase or decrease depending on whether the (b) distortion
angle of the prism is less than or greater than 60º (c) coma
8. When a drop of oil is spread on a water surface, it displays (d) chromatic aberration
beautiful colours in daylight because of 17. If the light moving in a straight line bends by a small but
(a) dispersion of light (b) reflection of light fixed angle, it may be a case of
(c) polarization of light (d) interference of light (a) reflection (b) refraction
(c) diffraction (d) both (a) & (b)
9. A bird in air looks at a fish vertically below it and inside
18. What causes chromatic aberration?
water h1 is the height of the bird above the surface of
(a) Marginal rays
water and h 2, the depth of the fish below the surface of (b) Central rays
water. If refractive index of water with respect to air be m, (c) Difference in radii of curvature of its surfaces
then the distance of the fish as observed by the bird is (d) Variation of focal length of lens with colour
h2 19. The focal length of a converging lens are fV and fR for
(a) h1 + h2 (b) h1 + violet and red light respectively. Then
m
(a) fV> fR
(c) m h1 + h2 (d) m h1 + m h2
(b) fV = fR
10. An electromagnetic radiation of frequency n, wavelength
l, travelling with velocity v in air, enters a glass slab of (c) fV < fR
refractive index m. The frequency, wavelength and velocity (d) any of the three is possible depending on the value
of light in the glass slab will be respectively of the average refractive index m
20. A narrow beam of white light goes through a slab having
n l v l v parallel faces
(a) , and (b) n, and
m m m m m (a) the light never splits in different colours
(b) the emergent beam is white
v 2n l (c) the light inside the slab is split into different colours
(c) n, 2 l and (d) , and v
m m m (d) the light inside the slab is white
21. By properly combining two prisms made of different
11. Yellow light is refracted through a prism producing materials, it is not possible to have
minimum deviation. If i 1 and i 2 denote the angle of (a) dispersion without average deviation
incidence and emergence for this prism, then (b) deviation without dispersion
(a) i1 = i2 (b) i1 > i2 (c) both dispersion and average deviation
(c) i1 < i2 (d) i1 + i2 = 90º (d) neither dispersion nor average deviation
Free eBooks on @neetquestionpaper2020

Ray Optics and Optical Instruments 847

22. In producing a pure spectrum, the incident light is passed 23. Which of the following quantities increase when
through a narrow slit placed in the focal plane of an wavelength is increased? (consider only the magnitudes)
achromatic lens because a narrow slit (a) The power of a converging lens
(a) produces less diffraction
(b) The focal length of a converging lens
(b) increases intensity
(c) allows only one colour at a time (c) The power of a diverging lens
(d) allows a more parallel beam when it passes through (d) The focal length of a diverging lens
the lens
ANSWER KEY
1. (b, c) 2. (b) 3. (d) 4. (b) 5. (d) 6. (a) 7. (b) 8. (d) 9. (b) 10. (b) 11.(a) 12. (b)
13.(b) 14. (a) 15.(a) 16. (d) 17. (d) 18. (d) 19. (c) 20. (b, c) 21. (d) 22. (d) 23. (b, d)

OPTICAL INSTRUMENTS (ii) Compound Microscope


(i) Simple Microscope
It is known as simple magnifier & consist of a convergent
lens with object between its focus & optical centre & eye
close to it.

M.P. of compound microscope is defined as


visual angle with the instrument q
M.P = =
max. visual angle for unaided eye q 0

h h1
where q 0 = , q=
D ue
h1 D æ h1 öæ D ö
so M.P. = ´ = ç ÷ç ÷
u e h è h øçè u e ÷ø
The magnifying power of a simple microscope (M.P.) is
T v h1 v
visual angle with instrument q (since for objective m = = Þ = - , as u is –ive)
M.P. = = o u h u
max visual angle for unaided eye q0
-v æ D ö
so M .P. =
h h h q h D D u çè ue ÷ø
Here q 0 = , q = 1 = so M.P. = = ´ =
D v u q0 v h u
(a) When image is formed at least distance of distinct
(a) If image is at infinity [far point] then from lens formula
v0 æ Dö
1 1 1 1 1 1 D vision, M =
ç1 + ÷ = M 0 ´ M e
- = Þ - = i.e., u = f & M.P. = u0 è fe ø
v u f a -u f f
(b) When the final image is formed at infinity
In this case M.P. is minimum if eye is least strained.
(b) If image is at D [near point] then u = –D -v0 æ D ö
M = ç ÷
1 1 1 u0 è f e ø
and from lens formula - =
v u f
(iii) Astronomical Telescope
D D æ Dö
we get = (1 + ) so, M .P = ç1 + ÷ (a) If the final image is formed at a distance D,
u f è fø
In this case M.P. is maximum and as final image is - f0 æ fe ö
M = ç1 + D ÷
close to eye, eye is under maximum strain. fe è ø
Free eBooks on @neetquestionpaper2020

848 Physi cs

fe D
and length of tube is L = f 0 + 3.
fe + D
(b) If the final image is formed at infinity then
f0
M = and the length of tube is l = f 0 + f e
fe
(iv) Galilean Telescope
f0
M = ; Length of tube L = fo – fe
fe
4. Some common eye defects are myopia, hypermetropia,
(v) Terrestrial Telescope astigmatism and presbyopia.
f0
M = ; 5.
fe M yopia (or short H ypermetropia (or long -
sightedness) sightedness)
Length of tube L = fo + fe+ 4f, where f is the focal length of
erecting lens, which is used in this telescope. 1 Eye can see near objects 1 Eye can see far off objects
clearly but cannot see far clearly but cannot see near
Resolving Power of Microscope and Telescope objects clearly because objects clearly because the
the li ght from the for off light from the near by
2 m sin q
(i) The resolving power of microscope is R = object arri ving the eye object arriving the eye lens
l lens may get converged in may get converged at a
where m = refractive index of medium between object and
front of retina. point behind the retina.
objective of microscope and q = angle subtended by a
2 It can be corrected by 2 It can be corrected by
radius of the objective on one of the objects. (When both
concave lens (power of convex lens (Power of
objects are not self illuminous).
concave lens is –ve). convex lens is +ve).
a
(ii) The resolving power of a telescope is R = where
1.22l
a = diameter of objective of telescope. 6. Astigmatism : It is due to different curvature of cornea in
horizontal and vertical plane. It is corrected by using
Keep in Memory
cylindrical lens.
7. Presbyopia : The eye with this defect cannot see near
1. Refracting type Refl ecting type telescope objects as well as far off objects clearly.
telescop e (use of lenses) (use of mi rrors)
Example 27.
1 It suffers from spherical 1 It is alm ost free from A small telescope has an objective of focal length 140cm
aberration and chromatic spherical aberration and
aberration. asbolutely free from
and an eye piece of focal length 5.0cm. What is the
chromatic aberration. magnifying power of the telescope for viewing distant
2 T he lenses used have 2 The aperature of m irror s objects when
small aperature and used is l arge and therefore (a) the telescope is in normal adjustment?
therefore light gathering light gathering power is
(b) the final image is formed at the least distance of
power is small. large.
distinct vision?
Solution :
Here, f0 = 140 cm, fe = 5.0 cm
2. (a) The magnifying power in normal adjustment is given
f 140
by m = 0 = = 28
fe 5
(b) When image is formed at least distance of distinct
vision
f0 é fe ù é 5ù 6
m= ê1 + D ú = 28ê1 + 25 ú = 25 ´ 5 = 33.6
fe ë û ë û

where D = 25cm

Example 28.
A compound microscope has an objective of focal length
1 cm and an eyepiece of focal length 2.5 cm. An object has
Free eBooks on @neetquestionpaper2020

Ray Optics and Optical Instruments 849


to be placed at a distance of 1.2 cm away from the objective,
1 1 1
for normal adjustment. Find (a) the angular magnification = +
and (b) the length of the microscope tube. f ¥ v
Solution : 1 1 1 1
(a) If the first image is formed at a distance v from the or, = - =-
f ¥ 50 50
objective, then we have
Þ f = –50 cm = – 0.5 m
1 1 1
- = or v = 6 cm. 1 1
v (-1.2 cm) 1 cm P= =- = -2 dioptre
f 0.5
The angular magnification in normal adjustment is, Example 31.
v D 6 cm 25 cm In order to correct his near point to 25 cm a man is given
m= =- . = -50. spectacles with converging lenses of 50 cm focal length
u fe 1.2 cm 2.5 cm
and to correct his far point to infinity, he is given diverging
(b) For normal adjustment, the first image must be in the lenses of 200 cm focal length. Determine his far and near
focal plane of the eyepiece. points when not wearing the spectacles.
The length of the tube is, therefore, Solution :
L = v + f e = 6 cm + 2.5 cm = 8. 5 cm For near point correction
u = 25 cm, f = 50 cm, v = ?
Example 29.
A person cannot see objects in nearer than 500 cm from 1 1 1
+ =
the eye. Determine the focal length and the power of u v f
glasses which enable him to read a book 25 cm from his
1 1 1 u -f
eye. = - =
Solution : v f u uf
Let f be the focal length of the glass. Then for the glass, uf 25 ´ 50
u = 25 cm, v = –500 cm v= = = -50 cm
u - f 25 - 50
1 1 1 i.e., his near point is 50 cm.
Q = +
f u v For far point correction
1 1 1 20 - 1 1 1 1
or, = - = u = ¥ , f = –200 cm, v = ? Q + =
f 25 500 500 u v f

500 1 1 1 1 1
or, f = = 26.3 cm = 0.263 m or + =- or =-
19 ¥ v 200 v 200
or v = –200 cm
1 1 i.e., his far point is 200 cm.
P(power) = = = 3.8 dioptre
f 0.263
Example 30. Example 32.
What is the power of the spectacles required (a) by a A person with normal vision has a range of
hypermetropic eye whose near point is 125 cm (b) by a accommodation from 25 cm to infinity. Over what range
myopic eye whose far point is 50 cm ? would he be able to see objects distinctly when wearing
Solution : the spectacles of a friend whose correction is +4 dioptres.
(a) u = 25 cm, v = –125 cm Solution :
1 1 1 1
= + P = 4 dioptres, \ f = m = 25 cm
f u v 4
For near point, v = –25 cm,
1 1 1 5 -1
or, = - = vf -25 ´ 25
f 25 125 125 u= = = 12.5cm
v - f -25 - 25
125 For far point, v = ¥
or, f= = 31.25 cm = 0.3125 m
4
1 1 1
1 1 Q + =
P= = = 3.2 dioptre u v f
f 0.3125
1 1 1
(b) u = ¥ , v = –50 cm + =
u ¥ 25
1 1 1
Q = + or u = 25 cm
f u v Hence, the range of distinct vision is from 12.5 cm to 25 cm.
Free eBooks on @neetquestionpaper2020

850 Physi cs
Example 33. i.e., he should use the converging lens of focal length
Where is the near point of an eye for which a spectacle 28.6 cm.
lens of +1 dioptre is prescribed ? Let x be the object distance for v = 150 cm, then
Solution :
1 1 1 7
P = +1 dioptre. + = =
x 150 f 200
1 1
\f = = = 1 m = 100 cm 1 7 1 21 - 4
P 1 Þ = - =
For the spectacle lens, x 200 150 600
u = 25 cm, f = 100 cm, v = ? 600
Þ x= = 35.3cm
1 1 1 17
Q + =
u v f \ Range of distinct vision is 25 cm to 35.3 cm.

1 1 1 u -f Example 36.
or = - = If the range of vision of short-sighted man is from 10 cm to
v f u uf
20 cm from the eye, what lens should be used in order to
uf 25 ´ 100 enable him to see distinct objects clearly ? What would be
or v = = = -33.33cm the range of accommodation when using this lens ?
u - f 25 - 100
Solution :
i.e., the near point is 33.33 cm from the eye.
For seeing distant objects
Example 34. u = ¥, v = –20 cm, f = ?
A certain person can see clearly at distance between 20 cm
1 1 1 1 1
and 200 cm from his eye. What sepctacles are required to = + = -
enable him to see distant objects clearly and what will be f u v ¥ 20
his least distance of distinct vision when he is wearing them? i.e., he should use the diverging lens of focal length,
Solution : –20 cm.
For seeing distant objects For finding the near point when using the lens
u = ¥ , v = –200 cm, f = ? u = ?, v = –10 cm, f = –20 cm

1 1 1 vf -10( -20)
Q = + u= = = 20 cm
f u v v-f -10 - (-20)
\ the range of vision is 20 cm to infinity.
1 1 1 1
or, = - =- or f = -200cm
f ¥ 200 200 Example 37.
For finding the least distance of distinct vision A person's left eye is corrected by a –8.5 diopter lens,
u = ?, v = –20 cm, f = –200cm. 2.0 cm from the eye.
(a) Is the person near or far sighted ?
vf -20( -200) (20 ´ 200) (b) What is this person's far point without glasses ?
u= = = = 22.2 cm
v - f -20 - ( -200) 180 Solution :
i.e., his least distance of distinct vision is 22.2 cm when he As the diverging lens (–ve power) is used to correct the
is wearing spectacles. eye, the lens is used to diverge the rays coming from distant
objects. So the person is near sighted.
Example 35.
An elderly person cannot see clearly, without the use of 1
spectacles, objects nearer than 200 cm. What spectacles For the lens, P = –8.5 diopter, f = - m = -11.8 cm
8.5
will he need to reduce this distance to 25 cm ? If his eyes u = –¥, v = ?
can focus rays which are converging to points not less
than 150 cm behind them, calculate his range of distinct 1 1 1 1 1
Q + = or =- cm
vision when using the spectacles. v u f v 11.8
Solution : v = –11.8 cm
Here u = 25 cm, v = – 200 cm i.e., the far point is 11.8 cm from the lens or (11.8 + 2) or
1 1 1 13.8 cm from the eye.
Q = +
f u v Example 38.
A lady uses +1.5D glasses to have normal vision from 25 cm.
1 1 1 8 -1
or, = - = onwards. She uses a 20D lens as a simple microscope to see
f 25 200 200 an object. Calculate the maximum magnifying power if she
200 uses the microscope (a) together with her glass (b) without
Þf= = 28.6 cm the glass.
7
Free eBooks on @neetquestionpaper2020

Ray Optics and Optical Instruments 851


Solution : M -30
D 1 For objective, M0 = = = -5 ,
(a) M = 1+ = 1 + PD = 1 + 20 ´ = 6 Me 6
f 4 For objective, if u0 = – x cm, v0 = 5x cm.
100 1000 200 1 1 1 1 1 1
(b) focal length of glasses = = = Again, - = or - =
1.5 15 3 v u f 5x - x 1.25
1 1 3 On simplification, x = 1.5
- =
v -25 100
\ u0 = – 1.5 cm, v0 = 7.5 cm.
1 3 1 3-8 5 1 v
or = - = =- =- For eyepiece, Me = e
v 200 25 200 200 40 ue
v = – 40 cm. ve -25
or u e = = = - 4.17 cm.
Me 6
æ 40 ö
Now, M = 1 + Pv = 1 + 20 ç = 1+ 8 = 9 Distance between objective and eyepiece
è 100 ÷ø
Example 39. = vobjective + | u eyepiece | = 7.5 + 4.17 = 11.67 cm.
An angular magnification (magnifying power) of 30 × is Example 40.
desired using an objective of focal length 1.25 cm. and an A small telescope has an objective lens of focal length
eyepiece of focal length 5 cm. in a compound microscope. 144 cm. and eye-piece of focal length 6.0 cm. What is the
What is the separation between objective and the eyepiece ? magnifying power of the telescope ? What is the separation
Solution : between objective and eye-piece ? Assume normal
Let final image be formed at lease distance of distinct vision adjustment.
Solution :
æ 25 ö
For eyepiece, Me = ç1 + ÷ = æç1 + ö÷ = 6
25
f0 144
è fe ø è 5ø M= = = 24
fe 6
Now, M = M0 × Me
L = f0 + fe = (144 + 6) cm. = 150 cm.

24.3
Solve following problems with the help of above text and 3. To increase the angular magnification of a simple
examples. microscope, one should increase
1. The image formed by an objective of a compound (a) the focal length of the lens
microscope is (b) the power of the lens
(a) real and diminished (c) the aperture of the lens
(b) real and enlarged (d) the object size
(c) virtual and enlarged 4. In which of the following the final image is erect ?
(d) virtual and diminished (a) Simple microscope
2. An astronomical telescope has a large aperture to (b) Compound microscope
(a) reduce spherical aberration (c) Astronomical telescope
(b) have high resolution (d) None of these
(c) increases span of observation
(d) have low dispersion
ANSWER KEY
1. (b) 2. (b) 3. (b) 4. (a)

PHOTOMETRY Df
Luminous intensity, I =
Ray optics is based on the assumption that light travels along Dw
straight line. For isotropic point source,
(i) Luminous flux (f) of a source of light = amount of visible Solid angle,
light energy emitted per second from the source.
The SI unit of luminous fulx (f) is lumen. DA(surface area) 4pr 2
(ii) Luminous intensity (I) of a light source = luminous flux w= 2
= = 4p steradian
r r2
emitted per unit solid angle in any direction.
Its SI unit is candela. ( where DA = 4pr2 = total surface area of sphere of radius r )
Free eBooks on @neetquestionpaper2020

852 Physi cs
DA

o r

Isotropic point source


Let central ray of source s falls perpendicularly on surface
f
so I= or f = 4p. I DA, then luminous flux Df is given by
4p
Df = I×Dw ...(i)
Solid angle : We know that arc of a circle subtends an angle q on DA
the centre of circle O where Dw =
r2
DA
Þ Df = I ´ 2 ...(ii)
o q s r
r Df I I
or = 2 or E = 2 ...(iii)
S Arcof circle DA r r
i.e., q = = ...(i)
r Radius of circle where E is called illuminance or intensity of illumination.
(a) The unit of q (plane angle) is radian. I
Similarly in the case of a sphere, the surface area of sphere If in eq. (iii) I is constant for a given source then E µ 2 .
r
subtends an angle on the centre of sphere O, which is called So intensity of illumination of any source is inversely
solid angle & is denoted by w. proportional to square of the distance between light source
Let radius of sphere is r and a small area DA on its surface & surface. This is called inverse square law.
subtends a solid angle w at the centre then
DA Lambert's Cosine Law for Illuminance
w= = constant ..(ii)
r2 Let S is unidirectional point source & its luminous intensity is I.
There is a surface of area DA at distance r from S, which is kept in
such a way that light from S falls obliquely on it and central ray
makes an angle q with normal to DA.
Then by fig. DA¢ = DA cos q
(b)

The unit of solid angle is steradian. If in eq (ii)


DA = r2 , then w = 1 steradian
If DA = 4pr2 (total surface area of sphere)
then w = 4p steradian.
(iii) Illuminance (E) of a surface is the luminous flux incident
normally on unit area of the surface. According to definition of luminous intensity :
Its unit is lux. Df = I × Dw
DA' DAcos q I ´ DA cos q
Df where Dw = 2 = Þ Df =
E= r r 2 r2
DA
Df I cos q
For point source, the total normal area will be 4pr2, or E= =
DA r2
f f 1
so E= = 2
Þ Eµ 2 For any given source (I constant) & at a fixed distance (r constant)
A 4pr r E µ cos q
(iv) Luminance or brightness of a surface is the luminous flux i.e., the, intensity of illumination of a surface is proportional to the
reflected into our eyes from unit area of the surface. cosine of angle of the inclination of the surface. This is called
The unit of Brightness is lambert. Lambert’s cosine law. As q increases, cos q decreases &
(v) Inverse square law for illuminance : Let S is a unidirectional consequently E decreases.
point source, whose luminous intensity is I. It has some q is the angle between normal to the area and direction of light
surface DA at distance r from source S. propagation.
Free eBooks on @neetquestionpaper2020

Ray Optics and Optical Instruments 853


Example 41.
Ih
What is the effect on the intensity of illumination on a \ E2 = ...(3)
table if a lamp hanging 2 m directly above it is lowered by (h + r 2 )3 / 2
2
0.5 m? Dividing eq. (1) by (2), we get
Solution :
I I E1 I / h2 (h 2 + r 2 )3 / 2
E1 = 2 and E 2 = 2 = =
r1 r2 E 2 I h /(h 2 + r 2 ) 3 / 2 h3
2 2
E 2 æ r1 ö æ 2 ö æ 4 ö
\ = çç ÷÷ = ç ÷ =ç ÷ æ h2 + r2 ö
3/ 2
æ
3/ 2
E1 è r2 ø è 1. 5 ø è 2.25 ø r 2 ö÷
=ç ÷ = ç1 +
Fractional increase in the intensity ç h2 ÷ ç h2 ÷
è ø è ø
E 2 - E1 æ E 2 ö æ 4 ö Example 43.
= = çç - 1÷÷ = ç - 1÷ ´ 100 = 78%
E1 è E1 ø è 2 .25 ø A lamp of power P is suspended at the centre of a circular
Example 42. table of radius r. What should be the height of the lamp
An isotropic point source of light is suspended h metres above the table so that maximum intensity is produced at
vertically above the centre of a circular table of radius the edge?
r metres. Then determine the ratio of illuminance at the Solution :
centre to that at the edge of the table. See figure, the intensity of illumination at edge
Solution : (i.e., at point A)
See fig. at point O, the light is
falling normally. I cos q I1 cos q
E= =
Hence illumination at O is given by ( LA ) 2
(h 2 + r 2 )
I I
E1 = = ...(1) h
2
(LO) h2 From figure. cos q =
The illumination at the edge A (h + r 2 )
2

is given by
Ih
I cos q I cos q \E =
E2 = = ...(2)
( LO ) 2 (h 2 + r 2 ) (h 2 + r 2 )3
h For maximum intensity, dE/dh = 0
From figure cos q =
2 2
(h + r ) Applying this condition, we get h = r 2

24.4
Solve following problems with the help of above text and 1 1
examples. (a) 2 (b)
r r3
1. The one parameter that determines the brightness of a 1
light source sensed by an eye is (c) (d) None of these
r
(a) energy of light entering the eye per second
4. Select the incorrect option.
(b) wavelength of the light
(a) Luminous flux and radiant flux have same dimensions
(c) total radiant flux entering the eye
(b) Luminous flux and luminous intensity have same
(d) total luminous flux entering the eye
dimensions
2. As the wavelength is increased from violet to red, the
(c) Radiant flux and power have same dimensions
luminosity
(d) Relative luminosity is a dimensionless quantity
(a) continuously increases
5. Luminous flux and luminous intensity are related as
(b) continuously decreases
(c) first increases then decreases I
(a) f = 4 p I (b) f =
(d) first decreases then increases 4p
3. The intensity produced by a long cylindrical light source 4p
(c) f = (d) I = 4 p f
at a small distance r from the source is proportional to I

ANSWER KEY
1. (d) 2. (c) 3. (c) 4. (a) 5. (a)
Free eBooks on @neetquestionpaper2020

854 Physi cs

Very Short / Short Answer Questions (iii) In a single slit diffraction experiment, the width of the
slit is reduced to half its original width. How would this
1. Deduce, with the help of ray diagram the expression for the
affect the size and intensity of the central maximum?
mirror equation in case of convex mirror.
14. You are given three lenses L1, L2 and L3each of focal length
[Outside Delhi - 2012 COMPTT.]
10 cm. An object is kept at 15 cm in front of L1, as shown.
2. (a) Draw a ray diagram for a concave mirror showing the
The final real image is formed at the focus ‘T’ of L3. Find the
image formation of an object placed anywhere in front
separations between L1, L2 and L3.
of a mirror.
[Outside Delhi - 2012 COMPTT.]
(b) Using the ray diagram, obtain the expression for its
linear magnification. [Outside Delhi - 2012 COMPTT.] L1 L2 L3
3. Why are convex mirrors used as side view mirrors in
vehicles? [Delhi Board - 2011 COMPTT.] I
15 cm 10 cm
4. How would a biconvex lens appear when placed in a trough
of liquid having the same refractive index as that of the
lens? [Delhi Board - 2012 COMPTT.] 15. Drive the prism formula,
5. State snell’s law of refraction.
(A + dm )
6. Define critical angle. sin
(a) n12 = 2
7. How are the rainbows formed? A
Sin
8. What do you mean by the total internal reflection? Give the 2
two conditions for TIR of light to take place.
(b) Draw the graph showing the variation of the angle of
9. Derive the relation, d = ( n12 –1) A. deviation with angle of incidence, through a prism.
10. Explain with the help of lens maker’s formula. Why does a 16. A small object is placed at distance of 40 cm from a convex
convex lens behave as spherical refracting surface of radius of curvature 15 cm. If
(i) converging lens when immersed in water (m = 1.33) and the surface separates air from glass of refractive index 1.5,
(ii) a diverging lens when immersed in CS2 solution find the position of the image.
(m = 1.6). 17. A convex lens of focal length 20 cm and a concave lens of
focal length 5 cm are kept along the same axis with a
Long Answer Questions
separation ‘d’ between them. What is the value of d, if a
11. Define power of a lens. Write its S.I. unit. Two thin convex parallel beam of light incident on convex lens, leaves the
lenses of focal lengths f1 and f2 are placed in contact concave lens as a parallel beam?
coaxially. Derive the expression for the effective focal length Multiple Choice Questions
of the combination. [Delhi Board - 2010 COMPTT.]
12. Draw a ray diagram to show the formation of the image of an 18. What will be the colour of the sky as seen from the earth if
object placed on the axis of a convex refracting surface, of there were no atmosphere?
radius of curvature ‘R’, separating the two media of (a) Black (b) Blue
refractive indices ‘n1’ and ‘n2’ (n2 > n1). Use this diagram to (c) Orange (d) Red
n n n –n 19. A man wants to see two poles, separately, situated at 11 km.
deduce the relation 2 – 1 = 2 1 , where u and v
v u R The minimum distance (approximately) between these poles
represent respectively the distance of the object and the
will be
image formed. [Delhi Board - 2011 COMPTT.]
(a) 5 m (b) 0.5 m
13. Answer the following : [Delhi Board - 2011 COMPTT.] (c) 1 m (d) 3 m
(i) Do the frequency and wavelength change when light 20. The index of refraction of diamond is 2.0. The velocity of
passes from a rarer to a denser medium? light in diamond is approximately
(ii) Why is the value of the angle of deviation for a ray of
(a) 1.5 × 1010 cm/sec (b) 2 × 1010 cm/sec
light undergoing refraction through a glass prism 10
(c) 3.0 × 10 cm/sec (d) 6 × 1010 cm/sec
different for different colours of light?
Free eBooks on @neetquestionpaper2020

Ray Optics and Optical Instruments 855


21. In a room containing smoke particles, the intensity due to a (a) It may be erect or inverted
source of light will (b) It may be magnified or diminished
(a) obey the inverse square law (c) It may be real or virtual
(b) be constant at all distances (d) Real image may be between the pole and focus or beyond
(c) increase with distance from the source than the focus
inverse fourth power law 24. Critical angle of light passing from glass to water is minimum
(d) fall faster with distance from the source than the for
inverse fourth power law (a) red colour (b) green colour
22. What causes chromatic aberration? (c) yellow colour (d) violet colour
(a) Non - paraxial rays 25. A concave lens of glass, refractive index 1.5 has both surfaces
(b) Paraxial rays of same radius of curvature R. On immersion in a medium of
(c) Variation of focal length with colour refractive index 1.75, it will behave as a
(d) Difference in radii of curvature of the bounding surfaces (a) convergent lens of focal length 3.5 R
of the lens (b) convergent lens of focal length 3.0 R
23. Which of the following is not the case with the image formed (c) divergent lens of focal length 3.5 R
by a concave lens? (d) divergent lens of focal length 3.0 R

1. A convex lens of focal length 80 cm and a concave lens of 6. A luminous object is placed at a distance of 30 cm from the
focal length 50 cm are combined together. What will be their convex lens of focal length 20 cm. On the other side of the
resulting power? [CBSE PMT 1996] lens, at what distance from the lens a convex mirror of radius
(a) + 6.5 D (b) –6.5 D (c) + 7.5 D (d) –0.75 D of curvature 10 cm be placed in order to have an upright
2. If fV and fR are the focal lengths of a convex lens for violet image of the object coincident with it? [CBSE PMT 1998]
and red light respectively and FV and FR are the focal lengths (a) 12 cm (b) 30 cm (c) 50 cm (d) 60 cm
of concave lens for violet and red light respectively, then we
7. Light enters at an angle of incidence in a transparent rod of
have [CBSE PMT 1996]
refractive index n. For what value of the refractive index of
(a) fV < fR and FV > FR (b) fV < fR and FV < FR
the material of the rod the light once entered into it will not
(c) fV > fR and FV > FR (d) fV > fR and FV < FR
leave it through its lateral face whatsoever be the value of
3. Light travels through a glass plate of thickness t and
angle of incidence? [CBSE PMT 1998]
refractive index m. If c is the speed of light in vacuum, the
time taken by light to travel this thickness of glass is (a) n > 2 (b) n = 1 (c) n = 1.1 (d) n = 1.3
[CBSE PMT 1996]
8. A plano-convex lens is made of material of refractive index
t
(a) mtc (b) tc (c) (d) mt 1.6. The radius of curvature of the curved surface is 60 cm.
m mc c The focal length of the lens is [CBSE PMT 1999]
4. One face of a rectangular glass plate 6 cm thick is silvered. An (a) 50 cm (b) 100 cm (c) 200 cm (d) 400 cm
object held 8 cm in front of the first face, forms an image 12 cm
behind the silvered face. The refractive index of the glass is 9. The refractive index of the material of the prism is 3 ; then
[CBSE PMT 1996] the angle of minimum deviation of the prism is
(a) 0.4 (b) 0.8 (c) 1.2 (d) 1.6 [CBSE PMT 1999]
5. An electromagnetic radiation of frequency n, wavelength l, (a) 30º (b) 45º (c) 60º (d) 75º
travelling with velocity v in air enters in a glass slab of 10. The radius of curvature of a thin plano-convex lens is 10 cm
refractive index (m). The frequency, wavelength and velocity (of curved surface) and the refractive index is 1.5. If the
of light in the glass slab will be respectively
plane surface is silvered, then it behaves like a concave
[CBSE PMT 1997] mirror of focal length [CBSE PMT 2000]
l v v (a) 10 cm (b) 15 cm (c) 20 cm (d) 5 cm
(a) n, and (b) n, 2l and
m m m 11. A person is six feet tall. How tall must a vertical mirror be if
n l v 2p l he is able to see his entire length? [CBSE PMT 2000]
(c) , and (d) , and v
m m m m m (a) 3 ft (b) 4.5 ft (c) 7.5 ft (d) 6 ft
Free eBooks on @neetquestionpaper2020

856 Physi cs
12. An air bubble in a glass slab (m = 1.5) is 5 cm deep when 18. Two thin lenses of focal lengths f1 and f2 are in contact and
viewed from one face and 2 cm deep when viewed from the coaxial. Its power is same as power of a single lens given by
opposite face. The thickness of the slab is [CBSE PMT 2008]
[CBSE PMT 2000]
(a) 7.5 cm (b) 10.5 cm (c) 7 cm (d) 10 cm f1 + f 2 æ f1 ö æ f2 ö f1 + f 2
13. A light ray falls on a rectangular glass slab as shown. The (a) (b) ç ÷ (c) ç ÷ (d)
f1f 2 çf ÷ çf ÷ 2
index of refraction of the glass, if total internal reflection is è 2ø è 1ø
to occur at the vertical face, is [CBSE PMT 2002] 19. A ray of light travelling in a transparent medium of refractive
45º index m , falls on a surface separating the medium from air at
an angle of incidence of 45°. For which of the following
value of m the ray can undergo total internal reflection?
Glass [CBSE-PMT 2010]
(a) m = 1.33 (b) m = 1.40 (c) m = 1.50 (d) m = 1.25
20. Which of the following is not due to total internal reflection?

(a) 3 / 2 (b)
( 3 + 1) (c)
(
2 +1 ) (d) 5/2
(a) Working of optical fibre [CBSE-PMT 2011]
(b) Difference between apparent and real depth of pond
2 2
(c) Mirage on hot summer days
14. An equiconvex lens is cut into two halves along (i) XOX'
(d) Brilliance of diamond
and (ii) YOY' as shown in the figure. Let f, f', f'' be the focal
21. A biconvex lens has a radius of curvature of magnitude 20
lengths of the complete lens, of each half in case (i), and of
cm. Which one of the following options best describe the
each half in case (ii), respectively [CBSE PMT 2003]
image formed of an object of height 2 cm placed 30 cm from
the lens? [CBSE-PMT 2011]
(a) Virtual, upright, height = 1 cm
(b) Virtual, upright, height = 0.5 cm
(c) Real, inverted, height = 4 cm
(d) Real, inverted, height = 1cm
22. A thin prism of angle 15º made of glass of refractive index
µ1 = 1.5 is combined with another prism of glass of refractive
index µ2 = 1.75. The combination of the prism produces
Choose the correct statement from the following dispersion without deviation. The angle of the second prism
(a) f ' = 2f, f '' = 2f (b) f ' = f, f '' = 2f should be [CBSE-PMT 2011 M]
(c) f ' = 2f, f '' = f (d) f ' = f, f '' = f (a) 7° (b) 10° (c) 12° (d) 5°
15. A convex lens is dipped in a liquid whose refractive index is 23. For the angle of minimum deviation of a prism to be equal to
equal to the refractive index of the lens. Then its focal length its refracting angle, the prism must be made of a material
will [CBSE PMT 2003] whose refractive index [CBSE-PMT 2012 M]
(a) remain unchanged
(a) lies between 2 and 1
(b) become zero
(c) become infinite (b) lies between 2 and 2
(d) become small, but non-zero (c) is less than 1
16. The refractive index of the material of a prism is Ö2 and its (d) is greater than 2
refracting angle is 30º. One of the refracting surfaces of the 24. A rod of length 10 cm lies along the principal axis of a
prism is made a mirror inwards. A beam of monochromatic concave mirror of focal length 10 cm in such a way that its
light enters the prism from the mirrored surface if its angle end closer to the pole is 20 cm away from the mirror. The
of incidence of the prism is [CBSE PMT 2004] length of the image is : [CBSE-PMT 2012 M]
(a) 30º (b) 45º (c) 60º (d) 0º (a) 10 cm (b) 15 cm (c) 2.5 cm (d) 5 cm
17. A telescope has an objective lens of 10 cm diameter and is 25. When a biconvex lens of glass having refractive index 1.47
situated at a distance of one kilometer from two objects. is dipped in a liquid, it acts as a plane sheet of glass. This
The minimum distance between these two objects, which implies that the liquid must have refractive index
can be resolved by th e telescope, when the mean (a) equal to that of glass [CBSE-PMT 2012 S]
wavelength of light is 5000 Å, is of the order of (b) less than one
[CBSE PMT 2004] (c) greater than that of glass
(a) 5 cm (b) 0.5 m (c) 5 m (d) 5mm (d) less than that of glass
Free eBooks on @neetquestionpaper2020

Ray Optics and Optical Instruments 857


26. A ray of light is incident at an angle of incidence, i, on one 34. Which of the following is used in optical fibres?
face of prism of angle A (assumed to be small) and emerges (a) Total internal reflection [AIEEE 2002]
normally from the opposite face. If the refractive index of (b) Scattering
the prism is m, the angle of incidence i, is nearly equal to
(c) Diffraction
[CBSE-PMT 2012 S]
(d) Refraction
mA A A 35. Consider telecommunication through optical fibres.
(a) mA (b) (c) (d)
2 m 2m Which of the following statements is not true?
27. A concave mirror of focal length ‘f1’ is placed at a distance [AIEEE 2003]
of 'd’ from a convex lens of focal length ‘f2’. A beam of light (a) Optical fibres can be of graded refractive index
coming from infinity and falling on this convex lens-concave (b) Optical fibres are subject to electromagnetic
mirror combination returns to infinity. interference from outside
The distance ‘d’ must be equal to [CBSE-PMT 2012 S]
(c) Optical fibres have extremely low transmission loss
(a) f1 + f2 (b) –f1 + f2 (c) 2f1 + f2 (d) –2f1 + f2
(d) Optical fibres may have homogeneous core with a
28. The magnifying power of a telescope is 9. When it is suitable cladding
adjusted for parallel rays the distance between the objective
36. The image formed by an objective of a compound
and eyepiece is 20 cm. The focal length of lenses are
microscope is [AIEEE 2003]
[CBSE-PMT 2012 S]
(a) virtual and diminished
(a) 10 cm, 10 cm (b) 15 cm, 5 cm
(c) 18 cm, 2 cm (d) 11 cm, 9 cm (b) real and diminished
29. A plano convex lens fits exactly into a plano concave lens. (c) real and enlarged
Their plane surfaces are parallel to each other. If lenses are (d) virtual and enlarged
made of different materials of refractive indices m1 and m2 37. To get three images of a single object, one should have two
and R is the radius of curvature of the curved surface of the plane mirrors at an angle of [AIEEE 2003]
lenses, then the focal length of the combination is (a) 60° (b) 90° (c) 120° (d) 30°
(NEET 2013) 38. A light ray is incident perpendicularly to one face of a 90°
R R prism and is totally internally reflected at the glass-air
(a) (b)
2 ( m1 - m 2 ) (m1 - m 2 ) interface. If the angle of reflection is 45°, we conclude that
2R R the refractive index [AIEEE 2004]
(c) (d)
(m 2 - m1 ) 2 ( m1 + m 2 )
30. For a normal eye, the cornea of eye provides a converging
power of 40D and the least converging power of the eye
lens behind the cornea is 20D. Using this information, the
distance between the retina and the eye lens of the eye can
be estimated to be (NEET 2013)
(a) 2.5 cm (b) 1.67 cm (c) 1.5 cm (d) 5 cm
31. An astronomical telescope has a large aperture to
(a) reduce spherical aberration [AIEEE 2002]
(b) have high resolution 1
(a) n> (b) n> 2
(c) increase span of observation 2
(d) have low dispersion 1
(c) n< (d) n< 2
32. If two mirrors are kept at 60° to each other, then the number 2
of images formed by them is [AIEEE 2002]
39. A plano convex lens of refractive index 1.5 and radius of
(a) 5 (b) 6 (c) 7 (d) 8
curvature 30 cm. Is silvered at the curved surface. Now this
33. Wavelength of light used in an optical instrument are
o o lens has been used to form the image of an object. At what
l1 = 4000 A and l 2 = 5000 A , then ratio of their
distance from this lens an object be placed in order to have
respective resolving powers (corresponding to ll and l2) is
a real image of size of the object [AIEEE 2004]
[AIEEE 2002]
(a) 60 cm (b) 30 cm (c) 20 cm (d) 80 cm
(a) 16 : 25 (b) 9 : 1 (c) 4 : 5 (d) 5 : 4.
Free eBooks on @neetquestionpaper2020

858 Physi cs
40. A fish looking up through the water sees the outside world 0 1L 5L 6L
contained in a circular horizon. If the refractive index of
heat A B 3K E
4
water is and the fish is 12 cm below the surface, the radius 1L
3 6K
of this circle in cm is [AIEEE 2005] C 4K
36 3L 2K
(a) (b) 36 7 (c) 4 5 (d) 36 5
7 4L D 5K
41. A thin glass (refractive index 1.5) lens has optical power of (a) heat flow through A and E slabs are same.
– 5 D in air. Its optical power in a liquid medium with refractive (b) heat flow through slab E is maximum.
index 1.6 will be [AIEEE 2005] (c) temperature difference across slab E is smallest.
(a) – 1D (b) 1 D (c) – 25 D (d) 25 D (d) heat flow through C = heat flow through B + heat flow
42. The refractive index of a glass is 1.520 for red light and 1.525 through D.
for blue light. Let D1 and D2 be angles of minimum deviation 47. An object 2.4 m in front of a lens forms a sharp image on a
for red and blue light respectively in a prism of this glass. film 12 cm behind the lens. A glass plate 1 cm thick, of
Then, [AIEEE 2006] refractive index 1.50 is interposed between lens and film
(a) D1 < D2 with its plane faces parallel to film. At what distance (from
lens) should object shifted to be in sharp focus of film?
(b) D1 = D2
[AIEEE 2012]
(c) D1 can be less than or greater than D2 depending upon (a) 7.2 m (b) 2.4 m (c) 3.2 m (d) 5.6 m
the angle of prism 48. Diameter of a plano-convex lens is 6 cm and thickness
(d) D1 > D2 at the centre is 3 mm. If speed of light in material of lens is
43. Two lenses of power –15 D and +5 D are in contact with 2 × 108 m/s, the focal length of the lens is [JEE Main 2013]
each other. The focal length of the combination is (a) 15 cm (b) 20 cm (c) 30 cm (d) 10 cm
[AIEEE 2007] 49. The graph between angle of deviation (d) and angle of
(a) + 10 cm (b) – 20 cm incidence (i) for a triangular prism is represented by
[JEE Main 2013]
(c) – 10 cm (d) + 20 cm
44. A student measures the focal length of a convex lens by
(a) (b)
putting an object pin at a distance ‘u’ from the lens and d d
measuring the distance ‘v’ of the image pin. The graph
between ‘u’ and ‘v’ plotted by the student should look like
[AIEEE 2008]
o o
v(cm) i i
v(cm)

(a) (b) (c) (d)


O O u(cm)
d d
u(cm)

v(cm)
v(cm)
o o
i i
(c) (d) O 50. A glass prism of refractive index 1.5 is immersed in water
O u(cm) u(cm)
(refractive index 4/3). A light beam incident normally on the
45. An experment is performed to find the refractive index of face AB is totally reflected to reach on the face BC if
glass using a travelling microscope. In this experiment [IIT JEE 1981]
distances are measured by [AIEEE 2008] A B
(a) a vernier scale provided on the microscope q
(b) a standard laboratory scale
(c) a meter scale provided on the microscope
(d) a screw gauge provided on the microscope
46. A composite block is made of slabs A, B, C, D and E of
different thermal conductivities (given in terms of a constant
K and sizes (given in terms of length, L) as shown in the 8 2 8
(a) sin q ³ (b) < sin q <
figure. All slabs are of same width. Heat ‘Q’ flows only from 9 3 9
left to right through the blocks. Then in steady state 2
[AIEEE 2011] (c) sin q £ (d) None of these
3
Free eBooks on @neetquestionpaper2020

Ray Optics and Optical Instruments 859


51. A ray of light from a denser medium strike a rarer medium at (a) 0.50 ± 0.05 cm (b) 0.50 ± 0.10 cm
an angle of incidence i (see Fig). The reflected and refracted (c) 5.00 ± 0.05 cm (d) 5.00 ± 0.10 cm
rays make an angle of 90° with each other. The angles of
55. Rays of light from Sun falls on a biconvex lens of focal
reflection and refraction are r and r’. The critical angle is
length f and the circular image of Sun of radius r is formed
[IIT JEE 1983]
on the focal plane of the lens. Then [IIT JEE 2006]
(a) area of image is pr2 and area is directly proportional to
f
i r
(b) area of image is pr2 and area is directly proportional to
f2
r' (c) intensity of image increases if f is increased
(d) if lower half of the lens is covered with black paper area
(a) sin–1(tan r) (b) sin–1(tan i) will become half
(c) sin–1(tan r¢) (d) tan–1(tan i) 56. A biconvex lens of focal length 15 cm is in front of a plane
52. An equilateral prism is placed on a horizontal surface. A ray mirror. The distance between the lens and the mirror is
PQ is incident onto it. For minimum deviation 10 cm. A small object is kept at a distance of 30 cm from the
[IIT JEE 2004 S] lens. The final image is [IIT-JEE 2010]
(a) virtual and at a distance of 16 cm from the mirror
R (b) real and at a distance of 16 cm from the mirror
Q S
(c) virtual and at a distance of 20 cm from the mirror
P
(d) real and at a distance of 20 cm from the mirror
57. Two transparent media of refractive indices m1 and m3have
(a) PQ is horizontal a solid lens shaped transparent material of refractive index
(b) QR is horizontal m2 between them as shown in figures in column II. A ray
(c) RS is horizontal traversing these media is also shown in the figures. In
(d) Any one will be horizontal column I different relationships between m1, m2, and m3 are
53. A ray of light passes through four transparent media with given. Match them to the ray diagrams shown in Column II.
refractive indices µ1, µ2, µ3 and µ4 as shown in the figure. [IIT-JEE 2010]
The surfaces of all media are parallel. If the emergent ray CD Column I Column II
is parallel to the incident ray AB, we must have
[IIT JEE 2001 S]

µ1 µ2 µ3 D (A) m1 < m 2 (p) m3 m2 m1


µ4

C
B
A

(a) µ1 = µ2 (b) µ2 = µ3 (c) µ3 = µ4 (d) µ4 = µ1


54. The graph shows relationship between object distance and
(B) m1 > m 2 (q) m3 m2 m1
image distance for an equiconvex lens. Then, focal length
of the lens is [IIT JEE 2006]

31
v cm
30

(C) m 2 = m3 (r)
10 m3 m2 m1

0 (–9, +9)
u cm –31 –30 –20 –10
Free eBooks on @neetquestionpaper2020

860 Physi cs
60. The image of an object, formed by a plano-convex lens at a
distance of 8 m behind the lens, is real is one-third the size
2
of the object. The wavelength of light inside the lens is
3
(D) m 2 > m 3 (s) times the wavelength in free space. The radius of the curved
surface of the lens is (JEE Adv. 2013)
m3 m2 m1
(a) 1 m (b) 2 m (c) 3 m (d) 6 m
1
61. A ray of light travelling in the direction (iˆ + 3 ˆj ) is
2
incident on a plane mirror. After reflection, it travels along
1
the direction (iˆ - 3 ˆj ) . The angle of incidence is
2
(t) JEE Adv. 2013)
m3 m2 m1 (a) 30° (b) 45° (c) 60° (d) 75°
62. A right angled prism of refractive index m1 is placed in a
4 rectangular block of refractive index m2, which is surrounded
58. Water (with refractive index = ) in a tank is 18 cm deep. Oil
3 by a medium of refractive index m3, as shown in the figure.
7
of refractive index lies on water making a convex surface A ray of light 'e' enters the rectangular block at normal
4 incidence. Depending upon the relationships between m1,
of radius of curvature ‘R = 6 cm’ as shown. Consider oil to
m2 and m3, it takes one of the four possible paths 'ef', 'eg', 'eh'
act as a thin lens. An object ‘S’ is placed 24 cm above water or 'ei'. (JEE Adv. 2013)
surface. The location of its image is at ‘x’ cm above the
bottom of the tank. Then ‘x’ is [IIT-JEE 2011]
f
S
(a) 5 45°
R = 6 cm m = 1.0 e g
(b) 6
m = 7/4 m1 h
(c) 8 i
m = 4/3
(d) 9
m3
m2
59. A bi-convex lens is formed with two thin plano-convex lenses
as shown in the figure. Refractive index n of the first lens is
Match the paths in List I with conditions of refractive indices
1.5 and that of the second lens is 1.2. Both the curved surface
in List II and select the correct answer using the codes
are of the same radius of curvature R = 14 cm. For this bi-
given below the lists:
convex lens, for an object distance of 40 cm, the image
distance will be [IIT-JEE 2012] List I List II
P. e®f 1. m1 > 2m 2
Q. e®g 2. m2 > m1 and m2 > m3
R. e®h 3. m1 = m2
S. e®i 4. m2 < m1< 2m 2 and m2 > m3
Codes:
P Q R S
(a) 2 3 1 4
(a) -280.0 cm (b) 40.0 cm (b) 1 2 4 3
(c) 21.5 cm (d) 13.3 cm (c) 4 1 2 3
(d) 2 3 4 1
Free eBooks on @neetquestionpaper2020

Ray Optics and Optical Instruments 861

1. Which of the following properties shows that light is a 8. Luminous flux and luminous intensity are related as
transverse wave ? I
(a) f = 4p I (b) f =
(a) Reflection (b) Interference 4p
4p
(c) Diffraction (d) Polarization (c) f = (d) I = 4 p f
I
2. Two sources are called coherent if they produce waves 9. The luminous intensity of 100 W unidirectional bulb is 100
(a) of equal wavelength candela. The total luminous flux emitted from bulb will be
(b) of equal velocity (a) 100 p lumen (b) 200 p lumen
(c) having same shape of wavefront (c) 300 p lumen (d) 400 p lumen
10. A lamp of 250 candle power is hanging at a distance of 6 m
(d) having a constant phase difference from a wall. The illuminance at a point on the wall at a
3. Which of the following properties of light conclusively minimum distance from lamp will be
support wave theory of light ? (a) 9.64 lux (b) 4.69 lux
(a) Light obeys laws of reflection (c) 6.94 lux (d) None of these
(b) Speed of light in water is smaller than the speed in 11. An electric bulb illuminates a plane surface. The intensity
vacuum of illumination on the surface at a point 2 m away from the
bulb is 5 × 10–4 phot (lumen/cm2). The line joining the bulb
(c) Light doesn't show interference to the point makes an angle of 60º with the normal to the
(d) Light shows photoelectric effect surface. The luminous intensity of the bulb in candela
4. A normal eye is not able to see objects closer than 25 cm (candle power) is
because (a) 40 3 (b) 40
(a) the focal length of the eye is 25 cm (c) 20 (d) 40 × 10–4
(b) the distance of the retina from the eye-lens is 25 cm 12. If two mirrors are kept at 60º to each other, then the number
(c) the eye is not able to decrease the distance between of images formed by them is
the eye-lens and the retina beyond a limit (a) 5 (b) 6 (c) 7 (d) 8
(d) the eye is not able to decrease the focal length beyond 13. Wavelength of light used in an optical instrument are
l1 = 4000Å and l2 = 5000 Å, then ratio of their respective
a limit
resolving powers (corresponding to l1 and l2) is
5. The one parameter that determines the brightness of a light (a) 16 : 25 (b) 9 : 1 (c) 4 : 5 (d) 5 : 4
source sensed by an eye is 14. The critical angle for light going from medium X into medium
(a) energy of light entering the eye per second Y is q. The speed of light in medium X is v, then speed of
(b) wavelength of the light light in medium Y is
(a) v(1 – cos q) (b) v/sin q
(c) total radiant flux entering the eye
(c) v/cos q (d) v cos q
(d) total luminous flux entering the eye 15. Monochromatic light of wavelength l1 travelling in a
6. As the wavelength is increased from violet to red, the medium of refractive index m1 enters a denser medium of
luminosity refractive index m2. The wavelength in the second medium
(a) continuously increases is
(a) l1 (m1/m2) (b) l1 (m2/m1)
(b) continuously decreases
(c) l1 (m2 – m1)/m2 (d) l1 (m2 – m1)/m1
(c) increases then decreases 16. A vessel of depth 2d cm is half filled with a liquid of refractive
(d) decreases then increases index m1 and the upper half with a liquid of refractive index
7. The intensity produced by a long cylindrical light source at m2. The apparent depth of the vessel seen perpendicularly
a small distance r from the source is proportional to is

1 1 æ m1 m 2 ö æ 1 1 ö
(a) (b) (a) ç ÷d (b) çç m + m ÷÷ d
r 2
r 3 çm +m ÷
è 1 2 ø è 1 2ø

1 æ 1 1 ö æ 1 ö
(c) (d) None of these ç ÷ ç ÷
r (c) ç m + m ÷ 2d (d) ç m m ÷ 2d
è 1 2ø è 1 2ø
Free eBooks on @neetquestionpaper2020

862 Physi cs
17. Light travels through a glass plate of thickness t and having
æ 10 t 2 ö æ t x ö
refractive index m. If c be the velocity of light in vacuum, the (a) sin -1 çç ÷÷ (b) sin -1 çç 2 ÷÷
time taken by the light to travel this thickness of glass is è t1x ø è 10 t 1 ø
t mt tc
(a) (b) t mc (c) (d) æ 10 t 1 ö æ t x ö
mc c m (c) sin -1 çç ÷÷ (d) sin -1 çç 1 ÷÷
è t 2x ø è 10 t 2 ø
18. The refractive index of water is 1.33. What will be speed of
light in water ? 27. A double convex lens of focal length 6 cm is made of glass
(a) 3 × 108 m/s (b) 2.25 × 108 m/s of refractive index 1.5. The radius of curvature of one surface
8
(c) 4 × 10 m/s (d) 1.33 × 108 m/s is double that of other surface. The value of small radius of
curvature is
19. A beam of monochromatic blue light of wavelength 420 nm
in air travels in water (m = 4/3). Its wavelength in water will (a) 6 cm (b) 4.5 cm (c) 9 cm (d) 4 cm
be 28. A prism has a refracting angle of 60º. When placed in the
(a) 280 nm (b) 560 nm (c) 315 nm (d) 400 nm position of minimum deviation, it produces a deviation of
20. When light falls on a given plate at angle of incidence of 30º. The angle of incidence is
60º, the reflected and refracted rays are found to be normal (a) 30º (b) 45º (c) 15º (d) 60º
to each other. The refractive index of the matertial of the 29. A ray of light passes through an equilateral prism such that
plate is then the angle of incidence is equal to the angle of emergence
(a) 0.866 (b) 1.5 (c) 1.732 (d) 2 and the latter is equal to 3/4th of the angle of prism. The
21. A convex mirror of focal length f produces an image angle of deviation is
(1/n)th of the size of the object. The distance of the object (a) 45º (b) 39º (c) 20º (d) 30º
from the mirror is 30. An achromatic convergent doublet of two lenses in contact
(a) (n – 1) f (b) f/n has a power of + 2D. The convex lens has power + 5D. What
(c) (n + 1) f (d) nf is the ratio of dispersive powers of convergent and divergent
22. A concave mirror of focal length f produces an image n lenses ?
times the size of object. If image is real, then distance of (a) 2 : 5 (b) 3 : 5 (c) 5 : 2 (d) 5 : 3
object from mirror, is 31. The dispersive power of material of a lens of focal length 20
(a) (n – 1) f (b) { (n – 1)/n} f cm is 0.08. What is the longitudinal chromatic aberration of
(c) { (n + 1)/n} f (d) (n + 1) f the lens ?
23. In a concave mirror, an object is placed at a distance x1 from (a) 0.08 cm (b) 0.08/20 cm
focus, and image is formed at a distance x2 from focus. Then (c) 1.6 cm (d) 0.16 cm
focal length of mirror is 32. The magnifying power of a telescope is 9. When it is
x1 - x 2 adjusted for parallel rays, the distance between the objective
(a) x1 x 2 (b) and the eye piece is found to be 20 cm. The focal length of
2
lenses are
x1 + x 2 x1 (a) 18 cm, 2 cm (b) 11 cm, 9 cm
(c) (d) x2 (c) 10 cm, 10 cm (d) 15 cm, 5 cm
2
33. The focal length of the objective of a telescope is 60 cm. To
24. A convex lens of focal length f1 and a concave lens of focal obtain a magnification of 20, the focal length of the eye
length f2 are placed in contact. The focal length of the piece should be
combination is (a) 2 cm (b) 3 cm (c) 4 cm (d) 5 cm
(a) (f1 + f2) (b) (f1 – f2) 34. The focal lengths of objective and eye lens of an
f1f 2 f1f 2 astronomical telelscope are respectively 2 meter and 5 cm.
(c) f 2 - f1 (d) f1 + f 2 Final image is formed at (i) least distance of distinct vision
(ii) infinity Magnifying power in two cases will be
25. A lens of power + 2 diopter is placed in contact with a lens
(a) – 48, – 40 (b) – 40, – 48
of power – 1 diopter. The combination will behave like
(c) – 40, + 48 (d) – 48, + 40
(a) a convergent lens of focal length 50 cm
(b) a divergent lens of focal length 100 cm 35. Amount of light entering into the camera depends upon.
(c) a convergent lens of focal length 100 cm (a) focal length of objective lens
(d) a convergent lens of focal length 200 cm (b) product of focal length and diameter of the objective
26. Light takes t1 sec to travel a distance x in vacuum and the lens
same light takes t2 sec to travel 10 cm in a medium. Critical (c) distance of object from camera
angle for corresponding medium will be (d) aperture setting of the camera
Free eBooks on @neetquestionpaper2020

Ray Optics and Optical Instruments 863


36. In optical fibres, propagation of light is due to 44. A concave mirror of focal length f. in vacuum is placed in a
(a) diffraction (b) total internal reflection medium of refractive index 2. Its focal length in the medium
(c) reflection (d) refraction is
37. Rectilinear motion of light in a medium is caused due to f
(a) high frequency (a) (b) f (c) 2 f (d) 4 f
2
(b) short wavelength
(c) velocity of light 45. The maximum and minimum distance between a convex lens
(d) uniform refractive index of the medium and an object, for the magnification of a real image to be
38. Resolving power of a telescope increases with greater than one are
(a) increase in focal length of eye-piece (a) 2f and f (b) f and zero
(b) increase in focal length of objective (c) ¥ and 2f (d) 4f and 2f
(c) increase in aperture of eye piece 46. A plane convex lens of focal length 16 cm, is to be made of
(d) increase in apeture of objective glass of refractive index 1.5. The radius of curvature of the
39. We wish to see inside an atom. Assume the atom to have a curved surface should be
diameter of 100 pm. This means that one must be able to (a) 8 cm (b) 12 cm (c) 16 cm (d) 24 cm
resolve a width of say 10 pm. If an electron microscope is 47. A real image is formed by a convex lens. If we put a concave
used the energy required should be lens in contact with it, the combination again forms a real
(a) 1.5 keV (b) 50 keV image. The new image
(c) 150 keV (d) 1.5 MeV (a) is closer to the lens system.
40. Which of the following is false ? (b) is farther form the lens system.
(a) Convex lens always forms image with m < 1 (c) is at the original position.
(b) A simple mirror produces virtual, erect and same-sized (d) may be anywhere depending on the focal length of
image the concave lens.
(c) A concave mirror produces virtual, erect and magnified 48. A plano-convex lens of focal length 30 cm has its plane
image surface silvered. An object is placed 40 cm from the lens on
(d) A convex lens can produce real and same-sized image the convex side. The distance of the image from the lens is
41. A plane mirror reflects a beam of light to form a real image. (a) 18 cm (b) 24 cm (c) 30 cm (d) 40 cm
The incident beam is 49. Two convex lenses of focal lengths f1 and f2 are mounted
coaxially separated by a distance. If the power of the
(a) parallel (b) convergent
combination is zero, the distance between the lenses is
(c) divergent (d) any one of the above
42. An object is placed at a distance 2f from the pole of a convex (a) | f1 - f 2 | (b) f1 + f2
mirror of focal length f. The linear magnification is
f1f 2 f1f2
1 2 3 (c) (d)
(a) (b) (c) (d) 1 | f1 - f2 | f1 + f2
3 3 4
50. A red flower kept in green light will appear
43. A beam of light consisting of red, green and blue colours is (a) red (b) green (c) white (d) black
incident on a right-angled prism as shown. The refractive
51. Chromatic aberration in a lens is caused by
index of the material of the prism for the above red, green
(a) reflection (b) interference
and blue wavelengths are 1.39, 1.44 and 1.47 respectively.
The prism will (c) diffraction (d) dispersion
52. If D is the deviation of a normally falling light beam on a thin
B prism of angle A and d is the dispersive power of the same
prism then
90° (a) D is independent of A.
(b) D is independent of refractive Index.
45° 45° (c) d is independent of refractive index.
A C
(d) d is independent of A.
(a) separate part of the red colour from the green and 53. In primary rainbow what is the order of colours observed
blue colours. from earth ?
(b) separate part of the blue colour from the red and (a) Violet innermost, red outermost.
green colours. (b) Red innermost, violet outermost.
(c) separate all the three colours from one another. (c) Random.
(d) not separate even partially any colour from the other (d) White and dark alternatively
two colours.
Free eBooks on @neetquestionpaper2020

864 Physi cs
54. In secondary rainbow what is the order of colours observed be placed between them such that illuminance on one of its
from earth? faces is four times that on another face ?
(a) Violet innermost, red outermost. (a) 0.2 m (b) 0.4 m (c) 0.8 m (d) 1.6 m
(b) Red innermost, violet outer most. 66. A lamp is hanging along the axis of a circular table of radius
(c) Random. r. At what height should the lamp be placed above the table,
1
(d) White and dark alternatively. so that the illuminance at the edge of the table is of that
8
55. Why is refractive index in a transparent medium greater than at its centre?
one ?
(a) Because the speed of light in vaccum is always less (a) r/2 (b) r/ 2 (c) r/3 (d) r / 3
than speed in a transparent medium 67. A ray of light is incident on the surface of a glass plate at an
(b) Because the speed of light in vaccum is always greater angle of incidence equal to Brewster’s angle f. If m represents
than speed in a transparent medium the refractive index of glass with respect to air, then the
(c) Frequency of wave changes when it crosses medium angle between the reflected and the refracted rays is
(d) None of these
(a) 90° + f (b) sin –1 (m cos f)
56. For normal vision, what is distance of object from eye?
(a) 30 cm (b) 25 cm (c) infinite (d) 20 cm æ sin f ö
57. Two convex lenses of focal lengths 0.3 m and 0.05 m are used (c) 90º (d) 90 º - sin -1 çç ÷÷
to make a telescope. The distance kept between the two in è m ø
order to obtain an image at infinity is 68. A rectangular block of glass is placed on a mark made on the
(a) 0.35 m (b) 0.25 m (c) 0.175 m (d) 0.15 m surface of the table and it is viewed from the vertical position
58. The refractive indices of glass and water with respect to air of eye. If refractive index of glass be m and its thickness d,
are 3/2 and 4/3 respectively. Then the refractive index of then the mark will appear to be raised up by
glass with respect to water is
(a) 8/9 (b) 9/8 (c) 7/6 (d) 2 (m + 1)d (m - 1)d
(a) (b)
59. The wavelength of a monochromatic light in vacuum is l. It m m
travels from vacuum to a medium of absolute refractive index
µ. The ratio of wavelength of the incident and refracted (m + 1) (m - 1) m
(c) (d)
wave is md d
(a) µ2 : 1 (b) 1 : 1 (c) µ : 1 (d) 1 : µ 69. Light passes through a glass plate of thickness d and
60. An object is placed at a distance of 40 cm in front of a refractive index m. For small angle of incidence i, the lateral
concave mirror of focal length 20 cm. The image produced is displacement is
(a) real, inverted and smaller in size (a) id (b) id (m – 1)
(b) real, inverted and of same size
(c) real and erect i d (m - 1) id m
(c) (d)
(d) virtual and inverted m m –1
61. The frequency of a light wave in a material is 2 × 1014 Hz and 70. A glass slab of thickness 4 cm contains the same number of
wavelength is 5000 Å. The refractive index of material will waves as 5 cm of water when both are traversed by the same
be monochromatic light. If the refractive index of water is 4/3,
(a) 1.50 (b) 3.00 (c) 1.33 (d) 1.40 what is that of glass?
62. A ray incident at 15° on one refracting surface of a prism of (a) 5/3 (b) 5/4 (c) 16/15 (d) 1.5
angle 60° suffers a deviation of 55°. What is the angle of 71. An air bubble in glass slab (m = 1.5) from one side is 6 cm
emergence ?
and from other side is 4 cm. The thickness of glass slab is
(a) 95° (b) 45° (a) 10 cm (b) 6.67 cm
(c) 30° (d) None of these (c) 15 cm (d) None of these
63. When red glass is heated in dark room, it will seem 72. A vessel is half filled with a liquid of refractive index m. The
(a) green (b) purple other half of the vessel is filled with an immiscible liquid of
(c) black (d) yellow refrative index 1.5 m. The apparent depth of the vessel is
64. A man’s near point is 0.5 m and far point is 3 m. Power of 50% of the actual depth. Then m is
spectacle lenses required for (i) reading purposes, (ii) seeing (a) 1.4 (b) 1.5 (c) 1.6 (d) 1.67
distant objects, respectively, are 73. A man 160 cm high stands in front of a plane mirror. His eyes
(a) –2 D and + 3 D (b) +2 D and –3 D are at a height of 150 cm from the floor. Then the minimum
(c) +2 D and –0.33 D (d) –2 D and + 0.33 D length of the plane mirror for him to see his full length image
65. Two light sources with equal luminous intensity are lying at is
a distance of 1.2 m from each other. Where should a screen (a) 85 cm (b) 170 cm (c) 80 cm (d) 340 cm
Free eBooks on @neetquestionpaper2020

Ray Optics and Optical Instruments 865


74. It is desired to photograph the image of an object placed at 83. Two lens of focal length f1 and f2 are kept in contact coaxially.
a distance of 3 m from plane mirror. The camera, which is at The resultant power of combination will be
a distance of 4.5 m from mirror should be focussed for a f1 + f 2
f1f 2
distance of (a) (b)
f1 - f 2 f1f 2
(a) 3 m (b) 4.5 m (c) 6 m (d) 7.5 m
f1 f 2
75. Two thin lenses are in contact and the focal length of the (c ) f1 + f 2 (d) +
combination is 80 cm. If the focal length of one lens is 20 cm, f 2 f1
84. When white light enters a prism, its gets split into its
then the power of the other lens will be
constituent colours. This is due to
(a) 1.66 D (b) 4.00 D
(a) high density of prism material
(c) – 100 D (d) – 3.75 D
(b) because m is different for different wavelength
76. A thin convergent glass lens (mg = 1.5) has a power of
+ 5.0 D. When this lens is immersed in a liquid of refractive (c) diffraction of light
index m, it acts as a divergent lens of focal length 100 cm. (d) velocity changes for different frequency
The value of m must be 85. A pencil of light rays falls on a plane mirror and form a real
(a) 4/3 (b) 5/3 (c) 5/4 (d) 6/5 image, so the incident rays are
77. A ray of light passes through an equilateral prism (a) parallel (b) diverging
(m = 1.5). The angle of minimum deviation is (c) converging (d) statement is false
(a) 45º (b) 37º 12' (c) 20º (d) 30º 86. Astronauts look down on earth surface from a space ship
78. Two lenses in contact form an achromatic lens. Their focal parked at an altitude of 500 km. They can resolve objects of
lengths are in the ratio 2 : 3. Their dispersive powers must the earth of the size (It can be assumed that the pupils
be in the ratio of diameter is 5mm and wavelength of light is 500 nm)
(a) 1 : 3 (b) 2 : 3 (c) 3 : 2 (d) 3 : 1 (a) 0.5 m (b) 5 m (c) 50 m (d) 500 m
79. A combination is made of two lenses of focal length f and f' 87. Spherical aberration in a thin lens can be reduced by
in contact, the dispersive powers of the materials of the (a) using a monochromatic light
lenses are w and w'. The combination is achromatic, when (b) using a doublet combination
(a) w = w0, w' = 2w0 f ¢ = 2f (c) using a circular annular mark over the lens
(b) w = w0, w' = 2w0 f ¢ = f/2 (d) increasing the size of the lens
(c) w = w0, w' = 2w0 f ¢ = –f/2 88. A lens produces an image of an object on a screen. If a slab
(d) w = w0, w' = 2w0 f ¢ = –2f of refractive index n is placed in between lens and screen,
80. An achromatic convergent lens of focal length 20 cms is the screen has to be moved by distance d behind. The
made of two lenses (in contact) of materials having dispersive thickness of slab is
powers in the ratio of 1 : 2 and having focal lengths f1 and f2.
n -1 (n - 1) d nd
Which of the following is true ? (a) nd (b) (c) (d)
(a) f1 = 10 cms, f2 = –20 cms nd n n -1
(b) f1 = 20 cms, f2 = 10cms 89. An object is moved along the principal axis of a converging
(c) f1 = –10 cms, f2 = –20 cms lens from a position 5 focal lengths from the lens to a position
(d) f1 = 20 cms, f2 = –20 cms that is 2 focal lengths from the lens. Which statement about
81. A simple telescope, consisting of an objective of focal length the resulting image is most accurate?
60 cm and a single eye lens of focal length 5 cm is focussed on (a) The image increases in size and decreases in distance
a distant object in such a way that parallel rays emerge from from the lens
the eye lens. If the object subtends an angle of 2º at the (b) The image increases in size and increases in distance
objective, the angular width of the image is from the lens
(a) 10º (b) 24º (c) 50º (d) (1/6)º (c) The image decreases in size and decreases in distance
82. An astronomical telescope has an angular magnification of from the lens
magnitude 5 for distant objects. The separation between (d) The image decreases in size and increases in distance
the objective and the eye-piece is 36 cms and the final image from the lens
is formed at infinity. The focal length f0 of the objective and 90. An object is placed upright on the axis of a thin convex lens
fe of the eye piece are at a distance of four focal lengths (4f) from the center of the
(a) f0 = 45 cm and f2 = –9 cm lens. An inverted image appears at a distance of 4/3 f on the
(b) f0 = 50 cm and fe = 10 cm other side of the lens. What is the ratio of the height of he
(c) f0 = 7.2 cm and fe = 5 cm image of the height of the object?
(d) f0 = 30 cm and fe = 6 cm (a) 1/3 (b) 3/4 (c) 4/3 (d) 3/1
Free eBooks on @neetquestionpaper2020

866 Physi cs
91. A paper, with two marks having separation d, is held normal 99. The graph shows the variation of magnification m produced
to the line of sight of an observer at a distance of 50m. The by a convex lens with the image distance v. The focal length
diameter of the eye-lens of the observer is 2 mm. Which of
of the lens is
the following is the least value of d, so that the marks can be
seen as separate ? (The mean wavelength of visible light m
( a + c, b )
may be taken as 5000 Å)
(a) 1.25 m (b) 12.5 cm (c) 1.25 cm (d) 2.5 mm
92. An object is placed between two parallel mirrors. The number b
of images formed is
a c v
(a) 2 (b) 4 (c) 8 (d) infinite
(a , b)
93. A diver inside water sees the setting sun at
(a) 41° to the horizon (b) 49° to the horizon b c ab
(a) (b) (c) b (d)
(c) 0° to the horizon (d) 45° to the horizon c b c
94. A concave mirror forms the image of an object on a screen. 100. A real image of an object is formed by a convex lens at the
If the lower half of the mirror is covered with an opaque bottom of an empty beaker. The beaker is now filled with a
card, the effect would be to make the
liquid of refractive index 1.4 to a depth of 7 cm. In order to
(a) image less bright.
get the image again at the bottom, the beaker should be
(b) lower half of the image disappear.
moved
(c) upper half of the image disappear.
O
(d) image blurred.
95. The distance between an object and its real image formed
by a convex lens cannot be
(a) greater than 2 f (b) less than 2 f
(c) greater than 4 f (d) less than 4 f
96. The layered lens as shown is made of two types of
transparent materials-one indicated by horizontal lines and
I
the other by vertical lines. The number of images formed of
an object will be (a) downward by 2 cm (b) upward by 2 cm
(c) downward by 3 cm (d) upward by 3 cm
101. A convex lenses of focal length 40 cm is held coaxially 12 cm
above a concave mirror of focal length 18 cm. An object
held x cm above the lens gives rise to an image coincident
with it. The x is equal to
O
(a) 1 (b) 2 (c) 3 (d) 6
x cm
97. In the displacement method, a concave lens is placed in
between an object and a screen. If the magnification in the
two positions are m1 and m2 (m1 > m2), and the distance
between the two positions of the lens is x, the focal length
12 cm
of the lens is
x x
(a) (b)
m1 + m 2 m1 - m 2 (a) 12 cm (b) 15 cm (c) 18 cm (d) 30 cm
102. A convex lens is immersed in a liquid of refractive index
x x greater than that of glass. It will behave as a
(c) (d)
(m1 + m 2 ) 2 (m1 - m 2 ) 2 (a) convergent lens (b) divergent lens
98. A thin lens has focal length f, and its aperture has diameter (c) plane glass (d) homogeneous liquid
D. It forms an image of intensity I. If the central part of the 103. The plane face of a plano-convex lens is silvered. If µ be the
D refractive index and r the radius of curvature of the curved
aperture, of diameter , is blocked by an opaque paper,, surface, then the system behaves like a concave mirror of
2
the focal length of the lens and the intensity of image will radius
become
r r
f I I 3f I 3I (a) (b) (c) r µ (d) r (µ – 1)
(a) , (b) f, (c) , (d) f, µ µ -1
2 2 4 4 2 4
Free eBooks on @neetquestionpaper2020

Ray Optics and Optical Instruments 867


104. In an experiment to determine the focal length (f ) of a Directions for Qs. (110 to 113) : Each question contains
concave mirror by the u - v method, a student places the STATEMENT-1 and STATEMENT-2. Choose the correct answer
object pin A on the principal axis at a distance x from the (ONLY ONE option is correct ) from the following-
pole P. The student looks at the pin and its inverted image (a) Statement -1 is false, Statement-2 is true
from a distance keeping his/her eye in line with PA. When
the student shifts his/her eye towards left, the image appears (b) Statement -1 is true, Statement-2 is true; Statement -2 is a
to the right of the object pin. Then, correct explanation for Statement-1
(a) x < f (b) f < x < 2f (c) Statement -1 is true, Statement-2 is true; Statement -2 is not
(c) x = 2f (d) x > 2f a correct explanation for Statement-1
105. A ray of light traveling in water is incident on its surface (d) Statement -1 is true, Statement-2 is false
open to air. The angle of incidence is q, which is less than
the critical angle. Then there will be 110. Statement 1: Two convex lenses joined together cannot
(a) only a reflected ray and no refracted ray produce an achromatic combination.
(b) only a refracted ray and no reflected ray Statement 2 : The condition for achromatism is
(c) a reflected ray and a refracted ray and the angle between w1 w2
them would be less than 180°–2q + = 0 where symbols have their usual meaning.
f1 f 2
(d) a reflected ray and a refracted ray and the angle between 111. Statement 1: Critical angle is minimum for violet colour.
them would be greater than 180°–2q
æ1ö
106. Air has refractive index 1.0003. The thickness of air column, Statement 2 : Because critical angle qc = sin -1 ç ÷ and
which will have one more wavelength of yellow light (6000 èmø
1
Å) than in the same thickness of vacuum is mµ .
l
(a) 2 mm (b) 2 cm (c) 2 m (d) 2 km. 112. Statement 1: Optical fibres are used to transmit light without
107. The position of final image formed by the given lens any appreciable loss in its intensity over distance of several
combination from the third lens will be at a distance of kilometers.
(f1 = + 10 cm, f2 = – 10 cm and f3 = + 30 cm). Statement 2 : Optical fibres are very thick and all the light is
passed through it without any loss.
113. Statement 1 : If P1 and P2 be the powers of two thin lenses
located coaxially in a medium of refractive index µ at a
distance d, then the power P of the combination is
P = P1 + P2 – P1P2d/µ
Statement 2 : Because for above given system equivalent
30 cm 5 cm 10 cm f1f 2 1
focal length is given by F = and P = .
f1 + f2 - d / m F
(a) 15 cm (b) infinity (c) 45 cm (d) 30 cm Directions for Qs. (114 to 118) : Read the following passage(s)
108. A ray of light is travelling from glass to air. (Refractive carefully and answer the questions that follows:
index of glass = 1.5). The angle of incidence is 50°. PASSAGE 1
The deviation of the ray is The ciliary muscles of eye control the curvature of the lens in the
(a) 0° (b) 80° eye and hence can alter the effective focal length of the system.
When the muscles are fully relaxed, the focal length is maximum.
é sin 50° ù -1 é sin 50° ù
(c) 50° - sin -1 ê ú (d) sin ê ú - 50°
When the muscles are strained the curvature of lens increases
ë 1.5 û ë 1.5 û (that means radius of curvature decreases) and focal length
109. A vessel of height 2d is half-filled with a liquid of refractive decreases. For a clear vision the image must be on retina. The
image distance is therefore fixed for clear vision and it equals the
index 2 and the other half with a liquid of refractive index
distance of retina from eye-lens. It is about 2.5 cm for a grown-up
n. (The given liquids are immiscible). Then the apparent person.
depth of the inner surface of the bottom of the vessel
A person can theoretically have clear vision of objects situated at
(neglecting the thickness of the bottom of the vessel) will
any large distance from the eye. The smallest distance at which a
be
person can clearly see is related to minimum possible focal length.

(a)
n
(b)
(
d n+ 2 ) The ciliary muscles are most strained in this position. For an
average grown-up person minimum distance of object should be
(
d n+ 2 ) n 2 around 25 cm.
A person suffering for eye defects uses spectacles (Eye glass).
n 2 nd The function of lens of spectacles is to form the image of the
(c) (d)
(
d n+ 2 ) d + 2n objects within the range in which person can see clearly. The
Free eBooks on @neetquestionpaper2020

868 Physi cs
image of the spectacle-lens becomes object for eye-lens and whose
image is formed on retina.
varies continuously. Now, å nl is the total optical path, so that
Fermat’s principle states then that path of a ray is such that the
The number of spectacle-lens used for the remedy of eye defect is optical path is a stationary value. This principle is obviously in
decided by the power of the lens required and the number of agreement with the fact that the ray are straight lines in a
spectacle-lens is equal to the numerical value of the power of lens homogenous isotropic medium. It is found that it also agrees with
with sign. For example power of lens required is +3D (converging the classical laws of reflection and refraction.
lens of focal length 100/3 cm) then number of lens will be +3.
116. If refractive index of a slab varies as m = 1 + x2 where x is
For all the calculations required you can use the lens formula and
measured from one end, then optical path length of a slab of
lens maker’s formula. Assume that the eye lens is equiconvex
thickness 1 m is
lens. Neglect the distance between eye lens and the spectacle
(a) (4/3) m (b) (3/4) m
lens.
(c) 1 m (d) None of these
114. Maximum focal length of eye lens of normal person is 117. The optical path length followed by ray from point
(a) 25 cm (b) 2.5 cm. A to B given that laws of reflection are obeyed as shown in
(c) 25/9 cm. (d) 25/11 cm. figure is
115. A nearsighted man can clearly see object only upto a distance
of 100 cm and not beyond this. The number of the spectacles
lens necessary for the remedy of this defect will be
(a) +1 (b) – 1
(c) + 3 (d) – 3

PASSAGE 2 (a) maximum (b) minimum


(c) constant (d) None of these
The laws governing the behavior of the rays namely rectilinear
118. The optical path length followed by ray from point A to B
propagation, laws of reflection and refraction can be summarised
given that laws of reflection are obeyed as shown in figure
in one fundamental law known as Fermat’s principle. According
to this principle a ray of light travels from one point to another is.
such that the time taken is at a stationary value (maximum or
minimum). If c is the velocity of light in a vacuum, the velocity in
c
a medium of refractive index n is , hence time taken to travel a
n
nl
distance l is . If the light passes through a number of media,
c (a) maximum (b) minimum
æ 1ö 1
the total time taken is çè ÷ø å nl or ò ndl if refractive index (c) constant (d) None of these
c c
Free eBooks on @neetquestionpaper2020

Ray Optics and Optical Instruments 869

than ic, then total internal reflection occurs. It takes


Exercise 24.1
place when ray of light travels from optically denser
medium (m1 > m2) to optically rarer medium.
3. (a) They are not constant with time, but they vary with
time period. Actually the sketch between the amplitude 1 1 æ 1 ö
r r 5. (d) sin C = = \ C = sin -1 çç ÷÷ = 45 º
of E / B with time t is sine curve. m 2 è 2ø
5. (c) Because they form nearly point image of point source.
sin C 1 sin 45º 1
Now = or =
sin r m sin r 2
Exercise 24.2
sin r = 1 or r = 90º
sin i v1 m 2 6. (a) The angular dispersion q i.e., the angle between the
1. (b,c) According to Snell’s law sin r = v = m extreme rays of light,
2 1
q = (dV – dR) where dV = (mV–1)A, dR = (mV–1)A & A is
(a) From fig. 1 we see that angle of prism.
So if refractive index increases, then d increases & hence
q increases.
i medium 1 (m1)
denser mV - m R dm
7. (b) Dispersive power of a prism w = = ,
rare 90 – r my -1 m -1
r
medium 2 (m2) mV + mR
where m = m y =
2
r > i Þ v2 > v1 from Snell’s Law 8. (d) The colours are seen due to interference of light. The
So v2 = nl2 > v1 = nl1 Þ l2 > l1 colours seen in reflected light are complementary with
(Frequency of wave does not change on refraction) the colours seen in transmitted light.
(b) From fig.2, we see that r < i Þ v1> v2 9. (b) Apparent position of fish as seen by bird
= [h 1 + (h2/m)]
10. (b) We know that frequency of electromagnetic radiation
i medium 1 (m1) remains the same when it changes the medium. Further
rare
denser wavelength of light in vacuum l v
m= =
r wavelength of light in medium l m
medium 2 (m2)
lv l
lm = =
Þ l1 > l2 m m
So both options b & c are correct.
2. (b) Frequency does not change on refraction. velocity of light in vacuum
Similarly, m =
3. (d) A laser source gives coherent, monochromatic & velocity of light in medium
directional light, while other source does not give.
v
lm =
sin i m 2 m
4. (b) According to Snell’s Law, =
sin r m1
11. (a) In the position of minimum deviation, i1 = i2.
where r = 90º for particular incidence angle called critical
w1 w 2 f w
angle. When the incidence angle is equal to or greater 12. (b) + = 0 or 1 = - 1
f1 f2 f2 w2
Free eBooks on @neetquestionpaper2020

870 Physi cs
14. (a) The basic principle of communication in fibre optics is Exercise 1 : NCERT Based Questions
based on the phenomenon of total internal reflection.
sin i
16. (d) Since refractive index for different wave length of light 5. = n 21
sin r
is different. Hence the different colours of light forms
images at different position. This phenomenon is called 6. Angle of incidence in the denser medium corresponding to
chromatic aberration. which the angle of refraction is 90° in the rarer medium.
17. (d) It occurs in both reflection & refraction & the angle of 7. Due to the combined effect of dispersion, refraction and
bending rays is constant. reflection of sunlight by water droplets.
18. (d) The cause of chromatic aberration is that lens focusses 17. d = 15 cm
different colours at different points. 18. (a) 19. (d) 20. (a) 21. (d)
19. (c) Since lR > lV mR< mV 22. (c) 23. (d) 24. (d) 25. (a)

æ 1ö æ 1 ö Exercise 2 : PAST Competition MCQs


çèQ m µ ÷ø Þ f V < fR çèQ µ (m - 1)÷ø
l f n
1 1
20. (b,c) If faces of prism on which light is incident & from which 1. (d) We know that f = å fi
i =1
it emerge are parallel, then angle of prism will be zero &
so deviation will also be zero i.e., the prism will act as a
1 1 1
transparent prism. = +
f f1 f 2
21. (d) We can combine two prisms in such a way
(i) deviation is zero but dispersion not f1 = 80 cm, f2 = –50 cm
(ii) dispersion is zero but deviation is not. 1 1 1 1
= - Þ P = = 1.25 – 2 = – 0.75D
But in any situation both deviation & dispersion f 80 50 f
can not be zero simultaneously. 100 100
22. (d) A narrow slit, cut unparallel rays & makes the beam
parallel. 1 æ 1 1 ö
2. (b) = (m - 1)çç - ÷÷
f è R1 R 2 ø
1 1
23. (b, d) Since m µ & µ ( m -1) Þ f µ l
l f B C
According to Cauchy relation m = A + 2
+ ......
So if wavelength is increased, then, focal lengths of l l4
both converging & diverging lens increase.
Hence f µ l .
Hence red light having maximum wavelength has
Exercise 24.3 maximum focal length.

1. (b) The image formed by objective lens of compound fV < fR and also FV < FR
microscope is real and enlarged, while final image formed 3. (d) Total thickness = t; Refrative index = m
by compound microscope is inverted, virtual, enlarged c
and at a distance D to infinite or from an eye, on same Speed of light in Glass plate =
m
side of eye piece.
2. (b) The aperture of objective lens of Astronomical é Speed of light in vacuum ù
telescope is large to get better resolution. Since êQ v = ú
ë R.I. of Medium û
D
resolution of telescope power is R = , where D t mt
1.22l Time taken = c = c
is the diameter of the objective lens of Telescope. m
3. (b) One should increase the power of lens i.e., decrease
(where t = thickness of glass plate)
the focal length of a lens.
4. (c) Thickness of glass plate (t) = 6 cm;
4. (a) In simple microscope the final image is erect. Distance of the object (u) = 8 cm. and
distance of the image (v) = 12 cm.
Exercise 24.4 Let x = Apparent position of the silvered surface in cm.
Since the image is formed due to relfection at the silvered
1 (d) 2. (c) 3. (c) 4. (a) 5. (a) face and by the property of mirror image distance of object
Free eBooks on @neetquestionpaper2020

Ray Optics and Optical Instruments 871


from the mirror = Distance of image from the mirror 1
or x + 8 = 12 + 6 – x or x = 5 cm. Also sin C =
n
Therefore refractive index of glass
Re al depth 6 sin 2 i 1 sin 2 i 1
= Apparent depth = 5 = 1.2 . \1 – 2
> 2
or 1 > 2
+
n n n n2

5. (a) When electromagnetic wave enters in other medium,


1
or (sin 2 i + 1) < 1 or n 2 > sin 2 i + 1
frequency reamains unchanged while wavelength and n 2
1 Maximum value of sin i = 1
velocity become times.
m
\ n2 > 2 Þ n > 2
so, e.m. entering from air to glass slab (m), frequency
8. (b) R1 = 60 cm, R2 = µ , m = 1.6
l
remains n, wavelength l' =
m 1 æ 1 1 ö
v = (m - 1)çç - ÷
÷
velocity of light in medium v' = f è R1 R 2 ø
m
f = 20 cm
6. (c) 1 æ 1 ö
= (1.6 - 1)ç ÷ Þ f = 100 cm.
l f è 60 ø
O
9. (c) Angle of minimum deviation
30 cm 10 cm
æ A + dm ö æ 60° + d m ö
60 cm sin ç sin ç
è 2 ÷ø è 2 ø
÷
m= Þ 3=
1 1 1 1 1 1 æ Aö æ 60° ö
- = ; - = sin ç ÷ sin ç
v u f v - 30 20
Þ v = 60 cm è 2ø è 2 ÷ø
Coincidence is possible when the image is formed at
the centre of curvature of the mirror. Only then the rays æ d ö 3 d
refracting through the lens will fall normally on the Þ sin ç 30° + m ÷ = Þ 30° + m = 60°
è 2ø 2 2
convex mirror and retrace their path to form the image
at O. So the distance between lens and mirror = 60 – 10 Þ dm = 60°.
= 50 cm. 10. (c) The silvered plano convex lens behaves as a concave
7. (a) Let a ray of light enter at A and refracted beam is AB. mirror; whose focal length is given by
This is incident at an angle q. For no refraction at the
lateral face q > C 1 2 1
= +
sin q > sin C But q + r = (90°) F f1 f m

B If plane surface is silvered

q R2 ¥
fm = = =¥
A 2 2
r
i
1 æ 1 1 ö
\ = (m – 1)çç – ÷
÷
f1 è R1 R 2 ø
\ sin (90° – r) > sin C or cos r > sin C ..(1) æ 1 1 ö m –1
= (m – 1)ç – ÷ =
sin i sin i èR ¥ø R
From Snell’s law n = Þ sin r =
sin r n
1 2(m – 1) 1 2(m – 1)
\ = + =
æ sin 2 i ö F R ¥ R
\ cos r = 1 - sin 2 r = ç1 - 2 ÷
è n ø R
F=
2(m – 1)
sin 2 i
\ equation (1) gives, 1 – > sin C Here R = 20 cm, m = 1.5
n2
20
sin 2 i \ F= = 20cm
Þ1– > sin 2 C 2(1.5 – 1)
n2
Free eBooks on @neetquestionpaper2020

872 Physi cs
11. (a) To see his full image in a plane mirror a person requires
1 æ 1 1 ö
a mirror of at least half of his height. 14. (b) = (m - 1)çç - ÷
÷
f è R1 R 2 ø
H in this case,
M R1 and R2 are unchanged
E So, f will remain unchanged for both pieces of the
H
H 2 lens
\ f= f'
M'
1 1 1
L = +
f f1 f 2
This is combination of two lenses
Re al depth (R1 )
12. (b) 1.5 = of equal focal length
Apparent depth (5 cm)
é1 1 1 2 ù
\ R1 = 1.5 × 5 = 7.5 cm \ ê = ¢ + ¢ = ¢ú Þ f '' = 2f
ëf f f f û

R1 5 cm
15. (c) 1
f
= (l mg - 1)æçç R1 - R1ö
÷÷ where l m g = 1 is given, we get
è 2ø
air bubble
R2 1 æ 1 1 ö
Þ = (1 - 1)çç - ÷÷ = 0 or Þ f = ¥
2 cm f R
è 1 R 2ø

16. (b) The angle must be equal to the critical angle,


R
For opposite face, 1.5 = 2 Þ R2 = 3.0 cm æ1ö æ 1 ö
2 C = sin -1 çç ÷÷ = sin -1 çç ÷÷ = 45º
\ Thickness of the slab = R1 + R2 = 7.5 + 3 m
è ø è 2ø
= 10.5 cm
x 1.22l 1.22 ´ 5 ´10 3 ´ 10 -10 ´ 10 3
sin 45 17. (d) Here = or x =
1 10 ´ 10 - 2
(a) For point A, a m g = 1000 D
sin r Þ sin r =
13.
2 a mg or x = 1.22 × 5 × 10–3 m = 6.1 m
for point B, sin (90 – r) = gma x is of the order of 5 mm.
45º Air
1 1 1 f 2 + f1 1 f1 + f 2
(90 – r) is critical angle. 18. (a) = + = ; P= =
90 – r r A F f1 f 2 f1f 2 F f1f 2
1 90 – r 19. (c) For total internal reflection,
\ cos r = g m a =
m
a g B
1
m³ >1.414
Glass sin C ³ 2
1
Þ a mg = Þ µ = 1.50
cos r
20. (b) Difference between apparent and real depth of a pond
1 1 is due to the refraction of light, not due to the total
= =
1 - sin 2 r 1 internal reflection. Other three phenomena are due to
1-
2 amg2 the total internal reflection.
21. (c) R = 20 cm
h0 = 2
2 1 2 a m 2g
Þ a mg = = u = –30 cm
1 2 a m 2g - 1
1- 1 æ 1 1 ö
2 a m g2 We have, = (m - 1) ç -
f è R1 R 2 ÷ø

3 æ 3 ö é 1 æ 1 öù
= çè - 1÷ø ê - çè - ÷ø ú
2
Þ 2 a mg - 1 = 2 Þ a mg =
2 2 ë 20 20 û
Free eBooks on @neetquestionpaper2020

Ray Optics and Optical Instruments 873

1 æ3 ö 2 A A A
Þ = ç - 1÷ ´ 2 sin cos = n sin
f è 2 ø 20 2 2 2
\ f = 20 cm A
2 cos =n
1 1 1 1 1 1 2
= - Þ = +
f v u 20 v 30 when A = 90° = imin
1 1 1 10 then nmin = 2
= - =
v 20 30 600 i = A = 0 nmax = 2
v = 60 cm
hi v 24. (d) f = 10 cm
m= = Þ hi = v ´ h 0 = 60 ´ 2 = – 4 cm
h0 u u 30
So, image is inverted. B A
22. (b) Deviation = zero
–10 cm –20 cm
So, d = d1 + d2 = 0
Þ (m1 – 1)A1 + (m2 – 1) A2 = 0
Þ A2 (1.75 – 1) = – (1.5 – 1) 15°
The focal length of the mirror
0.5
Þ A2 = - ´ 15° 1 1 1
0.75 – = +
f v u
or A2 = – 10°.
For A end of the rod the image distance
Negative sign shows that the second prism is inverted When u1 = – 20 cm
with respect to the first.
-1 1 1
Þ = -
A 10 v1 20
23. (b) A Prism angle
Angle of
Smin
minimum 1 -1 1 -2 + 1
= +
deviation v1 10 20 = 20
Incident C e Angle of v1 = – 20 cm
r1 r2
angle emergence
For when u2 = – 30 cm
1 1 1
= -
B C f v2 30

1 -1 1 - 30 + 10 -20
The angle of minimum deviation is given as = + =
v2 10 30 = 300 300
d min = i + e–A v2 = – 15 cm
for minimum deviation L = v2 – v1 = – 15 – (– 20)
L = 5 cm
d min = A then
2A = i + e 1 æ mg öæ 1 1 ö
25. (a) =ç - 1÷ ç - ÷
f è m m ø è R1 R2 ø
in case of d min i = e
1 æ 1 1ö
A If mg = mm, then = (1 - 1) ç - ÷
2A = 2i r1 = r2 = f è R1 R2 ø
2
i = A = 90° 1
Þ =0
f
from smell’s law
1 sin i = n sin r1 1
f = =¥
0
A
sin A = n sin This implies that the liquid must have refractive index
2 equal to glass.
Free eBooks on @neetquestionpaper2020

874 Physi cs
26. (a) For normally emerge e = 0 1 1 1
1 1
Therefore r2 = 0 and r1 = A 30. (a) = – = =
f v u 1 u
Snell’s Law for Incident ray’s 40
1
1sin i = m sin r1 = msin A Þ u= m = 2.5 cm
40
For small angle 31. (b) Large aperture increases the amount of light gathered
i = mA by the telescope increasing the resolution.
32. (a) No. of images, n = (360/ q ) – 1. As q = 60°
so n = 5.
33. (d) Resolving power a(1/l).
27. (c) C
(R.P)1 l 2 5
Hence, = = .
f2 (R.P) 2 l1 4
34. (a) 35. (d) 36. (c)
f1 2f2
d 360
37. (b) According to Image formula, n = -1
q
d = f1 + 2f2
360
Þ 3= - 1 Þ 360 = 4 Þ q = 360 = 90
f q q q
28. (c) M.P. = 9 = 0
fe 38. (b) The incident angle is 45° incident angle > critical angle,
Þ f0 = 9fe ...(1) i > ic
f0 + fe = 20 ...(2) \ sin i > sin i c or sin 45 > sin i c
On solving 1 1 1
1 \ sin 45° > or >
f0 = 18 cm = focal length of the objective sin i c =
n n 2 n
fe = 2 cm = focal length of the eyepiece
Þn> 2

29. (b) 39. (c)

C O
X
µx

For the image to be real and of same size as object, final


image should be formed at the position of object itself.
Let x be the distance of object from plane surface.
1 1 1 Apparent distance from surface = mx
= + This should be centre of curve
f f1 f2
\ mx = 30 Þ 1.5x = 30 Þ x = 20
æ1 1 ö æ 1 1 ö
= (m1 – 1) ç - ÷ + (m2 – 1) ç ¥ - R ÷ 1 3
è ¥ -R ø è ø 40. (a) sin qc = = or tan q c = 3 = 3 = R
m 4 16 - 9 7 12
( m1 - 1) ( m 2 - 1) 1 m1 - m 2
= – Þ = R
R R f R

R qc qc
12 cm
Þf=
m1 - m 2

R
Hence, focal length of the combination is .
m1 - m 2
36
Þ R= cm
7
Free eBooks on @neetquestionpaper2020

Ray Optics and Optical Instruments 875

1 æ 1. 5 ö æ 1 1 ö 1 1 1
41. (b) =ç - 1÷ çç - ÷÷ .............. (i) - =
fa è 1 ø è R1 R 2 ø v u f
where f is a positive constant for a given convex lens.
1 æ mg öæ 1 1 ö 45. (a) To find the refractive index of glass using a travelling
= çç - 1÷÷ çç - ÷
÷
fm è mm microscope, a vernier scale is provided on the
ø è R1 R 2 ø
microscope.
1 æ 1.5 ö æ 1 1 ö 46. (a,c,d)
=ç - 1÷ çç - ÷ ............. (ii) It is given that heat Q flows only from left to right
fm 1 . 6 R R ÷
è øè 1 2ø
through the blocks. Therefore heat flow through A and
E slabs are the same.
æ ö \ (a) is correct option
f m ç 1.5 - 1 ÷
Dividing (i) by (ii), =ç ÷ =–8
f a ç 1.5 ÷
ç -1 ÷ A B
è 1.6 ø 3K QB RB E
2K 6K
C RC
1 1
Pa = - 5 = Þ fa = -
fa 5 4K QC
QA QE

1 8
Þ f m = - 8 ´ f a = -8 ´ - = RA D RE
5 5 5K QD RD
QA = QB + QC + QD = QE
m 1.6
Pm = = ´ 5 = 1D
fm 8
Since heat flow through slabs A and E is same, (b) is
42. (a) For a thin prism, D = (m – 1) A not correct.
Since lb < lr Þ mr < mb Þ D1 < D2
l
43. (c) Power of combination is given by We know that resistance to heat flow is R =
KA
P = P1 + P2 = (– 15 + 5) D = – 10 D.
Let the width of slabs be Z. Then
1 1 1
Now, P = Þ f = = metre L 1 4L 4
f P -10 RA = = ,R = =
2 K (4L) Z 8KZ B 3K ( LZ ) 3KZ
æ1 ö
\ f = - ç ´100 ÷ cm = -10 cm. 4L 1 4L 4
è 10 ø RC = = , RD = =
4 K (2 LZ ) 2 KZ 5K ( LZ ) 5 KZ
44. (c) This graph suggest that when
u = – f, v = + µ L 1
RE = =
v (cm) 6 K (4 LZ ) 24 KZ
Now, DT = QR
As R E is least, DTE is also smallest ie since the
resistance to heat flow is least for slab E, the
temperature difference across is smallest.
f
\ Option (c) is the correct answer.
–f u (cm) Also
DTC DTC
When the object is moved further away from the lens, QC = = = 2 KZ ( DTC )
RC 1/ 2 KZ
v decreases but remains positive. When u is at – µ ,
v = f. DTC 3KZ ( DTC )
DTB
This is how image formation takes place for different QB = = = [Q DTB = DTC]
RB 4 / 3KZ 4
positions of the object in case of a convex lens.
ALTERNATIVELY : This graph obeys the lens DTD DTC 5KZ ( DTc )
equation QD = = =
RD 4 / 5KZ 4
Free eBooks on @neetquestionpaper2020

876 Physi cs

[Q DTD = DTC ] When q is the angle of incidence at P


l
3 KZ (DTC ) 5KZ ( DTC )
QB + QD = + B A
4 4 q
q
8 KZ (DTC ) R P
= = 2 KZ (DTC ) = QC
4
\ (d) is the correct option.
C
47. (d) The focal length of the lens
a
1 1 1 1 1 20 + 1 21 w gh 1. 5
= - = + = = Now, gm =
w
= = 1.125
f v u 12 240 240 240 gm
4/3

240 1 8
f = cm Putting in (i), sin q = =
21 1.125 9
æ 1ö 8
Shift = t ç 1 - ÷ \ sin q should be greater than .
è mø 9

æ 1 ö 1 1 sin 90° 1
1ç1 - ÷ = 1´ 51. (a) 2m = = [For critical angle]
è 3/ 2 ø 3 sin C sin C

1 35 æ 1 ö
Now v' = 12 - = cm C = sin -1 ç ÷
3 3 \ ç 1m÷ … (i)
è2 ø
Now the object distance u.
Applying Snell’s law at P, we get
1 3 21 1 é 3 21 ù
= - = -
u 35 240 5 êë 7 48 úû 1 sin r ¢ sin(90 - r)
[Q i = r; r '+ r = 90°]
2m = =
sin i sin r
1 1 é 48 - 49 ù
= 1 cos r ... (ii) From (i) and (ii)
u 5 êë 7 ´16 úû m=
2 sin r
u = –7 ×16 × 5 = – 560 cm = – 5.6 m
C = sin–1 (tan r)
Velocity of light in vacuum R = 3cm 52. (b) For minimum deviation, incident angle is equal to
48. (c) Q n =
Velocity of light in medium emerging angle.
3 3mm \ QR is horizontal.
\ n= 53. (d)
2
54. (c) We know that in case of a convex lens when object is
3 + (R – 3mm)2 = R2
2
placed at C', the image is obtained at C. This situation
Þ 32 + R2 – 2R(3mm) + (3mm)2 = R2 is represented in the graph by the point corresponding
Þ R » 15 cm to u = –10 cm, v = 10 cm. Therefore R = 10
R
1 æ 3 öæ 1 ö Þ = 5 cm = f
= ç –1 ÷ç ÷ Þ f = 30 cm 2
f è 2 øè 15 ø
Lens formula is
49. (c) For the prism as the angle of incidence (i) increases,
the angle of deviation (d) first decreases goes to 1 1 1 df dv du
= - Þ 2 = 2+ 2
minimum value and then increases. f v u f v u
50. (a) The phenomenon of total internal reflection takes place (for maximum error in f)
during reflection at P.
1 df 0.1 0.1
sin q = Þ = + Þ df = 25 ´ 0.1 ´ 2 ´ 0.01 = 0.05
2
w
m
g
… (i) 25 (10) (10) 2
Free eBooks on @neetquestionpaper2020

Ray Optics and Optical Instruments 877


v will bend towards the normal. Therefore p, r are the
correct options
(B) When m1> m2, the ray of light while entering the lens
will bend away from the normal. Therefore q,s,t are the
correct options.
10
(C) When m2 = m3, the ray of light while coming out from
the lens does not deviate from its path. Therefore p,r,t
u are the correct option.
–10
(D) m2> m3, the ray of light coming out of the lens deviates
55. (b) From the figure in DABC,
away from the normal. Therefore q,s are the correct
AB options.
tan b = Þ AB = AC tan b
AC 58. (b) For the convex spherical refracting surface of oil we
apply
2r = f tan b Þ pr 2 µ f 2
Parallel -m1 m 2 m 2 - m1
Rays from + =
Focal u v R
plane object placed
at infinity
7
-1
-1 7/4 4
f \ + =
A ( -24) v 6 0
2r b C
24 cm
\ v = 21 cm
B For water-oil interface
Image formed oil
on focal plane Biconvex lens -7 4
56. (b) Focal length of the biconvex lens is 15 cm. A small 4 + 3 =0 V¢
object is placed at a distance of 30cm from the lens i.e. +21 V ¢ water
at a distance of 2f. Therefore the image should form at I
\ V¢ = 16 cm.
30cm from the lens at I1. This is the image distance from water-oil interface.
Therefore the distance of the image from the bottom of
the tank is 2 cm.
59. (b) The focal length (f1) of the lens with n = 1.5 is given by
O I3 I2 I1 1 é 1 1 ù
= (n1 - 1) ê - ú
f1 R
ë 1 R 2û

10cm
é 1 1ù 1
But since the ray strike the plane mirror before reaching = (1.5 - 1) ê - ú =
ë 14 ¥ û 28
I1, the image I1 acts as the virtual object for reflection
on plane mirror kept at a distance of 20 cm from it. The focal length (f2) of the lens with n = 1.2 is given by
It should produce an image I2 but as the ray encounters
1 é 1 1 ù
the lens, it gets refracted and the final image is formed = (n2 - 1) ê - ú
f2 ë R1 R2 û
at I3. For the last refraction from the biconvex lens,
u = 10 cm.
é1 1 ù 1
= (1.2 - 1) ê - =
1 1 1
Applying lens formula - =
ë ¥ -14 úû 70
v u f
The focal length F of the combination is
1 1 1 1 1 1 25 1 1 1 1
Þ - = Þ = + = = + =
v 10 15 v 15 10 150 F f1 f 2 20
Þ v = 6 cm. Applying lens formula for the combination of lens
Therefore a real image is formed at a distance of 16 cm
from the plane mirror. 1 1 1 1 1 1
- = Þ - =
57. A-p, r; B-q,s,t; C-p,r,t, D-q,s V U F V -40 20
(A) When m1 < m2, the ray of light while entering the lens Þ V = 40 cm
Free eBooks on @neetquestionpaper2020

878 Physi cs
60. (c) For a plano convex lens e ® i. At the prism surface, total internal reflection
h
1 (µ - 1) 1 1 has taken place. For this sin 45° > 2
= = - ...(i) h1
f R v u
\ m1 > 2 m2.
la l 3 (d) is the correct option.
Here µ = = a = = 1.5
lm 2
l 2
3 a Exercise 3 : Conceptual & Applied MCQs
where la = wavelength of light in air 1. (d) The polarization phenomenon, verifies the transverse
lm = wavelength of light in water nature of light. Since sound has longitudinal nature,
so it does not show polarization effect.
2. (d) Two source are called coherent, if they have constant
phase difference i.e., phase difference between two
wave is constant with respect to time.
3. (b)
4. (d) Because, the focal length of eye lens can not decrease
v = 8m
beyond a certain limit.
v 1
Also m = =- \ u = -24 cm. 5. (d) It is the total luminous flux.
u 3
6. (c)
1.5 - 1 1 æ 1 ö 1 1 1 7. (c) The intensity of cylindrical source at small distance r is
From (i) = -ç ÷= + =
R 8 è -24 ø 8 24 6 inversely proportional to r.
\ R = 3m option (c) is correct 1 1
Iµ (since A µ & I µ A2 )
r r
æ iˆ 3 ˆö æ ˆi 3 ˆö
ç 2 + 2 j÷ .ç 2 - 2 j÷ 8. (a) f = 4 p I is the correct relation.
è ø è ø
61. (a) cos(180° - 2a ) =
2
æ 3ö
2 2 9. (d) f = 4 p I = 4 p (100) = 400 p lumen.
æ 1ö æ 1ö æ 3ö
çè ÷ø + ç ÷ çè ÷ø + ç - ÷ 10. (c) At minimum distance, incidence is normal. Therefore,
2 è 2 ø 2 è 2 ø
I 250
1 E= 2
= = 6.94 lux
\ cos(180° - 2a ) = - r 62
2
11. (b) r = 2 m = 200 cm
E = 5 × 10–4 phot, q = 60º
From,

I cos q E r 2 5 ´ 10 -4 (200)2
E= , I= = = 40 C.P
r2 cos q cos 60

360 º
\ 180° – 2a = 120° 12. (a) Number of images ( n 1 ) = -1
q
\ a = 30°
Option (a) is correct. where q = angle between mirrors
62. (a) e ® f. For the ray to bend towards the normal at the Here, q = 60º
prism surface m2 > m1. The ray then moves away 360 º
from the normal when it emerges out of the So, number of images n 1 = -1 = 5
60 º
rectangular block. Therefore m2 > m3.
e ® g. As there is no deviation of the ray as it emerges 1
out of the prism, m2 = m1. 13. (d) Resolving power of an optical instrument µ
l
e ® h. As the ray emerges out of prism, it moves away
from the normal. Therefore m2 < m1 . As the ray Resolving power at λ1 l
= 2
moves away from the normal as it emerges out Resolving power at l 2 l1
of the rectangular block, therefore m2 > m3.
Free eBooks on @neetquestionpaper2020

Ray Optics and Optical Instruments 879

æ 1 ö 1 1 1 f -f f1f 2
çç Limit of resolution µ ÷÷ 24. (c) = + = 2 1; f=
è resolving power ø f f1 - f 2 f1f 2 f 2 - f1
25. (c) P = P1 + P2 = + 2 – 1 = + 1 dioptre, lens behaves as
5000 5 convergent
\ Ratio of resolving power = = =5:4
4000 4
1 1
14. (b) F= = = 1 m = 100 cm
P 1
c v c c
15. (a) la = or l m = = (Q m m = ) 26. (c) c=
x
, v=
10
n n m m n v t1 t2

c c 1 v 10 t 1 æ 10 t 1 ö
\ l1 = and l 2 = sin i c = = = ´ ; i c = sin -1 çç ÷
m1n m 2n m c t2 x ÷
è t 2x ø
or l1 m1 = l 2 m 2 or l 2 = l1 (m1 / m 2 ) 27. (b) If R1 = R, R2 = –2 R
16. (b) Apparent depth = d/m1 + d/m2 1 æ 1 1 ö
= (m - 1) çç - ÷
÷
velocity of light in vacuum f è R1 R 2 ø
17. (c) m=
velocity of light in glass plate
1 æ1 1 ö 0 .5 ´ 3
= (1.5 - 1) çç + ÷=
or m =
c
or c ¢ =
c 6 è R 2 R ÷ø 2R
c¢ m
R = 4.5 cm
mt A + d m 60 + 30
Time taken = distance/velocity = t /( c / m ) = 28. (b) i= = = 45 º
c
2 2
v2 m 1 v 8 3
18. (b) = 1 = or v 2 = 1 = 2.25 ´ 10 m / s 29. (d) i1 = i 2 = A
v1 m 2 1 .33 1.33 4

c l 420 As A + d = i1 + i 2
19. (c) v= or l m = 0 \ lm = = 315 nm
m m ( 4 / 3) 3 3 A 60 º
\ d = i1 + i 2 - A = A+ A-A = = = 30 º
20. (c) Here i = 60º. As the angle between reflected and refracted 4 4 2 2
ray is 90º, then i + r = 90 or r = 30º 30. (b) Here, P1 = 5 D
sin i sin 60 3 /2 P2 = P – P1 = 2 – 5 = –3 D
Now m = = = = 3 = 1.732
sin r sin 30 1/ 2 w1 f - P2 3
=- 1 = =
The angle for which i + r = 90º , called Brewster' Angle. w2 f2 P1 5

1 -v 31. (c) Longitudinal chromatic aberration = w f


m= = u
21. (a) Þv= = 0.08 × 20 = 1.6 cm
n (-u ) n
f0
1 1 1 n 1 1 32. (a) =9, \ f0 = 9 fe
As + = \ - = fe
v u f u u f
Also f0 + fe = 20 (Q final image is at infinity)
u = (n – 1) f
9 fe + fe = 20, fe = 2 cm, \ f0 = 18 cm
1 1 1 33. (b) In normal adjustment,
22. (c) Given v = nu As + =
v u f
f0 f 60
M= = 20 , f e = 0 = = 3 cm
1 1 1
\ + = or u = (n + 1) f fe 20 20
nu u f n
23. (a) Here, u = f + x1, n = f + x2 f0 æ f e ö - 200 æç 1 + 5 ö÷ = - 48
34. (a) (i) M = - ç1 + ÷ =
fe è dø 5 è 25 ø
un
use f = and solve to get f = x 1x 2
u+n
Free eBooks on @neetquestionpaper2020

880 Physi cs
(since least distance d = 25cm)
1 æ1 1ö
= (1.5 - 1)ç - ÷
f 200 16 èR ¥ø
(ii) M = - 0 = - = -40
fe 5
1 1
35. (d) Amount of light entering into the camera depends upon Þ = 0.5 ´ Þ R = 8 cm
16 R
aperture setting the camera.
36. (b) Optical fibre is a device which transmits light introduced 47. (b) When we bring in contact a concave lens the effective
at one end to the opposite end, with little loss of the focal length of the combination decreases.
light through the sides of the fibre. It is possible with the 1 1 1 1 1 1
help of total internal reflection. - = Þ = +
v u f v u f
according to above relation as f reduces, v increases.
48. (b) 49. (b) 50. (d) 51. (d) 52. (d)
53. (a) 54. (b) 55. (b) 56. (b)
57. (a) In a telescope, to obtain an image at infinity or in normal
adjustment, the distance between two convex lenses
one called objective (greater focal length) and the other
called eye piece (shorter focal length) is L.
37. (d) If the medium is heterogeneous having a gradient of L = f0 + fe = 0.3 + 0.05 = 0.35 m.
refractive index. Then light rays will not follow a
58. (b) given : a mg = 3 ,a m w = 4
rectilinear (straight line path). 2 3
38. (d) Resolving power = l plane transmission grating Q a m w ´ w m g = a mg
dl
Resolving power for telescope a
w mg 3/ 2 9
1 d d \ mg = = = .
a 4/3 8
= = = 0 mw
limit of resolution 1.22l d l
59. (c)
by increasing the aperture of objective resolving power
60. (b) Object distance u = – 40 cm
can be increased.
Focal length f = – 20 cm
39. (b)
According to mirror formula
40. (a) Convex lens can form image with m < 1, m > 1 and
m = 1 depending upon the position of the object. 1 1 1 1 1 1
+ = or = -
Convex lens forms magnified image (m > 1) when the u v f v f u
object is pole and 2f, same size as the object (m = 1)
when the object is at 2f and smaller image (m < 1), when 1 1 1 1 1
or + - = +
the object is beyond 2f. v -20 ( -40 ) -20 40
41. (b) Virtual object forms real image on a plane mirror, so
rays convergent. 1 -2 + 1 1
= =- or v = -40 cm.
v 40 40
1 1 1 1 3 2
42. (a) - = Þ = Þv= f Negative sign shows that image is infront of concave
v 2f f v 2f 3
mirror. The image is real.

\m =
v 2 f
= =
1
Magnification, m =
-v
=-
( -40 ) = -1
u 3 2f 3 u ( -40 )
43. (a) Difference in refractive indices of blue and green colour
The image is of the same size and inverted.
are less so they are seen together and red is seen
separate because deviation depends on refractive velocity of light in vacuum (c)
index. 61. (b) m =
velocity of light in medium (v)
44. (b) Medium doesn't effect focal length of a mirror.
But v = ul = 2 × 1014 × 5000 × 10–10
45. (a) In the medium, v = 108 m/s.
1 æ 1 1 ö
46. (a) = (m - 1)çç - ÷ v vac 3 ´108
f ÷ \m= = = 3.
è R1 R 2 ø vmed 108
Free eBooks on @neetquestionpaper2020

Ray Optics and Optical Instruments 881


62. (d) Here, i1 = 15°, A = 60°, d = 55°, i2 = e = ?
d
As i1 + i2 = A + d Further, = cos r » 1
BC
i2 = A + d – i1 = 60° + 55° – 15° = 100°.
(When i is small r is small)
63. (a) When a red glass is heated to a high temperature it will
\ BC » d
glow with green light.
64. (c) For reading purposes : x
From eq. (1), » (i - r) or x » d (i - r)
u = – 25 cm, v = – 50 cm, f = ? d
1 1 1 1 1 1 100
= - =- + = ; P=
= +2 D æ rö
f v u 50 25 50 f or x » i d ç1 - ÷
è iø
For distant vision, f' = distance of far point = –3 m
sin i i æ 1ö
1 1 Now sin r = r = m \ x » i d çç1 - ÷÷
P= = - D = -0.33 D
f¢ 3 è mø
65. (c) E2 = 4 E1. If x is distance from 1st source, 4
5
70. (a) Given that w mg = and a m w =
Ι 4Ι 1 2 4 3
then, = or =
2 2 1 .2 - x
(1.2 - x) x x
5 4 5
3x = 2.4, x = 0.8 m \ a mg = w mg ´a m w = ´ =
4 3 3
1 h 1 1 real depth
1 ´ =
66. (d) E2 = E1 or, ( r 2 + h 2 )
r2 + h2 8 h
2 71. (c) We know that m = apparent depth
8
(by Lambert's cosine law) Let the thickness of the slab be t and real depth of the
bubble from one side be x. Then
or, ( r 2 + h 2 ) 3 / 2 = (2 h ) 3 or, (r 2 + h 2 )1 / 2 = 2 h
x (t - x) x t-x
or, r2 + h2 = 4 h2 m= = or 1 .5 = = .
6 4 6 4
h = r/ 3
t-9
67. (c) When, the incidence angle of ray is equal to Brewster’s This gives x = 9 and 1 . 5 = or t = 15 cm
4
angle then angle between reflected & refracted ray is
72. (d) Let d be the depth of two liquids.
90º (i.e., p/2)
Then apparant depth
incident
reflected ( d / 2) ( d / 2) d
ray + = or 1 + 2 = 1
i=

m 1. 5 m 2 m 3m
f

p/2
Solving we get m = 1.671
73. (c) The minimum length of the mirror is half the length of
the man. This can be proved from the fact that Ði = Ðr.
refracted ray
74. (d) Distance of image from plane mirror = 3 m at the back.
Apparent depth 1 To photograph the image, camera must be focussed
68. (b) Since =
Realdepth m for a distance of 4.5 + 3 = 7.5 m.

Þ Apparent depth = d/m 100 100


75. (d) P2 = P - P1 = - = -3 .75 D
So mark raised up = Real depth – Apparent depth 80 20

d æ 1 ö æ m -1 ö
= d- = d çç1 - ÷÷ = çç ÷÷d æ mg ö
m è mø è m ø ç - 1 ÷
Pa çè m a ÷
ø= +5
69. (c) From figure 76. (b) = = -5
i P1 æ m g ö - 100 / 100
ç ÷
x B ç m - 1÷
= sin (i - r) » (i - r ) ...(1) (i–r) è 1 ø
BC r
d m æ m g ö mg
D - 5 çç - 1÷÷ = -1
x è m 1 ø ma
C
Free eBooks on @neetquestionpaper2020

882 Physi cs
84. (b) Refractive index of medium is given by
1 .5 -1 1 .5 5
-1 = (1 .5 - 1) = -0 .1 ; m1 = =
m1 B
5 0.9 3 m =A+ ( where A and Bareconstant ) .
l2
sin (A + d m ) / 2 Light has seven different colour, so its each colour has
77. (b) m=
sin A / 2 different wavelength and so different refractive index.
Due to difference in refractive index different refractive
sin ( A + d m )
Þ = m sin A / 2` = 1.5 × sin 30º = 0.75 æ sin i ö
2 angle ç m = ÷.
è sin r ø
A + dm
= sin -1 (0 .75 ) = 48 º 3 6 ¢ \ dm = 37º12' So this is due to dependence on wavelength of
2 refractive index.
w1 w 2 85. (c) When a object is real plane mirror form a virtual image
78. (b) Condition for achromatism is + =0 and when object is virtual image will be real. Thus in
f1 f2
this question object is virtual. Virtual object means
object is at infinity. So rays (incident) converge on the
f1 w 2
\ = - 1 = (leaving sign) mirror.
f2 w2 3
79. (d) The necessary condition is
w f
= - which is satisfied by (d)
w¢ f¢

f1 w 1
80. (a) =- 1 =- \ f 2 = -2 f1
f2 w2 2 86. (c) Resolving power of eye = l / a

1 1 1 500 ´ 10 -9
As = + = = 10 -4 radians
F f1 f 2 5 ´ 10 -3

1 1 1 1 Now,\ arc = angle´ radius = 10–4 × (500 × 103)m = 50m


\ = - = \ f1 = 10 cm 87. (c) Spherical abberation occurs due to the inability of a
20 f1 2 f1 2 f1
lens to coverage marginal rays of the same wavelength
f2 = –20 cm to the focus as it converges the paraxial rays, This
b f0 f0 60 defect can be removed by blocking marginal rays. This
81. (b) M= = \ b= µ= ´ 2° = 24° can be done by using a circular annular mask over the
µ fe fe 5
lens.
f0 88. (d) Shift in the position of image after introducing slab,
82. (d) M= = 5 , L = f0 + fe = 36 æ 1ö
fe (d) = t ç1 - ÷
è nø
\ fe = 6 cm, f0 = 30 cm nd
nd = t (n – 1) Þ t =
83. (b) When two lenses are placed coaxially then their (n - 1)
equivalent focal length F is given as 89. (b) The easiest way to answer this question is with a fast
sketch. For a given object’s position, draw two rays
1 1 1 ff from the top of the object. One ray is parallel to the
= + ÞF= 1 2
F1 f1 f 2 f1 + f 2 principal axis and passes through the focal point on
the opposite side of the lens. The other ray passes
f1 f2 through the center of the lens. The top of the image
appears where these two rays intersect.

Object A Object B
F Image A Image B

1 f +f F
Now power = = 1 2
focal length f1f 2
Free eBooks on @neetquestionpaper2020

Ray Optics and Optical Instruments 883


Change the object’s position and repeat the process. 100. (a) 101. (b)
You will observe that as the object approaches the lens
while remaining beyond the focal length, the image 1 æ m 2 öæ 1 1 ö
102. (b) =ç - 1÷ç - ÷
produced on the opposite side of the lens moves away f çè m1 ÷øçè R1 R 2 ÷ø
from the lens and increases in size. As an aside, the
image is real and inverted. m2
90. (a) The ratio of object to image distance equals the ratio of As <1
m1
object to image height. The ratio of image to object height
is found by rearranging the ratios to give 4f /(4/3)f = 1/3. \ f is negative. It acts as divergent lens.
The image is demagnified by a factor of 3. Thus, answer 103. (b)
choice A is the best answer. 104. (b) As shown in the figure, when the object (O) is placed
l between F and C, the image (I) is formed beyond C. It is
91. (b) Angular limit of resolution of eye, q = , where d is in this condition that when the student shifts his eye
d
diameter of eye lens. towards left, the image appears to the right of the object
Also if y is the minimum just resolution separation pin.
between two objects at distance D from eye then
y
q=
D
y l lD
Þ = Þy= ...............(1)
D d d
Here : wavelength l = 5000Å = 5 ´ 10 -7 m
D = 50m
105. (c) The ray is partly reflected and partly refracted.
Diameter of eye lens = 2 mm = 2 ´ 10 -3 m ÐMOB = 180 – 2q
From eq. (1) minimum separation is But the angle between refracted and reflected ray is
ÐPOB. Clearly ÐPOB is less than ÐMOB.
5 ´ 10 -7 ´ 50
y= = 12.5 ´ 10 -3 m = 12.5 cm
2 ´ 10 -3

360° 360°
92. (d) n. = Þ = infinity
q 0
93. (a)
94. (a) Due to covering the reflection from lower part is not
there so it makes the image less bright.
95. (d) Least distance is 4f when object is at radius of curvature, 106. (a)
and greatest is infinity. 107. (d) For 1st lens, u1 = – 30, f1 = + 10cm,
96. (b) Due to difference in refractive indices images obtained
will be two. Two mediums will form images at two 1 1 1
Formula of lens, + =
different points due to difference in focal lengths. v1 30 10
97. (b) 98. (d)
v or, v1 = 15 cm at I1 behind the lens.
99. (b) m = The images I1 serves as virtual object for concave lens.
u
1 1 1 For second lens, which is concave, u2 = (15 – 5) = 10
- = cm. I1 acts as object. f2 = – 10 cm.
v u f
Multiply the equation by v The rays will emerge parallel to axis as the virtual object
is at focus of concave lens, as shown in the figure. Image
v v v v of I1 will be at infinity. These parallel rays are incident on
1- = Þ = 1-
u f u f the third lens viz the convex lens, f3 = + 30 cm. These
v parallel rays will be brought to convergence at the focus
\ m = 1-
f of the third lens.
1 b c \ Image distance from third lens = f3 = 30 cm
Slope = - = Þf =
f c b
Free eBooks on @neetquestionpaper2020

884 Physi cs

d d
\ Due to 1st liquid, 2= or x1 =
x1 2
d
Due to the second liquid, n =
x2
d
\ x2 =
n
d d
\ Total apparent depth = x1 + x2 = +
2 n
aµ = 1.5
108. (b) g
\ 1.5 = cosecC or C = 42°. Critical angle for glass Total apparent depth =
(
d n+ 2 )
= 42°. Hence a ray of light incident at 50° in glass n 2
medium undergoes total internal reflection. d denotes 110. (b) 111. (b) 112. (d)
the deviation of the ray.
113. (b) 114. (d) 115. (a)
\ d = 180° – (50° + 50°) or d = 80°.
116. (a) 117. (b) 118. (a)
Real depth d
109. (b) m = =
apparent depth x
Free eBooks on @neetquestionpaper2020

25
Wave Optics
WAVEFRONT HUYGENS WAVE THEORY
The locus of all particles of the medium vibrating in the same (Geometrical method to find the secondary wavefront)
phase at a given instant is called a wavefront. Depending on (i) Each point source of light is a centre of disturbance from
the shape of source of light, wavefront can be of three types. which waves spread in all directions.
(i) Spherical wavefront: A spherical wavefront is produced by a (ii) Each point on primary wavelets acts as a new source of
point source of light. This is because the locus of all such distrubance and produces secondary wavelets which travel
points which are equidistant from the point source will be a in space with the speed of light.
sphere. Spherical wavefronts are further divided into two (iii) The forward envelope of the secondary wavelets at any
headings: (i) converging spherical and (ii) diverging spherical instant gives the new wavefront.
wavefront.
A² A A¢
Primary A² A A¢ Secondary
wavefront wavefront

O O O

Spherical Converging Diverging


wavefront spherical wavefront spherical wavefront Secondary
O¢ wavelets
(ii) Cylindrical wavefront: When the B² B B¢
B² B B¢
source of light is linear in shape
such as a slit, the cylindrical (iv) In a homogeneous medium the wavefront is always
wavefront is produced. This is S perpendicular to the direction of wave propagation.
because all the points equidistant
from a line source lie on the surface With the help of Huygen’s wave theory, law of
of a cylinder. O reflection and refraction, total internal reflection and dispersion
Cylindrical wavefront can be explained easily. This theory also explain interference,
diffraction and polarization successfully.
(iii) Plane wavefront: A small part Drawbacks of Huygens Wave Theory
of a spherical or cylindrical
(a) This theory cannot explain photo-electric effect, compton,
wavefront due to a distant
and Raman effect.
source will appear plane and
(b) Hypothetical medium in vacuum is not true imagination.
hence it is called plane wave-
front. The wavefront of parallel (c) The theory predicted the presence of back wave, which
rays is a plane wavefront. proved to be failure.
Plane wavefront
Free eBooks on @neetquestionpaper2020

886 Physi cs
REFLECTION AND REFRACTION OF PLANE WAVES According to Huygens principle, every point on AB is a source
USING HUYGENS PRINCIPLE of secondary wavelets. Let the secondary wavelets from B strike
XY at A¢ in t seconds.
Reflection on the Basis of Wave Theory
\ BA¢ = c1 × t … (v)
According to Huygens principle, every point on AB is a source
of secondary wavelets. Let the secondary wavelets from B strike The secondary wavelets from A travel in the denser medium with
reflecting surface M1M2 at A¢ in t seconds. a velocity c2 and would cover a distance (c2 × t) in t seconds.
A¢B¢ is the true refracted wavefront. Let r be the angle of refraction.
\ BA ' = c ´ t … (i) As angle of refraction is equal to the angle which the refracted
where c is the velocity of light in the medium. plane wavefront A¢B¢ makes with the refracting surface AA¢,
therefore, ÐAA ' B ' = r .
3 1¢
B B¢ Let ÐAA ' B ' = r , angle of refraction.
BA ' c1 ´ t
2
N 2¢ In DAA¢B, sin i = =
D D¢ AA ' AA '
1 i r 3¢ AB ' c2 ´ t
i r In D AA¢B¢, sin r = =
A P A¢ AA ' AA '
M1 M2
sin i c1
The secondary wavelets from A will travel the same distance c × \ = =µ [using (iv)]
t in the same time. Therefore, with A as centre and c × t as radius, sin r c2
draw an arc B¢, so that sin i
Hence m= … (vi)
AB¢ = c × t … (ii) sin r
A¢B¢ is the true reflected wavefront. which proves Snell’s law of refraction.
angle of incidence, i = ÐBAA ' It is clear from fig. that the incident rays, normal to the interface
XY and refracted rays, all lie in the same plane (i.e., in the plane of
and angle of reflection, r = ÐB ' A ' A
the paper). This is the second law of refraction.
In Ds AA¢B and AA¢B¢, Hence laws of refraction are established on the basis of wave
AA¢ is common, BA ' = AB ' = c ´ t , and ÐB = ÐB ' = 90° theory.
\ Ds are congruent \ ÐBAA ' = ÐB ' A ' A, i.e., Ði = Ðr … (iii) Keep in Memory
Which is the first law of reflection.
Further, the incident wavefront AB, the reflecting surface M1M2 1. In 1873, Maxwell showed that light is an electromegnetic
and the reflected wavefront A¢B¢ are all perpendicular to the plane wave i.e. it propagates as transverse non-mechnical wave
of the paper. Therefore, incident ray, normal to the mirror M1M2 at speed c in free space given by
and reflected ray all lie in the plane of the paper. This is second
1
law of reflection. c= = 3 ´ 108 ms -1
Refraction on the Basis of Wave Theory m 0e 0
XY is a plane surface that separates a denser medium of refractive 2. There are some phenomenon of light like photoelectric
index µ from a rarer medium. If c1 is velocity of light in rarer effect, Compton effect, Raman effect etc. which can be
medium and c2 is velocity of light in denser medium, then by explained only on the basis of particle nature of light.
definition. 3. Light shows the dual nature i.e. particle as well as wave
c1 nature of light. But the wave nature and particle nature
µ= c … (iv) both cannot be possible simultaneously.
2 4. Interference and diffraction are the two phenomena that can
3 be explained only on the basis of wave nature of light.

B INTERFERENCE OF LIGHT WAVES AND YOUNG'S


2
DOUBLE SLIT EXPERIMENT
Rarer-C1
D N The phenomenon of redistribution of light energy in a medium
1 i on account of superposition of light waves from two coherent
P sources is called interference of light waves.
i A¢
X A r Y Young performed the experiment by taking two coherent sources
of light. Two source of light waves are said to be coherent if the
D¢ r initial phase difference between the waves emitted by the source

Denser-C2 remains constant with time.

2¢ (i) The rays of light from two coherent sources S1 and S2

superpose each other on the screen forming alternately
AB is a plane wave front incident on XY at ÐBAA ' = Ði . 1, 2, 3 maxima and minima (constructive and destructive
are the corresponding incident rays normal to AB. interference).
Free eBooks on @neetquestionpaper2020

Wa ve Optics 887

P or I = I1 + I 2 - 2 I1 I 2 ...(6)
S1 y
M1 It means that resultant intensity I is less than the sum
of individual intensities. Now as the position of point
S d O P on the screen changes, then the path difference at
point P due to these two waves also changes &
S2 M2
intensity alternately becomes maximum or minimum.
D Screen These bright fringes ( max. intensity) & dark fringes
(min. intensity) make an interference pattern.
(ii) Let the equation of waves travelling from S1 & S2 are It must be clear that there is no loss of energy
y1 = A1Sin wt ...(1) ( at dark fringe) & no gain of energy ( at bright fringe),
y 2 = A 2Sin wt ...(2) but, only there is a redistribution of energy.
where A1 & A2 are amplitudes of waves starting from S1 The shape of fringe obtained on the screen is
& S2 respectively. These two waves arrive at P by traversing approximately linear.
different distances S2P & S1P. Hence they are superimposed (B) Position of fringe:
with a phase difference (at point P) given by (i) If D = S2P – S1P = nl, then we obtain bright fringes at
poin t P on the screen and it corresponds to
2p
d ( phase difference ) = ´ D ( path difference ) constructive interference. So from equation (4) the
l position of n th bright fringe
2p yd
= (S2 P - S1P). ....(3) D = S 2 P - S1P = nl =
l D
2 æ nD ö
æ dö or y = ç ÷l
where S2 P (from fig) = D 2 + ç y + ÷ ...(7)
è ø 2 è d ø th
(Position of n bright fringe)
1 ( y + d / 2) 2
» D+ [\ D >> ( y + d)] l
2 2D (ii) If D = S2 P - S1P = (2n + 1) , then we obtain dark
2
( y – d / 2) 2 fringe at point P on the screen and corresponds to
Similarly, S1P » D +
4D destructive interference. So from equation(4), the
yd position of, n th dark fringe is
so, S2 P - S1P = ....(4)
D l yd
D = S2 P - S1P = (2n + 1) =
(A) Conditions for maximum & minimum intensity : 2 D
(i) Conditions for maximum intensity or constructive (2n + 1) Dl
interference : If phase difference or y = ...(8)
2d
d = 0, 2p, 4p – – – 2np (Position of nth dark fringe)
or, path difference D = S2 P - S1P = 0, l, 2l - - - nl (C) Spacing or fringe width :
then resultant intensity at point P due two waves Let yn and yn+1 are the distance of n th and (n+1)th bright
fringe from point O then
emanating from S1 & S2 is
Dnl D(n + 1)l
yn = & y n +1 =
I = A2 = A12 + A22 + 2 A1 A2 cos d (\ I µ A 2 ) d d
or I = ( A1 + A2 )2 So spacing b between nth and (n+1)th bright fringe is
Dl
or I = I1 + I 2 + 2 I1I 2 ....(5) b = y n +1 - y n = ...(9)
d
It means that resultant intensity is greater than the Since it is independent of n, so fringe width or spacing
sum of individual intensity ( where A is the amplitude between any two consecutive bright fringes is same.
of resultant wave at point P). Similarly the fringe width between any two consecutive
(ii) Conditions for minimum intensity or destructive dark fringe is
interference : If phase difference,
Dl
d = p, 3p, 5p - - - -(2n + 1)p b= ...(10)
d
or, path difference (D) Conditions for sustained interference:
l 3l l (i) The two sources should be coherent i.e they should
D = S2 P - S1P = , - - - (2n + 1) have a constant phase difference between them.
2 2 2
then resultant intensity at point P is (ii) The two sources should give light of same frequency
(or wavelength).
I = A2 = A12 + A22 – 2 A1 A2 cos d (iii) If the interfering waves are polarized, then they must
or I = ( A1 - A2 ) 2 be in same state of polarization.
Free eBooks on @neetquestionpaper2020

888 Physi cs
(E) Conditions for good observation of fringe: COHERENCE
(i) The distance between two sources i.e. d should be The phase relationship between two light waves can very from
small. time to time and from point to point in space. The property of
(ii) The distance of screen D from the sources should be definite phase relationship is called coherence.
quite large. 1. Temporal coherence : A light wave (photon) is produced
(iii) The two interfering wavefronts must intersect at a very when an excited atom goes to the ground state and emits
small angle. light.
(F) Conditions for good contrast of fringe : (i) The duration of this transition is about 10–9 to 10–10
(i) Sources must be monochromatic i.e they emit waves sec. Thus the emitted wave remains sinusoidal for this
of single wavelength. much time. This time is known as coherence time ( t c ).
(ii) The amplitude of two interfering waves should be (ii) Definite phase relationship is maintained for a length
equal or nearly equal.
L = ct c called coherence length.
(iii) Both sources must be narrow.
(iv) As Intensity I is directly proportional to the square of For neon l = 6238 Å , t c » 10-10 sec. and L = 0.03 m.
amplitude, hence Intensity of resultant wave at P,
For cadmium l = 6238 Å , t c = 10-9 and L = 0.3 m
I = I1 + I2 + 2 I1 I 2 cos f; if I1 = I 2 = I 0 . , then
For Laser t c = 10-5 sec and L = 3 km.
I = 4I 0 cos 2 f( 2) (iii) The spectral lines width Dl is related to coherence
length L and coherence time tc.
( )
2
(v) Imax = I1 + I2 . l2 l2
Dl » or Dl »
If I1 = I2 = I0, then Imax = 4I0 ct c L
2. Spatial coherence : Two points in space are said to be
( )
2
Imin = I1 - I 2 , if I1 = I2 = I0, then Imin = 0 spatially coherence if the waves reaching there maintains a
2 constant phase difference. Points P and Q are at the same
I æ I1 + I2 ö distance from S, they will always be having the same phase.
(vi) max = ç ÷ .
Imin è I1 - I2 ø Points P and P¢ will be spatially coherent if the distance
between P and P¢ is much less than the coherence length i.e.
b l
(vii) Angular fringe-width q0 = = PP ¢ << ct c
D d
(viii) The width of all interference fringes are same. Since

fringe width b is proportional to l, hence fringes with
red light are wider than those for blue light. P
(ix) If the interference experiment is performed in a medium
of refractive index m instead of air, the wavelength of Q
l
light will change from l to .
m Monochromatic
Dælö b source of light
i.e. b´= ç ÷ =
d èmø m Methods of Obtaining Coherent Sources
(x) If a transparent sheet of refractive index m and Two coherent sources are produced from a single source of light
thickness t is introduced in one of the paths of by two methods :
interfering waves, then due to its presence optical path (i) By division of wavefront and (ii) By division of amplitude.
will become mt instead of t. Due to this a given fringe (i) Division of wavefront : The wavefront emitted by a narrow
from present position shifts to a new position. So the source is divided in two parts by reflection, refraction or
lateral shift of the fringe, diffraction. The coherent sources so obtained are imaginary.
D b Example : Fresnel’s biprism, Llyod’s mirror, Young’s double
y0 = (m - 1)t = (m - 1)t
d l slit, etc.
(xi) In Young’s double slit experiment (coherent sources (ii) Division of amplitude : In this arrangement light wave is
in phase): Central fringe is a bright fringe. It is on the partly reflected (50%) and partly transmitted (50%) to
prerpendicular bisector of coherent sources. Central produced two light rays. The amplitude of wave emitted by
fringe position is at a place where two waves having an extended source of light is divided in two parts by partial
equal phase superpose. reflection and partial refraction. The coherent sources
(xii) Young’s experiment with the white light will give white obtained are real and are obtained in Newton’s rings,
central fringe flanked on either side by coloured bands. Michelson’s interferometer, etc.
Free eBooks on @neetquestionpaper2020

Wa ve Optics 889
Incoherence of Two Conventional Light Sources (ii) This interference pattern is frequently seen in a ripple tank
Let two conventional light sources L1 and L2 (like two sodium when one uses a wave train to demonstrate the law of
lamps or two monochromatic bulbs) illuminate two pin holes S1 reflection.
and S2. Then we will find that no interference pattern is seen on Dl
(iii) In this case, fringe width b =
the screen. d
The reason is as follows : In conventional light source, light Optical path : (Equivalent path in vacuum or air) In case of medium
comes from a large number of independent atoms, each atom of refractive index m and thickness t, the optical path = mt.
emitting light for about 10–9 seconds i.e., light emitted by an
atom is essentially a pulse lasting for only 10–9 seconds. Interference in Thin Films
We are familiar with the colours produced by a thin film of oil on
the surface of water and also by the thin film of a soap bubble.
Hooke observed such colours in thin films of mica and similar
thin transparent plates. Young was able to explain the phenomenon
L1 S1 on the basis of interference between light reflected from the top
and bottom surface of a thin film. It has been observed that
interference in the case of thin films takes place due to
S2 (i) reflected light and (ii) transmitted light.
L2 Interference due to reflected light
From the figure, the optical path difference between the reflected
ray (AT) from the top surface and the reflected ray (CQ) from the
Screen
bottom surface can be calculated. Let it be x, then
Even if all the atoms were emitting light pulses under similar
conditions, waves from different atoms would differ in their initial T
phases. Consequently light coming out from the holes S1 and S2 S
N Q
will have a fixed phase relationship only for 10–9 sec. Hence any i Air
interference pattern formed on the screen would last only for i
10–9 sec. (a billionth of a second), and then the pattern will A C
t r m
change. The human eye can notice intensity changes which last M
at least for a tenth of a second and hence we will not be able to B Air
see any interference pattern. Instead due to rapid changes in the r
pattern, we will only observe a uniform intensity over the screen. F
LIoyd’s Mirror P
x = m(AB + BC) - AN
The two sources are slit S ( parallel to mirror ) and its virtual image
S'. On simplification, we get
x = 2mt cos r
Super-position Screen l
occurs in this region 1. If 2mt cos r = (2n + 1) , where n = 0,1,2, ..............then
S 2
constructive interference takes place and the film appears
d O bright.
Flat black 2. If 2mt cos r = nl , where n = 0, 1, 2, 3,............ then destructive
S'
glass interference takes place and the film appears dark.
D Interference due to transmitted light
P
Lloyd mirror arrangement
S r
(i) If screen is moved so that, point O touches the edge of
i
glass plate, the geometrical path difference for two wave C Air
trains is zero. The phase change of p radian on reflection at A
denser medium causes a dark fringe to be formed. t r m
rr M
· The fring width remains unchanged on introduction r D
of transparent film. i Air
B
· If the film is placed in front of upper slit S1, the fringe N Q
pattern will shift upwards. On the other hand if the R
The optical path difference between the reflected ray (DQ) and
film is placed in front of lower slit S2, the fringe pattern
the transmitted ray (NR) is given by
shifts downwards.
x = m(BC + CD) - BN
Free eBooks on @neetquestionpaper2020

890 Physi cs
On simplification, we get Newton's rings by transmitted light
x = 2mt cos r Here, interference takes place due to transmitted light.
1. If 2mt cos r = nl , where n = 0, 1, 2, 3, .............then
constructive interference takes place and the film appears
bright.
l L
2. If 2mt cos r = (2n + 1) , n = 0, 1, 2, ....... then destructive
2
interference takes place and the film appears dark.
G
Newton's Rings
Newton observed the formation of interference rings when a
plano-convex lens is placed on a plane glass plate. When viewed
with white light, the fringes are coloured while with monochromatic Therefore,
light, the fringes are bright and dark. These fringes are produced For bright rings, 2mt cos q = nl, n = 0,1, 2,....
due to interference between the light reflected from the lower
surface of the lens and the upper surface of the glass plate. l
Interference can also take place due to transmitted light. For dark rings, 2mt cos q = (2n - 1) , n = 0,1, 2,......
2
Proceeding further, we get Radius of bright ring,
Air film
(2n - 1)lR
r=
2

Newton's rings by reflected light :


Here, interference takes place due to reflected light. Therefore, (i) The centre is bright and alternately bright and dark
for bright rings, rings are obtained.
l (ii) The ring pattern due to reflected light is just opposite
2 mt cos q = (2n - 1) where n = 1, 2, 3, ......
2 to that of transmitted light.
And for dark rings, 2 mt cos q = nl , n = 1, 2, 3, ......
Keep in Memory

1. If Dn and Dn + m be the diameters of n th and (n + m)th dark


rings then the wavelength of light used is given by

L (D n + m ) 2 - (Dn ) 2
l=
4mR
Air film
where, R is the radius of curvature of the lens.
G
2. If Dn = diameter of nth dark ring when air is present betwen
Proceeding further, we get the radius of rings as follows: the glass plate and the lens
Dn+m = diameter of (n+m)th dark ring when air is present
(2n - 1)lR between the glass plate and the lens
For bright rings, r =
2 D¢n = diameter of n th dark ring when a liquid is poured
For dark rings, r = nlR , where R = radius of curvature of between the plate and the lens
lens. D¢n+m = diameter of (n+m)th dark ring when a liquid is
poured between the plate and the lens
Then the refractive index of the liquid is given by
(i) The centre is dark and alternately dark and bright rings
are produced. (D n + m )2 - (Dn )2
(ii) While counting the order of the dark rings 1, 2, 3, etc. m= or,,
(D n¢ + m ) 2 - (D n¢ )2
the central ring is not counted. Therefore,
for 1st dark ring, n = 1 and r1 = lR 4mlR
m=
for 2nd dark ring, n = 2 and r2 = 2lR (D n¢ + m )2 - (D n¢ )2
Free eBooks on @neetquestionpaper2020

Wa ve Optics 891
Example 1. Example 4.
In Young’s expt., two coherent sources are placed 0.90 mm In young’s double slit experiment, the slits are illuminated
apart and fringes are observed one metre away. If it by white light. The distance between two slits is d and
produces second dark fringe at a distance of 1 mm from screen is D distance apart from the slits. Some wavelengths
central fringe, what would be the wavelength of are missing on the screen in front of one of the slits. These
monochromatic light used? wavelengths are
Solution :
d2 d2 2d2 d2
lD (a) λ = , (b) λ = ,
For dark fringes, x = (2 n - 1) D 3D D 4D
2d
-3
2xd 2 ´ 10 ´ 0.9 ´ 10 -3
\ l= = 2d 2 5d 2 d 2 d2
(2 n - 1) D (2 ´ 2 - 1) ´1 (c) λ = , (d) λ = ,
3D 3D D 2D
or, l = 0.6 ´ 10 -6 m = 6 ´10 -5 cm. Solution : (a)
Example 2.
Two beam of light having intensities I and 4 I interfere to P
produce a fringe pattern on a screen. The phase difference P1 x
S1
between the beams is p/2 at point A and p at point B. Then d/2
d
find the difference between the resultant intensities at A Source O
and B.
S2
Solution :
Here, I1 = I; I 2 = 4 I; q1 = p / 2, q2 = p D

I A = I1 + I 2 + 2 I1 I 2 cos q1 xd
Path difference at any point P in front of one of the slits =
= I + 4 I + 2 I ´ 4 I cos p / 2 = 5 I D
For missing wavelengths (i.e., for dark fringe)
I B = I1 + I 2 + 2 I1 I 2 cos q 2
l xd
= I + 4 I + 2 I 4 I cos p = 5 I - 4 I = I ; path difference = (2n - 1) =
2 D
\ IA - IB = 5 I - I = 4 I 2xd
Example 3. Þ l=
D(2n - 1)
A beam of light consisting of two wavelengths 6500 Å and but x = d/2, because point is P1, where we want missing
5200 Å is used to obtain fringes in a Young’s double slit wavelength.
experiment. (i) Find the distance of the third bright fringe
on the screen from central maximum for the wavelength d2
6500 Å. (ii) What is the least distance from the central \ l=
D(2n - 1)
maximum where the bright fringes due to both the
For n = 1, 2 ......... missing wavelengths are
wavelengths coincide? The distance between the slits is
2 mm and the distance between the plane of the slits and d2 d2 d2
the screen is 120 cm. , , ...............
D 3D 5D
Solution : Example 5.
lD In a biprism experiment, 5th dark fringe is obtained at a
(i) The fringe width b =
d point. If a thin transparent film is placed in the path of one
The distance of the third bright fringe is given by of waves, then 7th bright fringe is obtained at the same
3l D 3 ´ (6500 ´ 10-10 ) ´ 1.2 point. Determine the thickness of the film in terms of
x 3 = 3b = = = 0.117cm wavelength l and refractive index m .
d 2 ´ 10-3 Solution :
(ii) For least distance x, the nth bright fringe due to longer
wavelength coincides with the (n + 1)th bright fringe l D 9l D
For 5th dark fringe, x1 = (2 n - 1) =
of shorter wavelength. 2 d 2d
n ´ (6500 ´ 10 -10 )D (n + 1) (5200 ´ 10 -10 )D For 7th bright fringe, x 2 = n l
D 7lD
=
\ =
d d d d
Solving we get n = 4 D lD é 9ù D
but x 2 - x1 = (m - 1) t ; ê 7 - ú = (m - 1) t
Then least distance, where fringes due to both wavelengths d d ë 2û d
coincide is
2.5 l
4 ´ (6500 ´ 10-10 ) ´1.2 \ Thickness, t =
= = 0.156 cm (m - 1)
2 ´10 -3
Free eBooks on @neetquestionpaper2020

892 Physi cs
Example 6. p
In Young’s experiment, the interference pattern is found to Given, l = 6280 Å, a = 1° = radian
180
have an intensity ratio between the bright and dark fringes
6280 ´ 10-10
as 9. What is the ratio of (a) intensities (b) amplitudes of Thus d = ´ 180 = 3.6 ´ 10-5 m = 0.036 mm
the two interfering waves ? 3.14
Solution : Example 8.

In case of interference, I = I1 + I2 + 2 ( )
I1I 2 cos f
In Young’s experiment fringe width with l = 5900 Å is 0.45
mm. When a mica sheet (µ = 1.6) is introduced in the path
(a) For I to be maximum and minimum cos f is 1 and –1 of waves from one of the slits then the shift of the central
respectively, i.e., fringe is 1.9 mm. What is the thickness of the sheet ?
Solution :
I max = I1 + I 2 + 2 I1I 2 = ( I1 + I 2 ) 2 and It is known that if the path difference changes by l, then the
I min = I1 + I 2 - 2 I1I 2 = ( I1 ~ I 2 ) 2 central fringe shifts by b (width of one fringe). Therefore
According to given problem, when the path difference changes by (µ – 1) t the shift in

( ) =9, the central fringe should be, shift = (m - 1) tb / l


2
Imax I1 + I2
= It is given that, shift
( I2 )
I min 2 1
I1 - = 1.9 mm, µ = 1.6, l = 5900Å, b = 0.45 mm.
Therefore, thickness
Imax I1 + I2 3 -3 -10
i.e., I = = (shift) (l ) 1.9 ´ 10 ´ 5900 ´ 10
I1 - I2 1 t= =
min (m - 1) b (1.6 - 1) ´ 0.45 ´ 10-3
By comonendo and dividendo,
= 4.15 ´ 10-6 m = 4.15 µm
I1 3+1 I1 4
= i.e., I = 1 = 4 Example 9.
I2 3 - 1 2 In solar cells a silicon solar cell (µ = 3.5) is coated with a
(b) Now as for a wave I µ A 2 , thin film of silicon monoxide SiO (µ = 1.45) to minimize
2 2
reflective losses from the surface. Determine the minimum
I1 é A1 ù é A1 ù A thickness of SiO that produces the least reflection at a
=ê ú , ê ú = 4, i.e., 1 = 2
I2 ë A 2 û ë A 2 û A2 wavelength of 550 nm, near the centre of the visible
Example 7. spectrum.
In a Young’s double slit experiment the angular width of a Solution :
fringe formed on a distant screen is 1°. The wavelength of The reflected light is minimum when rays meet the condition
the light used is 6280 Å. What is the distance between the of destructive interference.
two coherent sources ? Both rays undergo a 180° phase change upon reflection.
Solution : The net change in phase due to reflection is therefore zero,
and the condition for a minimum reflection requires a path
l
The angular fringe width is given by a = difference of l m / 2 .
d
where l is wavelength and d is the distance between two
l l 550
l Hence, 2t = or t= = = 94.8 nm
coherent sources. Thus d = 2m 4m 4 (1.45)
a

25.1
Solve following problems with the help of above text and (a) A (b) D
examples. (c) B (d) C
1. The correct curve between refractive index m and 2. A very thin film in reflected white light appears
wavelength l will be (a) violet (b) white
(c) black (d) red
3. The fringe width for red colours as compared to that for
m B violet colour is approximately
D
(a) 3 times (b) 2 times
C (c) 4 times (d) 8 times
A 4. Intensity of light depends on
l (a) amplitude (b) frequency
(c) wavelength (d) velocity
Free eBooks on @neetquestionpaper2020

Wa ve Optics 893
5. In Young’s double slit experiment, the source S and two 15. Instead of using two slits, if we use two separate identical
slits A and B are lying in a horizontal plane. The slit A is sodium lamps in Young’s experiment, which of the
above slit B. the fringes are obtained on a vertical screen following will occur?
K. The optical path from S to B is increased by putting a (a) General illumination
transparent material of higher refractive index. The path (b) Widely separate interference
from S to A remains unchanged. As a result of this the (c) Very bright maxima
fringe pattern moves somewhat (d) Very dark minima
(a) upwards 16. The interfering fringes formed by a thin oil film on water
(b) downwards are seen in yellow light of sodium lamp. We find the fringes
(c) towards left horizontally (a) coloured
(d) towards right horizontally (b) black and white
6. The colour of bright fringe nearest to central achromatic (c) yellow and black
fringe in the interference pattern with white light will be (d) coloured without yellow
(a) violet (b) red 17. Which of the following is not essential for two sources of
(c) green (d) yellow light in Young’s double slit experiment to produce a
7. When a thin film of thickness t is placed in the path of sustained interference?
light wave emerging out of S1 then increase in the length (a) Equal wavelength
of optical path will be (b) Equal intensity
(a) (m – 1) t (b) (m + 1) t (c) Constant phase relationship
m (d) Equal frequency
(c) m t (d) 18. In Young’s double slit experiment, the minimum amplitude
t
8. The device which produces highly coherent sources is is obtained when the phase difference of super-imposing
(a) Fresnel biprism (b) Young’s double slit waves is (when n = 1, 2, 3, ...)
(c) Laser (d) Lloyd’s mirror (a) zero (b) (2 n – 1) p
9. Which of the following, cannot produce two coherent (c) n p (d) (n + 1) p
sources? 19. A thin sheet of glass (m = 1.5) of thickness 6 micron
(a) Lloyd’s mirror (b) Fresnel biprism introduced in the path of one of the interfering beams in a
(c) Young’s double slit (d) Prism double slit experiment shifts the central fringe to a position
10. Coherence is a measure of previously occupied by fifth bright fringe. Then the
(a) capability of producing interference by wave wavelength of light used is
(b) waves being diffracted (a) 6000 Å (b) 3000 Å
(c) waves being reflected (c) 4500 Å (d) 7500 Å
(d) waves being refracted 20. When monochromatic light is replaced by white light in
11. In Young’s double slit experiment, one slit is covered with Fresnel’s biprism arrangement, the central fringe is
red filter and another slit is covered by green filter, then (a) coloured (b) white
interference pattern will be (c) dark (d) None of these
(a) red (b) green 21. At the centre (t = 0) of Newton’s ring arrangment, we
(c) yellow (d) invisible observe a
12. Newton postulated his corpuscular theory of light on the (a) dark spot (b) bright spot
basis of (c) coloured spot (d) None of these
(a) Newton’s rings 22. The fringe width in a Young’s double slit experiment can
(b) rectilinear propagation of light be increased if we decrease
(c) colour through thin films (a) width of slits
(d) dispersion of white light into colours (b) separation of slits
13. Huygen’s concept of secondary wave (c) wavelength of light used
(a) allows us to find the focal length of a thick lens (d) distance between slits and screen
(b) is a geometrical method to find a wavefront 23. A monochromatic visible light consists of
(c) is used to determine the velocity of light (a) a single ray of light
(d) is used to explain polarisation (b) light of single wavelength
14. Two sources of light are said to be coherent, when they (c) light of many wavelength with a single colour
give light waves of same (d) light of many colours with a single wavelength
(a) amplitude and phase 24. When light suffers reflection at the interface between water
(b) wavelength and constant phase difference and glass, the change of phase in the reflected wave is
(c) intensity and wavelength (a) zero (b) p
(d) phase and speed (c) p/2 (d) 2p
Free eBooks on @neetquestionpaper2020

894 Physi cs

25. When a film is illuminated by white light, its upper portion


æxö æxö
appears dark. Path difference between two reflected beams (a) I 0 cos çç ÷÷ (b) I 0 cos 2 çç ÷÷
at the spot must be èbø èbø
(a) zero (b) l/2 (c) 2 l/2 (d) p
26. The correct formula for fringe visibility is æ px ö æI ö æ px ö
(c) I 0 cos 2 çç ÷÷ (d) ç 0 ÷ cos 2 çç ÷÷
I -I I +I è b ø è 4ø è b ø
(a) V = max min (b) V = max min
I max + I min I max - I min 31. The inverse square law of intensity (i.e., the intensity
I max I 1
(c) V= (d) V = min µ ) is valid for a
I min I max r2
27. Laser light is considered to be coherent because it consists
(a) point source (b) line source
of
(c) plane source (d) cylindrical source
(a) many wavelengths
(b) uncoordinated wavelengths 32. If Young’s double slit experiment is performed in water,
(c) coordinated waves of exactly the same wavelength (a) the fringe width will decrease
(d) divergent beam (b) the fringe width will increase
28. Wavelength of a laser beam can be used as a standard of (c) the fringe width will remain unchanged
(a) time (b) temperature (d) there will be no fringe
(c) angle (d) length 33. Four light waves are represented by
29. Soap bubble looks coloured due to (i) y = a1 sin wt (ii) y = a2sin (wt + e)
(a) dispersion (b) reflection
(iii) y = a1 sin 2wt (iv) y = a2sin 2(wt + e)
(c) interference (d) None of these
30. In a Young’s double-slit experiment, let b be the fringe We obtain sustained interference due to super-position
width, and let I0 be the intensity at the central bright fringe. of
At a distance x from the central bright fringe, the intensity (a) (i) and (ii) (b) (i) and (iii)
will be (c) (ii) and (iv) (d) (iii) and (iv)

ANSWER KEY
1. (a) 2. (c) 3. (b) 4. (a) 5. (b) 6. (a) 7. (a) 8. (c) 9. (d) 10.(a) 11. (d) 12. (b) 13. (b)
14. (b) 15.(a) 16. (a) 17. (b) 18. (b) 19. (a) 20.(b) 21. (a) 22. (a) 23. (b) 24. (b) 25. (b) 26.(a)
27. (c) 28. (d) 29. (c) 30. (c) 31.(a) 32. (a) 33. (a,b)

DIFFRACTION sin b p b sin q


E0 ' = E0 where b = and
When a wave is obstructed by an obstacle, the rays bend round b l
the corner. This phenomenon is known as diffraction. E0 = amplitude at the point P0 i.e. at q = 0
Fraunhoffer Diffraction by Single Slit sin 2 b
In Fraunhoffer diffraction experiment, the source and the screen The intensity at a general point P is given as I = I 0
b2
are effectively at infinite distance from the diffracting element.
In single slit diffraction, imagine aperature to be divided into two (ii) The graph for the variation of intensity as a function of
equal halves. Secondary sources in these two halves give first sinq is as follows :
minima at b sin q = l
I
I0
P
q
q
b P0
q I0 / 22 I0 / 62.5

-3l -2l -l O l 2l 3l sinq


b b b b
D b b

l
In general, b sin q = nl for minima and, b sin q = ( 2n + 1) for
(iii) The width of the central maxima is æç
2 lD ö
2 ÷ and angular
maxima. è b ø
(i) The points of the maximum intensity lie nearly midway
æ 2l ö
between the successive minima. The amplitude E0' of the width of central maxima is ç ÷ .
electric field at a general point P is è b ø
Free eBooks on @neetquestionpaper2020

Wa ve Optics 895
Fraunh offer Diffraction by a Circular Aperture Solution :
(i) The 1st dark ring is formed by the light diffracted from the For single-slit diffraction, the angular position of the first
circular aperture at an angle q with the axis where maximum is determined from the relation
1.22l 3l
sin q » where l = wavelength of light used, a sin q1¢ =
b 2
b = diameter of circular aperture It is given that a = 0.01 mm
Circular aperture = 1 × 10–5, q1¢ = 5.2°, sin q1¢ = 0.0906 . Therefore,
2 2
l= a sin q1¢ = ´ 10-5 ´ 0.0906 = 6040 Å
b q 3 3
Example 11.
In Fraunhaufer diffraction from a single slit of width 0.3
D mm the diffraction pattern is formed in the focal plane of a
Screen
lens of focal length 1m. If the distance of third minimum
(ii) If the screen is at a distance D (D >> b) from the circular from the central maximum is 5mm, then find the wavelength
aperture, the radius of the 1st dark ring is, of light used.
1.22lD Solution :

b The distance of n th minimum from the central maximum is
(iii) If the light transmitted by the hole is converged by a nlf
converging lens at the screen placed at the focal plane of given by X n =
a
1.22lf where it is given that
the lens, the radius of the 1st dark ring is R =
b a = 0.3 × 10–3 m, n = 3, f = 1m, Xn = 5 × 10–3 m
This radius is also called the radius of diffraction disc. Therefore,
For plane transmission diffraction grating aX n 0.3 ´ 10-3 ´ 5 ´ 10-3
(a + b) sin qn = nl for maxima, where a = width of transparent l= = = 5 ´ 10-7 m = 500 nm
nf 3 ´1
portion, b = width of opaque portion. Example 12.
Difference between Interference and Diffraction of light Angular width of central maximum in the Fraunhoffer-
Interference Diffraction diffraction pattern of a slit is measured. The slit is
illuminated by light of wavelength 6000 Å. When the slit
1. Interference is due to the 1. Diffraction is due to the
is illuminated by light of another wavelength, the angular
superposition of two superposition of two
width decreases by 30%. Calculate wavelength of this
wavefronts originating secondary wavelets
light. The same decrease in the angular width of central
from two coherent originating from the
sources. different points of the maximum is obtained when the original apparatus is
same wavefront. immersed in a liquid. Find refractive index of the liquid.
Solution :
2. In Interference pattern, 2. In diffraction pattern, the
Angular width is the angle subtended by the distance
all the maxima i.e. bright bright fringes are of
between first minima on either side at the centre of the slit.
fringes are of the same varying intensity.
intensity. It is given by f = 2 q , where q is the angle of diffraction.
For first diffraction minimum, a sin q = 1 l
3. In Interference pattern, the 3. In diffraction pattern, the
dark fringes are usually dark fringes are not l
or sin q = l/a or q=
almost perfectly dark. perfectly dark. a
4. In Interference pattern, the 4. In diffraction pattern, the 2l
\ Angular width f = 2 q = i.e. f µl
width of fringes (bright widths of fringes are not a
and dark) is equal. equal. f1 l1
= ;
5. In Interference, bands are 5. In diffraction, bands are f2 l 2
large in number. a few in number.
f2 70
6. In Interference, bands are 6. In diffraction, bands are \ l 2 = l1 = 6000 ´ = 4200 Å
f1 100
equally spaced. unequally spaced.
On immersing in liquid, a wavelength l = 6000 Å must be
Example 10. behaving as l' = 4200 Å to get the same decrease in angular
In a single slit diffraction experiment, the angular position width. Therefore, refractive index of medium
of the first (secondary) maximum is found to be 5.2°, when l 6000
the slit width is 0.01 mm. If sin 52° = 0.0906, then find the m= = = 1.43.
l¢ 4200
wavelength of light used.
Free eBooks on @neetquestionpaper2020

896 Physi cs

25.2
Solve following problems with the help of above text and The diffracted images A, B and C correspond to the first,
examples. second and third order diffraction. When the source is
1. The condition for observing Fraunhoffer diffraction from replaced by another source of shorter wave-length
a single slit is that the light wavefront incident on the slit
should be
(a) spherical (b) cylindrical
(c) plane (d) elliptical
2. The phenomenon of diffraction can be treated as
interference phenomenon if the number of coherent
sources is O A B C
(a) one (b) two
(c) zero (d) infinity (a) all the four will shift in the direction C to O
3. The position of the direct image obtained at O, when a (b) all the four will shift in the direction O to C
monochromatic beam of light is passed through a plane (c) the images C, B and A will shift towards O
transmission grating at normal incidence as shown in Fig. (d) the images C, B and A will shift away from O

ANSWER KEY
1. (c) 2. (d) 3. (c)

POLARISATION Polarization by Reflection (Brewster’s Law)


An ordinary source such as bulb consists of a large number of During reflection of a wave, we obtain a particular angle called
waves emitted by atoms or molecules in all directions angle of polarisation, for which the reflected light is completely
symmetrically. Such light is called unpolarized light (see fig - a) plane polarised.

Y Source
Reflected light
is polarised.
Direction of
ip
X wave motion
ip+rp=90º
Rarer
Z 90º
Denser
rp
Fig (a) Unpolarised light

m = tan (ip)
where, ip = angle of incidence, such that the reflected and refracted
Fig (b) Polarised light waves are perpendicular to each other.
If we confine the direction of wave vibration of electric vector in Law of Malus : If the electric vector is at angle q with the
one direction perpendicular to direction of wave propagation, transmission axis, light is partially transmitted. The intensity of
then such type of light is called plane polarised or linearly transmitted light is
polarised (with the help of polaroids or Nicol prism). The I = I 0 cos 2 q where I0 is the intensity when the incident electric
phenomenon by which, we restrict the vibrations of wave in a vector is parallel to the transmission axis.
particular direction (see fig-b) ^ to direction of wave
· Polarization can also be achieved by scattering of light
propagation is called polarization.
r
The plane of vibration is that which contains the vibrations of · (a) Plane polarized : oscillating E field is in a single plane.
r r
electric vector E and plane of polarisation is perpendicular to (b) Circularly polarized : tip of oscillating E field describes
the plane of vibration a circle.
· Tourmaline and calcite polarizes an e.m. wave passing r
through it. (c) Elliptically polarized : tip of oscillating E field
describes an ellipse.
Free eBooks on @neetquestionpaper2020

Wa ve Optics 897
Example 13 :
The intensity of the polarised light becomes 1/20th of its A N B
initial intensity after passing through the analyser. What
is the angle between the axis of the analyser and the initial 57º
amplitude of the light beam ? 33º 33º
O
Solution :
1
Here I = I0 = 0.05 I0
20 (a) the intensity is reduced down to zero and remains
zero
Using I = I0 cos2 q , we get 0.05 I0 = I0 cos 2 q (b) the intensity reduces down somewhat and rises again
(c) there is no change in intensity
Þ cos 2 q = 0.05 or cos q = 0.05 = 0.2236
(d) the intensity gradually reduces to zero and then again
\ q = cos-1 (0.2236) = 76°9 ¢ increases
Solution : (d)
Example 14 :
For complete polarisation of reflected light
A beam of polarised light makes an angle of 60° with the
axis of the polaroid sheet. How much is the intensity of m = tan f (f = Brewster’s angle)
light transmitted through the sheet ? \ f = tan–1 m = tan–1 (1.54) = 57º
Solution : From fig, angle of incidence = 90° – 33º = 57º
Here q = 60°, Hence the reflected light is completely polarised. When the
plane polarised light is viewed through a rotating nicol prism,
Using I = I0 cos2 q , we get the intensity gradually reduces to zero and then again
increases.
1 æ 1ö
I = I0 (cos 60°) 2 = I 0 çQ cos 60° = ÷ Example 17.
4 è 2ø
A beam of plane polarised light falls normally on a polariser
1 (cross sectional area = 3 ×10–4 m2) which rotates about
\ Intensity of transmitted light = ´ 100 = 25% the axis of the ray with an angular velocity of 31.4 rad/sec.
4
Find the energy of light passing through the polariser per
Thus, the intensity of the transmitted light is 25% of the revolution and the intensity of the emerged beam if flux of
intensity of incident light. energy of incident ray is 10–3 W.
Example 15 : Solution :
A ray of light strikes a glass plate at an angle of 60° with Here, A = 3 × 10–4 m2,
the glass surface. If the reflected and refracted rays are at
w = 31.4 rad/s, flux f = 10–3 W
right angles to each other, find the refractive index of the
glass. Time taken to complete one revolution,
Solution. 2 p 2 ´ 3.14
T= = = 0. 2 s
When the reflected and refracted rays are at right angle to w 31.4
each other, the angle of incident is known as angle of Energy transmitted per revolution
polarisation (ip).
1
Here, q = 60°, Using m = tan ip, we get = ´ incident flux density (f) × T
2
m = tan 60° = 3 = 1.732 1
= ´ 10-3 ´ 0.2 = 10 -4 J
Example 16 : 2
A beam of light AO is incident on a glass slab (m = 1.54) in 1 incident flux density
a direction as shown in fig. The reflected ray OB is passed Intensity of emergent beam = ´
2 Area
through nicol prism. On viewing through a nicol prism,
1 10 -3 5
we find on rotating the prism that = ´ = Watt m - 2
2 3 ´ 10 -4 3
Free eBooks on @neetquestionpaper2020

898 Physi cs

25.3
Solve following problems with the help of above text and 5. From Brewster’s law of polarisation, it follows that the
examples. anlge of polarisaiton depends upon
1. Light waves can be polarised because they (a) the wavelength of light
(a) have high frequencies (b) plane of polarisation’s orientation
(b) have short wavelength (c) plane of vibration’s orientation
(d) None of these
(c) are transverse
6. Optically active substances are those substances which
(d) can be reflected
(a) produces polarised light
2. When unpolarised light is incident on a plane glass plate (b) produces double refraction
at Brewster’s angle, then which of the following statements (c) rotate the plane of polarisation of polarised light
is correct? (d) converts a plane polarised light into circularly
(a) Reflected and refracted rays are completely polarised polarised light.
with their planes of polarization parallel to each other 7. Polaroid glass is used in sun glasses because
(b) Reflected and refracted rays are completely polarised (a) it reduces the light intensity to half on account of
with their planes of polarization perpendicular to each polarisation
other (b) it is fashionable
(c) Reflected light is plane polarised but transmitted light (c) it has good colour
is partially polarised (d) it is cheaper.
(d) Reflected light is partially polarised but refracted light 8. In the propagation of light waves, the angle between the
is plane polarised plane of vibration & plane of polarisaiton is
(a) 0º (b) 90º (c) 45º (d) 80º
3. Light transmitted by nicol prism is
9. In the propagation of electromagnetic waves, the angle
(a) unpolarised between the direction of propagation and plane of
(b) plane polarised polarisation is
(c) circularly polarised (a) 0º (b) 45º (c) 90º (d) 180º
(d) elliptically polarised 10. If for a calcite crystal, m0 and me are the refractive indices
4. If the light is polarised by reflection, then the angle of the crystal for O-ray and E-ray respectively, then along
between reflected and refracted light is the optic axis of the crystal
(a) 180º (b) 90º (a) m0 = me (b) m0 > me
(c) 45º (d) 36º (c) m0 < me (d) None of these
ANSWER KEY
1. (c) 2. (c) 3. (b) 4. (b) 5. (a) 6. (c) 7. (a) 8. (b) 9. (a) 10. (a)

RESOLVING POWER OF AN OPTICAL INSTRUMENT (b) When the source moves away from the stationary observer
The resolving power of an optical instrument, is its ability to
æ vö
distinguish between two closely spaced objects. or vice-versa, n´= n ç1 - ÷ (Red shift)
Diffraction occurs when light passes through the circular, or è ø c
nearly circular, openings that admit light into cameras, telescopes, where n´ = apparent frequency, n = active frequency
microscopes, and human eyes. The resulting diffraction pattern
places a natural limit on the resolving power of these instruments. v = velocity of source, c = velocity of light
For example, for normal vision, the limit of resolution of normal But in both cases, the relative velocity v is small.
human eye is ~0.1 mm from 25 cm. (i.e., distances less than 0.1 mm Example 18 :
cannot be resolved). For optical microscope the limit of resolution
~ 10–5 cm and for electron microscope ~5 Å or less. The time period of rotation of the sun is 25 days and its
0.61l radius is 7 × 108 m. What will be the Doppler shift for the
The limit of resolution of a microscope x = where a is the light of wavelength 6000 Å emitted from the surface of the
a
aperture of the microscope. sun?
DOPPLER’S EFFECT FOR LIGHT WAVES Solution :
(a) When the source moves towards the stationary observer v ælö æ 2p ö æ l ö
or the observer moves towards the source, the apparent Doppler’s shift d l = ´ l = R wç ÷ = R ç ÷ç ÷
c ècø è T øècø
frequency.
æ vö 7 ´ 108 ´ 2 ´ 22 6000
n´= n ç1 + ÷ ( Blue shift ) = ´ Å = 0.04 Å
è cø 25 ´ 24 ´ 60 ´ 60 ´ 7 3 ´ 108
Free eBooks on @neetquestionpaper2020

Wa ve Optics 899
Example 19 :
It is given that l = 6 ´ 10 -7 m , and the numerical aperture
How far in advance can one detect two headlights of a car
if they are separated by a distance of 1.57 m ? m sin q = 0.12 .
Solution : 0.61 ´ 6 ´ 10-7
The human eye can resolve two objects when the angle Therefore, x = = 3.05 ´ 10-6 m » 3 µm
0.12
between them is 1 minute of arc. Thus, we have Example 21 :
x
D= A person wants to resolve two thin poles standing near
q each other at a distance of 1 km. What should be the
1 p minimum separation between them?
Here x = 1.57 m, q = 1¢ = ´ rad ,
60 180 Solution :
1.57 10800 ´ 1.57 Angular limit of resolution of eye q = 1 minute of arc
Thus D = = = 5400 m = 5.4 km
1 p 3.14 = 1/60 degree.
´
60 180 Therefore, the minimum separation should be such that
Example 20 : x = Dq
The numerical aperture of a microscope is 0.12, and the 1 p
wavelength of light used is 600 nm. Then find its limit of with D = 1 km. = 103 m and q = ´ radian
60 180
resolution.
Solution : 10 3 ´ 3.14 31.4
Thus x = = = 0.29 m
The limit of resolution of a microscope is given by 60 ´ 180 108
0.61 l or x » 30 cm.
x=
m sin q

25.4
Solve following problems with the help of above text and (a) approaching the earth
examples. (b) receding away from the earth
1. When a source of light is receding away from an observer, (c) stationary
then the spectral lines will get displaced towards (d) rotating about its own axis
(a) violet region (b) red region 4. The limit of resolution of eye is approximately
(c) ultraviolet region (d) blue region (a) 1º (b) 1'
2. An observer measures speed of light to be c, when he is (c) 1 mm (d) 1 cm
stationary w.r.t. the source. If observer moves with velocity 5. What should be the refractive index of a transparent
v towards the source, then the velocity of light observed medium to be invisible in vacuum?
will be (a) 1 (b) < 1
(a) c (b) c + v (c) > 1 (d) None of these
6. The ability of an optical instrument to show the images of
(c) c – v (d) 1 - v 2 / c 2
two adjacent point objects as separate is called
3. An observer on earth observes that wavelength of spectral
(a) dispersive power (b) magnifying power
line in spectrum of a milky way shifts towards red end of
(c) resolving power (d) None of these
spectrum. According to the observer, the milky-way is

ANSWER KEY
1. (a) 2. (c) 3. (b) 4. (b) 5. (b) 6. (c)
Free eBooks on @neetquestionpaper2020

900 Physi cs

Very Short / Short Answer Questions 17. (a) In Young’s double slit experiment, derive the condition
for (i) constructive interference and (ii) destructive
1. What is a wavefront ? interference at a point on the screen.
2. What is the relation between the path difference and the (b) A beam of light consisting of two wavelengths, 800
phase difference? nm and 600 nm is used to obtain the interference fringes
3. What is the resolving power of a microscope? in a Young’s double slit experiment on a screen placed
4. What will be the effect on the fringes, if young’s double slit 1.4 m away. If the two slits are separated by 0.28 mm,
experiment set up is immersed in water? calculate the least distance from the central bright
5. State Huygen's postulates of wave theory. maximum where the bright fringes of the two
6. Give two points of difference between interference and wavelengths coincide. [Outside Delhi - 2011]
diffraction. 18. (a) How does an unpolarized light incident on polaroid
7. Explain the formation of secondary maxima in the diffraction get polarized?
pattern due to a single slit. Describe briefly, with the help of a necessary diagram,
8. What are polaroids? Write their uses. the polarization of light by reflection from a transparent
9. State two conditions required for obtaining coherent medium.
sources. [Delhi Board - 2012 COMPTT.] (b) Two polaroids ‘A’ and ‘B’ are kept in crossed position.
10. How does the angular separation between fringes in single- How should a third polaroid ‘C’ be placed between
them so that the intensity of polarized light transmitted
slit diffraction experiment change when the distance of
by polaroid B reduces to 1/8th of the intensity of
separation between the slit and screen is doubled?
unpolarized light incident on A?
[Outside Delhi - 2012]
[Outside Delhi - 2012]
11. Why are coherent sources necessary to produce a
sustained inteference pattern? [Delhi Board - 2010] Multiple Choice Questions
12. State briefly two features which can distinguish the 19. Which one of the following phenomena is not explained by
characteristic features of an interference pattern from those Huygen’s construction of wavefront?
observed in the diffraction pattern due to a single slit.
(a) Refraction (b) Reflection
[Outside Delhi - 2011 COMPTT.] (c) Diffraction (d) Origin of spectra
13. Laser light of wavelength 630 nm incident on a pair of slits 20. Which of the following phenomena is not common to sound
produces an interference pattern in which the bright fringes and light waves ?
are separated by 7.2 mm. Calculate the wavelength of
(a) Interference (b) Diffraction
another source of laser light which produces interference
(c) Coherence (d) Polarisation
fringes separated by 8.1 mm using same pair of slits.
21. Interference is possible in
[Outside Delhi - 2011 COMPTT.]
(a) light waves only
Long Answer Questions (b) sound waves only
(c) both light and sound waves
14. (a) Using Huygen's principle explain the refraction of a
(d) neither light nor sound waves
plane wave and hence prove the Snell's law of
22. A single slit diffraction pattern is obtained using a beam of
refraction.
red light. If the red light is replaced by the blue light, then
(b) What happens if a plane wavefront travelling in a the diffraction pattern
denser medium is incident on the interface of a rarer
(a) remains unchanged (b) becomes narrower
medium at an angle greater than critical angle?
(c) becomes broader (d) will disappear
15. In Young's double-slit experiment, the fring-width was found
23. In Young's double slit experiment, if the slit widths are in the
0.4 mm for the light of wavelength 6000Å. What will be the
ratio 1 : 2, the ratio of the intensities at minima and maxima
fringe width if entire apparatus is immersed in a liquid of
will be
refractive index 1.33?
(a) 1 : 2 (b) 1 : 3 (c) 1 : 4 (d) 1 : 9
16. Explain briefly, giving a suitable diagram, how an
24. If a wave can be polarized, it must be
unpolarised light incident on the interface separating two
(a) a transverse wave (b) a longitudinal wave
transparent media gets polarised on reflection. Deduce the
necessary condition for it. (c) a sound wave (d) a stationary wave
Free eBooks on @neetquestionpaper2020

Wa ve Optics 901
25. Interference was observed in interference chamber where 26. To demonstrate the phenomenon of interference, we require
air was present, now the chamber is evacuated, and if the two sources which emit radiation of
same light is used, a careful observer will see (a) nearly the same frequency
(a) no interference (b) the same frequency
(b) interference with brighter bands (c) different wavelengths
(c) interference with dark bands (d) the same frequency and having a definite phase
(d) interference fringe with larger width relationship

1. A lens having focal length f and aperture of diameter d 6. The maximum number of possible interference maxima for
d slit-separation equal to twice the wavelength in Young’s
forms an image of intensity I. Aperture of diameter in double-slit experiment is [AIEEE 2004]
2
central region of lens is covered by a black paper. Focal (a) three (b) five (c) infinite (d) zero
length of lens and intensity of image now will be 7. Two point white dots are 1 mm apart on a black paper.
respectively [CBSE-PMT 2010] They are viewed by eye of pupil diameter 3 mm.
I 3f I Approximately, what is the maximum distance at which these
(a) f and (b) and dots can be resolved by the eye? [Take wavelength of light
4 4 2
3I f I = 500 nm] [AIEEE 2005]
(c) f and (d) and (a) 1 m (b) 5 m (c) 3 m (d) 6 m
4 2 2
2. In Young’s double slit experiment, the slits are 2 mm apart and 8. A Young’s double slit experiment uses a monochromatic
are illuminated by photons of two wavelengths l1 = 12000Å source. The shape of the interference fringes formed on a
and l2 = 10000Å. At what minimum distance from the screen is [AIEEE 2005]
common central bright fringe on the screen 2 m from the slit (a) circle (b) hyperbola
will a bright fringe from one interference pattern coincide (c) parabola (d) straight line
with a bright fringe from the other ? (NEET 2013) 9. If I 0 is the intensity of the principal maximum in the single
(a) 6 mm (b) 4 mm (c) 3 mm (d) 8mm slit diffraction pattern, then what will be its intensity when
3. A parallel beam of fast moving electrons is incident normally the slit width is doubled? [AIEEE 2005]
on a narrow slit. A fluorescent screen is placed at a large
I0
distance from the slit. If the speed of the electrons is (a) 4 I 0 (b) 2 I 0 (c) (d) I 0
increased, which of the following statements is correct ? 2
(NEET 2013) 10. When an unpolarized light of intensity I 0 is incident on a
(a) The angular width of the central maximum of the polarizing sheet, the intensity of the light which does not
diffraction pattern will increase. get transmitted is [AIEEE 2005]
(b) The angular width of the central maximum will decrease. 1 1
(a) I0 (b) I0 (c) I 0 (d) zero
(c) The angular width of the central maximum will be 4 2
unaffected. 11. In a Young’s double slit experiment the intensity at a point
(d) Diffraction pattern is not observed on the screen in l
where the path difference is (l being the wavelength of
case of electrons. 6
4. To demonstrate the phenomenon of interference, we require I
light used) is I. If I0 denotes the maximum intensity, is
two sources which emit radiation of [AIEEE 2003] I0
(a) nearly the same frequency equal to [AIEEE 2007]
(b) the same frequency 3 1 3 1
(a) (b) (c) (d)
(c) different wavelengths 4 2 2 2
(d) the same frequency and having a definite phase Directions for Qs. 12 to 14 : Read the passage given below and
relationship answer the questions that follow :
5. The angle of incidence at which reflected light is totally PASSAGE
polarized for reflection from air to glass (refractive index n), An initially parallel cylindrical beam travels in a medium of
is [AIEEE 2004] refractive index m (I) =m0 + m2 I, where m0 and m2 are positive
(a) tan–1(1/n) (b) sin–1(1/n) constants and I is the intensity of the light beam. The intensity of
(c) sin–1(n) (d) tan–1(n) the beam is decreasing with increasing radius.
Free eBooks on @neetquestionpaper2020

902 Physi cs
12. As the beam enters the medium , it will [AIEEE 2010] 18. A beam of light of wavelength 600 nm from a distance
(a) diverge source falls on a single slit 1 mm wide and a resulting
(b) converge diffraction pattern is observed on a screen 2m away. The
(c) diverge near the axis and converge near the periphery distance between the first dark fringes on either side of
(d) travel as a cylindrical beam central bright fringe is [IIT JEE 1994]
13. The initial shape of the wavefront of the beam is (a) 1.2 cm (b) 1.2 mm (c) 2.4 cm (d) 2.4 mm
(a) convex [AIEEE 2010] 19. In a double slit experiment instead of taking slits of equal
widths, one slit is made twice as wide as the other. Then, in
(b) concave
the interference pattern [IIT JEE 2000 S]
(c) convex near the axis and concave near the periphery
(a) the intensities of both the maxima and the minima
(d) planar
increase
14. The speed of light in the medium is [AIEEE 2010]
(b) the intensity of the maxima increases and the minima
(a) minimum on the axis of the beam has zero intensity
(b) the same everywhere in the beam (c) the intensity of the maxima decreases and that of the
(c) directly proportional to the intensity I minima increases
(d) maximum on the axis of the beam (d) the intensity of the maxima decreases and the minima
15. In Young's double slit experiment, one of the slit is wider has zero intensity.
than other, so that amplitude of the light from one slit is 20. Monochromatic light of wavelength 400 nm and 560 nm
double of that from other slit. If Im be the maximum intensity, are incident simultaneously and normally on double slits
the resultant intensity I when they interfere at phase apparatus whose slits separation is 0.1 mm and screen
difference f is given by [AIEEE 2012] distance is 1m. Distance between areas of total darkness
Im Im æ 2 fö will be [IIT JEE 2004 S]
(a) (4 + 5 cos f) (b) ç 1 + 2 cos ÷ (a) 4mm (b) 5.6 mm (c) 14mm (d) 28mm
9 3 è 2ø
21. A source emits sound of frequency 600 Hz inside water.
Im æ 2 fö Im æ 2 fö The frequency heard in air will be equal to (velocity
(c) ç 1 + 4 cos ÷ (d) ç 1 + 8 cos ÷
5 è 2ø 9 è 2ø of sound in water = 1500 m/s, velocity of sound in air
16. Abeam of unpolarised light of intensity I0 is passed through = 300 m/s) [IIT JEE 2004 S]
a polaroid A and then through another polaroid B which is (a) 3000 Hz (b) 120 Hz
oriented so that its principal plane makes an angle of 45° (c) 600 Hz (d) 6000 Hz
relative to that of A. The intensity of the emergent light is 22. In Young’s double slit experiment intensity at a point is
[JEE Main 2013] (1/4) of the maximum intensity. Angular position of this
(a) I0 (b) I0/2 point is [IIT JEE 2005 S]
(c) I0/4 (d) I0/8 -1
(a) sin (l/d) -1
(b) sin (l/2d)
17. Two coherent point sources S1 and S2 are separated by a (c) sin-1(l/3d) (d) sin-1(l/4d)
small distance 'd' as shown in figure. The fringes obtained 23. Young’s double slit experiment is carried out by using green,
on the screen will be [JEE Main 2013] red and blue light, one color at a time. The fringe widths
recorded are bG, bR and bB, respectively. Then,
[IIT-JEE 2012]
(a) bG > bB > bR (b) bB > bG > bR
d (c) bR > bB > bG (d) bR > bG > bB
S1 S2 24. In the Young’s double slit experiment using a monochromatic
Screen
light of wavelength l, the path difference (in terms of an
D integer n) corresponding to any point having half the peak
intensity is (JEE Adv. 2013)

(a) points (b) straight lines l l


(a) (2n + 1) (b) (2n + 1)
(c) semi-circles (d) concentric circles 2 4
l l
(2 n + 1)
(c) 8 (d) (2n + 1)
16
Free eBooks on @neetquestionpaper2020

Wa ve Optics 903

1. The width of a slit is 0.012 mm. Monochromatic light is 8. The separation between successive fringes in a double slit
incident on it. The angular position of first bright line is arrangement is x. If the whole arrangement is dipped under
5.2º. The wavelength of incident light is water what will be the new fringe separation? [The
[sin 5.2º = 0.0906]. wavelenght of light being used is 5000 Å]
(a) 6040 Å (b) 4026 Å (a) 1.5 x (b) x
(c) 5890 Å (d) 7248 Å (c) 0.75 x (d) 2 x
2. A ray of light is incident on the surface of a glass plate at an 9. Light of wavelength 6328 Å is incident normally on a slit
angle of incidence equal to Brewster’s angel f. If m represents having a width of 0.2 mm. The angular width of the central
the refractive index of glass with respect to air, then the angle maximum measured from minimum to minimum of diffraction
between the reflected and the refracted rays is pattern on a screen 9.0 metres away will be about
(a) 90° + f (b) sin –1 (m cos f) (a) 0.36 degree (b) 0.18 degree
(c) 0.72 degree (d) 0.09 degree
æ sin f ö
(c) 90º (d) 90 º - sin -1 çç ÷÷ 10. If wavelength of a wave is l = 6000 Å, then wave number
è m ø will be
3. Light of wavelength 6.5 × 10–7 m is made incident on two (a) 1.66 × 107 m–1 (b) 1.66 × 106 m–1
slits 1 mm apart. The distance between third dark fringe and (c) 16.6 × 10 m –1 –1 (d) 166 × 103 m–1
fifth bright fringe on a screen distant 1 m from the slits will 11. A plane wave of wavelength 6250 Å is incident normally on
be a slit of width 2 × 10–2 cm. The width of the principal maximum
(a) 0.325 mm (b) 0.65 mm on a screen distant 50 cm will be
(c) 1.625 mm (d) 3.25 mm (a) 312.5 × 10–3 cm (b) 312.5 × 10–3 m
4. The max. intensity produced by two coherent sources of (c) 312.5 × 10 m –3 (d) 312 m
intensity I1 and I2 will be 12. A ray of light strikes a glass plate at an angle of 60º. If the
(a) I1 + I 2 (b) I12 + I 22 reflected and refracted rays are perpendicular to each other,
the index of refraction of glass is
(c) I1 + I 2 + 2 I1 I 2 (d) zero 1 3
(a) (b) 3 (c) (d) 1.732
5. The path difference between two wavefronts emitted by 2 2
2
coherent sources of wavelength 5460Å is 2.1 micron. The
phase difference between the wavefronts at that point is 13. The wavelength of Ha line in hydrogen spectrum was found
to be 6563 Å in the laboratory. If the wavelength of same
(a) 7.692 (b) 7.692 p
line in the spectrum of a milky way is observed to be 6568Å,
7.692 7.692 then recession velocity of milky way will be
(c) (d)
p 3p (a) 105 m/s (b) 1.05 × 106 m/s
6
(c) 10.5 × 10 m/s (d) 0.105 × 106 m/s
d
6. In Young’s expt., the distance between two slits is and 14. A star is receding away from earth with a velocity of
3
the distance between the screen and the slits is 3 D. The 105 m/s. If the wavelength of its spectral line is 5700 Å, then
1 Doppler shift will be
number of fringes in m on the screen, formed by (a) 200 Å (b) 1.9 Å
3
monochromatic light of wavelength 3l, will be (c) 20 Å (d) 0.2 Å
d d 15. The phenomenon by which stars recedes from each other
(a) (b) is explained by
9Dl 27 D l
(a) black hole theory (b) neutron star theory
d d
(c) (d) (c) white dwarf (d) red shift
81 D l Dl
16. Angular width (B) of central maxima of a diffraction pattern
7. In Young’s double slit expt. the distance between two of a single slit does not depend upon
sources is 0.1 mm. The distance of the screen from the (a) distance between slit and source
source is 20 cm. Wavelength of light used is 5460 Å. The
(b) wavelength of the light used
angular position of the first dark fringe is
(c) width of slit
(a) 0.08º (b) 0.16º
(d) frequency of light used
(c) 0.20º (d) 0.32º
Free eBooks on @neetquestionpaper2020

904 Physi cs
17. Which of the following does not support the wave nature 28. The condition for obtaining secondary maxima in the
of light? diffraction pattern due to single slit is
(a) Interference (b) Diffraction l
(c) Polarisation (d) Photoelectric effect. (a) a sin q = nl (b) a sin q = ( 2n - 1)
2
18. The colours seen in the reflected white light from a thin oil nl
film are due to (c) a sin q = ( 2n - 1) l (d) a sin q =
2
(a) diffraction (b) interference 29. Light from two coherent sources of the same amplitude A
(c) polarisation (d) dispersion and wavelength l illuminates the screen. The intensity of
19. Which of the following cannot be polarised ? the central maximum is I0. If the sources were incoherent,
(a) Radio waves (b) b rays the intensity at the same point will be
(c) Infrared rays (d) g rays (a) 4I0 (b) 1I0 (c) I0 (d) I0/2
30. In Young's double slit experiment with sodium vapour lamp
20. The phenomenon of interference is shown by
of wavelength 589 nm and the slits 0.589 mm apart, the half
(a) longitudinal mechanical waves only angular width of the central maximum is
(b) transverse mechanical waves only (a) sin–1 (0.01) (b) sin –1 (0.0001)
(c) non-mechanical transverse waves only –1
(c) sin (0.001) (d) sin–1 (0.1)
(d) All of the above 31. When light passing through rotating nicol is observed,
21. The transverse nature of light is shown by no change in intensity is seen. What inference can be
(a) interference of light drawn ?
(b) refraction of light (a) The incident light is unpolarized.
(c) polarization of light (b) The incident light is circularly polarized.
(d) dispersion of light (c) The incident light is unpolarized or circularly
22. If the intensities of the two interfering beams in Young’s polarized.
double-slit experiment are I1 and I2, then the contrast (d) The incident light is unpolarized or circularly polarized
between the maximum and minimum intensities is good or combination of both.
when 32. Huygen's concept of wavelets is useful in
(a) | I1 and I2 | is large (b) | I1 and I2 | is small (a) explaining polarisation
(c) either I1 or I2 is zero (d) I1 = I2 (b) determining focal length of the lenses
23. The idea of the quantum nature of light has emerged in an (c) determining chromatic aberration
attempt to explain (d) geometrical reconstruction of a wavefront
(a) interference 33. When the light is incident at the polarizing angle on the
(b) diffraction transparent medium, then the completely polarized light is
(b) polarization (a) refracted light
(d) radiation spectrum of a black body (b) reflected light
24.. The contrast in the fringes in an interference pattern (c) refracted and reflected light
depends on (d) neither reflected nor refracted light
(a) fringe width 34. In the phenomena of diffraction of light, when blue light is
(b) wavelength used in the experiment in spite of red light, then
(c) intensity ratio of the sources (a) fringes will become narrower
(d) distance between the slits (b) fringes will become broader
25. Polarisation of light establishes (c) no change in fringe width
(a) corpuscular theory of light (d) None of these
(b) quantum nature of light 35. The wavefronts of a light wave travelling in vacuum are
(c) transverse nature of light given by x + y + z = c. The angle made by the direction of
(d) all of the three propagation of light with the X-axis is
26. A slit of width a is illuminated by red light of wavelength (a) 0º (b) 45º
6500 Å. If the first minimum falls at q = 30°, the value of a is (c) 90º (d) cos –1 (1/ 3)
(a) 6.5 × 10–4 mm (b) 1.3 micron 36. In Fresnel’s biprism expt., a mica sheet of refractive index
(c) 3250 Å (d) 2.6 × 10–4 cm 1.5 and thickness 6 × 10–6 m is placed in the path of one of
27. Two beams of light of intensity I1 and I2 interfere to give interfering beams as a result of which the central fringe
an interference pattern. If the ratio of maximum intensity to gets shifted through 5 fringe widths. The wavelength of
that of minimum intensity is 25/9, then I1/I2 is light used is
(a) 5/3 (b) 4 (a) 6000 Å (b) 8000 Å
(c) 81/625 (d) 16 (c) 4000 Å (d) 2000 Å
Free eBooks on @neetquestionpaper2020

Wa ve Optics 905
37. Two sources of light of wavelengths 2500 Å and 3500 Å are 46. Two nicols are oriented with their principal planes making
used in Young’s double slit expt. simultaneously. Which an angle of 60º. Then the percentage of incident unpolarised
orders of fringes of two wavelength patterns coincide? light which passes through the system is
(a) 3rd order of 1st source and 5th of the 2nd (a) 100 (b) 50 (c) 12.5 (d) 37.5
(b) 7th order of 1st and 5th order of 2nd 47. A beam of unpolarised light passes through a tourmaline
(c) 5th order of 1st and 3rd order of 2nd crystal A and then through another such crystal B oriented
(d) 5th order of 1st and 7th order of 2nd so that the principal plane is parallel to A. The intensity of
38. A radar sends radiowaves of frequency v towards an emergent light is I0. Now B is rotated by 45º about the ray.
aeroplane moving with velocity va. A change Dn is observed The emergent light will have intensity
in the frequency of reflected waves which is higher than I0 / 2 (b) I 0 / 2
(a) (c) I0 2 (d) 2I 0
original frequency. The velocity of aeroplane is (va << c)
48. A rocket is receding away from earth with velocity 0.2 c.
c Dn 2 c Dn c Dn Dn
(a) (b) (c) (d) The rocket emits signal of frequency 4 × 107 Hz. The
n Dn 2n 2cn apparent frequency observed by the observer on earth will
39. In Young’s double slit experiment, we get 10 fringes in the be
field of view of monochromatic light of wavelength 4000Å. (a) 4 × 106 Hz (b) 3.2 × 107 Hz
If we use monochromatic light of wavelength 5000Å, then 6
(c) 3 × 10 Hz (d) 5 × 107 Hz
the number of fringes obtained in the same field of view is 49. The heavenly body is receding from earth, such that the
(a) 8 (b) 10 (c) 40 (d) 50 fractional change in l is 1, then its velocity is
40. With a monochromatic light, the fringe-width obtained in a
Young’s double slit experiment is 0.133 cm. The whole set- 3c c 2c
(a) c (b) (c) (d)
up is immersed in water of refractive index 1.33, then the 5 5 5
new fringe-width is 50. Two waves are propagating with same amplitude and nearly
(a) 0.133 cm (b) 0.1 cm same frequency, they result in
1.33 (a) beats (b) stationary wave
(c) 1.33 × 1.33 cm (d) cm
2 (c ) resonance (d) wave packet
41. A slit of width a is illuminated by white light. The first 51. When a compact disc is illuminated by small source of white
minimum for red light (l = 6500 Å) will fall at q = 30º when a light, coloured bands are observed. This is due to
will be (a) dispersion (b) diffraction
(a) 3250 Å (b) 6.5 × 10–4 cm (c) interference (d) reflection
(c) 1.3 micron (d) 2.6 × 10–4 cm 52. A nicol prism is based on the action of
42. The Fraunhoffer ‘diffraction’ pattern of a single slit is formed (a) refraction (b) double refraction
in the focal plane of a lens of focal length 1 m. The width of (c) dichroism (d) both (b) and (c)
slit is 0.3 mm. If third minimum is formed at a distance of 5 53. Ray optics is valid when characteristic dimensions are
mm from central maximum, then wavelength of light will be (a) of the same order as the wavelength of light
(a) 5000 Å (b) 2500 Å (b) much smaller than the wavelength of light
(c) 7500 Å (d) 8500 Å (c) much larger than the wavelength of light
43. Two points separated by a distance of 0.1 mm can just be (d) of the order of 1 mm
inspected in a microscope, when light of wavelength 600Å 54. Fluorescent tubes give more light than a filament bulb of
is used. If the light of wavelength 4800 Å is used, the limit same power because
of resolution will become (a) the tube contains gas at low temperature
(a) 0.80mm (b) 0.12 mm (c) 0.10 mm (d) 0.08 mm (b) ultraviolet light is converted into visible light by
44. Unpolarised light of intensity 32 W m–2 passes through fluorescence
three polarizers such that the transmission axis of the last (c) light is diffused through the walls of the tube
polarizer is crossed with that of the first. The intensity of (d) it produces more heat than bulb
final emerging light is 3 W m–2. The intensity of light 55. The deflection of light in a gravitational field was predicted
transmitted by first polarizer will be first by
(a) 32 W m–2 (b) 16 W m–2 (a) Einstein (b) Newton
(c) 8 W m –2 (d) 4 W m–2 (c) Max Planck (d) Maxwell
45. A parallel beam of monochromatic unpolarised light is 56. In young’s double-slit experiment, the intensity of light at a
incident on a transparent dielectric plate of refractive index
point on the screen where the path difference is l is I, l
1 being the wavelength of light used. The intensity at a point
. The reflected beam is completely polarised. Then the
3 l
where the path difference is will be
angle of incidence is 4
(a) 30º (b) 60º I I
(a) (b) (c) I (d) zero
(c) 45º (d) 75º 4 2
Free eBooks on @neetquestionpaper2020

906 Physi cs
57. Aperture of the human eye is 2 mm. Assuming the mean 64. Statement 1 : In YDSE, if a thin film is introduced in front of
wavelength of light to be 5000 Å, the angular resolution the upper slit, then the fringe pattern shifts in the downward
limit of the eye is nearly direction.
(a) 2 minute (b) 1 minute Statement 2 : In YDSE if the slit widths are unequal, the
(c) 0.5 minute (d) 1.5 minute minima will be completely dark.
58. If the polarizing angle of a piece of glass for green light is Imax
54.74°, then the angle of minimum deviation for an 65. Statement 1 : In YDSE, if I1 = 9I0 and I2 = 4I0 then = 25.
I min
equilateral prism made of same glass is
[Given, tan 54.74° = 1.414] Statement 2 : In YDSE Imax = ( I1 + I 2 ) 2 and
(a) 45° (b) 54.74°
(c) 60° (d) 30° I min = ( I1 - I 2 ) 2 .
59. In Young's double slit experiment, the fringes are displaced 66. Statement 1 : In Young’s double slit experiment if
by a distance x when a glass plate of refractive index 1.5 is wavelength of incident monochromatic light is just doubled,
introduced in the path of one of the beams. When this number of bright fringe on the screen will increase.
plate is replaced by another plate of the same thickness,
Statement 2: Maximum number of bright lunge on the screen
the shift of fringes is (3/2) x. The refractive index of the
is inversely proportional to the wavelength of light used
second plate is
67. Statement 1 : In YDSE number of bright fringe or dark fringe
(a) 1.75 (b) 1.50
can not be unlimited
(c) 1.25 (d) 1.00
Statement 2 : In YDSE path difference between the
60. A single slit Fraunhoffer diffraction pattern is formed with superposing waves can not be more than the distance
white light. For what wavelength of light the third between the slits.
secondary maximum in the diffraction pattern coincides
with the second secondary maximum in the pattern for red Directions for Qs. (68 to 76) : Read the following passage(s)
light of wavelength 6500 Å? carefully and answer the questions that follows:
(a) 4400 Å (b) 4100 Å PASSAGE 1
(c) 4642.8 Å (d) 9100 Å A student is performing Young’s double slit experiment. There are
61. When the angle of incidence is 60° on the surface of a two slits S1 and S2. Separation between them is d.
glass slab, it is found that the reflected ray is completely
polarised. The velocity of light in glass is Screen
S1
(a) 2 ´ 108 ms -1 (b) 3 ´ 108 ms -1 O d P
S2
(c) 2 ´ 108 ms -1 (d) 3 ´ 108 ms -1
D
62. In a two-slit experiment, with monochromatic light, fringes
are obtained on a screen placed at some distance from the There is large screen at a distance D(D >>d) from the slits. The
slits. If the screen is moved by 5 × 10–2 m towards the slits, set-up is shown in the following figure. A parallel beam of light is
the change in fringe width is 10–3 m. Then the wavelength incident upon it. A monochromatic light of wavelength l is used.
of light used is (given that distance between the slits is The initial phase difference between the two slits which behaves
0.03 mm) as two coherent sources of light is zero. The intensity of light
(a) 4500 Å (b) 5000 Å waves on the screen coming out of S1 and S2 are same and is I0.
(c) 5500 Å (d) 6000 Å. In this situation, the principal maximum is formed at the point P.
Directions for Qs. (63 to 67) : Each question contains At the point on screen where principal maximum is formed, phase
STATEMENT-1 and STATEMENT-2. Choose the correct answer difference between two interfering waves will be zero.
(ONLY ONE option is correct ) from the following- 68. Initially the distance of third minima from principal maxima
(a) Statement -1 is false, Statement-2 is true will be
(b) Statement -1 is true, Statement-2 is true; Statement -2 is a 3 lD 3lD 5 lD 5 lD
correct explanation for Statement-1 (a) (b) (c) (d)
2 d d 4 d 2 d
(c) Statement -1 is true, Statement-2 is true; Statement -2 is not 69. A glass slab of thickness t and refractive index m is
a correct explanation for Statement-1 introduced before S2, now P does not remain the point of
(d) Statement -1 is true, Statement-2 is false principal maximum. Suppose principal maximum forms at a
63. Statement 1 : Diffraction takes place for all types of waves point P’ on screen. Then PP’ is equal to
mechanical or non-mechanical, transverse or longitudinal. tD( m - 1) tD( m - 1)
Statement 2 : Diffraction's effect are perceptible only if (a) (b)
d 2d
wavelength of wave is comparable to dimensions of
diffracting device. D(m - 1) D(m - 1)
(c) (d)
t d
Free eBooks on @neetquestionpaper2020

Wa ve Optics 907
70. Use the statement given in previous question. Now parallel 71. At the angle of polarisation, the angle of inclination between
beam is incident at an angle a w.r.t. line OP, such that the reflected and refracted rays is
principal maximum again comes at point P. The figure is (a) p/8 (b) p/6
shown. The value of a is (c) p/4 (d) p/2
t(m - 1) r r
(a) sin -1 72. If in an unpolarised light E = 2iˆ + 3jˆ and H = 3iˆ - 2ˆj , then
d
the direction of propagation is given by
-1 t( m - 1)
S1
(b) cos aO r
d a P (a) -13k (b) -10ˆj
S t
-1 t(m - 1)D 2 m (c) -6iˆ + 6kˆ (d) 5iˆ - 4kˆ
(c) sin
d 73. An analyser examines two adjacent plane-polarised beams
-1 tD A and B whose planes of vibration are mutually
(d) sin
d perpendicular. In one position of the analyser, beam B shows
PASSAGE 2 zero intensity. From this position, a rotation of 30° shows
On the basis of the phenomenon of polarisation, it was established the two beams as matched in intensity. The intensity ratio
that light is a form of transverse wave motion. IA/IB of the two beams is
(a) 3 (b) 1/3
(c) 1/ 2 (d) 5
PASSAGE 3
A point source is emitting light of wavelength 6000 Å is placed at
Since light is an electromagnetic wave, it consists of electric vector a very small height h above a flat reflecting surface MN as shown
r r in the figure.
E and magnetic vector H varying periodically in phase, at right p Screen
angles to each other and also perpendicular to the direction of
propagation. In ordinary light, the electric vector vibrates in all
planes with equal probability at right angles to the direction of
propagation. Hence the ordinary light also called unpolarised D
light can be represented by a star as shown. S
Hence unpolarised light is symmetrical about the direction of h
propagation. By the phenomenon of reflection, light can be ////////////////////////////////////////////
M N
polarised. The polarised light has vibrations confined only to a
single line in a plane perpendicular to the direction of propagation The intensity of the reflected light is 36% of the incident intensity.
and is called plane polarised. Interference fringes are observed on a screen placed parallel to
the reflecting surface at a very large distance D from it.
To polarise light by reflection we make use of Brewster’s law
which states that the tangent of the angle of polarisation (p) is 74. The shape of the interference fringes on the screen is
numerically equal to the refractive index m of the medium. The (a) circle (b) ellipse
device that polarises unpolarised light is called the polariser and (c) parabola (d) straight line
the device that analyses the polarised light is called the analyser. 75. If the intensity at p be maximum, then the minimum distance
The amplitude of a plane polarised light incident on the analyser through which the reflecting surface MN should be
can be resolved into two components, one parallel to the plane of displaced so that at P again becomes maximum?
transmission of the analyser and the other perpendicular to it. (a) 3 × 10-10 m (b) 6 × 10-10 m
(c) 1.5 × 10 m -10 (d) 12 × 10-10 m
The parallel component, namely, (a cosq) is transmitted through
the analyser. 76. Ratio of maximum to minimum intensities at P is
(a) 2 : 1 (b) 4 : 1
(c) 8 : 1 (d) 16 : 1
Free eBooks on @neetquestionpaper2020

908 Physi cs

20. (b) At the centre, all colours meet in phase, hence central
Exercise 25.1
fringe is white.
21. (a) The centre of the pattern is dark on account of
1. (a) According to Cauchy’s formula destructive interference.
B lD
m =A+ + ... Therefore, as l increases, m decreases.
l2 22. (a) b =
d
Curve A is correct.
2. (c) When the film is thin, t ® 0, path diff. = l/2. Therefore, \ b increases on decreasing d, separation of slits.
in reflected light, the film appears black. 23. (b) Monochromatic light is light of single colour/
lD wavelength.
3. (b) As l r » 8000A º and l v » 4000 Å and b = i.e. 24. (b) Phase reversal occurs i.e. phase change = p takes place
d
on reflection, because glass is much denser than water.
b µ l , therefore b red is roughly double of b violet .
4. (a) According to wave theory, intensity of light is directly 26. (a) Fringe visibility (V) is given by
proportional to square of amplitude.
Ιmax - Ι min
5. (b) As optical path SB of lower slit is increased, therefore, V=
fringe pattern shifts somewhat downwards. Ιmax + Ι min .

lD 27. (c) Laser light is coherent, because it consists of coordinated


6. (a) As b = \ b µ l. (parallel) waves of exactly same wavelength (i.e
d
monochromatic wave).
As l for violet is least, therefore, fringe nearest to central
achromatic fringe will be violet. 29. (c) Interference at thin films causes colouring of soap
bubble.
7. (a) Increase in length of optical path
= m t – t = (m – 1) t. d
30. (c) D = x , where D is path difference between two
8. (c) Highly coherent sources are produced using laser. D
waves.
9. (d) A prism cannot produce coherent sources.
10. (a) Coherence is a measure of capability of producing 2p
\ phase difference = f = D.
interference by waves. l
11. (d) Interference pattern will be invisible, because red and Let a = amplitude at the screen due to each slit.
green are complimentary colours. \ I0 = k (2a)2 = 4ka2, where k is a constant.
12. (b) Light travels along straight lines is the basis of Newton’s For phase difference f,
corpuscular theory. amplitude = A = 2acos(f/2).
13. (b) Huyghen’s principle gives us a geometrical method of
[Since, a 2 = a 12 + a 22 + 2a1a 2 cos f , here a1 = a2]
tracing a wavefront.
14. (b) For coherent sources l is same and phase is also same Intensity I,
or phase diff. is constant. æ px ö
Ι = kA 2 = k(4a 2 ) cos 2 (f / 2) = Ι0 cos 2 ç D
15. (a) There will be general illumination as super imposing è b ø÷
waves do not have constant phase difference.
æ p xd ö æ px ö
16. (a) Bright fringes are yellow and dark fringes are black. = Ι 0 cos 2 ç . ÷ = Ι0 cos 2 ç ÷
17. (b) Wavelength/frequency must be same and phase èl Dø è bø
difference must be constant for producing sustained 31. (a) The inverse square law of intensity is valid only for a
interference. point source.
18. (b) For minima, phase diff. = odd integral multiple of 32. (a) The fringe width b in Young’s double slit experiment is
p = (2 n – 1) p. defined as
19. (a) nl = (m - 1)t ; lD
b= ®b µ l (for same D & d)
d
(m - 1) t (1.5 - 1) ´ 6 ´ 10 -6 now the wavelength of light decreases, as it go from
\ l= = = 6000Å
n 5 air to water. Hence fringe width b also decreases in water.
Free eBooks on @neetquestionpaper2020

Wa ve Optics 909
33. (a,b) For sustained interference 4. The fringes become narrower.
(1) Two source should be coherent i.e they should have 15. 0.3 mm
constant phase difference between them. 19. (d) 20. (d) 21. (c) 22. (b)
(2) Both source should give light of same frequency. 23. (d) 24. (a) 25. (d) 26. (d)
(3) If interfering waves are polarised, they must be in same
state of polarisation. Exercise 2 : PAST Competition MCQs

Exercise 25.2 1. (c) By covering aperture, focal length does not change.
1
But intensity is reduced by times, as aperture
1. (c) Because both source & screen are effectively at infinite 4
d
distance from the diffractive device diameter is covered.
2
2. (d) Diffraction on a single slit is equivalent to interference I 3I
of light from infinite number of coherent sources \ I' = I - =
4 4
contained in the slit. 3I
\ New focal length = f and intensity = .
Exercise 25.3 4
nlD
2. (a) Q y =
1. (c) Light waves can be polarized because they are d
transverse in nature. \ n1 l1 = n2l2
2. (c) At Brewster’s angle, only the reflected light is plane Þ n1 × 12000 × 10–10 = n2 × 10000 × 10–10
polarised, but transmitted light is partially polarised. or, n (12000 × 10–10) = (n + 1) (10000 × 10–10)
3. (b) Light transmitted by nicol prism is plane polarised. Þn=5
4. (b) On polarisation by reflection, the reflected and refracted
waves are at 90º to each other. (Q l 1 = 12000 ´ 10 -10 m; l 2 = 10000 ´ 10-10 m )
5. (a)
6. (c) Such substances rotate the plane of polarised light.
7. (a) Polaroid glass polarises light reducing the light intensity
to half its original value.
8. (b) Angle between plane of vibration and plane of
polarisation is 90º.
9. (a) Plane of vibration is ^ r to direction of propagation and
also ^ r to plane of polarisation. Therefore, angle nl1D
between plane of polarisation and direction of Hence, ycommon =
d
propagation is 0º.
10. (a) Along the optic axis, m0 = me.
=
(
5 12000 ´ 10-10 ´ 2 ) (Q d = 2 mm and D = 2m )
Exercise 25.4 2 ´ 10-3
= 5 × 12 × 10–4 m
1. (b) When source is receding away, apparent wavelength = 60 × 10–4 m
increases. Displacement is towards red region. = 6 × 10–3m = 6 mm
2. (a) Velocity of light (in vacuum) is an absolute constant
and does not depend upon the motion of the observer.
3. (b) Shifting towards red end means wavelength is increasing. Y
Therefore, milkyway is receding away from earth. S1
4. (b) Limit of resolution of eye is roughly 1 minute (angle). 3. (b) d q
5. (a) To be invisible in vacuum, m of medium must be equal to D
S2
m of vacuum, which is 1.
6. (c) This ability refers to resolving power of the instrument.
Y nlD é Dl ù
Exercise 1 : NCERT Based Questions Angular width, q = = êQ Y=
D dD ë d úû
1. A wavefront is defined as a surface of constant phase.
l
l so, q = , v ­l ¯ q ¯ [For central maxima n = 1]
2. Path difference (x) = × phase difference (f). d
2p Hence, with increase in speed of electrons angular
3. The resolving power of a microscope is given by 2n sin b . width of central maximum decreases.
1.22l 4. (d)
Free eBooks on @neetquestionpaper2020

910 Physi cs
5. (d) The angle of incidence for total polarization is given NOTE : This formula is applicable when I1= I2.
-1
by tan q = n Þ q = tan n Here f f = p 3
6. (b) For constructive interference d sin q = nl 2
I p æ 3ö 3
n \ = cos 2 = ç =
given d = 2l Þ sin q = I0 6 è 2 ÷ø 4
2
12. (b) In the medium, the refractive index will decrease from
n = 0,1, - 1,2, - 2 hence five maxima are possible
the axis towards the periphery of the beam.
y l Therefore, the beam will move as one move from the
7. (b) ³ 1.22
D d axis to the periphery and hence the beam will converge.
yd 10 - 3 ´ 3 ´ 10 - 3 = 30
Þ D£ = » 5m
(1.22) l (1.22) ´ 5 ´ 10 - 7 6.1
\ D max = 5m
Decreasing m
8. (d) The shape of interference fringes formed on a screen Axis
in case of a monochromatic source is a straight line.
Remember for double hole experiment a hyperbola is
generated.
2
æ sin f ö p æ ay ö
9. (b) I = I 0 çç ÷÷ and path difference f = ç ÷ 13. (d) Initially the parallel beam is cylindrical . Therefore, the
è f ø l èDø
wavefront will be planar.
For principal maxima y = 0
14. (a) The speed of light (c) in a medium of refractive index
\ f =0 (m) is given by
Hence intensity remains same. c
m = 0 , where c0 is the speed of light in vacuum
10. (b) I = I 0 cos 2 q c
c0
I0 \c=
Intensity of polarized light = m
2 c0
=
Þ Intensity of untransmitted light m0 + m2 ( I )
I0 I0 As I is decreasing with increasing radius, it is maximum
= I0 - = on the axis of the beam. Therefore, c is minimum on the
2 2
axis of the beam.
11. (a) For path difference of l, the phase difference is 2p
15. (d) Let a1 = a, I1 = a12 = a2
l
For path difference of , the phase difference is a2 = 2a, I2 = a22 = 4a2
6 I2 = 4I1
2p ´ l / 6 l Ir = a12 + a22 + 2a1a2 cos f
=
l 3
= I1 + I2 + 2 I1I2 cos f
p
\ Intensity I = I1 + I 2 + 2 I1 I 2 cos
3 Ir = I1 + 4I1 + 2 4I12 cos f
\ I = I1 + I2 + I1 I2 Þ Ir = 5I1 + 4I1 cos f … (1)
when I1= I2 = I' (say) then I = 3I'
Now, Imax = (a1 + a 2 ) 2 = (a + 2a)2 = 9a 2
( )
2
Imax = I1 + I2
Imax
Imax = 9I1 Þ I1 =
=( ) = (2 I')
2 2
I' + I' = 4I' 9
Substituting in equation (1)
T 3 5Imax 4I max
\ = Ir = + cos f
I max 4 9 9
ALTERNATIVELY : The intensity of light at any point I
of the screen where the phase different due to light Ir = max [ 5 + 4cos f]
9
coming from the two slits is f is given by f
I é ù
Ir = max ê5 + 8cos 2 - 4ú
æ fö 9 ë 2 û
I = Io cos2 ç ÷ where to is the maximum intensity..
è 2ø Imax é fù
2
Ir = êë1 + 8cos
9 2 úû
Free eBooks on @neetquestionpaper2020

Wa ve Optics 911
45° B In YDSE, I1 = I2 = I and Imax = 4 I
16. (c) I0 (I0/2) We are concentrating at a point where the intensity
(unpolarised) IR is one fourth of the max intensity.
\ I = I + I +2 I cos f
A 1 2p
Relation between intensities Þ - cos f Þ f =
2 3
æI ö [P1 note that we take the least value of the angle as
IR = ç 0 ÷ cos 2 (45°) the point is in central maxima]
è2ø
For a phase difference of 2p, the path difference
I 1 I is l
= 0´ = 0
2 2 4 2p
17. (d) It will be concentric circles. For a phase difference of , the path difference is
3
18. (d) The distance between the first dark fringe on either
l 2p l
side of the central bright fringe ´ =
= width of central maximum 2p 3 3
But the path difference (in terms of P and Q) is d sin q
2Dl 2 ´ 2 ´ 600 ´ 10-9 as shown in fig.
= =
a 10-3 \ d sin q =
l
Þ sin q =
l
= 2.4 × 10–3 m = 2.4 mm 3 3d
19. (a) æ l ö
Þ q = sin -1 ç ÷
20. (d) At the area of total darkness minima will occur for è 3d ø
both the wavelength. lD
(2n + 1) (2 m + 1) 23. (d) We know that b =
\ l1 = l2 d
2 2 Now, lR > lG > lB
Þ (2n + 1)l1 = 2(m + 1)l 2 \ bR > bG > bB
24. (b) The intensity I is given as
( 2n + 1) 560 7
or (2m + 1) = 400 = 5 f
I = Io cos 2 where Io is the peak intensity
2
or 10 n = 14 m + 2
Here I = o , \ o = I o cos 2 f \ f = p (2n + 1)
I I
by inspection for m = 2, n = 3
2 2 2 2
for m =7, n = 10 For a phase difference of 2p the path difference is l
the distance between them will be the distance p
between such points \ For a phase difference of (2n+1) the path
2
Dl1 ì ( 2n 2 + 1) – ( 2n1 – 1) ü l
i.e., Ds = difference is (2n +1) . option (b) is correct.
í ý 4
d î 2 þ
put n2 = 10, n1 = 3 Exercise 3 : Conceptual & Applied MCQs
on solving we get, Ds = 28 mm
1. (d) It is a one of Fraunhoffer diffraction from single slit.
21. (c) Frequency does not change with change of medium. so for bright fringe where a is the width of slit.
22. (c) Let P be the point on the central maxima whose
l
intensity is one fourth of the maximum intensity. a sin q = (2 n + 1)
2
For interference we know that
2 a sin q 2 ´1.2 ´10 -5 ´ 0.0906
I = I1 + I 2 + 2 I1I 2 cos f l= =
2 n +1 2 ´1 + 1
Where I is the intensity at P and I1, I2 are the intensity
= 7248 ´10 -10 m = 7248Å.
of light originating from A and B respectively and f is
the phase difference at P. 2. (c) Q i p = f, therefore, angle between reflected and
refracted rays is 90º.
D 5 ´ 6.5 ´ 10 -7 ´ 1
3. (c) x5 = n l = = 32.5 ´ 10 -4 m
P d 10 -3
1 Dl 5 ´ 6.5 ´ 10 -7 ´ 1
A x 3 = ( 2 n - 1) =
q 2 d 2 ´ 10 -3
d
s in
q = 16.25 ´ 10 -4 m
B d
x 5 - x 3 = 16.25 ´ 10-4 m = 1.625 mm.
Free eBooks on @neetquestionpaper2020

912 Physi cs
4. (c) As R2 = a2 + b2 + 2 ab cos f 15. (d) Doppler effect in light explains the phenomenon of
receding of stars and approaching of star by red shift
\ I max = I1 + I 2 + 2 I1 I 2 cos 0º
and blue shift respectively.
= I1 + I 2 + 2 I1 I 2 16. (a) For single slit diffraction pattern e sin q = l Angular
2p width, e = slit width
5. (b) Phase diff. = x
l ælö
\ q = sin -1 ç ÷
-6 èeø
2 p ´ 2.1´10
Path difference = = 7.692 p radian. It is independent of D, distance between screen and
5460 ´ 10 -10
slit.
l¢ D¢ 3 l 3 D lD 17. (d) Photoelectric effect does not support the wave nature
6. (c) b = = = 27 .
d¢ d/3 d of light.
No. of fringes = 1 / 3 = d . 18. (b)
b 81l D 19. (b) Longitudinal waves cannot be polarised.
7. (b) The position of n th dark fringe. So position of first 20. (d)
21. (c) 22. (d) 23. (d)
dark fringe in x1 = lD / 2d . 24. (c) 25. (c)
d = 20 cm, D = 0.1mm, l = 5460 Å, x1 = 0.16 26. (b) According to principle of diffraction, a sin q = nl
8. (c) When the arrangement is dipped in water; where, n = order of secondary minimum
x 3 or, a sin 30° = 1 × (6500 × 10–-10)
b¢ =b / m = = x = 0.75x or, a = 1.3 × 10–6 m, or, a = 1.3 micron.
4/3 4 2
9. (a) The angular width of central maxi. is Imax 25 æa +a ö 25
27. (d) = or ç 1 2 ÷ =
l 2 ´ 6328 ´ 10 -10 Imin 9 è a1 - a 2 ø 9
2q = 2 = radian . where a denotes amplitude.
a 2 ´ 10 - 4
180 a1 + a 2 5
= 6328 ´ 10-6 ´ degree = 0.36º = or 5a1 - 5a 2 = 3a1 + 3a 2
p a1 - a 2 3
or, 5a1 – 5a2 = 3a1 + 3a2 or 2a1 = 8a2
1 1
10. (b) Wave number, n = = = 1.66 ´10 6 m -1. 2
I1
l 6 ´ 10- 7 a1 æa ö
or, = 4 or ç 1 ÷ = 16 = .
11. (a) Width of central maximum a2 è a2 ø I2
2 l D 2 ´ 6250 ´ 10 -10 ´ 0.5 28. (b)
= = 29. (d) For two coherent sources, I1 = I2
a 2 ´ 10 -4
( )
2
= 3125 ´ 10-6 m = 312.5 ´ 10-3 cm. Imax = (A1 + A2)2 = I1 + I 2
This is given as I0 for maximum and zero for minimum.
If there are two noncoherent sources, there will be no
y2sinq = 2l
a maximum and minimum intensities. Instead of all the
y1sinq = l/a intensity I0 at maximum and zero for minimum, it will
a q o sinq = 0
be just I0/2.
Intensity y1sinq = – l/a
D y2sinq = – 2l l 589 ´ 10-9 1
a 30. (c) sin q = = = 10-3 = = 0.0001
d 0.589 ´ 10 -3 1000
Screen position of various minima for Fraunhoffer 31. (c)
diffraction pattern of a single slit of width a. 32. (d)
12. (d) As reflected and refracted rays are perpendicular to 33. (b) When the light is incident at the polarising angle on
each other, therefore, ip = i = 60º, where ip is called the transparent medium, the reflected light is
angle of polarisation. completely polarised.
34. (a)
m = tan ip = tan 60º = 3 = 1.732. r
35. (d) Let any R its components are
Dl (6586 - 6263) r r r r
13. (b) u= ´c = ´ 3 ´ 108 R = Rx + Ry + Rz
l 6563 r r r r
with | R |= R = | R |= R = Rx 2 + Ry 2 + Rz–
= 1.05 ´ 106 m / s.
Rx Ry Rz
u 10 5 & cosq x = , cosq y , cosq z =
14. (b) Dl = ´l = ´ 5700 = 1.9 Å . R R R
c 3 ´ 108
Free eBooks on @neetquestionpaper2020

Wa ve Optics 913
there cosqx, cosqy and cosqz one called direction 42. (a) a sin q = nl
cosines.
r ax
Hence x + y + z = c (= R ) = 3l
f
2 2 2
So, magnitude of c = I + I + I = 3 (since q is very small so sin q » tan q » q = x / f )
a x 0.3 ´10 -3 ´ 5 ´10 -3
1 or l = =
and cos q x = 3f 3 ´1
3
36. (a) Where n is equivalent number of fringe by which the = 5 ´ 10 -7 m = 5000 Å.
centre fringe is shifted due to mica sheet 43. (d) Limit of resolution L.R µ l
-6 l¢ 4800
(m - 1) t (1.5 - 1) 6 ´10 \ L.R ¢ = L.R ´ = 0.1 ´ = 0.08 mm.
l= =
n 5 l 6000
\ Resolution improves when light of lower wavelength
= 6 ´ 10 -7 m = 6000 Å is used.
37. (b) Let nth fringe of 2500 Å coincide with (n – 2)th fringe 44. (b) Intensity of polarised light transmitted from 1st
of 3500Å. polariser,
\ 3500 (n – 2) = 2500 × n I 1 = I0 cos2q
1000 n = 7000, n = 7 1
\ 7th order fringe of 1st source will coincide with 5th but (cos2q)av =
2
order fringe of 2nd source.
1 32
38. (c) In Radar, the source & receiver are together, the So I1 = I0 = = 16Wm -2
receiver being turned for frequencies other than the 2 2
45. (a) When angle of incidence i is equal to angle of
source or radar frequency.
polarisation i.e, then reflected light is completely plane-
To measure the speed of helicopter
polarised whose vibration is perpendicular to plane of
(i) The moving object of speed va receive a frequency incidence.
æ v ö 46. (c) Suppose intensity of unpolarised light = 100.
n ' = nç 1 + a ÷
è c ø \ Intensity of polarised light from first nicol prism
(ii) The object which receives n’ frequency now acts as a Ι 1
= 0 = ´ 100 = 50
moving source. The detector observes a frequency n0 2 2
æ ö æ v ö According to law of Malus,
ç ÷ ç1+ a ÷
1 c 2
n 0 = n' ç ÷ =ç ÷ æ1ö
Ι = Ι0 cos 2 q = 50 (cos 60º )2 = 50 ´ ç ÷ = 12.5
ç va ÷ ç va ÷
ç1- ÷ ç1- ÷ è2ø
è c ø è c ø 47. (a) According to law of Malus
v
Þ (n 0 - n ) = Dn = (n 0 + n ) a 2
c æ 1 ö Ι
Ι = Ι 0 cos 2 q = Ι 0 (cos 45º ) 2 = Ι 0 çç ÷÷ = 0
Dnc æ \n0 »n ö è 2ø 2
or v a = ç Þn +n»2n ÷ u
2n è 0 ø 48. (b) Dn = ´ n =
0.2 c
´ 4 ´ 10 7 = 0.8 ´ 10 7 Hz
39. (a) As b µ l c c
7
5 n ¢ = n - Dn = 4 ´ 10 7 - 0.8 ´ 10 7 = 3.2 ´10 Hz.
\ fringe width becomes times,
4 49. (a) When planet or stars are receding away from earth. If
4 f is frequency of vibration.
No, of fringes = ´ 10 = 8
5 c
Then , f =
b 0.133 l
40. (b) b¢ = = = 0.1 cm If v = velocity of body moving away
m 1.33
41. (c) The position of n th dark fringe in Fraunhoffer l' = apparent wavelength to an observer on the earth
Diffraction from a single slit is
l' =
(c + v )
a sin q = n l (c and v are in opposite to each other)
f
n l 1´ 6.5 ´ 10 -7 æc+vö æ vö
a= = , (for first min. n = 1) =ç ÷l l ' = ç1 + ÷ l
sin q sin 30º è c ø è cø
6.5 ´ 10 -7 l '- l v
= = 13 ´ 10 -7 m = 1.3 m m. =
1/ 2 l c
Free eBooks on @neetquestionpaper2020

914 Physi cs
Fractional change in wavelength æ A + dö æ 60° + d ö
sin ç sin ç
v è 2 ÷ø è 2 ÷ø
\ 1= Þ v =c \ m= =
c sin ( A / 2) sin ( 60° / 2)
50. (b) When two waves of same amplitude and nearly
1.141 ´ 1 æ 60° + d ö é
same frequency in opposite direction are propagated. or, = sin ç Q1.414 = 2 ùû
They produce stationary waves. For beats, there must 2 è 2 ÷ø ë
be a difference in frequency. 2 æ 60° + d ö 1 æ 60° + d ö
or, = sin ç or = sin ç
51. (b) The line rulings, each of 0.5 m m width,on a compact 2 è 2 ÷ø 2 è 2 ÷ø
disc function as a diffraction grating. When a small
æ 60° + d ö æ 60° + d ö
source of light illuminates a disc, diffracted light forms or, sin 45° = sin çè ÷ø or 45° = çè ÷
coloured ‘lanes’ that are the composite of the 2 2 ø
diffraction patterns from the ruling. 59. (a)
52. (d) When a ray of light enters nicol prism, it splits into
x=
( 2n + 1) l D
two plane polarised light in mutually perpendicular 60. (c)
2a
direction. One of this light undergoes total reflection
( 4 + 1) D ´ 6500Å
and absorption whereas other comes out as a plane For red light, x =
polarised light. 2a
53. (c) ( 6 + 1) D
For other light, x = ´ lÅ
54. (b) The fluorescent material present in the tube converts 2a
u.v. rays into visible light. x is same for each.
55. (b) Newton first predicted deflection of light by 5
gravitational field. \ 5 ´ 6500 = 7 ´ l Þ l = ´ 6500 = 4642.8 Å.
7
56. (b) For path difference l, phase 61. (b) aµg = tan qP where qP = polarising angle.
æ 2p 2p ö or, aµg = tan 60°
difference = 2p ç Q = x = .l = 2p ÷
è l l ø c
or, = 3
Þ I = I0 + I0 + 2I0 cos 2p vg
Þ I = 4I0 (\ cos 2p = 1)
c 3 ´ 108
l p or, vg = = = 3 ´ 108 ms -1
For x = , phase difference = 3 3
4 2
lD
p 62. (d) Fringe width b =
\ I' = I1 + I 2 + 2 I1 I 2 cos d
2 Where D is the distance between slit and the screen,
I I d is the distance between two slits, l is the wavelength
If I1 = I2 = I0 then I ' = 2I0 = 2. =
4 2 of light.
57. (b) If the angular limit of resolution of human eye is R lDD
then \ Db =
d
1.22l 1.22 ´ 5 ´ 10 -7
R= = rad Dbd 10 -3 ´ 0.03 ´ 10-3 10-3 ´ 3 ´ 10 -5
a 2 ´ 10 -3 or, l = = =
DD 5 ´ 10-2 5 ´ 10 -2
1.22 ´ 5 ´ 10 -7 180 –7
= 6 × 10 m = 6000 Å.
= ´ ´ 60 mi nute = 1 minute
-3 p 63. (c) 64. (a) 65. (b)
2 ´ 10
58. (d) By principle of polarization, µ = tanqp 66. (a) 67. (b) 68. (a)
or µ = tan 54.74° or µ = 1.414 69. (b) 70. (a) 71. (d)
For an equilateral prism, ÐA = 60° 72. (a) 73. (b) 74. (a)
75. (a) 76. (d)
Free eBooks on @neetquestionpaper2020

26
Dual Nature of
Radiation and Matter
CATHODE RAYS Millikan’s oil drop method for e/ m :
Discharge Tube Experiments In fig.(a) we consider a single drop of mass m carrying a negative
charge –q in the absence of electric field. Then
When a very strong potential difference is applied across the two
Fvis cous 1 = 6 r 1 –qE
electrodes in a discharge tube and the pressure of the air is lowered
gradually, then a stage is reached at which the current begins to
flow through the air with cracking noise. The potential at which
this happens is called sparking potential. n1 n2 E

mg
Aston’s Cathode glow
dark space mg
Faraday’s dark space
– + Fviscous 2 = 6ph rv1
(a) (b)
Fviscous 1 = mg [from Stoke’s law Fviscous = 6phrv]
Negative glow Positive striation or 6p h rv1 = mg …(1)
· As pressure is lowered to 0.1 m.m. Hg – cathode glow, where h is coefficient of viscocity of air, r is radius of drop and v1
Crooke’s dark space, negative glow, Faraday’s dark space is the terminal velocity of drop.
and striations are observed. In fig. (b) we consider a single drop of mass m, radius r carrying a
· At a pressure 0.01 m.m. Hg entire tube is dark (Crooke’s dark negative charge –q in the presence of electric field acting
space) except the glass wall behind anode. Colour is downward. Then by free body diagram (fig. (b)), we get
yellowish-green for soda glass and greyish-blue for lead (-q )E = mg + Fviscous 2 = mg + 6phrv2 …(2)
glass. where v2 is the terminal speed in this case. Then from eqn (1), we
· The luminous streaks travelling from cathode to anode, have.
below the pressure 0.01 m.m. Hg, are called cathode rays. 6 phr ( v1 + v 2 )
( -q ) = …(3)
Properties of Cathode Ray : E
(i) Emitted perpendicularly to cathode, (ii) Travel in straight lines and radius of drop from equation (1)
(iii) Carry energy (iv) Possesses momentum (v) Deflected by 4 3 9hv1
6 phrv1 = mg = pr r g or r = …(4)
electric and magnetic fields (vi) Excite fluroescence (vii) Ionise 3 2r g
gas (viii) Produce highly penetrating secondary radiation when where r is density of drop.
suddenly stopped (ix) Effect photographic plate etc. Millikan repeated these measurements on thousands of drops
J.J. Thomson’s e/m value of electron : and he found that the charge q calculated for each drop was some
integral multiple of an elementary charge e. (e = 1.6 × 10–19C).
E2 æeö
e/m = = 1.76 ´ 1011 C kg –1 . This value of ç m ÷ is for Hence, q = ne, n = 0, ±1, ±2 ...(5)
2VB 2 è ø
electron. This experiment gives the evidence of quantisation of charge.
Free eBooks on @neetquestionpaper2020

916 Ph ysi cs
EMISSION OF ELECTRON negative then, photoelectrons are repelled by negative cathode
Electrons from the metal surface are emitted by anyone of the and only those electrons reaches anode, who have energy equal
following physical processes : to or greater than eV. But if V is equal to V0, called stopping
(i) Thermionic emission : The emission of electrons by suitably potential (i.e., cut off. potential), no electrons will reach the anode
heating the metal surface. i.e., Maximum kinetic energy of electron = eV0
(ii) Field emission : The emission of electrons by applying very or Kmax = eV0 ...(1)
strong field of the order of 108 Vm–1 to a metal. where e is charge of electron (e = 1.6 × 10–19 coulomb).
(iii) Photo-electric emission : The emission of electrons when But some features of photoelectric effect cannot be explained by
light of suitable frequency illuminates metal surface. classical physics & the wave theory of light.
PHOTOELECTRIC EFFECT (EINSTEIN’S PHOTOELECTRIC (1) No photoelectrons are emitted, if the frequency of incident
EQUATION light is less than some cut-off frequency (i.e., threshold
frequency) n0. It is inconsistent with the wave theory of
In 19th century, experiments showed that when light is incident
light, which predicts that photoelectric effect occurs at any
on certain metallic surfaces, electrons are emitted from the
frequency provided intensity of incident light is sufficiently
surfaces. This phenomenon is known as the photoelectric effect
high.
& emitted electrons are called photoelectrons. The first
discovery of this phenomenon was made by Hertz. (2) The maximum kinetic energy of the photoelectrons is
independent of light intensiy, but increases with increasing
e C the frequency of incident light.
od Light
th (3) Electrons are emitted from surface almost instantaneously
Ca (less than 10–9 sec after the surface illumination), even at
C
Anode A
low intensity of incident light(classicaly we assume that the
electrons would require some time to absorb the incident
G light before they acquire enough kinetic energy to escape
V from metal).
These above points were explained by Einstein in 1905 by treating
the light as stream of particles.
Taking Max Planck assumptions, Einstein postulated that a beam
of light consists of small packets of energy called photons or
quanta. The energy E of a photon is equal to a constant h times
its frequency n
Battery i.e., hc ...(2)
E = hn =
When light strikes the cathode C (metallic surface), photo l
electrons are ejected. Electrons are collected at anode A,
where h is a universal constant called Planck’s constant &
constituting a current in the circuit. (Photoelectric effect)
numerical value of h = 6.62607 × 10–34 J.s
Fig. shows, when light strikes the cathode C, electrons are emitted
When a photon arrives at surface, it is absorbed by an electron.
& they are collected on anode A due to potential difference provided
This energy transfer is an All-or-None process, in contrast to
by battery and constitutes the current in the circuit (observed by
continuous transfer of energy in classical theory; the electrons
Galvanometer G.)
get all photon’s energy or none at all. If this energy is greater than
A plot of photoelecric current versus the potential difference V between the work function (f) of the metal (f is the minimum energy
cathode & anode is shown in fig below. required to remove the electron from metal surface), the electron
may escape from the surface. Greater intensity at a particular
frequency means greater number of photons per second absorbed
& consequently greater number of electrons emitted per second
& so greater current.
1 2
To obtain maximum kinetic energy K max = mvmax
2
for an emitted electron, applying law of conservation of energy.
According to it
1 2
Photoelectric current versus voltage for two light intensities. K max = mvmax = hn - f [f = hn0 ]
2
At a voltage less than –V0 the current is zero.
It is clear from fig. that photoelectric current increases as we K max = h( n - n0 ) ...(3)
increase the intensity of light & obtain saturation value at larger
value of potential difference V between cathode & anode. If V is or eV0 = Kmax = h( n - n0 ) ...(4)
Free eBooks on @neetquestionpaper2020

Dual Nature of Radiation and Matter 917

1 2 æ1 1 ö 3. Maximum kinetic energy versus potential graph.


or, mvmax = h (v - v0 ) = hc ç - ÷ = evs
2 è l l0 ø
n01 for metal 1 n02for metal 2
This is the Einstein’s photoelectric equation. 1 mv2
max
where V0 = cut-off potential 2
nmax = maximum velocity obtained by photoelectrons
n = frequency of incident light i.e., photon
n0 = cut off frequency or threshold frequency.
n01 n02 n
n0 is different for different metallic surfaces. For most metals the
threshold frequency is in ultarviolet region of spectrum. Keep in Memory
(Corresponding to l between 200 & 300 nm), but for potassium &
cesium oxides, it is in the visible spectrum (l between 400 & 700 nm). 1. Mass spectrograph is an appratus used to determine the
mass or the specific charge (e/m) of positive ions. Examples
Work Function (f) of Some Elements Given in Brackets :
are (a) Thomson mass spectrograph (b) Bain bridge mass
Al (4.3eV) Ni (5.1 eV) spectrograph (c) Aston mass spectrograph (d) Dempster
C (5.0 eV) Si (4.8 eV) mass spectrograph etc.
Cu (4.7 eV) Ag (4.3 eV) 2. In photoelectric effect all photoelectrons do not have same
Au (5.1eV) Na (2.7 eV) kinetic energy. Their KE ranges from zero to E max which
depends on frequency of incident radiation and nature of
where 1 eV = 1.602 × 10–19 joule. cathode.
Within the framework of photon theory of light (Quantum theory 3. The photoelectric effect takes place only when photons
of light) we can explain above failures of classical physics. strike bound electrons because for free electrons energy
(1) It is clear from eq. (3) that if energy of photon is less than and momentum conservations do not hold together.
the work function of metallic surface, the electrons will never 4. Cesium is the best photo sensitive material.
be ejected from the surface regardless of intensity of incident
5. Efficiency of photoemission,
light.
(2) Kmax is independent of intensity of incident light, but it Number of photoelectrons emitted
per unit area per unit time n
depends on the frequency of incident light i.e., Kmax µ n h= = e
Number of photons incident np
(frequency of light). per unit area per unit time
(3) Electrons are emitted almost instantaneously consistent with Intensity of emitted electrons I e
particle view of light in which incident energy is concentrated h= =
Intensity of incident radiation I p
in small packets (called photons) rather than over a large ne I
area (as in wave theory). Therefore, h = = e
np I p
Various Graphs Related to Photoelectric Effect
6. Maximum velocity of emitted electrons
1. Photocurrent versus intensity of light graph
I 2h(v - v0 ) 2hc(l 0 - l ) 2evs
n > n0 vmax = = =
(mA) m mll 0 m
7. Stopping potential
h(v - v0 ) hc(l - l0 )
Vs = =
e ell 0

de-BROGLIE EQUATION (DUAL NATURE OF MATTER)


Intensity In 1924, Louis de Broglie, wrote a doctoral dissertation in which
2
(W/m ) he proposed that since photons have wave and particle
2. Photocurrent versus potential graph characterstics, perhaps all forms of matter have wave as well as
n2 > n 1 > n 0 particle properties.
I I1 = I2 This is called dual nature of matter. According to which A matter
I n1 = n2 > n0 I2 > I1 (mA) particle moving with a velocity v can be treated as a wave of
(mA)
I2 wavelength l. This l is called de-Broglie wavelength & it is defined
I1 as :
h h ...(1)
l= =
P mv
–V0 V –V02 –V0 V where m is the mass of matter particle & these waves are called
1
matter waves.
Free eBooks on @neetquestionpaper2020

918 Ph ysi cs
Further with the analogy of photon, the frequency of matter waves associated with them and that the Bragg condition for X-ray
is diffraction holds true for electron also :
E Bragg’s equation
n= ...(2)
h nl = D sin q or nl = 2d sin f.
The dual nature of matter is quite apparent in these two equations Diffraction maximum of electrons accelerated with 54 volt is
(equations (1) & (2)). i.e., each equation contains both particle obtained at q = 50º for the Nickel crystal.
concepts (mv & E) & wave concepts (l & n). It is clear from next Explanation of Bohr’s quantum condition :
topic that Compton effect confirm the validity of p = h/l for (a) According to Bohr’s quantum conditions :
photons, and the photoelectric effect confirms the validity and
h
E = hn for photons. Angular momentum, mvrn = n
de-Broglie wavelength associated with electron accelerated under 2p
12.27 (b) Matter waves associated with the electrons moving in an
a potential difference V volt is given by l = Å orbit are stationary waves.
V (c) For the production of stationary waves in the orbit the
de-Broglie wave is not an electromagnetic wave but the matter
wave. circumference of the orbit should be integral multiple of
Wavelength of matter waves associated with accelerated charged wavelength of waves associated with the electron,
particles : h
i.e., 2prn = nl, where l =
If V is the accelerating voltage applied then : mv
(a) For the charged particle : nh
\ mvrn =
Energy E = qV ; 2p
2qV 2E COMPTON EFFECT
Velocity v = =
m m Further experimental proof for photon concept(i.e., particle nature
of light) was discovered in 1923 by American Physicist, A.H.
Momentum p = 2mE = 2mqV ;
Compton. According to which, when a monochromatic beam of
h h X-rays (wavelength l0) strikes the electron in a carbon target,
Wavelength l = =
2 mqV 2mE two types of X-rays are scattered. The first type of scattered
wave has same wavelength (l0) as the incoming X-rays, while
12.27
(b) For electron le = Å second type has a longer wavelength (l) than incident rays (First
V type of X-rays are called unmodified x-rays, while second type of
0.286 X-rays are called modified X-rays.) This change in wavelength i.e.
(c) For proton lp = Å
Dl = l – l0 is called Compton shift & this effect is called Compton
V
effect”.
0.101
(d) For alpha particle la = Å y
V
X-ra E hc/l
0.202 red
(e) For deuteron ld = Å a tte p=h/l
V Incident x-rays Sc
For neutral particles (neutron, atom or molecule) : q
h h E0=hc/l f
(a) If E is the energy of the particle, then, = m
p 2mE p0=h/l 0 Recoiling
Carbon target electron
h which consists
(b) If T is the temperature, then, l = of free electron
3mkT
DAVISSON-GERMER EXPERIMENT sinq
Idea of de-Broglie wave was tested beautifully in 1926 in an
experiment performed by C. Davission (1881-1958) and L.H. Jermer q
cosq
(1896-1971). They directed a beam of electrons at a crystal and
observed that the electrons scattered in various directions for a
given crystal orientation.
In this experiment the pattern
formed by the electrons
cosf
reflected from the crystal f
lattice of aluminium is almost
identical to that produced by sinf
X-rays. This strongly Diagram shows compton scattering of an x-rays by free electron in
suggests that the electrons a carbon target. The scattered x-rays has less energy than the
incident x-rays. The excess energy is taken by recoiling electrons.
have a wavelength l
Free eBooks on @neetquestionpaper2020

Dual Nature of Radiation and Matter 919


This effect cannot be explained by classical theory (by wave nature X-RAYS
of light). According to classical model, when X-rays of frequency · The X-rays were discvoered by Prof. Roentgen, a German
n0 is incident on the material containing electrons, then electrons scientist in 1885. He was awarded Nobel Prize for this
do oscillate & reradiate electromagnetic waves of same frequency
discovery in 1901. X-rays are electromagnetic waves.
n0 . Hence scattered X-rays has same frequency n0 & same
· The modern apparatus for the production of X-rays was
wavelength as that of incident X-rays.
developed by Dr. Coolidge in 1913.
Compton treated this processes as a collision between a photon
& an electron. In this treatment, the photon is assumed as a particle · X-rays are produced when fast moving electrons are
of energy suddenly stopped on a metal of high atomic number.
E = hn0 = hc/l0 ...(1) Properties of X-rays
Further, the rest mass of photon is zero (because photon travels (i) They are not deflected by electric or magnetic field.
with the speed of light) hence the momentum of photon can be (ii) They travel with the speed of light.
written as (iii) There is no charge on X-rays.
E hc h ...(2) (iv) X-rays show both particle and wave nature.
p= = =
c cl l (v) They are invisible.
To dervie the compton shift. Dl, we apply both conservation of Continuous and Characteristic X-rays
energy & momentum. Experimental observation and studies of spectra of X-rays reveal
Conservation of energy : that X-rays are of two types and so are their respective spectras.
hc hc Characteristic X-rays and Continuous X-rays.
= + Ke ...(3)
l0 l Characteristic X-rays: The spectra of this group consists of
Where hc/l is energy of scattered X-rays, Ke is kinetic energy of several radiations with specific sharp wavelengths and frequency
recoling electron & hc/l0 is the energy of incoming X-rays. Since similar to the spectrum (line) of atoms like hydrogen. The
the electron may travel with the speed of light, so we must use wavelengths of this group show characteristic discrete radiations
relativistic expression of Ke in equation (3), and we obtain
emitted by the atoms of the target material. The characteristic
hc hc X-rays spectra helps us to identify the element of target material.
= + g mc 2 - mc 2 ...(4)
l0 l When the atoms of the target material are bombard with high
where m is rest mass of electron and mc2 is the rest mass energy energy electrons (or hard X-rays), which posses enough energy
of the electron to penetrate into the atom, knock out the electron of inner shell
1 (say K shell, n = 1). When an electron is missing in the ‘K’ shell, an
where g =
1 - v2 / c 2 electron from next upper shell makes a quantum jump to fill the
Conservation of momentum : vacancy in ‘K’ shell. In the transition process the electron radiates
h h energy whose frequency lies in the X-rays region. The frequency
= cos q + gmv cos f x - component ...(5)
l0 l of emitted radiation (i.e. of ph oton ) is given by
h æ 1 1ö
0= sin q - gmv sin f y - component ...(6) v = RZe2 ç 2 - 2 ÷ ; where R is constant and Z is effective
l è n1 n2 ø e
where pe = gmv is the relativistic expression for momentum of atomic number. Generally Ze is taken to be equal to Z – s, where Z
recoiling electron. is proton number or atomic number of the element and s is called
By eliminating v & f from equation (4) to (6), we obtain the screening constant. Due to the presence of the other electrons.
h The charge of the nucleus as seen by the electron will be different
Dl = l - l 0 = (1 - cos q) ...(7)
mc in different shells.
or Dl = 0.0243(1 - cos q)Å ...(8)
ek
It is clear from expression (7) that compton shift Dl depends on hv
scattering angle q & not on the wavelength. (x-ray)
Keep in Memory K L
K L
1. The wave nature of light shows up in the phenomena of ei
interference, diffraction and polarisation whereas
photoelectric effect and compton effect shows particle nature ei
of light. Knocking out e - of K shell by incident electron e i
2. The maximum kinetic energy of the photoelectrons varies emission of X-ray photon (Ka- series)
linearly with the frequency of incident radiation but is Another vacancy is now created in the ‘L’ shell which is again
independent of its intensity. filled up by another electron jump from one of the upper shell (M)
Free eBooks on @neetquestionpaper2020

920 Ph ysi cs
which results in the emission of another photon, but of different Ka
X-rays frequency. This transition continues till outer shells are
Kb
reached. Thus resulting in the emission of series of spectral line. La

Intensity
Kg Lb
The transitions of electrons from various outer shells to the inner
Ma
most ‘K’ shell produces a group of X-rays lines called as
K-series. These radiations are most energetic and most
penetrating. K-series is further divided into Ka , Kb , K g …. 0.01 l min 0.1 1.0
depending upon the outer shell from which the transition is made. Wavelength (nm)

The jump of electrons from outer shells to ‘L’ shell results in


50 kV
L-Series X-rays

Relative intensity
40 kV
n=a
Na 30 kV
n=4 N
M a Mb 20 kV
n=3 M
La M-series
L b Lg 0 0.02 0.04 0.06 0.08 0.10
n=2 L Wavelength (nm)
L-series
Wavelength of X-Rays (Daume Hunt Law)
Ka
Kb
(i) When an electron is accelerated through a potential
Kg K difference V then
d

1
n=1 K the energy accquired by electron = eV = mv 2 .
K-series
2
(ii) When these high energy electrons fall on target T of high
and so on. atomic number, then X-rays are produced, whose wavelength
Continuous X-rays : In addition to characteristic X-rays tubes is given by
emit a continuous spectrum also. The characteristic line spectra is hc hc
superimposed on a continuous X-rays spectra of varying \ eV = hn = ; l= .
l eV
intensities. The wavelength of the continuous X-rays spectra are
hc
independent of material. One important feature of continuous X- (iii) The energy of X-rays of wavelength l is E = hn =
l
rays is that they end abruptly at a certain lower wavelength for a (iv) The shortest wavelength of X-rays emitted is
given voltage. If an electron beam of energy eV (electron volts) is
hc 1240 1
incident on the target material; the electrons are suddenly stopped. l= = nm i.e. l µ .
eV V V
If the whole of the energy is converted to continuous radiation,
then lmin (corresponding to energy maximum) = hc/Ve where V is It is called Daume Hunt law.
the voltage applied. Types of X-Rays
X-ray photon 1. Hard X- rays : The X-rays of high frequency or low
hv wavelength are said to be hard X-rays. They have higher
penetrating power.
K K¢
2. Soft X-rays : The X-rays of longer wavelength are called
Target atom
soft X-rays.
The classical theory of electromagnetism states that the suddenly
accelerated or decelerated electrons emit radiations of Moseley’s Law
electromagnetic nature called as bremsstrahlung (braking Moseley used different elements as target in the X-ray tube. He
found that Ka radiation of different elements were different
radiation) and wavelength of such radiation is continuous because
the loss in energy is statistical. At the peak, the probability of Mathematically, n = a( Z - b)
maximum number of electrons producing radiation. where a and b depend on the particular line of the radiation
The wavelength of X-rays emitted is minimum corresponding to
3
the electron which hits the target with maximum speed. This For Ka, a = Rc and b = 1
electron is completely stopped and will emit the photon of highest 4
energy. where R = Rydberg constant and c = speed of light
As the electrons lose energy by collision, longer wavelengths are In general the wavelength of K - lines are given as
produced the shape of the curve is statistical. 1 é 1 ù
= R ( Z - 1) 2 ê1 - ú where n = 2, 3, .....
l ë n2 û
Free eBooks on @neetquestionpaper2020

Dual Nature of Radiation and Matter 921


Absorption of X-rays h
4. The quantity (= 0.02426Å) is called Compton
· X-rays are absorbed by the materials according to the mc
wavelength.
relation I = I0 e–mx, where m is absorption coefficient and x
5. The maximum wavelength change possible in compton
is the thickness of the mateiral. Here I is the intensity after
effect is 0.05Å.
penetrating the material through distance x and I0 is the
6. Compton effect can't be observed for visible light rays.
initial intensity of the X-rays.
7. In compton effect, the direction of recoil electron is given
· The coefficient of absorption (m) of the material is given by
lsinq
0.6931 by tan f = .
m= where x1/2 is the distance after traversing which l '- l cos q
x1/2
8. The kinetic energy of recoil electron is given by
the intensity of X-rays is reduced to half.
é x(1 - cos q) ù hn
· Absorption coefficient depends on the nature of material T = hn ê ú , where x =
and wavelength of X-rays i.e. m = cZ4 l3. ë1 + x(1 - cos q) û mc 2
It means that (a) m µ z4 (b) m µ l3 (c) m µ n–3. 9. de-Broglie wavelength of a particle of K.E., Ek is given by
Fluorescence : Certain substances (like quinine sulphate, h
fluoricine, barium platinocyanide, uranium oxide etc.), when l= .
2mE k
illuminated with light of high frequency (ultraviolet, violet, etc.)
emit light of lower frequency. The phenomenon is called 10. de-Broglie wavelength for a charged particle with charge q
fluorescence. and accelerated through a potental difference V is given by
· When quinine sulphate is illuminated with ultraviolet or h
violet light it gives out blue light. The fluroescence of barium l= .
2mqV
sulphate as well as uranium oxide gives out green light when
illuminated with ultraviolet or violet light. 11. de-Broglie wavelength of a material particle at temperature
· The house hold tubes are painted from inside with T is given by
magnesium tungstate or zinc-beryllium silicate. They are h
fluorescent materials. The ultraviolet light generated inside l= . , where k is Boltzmann’s constant.
2mkT
the tube falls on the walls, where magnesium tungstate gives
blue light and zinc beryllium silicate gives yellow orange Application of X-rays
light. The mixture of the two produces white light. If the Following are some important and useful applications of X-rays.
inner side of the tube is painted with cadmium borate it 1. Scientific applications: The diffraction of X-rays at crystals
gives fluorescence of pink light and when painted with zinc opened new dimension to X-rays crystallography. Various
silicate, it gives fluorescence of green light. diffraction patterns are used to determine internal structure
· The fluorescence occurs as long as the material is of crystals. The spacing and dispositions of atoms of a crystal
illuminated. can be precisely determined by using Bragg’s law :
Phosphorescence : Fluorescent materials emit light only so long nl = 2d sin q.
as light is incident on them. There are certain susbstances which 2. Industrial applications: Since X-rays can penetrate through
continue emitting light for some time after the light incident on various materials, they are used in industry to detect defects
them is stopped. This phenomenon is called phosphorescence. in metallic structures in big machines, railway tracks and
For example, if we make blue light incident on a zinc-sulphide bridges. X-rays are used to analyse the composition of alloys
(ZnS) screen, then it produces phosphorescence of green colour. and pearls.
Calcium sulpide and barium sulphide, after absorbing sunlight, 3. In radio therapy: X-rays can cause damage to the tissues of
produce blue phosphorescence for some time. Time of body (cells are ionized and molecules are broken). So X-rays
phosphorescence is different for different materials. damages the malignant growths like cancer and tumors which
Keep in Memory are dangerous to life, when is used in proper and controlled
intensities.
1. The stopping potential (and hence the maximum kinetic
4. In medicine and surgery: X-rays are absorbed more in heavy
energy of emitted electrons) is independent of the intensity
elements than the lighter ones. Since bones (containing
of light but that the saturation current (and hence the number
of emitted photoelectrons) is proportional to the intensity. calcium and phosphorus) absorb more X-rays than the
2. Photoelctric effect doesn't take place below the threshold surrounding tissues (containing light elements like H , C , O ),
frequency for the photometal used. their shadow is casted on the photographic plate. So the
3. In compton effect, the change in wavelength is independent cracks or fracture in bones can be easily located. Similarly
of incident photon as well as of the nature of the scatterer, intestine and digestive system abnormalities are also detected
but depends only on the angle of scattering (q). by X-rays.
Free eBooks on @neetquestionpaper2020

922 Ph ysi cs
Example 1. hc
An electron is moving with velocity 107 m/s on a circular (b) E k = hn - W0 = - W0
l
path of radius 0.57 cm in a magnetic field of 10–2 Wb/m2.
Find the value of e/m for electron. (6.63 ´ 10 -34 ) (3 ´ 108 )
= - (2.9 ´ 10 -19 joule)
Solution : 5000 ´ 10 -10
e v = (4.0 – 2.9) ×10–19 = 1.1 × 10–19 joule.
e v B = ( mv 2 / r ) or =
m Br 1
(c) E k = mv 2max
e 107 2
\ = = 1.76 × 1011 coulomb/kg.
m 10 - 2 ´ 0.0057 æ 2 Ek ö é 2 ´ (1.1´ 10 -19 ) ù
v
\ max = ç ÷ = ê - 31 ú
Example 2. è m ø êë 9.0 ´10 úû
Find the ratio of specific charge e/m of a proton to that of 5
= 5.0 × 10 m/sec.
an a particle. (d) The K.E. of emitted electrons does not depend upon
Solution : the intensity of light. Hence if the intensity of incident
æ e ö light be doubled, the energy will remain unchanged.
æeö æeö 2e
ç ÷ =ç ÷
and ç ÷ = Example 5.
è m ø proton çè m p ÷ø m
è øa 4 mp What will be the ratio of the de-Broglie wavelength of
(e / m) proton (e / m p ) 2 proton and a particle of same energy?
\ = = or 2 : 1 Solution :
(e / m ) a (2 e / 4 m p ) 1
h h
Example 3. l= =
Determine the ratio of momentum of an electron and an mv 2mE
alpha particle which are accelerated from rest by a 1
potential difference of 100 V. \ E= mv2 or 2 m E = m2v2 or mv = 2mE
2
Solution :
lp æm ö 4
1
Q m v 2 = e V or v = (2 e V / m)1 / 2 so, = ç a÷= = 2 or l p : l a = 2 : 1
la ç mp ÷ 1
2 è ø
\ p = m v = (2 m e V)1 / 2 Example 6.
An electron of mass m, when accelerated through a potential
Now p e = (2 m e ´ e ´100)1 / 2 V, has de-Broglie wavelength l. What will be the de-Broglie
and p a = (4 m a ´ e ´100)1 / 2 wavelength associated with a photon of mass M accelerated
through the same potential difference.
\ p e / p a = (m e / 2 m a ) Solution :
Example 4. Momentum of electron,
The work function of cesium is 1.8 eV. Light of 5000 Å is Pe = ( 2 m e V ) , where e = 1.6 × 10 –19 coulomb
incident on it. Calculate (a) threshold frequency and
threshold wavelength. (b) maximum K.E. of emitted Momentum of photon Pp = (2 M e V)
electrons. (c) maximum velocity of emitted electrons lp h / Pp P 2meV æmö
(d) if the intensity of the incident light be doubled, then = = e = = ç ÷
what will be the maximum K.E. of the emitted le h / Pe Pp 2MeV èMø
electrons? (h = 6.6 × 10–34 joule second, mass of electron
æmö æmö
m = 9.0×10–31 kg and speed of light c = 3 x 108 m/s). \ lp = le ç ÷ = l ç ÷
Solution : èMø èMø
Example 7.
(a) W0 = h v 0 or v 0 = h / W0
If 5% of the energy supplied to a bulb is radiated as visible
W0 = 1.8 ´ (1.6 ´10 -19 ) = 2.9 × 10–19 joule light, how many quanta are emitted per sec by a 100 watt
lamp? Assume wavelength of visible light is 5.6 × 10–15 cm.
W0 2.9 ´ 10 -19 joule Solution :
\ n0 = =
h 6.6 ´ 10 -34 joule second 5
Energy radiated as visible light = ´ 100 = 5 J / s.
\ Threshold frequency = 4.4 × 1014 sec–1 100
Threshold wavelength Let n be the number of photons emitted per second. Then
hc
nhn = n =5
c 3.0 ´108 m / s l
l0 = =
n 0 4.4 ´1014 s -1 5 ´ 5.6 ´10 -7
5l = 1.4 ´1019
= 6.8 × 10–7 metre = 6800 Å \ n= =
hc (6.2 ´10 -34 ) (3 ´ 108 )
Free eBooks on @neetquestionpaper2020

Dual Nature of Radiation and Matter 923


Example 8. Example 11.
When one centimeter thick surface is illuminated with light Find the number of photons emitted per second by a
of wavelength l, stopping potential is V and when the same 25 watt source of monochromatic light of wavelength 6000Å.
surface is illuminated by light of wavelength 2 l, stopping Solution :
potential is V/3. Determine threshold wavelength for metallic Energy of one photon
surface.
hc 6.62 ´ 10 -34 ´ 3 ´ 108
Solution : E = hn = = = 3.315 ´ 10 -19 J
l 6000 ´ 10-10
æ1 1 ö
eV = hc ç - ÷ …(1) No. of photons emitted per second
è l l0 ø
Total energy emitted per second P
eV æ 1 1ö = =
= hc ç - ÷ …(2) Energy of the photon E
3 è 2l l 0 ø
æ1 1 ö 25
çç - ÷ = ´ 10 -19 = 7.54 ´ 1019
è l l 0 ÷ø 3.315
Dividing eq. (1) by eqn. (2), we get 3 = Example 12.
æ 1 1 ö
çç - ÷÷ Sun gives light at the rate of 1400 Wm –2 of area
è 2 l l 0ø perpendicular to the direction of light. Assume l (sunlight)
æ 1 1 ö 1 1 = 6000Å. Calculate the
or 3çç - ÷= -
÷
è 2l l 0 ø l l 0 (a) number of photons/sec arriving at 1 m2 area at that
3 1 3 1 1 2 part of the earth, and
- = - or = (b) number of photons emitted from the sun/sec assuming
2l l l 0 l 0 2l l 0
l0 = 4l the average radius of Earth’s orbit is 1.49 × 1011 mm.
Example 9. Solution :
Energy levels A, B, C of a certain atom correspond to I = 1400 W/m2 ; l = 6000 Å
increasing values of energy i.e., EA < EB <EC. If l1, l2 and hc
l3 be the wavelengths of radiation corresponding to the (a) Energy of the photon E, = hn =
l
transitions C to B, B to A and C to A respectively, which of (c = 3 × 108 m/sec)
the following relations is correct? Let n be the number of photons received /sec per unit area.
(a) l3 = l1 + l2 (b) l3 = l1l2/(l1 + l2)
IA (1400 ´ 1) ´ (6000 ´ 10 -10 )
(c) l1 + l2 +l3 = 0 (d) l32 = l12 + l22 n= = -34 8
= 4.22 ´ 10 21
E/photon 6.63 ´ 10 ´ 3 ´ 10
Solution : (b)
(b) Total energy emitted per second = power (watts)
hc
(E 2 - E1 ) = hn = No. of photons per second
l
Power of Sun (W) I ´ (4 pR 2 ) ´ (6000 ´ 10 -10 )
hc hc = =
\ = (E C - E B ), = (E B - E A ) E/photon 6.63 ´ 10 -34 ´ 3 ´ 108
l1 l2 45
= 1.178 × 10 (R = average radius of earth’s orbit)
hc
and = (E C - E A ) Example 13.
l3
In a photoelectric experiment, with light of wavelength l,
Now (E C - E A ) = (E C - E B ) + (E B - E A ) the fastest electron has speed v. If the exciting wavelength
hc hc hc 1 1 1 is changed to 3l/4, then find the speed of the fastest emitted
or = + or = +
l 3 l1 l 2 l 3 l1 l 2 electron.
1 l + l2 ll Solution :
\ = 1 or l 3 = 1 2
l3 l1l 2 l1 + l 2 1 hc
mv 2 = - f
Example 10. 2 l
Calculate the energy and momentum of a photon of 1 hc 4hc
wavelength 6600Å. m¢ = -f = -f
2 (3l / 4) 3l
Solution :
Energy of photon 4
hc 6.62 ´ 10 -34 ´ 3 ´ 108 Clearly, v ¢ > v
E= = = 3 ´ 10 -19 J 3
l -10
6600 ´ 10 Example 14.
Momentum of photon
What is the de-Broglie wavelength associated with (a) an
h 6.6 ´ 10-34 electron moving with a speed of 5.4 × 106 m/s, and (b) a
p= = = 10-27 kg m / sec
l 6600 ´ 10 -10 ball of mass 150 g travelling at 30.0 m/s?
Free eBooks on @neetquestionpaper2020

924 Ph ysi cs
Solution : Example 16.
(a) For the electron If the Ka radiation of Mo (Z = 42) has a wavelength of
Mass m = 9.11 ×10–31 kg, speed v = 5.4 × 106 m/s. Then, 0.71 Å, calculate wavelength of the corresponding radiation
momentum p = m v of Cu, i.e., Ka for Cu (Z = 29) assuming s = 1.
= 9.11 ×10–31 (kg) × 5.4 × 106 (m/s) Solution :
p = 4.92 × 10–24 kg m/s
According to Moseley’s law : v = a(Z - 1)
6.63 ´ 10 -34 Js
de-Broglie wavelength, l = h/p = 2
Þ (Z - 1) µ v (Z - 1)2 µ 1/ l
4.92 ´ 10 -24 kg m / s or
= 0.135 nm (ZM0 - 1) 2 l Cu
(b) For the ball: Mass m' = 0.150 kg, Þ =
(ZCu - 1) 2 l Mo
Speed v' = 30.0 m/s.
Then momentum p' = m'v' (Z M0 - 1) 2
= 0.150 (kg) × 30.0 (m/s) Þ l Cu = l Mo
(ZCu - 1) 2
p' = 4.50 kg m/s
2
de Broglie wavelength l' = h/p' æ 41ö
= 0.71 ´ ç ÷ Å = 1.52Å
6.63 ´ 10-34 Js è 28 ø
= = 1.47 × 10–34 m Example 17.
4.50 kg m / s
The de-Broglie wavelength of electron is comparable The wavelength of Ka x-rays of two metals ‘A’ and ‘B’ are
with X-ray wavelengths. However, for the ball it is about 4 1
10–19 times the size of the proton, quite beyond and respectively, where ‘R’ is Rydbergg
1875R 675R
experimental measurement.
constant. Find the number of elements lying between A and
Example 15.
B according to their atomic numbers.
What is the de Broglie wavelength associated with an
Solution :
electron, accelerated through a potential difference of 100
volt? 1 é1 1 ù
Solution : Using = R(Z - 1)2 ê 2 - 2 ú
l êë n 2 n1 úû
Accelerating potential V = 100 V. The de Broglie wavelength,
For a particle, n 1 = 2, n2 = 1
h 1.227
l= = nm 1875 R æ 3ö
p V For metal A : = R (Z1 - 1) 2 ç ÷ Þ Z1 = 26
4 è 4ø
1.227
l= nm = 0.123nm æ 3ö
100 For metal B : 675 R = R (Z2 - 1)2 ç ÷ Þ Z2 = 31
The de-Broglie wavelength associated with an electron in è 4ø
this case is of the order of X-ray wavelengths. Therefore, 4 elements lie between A and B.

26.1
Solve following problems with the help of above text and 3. A strong argument for the particle nature of cathode rays is
examples : that they
1. The specific charge for cathode rays is (a) produce fluoroscence
(b) travel through vacuum
(a) constant
(c) get deflected by electric and magnetic fields
(b) variable (d) cast shadow
(c) depend upon the material of the cathode 4. In J.J. Thomson’s method, electric field E, magnetic field B
(d) depend upon the nature of gas in the discharge tube and velocity v of the electrons were in mutually
2. Cathode rays are made to pass between the poles of a perpendicular direction. This velocity selector allows
magnet perpendicular to axis, the effect of the magnetic particles of velocity v to pass undeflected when
field is (a) v = BE (b) v = E/B
(a) to increase the velocity of rays (c) v = B/E (d) v = B2/E
5. The ratio of the specific charge (e) of a proton to that of an
(b) to deflect them towards the north pole
a particle is
(c) to deflect them towards the south pole (a) 1 : 2 (b) 1 : 1/4
(d) to deflect them upwards above the plane of paper (c) 1 : 4 (d) 1 : 1/2
Free eBooks on @neetquestionpaper2020

Dual Nature of Radiation and Matter 925

6. The mass of a photon at rest is 19. The photoelectric effect is based on the law of conservation
(a) hn/c2 (b) hn/c of
(c) hn (d) zero (a) momentum (b) energy
7. The idea of the quantum nature of light has emerged in an (c) angular momentum (d) mass
attempt to explain 20. The photoelectric effect can be understood on the basis of
(a) the thermal radiations of a black body (a) wave theroy of light only
(b) the interference of light (b) electromagnetic theory of light only
(c) radioactivity (c) quantum theory of light only
(d) thermionic emission (d) None of these
8. A photon is a/an 21. When light is incident on a metal surface the maximum kinetic
(a) quantum of light energy of emitted electrons
(b) quantum of matter (a) vary with intensity of light
(c) positively charged paticle (b) vary with frequency of light
(d) instrument for measuring light intensity (c) vary with speed of light
9. Of the following properties, the photon does not possess (d) vary irregularly
(a) rest mass (b) momentum 22. The maximum energy of electrons released in a photocell is
(c) energy (d) frequency independent of
10. A photon will have less energy, if its (a) the frequency of the incident light
(a) amplitude is higher (b) the intensity of the incident light
(b) frequency is higher (c) the nature of the cathode
(c) wavelength is longer (d) All of the above
(d) wavelength is shorter 23. Einstein’s photoelectric equation states that
11. The energy of photon of wavelength l is hn = W0 + Ek.
(a) cl/h (b) hl/c In this equatin, Ek refers to the
(c) hc/l (d) c/hl (a) kinetic energy of all the emitted electrons
12. It is essential to consider light as a stream of photons to (b) mean kinetic energy of the emitted electrons
explain (c) maximum kinetic energy of the emitted electrons
(a) diffraction of light (b) refraction of light
(d) minimum kinetic energy of the emitted electrons
(c) photoelectric effect (d) reflection of light
24. X-rays are used to irradiate sodium and copper surfaces in
13. Photoelectric effect was discovered by
two separate experiments and stopping potential
(a) Hertz (b) Hallwachs
determined. This stopping potential is
(c) Lenard (d) Millikan
(a) equal in both cases (b) greater for sodium
14. The photoelectrons emitted from a metal surface are such
(c) greater for copper (d) infinite in both cases
that their velocity
25. In the photoeletric effect, electrons are emitted
(a) is zero for all
(b) is same for all (a) at a rate that is proportional to the amplitude of the
(c) lies between zero and infinity incident radiation
(d) lies between zero and a finite maximum (b) with a maximum velocity proportional to the frequency
15. Photoelectric effect shows of the incident radiation
(a) wave like behaviour of light (c) at a rate that is independent of the emitter
(b) paritcle like behaviour of light (d) only if the frequency of the incident radiations is above
(c) both wavelike and paticle like behaviour a certain threshold value
(d) neither wave like nor particle like behaviour of light 26. The minimum energy required to eject an electron, from the
16. The best metal to be usd for photoemission is metal surface is called
(a) potassium (b) sodium (a) atomic energy (b) mechanical energy
(c) cesium (d) lithium (c) electrical energy (d) workfunction
17. A photoelectric cell converts 27. The work function for photoelectric effect
(a) light energy into heat energy (a) is different for different metals
(b) light energy to sound energy (b) is same for all metals
(c) light energy into electric energy (c) depends upon the intensity of incident light
(d) electric energy into light energy (d) depends upon the frequency of incident light
18. Light of a particular frequency n is incident on a metal 28. If the kinetic energy of a moving particle is E, then the
surface. When the intensity of incident radiation is de-Broglie wavelength is
increased, the photoelectric current 2mE
(a) decreases (a) l = h 2mE (b) l =
h
(b) increases
h hE
(c) remains unchanged (c) l= (d) l =
(d) sometimes increases and sometimes decreases 2mE 2mE
Free eBooks on @neetquestionpaper2020

926 Ph ysi cs

29. If a photon and an electron have same de-Broglie 32. The wavelength of characteristic X-rays depends upon
wavelength, then (a) temperature of target
(a) both have same kinetic energy (b) size of target
(b) proton has more K.E. than electron (c) atomic number of target
(c) electron has more K.E. than proton (d) mass of target
(d) both have same velocity 33. The intensity (I) of X-rays after traversing a distance x
through a matter is related to the coefficient of absorption
30. The magnitude of the de-Broglie wavelength (l) of electron
(m) of the material as
(e), proton (p), neutron (n) and a-particle (a) all having the
(a) I = I0 emx (b) I = I0 e–mx
same energy of 1 MeV, in the increasing order will follow
mx
(c) I = e /I0 (d) I = e–mx/I0
the sequence
(a) le, lp, ln, la (b) le, ln, lp, la 34. How is the coefficient of absorption (m) of a material is
(c) la, ln, lp, le (d) lp, le, la, ln related to wavelength (l) of X-ray?
31. On the basis of the uncertainty principle it can be proved (a) m µ l (b) m µ l2
(c) m µ l 3 (d) m µ l4
that
(a) electrons exist inside the nucleus 35. The coeff. of absorption (m) of a material for X-ray is related
(b) electrons do not exist inside the nucleus to atomic number (Z) of material as
(c) neutrons exist inside the nucleus (a) m µ Z3 (b) m µ Z4
(d) protons exist inside the nucleus (c) m µ Z–3 (d) m µ Z–4

ANSWER KEY
1. (a) 2. (d) 3. (c) 4. (b) 5. (d) 6. (d) 7.(a) 8.(a) 9.(a) 10.(c) 11.(c) 12.(c) 13.(a)
14.(d) 15.(b) 16.(c) 17.(c) 18.(b) 19.(b) 20.(c) 21.(b) 22.(b) 23.(c) 24.(b) 25.(d) 26.(d)
27.(a) 28.(c) 29.(c) 30.(c) 31.(b) 32.(c) 33.(b) 34.(c) 35.(b)
Free eBooks on @neetquestionpaper2020

Dual Nature of Radiation and Matter 927

Very Short / Short Answer Questions 16. Plot a graph showing variation of stopping potential (V0)
with the frequency (v) of the incident radiation for a given
1. A proton and an electron have same kinetic energy. Which photosensitive material. Hence state the significance of the
one has smaller de-Broglie wavelength and why? threshold frequency in photoelectric emission.
[Outside Delhi - 2012] Using the principle of energy conservation, write the
2. For a given photosensitive material and with a source of equation relating the energy of incident photon, threshold
constant frequency of incident rediation, how does the frequency and the maximum kinetic energy of the emitted
photocurrent vary with the intensity of incident light? photoelectrons. [Delhi Board - 2009 COMPTT.]
[Outside Delhi - 2011 COMPTT.] 17. Draw a schematic diagram of the experimental arrangement
3. Draw a plot showing the variation of photoelectric current used by Davisson and Germer to establish the wave-nature
with collector plate potential for two different frequencies, of electrons. Express the de-Broglie wavelength l associated
with electron in terms of the accelerating voltage V.
v1 > v2, of incident radiation having the same intensity. In
which case will the stopping potential be higher? Justify An electron and a proton have the same kinetic energy.
Which of the two will have larger wavelength and why?
your answer. [Outside Delhi - 2011]
[Outside Delhi - 2007]
4. Define the term ‘Stopping potential’ in relation to
18. Draw and explain Davisson and Germer experiment to
photoelectric effect. [Outside Delhi - 2011]
determine the wave nature of electrons.
5. Write Einstein’s photoelectric equation. State clearly the
19. Light of l = 5000 Å falls on a photosensitive plate with work
three salient features observe in photoelectric effect, which
function 1.9 eV. Find (1) energy of photon in eV, (ii) k.E of
can be explained on the basis of the above equation. photoelectrons emitted (iii) stopping potential.
[Outside Delhi - 2010] Take h = 6.6 × 10–34 J/s
6. Define work function.
Multiple Choice Questions
7. Define the phenomenon of photoelectric effect.
8. Write down the expression for Einstein’s photoelectric 20. In which of the following, emission of electrons does not
equation. take place?
9. Write de-Broglie’s relation. (a) Thermionic emission
10. The threshold wavelength l for photoelectric emission from (b) X-rays emission
a material is 5200 Å. Will the photoelectrons be emitted when (c) Photoelectric emission
this material is illuminated with 1 Watt, UV lamp? (d) Secondary emission
11. Explain why an electron cannot come out of the metal surface 21. In Davison-Germer experiment, an electron beam is incident
on its own? on a crystal. The reflected beam consists of
12. Derive an expression for stopping potential from Einsteins (a) photons (b) protons
photo electric equation. (c) x-rays (d) electrons
13. An increase in intensity of incident light doesn’t change 22. The de-Broglie wavelength of an electron moving in the nth
the maximum velocity of the emitted photoelectron. Why? Bohr orbit of radius r is given by
14. Show that de-Broglie wavelength. 2 pr nr nr
(a) (b) npr (c) (d)
n 2p p
l of electron of energy E is given by the relation,
23. The kinetic energy of electron (in electron volt) moving with
l = h / 2mE a velocity of 4 × 106 m/s will be
Long Answer Questions (a) 60 eV (b) 50 eV (c) 30 eV (d) 45.5 eV
24. Photoelectric effect is the phenomenon in which
15. Define the terms (i) ‘cut-off voltage’ and (ii) ‘threshold (a) photons come out of a metal when it is hit by a beam of
frequency’ in relation to the phenomenon of photoelectric electrons.
effect. (b) photons come out of the nucleus of an atom under the
action of an electric field.
Using Einstein’s photoelectric equation show how the cut-
off voltage and threshold frequency for a given (c) electrons come out of a metal with a constant velocity
which depends on the frequency and intensity of
photosensitive material can be determined with the help of a
incident light wave.
suitable plot/graph. [Outside Delhi - 2012]
Free eBooks on @neetquestionpaper2020

928 Ph ysi cs
(d) electons come out of a metal with different velocities (a) intensity
not greater than a certain value which depends only on (b) frequency
the frequency of the incident light wave and not on its (c) wavelength
intensity. (d) None of these
25. The momentum of a photon of wavelength l is 27. A photoelectric cell is a device which
(a) hl (b) h/l (c) l/h (d) h/cl (a) converts light into electricity
26. A photo sensitive metal is not emitting photo-electrons when (b) converts electricity into light
irradiated. It will do so when threshold is crossed. To cross (c) stores light
the threshold we need to increase (d) stores electricity

1. A source S1 is producing, 1015 photons per second of 8. Two radiations of photons energies 1 eV and 2.5 eV,
wavelength 5000 Å. Another source S2 is producing successively illuminate a photosensitive metallic surface of
1.02×1015 photons per second of wavelength 5100Å Then, work function 0.5 eV. The ratio of the maximum speeds of
(power of S2) to the (power of S1) is equal to : the emitted electrons is [CBSE PMT 2012 (M)]
[CBSE PMT 2010] (a) 1 : 4 (b) 1 : 2 (c) 1 : 1 (d) 1 : 5
(a) 1.00 (b) 1.02 (c) 1.04 (d) 0.98 9. A 200 W sodium street lamp emits yellow light of wavelength
2. The potential difference that must be applied to stop the 0.6 µm. Assuming it to be 25% efficient in converting
fastest photoelectrons emitted by a nickel surface, having electrical energy to light, the number of photons of yellow
work function 5.01 eV, when ultraviolet light of 200 nm falls light it emits per second is [CBSE PMT 2012 (S)]
on it, must be [CBSE PMT 2010] (a) 1.5 × 1020 (b) 6 × 1018
(a) 2.4 V (b) – 1.2 V (c) – 2.4 V (d) 1.2 V (c) 62 × 1020 (d) 3 × 1019
3. Photoelectric emmision occurs only when the incident light 10. Monochromatic radiation emitted when electron on
has more than a certain minimum [CBSE PMT 2011] hydrogen atom jumps from first excited to the ground state
(a) power (b) wavelength irradiates a photosensitive material. The stopping potential
(c) intensity (d) frequency is measured to be 3.57 V. The threshold frequency of the
4. In the Davisson and Germer experiment, the velocity of materials is [CBSE PMT 2012 (S)]
electrons emitted from the electron gun can be increased by (a) 4 × 1015 Hz (b) 5 × 1015 Hz
[CBSE PMT 2011] (c) 1.6 × 1015 Hz (d) 2.5 × 1015 Hz
(a) increasing the potential difference between the anode 11. For photoelectric emission from certain metal the cut-off
and filament frequency is n. If radiation of frequency 2n impinges on the
(b) increasing the filament current metal plate, the maximum possible velocity of the emitted
(c) decreasing the filament current electron will be (m is the electron mass) [NEET 2013]
(d) decreasing the potential difference between the anode (a) hn / m (b) 2 hn / m
and filament
(c) 2 hn / m (d) hn / ( 2m )
5. In photoelectric emission process from a metal of work
function 1.8 eV, the kinetic energy of most energetic 12. The wavelength le of an electron and lp of a photon are of
electrons is 0.5 eV. The corresponding stopping potential is same energy E are related by [NEET 2013]
[CBSE PMT 2011] (a) l p µ le (b) l p µ le
(a) 1.8 V (b) 1.2 V (c) 0.5 V (d) 2.3 V
1
6. The threshold frequency for a photosensitive metal is (c) lp µ (d) l p µ l e2
3.3 × 1014 Hz. If light of frequency 8.2 × 1014 Hz is incident le
on this metal, the cut-off voltage for the photoelectric 13. Sodium and copper have work functions 2.3 eV and 4.5 eV
emission is nearly [CBSE PMT 2011(M)] respectively. Then the ratio of the wavelengths is nearest to
(a) 2 V (b) 3 V (c) 5 V (d) 1 V [AIEEE 2002]
7. If the momentum of electron is changed by P, then the de (a) 1 : 2 (b) 4 : 1 (c) 2 : 1 (d) 1 : 4
Broglie wavelength associated with it changes by 0.5%. 14. Formation of covalent bonds in compounds exhibits
The initial momentum of electron will be (a) wave nature of electron [AIEEE 2002]
[CBSE PMT 2012 (M)] (b) particle nature of electron
P (c) both wave and particle nature of electron
(a) 200 P (b) 400 P (c) (d) 100 P (d) none of these
200
Free eBooks on @neetquestionpaper2020

Dual Nature of Radiation and Matter 929


15. Two identical photocathodes receive light of frequencies
f1 and f2 . If the velocites of the photoelectrons (of mass m)
coming out are respectively v1 and v2, then [AIEEE 2003]
(a) I (b) I
2 2 2h
(a) v1 - v 2 = (f1 - f 2 )
m
1/ 2 O l O l
é 2h ù
(b) v1 + v 2 = ê (f1 + f 2 )ú
ë m û
2h (c) (d) I
(c) v12 + v 2 2 = (f1 + f 2 ) I
m
1/ 2
é 2h ù
(d) v1 - v 2 = ê (f1 - f 2 )ú O l O l
ëm û DIRECTIONS Qs. no. 23 to 25 each has Statement-1 and
16. According to Einstein’s photoelectric equation, the plot of Statement-2 of the four choices given after the statements, choose
the kinetic energy of the emitted photo electrons from a the one that best describes the two statements.
metal Vs the frequency of the incident radiation gives as 23. Statement-1 : When ultraviolet light is incident on a
straight the whose slope [AIEEE 2004] photocell, its stopping potential is V0 and the maximum
(a) depends both on the intensity of the radiation and the kinetic energy of the photoelectrons is Kmax .When the
metal used ultraviolet light is replaced by X-rays, both V0 and Kmax
(b) depends on the intensity of the radiation increase.
(c) depends on the nature of the metal used Statement-2 : Photoelectrons are emitted with speeds
(d) is the same for the all metals and independent of the ranging from zero to a maximum value because of the range
intensity of the radiation of frequencies present in the incident light. [AIEEE 2010]
17. The work function of a substance is 4.0 eV. The longest (a) Statement-1 is true, Statement-2 is true ; Statement-2 is
wavelength of light that can cause photoelectron emission the correct explanation of Statement -1.
from this substance is approximately [AIEEE 2004] (b) Statement-1 is true, Statement-2 is true; Statement-2 is
(a) 310 nm (b) 400 nm not the correct explanation of Statement-1
(c) 540 nm (d) 220 nm (c) Statement-1 is false, Statement-2 is true.
18. A photocell is illuminated by a small bright source placed (d) Statement-1 is true, Statement-2 is false.
1 24. Statement-1: A metallic surface is irradiated by a
1m away. When the same source of light is placed m monochromatic light of frequency v > v0 (the threshold
2
away, the number of electrons emitted by photocathode frequency). The maximum kinetic energy and the stopping
would [AIEEE 2005] potential are Kmax and V0 respectively. If the frequency
(a) increase by a factor of 4 incident on the surface is doubled, both the Kmax and V0
(b) decrease by a factor of 4 are also doubled.
(c) increase by a factor of 2 Statement-2 : The maximum kinetic energy and the stopping
(d) decrease by a factor of 2 potential of photoelectrons emitted from a surface are linearly
dependent on the frequency of incident light.
19. If the kinetic energy of a free electron doubles, it’s de-Broglie
wavelength changes by the factor [AIEEE 2005] [AIEEE 2011]
(a) Statement-1 is true, Statement-2 is true, Statement-2 is
1 1
(a) 2 (b) (c) 2 (d) the correct explanation of Statement-1.
2 2 (b) Statement-1 is true, Statement-2 is true, Statement-2 is
20. The threshold frequency for a metallic surface corresponds not the correct explanation of Statement-1.
to an energy of 6.2 eV and the stopping potential for a (c) Statement-1 is false, Statement-2 is true.
radiation incident on this surface is 5 V. The incident radiation (d) Statement-1 is true, Statement-2 is false.
lies in [AIEEE 2006]
25. Statement-1: Davisson-Germer experiment established the
(a) ultra-violet region (b) infra-red region wave nature of electrons.
(c) visible region (d) X-ray region Statement-2 : If electrons have wave nature, they can
21. The time taken by a photoelectron to come out after the interfere and show diffraction. [AIEEE 2012]
photon strikes is approximately [AIEEE 2006] (a) Statement-1 is false, Statement-2 is true.
–4
(a) 10 s (b) 10 –10 s (c) 10 –16 s (d) 10–1 s (b) Statement-1 is true, Statement-2 is false
22. The anode voltage of a photocell is kept fixed. The (c) Statement-1 is true, Statement-2 is true, Statement-2 is
wavelength l of the light falling on the cathode is gradually the correct explanation for statement-1
changed. The plate current I of the photocell varies as
(d) Statement-1 is true, Statement-2 is true, Statement-2 is
follows [AIEEE 2006]
not the correct explanation of Statement-1
Free eBooks on @neetquestionpaper2020

930 Ph ysi cs
26. The anode voltage of a photocell is kept fixed. The 30. Photoelectric effect experiments are performed using three
wavelength l of the light falling on the cathode is gradually different metal plates p, q and r having work functions
changed. The plate current I of the photocell varies as fp = 2.0 eV, fq = 2.5 eV and fr = 3.0 eV, respectively. A light
follows [JEE Main 2013] beam containing wavelengths of 550 nm, 450 nm and 350
nm with equal intensities illuminates each of the plates. The
I I
correct I-V graph for the experiment is
[Take hc = 1240 eV nm] [IIT-JEE 2009]
(a) (b) I
I
O l O l p
q p
I I (a) (b)
r q
r
(c) (d) V V
O I
l O l I
27. In a photoelectric experiment anode potential is plotted r
against plate current [IIT JEE 2004s] q
(c) (d)
p
I r q
p
V V
C B 31. A silver sphere of radius 1 cm and work function 4.7 eV is
suspended from an insulating thread in freespace. It is under
A
continuous illumination of 200 nm wavelength light. As
photoelectrons are emitted, the sphere gets charged and
V
acquires a potential. The maximum number of photoelectrons
(a) A and B will have different intensities while B and C emitted from the sphere is A ´ 10z (where 1 < A < 10). The
will have different frequencies value of ‘z’ is [IIT-JEE 2011]
(b) B and C will have different intensities while A and C (a) 8 (b) 7
will have different frequencies (c) 6 (d) 4
(c) A and B will have different intensities while A and C 32. The work functions of Silver and Sodium are 4.6 and 2.3 eV,
will have equal frequencies respectively. The ratio of the slope of the stopping potential
(d) A and B will have equal intensities while B and C will versus frequency plot for Silver to that of Sodium is
have different frequencies [JEE Adv. 2013]
28. A proton has kinetic energy E = 100 keV which is equal to (a) 1 (b) 2
that of a photon. The wavelength of photon is l2 and that (c) 4 (d) zero
of proton is l1. The ratio of l2/l1 is proportional to
33. A pulse of light of duration 100 ns is absorbed completely
[IIT JEE 2004s] by a small object initially at rest. Power of the pulse is 30
(a) E2 (b) E1/2 (c) E–1 (d) E–1/2 mW and the speed of light is 3×108 ms–1 . The final
29. Electrons with de-Broglie wavelength l fall on the target in momentum of the object is [JEE Adv. 2013]
an X-ray tube. The cut-off wavelength of the emitted (a) 0.3 × 10–17 kg ms–1
X-rays is [IIT-JEE 2007] (b) 1.0 × 10–17 kg ms–1
2mcl 2 l0 =
2h (c) 3.0 × 10–17 kg ms–1
(a) l0 = (b)
h mc (d) 9.0 × 10–17 kg ms–1
2m 2 c 2 l 3
(c) l0 = (d) l0 = l
h2
Free eBooks on @neetquestionpaper2020

Dual Nature of Radiation and Matter 931

1. An electron is accelerated by a p.d. of 1000 V. Its velocity 13. The photoelectric work function for a metal surface is
will be 4.125 eV. The cut-off wavelength for this surface is
(a) 3.78 × 107 m/s (b) 1.89 × 107 m/s (a) 4125 Å (b) 3000 Å (c) 6000 Å (d) 2062 Å
7
(c) 5.67 × 10 m/s (d) 0.95 × 107 m/s 14. Einstein’s work on photoelectric effect provided support
2. The energy of a photon of green light of wavelength 5000Å for the equation
is (a) E = hn (b) E = mc2
(a) 3.459 × 10–19 joule (b) 3.973 × 10–19 joule -Rhc 1
–19 (c) E= (d) K.E. = mv 2
(c) 4.132 × 10 joule (d) 8453 × 10–19 joule n 2 2
3. If the energy of a photon is 10 eV, then its momentum is 15. Which of the following shows particle nature of light?
(a) 5.33 × 10–23 kg m/s (b) 5.33 × 10–25 kg m/s (a) Refraction (b) Interference
(c) 5.33 × 10–29 kg m/s (d) 5.33 × 10–27 kg m/s (c) Polarization (d) Photoelectric effect
4. The photoelectric threshold of Tungsten is 2300Å. The energy 16. Which of the following when falls on a metal will emit
of the electrons ejected from the surface by ultraviolet light photoelectrons ?
of wavelength 1800Å is (a) UV radiations (b) Infrared radiation
(a) 0.15 eV (b) 1.5 eV (c) 15 eV (d) 150 eV (c ) Radio waves (d) Microwaves
5. Ultraviolet radiation of 6.2 eV falls on an aluminium surface 17. 4eV is the energy of incident photon and the work function
(workfunction 4.2 eV). The kinetic energy in joule of the is 2eV. The stopping potential will be
faster electron emitted is approximately (a) 2V (b) 4V (c ) 6 V (d) 2 2 V
(a) 3 × 10–21 (b) 3 × 10–19
–17 3
(c) 3 × 10 (d) 3 × 10–15 18. The velocity of a body of rest mass m o is c (where c is
6. In a photoelectric experiment the stopping potential for the 2
the velocity of light in vacuum). The mass of this body is:
incident light of wavelength 4000Å is 2 volt. If the wavelength
be changed to 3000 Å, the stopping potential will be æ 3ö æ1ö
(a) ç ÷m (b) ç ÷m o
(a) 2 V (b) zero ç 2 ÷ o è2ø
è ø
(c) less than 2 V (d) more than 2 V
(c) 3m o (d) 2m0
7. A proton and a-particle are accelerated through the same
potential difference. The ratio of their de-Broglie wavelength 19. Radiations of intensity 0.5 W/m2 are striking a metal plate.
will be The pressure on the plate is
(a) 0.166 × 10–8 N/m2 (b) 0.332 × 10–8 N/m2
(a) 1 : 1 (b) 1 : 2 (c) 2 : 1 (d) 2 2 : 1 –8
(c) 0.111 × 10 N/m 2 (d) 0.083 × 10–8 N/m2
8. A particle with rest mass m0 is moving with speed of light c. 20. The ratio of de-Broglie wavelengths of proton and
The de-Broglie wavelength associated with it will be a -particle having same kinetic energy is
(a) µ (b) zero (c) m0 c/h (d) hn/m0c (a) 2 : 1 (b) 2 2 : 1 (c) 2 : 1 (d) 4 : 1
9. If the X-ray tube is working at 20 kV then the minimum
21. White X-rays are called white due to the fact that
wavelength of X-rays will be
(a) they are electromagnetic radiations having nature
(a) 0.31 Å (b) 0.62 Å (c) 0.93 Å (d) 0.47 Å same as that of white light.
10. The maximum distance between interatomic lattice planes is (b) they are produced most abundantly in X ray tubes.
15 Å. The maximum wavelength of X-rays which are (c) they have a continuous wavelength range.
diffracted by this crystal will be
(d) they can be converted to visible light using coated
(a) 15 Å (b) 20 Å (c) 30 Å (d) 45 Å screens and photographic plates are affected by
11. The glancing angle in a X-ray diffraction is 30º and the them just like light.
wavelength of X-rays used is 20 nm. The interplanar spacing 22. The maximum velocity of an electron emitted by light of
of the crystal dffracting these X-rays will be wavelength l incident on the surface of a metal of work-
(a) 40 nm (b) 20 nm (c) 15 nm (d) 10 nm function f is
12. As intensity of incident light increases
2( hc + lf ) 2( hc + lf )
(a) photoelectric current increase (a) (b)
ml ml
(b) K.E. of emitted photoelectron increases
(c) photoelectric current decreases 2( hc - lf ) 2(hl - f)
(c) (d)
(d) K.E. of emitted photoelectrons decreases ml m
Free eBooks on @neetquestionpaper2020

932 Ph ysi cs

23. Einstein’s photoelectric equation is E k = hn - f . In this 32. A photocell is illuminated by a small bright source placed 1 m
away. When the same source of light is placed 2 m away, the
equation Ek refers to
number of electrons emitted by photocathode are reduced by
(a) kinetic energy of all the emitted electrons a factor of
(b) mean kinetic energy of emitted electrons (a) 1/8 (b) 1/16 (c) 1/2 (d) 1/4
(c) maximum kinetic energy of emitted electrons 33. In a photoelectric effect measurement, the stopping potential
(d) minimum kinetic energy of emitted electrons for a given metal is found to be V0 volt when radiation of
24. The maximum kinetic energy (Emax) of photoelectrons wavelength l0 is used. If radiation of wavelength 2 l0 is
emitted in a photoelectric cell varies with frequency (n) as used with the same metal then the stopping potential (in
shown in the graph. The slope of the graph is equal to volt) will be
(a) charge of the electron E max V0
(a) (b) 2 V0
2
e hc hc
(b) of the electron (c) V0 + (d) V0 -
m 2el 0 2el 0
34. An electron of mass m and charge e initially at rest gets
(c) work function of the emitter accelerated by a constant electric field E. The rate of change
n0 n
(d) Plank’s constant of de-Broglie wavelength of this electron at time t ignoring
25. If E1, E2, E3 are the respective kinetic energies of an electron, relativistic effects is
an alpha-particle and a proton, each having the same -h -eht - mh -h
de-Broglie wavelength, then (a) 2 (b) (c) 2 (d)
e Et E e Et eE
(a) E1 > E3 > E2 (b) E2 > E3 > E1 35. The wavelength of Ka-line characteristic X-rays emitted by
(c) E1 > E2 > E3 (d) E1 = E2 = E3 an element is 0.32 Å. The wavelength of Kb-line emitted by
26. A light having wavelength 300 nm fall on a metal surface. the same element will be
The work function of metal is 2.54 eV, what is stopping (a) 0.32 Å (b) 0.39 Å (c) 0.49 Å (d) 0.27 Å
potential ? 36. The X-rays of wavelength 0.5 Å are scattered by a target.
(a) 2.3 V (b) 2.59 V (c) 1.59 V (d) 1.29 V What will be the energy of incident X-rays, if these are
27. A and B are two metals with threshold frequencies scattered at an angle of 72º ?
1.8 × 1014 Hz and 2.2 × 1014 Hz. Two identical photons of (a) 12.41 keV (b) 6.2 keV
energy 0.825 eV each are incident on them. Then (c) 18.6 keV (d) 24.82 keV
photoelectrons are emitted in (Take h = 6.6 × 10–34 Js) 37. In a photo-emissive cell with exciting wavelength l, the
(a) B alone (b) A alone fastest electron has speed V. If the exciting wavelength is
(c) neither A nor B (d) both A and B. changed to 3l/4, the speed of the fastest emitted electron
28. The de-Broglie wavelength of a proton (mass = 1.6 × will be :
10–27 kg) accelerated through a potential difference of 1 kV (a) n (¾)1 / 2 (b) v(4 / 3)1 / 2
is
(c) less the v(4 / 3)1 / 2 (d) greater than v(4 / 3)1 / 2
(a) 600 A (b) 0.9 × 10–12m
38. The momentum of photon whose frequency f is
(c) 7 Å (d) 0.9 nm.
hf hc h c
29. An ionisation chamber, with parallel conducting plates as (a) (b) (c ) (d)
anode and cathodes has singly charged positive ions per c f f hf
39. In case of electrons and photons having the same
cm3. The electrons are moving toward the anode with
wavelength. What is same for them?
velocity 0.4 m/s. The current density from anode to cathode
(a) Energy (b) Velocity
is X104 m A/m2. The velocity of positive ions moving
(c) Momentum (d) Angular momentum
towards cathode is
40. The variation of maximum kinetic energy of photoelectrons
(a) 0.1 m/s (b) 0.4 m/s (c) zero (d) 1.6 m/s
30. A parallel beam of light is incident normally on a plane surface with applied frequency (u) is
absorbing 40% of the light and reflecting the rest. If the (a) (b)
incident beam carries 60 watt of power, the force exerted by it on
K.E.max .
K .E.max .

the surface is
(a) 3.2 × 10–8 N (b) 3.2 × 10–7 N
(c) 5.12 × 10 N –7 (d) 5.12 × 10–8 N
31. Radiations of two photon’s energy, twice and ten times the (c) (d)
work function of metal are incident on the metal surface
K.E.max .

K .E.max .

successsively. The ratio of maximum velocities of


photoelectrons emitted in two cases is
(a) 1 : 2 (b) 1 : 3 (c) 1 : 4 (d) 1 : 1
Free eBooks on @neetquestionpaper2020

Dual Nature of Radiation and Matter 933

e2
41. The quantity has a value Y
2he 0 c
0.1 m
1 2
(a) ms -1 (b) ms -1
137 137 X
1 2
(c) (d)
137 137 1 2
42. A small photocell is placed at a distance of 4 m from a (a) 2.65 × 106 m/s (b) 7.02 × 1012 m/s
photosensitive surface. When light falls on the surface the (c) 1.87 × 106 m/s (d) 32 × 10–19 m/s
current is 5 mA. If the distance of cell is decreased to 1 m,
48. A steel ball of mass m is moving with a kinetic energy K. The
the current will become
de-Broglie wavelength associated with the ball is
æ 5ö
(a) 1.25 mA (b) ç ÷ mA h h
è 16 ø (a) (b)
2mK 2mK
(c) 20 mA (d) 80 mA
43. If 5% of the energy supplied to a bulb is radiated as visible h
(c) (d) meaningless
light, the number of visible quanta emitted per second by a 2mK
100 W bulb, assuming the wavelength of visible light to be 49. When the speed of electrons increase, then the value of its
5.6 × 10–5 cm, is specific charge
(a) 1.4 × 1019 (b) 1.4 × 1020 (a) increases
(c) 2 × 10 19 (d) 2 × 1020 (b) decreases
44. A material particle with a rest mass m0 is moving with speed (c) ramains unchanged
of light c. The de-Broglie wavelength associated is given
by (d) increases upto some velocity and then begins to
decrease
h m0c
(a) (b) (c) zero (d) ¥ 50. In an electron gun, the potential difference between the
m0c h filament and plate is 3000 V. What will be the velocity of
45. Which one of the following graphs represents the variation electron emitting from the gun?
of maximum kinetic energy (EK) of the emitted electrons (a) 3 × 108 m/s (b) 3.18 × 107 m/s
with frequency u in photoelectric effect correctly ?
(c) 3.52 × 107 m/s (d) 3.26 × 107 m/s
EK EK
51. Which metal will be suitable for a photoelectric cell using
light of wavelength 4000Å. The work functions of sodium
and copper are respectively 2.0 eV and 4.0 eV.
(a) (b) (a) Sodium (b) Copper
u u (c) Both (d) None of these
EK EK 52. What is the energy of ka X-ray photon of copper (Z = 29) ?
(a) 7.99 keV(b) 8.29 keV (c) 8.25 keV (d) 7.19 keV
53. When X-rays of wavelength 0.5 Å would be transmitted by
an aluminium tube of thickness 7 mm, its intensity remains
(c) (d)
one-fourth. The attenuation coefficient of aluminium for
u u these X-rays is
u0
46. In a photoelectric effect experiment, for radiation with (a) 0.188 mm–1 (b) 0.189 mm–1
frequency u0 with hu0 = 8eV, electrons are emitted with (c) 0.198 mm –1 (d) None of these
energy 2 eV. What is the energy of the electrons emitted for 54. An X-ray tube with Cu target is operated at 25 kV. The
incoming radiation of frequency 1.25 u0 ? glancing angle for a NaCl. Crystal for the Cu ka line is 15.8°.
(a) 1 eV (b) 3.25 eV Find the wavelength of this line.
(c) 4 eV (d) 9.25 eV. (d for NaCl = 2.82 Å, h = 6.62 × 10–27 erg-sec)
47. Two insulating plates are both uniformly charged in (a) 3.06 Å (b) 1.53 Å
such a way that the potential difference between them is
(c) 0.75 Å (d) None of these
V2 – V1 = 20 V. (i.e., plate 2 is at a higher potential). The
plates are separated by d = 0.1 m and can be treated as 55. The de-Broglie wavelength of a neutron at 927°C is l.
infinitely large. An electron is released from rest on the inner What will be its wavelength at 27 °C ?
surface of plate 1. What is its speed when it hits plate 2? l
(e = 1.6 × 10–19 C, me = 9.11 × 10–31 kg) (a) (b) l (c) 2 l (d) 4 l
2
Free eBooks on @neetquestionpaper2020

934 Ph ysi cs
56. In an electron gun the control grid is given a negative 64. The frequency and work function of an incident photon are
potential relative to cathode in order to v and f0. If v0 is the threshold frequency then necessary
(a) decelerate electrons condition for the emission of photoelectron is
(b) repel electrons and thus to control the number of
v0
electrons passing through it (a) v < v0 (b) v=
(c) to select electrons of same velocity and to converge 2
them along the axis. (c) v ³ v0 (d) None of these
(d) to decrease the kinetic energy of electrons 65. The work function of aluminium is 4.2 eV. If two photons,
57. A monochromatic source of light operating at 200 W emits each of energy 3.5 eV strike an electron of aluminium, then
4 × 1020 photons per second. Find the wavelength of light. emission of electrons
(a) 400 mm (b) 200 nm (a) will be possible
(c) 4 × 10–10 Å (d) None of these (b) will not be possible
58. An X-ray tube is operated at 15 kV. Calculate the upper limit (c) Data is incomplete
of the speed of the electrons striking the target. (d) Depends upon the density of the surface
(a) 7.26 × 107 m/s (b) 7.62 × 107 m/s
7 66. For intensity I of a light of wavelenght 5000Å the
(c) 7.62 × 10 cm/s (d) 7.26 × 109 m/s
photoelectron saturation current is 0.40 µA and stopping
59. All electrons ejected from a surface by incident light of
potential is 1.36 V, the work function of metal is
wavelength 200nm can be stopped before travelling 1m in
(a) 2.47 eV (b) 1.36 eV
the direction of uniform electric field of 4N/C. The work
function of the surface is (c) 1.10 eV (d) 0.43 eV
(a) 4 eV (b) 6.2 eV (c) 2 eV (d) 2.2 eV DIRECTIONS (Qs. 67 to 71) : Each question contains
60. The stopping potential (V0) versus frequency (v) plot of a STATEMENT-1 and STATEMENT-2. Choose the correct answer
substance is shown in figure, the threshold wavelength is (ONLY ONE option is correct ) from the following-
V0 (a) Statement-1 is false, Statement-2 is true
2 (b) Statement-1 is true, Statement-2 is true; Statement-2 is a
correct explanation for Statement-1
(c) Statement-1 is true, Statement-2 is true; Statement-2 is not
1
a correct explanation for Statement-1
(d) Statement-1 is true, Statement-2 is false
4 5 6 7 8 67. Statement-1 : In process of photoelectric emission, all
v × 1014 Hz emitted electrons do not have same kinetic energy.
(a) 5 × 1014m Statement-2 : If radiation falling on photosensitive surface
(b) 6000 Å of a metal consists of different wavelength then energy
(c) 5000 Å acquired by electrons absorbing photons of different
(d) Cannot be estimated from given data wavelengths shall be different.
61. A photon of 1.7 × 10–13 joule is absorbed by a material
68. Statement-1 : Though light of a single frequency
under special circumstances. The correct statement is
(monochromatic) is incident on a metal, the energies of
(a) Electrons of the atom of absorbed material will go the
emitted photoelectrons are different.
higher energy states
Statement-2 : The energy of electrons emitted from inside
(b) Electron and positron pair will be created
the metal surface, is lost in collision with the other atoms in
(c) Only positron will be produced
(d) Photoelectric effect will occur and electron will be the metal.
produced 69. Statement-1 : The de-Broglie wavelength of a molecule (in a
62. Light from a hydrogen discharge tube is incident on sample of ideal gas) varies inversely as the square root of
the cathode of a photoelectric cell, the work function of absolute temperature.
the cathode surface is 4.2 eV. In order to reduce the
Statement-2 : The rms velocity of a molecule (in a sample of
photocurrent to zero the voltage of the anode relative to the
ideal gas) depends on temperature.
cathode must be made
(a) – 4.2 V (b) – 9.4 V 70. Statement-1 : Photoelectric saturation current increases
(c) – 17.8 V (d) + 9.4 V with the increase in frequency of incident light.
63. X-rays are produced in X-ray tube operating at a given Statement-2 : Energy of incident photons increases with
accelerating voltage. The wavelength of the continuous increase in frequency and as a result photoelectric current
X-rays has values from increases.
(a) 0 to ¥ 71. Statement-1 : Photosensitivity of a metal is high if its work
(b) lmin to ¥, where lmin > 0 function is small.
(c) 0 to lmax, where lmax < ¥ Statement-2 : Work function = hf0 where f0 is the threshold
(d) lmin to lmax, where 0 < lmin < lmax < ¥ frequency.
Free eBooks on @neetquestionpaper2020

Dual Nature of Radiation and Matter 935


DIRECTIONS (Qs. 72 to 80) : Read the following passage(s) wavelength l dB of electrons can be calculated by the
carefully and answer the questions that follows: relationship (n is an integer)
PASSAGE-1 (a) d sin i = nldB (b) 2d cos i = nldB
A physicist wishes to eject electrons by shining light on a metal (c) 2d sin i = nldB (d) d cos i = nldB
surface. The light source emits light of wavelength of 450 nm. The 77. In an experiment, electrons are made to pass through a narrow
table lists the only available metals and their work functions. slit of width ‘d’ comparable to their de Broglie wavelength.
Metal W0 (eV) They are detected on a screen at a distance ‘D’ from the slit
Barium 2.5 (see figure).
Lithium 2.3
Tantalum 4.2
d
Tungsten 4.5 y=0
72. Which metal(s) can be used to produce electrons by the
photoelectric effect from given source of light ? D

(a) Barium only Which of the following graphs can be expected to represent
the number of electrons ‘N’ detected as a function of the
(b) Barium or lithium
detector position ‘y’(y = 0 corresponds to the middle of the
(c) Lithium, tantalum or tungsten
slit)
(d) Tungsten or tantalum y y
73. Which option correctly identifies the metal that will produce
the most energetic electrons and their energies ?
(a) Lithium, 0.45 eV (b) Tungston, 1.75 eV (a) N d (b) N d

(c) Lithium, 2.30 eV (d) Tungston, 2.75 eV


74. Suppose photoelectric experiment is done separately with y y
these metals with light of wavelength 450 nm. The maximum
magnitude of stopping potential amongst all the metals is-
(a) 2.75 volt (b) 4.5 volt (c) N d (d) N d

(c) 0.45 volt (d) 0.25 volt


PASSAGE-2 PASSAGE-3
Wave property of electrons implies that they will show diffraction Work function of metal A is equal to the ionization energy of
effects. Davisson and Germer demonstrated this by diffracting hydrogen atom in first excited state. Work function of metal B is
electrons from crystals. The law governing the diffraction from a equal to the ionization energy of He+ ion in second orbit. Photons
crystal is obtained by requiring that electron waves reflected from of same energy E are incident on both A and B. Maximum kinetic
the planes of atoms in a crystal interfere constructively (see figure). energy of photoelectrons emitted from A is twice that of
photoelectrons emitted from B.
78. Value of E (in eV) is
Inco m ng (a) 20.8 (b) 32.2 (c) 24.6 (d) 23.8
Electr ing i Outgoi s
ons ctro n 79. The difference in maximum kinetic energy of photoelectrons
Ele
from A and from B
d
(a) increases with increase in E
(b) decreases with increase in E
Crystal plane
(c) first increases than decrease with increase in E
75. Electrons accelerated by potential V are diffracted (d) remain constant
from a crystal. If d = 1Å and i = 30°, V should be about
80. For a photon of energy hn to eject an electron from a material
(h = 6.6 × 10 – 34 Js, me = 9.1 × 10–31 kg, e = 1.6 × 10 – 19 C)
with a work function f, the photon’s energy must be
(a) 2000 V (b) 50 V (c) 500 V (d) 1000 V
(a) greater than f
76. If a strong diffraction peak is observed when electrons are
(b) less than f
incident at an angle ‘i’ from the normal to the crystal planes
(c) less than maximum kinetic energy
with distance ‘d’ between them (see figure), de-Broglie
(d) equal to maximum kinetic energy
Free eBooks on @neetquestionpaper2020

936 Ph ysi cs

23. (c) In the given relation Ek stands for maximum K.E. of


Exercise 26.1
emitted photoelectron.

1. (a) Specific charge (e/m) for cathode rays is constant as e 24. (b) e Vs = hn - W0 ; As work function W0 is less for sodium
and m for cathode rays are fixed. than for copper, hence the value of stopping potential Vs
2. (d) The force on cathode rays (i.e. charge e) moving with is more for sodium than for copper.
r r 25. (d) Photoelectrons are emitted if the frequency of incident
velocity v due to a perpendicular magnetic field B is
r r r light is greater than the threshold frequency.
-e(v ´ B) , which acts perpendicular to plane of v and
r 26. (d) The minimum energy required for the emission of
B. electrons is called work function.
3. (c) As cathode rays are deflected by electric and mangetic 27. (a) The work function of different metals is different.
fields, it shows that cathode rays carry charged particles
(i.e. electrons) 1 2 h h
28. (c) E = mv or mv = 2 m E so l = =
2 mv 2mE
4. (b) e E = e vB or v = E/B
Specific ch arg e of proton h h v p me
5. (d) 29. (c) l = = ; then mp vp = me ve or =
Specific ch arg e of a particle m p v p me v e ve m p
e/m 1
= = 1 /(1 / 2) = 2 m p v 2p m 2
me
2e / 4m Ep 2 p æm ö <1
= = ´ç e ÷ =
mp
6. (d) The rest mass of a photon is zero Ee 1 me ç mp ÷
m e v e2 è ø
7. (a) Quantum theory of light came to explain the thermal 2
radiations of a black body. \ Ep < Ee
8. (a) A photon is a packet of energy called quantum of light h 1
energy 30. (c) l = so h µ
2m E m
9. (a) Photon has no rest mass
Since m a > m n > m p > m e
10. (c) Energy of a photon E = hc ; E is less if l is longer
l so de-Broglie wavelength in increasing order will be
11. (c) Energy of a photon E = hc/l la , ln , lp , le
12. (c) Photoelectric effect can be explained by quantum nature
of light i.e. light as a stream of photons. 31. (b) Uncertainity principle has helped to account that the
electron do not exist inside the nucleus.
13. (a) Hertz discovered first the photoelectric effect in 1887.
32. (c) The wavelength of characteristic X-rays depends upon
15. (b) Photoelectric effect is accounted by particle like
the nature of material of the target i.e. atomic number of
bahaviour of light (i.e. by quantum theory of light)
target.
16. (c) The best metal for photoelectric emission is one whose
34. (c) As m = cZ4l3 so m µ Z3
work-function is least. It is so for cesium.
35. (b) As m = cZ4l3 so m µ Z4
17. (c) Photoelectric cell converts light energy into electric
energy. Exercise 1 : NCERT Based Questions
18. (b) The photoelectric current µ Intensity of light.
19. (b) Photoelectric effect is based on law of conservation of 6. The minimum energy required to pull an electron out from
energy. the surface of the metal.
7. The phenomenon of emission of electrons when radiation
21. (b) Max. K.E. = hn – W0 ; so Max. K.E. µ n
is incident on a surface of metal.
22. (c) Max. K.E. of phtoelectrons emitted is independent of 8. Kmax = hv – f0
intensity of incident light.
Free eBooks on @neetquestionpaper2020

Dual Nature of Radiation and Matter 937


h h 7. (a) The de-Broglie’s wavelength associated with the
9. l == h
p mv moving electron l =
P
10. Yes, as l of UV is less than 5200 Å.
Now, according to problem
æ hö f
12. V0 = ç ÷ n - 0 dl dp
è eø e =-
l P
19. (i) energy of photon,
0.5 P
2.48 eV =
100 P '
(ii) k.E of photoelectron P¢ = 200 P
= hv – f0= 2.48 – 1.90 = 0.58 eV 8. (b) According to Einsten’s photoelectric effect, the K.E.
(iii) V0= stopping potential = 0.58 V of the radiated electrons

20. (b) 21. (d) 22. (a) 23. (d) K.Emax = E – W


1
24. (d) 25. (b) 26. (b) 27. (a) mv 2 = (1 – 0.5) eV = 0.5 eV
2 1
Exercise 2 : PAST Competition MCQs 1
mv 2 = (2.5 – 0.5) eV = 2 eV
2 2
hc
1. (a) Energy emitted/sec by S1 , P1 = n1 v1 0.5 1 1
l1 = = =
v2 2 4 2
hc
Energy emitted/sec by S2 , P2 = n2 9. (a) Give that, only 25% of 200W converter electrical energy
l2
P2 n2 l1 into light of yellow colour
\ = ×
P1 n1 l 2 æ hc ö 25
çè ÷ø ´ N = 200 ´
1.02 ´ 1015 5000 l 100
= × = 1.0
1015 5100 Where N is the No. of photons emitted per second,
hc hc 12375 h = plank’s constant, c, speed of light.
2. (d) Kmax = -W = - 5.01 = - 5.01
l l l (in Å) 200 ´ 25 l
N= ´
12375 100 hc
= –5.01 = 6.1875 – 5.01 = 1.17775 ; 1.2 V 200 ´ 25 ´ 0.6 ´ 10-6
2000 =
3. (d) For occurence of photoelectric effect, the incident light 100 ´ 6.2 ´ 10-34 ´ 3 ´ 108
should have frequency more than a certain minimum = 1.5 × 1020
which is called the threshold frequency (v0). 10. (c) n ® 2 – 1
1 E = 10.2 eV
We have, mv 2 = hv - hv0
2 kE = E – f
For photoelectric effect emission n > n0 Q = 10.20 – 3.57
where n is the frequency of the incident light. h u0 = 6.63 eV
4. (a) In the Davisson and Germer experiment, the velocity 6.63 ´ 1.6 ´ 10-19
of electrons emitted from the electron gun can be u0 = = 1.6 × 1015 Hz
increased by increasing the potential difference 6.67 ´ 10-34
between the anode and filament. 11. (b) From photoelectric equation,
5. (c) The stopping potential is equal to maximum kinetic hn' = hn + Kmax ...(i)
energy. 1
h.2n = hn + mV2max [\ n¢ = 2 n]
6. (a) K.E. = hn – hnth = eV0 (V0 = cut off voltage) 2
h 1
Þ V0 = (8.2 ´ 1014 - 3.3 ´ 1014 ) Þ hn = mV2max
e 2
6.6 ´ 10 -34 ´ 4.9 ´ 1014 2hn
= » 2V. Þ Vmax =
1.6 ´ 10 -19 m
Free eBooks on @neetquestionpaper2020

938 Ph ysi cs
E 22. (b) As l decreases, v increases and hence the speed of
12. (d) As P = photoelectron increases. The chances of photo electron
C
hC to meet the anode increases and hence photo electric
lp = ...(i) current increases.
E
hh 23. (d) We know that
le2 = ...(ii)
2mE eV0 = K max = hn - f
From equations (i) and (ii)
where, f is the work function .
lp µ le2
(l 0 )1 (W0 ) 2 4.5 Hence, as n increases (note that frequency of X-rays
13. (c) hc/ l 0 = W0 ; (l ) = ( W ) = 2.3 = 2 : 1. is greater than that of U.V. rays), both V0 and Kmax
0 2 0 1
increase. So statement-1 is correct
14. (a) Covalent bond formation is best explained by orbital
theory which uses wave phenomena. 24. (c) By Einstein photoelectric equation,
15. (a) Kmax = eV0 = hv – hv0
When v is doubled, Kmax and V0 become more than
16. (d) From Eqn K.E = hn - f
double.
slope of graph of K.E & n is h, 25. (c)
which is same for all metals. 26. (d) As l is increased, there will be a value of l above
17. (a) For the longest wavelength to emit photo electron which photoelectrons will be cease to come out so
hc hc photocurrent will become zero. Hence (d) is correct
=fÞl=
l f answer.
6.63 ´ 10-34 ´ 3 ´ 108 27. (a) From the graph it is clear that A and B have the same
Þl= -19 = 310 nm stopping potential and therefore the same frequency.
4 ´ 1.6 ´ 10
2 Also B and C have the same intensity.
I I1 æ r2 ö 1
18. (a) Iµ ; =ç ÷ = 28. (d) For photon E = hn
r2 I 2 çè r1 ÷ø 4
hc hc
I 2 ® 4 times I1 E= Þ l2 = ...(i)
l E
When intensity becomes 4 times, no. of photoelectrons
1
emitted would increase by 4 times, since number of for proton E = m p vp2
electrons emitted per second is directly proportional 2
to intensity. 2 2
1 mp vp
19. (d) de-Broglie wavelength, E= Þ p= 2mE
2 m
h h From De Broglie Eqn.
l= =
p 2.m.(K.E)
h h h
p= Þ l1 = = ...(ii)
1 l1 p 2mE
\ lµ
K. E
λ2 hc
= ¥ E –1/2
l λ1 E× h
If K.E is doubled, l becomes
2 2mE

20. (a) f = 6.2 eV = 6.2 ´ 1.6 ´10 -19 J 29. (a) The cut off wavelength is given by l 0 =
hc
eV
V = 5 volt
According to de Broglie equation
hc
- f = eV0
l h h h2 h2
l= = Þ l2 = ÞV=
hc 6.6 ´10 -34 ´ 3 ´108 p 2meV 2meV 2mel 2
Þl= = » 10- 7 m
f + eV0 1.6 ´10 -19 (6.2 + 5) From (i) and (ii)
This range lies in ultra violet range.
hc ´ 2mel 2 2mcl 2
21. (b) The order of time is nano second. l0 = =
eh 2 h
Free eBooks on @neetquestionpaper2020

Dual Nature of Radiation and Matter 939


30. (a) The energy possessed by photons of wavelength
E P ´ t 30 ´ 10 -3 ´ 100 ´ 10 -9
33. (b) p= = = = 10 -17 kg ms -1
1240 c c 3 ´ 108
550 nm is = 2.25 eV
550 option (b) is correct
The energy possessed by photons of wavelength
Exercise 3 : Conceptual & Applied MCQs
1240
450 nm is = 2.76 eV
450 1
1. (b) eV = m v 2 or v = 2eV / m
The energy possessed by photons of wavelength 2
1240 2. (b) E = hc/l = 6.6 × 10–34 × 3 × 108/5000 × 10–10
350 nm is = 3.54 eV
350 = 3.973 × 10–19 J
For metal plate p :
E 10 ´ 1.6 ´ 10 -19
fp = 2 eV. 3. (d) Momentum of a photon µ =
c 3 ´ 10 8
All the wavelengths are capable of ejecting electrons.
Therefore, the current is maximum. Also as the work = 5.33 ´10 - 27 kg ms -1
function is lowest in p, the kinetic energy of ejected
electron will be highest and therefore, the stopping hc æ 1 1 ö
potential is highest. 4. (a) E k = - (in eV)
c çè l l 0 ÷ø
For metal plate q :
fq = 2.5 eV. 6.6 ´ 10-34 ´ 3 ´ 108 æ 1010 1010 ö
= ç 1800 - 2300 ÷ = 0.15eV
Photons of wavelength 550 nm will not be able to eject 1.6 ´ 10-19 è ø
electrons and therefore, the current is smaller than p.
The work function is greater than q therefore the 5. (b) Ek = E – W0 = 6.2 – 4.2 = 2.0 eV
stopping potential is lower in comparison to p. = 2.0 × 1.6 × 10–19 = 3.2 × 10–19 J
For metal plate r :
hc
fr = 3 eV 6. (d) eVs = - W0 . If l decreases, V increases
l s
Only wavelength of 350 nm will be able to eject electrons
and therefore, current is minimum. Also the stopping 1
7. (d) q V = m v 2 or m v = 2q Vm ;
potential is least. 2
1 é hc ù h h 1
31. (b) Stopping potential = - f ú where So l= = i.e. l µ ;
e êë l û mv 2q V m qm
hc = 1240eV –nm
lp qama
1 é 1240 ù 1 so = = 2´ 4 = 2 2
1 la
= ê - 4.7 ú = [ 6.2 - 4.7 ] = ´ 1.5eV = 1.5V q pm p
e ë 200 û e e

1 q 1 ne h h 1- v2 / c2
But V = = 8. (b) l = = =0 (Q v = c)
4pe 0 r 4pe 0 r mv m0 v

Vr (4pe0 ) 1.5 ´ 10 -2 hc 12400 12400


\ n= = 9. (b) l min . = = = = 0.62Å
e 9 ´ 109 ´ 1.6 ´ 10 -19
eV V 20 ´103
\ n = 1.04 × 107 2d sin q 2 ´15 ´ sin 90º
Comparing it with A × 10z we get, z = 7 10. (c) l max . = = = 30Å
n min . 1
32. (a) For photoelectric effect
nl 1´ 20
hn f o
Vo 11. (b) 2 d sin q = nl or d = = = 20 nm
- = Vo 2 sin q 2 ´ sin 30º
e e
12. (a) Because when intensity of incident light increases, it
The slope is means that number of photons increases in incident light.
q n
h If number of incident photons increases, then number of
tan θ = = constant emitted photo electrons also increases, consequently
e
the photo electric current increases.
\ The ratio will be 1.
Free eBooks on @neetquestionpaper2020

940 Ph ysi cs
23. (c) 24. (d)
hc
13. (b) Since work function for a metal surface is W =
l0 1
25. (a) According to relation, E = mv 2
where l0 is threshold wavelength or cut-off wavelength 2
for a metal surface.
2E
here W = 4.125 eV = 4.125 × 1.6 × 10–19 Joule =v
m
6.6 ´ 10 -34 ´ 3 ´ 108
so l0 = = 3000Å
l=
h
4.125 ´ 1.6 ´ 10 -19 2mE
14. (a) Einstein’s photo electric effect & compton effect Because m1 < m3 < m2
established particle nature of light. These effects can be
explained only, when we assume that the light has particle So for same l, E1 > E 3 > E 2 .
nature (To explain, Interference & Diffraction the light 26. (c)
must have wave nature. It means that light has both
particle and wave nature, so it is called dual nature of 27. (b) Photoelectrons are emitted in A alone. Energy of
light) hu
electron needed if emitted from A = eV
15. (d) e
16. (a) Emission of electron from a substance under the action
of light is photoelectric effect. Light must be at a
\ EA =
( 6.6 ´10-34 ) ´ (1.8 ´1014 ) = 0.74 eV
sufficiently high frequency. It may be visible light, U.V,
X-rays. So U.V. cause electron emission. 1.6 ´10-19

17. (a) Einstein equation E = huo + K.E


EB =
( 6.6 ´10-34 ) ´ ( 2.2 ´1014 )
= 0.91 eV
where E = energy of incident photon.
1.6 ´10-19
huo = work function of metal Incident energy 0.825 eV is greater than EA (0.74 eV)
K.E = max. kinetic energy of e– but less than EB (0.91 eV).
\ 4 eV = 2 eV + K.E or K.E = 2 eV h h h
28. (b) l= = =
Stopping potential is the potential difference which may p 2mE 2mqV
stop this e–.
Let it be V, then eV = 2e Þ V = 2 volt. 6.6 ´ 10-34
\ l=

mo mo
( )( )
2 ´ 1.6 ´10-27 ´ 1.6 ´10 -19 ´ 1000
18. (d) m = = = 2m o
æ v2 ö
1/ 2 (1 - 3 4)1 / 2
ç1 - ÷ 6.6 ´ 10-34 ´ 10 22
ç c2 ÷ or l = = 0.9 ´ 10-12 m
è ø 1.6 20
19. (a) We know that P = Fv or F = P/v 29. (a) Here, No. of electrons
0.5 ne = 5 × 107/cm3 = 5 × 107 ×106 /m3
F= 8
= 0.166 ´ 10 -8 N / m 2 No. of positive ions,
3 ´ 10
np = 5 × 107 × 106 = 5 × 1013 /m3
h v = 0.4 m/s ; J = 4 × 10–6 A/m2 ; vp = ?
20. (c) de-Broglie wavelength, l =
2mEK.E Use the relation J = ne e ve + np e vp and solve it
for vp
lp ma 4m p 30. (b) Momentum of incident light per second
\ = = [Q E K.E (a) = E K.E ( p) ]
la mp mp E 60
P1 = = = 2 ´10 - 7
c 3 ´108
lp 2
\ = Momentum of reflected light per sec
la 1
60 E 60 60
21. (c) P2 = ´ = ´ = 1.2 ´ 10 -7
100 c 100 3 ´108
1 hc 2( hc - lf )
22. (c) mv 2 = -f Þ v =
2 l lm
Free eBooks on @neetquestionpaper2020

Dual Nature of Radiation and Matter 941


Force on the surface = change in momentum per sec
h
= P2 – (–P1) = P2 + P1 = (2 + 1.2) × 10–7 = 3.2 × 10–7 N 38. (a) Moment of photon = p =
l
1
31. (b) m n12 = 2 W0 - W0 = W0 and Q E = mc 2
2
But, p = mc
1 \ E = mc.c
m n 22 = 10 W0 - W0 = 9W0
2
hc
So, E = pc or E=
n1 W0 1 l
\ = =
n2 9 W0 3 hc h c
\ = pc or p= and l =
l l f
32. (d) Intensity µ 1/ (distance)2 ; No. of photoelectrons
emitted is proportional to intensity of incident light. hf
\p=
c
hc hc
33. (d) eV0 = - W0 and eV ¢ = - W0 h h
l0 2 l0 39. (c) Momentum p = Þl= (photon)
l p
Subtracting them we have
If l photon = l electron
hc é 1ù hc hc
e ( V0 - V¢) =
l0 ê1 - 2 ú = 2 l or V¢ = V0 - 2 e l p(momentum)will become same.
ë û 0 0
40. (b) As per Einstein’s photoelectric equation :
eE
34. (a) Here, u = 0 ; a = ;v=?;t=t E = hu = WF + KEmax
m
i.e. till a certain value of u , KE remains 0, it only starts
eE increasing once the Work function (WF) of the metal
\ v = u + at = 0 + t
m surface is achieved.

de-Broglie wavelength, l =
h
=
h
=
h e2 (1.6 ´ 10-19 ) 2
41. (c) =
mv m(eEt / m ) eEt 2he 0 c 2 ´ 6.26 ´ 10-34 ´ 8.857 ´ 10 -12 ´ 3 ´ 108
Rate of change of de-Broglie wavelength
1
= it is unit less quantity..
dl h æ 1 ö -h 137
= ç - ÷÷ =
ç
dt eE è t ø e E t 2
2
1
42. (d) Iµ
1 é1 1 ù 3R d2
35. (d) = R ( Z - a) 2 ê - ú = ( Z - a) 2
la ë12
22 û 4 43. (a) Energy associated with wave.
Let number of quanta = x
1 é1 1ù 8
= R ( Z - a) 2 ê - ú = R ( Z - a) 2 nhc
lb ë12
32 û 9 5% of 100 =
l
lb 27
\ = or l b =
27
la =
27
´ 0.32 = 0.27 Å 5 n ´ 6.26 ´ 10-34 ´ 3 ´ 10 -8
la 32 32 32 Þ 100 ´ =
100 5.6 ´ 10-7
hc 6.6 ´ 10 -34 ´ 3 ´ 108 5
36. (d) Energy = = J Þn= Þ n = 1.4 ´ 1019
l 0.5 ´ 10 -10 3.35 ´ 10-19
6.6 ´ 10 -34 ´ 3 ´ 108 h m0
= eV = 24.82 keV 44. (c) l= ,v= , v ® c, m ® ¥
5 ´ 10 -11 ´ 1.6 ´ 10 -19 mv 2
æ vö
1- ç ÷
1 hc hc è cø
37. (d) mv 2 = - Where l is exciting wavelength, l0
2 l l0
hence, l ® 0 .
is threshold wavelength & v is speed of fastest electron.
Free eBooks on @neetquestionpaper2020

942 Ph ysi cs
45. (d) hu – hu0 = EK, according to photoelectric equation, 52. (a) E(ka) = 10.2 (Z – 1)2 eV
when u = u0, EK = 0. = 10.2 × 282
Graph (d) represents EK – u relationship. = 10.2 × 784
46. (c) hu = Wex + maximum kinetic energy = 7.997 keV = 8 KeV
hu0 = 8 eV = Wex + 2 eV Þ Wex = 6 eV
I0
For incoming radiation, energy is 53. (c) = 4, x = 7mm (given)
I
h × 1.25 u0 = 10 eV, Wex = 6 eV
\ Kinetic energy (maximum) = 4 eV. I
2.303 log10 0
Þ m= I
1
47. (a) eV = mv 2 x
2
2.303 log10 4 2.303 ´ 0.6023
m= =
2eV 2 ´ 1.6 ´10 -19 ´ 20 7 7
Þv= =
m 9.1´ 10-31 m = 0.198 mm–1
54. (b) According to Bragg’s law, Þ 2d sin q = nl,
= 2.65 ´10 6 m / s n = 1 for first order
h Þ 2 × 2.82 sin 15.8 = l Þ l = 5.64 × 0.2723 = 1.53 Å
48. (c) de-Broglie’s relation, l =
p 55. (c) de-Broglie wavelength of a material particle at
temperature T is given by
momentum p = 2mE
h
l= . , where k is Boltzmann's constant.
h h 2mkT
Þl=
2mE
=
2mK
(Q E = K )
1
Þ lµ
49. (b) Here the velocity of electron increases, so as per T
Einstein’s equation mass of the electron increases,
e l2 T1
\ =
hence the specific charge decreases. l1 T2
m
50. (d) V = 3000 volt.
l2 1200
or = =2
1 2eV l1 300
mv2 = eV Þ v =
2 m l 2 = 2l1 = 2l
\

2 ´1.6 ´ 10-19 ´ 3000 56. (b)


\ v=
9.1 ´ 10 -31 200
57. (a) The energy of each photon = = 5 × 10–19 J
= 32.6 × 106 = 3.26 × 107 m/s. 4 ´ 1020
hc hc
51. (a) Q l0 = Wavelength = l =
f E

(6.63 ´ 10-34 ) ´ (3 ´ 108 )


6.6 ´ 10-34 ´ 3 ´ 108 =
\ (l0)sodium= = 6188 Å 5 ´ 10-19
2 ´ 1.6 ´ 10-19
Þ l = 4.0 × 10–7 = 400 nm
1 (l ) (f) copper 58. (a) The maximum kinetic energy of an electron accelerated
Q l0 µ Þ 0 sodium =
f (l 0 )copper (f)sodium 1
through a potential difference of V volt is mv2 = eV
2
2
Þ (l0)copper = × 6188 = 3094 Å 2eV
4 \ maximum velocity v =
m
To eject photo-electrons from sodium the longest
wavelength is 6188 Å and that for copper is 3094 Å.
2 ´ 1.6 ´ 10-19 ´ 15000
Hence for light of wavelength 4000 Å, sodium is v= = 7.26 × 107 m/s
suitable. 9.1 ´ 10-31
Free eBooks on @neetquestionpaper2020

Dual Nature of Radiation and Matter 943


59. (d) The electron ejected with maximum speed vmax are 70. (d) Photoelectric saturation current is independent of
stopped by electric field E =4N/C after travelling a frequency. It only depends on intensity of light.
distance d =1m 71. (b) Less work function means less energy is required for
ejecting out the electrons.
1
mv 2max = eEd = 4eV
2 12400
72. (b) DE =
4.500Å
1240
The energy of incident photon = = 6.2 eV D = 2.75 eV
200
For photoelectric effect, DE > W0 (work function).
From equation of photo electric effect
73. (a) DE = W0 + E ; (Ek) = DE – W0
1 For maximum value of (Ek), W0 should be minimum
mv 2max = hn - f0
2 W0 for lithium = 2.3 eV
\ f0 = 6.2 - 4 = 2.2 eV \ (Ek) = 2.75 – 2.3 = 0.45 eV
74. (c) The maximum magnitude of stopping potential will be
c 3 ´ 108 for metal of least work function.
60. (b) l0 = = = 6 ´ 10-7 m = 6000Å
v0 5 ´ 1014 \ required stopping potential is
61. (b) For electron and positron pair production, minimum hv - f 0
energy is 1.02 MeV. Vs = = 0.45 volt.
e
1.7 ´ 10-13 75. (b) The path difference between the rays APB and CQD is
-3
Energy of photon is given 1.7 ´ 10 J = Dx = MQ + QN = d cos i +d cos i
1.6 ´ 10-19
Dx = 2d cos i
= 1.06 MeV.
We know that for constructive interference the path
Since energy of photon is greater than 1.02 MeV. difference is nl
so electron positron pair will be created.
62. (b) E = W0 + eV0 A B
For hydrogen atom, E = +13.6 eV C D
\ + 13.6 = 4.2 + eV0
i
(13.6 - 4.2)eV
Þ V0 = = 9.4V
e d P
M i N
Potential at anode = – 9.4 V Q
63. (b) 64. (c)
65. (b) For emission of electrons incident energy of each \ nl = 2d cos i
photon must be greater than work function (threshold Also by de-Broglie concept
energy).
66. (c) By using E = W0 + Kmax h h h
l= = =
p 2mK.E 2meV
12375
E= =2.475 eV and Kmax = eV0 = 1.36 eV
5000 nh
\ = 2d cos i
So 2.475 = W0 + 1.36 Þ W0 = 1.1 eV. 2meV
67. (b) Both statement I and II are true; but even it radiation of
single wavelength is incident on photosensitive h2
surface, electrons of different KE will be emitted. Here n =1 : V =
8med2 cos2 i
68. (a) When a light of single frequency falls on the electrons
of inner layer of metal, then this electron comes out of (6.6 ´10 -34 )2
the metal surface after a large number of collisions with = = 50 V
atom of it's upper layer. 8 ´ 9.1´10 -31 ´1.6 ´10 -19 ´ (10 -10 ) 2 ´ cos 2 30
69. (a) de-Broglie wavelength associated with gas molecules Alternatively
varies as 1 Using Bragg’s equation 2d sinq = nl
1 Here n = 1, q = 90 – i = 90 – 30 = 60°

T \ 2d sin q = l .......(i)
Free eBooks on @neetquestionpaper2020

944 Ph ysi cs
77. (d) The electron beam will be diffracted and the maxima is
obtained at y = 0. Also the distance between the first
minima on both sides will be greater than d.
Sol. for (78-80)
q
13.6 13.6 ´ 4
fA = eV, fB = eV
4 4
E - fA = (KE)A ,
12.27 -10
Also, l = ´ 10 m .......(ii) E - fB = (KE)B
V
KEA = 2KEB
From eqn (i) & (ii)
E - fA = 2 (E - fB )
12.27
2 ´10-10 ´ sin 60o = ´ 10-10
V E = 2fB - fA

13.6
(12.27)2 . = 2 ´ 13.6 - = 23.8eV
\ V= = 50V 4
3
78. (d) 79. (d) 80. (a)
76. (b) 2d cos i = nldB
Free eBooks on @neetquestionpaper2020

27
Atoms
THOMSON'S ATOMIC MODEL Conclusions :
This model suggests an atom to be a tiny sphere of radius • The entire positive charge and almost whole mass of the
» 10 -10 m , containing the positive charge. The atom is electrically atom is concentrated in small centre called a nucleus.
• The electrons revolving round the nucleus could not
neutral. It contains an equal negative charge in the form of deflected the path of a-particles. This suggests that
electrons, which are embedded randomly in this sphere, like seeds electrons are very light.
in a watermelon. In 1911 Rutherford , proposed a new type of model of the atom.
This model failed to explain (i) large scattering angle of According to this model, the positive charge of the atom, instead
a-particle and (ii) origin of spectral lines observed in the spectrum of being uniformly distributed throughout a sphere of atomic
of hydrogen atom. dimension is concentrated in a very small volume at its centre.
ALPHA-PARTICLE SCATTERING AND RUTHER-FORD This central core, called nucleus, is surrounded by clouds of
NUCLEAR MODEL OF ATOM electrons makes the entire atom electrically neutral.
In Rutherford a- particle scattering experiment a very fine beam According to Rutherford scattering formula, the number of
of a-particle passes through a small hole in the lead screen. a-particles scattered at angle q by a target,
This well collimated beam is then allowed to fall on a thin gold N µ cosec4 (q/2)
foil. While passing through the gold foil, a-particles are a
scattered through different angles. A zinc sulphide screen is
placed out the other side of the gold foil, this screen is movable, b
so as to receive the a-particles, scattered from the gold foil at +
angles varying from 0 to 180°. When an a-particle strikes the
screen, it produces a flash of light.
Nucleus
2
2 Ze cot(q / 2)
Impact parameter b =
4pe0 mv02
Distance of closest approach
2Zeq 2( Ze) (2e)
r0 = =
4pe 0 mv02 4pe0 mv02
Result of Rutherford scattering experiment : Nucleus- is central,
massive, positively charged core, its size of the order of 10–15 m,
It was found that : number of electrons surrounding nucleus is such that atom is
• Most of the a-particles went straight through the gold foil electrically neutral.
and produced flashes on the screen as if there were nothing Unit for nuclear dimension measurement : 1 fermi = 10–15m.
inside gold foil. This suggests that the most part of the BOHR’S ATOMIC (HYDROGEN ATOM) MODEL
atom is empty.
In 1913 Bohr gave his atomic theory primarily to explain, the
• Few particles collided with the atoms of the foil which have spectra of hydrogen and hydrogen-like atoms. His theory,
scattered or deflected through considerable large angles. contained a combination of views from Plank’s quantum theory,
Very few particles even turned back towards source itself. Einstein’s photon concept and Rutherford model of atom. The
Free eBooks on @neetquestionpaper2020

946 Ph ysi cs
Bohr theory can explain, the atomic spectra of hydrogen atom Þ r = 0.529 n2 Å for hydrogen atom and
and hydrogen-like ions such as He+, Li2+, Be3+.......(one electron
n2
ions). But his theory failed to explain, the spectra of more complex r = 0.529 × for hydrogen like ions.
atom and ions. Z
From equation (4) & (1) we obtain,
The basic postulates of Bohr’s model are : Velocity of electron in nth state
(i) The electron moves in circular orbits around the nucleus
under the influence of coulombic force of attraction between n 2h 2 k 2e4 ke 2
v2 = = Þ v=
the electron and the positively charged nucleus (as shown
2 2
m r 2
n h 2 nh
in figure below). e 2
æ C ö 1
or v = =ç ÷ ´ (for hydrogen atom ) ...(6)
– 2e o nh è 137 ø n
e
r æ C öZ
v=ç
+
v
Fc è 137 ÷ø n for hydrogen like ions

C
= 2.19 ´ 10 6 ms -1
137
The total energy of electron is given by
Bohr’s model of hydrogen atom E = K.E. + P.E. = Kinetic energy + Potential energy
(ii) The electron rotates about the nucleus in certain stationary
circular orbits, for which the angular momentum of electron 1 ke ( -e)
= mv 2 +
2 r
about the nucleus is an integral multiple of h = h , where
2p - ke 2 - me 4
E= = ...(7)
h is plank’s constant 2r 8e o n 2 h 2
nh
i.e., Angular Momentum, mvr = = nh ...(1) (Allowed energy state)
2p After substituting numerical values in eqn.(7), we obtain
(where n = 1, 2, 3......... principal quantum number)
(iii) When the electron is in one of its stationary orbits, it does -13.6
E= eV (for hydrogen atom) ...(8)
not radiate energy, hence the atom is stable.These stationary n2
orbits are called allowed orbits.
- 13.6 Z 2
(iv) The atom radiates energy when the electron “jumps” from E= eV/atom for hydrogen like ions.
one allowed stationery state to another. The frequency of n2
radiation follows the condition The lowest energy state, or ground state, corresponds to n = 1 is
hn = Ei – Ef ...(2)
Where Ei and Ef are total energies of initial and final stationary - me 4
E0 = = -13.6eV
states. This difference in energy (Ei -Ef) between two 8e o h 2
allowed stationory states is radiated/absorbed in the form The next state corresponds to n = 2 i.e., first excited state has an
of a packet of electromagnetic energy (hn - one photon of energy, E = –3.4 eV
frequency n) called a photon.
Now we calculate the allowed energies of hydrogen atom, Limitations of Bohr's Model
For moving an electron in a circular orbit the required 1. It could not explain the spectra of atoms containing more
centripetal force is provided by the coulomb force of attraction than one electron.
which acts between nucleus [Ze+, here Z = 1 (atomic number)
for hydrogen atom] & electron (e–), 2. There was no theoretical basis for selecting mvr to be an
2 2 integral multiple of h / 2p .
i.e., mv = k e ...(3) 3. It involved the orbit concept which could not be checked
r 2
r experimentally.
1 4. It could not explain Zeeman & Stark effect and fine lines of
where k = is electrostatic constant & e o is
4pe o spectra.
permittivity of free space. 5. It was against de-Broglie concept and uncertainty principle.
Eliminating v from eqn. (1) and (3) we obtain radius of nth
orbit Keep in Memory

n2 h 2 e n 2 h 2 (where n = 1, 2, 3 .....) 1. Total energy of electron = – Kinetic energy


r= = o ...(4)
mke 2 pme 2 Potential energy
=
Equation (4) gives the radii of various orbits (have discrete 2
values). 2. The reference level for potential energy has been taken as
The smallest radius (also called Bohr radius) corresponds infinity
to n = 1 is 3. The energy gap between two successive levels decreases
2
as the value of n increases
h 4. The radius difference between the successive orbit (or
r0 = 2
@ 0.527 Å ...(5)
mke shells) increases as the value of n increases
Free eBooks on @neetquestionpaper2020

Atoms 947
5. The velocity of electrons around the nucleus goes on This is because in above calculations we assumed that electron
decreasing as n increases revolves around a massive fixed nucleus of mass M. But in reality,
6. The time period of the electron in an orbit T 2 µ r 3 the electron and nucleus each revolve round their common center
of mass i.e., the motion of nucleus cannot be ignored. The
7. Maximum number of spectral lines that can be emitted when correction for nuclear motion amounts to replacing electronic
n ( n - 1) mass m by reduced mass m which is defined as
an electron jumps from n th orbit is
2 mM
m= ...(3)
ENERGY LEVELS AND THE LINES SPECTRA OF m+ M
HYDROGEN ATOM So total energy by taking this correction is
An energy level diagram of the hydrogen atom is shown in figure. -m e4
E= ...(4)
The upper most level corresponding to n® ¥ , represents the
8e o h2 n2
state for which the electron is completely removed from the atom.
If we are dealing with hydrogen like ions such as – He+, Li2+,
¥ Be3+, Be4+ (one electron ions), each can be considered as a system
0 eV
of two charges, the electron of mass m & charge –e & nucleus of
mass M and charge +Ze, where Z is atomic number. The radii of
circular orbits for these one electron ions can be written as
–0.54 eV
e n2 h2 n2h2
–0.85 eV r= o = (n = 1, 2, 3............) ...(5)
Paschen pm Ze 2 mkZe 2
–1.51 eV series and the allowed energies are given by
Balmer
series -m Z 2 e 4
E@ (n = 1, 2, 3.........) ...(6)
–3.4 eV 8e o h 2 n 2
Wavelength Limits in Various Spectral Series of Hydrogen Atom
(i) For Lyman series (lies in ultraviolet region)
–13.6 eV (Ground state) l m ax = 1216 Å , l m in = 912 Å
Here n i = 1, n f = 2, 3...............
Some transitions for Lyman, Balmer & Paschen series are shown.
The quantum numbers are at left & energies of levels are at right. (ii) For Balmer series (lies in visible region)
E = 0 for r = ¥ (Since n = ¥ ) l max = 6563 Å and l min = 3646 Å
If the electron jumps from allowed state ni to allowed state nf, Here n i = 2, n f = 3, 4, 5...............
then frequency of emitted photon is given by
(iii) For Paschen series (lies in infrared region)
E f - Ei me 4 æ 1 1 ö ...(1) l max = 18751Å and l min = 8107 Å
n= = ç - ÷
h 8eo h3 çè ni2 n 2f ÷ø Here n i = 3, n f = 4, 5, 6...............
and the wavelength of emitted photon is (iv) For Brackett series (lies in infrared region)
æ 1 l max = 40477 Å and l min = 14572Å
1 n me4 1 ö
= = ç 2- 2÷ Here n i = 4, n f = 5, 6, 7.......... .....
l c 8e o ch3 çè ni n f ÷ø
(v) For p-fund series (lies in infrared region)
1 æ 1 1 ö l max = 74515 Å and l min = 22768 Å
= Rç - ÷ for hydrogen atom ...(2) Here n i = 5, n f = 6, 7, 8.......... .....
l çè ni2 n 2f ÷ø
Keep in Memory
1 é
2 1 1 ù
and n = = RZ ê 2 - 2 ú ( for H-like atoms) 1. The first line of Lyman series is when electron jumps from
l êë n1 n2 úû 2 ® 1, It is also called a – line
where R = 1.096776 × 10 m–1 is known as Rydberg constant. By
7
The second line of lyman series is when electron jumps
using this expression we can calculate the wavelengths for various from 3 ® 1, It is also called b – line
series (Lyman, Balmer...) in hydrogen spectrum, i.e. The limiting line of lyman series is when electron jumps
(i) Lyman series ni = 1 & nf = 2, 3, 4............... from ¥ ® 1
(ii) Balmer series ni = 2, & nf = 3, 4, 5............... 2. Energy of electrons in different orbits in an atom varies
(iii) Paschen series ni = 3 & nf = 4, 5, 6.............. inversely with the square of the number of orbits. So, energy
(iv) Bracett series ni = 4 & nf = 5, 6, 7............... of electrons increases (decreases in negative) as the orbit
(v) P fund series ni = 5 & nf = 6, 7, 8............... becomes higher.
First three series of hydrogen atom are shown in figure.
But in practice, the value of Rydberg constant varies between 3. If energy of a particular orbit is E for H-atom then its value
for a H-like atom with atomic number Z is given by
R
and R E' = E × Z2.
2
Free eBooks on @neetquestionpaper2020

948 Ph ysi cs
4. If the radius of a particular orbit of H-atom is R then its valu Example 3.
for a H-like atom is given by Ionization potential of hydrogen atom is 13.6 eV. Hydrogen
R atom in the ground state is excited by monochromatic
R' = . radiation of photon energy 12.1 eV. Find the number of
Z
5. If velocity of an electron in a particular orbit of H-atom be v spectral lines emitted by hydrogen atoms according to Bohr’s
then its value for a H-like atom is given by theory
v'= v × Z. Solution :
6. If kinetic energy and potential energy of an electron in a Given that (I.P.)H = 13.6 eV
particular orbit of H-atom be T and V respectively then \ (E1)H = –(I.P.)H = –13.6 eV.
their corresponding values for H-like atom are given by
T' = T × Z2 and V' = V× Z2. After absorbing 12.1 eV energy, the energy becomes
–13.6 eV + 12.1 eV = –1.5 eV.
COMMON DEFAULT This energy corresponds to n = 3 level i.e., hydrogen atoms
û Incorrect. Bohr's formula for spectral lines does not are excited to n = 3 level.
differentiate between isotopes. For example the first line of
Lyman series in hydrogen and deuterium will have same Now following three transitions are possible
wavelength because n = 3 to n = 1, n = 3 to n = 2, and n = 2 to n = 1.
1 é1 1ù Hence no. of spectral lines = 3
= R (12 ) ê 2 - 2 ú
l ë1 2 û Example 4.
ü Correct. The value of R will be different for hydrogen and The ionisation energy of hydrogen atom is 13.6 eV.
deuterium and therefore λ will be different for the two cases. Following Bohr’s theory, find the energy corresponding
In fact λ D < λ H (Q R D > R H ) to a transition between the 3rd and the 4th orbit is
Solution :
Example 1. En = 13.6/n2
A hydrogen atom in the ground state is excited by
radiations of wavelength 975Å. Find (a) the energy state E3 = –(13.9/9) = –1.51 eV
to which the atom is excited. (b) how many lines will be and E4 = – (13.6/16) = –0.85 eV
possible in emission spectrum ? Now E4 – E3 = –0.85 – (–1.51) = 0.66 eV
Solution : Example 5.
(a) l = 975Å = 975 ´ 10 -10 m If the wavelength of the first line of the Lyman series for the
1 é1 1 ù hydrogen atom is 1210 Å, then what will be the wavelength
= R ê 2 - 2ú of the first line of the Balmer series of the hydrogen
l ë1 n û
spectrum?
1 é1 1 ù
\ -10
= 1.1 ´ 10-7 ê 2 - 2 ú Solution :
975 ´ 10 ë1 n û
or n = 4 1 é 1 1 ù
n (n - 1) We know that, = Rê - ú
(b) Q Number of spectral lines (N) = l êë n12 n 22 úû
2
4 ´ (4 - 1) For first line of Lyman series, n1 =1 and n2 = 2
\ N= =6
2 4
1 é1 1 ù 3 R
Possible transition \ = Rê - ú = or R =
4 ® 3, 4 ® 2, 4 ® 1, 3 ® 2, 3 ® 1, 2 ® 1. l1 ë 1 4 û 4 3l 1

Example 2. For first line of Balmer series n 1 =2 and n2 = 3


Find the longest and shortest wavelength when a hydrogen
atom in the ground state is excited by radiations of 1 é1 1ù 5R 5 é 4 ù
wavelength 975Å. \ l = R ê 4 - 9 ú = 36 = 36 ê 3 l ú
2 ë û ë 1û
Solution :
hc 12400 é 1 5 4 1
l= »
-34
Å ëQ h = 6.6 ´ 10 Js ; C = 3 ´ 10 m/s ùû
8 or, = ´ ´
eE E (eV) l 2 36 3 1210
\ For longest wavelength 36 ´ 3 ´ 1210
or, l 2 = = 6434 Å
12400 12400 5´ 4
l max = =
E 4®3 0.66 = 18787.8 Å Example 6.
For smallest wavelength If the electron in hydrogen atom jumps from the third orbit
hc 12400 12400 to second orbit, then find the wavelength of the emitted
l min = = = » 973 Å radiation.
eE E 4®1 12.75
Free eBooks on @neetquestionpaper2020

Atoms 949
Solution : Solution : (a)

é 1 1 æ hc Eö
1 1 ù 1 é1 1ù 5R Eµ and p µ E çè E = , p = ÷ø
= Rê - ú or = Rê - ú = l l c
l 2 2
ëê n1 n 2 úû l ë 4 9 û 36
1 1
Further E µ \ b = c and a =
36 n2 b
\ l=
5R
(1/ 2)2 - (1/ 3)2 5
Example 7. and c = 2
=
1 - (1/ 2) 27
Making use of physical constants given below, calculate
for hydrogen atom 5 27
\ b=c= and a=
(i) time taken by the electron to traverse the first Bohr 27 5
orbit Example 9 :
(ii) Rydberg’s constant. How many different wavelengths may be observed in the
spectrum from a hydrogen sample if the atoms are excited
(e = 1.6 × 10–19 C, m = 9.1 × 10–31 kg., h = 6.6 × 10–34
to states with principal quantum number n ?
joule sec., e 0 = 8.86 × 10 –12 Coulomb 2 /N-m 2 ,
Solution :
v1 = 2.19 × 106 m/sec and r 1 = 0.528 Å)
From the nth state, the atom may go to (n – 1)th state, ….,
Solution : 2nd state or 1st state. So there are (n – 1)th possible transi-
tions starting from the n th sate. The atoms reaching (n –
2pr1 2 ´ 3.14 ´ 0.528 ´ 10 -10
(i) T= = 1)th state may make (n – 2) different transitions. Similarly for
v1 2.19 ´ 10 6 other lower states, the total number of possible transitions
= 1.5 × 10–16 sec are
(ii) Rydberg’s constant R = m e4/8e02c h3 n(n - 1)
(n - 1) + (n - 2) + (n - 3) + .....2 + 1 =
2
(9.1 ´ 10 -31 ) (1.6 ´ 10 -19 ) 4 Example 10.
= In the hydrogen atom, an electron makes a transition
8 ´ (8.86 ´ 10 -12 ) 2 ´ (3 ´ 108 ) ´ (6.6 ´ 10 -34 )3 from n = 2 to n = 1. The magnetic field produced by the
= 1.101 × 10–7 m–1 circulating electron at the nucleus
Example 8 : (a) decreases 16 times (b) increases 4 times
An electron in hydrogen atom first jumps from second ex- (c) decreases 4 times (d) increases 32 times
cited state to first excited state and then from first excited Solution :(d)
state to ground state. Let the ratio of wavelength, momen-
tum and energy of photons emitted in these two cases be a, m0 I e
Q B= and I =
b and c respectively. Then 2r T
1 9 m e 1
(a) c = (b) a = B= 0 Þ Bµ [Q r µ n 2 , T µ n 5 ] ;
a 4 2rT n5
27 4
(c) b = (d) c =
5 9

27.1
Solve following problems with the help of above text and 3. As one considers orbits with higher values of n in a
examples. hydrogen atom, the electric potential energy of the atom
1. Rutherford’s a-particle scattering experiment concludes (a) decreases (b) increases
that (c) remains the same (d) does not increase
(a) there is a heavy mass at centre 4. Which of the following parameters is the same for all
(b) electrons are revolving around the nucleus hydrogen-like atoms and ions in their ground states?
(c) both (A) and (B) (a) Radius of the orbit
(d) None of these (b) Speed of the electron
2. The minimum orbital angular momentum of the electron in (c) Energy of the atom
a hydrogen atom is (d) Orbital angular momentum of the electron
(a) h (b) h/2 5. An electron with kinetic energy 5 eV is incident on a
(c) h/2p (d) h/l hydrogen atom in its ground state. The collision
Free eBooks on @neetquestionpaper2020

950 Ph ysi cs
(a) must be elastic (a) 90º (b) 270º
(b) may be partially elastic (c) 0º (d) 180º
(c) must be completely inelastic 10. It is possible to understand nuclear fission on the basis of
(d) may be completely inelastic the
6. Which of the following in a hydrogen atom is independent (a) meson theory of the nuclear force
of the principal quantum number n? (The symbols have (b) proton-proton cycle
their usual meanings). (c) independent particle model of the nucleus
(a) nn (b) Er (d) liquid drop model of the nucleus
(c) En (d) nr 11. Hydrogen atom excites energy level from fundamental state
7. Ionization energy of a hydrogen-like ion A is greater than to n = 3. Number of spectral lines, according to Bohr, is
that of another hydrogen-like ion B. If r, u, E and L represent (a) 4 (b) 3
the radius of the orbit, speed of the electron, energy of the (c) 1 (d) 2
atom and orbital angular momentum of the electron 12. In the Bohr 's model of a hydrogen atom, the centripetal force
respectively then in ground state is furnished by the coulomb attraction between the proton and
(a) rA > rB (b) uA > uB the electron. If a0 is the radius of the ground state orbit, m is the
(c) EA > EB (d) LA > LB mass and e is charge on the electron and Î0 is the vacuum
8. When a hydrogen atom is raised from the ground state to
permittivity, the speed of the electron is
an excited state
(a) P.E. increases and K.E. decreases e
(b) P.E. decreases and K.E. increases (a) Zero (b) Î0 a 0 m
(c) Both energy (K.E. and P.E.) increases
(d) Both K.E. and P.E. decreases e 4p Î0 a 0 m
9. In Rutherford's a -particle scattering experiment, what will (c) (d)
be correct angle for a scattering for an impact parameter b 4pÎ0 a 0 m e
=0?
ANSWER KEY
1. (a) 2. (c) 3. (b) 4. (d) 5. (a) 6. (b) 7. (b) 8. (a) 9. (d) 10. (d) 11. (b) 12. (c)

Very Short / Short Answer Questions 8. How much energy is required to take an electron from the
ground state to the first excited state?
1. The electron in a given Bohr orbit has a total energy of 9. In Bohr’s theory of H-atom. What is the reason for greater
– 1.5 eV. Calculate its [Delhi Board - 2011 COMPTT.] potential energy than kinetic energy?
(i) kinetic energy 10. What are the drawback’s of Rutherford’s atomic model?
(ii) potential energy 11. The energy levels of an atom are as shown in figure. Which
2. The ground state energy of hydrogen atom is –13.6 eV. of these transitions will result in emission of photon of
What are the kinetic and potential energies of electron in wavelength 275 nm?
this state? [Outside Delhi - 2012] 0 eV
3. Write the expression for Bohr’s radius in hydrogen atom. –2 eV
[Delhi Board - 2010] A
4. The radius of inermost electron orbit of a hydrogen atom is –4.5 eV
B C
5.3 × 10–11 m. What is the radius of orbit in the second
excited state? [Delhi Board - 2010] D
5. The energy of the electron, in the ground state of hydrogen, –10 eV
is –13.6 eV. Calculate the energy of the photon that would 12. What is the difference between Rutherford’s model and
be emitted if the electron were to make a transition Bohr’s model of an atom?
corresponding ot the emission of the first line of the 13. The energy of H-atom in its ground state is –13.6 eV. Find
(i) Lyman series the energy of states corresponding to states whose quantum
(ii) Balmer series of the hydrogen spectrum. number are 2 and 3.
[Outside Delhi - 2009 COMPTT.] Long Answer Questions
6. Which spectral series is in the visible region?
7. Explain why spectrum of hydrogen has many lines, although 14. (a) Using Bohr’s theory of hydrogen atom, derive the
H-atom contains only one electron. expression for the total energy of the electron in the
stationary states of the atom.
Free eBooks on @neetquestionpaper2020

Atoms 951
(b) If electron in the atom is replaced by a particle (muon) (c) a-particles are nuclei of Helium atoms.
having the same charge but mass about 200 times as (d) Atoms can exist with a series of discrete energy levels.
that of the electron to form a muonic atom, how would 22. The proof of quantization of energy states in an atom is
(i) the radius and obtained by the experiment performed by
(ii) the ground state energy of this be affected? (a) Thomson (b) Millikan
(c) Calculate the wavelength of the first spectral line in (c) Rutherford (d) Franck and Hertz
the corresponding Lyman series of this atom. 23. The angular speed of the electron in the nth orbit of Bohr
[Delhi Board - 2012 COMPTT.] hydrogen atom is
15. (a) Using de Broglie’s hypothesis, explain with the help (a) directly proportional to n
of a suitable diagram, Bohr’s second postulate of (b) inversely proportional to n
quantization of energy levels in a hydrogen atom.
(c) inversely proportional to n 2
(b) The ground state energy of hydrogen atom is – 13.6 eV.
(d) inversely proportional to n3
What are the kinetic and potential energies of the
24. According to Bohr’s model of hydrogen atom
electron in this state? [Outside Delhi - 2011]
(a) the linear velocity of the electron is quantised.
16. State the basic assumption of the Rutherford model of the
(b) the angular velocity of the electron is quantised.
atom. Explain, in brief, why this model cannot account for
(c) the linear momentum of the electron is quantised.
the stablility of an atom.
(d) the angular momentum of the electron is quantised.
[Delhi Board - 2010 COMPTT.]
25. As the quantum number increases, the difference of energy
17. What do you understand by energy level diagram? Discuss
between consecutive energy levels
the various series of hydrogen spectrum with its help.
(a) remain the same
18. Calculate the approximate quantum number (n) for an
(b) increases
electron of hydrogen atom revoloving in an orbit of radius
(c) decreases
1mm.
(d) sometimes increases and sometimes decreases.
19. At what speed must an electron revolve around the nucleus
26. Line spectrum is obtained whenever the incandescent
of H-atom so that it does not collapse into the nucleus by
vapours at low pressure of the excited substance are in
electrostatics attration?
their
(Given : me = 9.1 × 10–31 Kg, r = 0.5 × 10–10 m,
(a) atomic state (b) molecular state
e = 1.6 × 10–19 C)
(c) nuclear state (d) None of these
Multiple Choice Questions
27. According to the Bohr theory of H-atom, the speed of the
20. In Millikan’s oil drop experiment, an oil drop is observed to electron, its energy and the radius of its orbit varies with
move vertically upward. The upward motion of the drop is the principal quantum number n, respectively, as
due to 1 2 1 1 2
(a) gravity (b) viscosity (a) ,n , (b) n, 2 , n
n 2
(c) buoyancy (d) electric field n n
21. The following statements are all true. Which one did 1 1 1 1 2
(c) n, 2 , 2 (d) , ,n
Rutherford consider to be supported by the results of n n n n2
experiments in which a-particles were scattered by gold 28. In a hydrogen atom following the Bohr’s postulates the
foil? product of linear momentum and angular momentum is
(a) The nucleus of an atom is held together by forces proportional to (n)x where ‘n’ is the orbit number. Then ‘x’
which are much str onger than electrical or is
gravitational forces. (a) 0 (b) 2
(b) The force of repulsion between an atomic nucleus (c) –2 (d) 1
and an a-particle varies with distance according to
inverse square law.

1. In terms of Bohr radius r0, the radius of the second Bohr orbit 3. When hydrogen atom is in its first excited level, it’s radius is
of a hydrogen atom is given by [CBSE PMT 1992] [CBSE PMT 1997]
(a) 4 r0 (b) 8 r0 (a) four times, it ground state radius
(b) twice times, it ground state radius
(c) 2 r0 (d) 2 r0
(c) same times, it ground state radius
2. Doubly ionised helium atom and hydrogen ions are (d) half times, it ground state radius.
accelerated, from rest, through the same potential 4. The energy of hydrogen atom in nth orbits is En, then the
difference. The ratio of final velocities of helium and energy in n th orbit of single ionised helium atom will be :
hydrogen is [CBSE PMT 1994] [CBSE PMT 2001]
(a) 1 : 2 (b) 2 :1 (a) 4 En (b) En/4
(c) 2 En (d) En/2
(c) 1 : 2 (d) 2 : 1
Free eBooks on @neetquestionpaper2020

952 Ph ysi cs
5. Ionization potential of hydrogen atom is 13.6 eV. Hydrogen 16. If 13.6 eV energy is required to ionize the hydrogen atom,
atoms in the ground state are excited by monochromatic then the energy required to remove an electron from n = 2 is
radiation of photon energy 12.1 eV. The spectral lines [AIEEE 2002]
emitted by hydrogen atom according to Bohr’s theory will (a) 10.2 eV (b) 0 eV (c) 3.4 eV (d) 6.8 eV.
be [CBSE PMT 2006] 17. Which of the following atoms has the lowest ionization
(a) one (b) two (c) three (d) four potential ? [AIEEE 2003]
6. Which of the following transitions in hydrogen atoms emit 14 133 40 16
photons of highest frequency? [CBSE PMT 2006] (a) N (b) Cs (c) Ar (d) O
7 55 18 8
(a) n = 1 to n = 2 (b) n = 2 to n = 6 18. The wavelengths involved in the spectrum of deuterium
(c) n = 6 to n = 2 (d) n = 2 to n = 1 2
7. The energy of a hydrogen atom in the ground state is ( D) are slightly different from that of hydrogen
1
– 13.6 eV. The energy of a He+ ion in the first excited spectrum, because [AIEEE 2003]
state will be [CBSE PMT 2010] (a) the size of the two nuclei are different
(a) –13.6 eV (b) – 27.2 eV (b) the nuclear forces are different in the two cases
(c) – 54.4 eV (d) – 6.8 eV (c) the masses of the two nuclei are different
1 2 (d) the attraction between the electron and the
8. An alpha nucleus of energy mv bombards a heavy nucleus is different in the two cases
2
nuclear target of charge Ze. Then the distance of closest 19. If the binding energy of the electron in a hydrogen atom
approach for the alpha nucleus will be proportional to is 13.6eV, the energy required to remove the electron
[CBSE PMT 2010] from the first excited state of Li + + is [AIEEE 2003]
1 1 1 (a) 30.6 eV (b) 13.6 eV (c) 3.4 eV (d) 122.4 eV
(a) (b) v 2 (c) (d) 20. The diagram shows the energy levels for an electron in a
Ze m v4
certain atom. Which transition shown represents the
9. The wavelength of the first line of Lyman series for emission of a photon with the most energy? [AIEEE 2005]
hydrogen atom is equal to that of the second line of Balmer
(a) IV n=4
series for a hydrogen like ion. The atomic number Z of n=3
hydrogen like ion is [CBSE PMT 2011] (b) III
(a) 3 (b) 4 (c) 1 (d) 2 n=2
10. An electron in the hydrogen atom jumps from excited state (c) II
n to the ground state. The wavelength so emitted (d) I n=1
illuminates a photosensitive material having work function I II III IV
2.75 eV. If the stopping potential of the photoelectron is 1 2
21. An alpha nucleus of energy mv bombards a heavy
10 V, the value of n is [CBSE PMT 2011 (M)] 2
(a) 3 (b) 4 (c) 5 (d) 2 nuclear target of charge Ze. Then the distance of closest
11. Out of the following which one is not a possible energy for approach for the alpha nucleus will be proportional to
a photon to be emitted by hydrogen atom according to [AIEEE 2006]
Bohr’s atomic model? [CBSE PMT 2011 (M)]
(a) 1.9 eV (b) 11.1 eV (c) 13.6 eV (d) 0.65 eV 1 1 1
(a) v 2 (b) (c) 4 (d)
12. The transition from the state n = 3 to n = 1 in a hydrogen m v Ze
like atom results in ultraviolet radiation. Infrared radiation 22. Suppose an electron is attracted towards the origin by a
will be obtained in the transition from
[CBSE PMT 2012 (M)] k
force where ‘k’ is a constant and ‘r’ is the distance of the
(a) 2 ® 1 (b) 3 ® 2 (c) 4 ® 2 (d) 4 ® 3 r
13. Electron in hydrogen atom first jumps from third excited electron from the origin. By applying Bohr model to this
state to second excited state and then from second excited system, the radius of the nth orbital of the electron is found
to the first excited state. The ratio of the wavelength l1 : l2 to be ‘r n’ and the kinetic energy of the electron to be ‘Tn’.
emitted in the two cases is [CBSE PMT 2012 (S)] Then which of the following is true?
(a) 7/5 (b) 27/20 (c) 27/5 (d) 20/7 [AIEEE 2008]
14. An electron of a stationary hydrogen atom passes from
1 2
the fifth energy level to the ground level. The velocity (a) Tn µ 2 , rn µ n (b) Tn independent of n, rn µ n
that the atom acquired as a result of photon emission will n
be [CBSE PMT 2012 (S)] 1 1
(c) Tn µ , rn µ n (d) Tn µ , rn µ n 2
24hR 25hR 25m 24 m n n
(a) (b) (c) (d) 23. The transition from the state n = 4 to n = 3 in a hydrogen like
25m 24m 24hR 25hR
15. The ratio of longest wavelengths corresponding to Lyman atom results in ultraviolet radiation. Infrared radiation will
and Blamer series in hydrogen spectrum is [ NEET -2013] be obtained in the transition from [AIEEE 2009]
3 7 9 5 (a) 2 ® 1 (b) 3 ® 2
(a) (b) (c) (d) (c) 4 ® 2 (d) 5 ® 4
23 29 31 27
Free eBooks on @neetquestionpaper2020

Atoms 953
24. If a source of power 4kW produces 1020photons/second, After a time interval of the order of micro second another
the radiation belongs to a part of the spectrum called photon collides with same hydrogen atom inelastically with
[AIEEE 2010] an energy of 15 eV. What will be observed by the detector?
(a) X -rays (b) ultraviolet rays (a) One photon of energy 10.2 eV and an electron of
(c) microwaves (d) g -rays energy 1.4 eV [IIT JEE 2005s]
(b) 2 photon of energy of 1.4 eV
25. Energy required for the electron excitation in Li ++ from
(c) 2 photon of energy 10.2 eV
the first to the third Bohr orbit is [AIEEE 2011]
(d) One photon of energy 10.2 eV and another photon of
(a) 36.3 eV (b) 108.8 eV 1.4 eV
(c) 122.4 eV (d) 12.1 eV 30. The largest wavelength in the ultraviolet region of the
26. Hydrogen atom is excited from ground state to another hydrogen spectrum is 122 nm. The smallest wavelength
state with principal quantum number equal to 4. Then the in the infrared region of the hydrogen spectrum (to the
number of spectral lines in the emission spectra will be nearest integer) is [IIT-JEE 2007]
[AIEEE 2012] (a) 802 nm (b) 823 nm (c) 1882 nm (d) 1648 nm
(a) 2 (b) 3 (c) 5 (d) 6 31. The wavelength of the first spectral line in the Balmer series
27. In a hydrogen like atom electron makes transition from of hydrogen atom is 6561 A°. The wavelength of the second
an energy level with quantum number n to another with spectral line in the Balmer series of singly-ionized helium
quantum number (n – 1). If n>>1, the frequency of atom is [IIT-JEE 2011]
radiation emitted is proportional to [JEE Main 2013] (a) 1215 A° (b) 1640 A°
1 (c) 2430 A° (d) 4687 A°
1 1 1
(a) (b) 2
(c) 3 (d) 32. The radius of the orbit of an electron in a Hydrogen-like
n n n /2 n3 atom is 4.5 a0, where a0 is the Bohr radius. Its orbital angular
28. Ka wavelength emitted by an atom l is given by an atom
of atomic number Z = 11 is l. Find the atomic number for 3h
momentum is . It is given that h is Planck constant and
an atom that emits Ka radiation with wavelength 4l. 2p
(a) Z = 6 (b) Z = 4 [IIT JEE 2005s] R is Rydberg constant. The possible wavelength(s), when
the atom de-excites, is (are) [JEE Adv. 2013[
(c) Z = 11 (d) Z = 44
29. A photon collides with a stationary hydrogen atom in ground 9 9 9 4
(a) (b) (c) (d)
state inelastically. Energy of the colliding photon is 10.2 eV. 32R 16R 5R 3R

1. In an atom, the two electrons move round the nucleus in 6. A 15.0 eV photon collides with and ionizes a hydrogen
circular orbits of radii R and 4R. The ratio of the time taken atom. If the atom was originally in the ground state
by them to complete one revolution is (ionization potential =13.6 eV), what is the kinetic energy of
(a) 1/4 (b) 4/1 (c) 8/1 (d) 1/8 the ejected electron?
2. The ratio of the energies of the hydrogen atom in its first to (a) 1.4 eV (b) 13.6 eV (c) 15.0 eV (d) 28.6 eV
second excited states is 7. The ratio of areas between the electron orbits for the first
(a) 1/4 (b) 4/9 (c) 9/4 (d) 4 excited state to the ground state for the hydrogen atom is
3. For the principal quantum number n = 4, the total number of (a) 2 : 1 (b) 4 : 1 (c) 8 : 1 (d) 16 : 1
different possible values of the azimuthal quantum number 8. The angular momentum of the electron in hydrogen atom in
l, is the ground state is
(a) 2 (b) 8 (c) 4 (d) 3 h h h
4. With increasing quantum number, the energy difference (a) 2h (b) (c) (d)
2 2p 4p
between adjacent energy levels in atoms 9. If e is electronic charge, the equivalent current for an electron
(a) decreases revolving with a frequency n in the ground state of
(b) increases hydrogen atom is
(c) remains constant e e
(d) decreases for low Z and increases for high Z (a) 2pne (b) (c) (d) en
5. In a hypothetical Bohr hydrogen atom, the mass of the electron pn 2pn
10. The potential energy associated with an electron in the
is doubled. The energy E '0 and radius r0' of the first orbit will orbit
be (r0 is the Bohr radius) (a) increases with the increases in radii of the orbit
(a) –11.2 eV (b) –6.8 eV (b) decreases with the increase in the radii of the orbit
(c) –13.6 eV (d) –27.2 eV (c) remains the same with the change in the radii of the orbit
(d) None of these
Free eBooks on @neetquestionpaper2020

954 Ph ysi cs
11. The principle of LASER action involves 22. The ratio between Bohr radii is
(a) amplification of particular frequency emitted by the (a) 1 : 2 : 3 (b) 2 : 4 : 6 (c) 1 : 4 : 9 (d) 1 : 3 : 5
system 23. The kinetic energy of the electorn in an orbit of radius r in
(b) population inversion hydrogen atom is (e = electronic charge)
(c) stimulated emissin e2 e2
e2 e2
(d) All of the above. (a) (b) (c) (d)
12. In which of the following series, does the 121.5 nm line of r2 2r r 2r 2
24. According to the Rutherford’s atomic model, the electrons
the spectrum of the hydrogen atom lie ?
inside the atom are
(a) Lyman series (b) Balmer series (a) stationary (b) not stationary
(c) Paschen series (d) Brackett series. (c) centralized (d) None of these
13. For the azimuthal quantum number l = 3, the total number of 25. According to classical theory, the circular path of an
different possible values of the magnetic azimuthal quantum electron in Rutherford atom is
number m1, is (a) spiral (b) circular
(a) 3 (b) 4 (c) 5 (d) 7 (c) parabolic (d) straight line
14. Energy levels, A, B, C of a certain atom correspond to 26. Rutherford’s a-particle experiment showed that the atoms
increasing values of energy i.e. EA < EB < EC. If l1, l2, l3 are have
the wavelengths of radiations corresponding to the (a) Proton (b) Nucleus
transitions C to B, B to A and C to A respectively, which of (c) Neutron (d) Electrons
the following relations is correct ? 27. Electrons in the atom are held to the nucleus by
ll (a) coulomb’s force (b) nuclear force
(a) l 3 = l1 + l 2 (b) l 3 = 1 2 (c) vander waal’s force (d) gravitational force
l1 + l 2
28. Which of the following series in the spectrum of hydrogen
(c) l1 + l 2 + l 3 = 0 (d) l33 = l21 + l22 atom lies in the visible region of the electromagnetic
15. Consider the spectral line resulting from the transition spectrum?
n = 2 ® n = 1 in the atoms and ions given below. The (a) Paschen series (b) Balmer series
shortest wavelength is produced by (c) Lyman series (d) Brackett series
(a) hydrogen atom (b) deuterium atom 29. The ratio of maximum to minimum wavelength in Balmer
(c) singly ionized Helium (d) doubly ionised Lithium series is
16. A hydrogen atom emits a photon corresponding to an (a) 3 : 4 (b) 1 : 4 (c) 5 : 36 (d) 5 : 9
electron transition from n = 5 to n = 1. The recoil speed of 30. What element has ka line of wavelength 1.785 Å?
hydrogen atom is almost
R = 109737 cm-1.
(a) 10–4 ms–1 (b) 2 × 10–2 ms–1
–1 (a) Platinum (b) Zinc (c) Iron (d) Cobalt
(c) 4 ms (d) 8 × 102 ms–1
31. If the ka radiation of Mo (Z = 42) has a wavelength of
17. In Bohr’s model of the hydrogen atom, let R, V, T and E 0.71Å. Calculate the wavelength of the corresponding
represent the radius of the orbit, speed of the electron, time radiation of Cu (Z = 29).
period of revolution of electron and the total energy of the (a) 1.52Å (b) 2.52Å (c) 0.52Å (d) 4.52Å
electron respectively. The quantity proportional to the 32. The angular momentum of electron in n th orbit is given by
quantum number n is
(a) VR (b) E (c) r (d) T h h 2 h
(a) nh (b) (c) n (d) n
18. Let n1 be the frequency of the series limit of the Lyman series, 2 pn 2p 2p
n2 be the frequency of the first line of the Lyman series, and 33. The extreme wavelengths of Paschen series are
n3 be the frequency of the series limit of the Balmer series. (a) 0.365 mm and 0.565 mm (b) 0.818 mm and 1.89 mm
(a) n1 – n2 = n3 (b) n2 – n1 = n3 (c) 1.45 mm and 4.04 mm (d) 2.27 mm and 7.43 mm
34. The third line of Balmer series of an ion equivalnet to
(c) n3 = 1 ( n1 + n2 ) (d) n1 + n2 = n3 hydrogen atom has wavelength of 108.5 nm. The ground
2 state energy of an electron of this ion will be
19. The approximate value of quantum number n for the circular (a) 3.4 eV (b) 13.6 eV
orbit of hydrogen 0.0001 mm in diameter is (c) 54.4 eV (d) 122.4 eV
(a) 1000 (b) 60 (c) 10000 (d) 31 35. The first line of Balmer series has wavelength 6563 Å. What
20. The ionization energy of Li++ is equal to will be the wavelength of the first member of Lyman series
(a) 9hcR (b) 6hcR (c) 2hcR (d) hcR. (a) 1215.4 Å (b) 2500 Å
21. The Rutherford a-particle experiment shows that most of (c) 7500 Å (d) 600 Å
the a-particles pass through almost unscattered while some 36. Hydrogen atom excites energy level from fundamental state
are scattered through large angles. What information does to n = 3. Number of spectral lines according to Bohr, is
it give about the structure of the atom? (a) 4 (b) 3 (c) 1 (d) 2
(a) Atom is hollow. 37. The ratio of the longest to shortest wavelengths in Brackett
(b) The whole mass of the atom is concentrated in a small series of hydrogen spectra is
centre called nucleus
(c) Nucleus is positively charged 25 17 9 4
(a) (b) (c) (d)
(d) All the above 9 6 5 3
Free eBooks on @neetquestionpaper2020

Atoms 955
38. In a Rutherford experiment, the number of particles scattered æf ö
at 90° angle are 28 per minute then number of scattered log ç n ÷
è f1 ø
particles at an angle 60° and 120° will be
(a) 117 per minute, 25 per minute (c) (d) Both (a) and (b)
(b) 50 per minute, 12.5 per minute O log n
(c) 100 per minute, 200 per minute 43. Excitation energy of a hydrogen like ion in its excitation
(d) 112 per minute, 12.4 per minute state is 40.8 eV. Energy needed to remove the electron from
39. In Hydrogen spectrum, the wavelength of Ha line is 656 the ion in ground state is
nm, whereas in the spectrum of a distant galaxy, Ha line (a) 54.4 eV (b) 13.6 eV (c) 40.8 eV (d) 27.2 eV
wavelength is 706 nm. Estimated speed of the galaxy with 44. The ionisation potential of H-atom is 13.6 V. When it is
respect to earth is excited from ground state by monochromatic radiations of
(a) 2 × 108 m/s (b) 2 × 107m/s 970.6 Å, the number of emission lines will be (according to
(c) 2 × 106 m/s (d) 2 × 105 m/s Bohr’s theory)
40. The energy levels of the hydrogen spectrum is shown in (a) 10 (b) 8 (c) 6 (d) 4
figure. There are some transitions A, B, C, D and E. 45. Electrons in a certain energy level n = n1, can emit 3 spectral
Transition A, B and C respectively represent lines. When they are in another energy level, n = n 2. They
n=¥ – 0.00 eV can emit 6 spectral lines. The orbital speed of the electrons
in the two orbits are in the ratio of
n=6 – 0.36 eV (a) 4 : 3 (b) 3 : 4 (c) 2 : 1 (d) 1 : 2
n=5 – 0.54 eV 46. The energy of electron in the nth orbit of hydrogen atom is
n=4 – 0.85 eV
C -13.6
n=3
B D – 1.51 eV expressed as E n = eV. The shortest and longest
n=2 – 3.39 eV n2
wavelength of Lyman series will be
A E (a) 910 Å, 1213 Å (b) 5463 Å, 7858 Å
(c) 1315 Å, 1530 Å (d) None of these
n=1 – 13.5 eV 47. A hydrogen atom in its ground state absorbs 10.2 eV of
(a) first member of Lyman series, third spectral line of energy. The orbital angular momentum is increased by
Balmer series and the second spectral line of Paschen (a) 1.05 × 10–34 J-s (b) 3.16 × 10–34 J-s
series –34
(c) 2.11 × 10 J-s (d) 4.22 × 10–34 J-s
(b) ionization potential of hydrogen, second spectral line 48. Taking Rydberg’s constant RH = 1.097 × 107m, first and
of Balmer series, third spectral line of Paschen series second wavelength of Balmer series in hydrogen spectrum
(c) series limit of Lyman series, third spectral line of Balmer is
series and second spectral line of Paschen series (a) 2000 Å, 3000 Å (b) 1575 Å, 2960 Å
(d) series limit of Lyman series, second spectral line of (c) 6529 Å, 4280 Å (d) 6552 Å, 4863 Å
Balmer series and third spectral line of Paschen series 257
49. If the atom 100Fm follows the Bohr model and the radius
41. Which of the following statements are true regarding Bohr’s
model of hydrogen atom? of 100Fm257 is n times the Bohr radius, then find n.
(I) Orbiting speed of electron decreases as it shifts to (a) 100 (b) 200 (c) 4 (d) 1/4
discrete orbits away from the nucleus 50. In Rutherford scattering experiment, the number of
(II) Radii of allowed orbits of electron are proportional to a-particles scattered at 60° is 5 × 106. The number of
the principal quantum number a-particles scattered at 120° will be
3
(III) Frequency with which electrons orbit around the (a) 15 × 106 (b) × 106
nucleus in discrete orbits is inversely proportional to 5
5
the cube of principal quantum number (c) × 106 (d) None of these
9
(IV) Binding force with which the electron is bound to the +
51. The energy of He in the ground state is – 54.4 eV, then the
nucleus increases as it shifts to outer orbits energy of Li++ in the first excited state will be
Select correct answer using the codes given below. (a) – 30.6 eV (b) 27.2 eV
Codes : (c) – 13.6 eV (d) – 27.2 eV
(a) I and II (b) II and IV 52. The wavelength of radiation is l0 when an electron jumps
(c) I, II and III (d) II, III and IV from third to second orbit of hydrogen atom. For the electron
42. If in hydrogen atom, radius of n th Bohr orbit is rn, frequency to jump from the fourth to the second orbit of the hydrogen
of revolution of electron in n th orbit is fn, choose the correct atom, the wavelength of radiation emitted will be
option. 16 20 27 25
(a) l 0 (b) l 0 (c) l0 (d) l0
rn 25 27 20 16
ær ö
lo g ç n ÷ 53. If the frequency of Ka X-rays emitted from the element with
è r1 ø
atomic number 31 is n, then the frequency of Ka X-rays
(a) (b) emitted from the element with atomic number 51 would be
5 51 25 9
O n O log n (a) n (b) n (c) n (d) n
3 31 9 25
Free eBooks on @neetquestionpaper2020

956 Ph ysi cs
54. A hydrogen atom is in ground state. Then to get six lines in Statement-2 : According to classical physics all moving
emission spectrum, wavelength of incident radiation should be electrons radiate.
(a) 800 Å (b) 825 Å (c) 975 Å (d) 1025 Å 61. Statement 1 : Between any two given energy levels, the
55. In the hydrogen atom, an electron makes a transition from number of absorption transitions is always less than the
n = 2 to n = 1. The magnetic field produced by the circulating number of emission transitions.
electron at the nucleus Statement 2 : Absorption transitions start from the lowest
(a) decreases 16 times (b) increases 4 times energy level only and may end at any higher energy level.
(c) decreases 4 times (d) increases 32 times But emission transitions may start from any higher energy
56. One of the lines in the emission spectrum of Li2+ has the level and end at any energy level below it.
same wavelength as that of the 2nd line of Balmer series in 62. Statement 1 : In Lyman series, the ratio of minimum and
hydrogen spectrum. The electronic transition corresponding 3
to this line is n = 12 ® n = x. Find the value of x. maximum wavelength is .
4
(a) 8 (b) 6 (c) 7 (c) 5 Statement 2 : Lyman series constitute spectral lines
57. The wavelength Ka of X-rays for two metals ‘A’ and ‘B’ are corresponding to transition from higher energy to ground
4 1 state of hydrogen atom.
and respectively, where ‘R’ is Rydbergg
1875R 675R Directions for Qs. (63 to 66) : Read the following passage
constant. Find the number of elements lying between A carefully and answer the questions that follows:
and B according to their atomic numbers
PASSAGE
(a) 3 (b) 1 (c) 4 (d) 5
According to Thomson's model, every atom consists of a
Directions for Qs. (58 to 62) : Each question contains
positively charged sphere of radius 10–10m in which entire mass
STATEMENT-1 and STATEMENT-2. Choose the correct answer
and positive charge of the atom are uniformly distributed. Inside
(ONLY ONE option is correct ) from the following.
the sphere, electrons are embedded like seeds in a watermelon.
(a) Statement-1 is True, Statement-2 is True; Statement-2 is a
According to Rutherford, entire positive charge and mass of the
correct explanation for Statement -1
atom are concentrated in a tiny central core of the atom, which is
(b) Statement-1 is True, Statement -2 is True; Statement-2 is
called atomic nucleus. Size of nucleus » 10–15. The nucleus
NOT a correct explanation for Statement - 1
contains protons and neutrons. Negatively charged electrons
(c) Statement-1 is True, Statement- 2 is False
revolve around the nucleus in circular orbits.
(d) Statement-1 is False, Statement -2 is True
58. Statement-1 : Balmer series lies in the visible region of 63. The size of atom is of the order of
electromagnetic spectrum. (a) 10–14 m (b) 10–12 m (c) 10–10 m (d) 1010 m
1 é1 1 ù 64. The size of atomic nucleus is of the order of
Statement-2 : = R ê 2 - 2 ú , where n = 3, 4, 5. (a) 10–15 m (b) 10–10 m (c) 1014 m (d) 1010 m
l ë2 n û
59. Statement-1 : The force of repulsion between atomic nucleus 65. The ratio of atomic volume to nuclear volume is of the order
and a-particle varies with distance according to inverse of
square law. (a) 1015 (b) 10–12 (c) 108 (d) 1010
Statement-2 : Rutherford did a-particle scattering experiment. 66. Nucleon is a common name for
60. Statement-1 : Bohr had to postulate that the electrons in
stationary orbits around the nucleus do not radiate. (a) Proton and electron (b) electron and neutron
(c) neutron only (d) neutron and proton

Exercise 27.1 8. (a) In excited state, r increases,


Ze 2
K.E. = decreases
1. (a) Heavy mass at the centre of atom is responsible for 2r
large angle scattering of alpha particles Ze 2
2. (c) The orbital angular momentum of electron in hydorgen P.E. = - , becomes less negative i.e. more positive.
r
nh 9. (d) When b = 0, scattering angle, q = 180º
atom is , where n is Principal Quantum number..
2p 10. (d) Nuclear fission is understood in terms of liquid drop
4. (d) The orbital angular momentum of electron is model of nucleus
independent of mass of orbiting particle & mass of n ( n - 1) 3 (3 - 1)
11. (b) No. of spectral lines = = =3
nuclei. 2 2
ke2 n 2-h 2 1 v me4 12. (c) Centripetal force = force of attraction of nucleus on
6. (b) v = ,r = , n = = , E =
electron
nh- mke2 T 2 pr 8e o n 2 h 2
Free eBooks on @neetquestionpaper2020

Atoms 957

mv 2 1 e2 e 1 2 1 qa Ze
= v= mv =
a0 4 p e o a 20 4p e o m a 0 2 4 pe0 r0

Exercise 1 : NCERT Based Questions where r0 is the distance of closest approach


6. Balmer series. 2 qa Ze
r0 =
8. 10.2 eV 4p e0 mv 2
9. Electron revolving in orbit is bound to the nucleus in 1 1
stationary orbit and cannot leave the nucleus on its own. Þ r0 µ Ze µ qa µ µ
11. The wavelength of emitted photon is 275 nm which would m v2
have an energy, E = 4.5 eV. Hence, correct option is (c).
13. E2 = – 3.4 eV 9. (d) For first line of Lyman series of hydrogen
E3 = – 1.51 eV hc æ1 1 ö
= Rhc ç - ÷
18. n 2 @ 4344 l1 è1 2
22 ø
19. 2.25 × 106 m/s. For second line of Balmer series of hydrogen like ion
20. (d) 21. (b) 22. (d) 23. (d) 24. (d)
hc æ 1 1 ö
25. (c) 26. (a) 27. (d) 28. (a) = Z2 Rhc ç - ÷
l2 è2 2
42 ø
Exercise 2 : PAST Competition MCQs
By question, l1 = l2
1. (a) As r µ n 2 , therefore, radius of 2nd Bohr’s orbit = 4 r 0
æ1 1 ö 2 æ1 1 ö
1 2q V q Þ ç - ÷ = Z ç - ÷ or Z = 2
2. (a) q V = m v 2 or v = i.e. v µ è1 2 ø è 4 16 ø
2 m m 10. (b) KEmax = 10eV
f = 2.75 eV
v He q He m H 2e m 1
\ = ´ ´ = Total incident energy
vH q H m He = e 4m 2 E = f + KEmax = 12.75 eV
3. 2
(a) rn= ro.n , where ro is radius of ground-state & rn is radius \ Energy is released when electron jumps from the
of nth state. (For first excited state n = 2). excited state n to the ground state.
4. (a) Because energy in n th orbit En µ Z2 for hydrogen Q E4 – E1 = {– 0.85 – (–13.6) ev}
Z = 1, and for singly ionised helium Z = 2. = 12.75eV
\ value of n = 4
5. (c) Total energy of electron in excited state = –13.6 + 12.1 11. (b) Obviously, difference of 11.1eV is not possible.
= –1.5 eV, which corresponds to third orbit. The possible
spectral lines are when electron jumps from orbit 3rd to –0.58eV
2nd ; 3rd to 1st and 2nd to 1st. –0.85eV
6. (d) We have to find the frequency of emitted photons. –1.51eV 12.09eV n=2
For emission of photons the transition must take place –3.4eV 10.2eV
from a higher energy level to a lower energy level which –13.6eV n=1
are given only in options (c) and (d). 1
Frequency is given by 12. (d) Q The frequency of the transition v µ 2 , when
n
æ 1 1 ö n = 1, 2, 3.
hn = -13.6 ç 2 - 2 ÷ 13. (c)
çn ÷
è 2 n1 ø n=3
For transition from n = 6 to n = 2, Case (I)
-13.6 æ 1 1 ö 2 æ 13.6 ö
n1 = ç 2 - 2 ÷ = ´ç
Energy states

÷
h è6 2 ø 9 è h ø
For transition from n = 2 to n = 1, n=2
Case (II)
-13.6 æ 1 1 ö 3 æ 13.6 ö
n2 = ç - ÷ = ´ç ÷.
h è 22 12 ø 4 è h ø II
\ n1 < n 2 n=1 1
7. (a) Energy of a H-like atom in it's nth state is given by The wave number ( n) of the radiation =
2 13.6 l
En = - Z ´ 2 eV é1 ù
n 1
For, first excited state of He+, n = 2, Z = 2 = R¥ ê - ú
2 2
4 êë n1 n2 úû
\ EHe + = - 2 ´13.6 = -13.6 eV Now for case (I) n1 = 3, n2 = 2
2
8. (c) Kinetic energy of alpha nucleus is equall to 1 é 1 -1ù
= R¥ ê , R = Rydberg constant
electrostatic potential energy of the system of the alpha l1 ë 9 4 úû ¥
particle and the heavy nucleus. That is,
Free eBooks on @neetquestionpaper2020

958 Ph ysi cs
1 é 4 - 9 ù -5R¥ - 36 21. (b) Work done to stop the a particle is equal to K.E.
= R¥ ê = l1 =
l1 ë 36 úû 36 Þ 5 R¥ 1
\ qV = mv 2 Þ q ´
K ( Ze ) 1
= mv 2
2 r 2
1 é 1 1 ù -3R¥
= R¥ ê - ú = 2(2e)K(Ze) 4KZe 2
l2 ë 4 1û 4 Þr= =
2
mv mv 2
-4 1 1
l2 = Þrµ and r µ .
3 R¥ 2 m
v
l1 -36 3R¥ k k mv 2
Þ = ´ 22. (b) When F =
= centripetal force, then =
l 2 5R¥ -4 r r r
Þ mv2 = constat Þ kinetic energy is constant
l1 27 Þ T is independent of n.
=
l2 5 23. (d) l IR > l UV also wavelength of emitted radiation
14. (a) For emission, the wave number of the radiation is 1
lµ .
given as DE
æ 1 24. (a) Power, P = nhn, n = no, of photons per second
1 1ö
= Rz 2 ç 2 - 2 ÷ P
l è n1 n2 ø Þ n=
nh
R = Rydberg constant, Z = atomic number 4 ´ 103
= 20 = 6 × 1016 Hz
æ1 1ö æ 1ö 1 24 10 ´ 6.63 ´10 -34
= R çè 2 - 2 ÷ø = R çè 1 - ÷ø Þ = R 25. (b) Energy of excitation,
1 5 25 l 25
linear momentum æ 1 1ö
D E = 13.6 p2 çè h - h ÷ø eV
h 24 1 2
P= = h × R× (de-Broglie hypothesis)
l 25 æ1
-

Þ DE = 13.6 (3)2 çè ÷ = 108.8 eV
24hR 24hR 12 32 ø
Þ mv = Þ V = 26. (d) For gr ound state, the prin cipal quantum no
25 25m
15. (d) For Lyman series (2 ® 1) (n) = 1. There is a 3rd excited state for principal
quantum number.
1 é 1 ù 3R
= R ê1 - 2 ú =
lL ë 2 û 4 4 3rd excited state
Pincipal quantum no. (n)

For Balmer series (3 ® 2)


1 é 1 1 ù 5R
Energy states

lB
=R ê 4 - 9 ú = 36 3 2nd excited state
ë û
4
lL 3R 4 æ5ö 5 2 1st excited state
Þ = = ç ÷=
lB 36 36 è 3 ø 27
5R 1 ground state
13.6 13.6 Possible number of spectral lines
16. (c) En = – 2
Þ E2 = – = –3.4 eV..
n 22 The possible number of the spectral lines is given
17. (b) 18. (c) 19. (b) n(n - 1) 4(4 - 1)
= = =6
é 1 1 ù 2 2
20. (b) E = Rhc ê - ú 27. (d) From, DE = hv
2
êë n1 n 2 2 úû
DE é 1 1 ù k(2n - 1)
E will be maximum for the transition for which n= =kê 2
- 2ú = 2
h ë (n - 1) n û n (n - 1)2
é 1 1 ù 2k
ê 2 - 2 ú is maximum. Here n 2 is the higher » 3
ëê n1 n 2 ûú n
1
energy level. or, n µ 3
n
é 1 1 ù 28. (a) According the Moseley’s law
Clearly, ê - ú is maximum for the third
êë n1 2
n 2 2 úû c 2 2
f = a (z - b) Þ f = a 2 (z - b) 2 Þ = a (z - b) ...(i)
l
transition, i.e. 2 ® 1 . I transition represents the
absorption of energy. for k a line, b = 1
Free eBooks on @neetquestionpaper2020

Atoms 959

l 2 (z1 - 1) 2 4l (11 - 1) 2 we know that


From (i) , = Þ = 1 é1 1 ù
l1 (z 2 - 1) 2 l (z 2 - 1) 2 = R z2 ê 2 - 2 ú
10 l ëê n1 n 2 ûú
Þ z2 -1 = Þ z2 = 6
2 1 é1 1 ù
29. (a) Initially a photon of energy 10.2 eV collides \ = 4R ê 2 - 2 ú
inelastically with a hydrogen atom in ground state l êë n1 n 2 úû
For hydrogen atom E1 = -13.6eV ; 9 9
For n2 = 3, n1 = 1 we get l = =
13.6 8 ´ 4R 32R
E2 = - = -3.4eV \ E 2 - E1 = 10.2eV
4 36 9
The electron of hydrogen atom will jump to second For n2 = 3, n1 = 2 we get l = =
5 ´ 4R 5R
orbit after absorbing the photon of energy 10.2 eV.
The electron jumps back to its original state is less 4 1
For n2 = 2, n1 = 1 we get l = =
than microsecond and releases a photon of energy 3´ 4R 3R
10.2 eV. Another photon of energy 15 eV strikes the (a), (c) are correct options
hydrogen atom inelastically. This energy is sufficient
to knock out the electron from the atom as ionization Exercise 3 : Conceptual & Applied MCQs
energy is 13.6 eV. The remaining energy of 1.4 eV is
left with electron as its kinetic energy. R1 n12 1 n1 1
1. (d) = = \ =
30. (b) The smallest frequency and largest wavelength in ul- R 2 n 22 4 n2 2
tra-violet region will be for transtion of electron from 3 3
orbit 2 to orbit 1. T1 æ n1 ö æ 1 ö 1
= çç ÷÷ = ç ÷ =
1 æ 1 1 ö T2 è n 2 ø è 2 ø 8
\ l = R çç 2 - 2 ÷÷ 2. (c) Ist excited state corresponds to n = 2
è n1 n2 ø 2nd excited state corresponds to n = 3
1 é1 1 ù é 1 ù 3R
Þ = R ê - ú = R ê1 - ú = E1 n 22 32 9
122 ´ 10 -9 m ë 12 2 2 û ë 4û 4 \ = = =
4 E 2 n12 2 2 4
Þ R= 3. (c) l = 0 to (n – 1) i.e. 0, 1, 2, 3,. In all 4 values.
3 ´ 122 ´ 10-9 m
The highest frequency and smallest wavelength for 4. (a) As n increases, energy difference between adjacent
infrared region will be for transition of electron from energy levels decreases.
¥ to 3rd orbit. 1 1
5. (d) As r µ \ r0¢ = r0
1 æ 1 1 ö m 2
\ = Rç - ÷ '
l ç 2
n 2 2 ÷ø As E µ m \ E = 2(–13.6) = –27.2 eV
è n1 0
1 4 æ 1 1ö 6. (a) Conservation of energy requires that the 15.0 eV
Þ = ç - ÷ photon energy first provides the ionization energy
l 3 ´ 122 ´ 10 -9 è 32 ¥ ø to unbind the electron, and then allows any excess
1 é1 1ù energy to become the electron’s kinetic energy. The
31. (a) We know that = RZ 2 ê 2 - 2 ú kinetic energy in this case is 15.0 eV – 13.6 eV = 1.4 eV.
l êë n1 n2 úû
The wave length of first spectral line in the 7. (d) r µ n 2 Þ pr 2 µ n 4
Balmer series of hydrogen atom is 6561Å . Here n2 = 3
and n1 = 2 So area µ n 4 Þ Ratio is 24 : 1 Þ 16 : 1.
8. (c) According to Bohr's theory,
1 æ 1 1 ö 5R
\ = R(1) 2 ç - ÷ = ...(i) nh
6561 è 4 9 ø 36 Angular momentum, mvr =
2p
For the second spectral line in the Balmer series of h
singly ionised helium ion n2 = 4 and n1 = 2 ; Z = 2 So in ground state, angular momentum = .
2p
1 2 é1 1 ù 3R 1
\ = R (2) ê - ú = ...(ii) 9. (d) Charge = e, frequency = n, time period =
l ë 4 16 û 4 n
Dividing equation (i) and equation (ii) we get e
So, equivalent current = = en
l 5R 4 5 1/ n
= ´ =
6561 36 3 R 27 - Ze 2
10. (b) P.E. = . Negative sign indicates that revolving
\ l = 1215 Å 4pe 0 r
nh 3h electron is bound to the positive nucleus.
32. (a, c)Angular momentum = = . Therefore n = 3.
2p 2p So, it decreases with increase in radii of orbit.
a n2 11. (d) Laser action involves all of the following.
Also rn = o = 4.5a o
z (i) Amplification of particular frequency.
n2 9 (ii) Population inversion.
\ = 4.5 Þ = 4.5 Þ z = 2 (iii) Stimulated emission
z z
Free eBooks on @neetquestionpaper2020

960 Ph ysi cs
12. (a) Since 121.5 nm line of spectrum of hydrogen atom lies For Balmer series n = 2
in ultraviolet region, therefore it is Lyman series. 1 æ 1 1 ö 1 æ 1 1 ö
= RZ2 ç - and l = RZ2 ç - ÷
13. (d) Different values of magnetic azimuthals quantum ÷ è2 2
32 ø
l max è 22 ¥ 2 ø min
number are m1 = 2 l + 1 = 2(3) + 1 = 7 1 1
14. (b) As energy emitted varies inversely as wavelength, -
1/ l min (1/ 2 2 - 1/ 32 ) 4 9 = 1 - 4 = 5 / 9
therefore or = =
1/ l max (1/ 2 2 ) 1/ 4 9
1 1 1 l + l1 ll
= + = 2 Þ l3 = 1 2 l 5
l 3 l1 l 2 l1 l 2 l1 + l 2 Þ max =
é ù l min 9
1 1 1 1
15. (d) We know that, = RZ2 ê 2 - 2 ú Þ µ Z2 1 é1 1 ù
l êë n 2 n1 úû l 30. (d) For ka line = R (Z – 1)2 ê 2 - 2 ú
lk ë1 2 û
1 a
l is shortest when is largest i.e., when Z is big . Z is
l 4 1 1 4 1 1
highest for lithium. (Z – 1)2 = = ´ - 8
´
3 l ka R 3 1.785 ´ 10 109737
1 Dp
16. (c) Dp ´ l ~ h Þ =
l h Þ (Z – 1)2 = 680.6 Þ Z – 1 = 26 Þ Z = 27
1 æ 1 1 ö Thus, the element is cobalt.
ç ÷
We know that l = R ç 2 - 2 ÷
è n1 n 2 ø 31. (a) From Mosley's law, we have,
æ 1 c
1 ö÷ (Z – 1)2 µ n \ (Z – 1)2 = A l
Þ m H v H = hR H ç -
ç 2 2÷ ka
è n1 n 2 ø where A is some constant,
æ 24ö (ZMO - 1)2 l Cu
2
l Cu
v = 6.26 ´ 10-34 ´ R H ç ÷ = 4 ms-1 æ 41ö
è 25ø \ = or çè ÷ø =
(ZCu - 1) 2 l MO 28 0.71
2
17. (a) R µ n2 æ 41ö
1 \ l Cu = 0.71 × ç ÷ = 1.52Å
V µ so VR is µ n è 28 ø
n 32. (c) According to Bohr’s second postulate.
1
T µ n3 Þ E µ 1 é 1 1 ù
n2 33. (b) In Paschen series l = Rê 2 - 2 ú
18. (a) Series limit of Lyman series is ¥ ® 1 n1 max ë (3) (4) û
144 144
First line of Lyman series is 2 ® 1 n2 Þ l max = = = 1.89 ´ 10-6 m = 1.89 mm
7R 7 ´ 1.1 ´ 107
Series limit for Balmer series ¥ ® 2
9 9
\ v1 - v 2 = v3 Similarly l min = = = 0.818 mm
R 1.1 ´ 107
19. (d) r = 0.528n 2 ´ 10 -10 34. (c) For third lineof Balmer seriesn1 = 2, n2 = 5
æ 1 ´ 10 - 7 ö 1 é1 1 ù n2n2
ç
2
÷ = 0.528n ´ 10
- 10 \ = RZ 2 ê 2 - 2 ú gives Z2 = 2 1 22
è 2 ø l ë n1 n 2 û (n 2 - n1 )lR
5 ´ 10 - 8 On putting values Z = 2
Þ n2 = Þ n » 31
0.528 ´ 10 - 10 13.6Z2 -13.6(2) 2
20. (a) Ionisation energy of Li++ = 9hcR From E = - = = -54.4 eV
Ionisation energy = RchZ2 = Rch(3)2 n2 (1) 2
(as Z = 3 for Li++) 1 é1 1 ù 5R 1
= 9hcR. 35. (a) = Rê 2 - 2ú = ,
l Balmer ë2 3 û 36 l Lyman
21. (d) 22. (c)
23. (b) Potential energy of electron in n th orbit of radius r in é 1 1 ù 3R
= Rê - ú =
e2 ë12 22 û 4
H-atom U = - (in CGS)
r 5
1 e2 \ l Lyman = l Balmer ´ = 1215.4Å
Q K.E. = | P.E. | Þ K = 27
2 2r n(n - 1) 3(3 - 1)
24. (b) 25. (a) 26. (b) 27. (a) 36. (b) No. of lines N E = = =3
28. (b) Transition from higher states to n = 2 lead to emission 2 2
of radiation with wavelengths 656.3 nm and 365.0 nm. 1 é1 1ù 9
37. (a) For Bracket series l =Rê 2 - 2ú= R
These wavelengths fall in the visible region and max ë4 5 û 25 ´ 16
constitute the Balmer series.
1 é1 1 ù R l 25
1 æ 1 1 ö and l = R ê 2 - 2 ú = Þ max =
29. (d) = RZ2 ç - ÷ min ë4 ¥ û 16 l min 9
l ç 2 2÷
è n1 n 2 ø
Free eBooks on @neetquestionpaper2020

Atoms 961
38. (d) No. of particles scattered through an angle n1 (n1 - 1)
\3 = , in first case.
k Z2 2
q = N( q ) =
æqö 2
Or n1 - n1 - 6 = 0 or (n1 - 3)(n1 + 2) = 0
sin 4 ç ÷ (K.E.)2
è 2ø Take positive root.
4kcz 2 kz 2 28 \ n1 = 3
\ 28 = for q = 90° \ = =7 n (n - 1)
(K.E.) 2
(K.E.) 2 4 Again, 6 = 2 2 , in second case.
2
7
\ N(60°) = = 16 × 7 = 112 /min. Or n 22 - n 2 - 12 = 0 or (n 2 - 4)(n 2 + 3) = 0.
4 æ 60° ö
sin ç ÷ Take positive root, or n 2 = 4
è 2 ø
7 2pKZe 2
N(120°) = = 12.4 / min Now velocity of electron u =
4 120° ö
æ nh
sin ç ÷
è 2 ø u1 n 2 4
\ = = .
1 1 c–v u2 n1 3
39. (b) =
l' l c+ v 1 é 1 1 ù 4
46. (a) l = R ê 2 - 2 ú Þ l max = » 1213Å
Here, l ' = 706 nm, l = 656 nm max ë (1) (2) û 3R
2 2
c–v æ l ö æ 656 ö 1 é 1 1ù 1
\ =ç ÷ =ç ÷ = 0.86 and l = R ê 2 - ú Þ l min = » 910Å.
c+ v è l'ø è 706 ø min ë (1) ¥ û R
v 0.14 47. (a) Electron after absorbing 10.2 eV energy goes to its
Þ = Þ v = 0.075 × 3 × 108 = 2.25 × 107m/s
c 1.86 first excited state (n = 2) from ground state (n = 1).
40. (c) Transition A (n = ¥ to 1) : Series lime of Lyman series h
Transition B (n = 5 to n = 2) : Third spectral lien of \ Increase in momentum =
2p
Balmer series
6.6 ´ 10-34
Transition C (n = 5 to n = 3) : Second spectral line of = = 1.05 ´ 10-34 J-s.
Paschen series 6.28
41. (a) Orbital speed varies inversely as the radius of the orbit. 1 é1 1 ù
1 48. (d) l = R ê n 2 - n 2 ú . For first wavelength, n1 = 2, n2 = 3
vµ ë 1 2û
n Þ l1 = 6563 Å. For second wavelength, n1 = 2, n2 = 4
42. (d) Radius of n th orbit rn µ n2, graph between r n and n is
2 Þ l2 = 4861 Å
rn æ n ö æ rn ö 49. (d) For an atom following Bohr’s model, the radius is given
a parabola. Also, = ç ÷ Þ log e ç ÷ = 2log e (n)
r1 è 1 ø è r1 ø
r0 m 2
Comparing this equation with y = mx + c, by rm = where r0 = Bohr’s radius and m = orbit
Z
Graph between log e æç n ö÷ and loge (n) will be a straight
r number.
è r1 ø For Fm, m = 5 (Fifth orbit in which the outermost
line, passing from origin. electron is present)
Similarly it can be proved that graph between r 52 1
æf ö \ rm = 0 = nr0 (given) Þ n =
log e ç n ÷ and log n is a straight line. But with 100 4
è f1 ø e
N 4
sin (q1 / 2)
1
negative slops. 50. (c) N µ ; 2 =
é1 1 ù
4
sin q / 2 N1 sin 4 (q2 / 2)
43. (a) Excitation energy DE = E2 –E1 = 13.6 Z2 ê 2 - 2 ú
ë1 2 û N2 sin 4 (60° / 2) N2 sin 4 30°
or = or =
6 4
3
Þ 40.8 = 13.6 ´ ´ Z2 Þ Z = 2.
5 ´ 10 sin (120° / 2) 5 ´ 106 sin 4 60°
4 4 4
æ 1ö æ 2 ö 5
Now required energy to remove the electron from or N 2 = 5 ´ 106 ´ ç ÷ ç = ´ 10 6
è 2 ø è 3 ÷ø 9
+13.6Z2
ground state = = 13.6(Z) 2 = 54.4 eV. 51. (a) Energy of electron in n th orbit is
(1)2 Z2
1 é1 1 ù En = - (Rch) 2 = -54.4 eV
44. (c) = Rê 2 - 2ú n
l ë n1 n 2 û For He+ is ground state
1 é1 1 ù (2)2
Þ -10
= 1.097 ´ 107 ê 2 - 2 ú Þ n 2 = 4 E1 = - (Rch) = -54.4 Þ Rch = 13.6
970.6 ´ 10 ë1 n 2 û (1)2
n(n - 1) 4 ´ 3 \ For Li++ in first excited state (n = 2)
\ Number of emission line N = = =6
2 2 (3)2
n(n - 1) E' = – 13.6 × = -30.6 eV
45. (a) Number of emission spectral lines, N =
2 (2) 2
1 æ 1 1ö
52. (b) = R ç 2 - 2÷
l è n1 n 2 ø
Free eBooks on @neetquestionpaper2020

962 Ph ysi cs

1 æ 1 1ö æ 1 1 ö 5R 1 é1 1 ù
=R çè 2 - 2 ÷ø = R çè 4 - 9 ÷ø = 36 and l = Rê 2 - 2ú
l0 2 3 min ë2 ¥ û
1 æ 1 1ö æ 1 1 ö 3R 4 4
=R ç - ÷ =R çè - ÷ø = l min = = = 3646 Å
l è2 2
42 ø 4 16 16 R 1.097 ´ 107
The wavelength 6563 Å and 3646 Å lie in visible region.
l 5 16 20
= ´ = Therefore, Balmer series lies in visible region.
l 0 36 3 27
59. (b) Rutherford confirmed that the repulsive force on
53. (c) n µ (Z - 1) for Ka X-ray a-particle due to nucleus varies with distance
2 2 according to inverse square law and that the positive
n 2 æ Z2 - 1ö æ 51 - 1ö 25 charges are concentrated at the centre and not
Þ =ç ÷ Þ n 2 = ç ÷ n= n
n1 è Z1 - 1 ø è 31 - 1 ø 9 distributed throughout the atom.
54. (c) Number of possible spectral lines emitted when 60. (b) Bohr postulated that electrons in stationary orbits
an electron jumps back to ground state from n th around the nucleus do not radiate.
n(n - 1) This is the one of Bohr’s postulate. According to this
orbit =
2 the moving electrons radiates only when they go from
n(n - 1) one orbit to the next lower orbit.
Here, =6Þn =4 61. (a) Absorption transition
2
Wavelength l from transition from n = 1 to n = 4 is C
given by,
1 æ1 1 ö 16 B
= Rç - ÷Þl= = 975 Å
l è 1 42 ø 15R
m I e A
55. (d) Q B = 0 and I =
2r T
m 0e Two possibilities in absorption transition.
B= [ r µ n 2 , T µ n5 ] ; 1
Bµ 5
2rT n
56. (b) For 2nd line of Balmer series in hydrogen spectrum
1 æ 1 1ö 3
= R (1) ç 2 - 2 ÷ = R
l è 2 4 ø 16
é1 æ 1 1 ö 3R ù
For Li2+ ê l = R ´ 9 çè 2 - 2 ÷ø = 16 ú Three possibilities in emission transition.
ë x 12 û
which is satisfied by n = 12 ® n = 6. Therefore, absorption transition < emission.
62. (b)
1 é1 1 ù
57. (c) Using = R(Z - 1)2 ê 2 - 2 ú 63-66. ratom » 10-10 m
l êë n 2 n1 úû
For a particle, n 1 = 2, n2 = 1 rnucleus » 10-15 m
1875 R æ 3ö 3
For metal A : = R (Z1 - 1) 2 ç ÷ Þ Z1 = 26 Vatomic æ ratom ö
4 è 4ø = = 1015
Vnucleus çè rnucleus ÷ø
2 æ 3ö
For metal B : 675 R = R (Z2 - 1) çè ÷ø Þ Z2 = 31 63. (c) 64. (a) 65. (a) 66. (d)
4
Therefore, 4 elements lie between A and B. kq1q2
U=
= 220 MeV. r
58. (a) The wavelength in Balmer series is given by q1 = +2e, q2 = + 82e
1 é1 1 ù 9 ´ 109 ´ 2e ´ 82e
= R ê 2 - 2 ú , n - 3, 4,5,... U= = 5.82 ´ 10-13 J
l ë2 n û 6.5 ´ 10-14

1 é1 1ù U = 3.63 MeV
=Rê 2 - 2ú
l max ë2 3 û From conservation of energy
36 36 K1 + U1 = K2 + U2
l max = = = 6563 Å K1 + 0 = 0 + 3.63 MeV
5R 5 ´ 1.097 ´ 107
K1 = 3.63 MeV
Free eBooks on @neetquestionpaper2020

28
Nuclei

SOME IMPORTANT FACTS ABOUT ATOMIC MASS, SIZE MASS ENERGY AND NUCLEAR BINDING ENERGY
AND COMPOSITION OF NUCLEUS Einstein’s mass energy equivalence i.e., E = mc2 gives :
1. Proton was discovered by Goldstein 1 a.m. u º 931.5 Me.V, where 1 eV = 1.6 ´10 -19 J.
2. Atomic mass unit, 1.a.m.u = 1/12th of mass of C-12 isotope, and 1Mev = 1.6 × 10–13 J
1 a.m.u = 1.660565 × 10–27 kg Binding Energy
3. Mass of a proton, mp = 1.0073 a.m.u = 1.6726 × 10–27 kg Binding energy of a nucleus is the energy with which nucleons
4. Chadwick’s experiment : Neutrons were detected. are bound in the nucleus. It is measured by the work required to
be done to separate the nucleons an infinite distance apart from
é 42 He + 94 Be ®12 1 ù
6 C +0 nû the nucleus, so that they may not interact with one another.
ë
5. Mass of neutron, mn = 1.00866 a.m.u = 1.6749 × 10–27 kg Total BE = [ ZmP + ( A - Z ) mn - m N ] c 2
6. Mass of electron = 9.1 ×10–31 kg mp = mass of proton; mn = mass of neutron
mN = mass of nucleons (protons + neutrons); Z = Atomic number;
7. Mass number, A = total number of nucleons (neutrons +
A = atomic mass
protons present in the nucleus of an atom)
Binding Energy per Nucleon
8. Atomic number, Z = number of protons = number of
The binding energy per nucleon of a nucleus is the average
electrons
energy required to extract a nucleon from the nucleus.
9. Types of nuclei : Binding energy per nucleon
(i) Isotopes : The atoms of the element which have the Total binding energy BE Dmc2
same atomic number but different atomic mass B= = =
numbers. e.g., 1H1, 1H2, 1H3 ; 8O16, 8O17, 8O18 Total number of nucleons A A
(ii) Isobars : The atoms of differents element which have c2
= [Zm p + ( A - Z )mn - M ( Z X A )]
the same atomic mass number but different atomic A
numbers. e.g., 6C14, 7N14, 18Ar40, 20Ca40 etc. The graph of binding energy per nucleon with mass number A is
(iii) Isotones : The nuclides which contain the same number as shown below.
16
8O

of neutrons e.g., 2 H32 ,2 He24 ; 2 Be95 , 5 Be10


5 etc.
8.0
12
6C
(MeV)

(iv) Isomers : having same mass number, same atomic 6.0


2He
4
Binding energy

number but different radioactive properties.


per nucleon

4.0
10. Rest mass of nucleus is less than sum of rest masses of
constituent nucleons, the difference is called mass defect. 2.0
11. Size of the nucleus : Radius of nucleus, R = R0 A1/ 3 where
56
R0 = 1.1 × 10–15 m. 0
50 100 150 200 250
Nuclear density of all elements ~ 1017 kg m–3. Mass number A
Free eBooks on @neetquestionpaper2020

964 Physi cs
Binding energy per nucleon gives a measure of stability of Features of Radioactivity :
nucleus. More is binding energy per nucleon more is the stability (i) It is a statistical process.
of nucleus. Binding energy per nucleon is small for lighter nuclei (ii) When a nucleus undergoes alpha or beta decay, its atomic
i.e. 1H1, 1H2 etc. number and mass number changes (in b-decay only atomic
For A < 28 at A = 4n the curve shows some peaks at 2He4, 4Be8, number changes) & it transforms into a new element.
16 16 20 24
6C , 8O , 10Ne , 12Mg .
This represents extra stability of these elements with respect to (a) ZX
A
= Z- 2 Y A - 4 + 2 He 4 (a-particle), it means that
their neighbours. by emission of alpha particle (a-particle), it loses
Keep in Memory 2 units of charge and 4 units of mass.

(b) ZX = Z-1Y A + b + (positron). It means that by


A
1. A nuclide is a specific nucleus of an atom characterised as
A emission of beta particle (b+-particle), nucleus loses
ZX where A = mass number and Z = atomic number. one unit of charge. It is surprising to note that a
2. Binding energy per nucleon is nearly 8.4 MeV for nuclei in nucleus does not contain b+ then how is it emitted.
the range of mass number 40 to 120. Reason : During a b+ particle(i.e., positron) decay, a
3. Binding energy is highest in Fe56.( 8.8 MeV) protron converts into a neutron
4. Binding energy curve predicts :
p + = n 0 + b + + u (neutrino).
(a) Fission : Breaking up of a heavy nucleus
(A > 200) into two nuclei of approximately equal size, A b– particle (i.e., electron) decays, when inside the
and release of energy. nucleus a neutron converts into a proton i.e.,
(b) Fusion : Lighter nuclei ( A < 20) combine together to
form heavier nucleus and release of energy. n 0 = p + + b - + u (antineutrino)
(c) BE/ A varies by less than 10% above A = 10 suggests
that each nucleon interacts with its neighbouring
nucleon only. Since b-particle is an electron(or positron), so the loss
(d) For A > 56, BE/A decreases because of the of mass in this decay is negligible.
destabilising effect of long-range coloumbic force. In b+decay the daughter element is one place forward
in the periodic table.
NUCLEAR FORCE
(iii) When a nucleus emits a gamma ray, neither the mass nor
It is the force acts in the nucleus between the nucleons and is
the charge of the nucleus changes
responsible for binding the nucleon.
Characteristics of Nuclear Force : i.e.,
1. It is a short range force effective only in range 10–15 m A A
ZX = Z X + g (gamma ray or photon)
2. It is charge independent. It acts between proton-proton, Excited state Ground state
proton-neutron and neutron – neutron.
The gamma ray(g-ray) is photon & it carries away some
3. It is not a central force.
energy from the nucleus & nucleus returns from excited
4. It is spin dependent. state(unstable state) to ground state (stable state)
5. It is 1038 times stronger than gravitational force and 102
a and b-particles are not emitted simultaneously.
times stronger than electric force.
6. The main cause of nuclear force is the exchange of g rays are emitted after the emission of a and b-particle. a,
p- mesons between nucleus b and g-rays are known as Bequerel rays
The energy spectrum in the case of b-particles is continous
p ® n + p + , p ® p + p° , n ® p + p – but that of a and g-rays is a line spectrum. This means that
RADIOACTIVITY b particles are emitted with any amount of kinetic energy.
It is the spontaneous disintegration of the heavy nucleus of an Properties of a, b & g-rays
atom (It occurs without external provocation). (A) Properties of a-rays
There are three main types of radioactive radiations. (a) It is a positively charged particle & contains a
(i) a-rays (i.e., Helium nuclei or a – particles) charge of 3.2 × 10–19 coulomb(exactly double the
(ii) b-rays (i.e., electron or positron or b – particles) charge of electron).
(iii) g-rays (photons or gamma radiations) (b) The mass of a-particles is 6.645 × 10–27kg(It is
It is a process by which an unstable nuclei achieves stability. equal to mass of a helium nucleus). Actually a-
This process is not affected by particle is nucleus of helium, hence it is called
(a) chemical combination doubly ionised helium.
(b) changing physical environment other than nuclear (c) They (a-particles) get deflected in both electric
bombardment. & magnetic fields.
Free eBooks on @neetquestionpaper2020

Nuclei 965
(d) The velocity of a-particle is very less than the Property a -rays b -rays g -rays
velocity of light i.e., Va » c , where c is velocity Mass 4mp me Zero
10 Charge +2e e Neutral
of light.
(e) The range of a-particle in air depends on 2 0.33 to c
Velocity » 2 × 10 m/s 0.99c
radioactive substance.
(f) The ionisation power of a-particle is higher than Penetrating
. power 1 100 1000
both b (100 times of b & 10,000 times of g) and g
particle. Ionisation 100
10,000 1
(g) The penetrating power of a particle is lowest (in power
comparison to b & g particles). It is 1/100 times of Spectrum Line Continuous Line
b-particles & 1/10,000 times of g-rays.
Rutherford and Soddy law for radioactive decay
(h) The a-particles can produce fluorescence in
barium platinocynide and zinc sulphide. It states that “at any time the rate at which particular
(i) They show little effect on photographic plate. decay occurs in a radioactive substance is proportional
to number of radioactive nuclei present.
(j) They show heating effect on stopping.
If N is the number of nuclei at any time t & at t + dt time, it
(B) Properties of b rays or b-particles : decrease to N-dN then the rate of decay of these nuclei is
(a) The beta particles (i.e., b– or b+) may be positive
-dN
& negative particle & contain ± 1.6 ´ 10 -19 C of (negative sign comes because N decreases as t
dt
charge. Actually b– is electron & b+ is positron. increases). So according to Rutherford & Sodi,
(b) They get deflected in both electric & magnetic
- dN
field. µ N or dN = lN ...(1)
(c) The velocity of b-particle varies between 0.01c dt dt
to .99c, where c is velocity of light. Where l is decay constant (i.e., probabilily per unit time for
(d) The mass of b particle is relativistic, because its a nucleus to decay) and it is constant for a particular nuclei,
velocity is comparable to velocity of light but different for different nuclei. By integration of equation
(e) They have both ionisation & penetration power. (1) w.r. to time we get
Ionisation power less than a-particle and N = No e–lt ...(2)
penetration power more than a-particle. where No is the number of nuclei at t = 0.
(f) Th ey produce fluorescence on barium
Activity : The number of decays per unit time or decay rate
platinocynide & zinc sulphide.
is called activity(R)
(C) Properties of g-rays (or gamma radiation):
é dN ù -lt
(a) They are electromagnetic waves as x-rays. [R ] = ê ú = N o le = R o e -lt where R = N l .
ë dt û o o
(b) They are not deflected in electric & magnetic field,
it means that they are chargeless. The S.I. unit of R is bequerel,
(c) The velocity of g-particle is equal to velocity of 1 bequerel = 1 Bq = 1 decay/sec
light. and 1 curie = 1Ci = 3.7 × 1010 decay/sec
The other unit of radioactivity is rutherford.
(d) The ionisation power of gamma rays is less than
b & a rays but penetration power more than b 1 rutherford = 106 dacay/sec
and a-rays. 2.303 N é N 0 m0 ù
Þ t= log 0 êand =
(e) The g-particles are emitted from the nucleus, while l N ë N m úû
X-rays are obtained, when electron goes from
m0 = mass at t =0 and m = mass at t = t
one state to another in an atom.
(f) When g-rays photon strikes nucleus in a COMMON DEFAULT
substance, then it gives rise to a phenomenon of O Incorrect. Since b-particles (electrons) are emitted from the
pair production i.e., nucleus shows that electrons exist in nucleus
P Correct. b-particle cannot exists in nucleus. It is created
hn ¾¾
® b- + b+ and ejected at once at the time of b-decay. b-particle cannot
( g-rays or photon) (Pair production) exist in the nucleus because its wavelength is greater than
the size of nucleus
The minimum energy of g-rays required for this
phenomena is 1.02 MeV, because the rest mass Half Life of a Radioactive Substance
energy of b ± particle is 0 .51 MeV.. Half life of a substance is the time, it takes for half of a given
number of radioactive nuclei to dacay
Free eBooks on @neetquestionpaper2020

966 Physi cs

No Series Mass Starting Stable end Natural /


Let at = T1/2 N= then by eq. (2)
2 number isotope product Artificial
Thorium 4n 232 208
No -lT 0.693 log e 2
1 90 Th 82 Pb Natural
2 or T1/2 = = Neptunium 4n+1 237 209
= No e 93 Np 83 Bi Artificial
2 l l Uranium 4n+2 238 206
92 U 82 Pb Natural
Actinium 4n+3 235 207
N0 92 U 82 Pb Natural
Also N = for n half-lives
2n Radioactive Equilibrium
When the rate of formation of daughter nuclei becomes equal to
m0
Þ m= where m0 is mass of radioacive substance at t = 0 rate of its decay then this is called as state of radioactive
2n equilibrium
and m is mass at t = t. NA NB
NA lA = NB lB = ............ or = = ............
N TA TB
Number of remaining nuclei

Carbon Dating
N0 Carbon dating is the process of determination of time interval
N0 /2 which has passed by making use of radioactive decay of a sample
containing radioactive substance (6C14). It helps in calculating
N0 /4 age of geological specimens like rocks, biological specimens likes
bones of animals or trunk of trees and age of earth. The isotope
N0 /8 of carbon 6C14 is radioactive. It is formed in atmosphere by
bombardment of nitrogen atoms with cosmic rays
14 + n 1 ® C14 + H1
t 7N 0 6 1
14
The 6C combines with oxygen to form carbondioxide which is
t=o
time absorbed by plants so concentration of 6C14 is constant with
time. The living plants and animals have a fixed ratio of 6C14 to
Mean Life of a Radioactive Substance: ordinary carbon 6C12. When a plant or animal dies the content of
Mean life (average life) t is defined as the averge time the nucleus 14 12
6C decreases while that of 6C remains constant. The ratio of
survive before it decays. two indicates the time that has passed since death of plant or
animal. The time interval is calculated from the laws of radioac-
1 T1/ 2
It is given as : t = = tive disintegration
l 0.693
1 N 2.303 N æ N0 A ö
The equivalent l and t for two nuclei A and B, t= log e 0 = log10 0 ç = 0÷
l N l N è N Aø
t AtB where No is number of 6C14 nuclei at time of death, l is decay
l = l A + l B and t = constant of 6C14 and N is number of 6C14 nuclei currently present
t A + tB
in sample.
RADIOACTIVE SERIES
Keep in Memory
The heavy nuclides change their mass number by a decay and
atomic number by a and b decay.
They can decay to stable end products by four paths. The four 1. Specific activity is the activity of 1 gram of material.
paths have mass numbers given as 4n, 4n + 1, 4n + 2, 4n + 3 where 2. Geiger Muller Counter is used for detecting a and b
n is integer. particles.
Last element of series is stable and has a decay constant zero. 3. Cloud chamber is used for detecting radioactive radiations
There are four radioactive series : and for determining their paths, range and energy.
Uranium 92U238 ® 82Pb206 4. Baryon number B = 1, for a neutron and a proton.
5. Lepton number (L) :L = 1 for electron, and neutrino
(Half life T1 / 2 = 4.47 ´109 years ) L = –1 for positron and antineutrino.
Actinium (natural) 92U235 ® 82Pb207 6. Radioactive isotope Uses
Iodine-131 For detecting the activity of
(Half life T1 / 2 = 7.04 ´108 years )
thyroid gland
Thorium 232 ® 208
92Th 82Pb Chromium-51 To locate the exact position of
(Half life T1 / 2 = 1.41´1010 years ) haemorrhage
Phosphorous-32 In agriculture
Neptunium 93Nb237 ® 83Bi209 C – 14 Carbon dating,
(Half life T1 / 2 = 2.14 ´ 106 years ) Photosynthesis in plants
Co60 Cancer treatment
But only first three series occur in nature & fourth one is artificial.
Na24 For circulation of blood
Free eBooks on @neetquestionpaper2020

Nuclei 967
NUCLEAR REACTION 2
1 1
Nuclear reaction obeys following conservation laws : 1 H +1 H ¾¾
®1 H +10 e + n + 0.42MeV
(1) Charge conservation 3
2 1
(2) Conservation of linear momentum 1 H +1 H ¾
¾® 2 He + g + 5.5 MeV
(3) Conservation of angular momentum 4
3 3
(4) Conservation of energy (Rest mass energy + K.E.) 2 He + 2 ¾® 2 He +11 H +11 H + 12.8MeV
He ¾
Standard way of representing a nuclear reaction 4
For a nuclear reaction 411 H ¾
¾® 2 He + 2 10 e + 2n + 2 g + 24.6 MeV
1
A 4
®1 H + zA++13 Y Stars with mass 0.4 to 2.5 solar mass produce energy by carbon-
z X +2 He ¾¾
nitrogen cycle. Stars with lower mas produce energy by proton-
Standard way A
z X ( a, p) zA++13 Y proton cycle.
Nuclear Fission (By Otto Hans and Fstrassmann) Nuclear holocaust : It is the name given to large scale destruction
Nuclear fission is the disintegration of a heavy nucleus upon which will be created upon the use of piled up nuclear weapons.
bombardment by a projectile, such that the heavy nucleus splits It is believed that if the existing nuclear weapons are used, then
up into two or more segments of comparable masses with an the radioactive waste will hang like a cloud in the earth's
enormous release of energy. atmosphere. This clould will be capable to absorb solar radiation
235 1
92 U + 0 n ® 141 92
53 Ba +36 Kr + 3 ( n) + 200 MeV
1
0
due to which these radiation will not reach earth. This would
result to a long nuclear winter.
The most of the energy released is by the mode of kinetic energy Radiation hazards : The g-radiations are highly energetic and
of fission segment. causes pathological and genetic damage.
Uncontrolled Chain Reaction : It is the principle of atom bomb
Example 1.
(destructive use). The number of fission in this case goes on
increasing at a tremendous rate leading to the creation of a huge The radius of a nucleus with nucleon number 16 is 3 × 10–15 m.
amount of energy in a very small time. What would be the radius of other nucleus with nucleon
The number of fissions in this case is maintained constant. Nuclear number 128?
reactor has beed devised for this purpose. Solution :
The main parts of nuclear reactor are We know that
(a) Nuclear fuel : U233, U235, Pu239 etc.
(b) Moderator : Graphite, heavy water (D2O). To slow down R µ A1 / 3
the neutrons (or slow down the nuclear reaction). 1/ 3 1/ 3
(c) Control rods : (Cadmium, boron). To absorb excess R æA ö æA ö
\ 2 = çç 2 ÷÷ or R 2 = R1 çç 2 ÷÷
neutrons. It controls the chain reaction. R1 è A1 ø è A1 ø
(d) Coolant : (water etc). To remove the heat produced in the
so, R2 = 3 × 10–15 × (128/16)1/3
core to heat exchanger for production of electricity.
The reaction of controlled chain reactor is also called = (3 × 10–15) × 2 = 6 × 10–15 m
critical reaction. Example 2.
Critical mass : It is the minimum amount of fissionable material What is the power output of 92U235 reactor if it takes 30
days to use up 2 kg of fuel, and if each fission gives 185 MeV
required to carry out fission reaction. It is 10 kg for U 236
of usable energy? Avogadro’s number N = 6.02 × 1026 per
Rate of neutron production kilomol.
Reproduction factor K =
Rate of neutron absorption Solution :
k = 1 for controlled reaction Number of atoms in 2 kg fuel
k ³ 1 for uncontrolled reaction 2
Breeder reactor :It converts U238 non-fissionable to a fissionable = kilomol × (6.02 × 1026 per kilomol)
235
material 239 235
Pu or U. = 5.12 × 1024
Nuclear Fusion : Fission rate = number of atoms fissioned in 1 sec.
Nuclear fusion is the fusion of two or more light nuclei to form a 5.12 ´10 24
= = 1.975 × 1018 per second
heavy nucleus with a release of huge amount of energy. 30 ´ 24 ´ 60 ´ 60
For a nuclear fusion to take place, very high temperature is Each fission gives 185 MeV of energy.
required to overcome the coulombic repulsive forces acting Hence energy obtained in one second i.e., power output
between the nuclei. It is the principle of hydrogen bomb. P = 185 × (1.975 × 1018) MeV per second
The nuclear fusion reaction, which is the source of the energy of = 185 × (1.975 × 1018) × (1.6 × 10–19)
sun/ star are proton-proton cycle. = 58.46 × 106 joule per second (watt) = 58.46 MW
Free eBooks on @neetquestionpaper2020

968 Physi cs
Example 3. Solution : (c)
The activity of a radioactive element decreased to one dN x
= l1 N x = l1 ( N 0 / 4) (Q N = N0/2n) and
third of the original activity Ι0 in a period of nine years. dt
What will be its activity after a further lapse of nine years? dN Y
= l 2 N Y = l 2 ( N 0 / 2)
Solution : dt
æ I ö æ 1 öt / T 1 æ1ö
9/T
æ dN ö æ dN Y ö l1 T 2 1
çç ÷ = ç ÷ or = ç ÷ \ ç X÷ çè ÷= = 2 = =
÷ è dt ø dt ø 2l 2 2T1 2 ´1 1
è I0 ø è 2 ø 3 è2ø
2 æ 0.693 ö
æ I¢ ö æ 1 ö18 / T éæ 1 ö
9/T ù
æ1ö
2 çQ l = ÷
ç ÷=ç ÷ êç ÷ ú =ç ÷ è T ø
çI ÷ =
è 0 ø è2ø êëè 2 ø úû è3ø Example 7.
\ Ι¢ = Ι 0 / 9 The half life of radium is 1620 years and its atomic weight
is 226. Find the number of atoms that will decay from its
Example 4. 1 gm sample per second.
The radioactivity of a sample is R1 at a time T1 and R2 at a Solution :
time T2. If the half-life of the specimen is T, the number of
According to Avogadro’s hypothesis,
atoms that have disintegrated in the time (T1 – T2) is
proportional to 6.02 ´10 23
N0 = = 2.66 ´10 21
(a) (R1T1 – R2T2) (b) (R1 – R2) 226
(c) (R1 – R2)/T (d) (R1 – R2) T 0.6931
Solution : (d) Half life = T = = 1620 years
l
Radioactivity at T1 , R1 = l N1
0.6931
Radioactivity at T2, R2 = l N2 \ l= 7
= 1.35 ´10 -11 s -1
\ Number of atoms decayed in time 1620 ´ 3.16 ´ 10
Because half life is very much large as compared to its time
(R 1 - R 2 ) (R1 - R 2 ) T interval, hence N » N0
(T1 – T2) = (N1 –N2) = =
l 0.693
dN
i.e., (T1 – T2) µ (R1 - R 2 ) T = lN = l N 0 or dN = lN 0 dt
dt
Example 5.
At any instant, the ratio of the amount of radioactive \ dN = (1.35 ´ 10 -11 ) (2.66 ´10 21 ) (1) = 3.61 ´ 1010 .
substances is 2 : 1. If their half lives be respectively 12 and Example 8.
16 hours, then after two days, what will be the ratio of g-ray photon produces an electron-positron pair, each
amount of the substances? moving with a K.E. of 0.01 MeV. Find the energy of the
Solution : g-ray photon.
Solution :
48
For first substance n = =4 1 electron mass = (9.1 × 10–31) × (3 × 108)2 J
12
4 (9.1 ´ 10 -31 ) (3 ´ 108 ) 2
æ1ö = MeV = 0.51 MeV
\ N1 = N 01 ç ÷ = N 01 / 16 ...(1) 1.6 ´ 10 -13
è2ø \ Energy of incident g-ray photon = (2 × 0.51 + 2 × 0.01)
48 MeV = 1.04 MeV
For second substance, n = =3
16 Example 9.
3 A radioactive material has a mean lives of 16 year and
æ1ö 405 year for a and b emission respectively. The material
N 2 = N 02 ç ÷ = N 02 / 8 ...(2)
è2ø decays by simultaneous a and b emission. Determine the
time in which 1/4th of the material is intact.
N1 N 01 1 1
Now, = ´ = 2 ´ = 1 (Q N01 = 2 N02) Solution :
N 2 N 02 2 2 Let la and lb are the decay constants for a and b emission
\ N1 : N2 = 1 : 1 respectively. The decay constant l for composite decay is
Example 6. l = la + lb.
Two radioactive substances X and Y initially contain equal 1 1 1 Ta Tb
number of nuclei. X has a half life of 1 hour and Y has half Further, = + or T =
T Ta Tb Ta + Tb
life of 2 hours. After two hours the ratio of the activity of X
to the activity of Y is 1620 ´ 405
\ T= = 325 year
(a) 1 : 4 (b) 1 : 2 1620 + 405
(c) 1 : 1 (d) 2 : 1
Free eBooks on @neetquestionpaper2020

Nuclei 969
Here 1/4th of the material is to remain intact i.e., 3/4th of the Number of deutrons used per second
material must have undergone decay
1016
N 1 1 -(0.693 / 325) t = = 2.82 ´ 10 27
\ = = e -lt or = e 1.6 ´ 10-19 ´ 22.13 ´ 106
N0 4 4 Time in which deutrons will be used
Solving we get t = 648 years. Number of deutrons
Example 10. =
Number of deutrons used per second
A nucleus breaks into two parts whose velocity is in ratio
of 2:1. Find the ratio of their radius. 1040
Solution : = 27
= 3.5 ´ 1012 sec
2.82 ´ 10
As per conservation of momentum m1v1 + m2v2 = 0 Example 13.
m1 v2 One gram of Radium emits 3.7 × 1010 a particles per sec-
so =
m 2 v1 ond. Calculate half life and mean life of Radium. Given:
Atomic mass of Radium = 226.
1/3 1/3 1/3
R1 æ A1 ö æ m1 ö æ 1ö Solution :
Ratio of radii = = = çè ÷ø
R 2 çè A 2 ÷ø çè m ÷ø
2 2 Rate of decay of Radium = rate of emission of a particles
\ R1 : R2 = 1 : 2 1/3 -dN
or = lN = 3.7 × 1010 per second
Example 11. dt
Calculate the binding energy per nucleon for 17C35. Given 6.023 ´ 1023 ´ 1
M (Cl 35 ) = 34.9800 amu. mn = 1.008665 amu and Number of active atoms N =
mP = 1.007825 amu. 226
Solution : 0.693 6.023 ´ 1023
\ lN = ´ = 3.7 ´ 1010
BE = ZmP + (A – Z) mn – M (Cl35) = 17 × 1.007825 + 18 × T 226
1.008665 – 34.9800 = 0.308995 amu or half life T = 1583 years
BE = 0.308995 × 931.5 = 287.83 MeV Mean life t = 1.44 T = 1.44 × 1580 = 2279 years
BE 287.75 Example 14.
B= = = 8.22 MeV Determine the disintegration energy of the process
A 35
Example 12. 210
¾¾ ®82 Pb 206 + 2 He 4 + Q if an a particle of en-
84 P0
A star initially has 1040 deutrons. It produces energy by ergy 5.3 MeV is emitted in it.
processes
Solution :
2 + 1H2 ¾¾® 1H3 + p
1H A-4
Kinetic energy of a particle E a = Q
and 1 + 1H2 H3¾¾® 2 He4
+n A
If average power radiated by star is 1016W then find time A 210
so Q = Ea = ´ 5.3 = 5.40 MeV
in which deutron is exhausted. A-4 210 - 4
M(1H2) = 2.01471 amu M(2He4) = 4.00388 amu Example 15.
mP = 1.00783 amu and mn = 1.00866 amu The activity of a radioactive substance drops to 1/32 of its
Solution : initial value in 7.5 h. Find the half life.
Solution :
Adding the two equation 31H2 ¾¾® 2He4 + p + n
t /T 7.5/T
mass defect = 3 × 2.01471 – 4.00388 – 1.00783 – 1.00866 A æ 1ö 1 æ 1ö
Using =ç ÷ or =ç ÷
= 0.02376 amu = 0.02376 × 931.5 MeV A0 è 2ø 32 è 2 ø
= 22.13 MeV 5 7.5/T
Power of star = 1016W = 1016 J/s æ 1ö æ 1ö 7.5
or çè ÷ø =çè ÷ø or 5= i.e. T = 1.5 hour
2 2 T
Free eBooks on @neetquestionpaper2020

970 Physi cs

28.1
Solve following problems with the help of above text and 11. Control rods used in nuclear reactors are made of
examples. (a) stainless steel (b) graphite
1. The nuclear radius is of the order of (c) cadmium (d) plutonium
(a) 10–10 m (b) 10–6 m 12. Boron rods in a nuclear reactor are used to
(a) absorb excess neutrons
(c) 10–15 m (d) 10–14 m
(b) absorb alpha particle
2. When the number of nucleons in nuclei increases, the
(c) slow down the reaction
binding energy per nucleon
(d) speed up the reaction
(a) increases continuously with mass number 13. What is the mass of one curie of U234 ?
(b) decreases continuously with mass number (a) 3.7 × 1010 g (b) 3.7 × 10–10 g
(c) remains constant with mass number –34
(c) 6.25 × 10 g (d) 1.438 × 10–11 g
(d) first increases and then decreases with increase of 14. A moderator is used in nuclear reactors in order to
mass number (a) slow down the speed of the nuetrons
3. Neutron decay in free space is given as follows (b) acceleerate the neutrons
1 (c) increase the number of neutrons
0n ®1 H1 + -1 e 0 + [ ] (d) decrease the number of neutrons
Then the parenthesis [ ] represents a 15. Thermal neutrons are those which
(a) neutrino (b) photon (a) are at very high temperature
(c) antineutrino (d) graviton (b) move with high velocities
4. Radioactivity is (c) have kinetic energies similar to those of the
(a) irreversible process surrounding molecules
(b) self disintegration process (d) are at rest
(c) spontaneous 16. Nuclear energy is released in fission since binding energy
(d) all of the above per nucleon is
(a) sometimes larger and sometimes smaller
5. g-rays are deflected by
(b) larger for fission fragments than for parent nucleus
(a) an electric field but not by a magnetic field
(c) same for fission fragments and nucleus
(b) a magnetic field but not by an electric field
(d) smaller for fission fragments than for parent nucleus
(c) both electric and magnetic field
17. Fusion reactions take place at high temperature because
(d) neither by electric field nor by magnetic field
(a) atoms are ionised at high temperature
6. Beta rays emitted by a radioactive material are (b) molecules break up at high temperature
(a) electromagnetic radiations (c) nuclei break up at high temperature
(b) the electrons orbiting around the nucleus (d) kinetic enrgy is high enough to overcome repulsion
(c) charged particles emitted by nucleus between nuclei
(d) neutral particles 18. In a given reaction
7. The antiparticle of electron is
(a) positron (b) a-particle zA ® z +1 Y A ® z -1 K A - 4 ® z -1 K A - 4
A

(c) proton (d) b-particle Radioactive radiations are emitted in the sequence of
8. Particles which can be added to the nucleus of an atom (a) a, b, g (b) g, a, b
without changing its chemical properties are called (c) b, a, g (d) g, b, a
(a) neutrons (b) electrons 19. The mass number of He is 4 and that for sulphur is 32. The
(c) protons (d) alpha particles radius of sulphur nuclei is larger than that of helium by
9. Which of the following is not a mode of radioactive decay ? (a) (b) 4
8
(a) Positron emission (b) Electron capture
(c) Fusion (d) Alpha decay (c) 2 (d) 8
20. A nucleus splits into two nuclear parts which have their
10. The half-life period and the mean life period of a radioactive
velocity ratio equal to 2 : 1. What will be the ratio of their
element are denoted respectively by Th and Tm. Then
nuclear radius?
(a) Th = Tm (b) Th > T m
(a) 21/3 : 1 (b) 1 : 21/3
(c) Th < Tm (d) Th ³ Tm 1/2
(c) 3 : 1 (d) 1 : 31/2

ANSWER KEY
1. (d) 2. (d) 3. (c) 4. (d) 5. (d) 6. (c) 7. (a) 8.(a) 9. (c) 10.(c)
11.(c) 12.(a) 13.(d) 14.(c) 15.(c) 16.(b) 17.(d) 18.(c) 19.(c) 20.(b)
Free eBooks on @neetquestionpaper2020

Nuclei 971

Very Short / Short Answer Questions 17. Distinguish between the phenomena of nuclear fission and
1. How is the radius of a nucleus related to its mass number? fusion.
[Outside Delhi - 2011 COMPTT.] Explain, using the graph for the B.E. / A versus mass number
2. How is the size of a nucleus experimentally determined? (A). How the release in energy can be accounted for in the
Write the relation between the radius and mass number of two processes. [Outside Delhi - 2010 COMPTT.]
the nucleus. [Delhi Board - 2011 COMPTT.] 18. Draw the binding energy per nucleon curve and draw
3. Two nuclei have mass numbers in the ratio of 2 : 5. What is conclusions from what you observe.
the ratio of their nuclear densities? [Delhi Board - 2009] 19. Chlorine has two isotopes having masses 34.98 u and
4. Assuming the nuclei to be speherical in shape, how does 36.98 u. Their relative abundance are 75.4% and 24.6%. What
the surface area of a nucleus of mass number A1 compare
is the average mass of a chlorine atom.
with that of a nucleus of mass number A2?
[Outside Delhi - 2008 COMPTT.] Multiple Choice Questions
5. State two characteristic properties of nuclear force :
[Outside Delhi - 2008] 20. In gamma ray emission from a nucleus
6. What are moderators in a nuclear reactor? (a) only the proton number changes
7. What is multiplication factor of a fission reaction? (b) both the neutron number and the proton number
8. Why is nuclear fission an important source of power change
generation? (c) there is no change in the proton number and the
9. How many electrons, protons and neutrons are there in neutron number
27
13 X element? (d) only the neutron number changes
10. A radioactive isotope of silver has half life of 20 minutes.
21. Fusion reaction occurs at temperatures of the order of
What fraction of the original mass would remain after 1
hour? (a) 103 K (b) 107 K (c) 10 K (d) 104 K
22. An element A decays into an element C by a two step process
235 1
11. Complete the equation . U + 0n ¾¾
®
A ® B + 2He4 and B ® C + 2e - . Then,
92
12. Calculate value of 1 a. m. u.
(a) A and C are isotopes (b) A and C are isobars
13. In general we say that mass energy interconversion takes
place only in nuclear reaction and not in chemical reactions (c) B and C are isotopes (d) A and B are isobars
which is wrong. Explain briefly. 23. Nuclear forces are
14. What is the role of control rods in a reactor. Why are they (a) spin dependent and have no non-central part
made of cadmium? (b) spin dependent and have a non-central part
Long Answer Questions (c) spin independent and have no non-central part
(d) spin independent and have a non-central part
15. Answer the following giving reasons
24. Actinium 231, 231 AC89, emit in succession two b particles,
(i) Why is the binding energy per nucleon found to be
four a-particles, one b and one a plus several g rays. What
constant for nuclei in the range of mass number (A)
lying between 30 and 170? is the resultant isotope?
(ii) When a heavy nucleus with mass number A = 240 (a) 221 Au79 (b) 211 Au 79
breaks into tow nuclei, A = 120, energy is released in (c) 221 Pb 82 (d) 211 Pb 82
the process. [Outside Delhi - 2012 COMPTT.] 25. In which sequence the radioactive radiations are emitted in
(iii) In B-decay, the experimental detection of neutrinos the following nuclear reaction?
(or antineutrinos) is found to be extremely difficult. A A
ZX ¾¾
® Z + 1Y ¾¾
®
16. Draw a plot of potential energy of a pair of nucleons as a
function of their separations. Mark the regions where the A–4 ¾¾
®Z KA–4
Z–1K – 1
nuclear force is (i) attractive and (ii) repulsive. Write any (a) g, a, b (b) a, b, g
tow characteristic features of nuclear forces.
(c) b, g, a (d) b, a, g
[Outside Delhi - 2012]
Free eBooks on @neetquestionpaper2020

972 Physi cs
26. Radioactive substance emits 27. Half life of a radioactive substance is 20 minute. Difference
(a) a-rays (b) b-rays between points of time when it is 33% disintegrated and
(c) g-rays (d) All of the above 67% disintegrated is approximately
(a) 40 minute (b) 10 minute
(c) 15 minute (d) 20 minute

1. The activity of a radioactive sample is measured as 9750


8. The mass of a 37 Li nucleus is 0.042 u less than the sum of
counts per minute at t = 0 & 975 counts per minute at T = 5
minutes. The decay constant is approximately. the masses of all its nucleons. The binding energy per
[CBSE PMT 1997]
nucleon of 37 Li nucleus is nearly [CBSE PMT 2010]
(a) 0.922 per minutes (b) 0.691 per minutes
(c) 0.461 per minutes (d) 0.230 per minutes (a) 46 MeV (b) 5.6 MeV
2. In a fission reaction [CBSE PMT 1997] (c) 3.9 MeV (d) 23 MeV
236
92 U ®117 X +117 Y + n + n 9. The activity of a radioactive sample is measured as
the binding energy per nucleon of X & Y is 8.5 MeV. Whereas N0 counts per minute at t = 0 and N0/e counts per minute at
of 236U is 7.6 MeV. The total energy liberated will be about t = 5 minutes. The time (in minutes) at which the activity
(a) 2000 MeV (b) 200 MeV reduces to half its value is [CBSE PMT 2010]
(c) 2 MwV (d) 200 KeV
5
3. Half lives of two radio active substance A & B are (a) log e 2 / 5 (b) log 2
respectively 20 minutes & 40 minutes. Initially the samples e
of A & B have equal numbers of nulcei. After 80 minutes the (c) 5 log 102 (d) 5 loge 2
ratio of remaining numbers of A & B nuclei is 10. The half life of a radioactive isotope 'X' is 50 years. It decays
[CBSE PMT 1998] to another element 'Y' which is stable. The two elements 'X'
(a) 1 : 16 (b) 4 : 1 and 'Y' were found to be in the ratio of 1 : 15 in a sample of
(c) 1 : 4 (d) 1 : 1 a given rock. The age of the rock was estimated to be
4. A duetron strikes 8 O16 nucleus with subsequent emission [CBSE PMT 2011]
of alpha particle. Identify the nucleus so produced (a) 150 years (b) 200 years
[CBSE PMT 2002] (c) 250 years (d) 100 years
(a) 7 (b) 10 11. A radioactive nucleus of mass M emits a photon of
3 Li 5B
13 14 frequency n and the nucleus recoils. The recoil energy will
(c) 7N (d) 7N be [CBSE PMT 2011]
5. For a nuclear fusion process, suitable nuclei are 2
(a) any Nuclei [CBSE PMT 2002] (a) Mc – hn (b) h n / 2Mc2
2 2

(b) heavy Nuclei (c) zero (d) hn


(c) light Nuclei m
(d) nuclei lying in the middle of periodic table 12. A nucleus n X emits one a-particle and two b-particles.
6. A nuclear reaction is given by [CBSE PMT 2003] The resulting nucleus is [CBSE PMT 2011]
A A 0
Z X ® Z+1Y + -1 e + n , represents (a) m -6 (b) m -6
n -4 Z n Z
(a) fission (b) b-decay
(c) s -decay (d) fusion m-4 m-4
(c) n X (d) n-2 Y
7. Mp denotes the mass of a proton and Mn that of a neutron.
A given nucleus, of binding energy B, contains Z protons 13. Fusion reaction takes place at high temperature because
and N neutrons. The mass M(N, Z) of the nucleus is given [CBSE PMT 2011]
by (c is the velocity of light) [CBSE PMT 2004] (a) nuclei break up at high temperature
(a) M(N, Z) = NMn + ZMp + B/c2 (b) atoms get ionised at high temperature
(b) M(N, Z) = NMn + ZMp – Bc2 (c) kinetic energy is high enough to overcome the
(c) M(N, Z) = NMn + ZMp + Bc2 coulomb repulsion between nuclei
(d) M(N, Z) = NMn + ZMp – B/c2 (d) molecules break up at high temperature
Free eBooks on @neetquestionpaper2020

Nuclei 973
14. Two radioactive nuclei P and Q, in a given sample decay 21. If N0 is the original mass of the substance of half-life period
into a stable nucleus R. At time t = 0, number of P species t1/2 = 5 years, then the amount of substance left after 15
years is [AIEEE 2002]
are 4 N0 and that of Q are N0. Half-life of P (for conversion
(a) N0/8 (b) N0/16
to R) is 1 minute where as that of Q is 2 minutes. Initially (c) N0/2 (d) N0/4
there are no nuclei of R present in the sample. When number
of nuclei of P and Q are equal, the number of nuclei of R 22. When a U 238 nucleus originally at rest, decays by emitting
an alpha particle having a speed ‘u’, the recoil speed of the
present in the sample would be [CBSE PMT 2011(M)]
residual nucleus is [AIEEE 2003]
9N 0
4u
(a) 3N0 (b)
2 (a) (b) - 4u
238 234
5N0 4u
(c)
2
(d) 2N0 (c) (d) - 4u
234 238
15. The half life of a radioactive nucleus is 50 days. The time 23. A radioactive sample at any instant has its disintegration rate
2 5000 disintegrations per minute. After 5 minutes, the rate is
interval (t 2 – t1) between the time t2 when of it has
3 1250 disintegrations per minute. Then, the decay constant
1 (per minute) is [AIEEE 2003]
decayed and the time t1 when of it had decayed is
3 (a) 0.4 ln 2 (b) 0.2 ln 2
[CBSE PMT 2012 (M)] (c) 0.1 ln 2 (d) 0.8 ln 2
(a) 30 days (b) 50 days 24. A nucleus with Z= 92 emits the following in a sequence:
(c) 60 days (d) 15 days a, b - , b - a, a, a, a, a, b - , b- , a, b + , b + , a
16. If the nuclear radius of 27Al is 3.6 fermi, the approximate
Then Z of the resulting nucleus is [AIEEE 2003]
nuclear radius of 64Cu in fermi is [CBSE PMT 2012(S)] (a) 76 (b) 78
(a) 2.4 (b) 1.2 (c) 82 (d) 74
(c) 4.8 (d) 3.6 25. Which of the following cannot be emitted by radioactive
17. A mixture consists of two radioactive materials A1 and A2 substances during their decay ? [AIEEE 2003]
(a) Protons (b) Neutrinoes
with half lives of 20 s and 10 s respectively. Initially the
(c) Helium nuclei (d) Electrons
mixture has 40 g of A1 and 160 g of A2. The amount of the 26. In the nuclear fusion reaction [AIEEE 2003]
two in the mixture will become equal after 2 3 4
[CBSE PMT 2012 (S)] 1 H + 1 H ® 2 He + n

(a) 60 s (b) 80 s given that the repulsive potential energy between the two
(c) 20 s (d) 40 s nuclei is ~ 7.7 ´ 10 -14 J , the temperature at which the gases
18. A certain mass of Hydrogen is changed to Helium by the must be heated to initiate the reaction is nearly [Boltzmann’s
process of fusion. The mass defect in fusion reaction is constant k = 1.38 ´ 10 -23 J / K ]
0.02866 a.m.u. The energy liberated per a.m.u. is (a) 107 K (b) 105 K
(Given : 1 a.m.u = 931 MeV) [NEET -2013] (c) 10 K3 (d) 109 K
(a) 26.7 MeV (b) 6.675 MeV 27. A nucleus disintegrated into two nuclear parts which have
(c) 13.35 MeV (d) 2.67 MeV their velocities in the ratio of 2 : 1. The ratio of their nuclear
19. The half life of a radioactive isotope ‘X’ is 20 years. It decays sizes will be [AIEEE 2004]
to another element ‘Y’ which is stable. The two elements (a) 3½ : 1 (b) 1:21/3
‘X’ and ‘Y’ were found to be in the ratio of 1 : 7 in a sample (c) 21/3:1 (d) 1:3½
of a the given rock. The age of the rock is estimated to be 28. The binding energy per nucleon of deuteron ( H) and
2
1

( He ) is 1.1 MeV and 7 MeV respectively..


[NEET -2013]
helium nucleus 4
(a) 60 years (b) 80 years 2
(c) 100 years (d) 40 years If two deuteron nuclei react to form a single helium nucleus,
20. At a specific instant emission of radioactive compound is then the energy released is [AIEEE 2004]
deflected in a magnetic field. The compound can emit (a) 23.6 MeV (b) 26.9 MeV
[AIEEE 2002] (c) 13.9 MeV (d) 19.2 MeV
(i) electrons (ii) protons 27
29. If radius of the 13 Al nucleus is estimated to be 3.6 fermi
(iii) He2+ (iv) neutrons
The emission at instant can be then the radius of 125 nucleus be nearly[AIEEE 2005]
52 Te
(a) i, ii, iii (b) i, ii, iii, iv
(c) iv (d) ii, iii (a) 8 fermi (b) 6 fermi
(c) 5 fermi (d) 4 fermi
Free eBooks on @neetquestionpaper2020

974 Physi cs
37. In gamma ray emission from a nucleus [AIEEE 2007]
7
66
30. Starting with a sample of pure Cu ,
of it decays into (a) only the proton number changes
8 (b) both the neutron number and the proton number
Zn in 15 minutes. The corresponding half life is change
[AIEEE 2005] (c) there is no change in the proton number and the
(a) 15 minutes (b) 10 minutes neutron number
(d) only the neutron number changes
1
(c) 7 minutes (d) 5 minutes 38. The half-life period of a radio-active element X is same as
2 the mean life time of another radio-active element Y. Initially
31. A nuclear transformation is denoted by X (n, a ) 73 Li . Which they have the same number of atoms. Then[AIEEE 2007]
(a) X and Y decay at same rate always
of the following is the nucleus of element X ?[AIEEE 2005] (b) X will decay faster than Y
(a) 10 (b) 12 (c) Y will decay faster than X
5 Be C6
(d) X and Y have same decay rate initially
(c) 11 (d) 9 39. There is a plot of binding energy per nucleon Eb, against
4 Be 5B
32. When 7 the nuclear mass M; A, B, C, D, E, F correspond to different
3Li
nuclei are bombarded by protons, and the
resultant nuclei are 4Be8, the emitted particles will be nuclei.
[AIEEE 2006]
(a) alpha particles (b) beta particles
(c) gamma photons (d) neutrons
33. The energy spectrum of b-particles [number N(E) as a
function of b-energy E] emitted from a radioactive source is
[AIEEE 2006]

Consider four reactions [AIEEE 2009]


(a) N(E) (b) N(E) (i) A + B ® C + e (ii) C ® A + B + e
(iii) D + E ® F + e and (iv) F® D + E + e,
E E
E0 E0 where e is the energy released? In which reactions is e
positive?
(a) (i) and (iii) (b) (ii) and (iv)
(c) N(E) (d) N(E) (c) (ii) and (iii) (d) (i) and (iv)
DIRECTIONS (Qs. 40 and 41) Read the passage given below and
E E
E0 E0 answer the questions that follow :
PASSAGE-1
34. If the binding energy per nucleon in 73 Li and 42 He nuclei
A nucleus of mass M + Dm is at rest and decays into two daughter
are 5.60 MeV and 7.06 MeV respectively, then in the reaction
M
nuclei of equal mass each. Speed of light is c.
p + 73 Li ¾¾ ® 2 42 He 2
energy of proton must be [AIEEE 2006] 40. The binding energy per nucleon for the parent nucleus is
(a) 28.24 MeV (b) 17.28 MeV E1 and that for the daughter nuclei is E2. Then
(c) 1.46 MeV (d) 39.2 MeV
[AIEEE 2010]
35. The 'rad' is the correct unit used to report the measurement
of [AIEEE 2006] (a) E2 = 2E1 (b) E1 > E2
(a) the ability of a beam of gamma ray photons to produce (c) E2 > E1 (d) E1 = 2 E2
ions in a target
(b) the energy delivered by radiation to a target 41. The speed of daughter nuclei is [AIEEE 2010]
(c) the biological effect of radiation
(d) the rate of decay of a radioactive source Dm 2Dm
(a) c (b) c
36. If MO is the mass of an oxygen isotope 8 O 17 ,M
P and MN
M + Dm M
are the masses of a proton and a neutron respectively, the
nuclear binding energy of the isotope is [AIEEE 2007] Dm Dm
(a) (MO –17MN)c2 (b) (MO – 8MP)c2 (c) c (d) c
M M + Dm
(c) (MO– 8MP –9MN)c2 (d) MOc2
Free eBooks on @neetquestionpaper2020

Nuclei 975
42. A radioactive nucleus (initial mass number A and atomic 49. The activity of a freshly prepared radioactive sample is
number Z emits 3 a - particles and 2 positrons. The ratio of 1010 disintegrations per second, whose mean life is 109 s.
number of neutrons to that of protons in the final nucleus The mass of an atom of this radioisotope is 10–25 kg. The
will be [AIEEE 2010]
mass (in mg) of the radioactive sample is [IIT-JEE 2011]
A- Z -8 A- Z -4 (a) 1 (b) 3
(a) (b)
Z -4 Z -8 (c) 5 (d) 6
A - Z - 12 A- Z -4
(c) (d) Paragraph for Questions 50 and 51
Z -4 Z -2
The mass of a nucleus A is less than the sum of the masses of
43. The half life of a radioactive substance is 20 minutes. The ZX
approximate time interval (t2 – t1) between the time t2 when (A-Z) number of neutrons and Z number of protons in the nucleus.
2 1 The energy equivalent to the corresponding mass difference is
of it had decayed and time t1 when of it had decayed is known as the binding energy of the nucleus. A heavy nucleus of
3 3
[AIEEE 2011] mass M can break into two light nuclei of masses m1 and m2 only
(a) 14 min (b) 20 min if (m1 + m2) < M. Also two light nuclei of masses m 3 and m4 can
(c) 28 min (d) 7 min undergo complete fusion and form a heavy nucleus of mass M'
44. Assume that a neutron breaks into a proton and an electron. only if (m3 + m4) > M'. The masses of some neutral atoms are
The energy released during this process is : (mass of neutron given in the table below:
= 1.6725 × 10–27 kg, mass of proton = 1.6725 × 10–27 kg, 1 1.007825 u 2 2.014102 u
1H 1H
mass of electron = 9 × 10–31 kg). [AIEEE 2012]
(a) 0.73 MeV (b) 7.10 MeV 3 3.016050 u 4 4.002603 u
1H 2 He
(c) 6.30 MeV (d) 5.4 MeV
6 6.015123 u 7 7.016004 u
45. A 280 days old radioactive substance show an activity of 3 Li 3 Li
6000 dps, 140 days later it’s activity becomes 3000 dps. 70 82
What was its initial activity [IIT JEE 2004s] 30 Zn 69.925325 u 34 Se 81.916709 u
(a) 20000 dps (b) 24000 dps 152 151.919803 u 206 205.974455 u
64 Gd 82 Pb
(d) 12000 dps (d) 6000 dps
209 208.980388 u 210 209.982876 u
46. If a star can convert all the He nuclei completely into oxygen 83 Bi 84 Po
nuclei. The energy released per oxygen nuclei is [Mass of (1u = 932 MeV/c2)
He nucleus is 4.0026 amu and mass of Oxygen nucleus is 50. The kinetic energy (in keV) of the alpha particle, when the
15.9994 amu] [IIT JEE 2005s] 210
nucleus 84 Po at rest undergoes alpha decay, is
(a) 7.6 MeV (b) 56.12 MeV (a) 5319 (b) 5422 (JEE Adv. 2013)
(c) 10.24 MeV (d) 23.9 MeV (c) 5707 (d) 5818
221 51. The correct statement is
47. 87 Ra is a radioactive substance having half life of 4 days.
(a) The nucleus 63 Li can emit an alpha particle
Find the probability that a nucleus undergoes decay after
two half lives [IIT-JEE 2006] (b) The nucleus 210 can emit a proton
84 Po
1 (c) Deuteron and alpha particle can undergo complete
(a) 1 (b)
2 fusion
(d) The nuclei 70 and 82 can undergo complete
3 1 30 Zn 34 Se
(c) (d)
4 4 fusion
48. In the options given below, let E denote the rest mass 52. Match List I of the nuclear processes with List II containing
energy of a nucleus and n a neutron.The correct option is parent nucleus and one of the end products of each process
[IIT-JEE 2007] and then select the correct answer using the codes given

( ) ( ) ( )
below the lists: (JEE Adv. 2013)
(a) E 236 U > E 137 I + E 97 Y + 2E(n) List I List II
92 53 39
15 15
P. Alpha decay 1. 8 O ® 7 O + ...
(b) (
92 ) ( ) ( )
E 236 U < E 137 I + E 97 Y + 2E(n)
53 39 Q. b+ decay 2. 138 234
92 U ® 90 Th + ...

(c) (
92 ) ( 56 ) (
36 )
E 236 U < E 140 Ba + E 94 Kr + 2E(n) R. Fission 3. 185 184
83 Bi ® 82 Pb + ...
239
® 140
( ) ( ) ( )
S. Proton emission 4. 94 Pu 57 La + ...
(d) E 236 U = E 140 Ba + E 94 Kr + 2E(n)
92 56 36
Free eBooks on @neetquestionpaper2020

976 Physi cs
Codes: 53. A freshly prepared sample of a radioisotope of half-life
P Q R S 1386 s has activity 103 disintegrations per second. Given
(A) 4 2 1 3 that ln 2 = 0.693, the fraction of the initial number of nuclei
(B) 1 3 2 4 (expressed in nearest integer percentage) that will decay in
(C) 2 1 4 3 the first 80 s after preparation of the sample is
(D) 4 3 2 1 (a) 4% (b) 8% (c) 10% (d) 25%
(JEE Adv. 2013)

1. If 1 mg of U235 is completely annihilated, the energy liberated 11. If u denotes 1 atomic mass unit. One atom of an element
is has mass exactly equal to Au, where A is mass number of
(a) 9 × 1010 J (b) 9 × 1019 J element.
(c) 9 × 10 J18 (d) 9 × 1017 J (a) A = 1 (b) A = 12
2. The energy released per fission of a 92U235 nucleus is nearly (c) A = 16
(a) 200 eV (b) 20 eV (d) A can take up any integral value from 1 to 110
(c) 200 MeV (d) 2000 eV 12. In the process of fission, the binding energy per nucleon
3. One milligram of matter convert into energy will give (a) increases
(a) 90 joule (b) 9 × 103 joule (b) decreases
5
(c) 9 × 10 joule (d) 9 × 1010 joule (c) remains unchanged
4. If Avogadro number is 6 × 1023, then number of protons, (d) increases for mass number A < 56 nuclei but
neutrons and electrons in 14 gms of 6C14 are respecitvely decreases for mass number A > 56 nuclei
(a) 36 × 1023, 48 × 1023, 36 × 1023 13 One curie is equal to
(b) 36 × 1023, 36 × 1023, 36 × 1023 (a) 3.7 × 1010 disintegration/sec
(c) 48 × 1023, 36 × 1023, 48 × 1023
(d) 48 × 1023, 48 × 1023, 36 × 1023 (b) 3.2 × 108 disintegration/sec
5. A radioactive substance has a half life of four months. Three (c) 2.8 × 1010 disintegration/sec
fourth of the substance will decay in (d) None of these
(a) three months (b) four months 14. A radioactive substance contains 10000 nuclei and its half-
(c) eight months (d) twelve months life period is 20 days. The number of nuclei present at the
6. If mass-energy equivalence is taken into account, when end of 10 days is
water is cooled to form ice, the mass of water should
(a) 7070 (b) 9000
(a) increase
(b) remain unchanged (c) 8000 (d) 7500
(c) decrease 15. The volume of a nucleus is directly proportional to
(d) first increase then decrease (a) A (b) A 3
7. The curve of binding energy per nucleon as a function of
atomic mass number has a sharp peak for helium nucleus. (c) A (d) A 1/3
This implies that helium 16. A gamma ray photon creates an electron-positron pair. If
(a) can easily be broken up the rest mass energy of an electron is 0.5 MeV and the total
(b) is very stable kinetic energy of the electron-positron pair is 0.78 MeV,
(c) can be used as fissionable meterial then the energy of the gamma ray photon must be
(d) is radioactive (a) 0.78 MeV (b) 1.78 MeV
8. The more readily fissionable isotope of uranium has an (c) 1.28 MeV (d) 0.28 MeV
atomic mass of 17. If the distance between nuclei is 2 × 10–13 cm, the density of
(a) 234 (b) 235 nuclear material is
(c) 236 (d) 238 (a) 3.21 × 10–12 kg/m3 (b) 1.6 × 10–3 kg/m3
9. The half life of the radioactive substance is 40 days. The 9
(c) 2 × 10 kg/m 3 (d) 1 × 1017 kg/m3
substance will disintegrate completely in 18. The half-life of radioactive Radon is 3.8 days. The time at
(a) 40 days (b) 400 days the end of which (1/20)th of the Radon sample will remain
(c) 4000 days (d) infinite time undecayed is (given log10e = 0.4343)
10. Which of the following is best nuclear fuel
(a) 13.8 days (b) 16.5 days
(a) thorium 236 (b) plutonium 239
(c) uranium 236 (d) neptunium 239 (c) 33 days (d) 76 days
Free eBooks on @neetquestionpaper2020

Nuclei 977
19. A freshly prepared radioactive source of half life 2 hr emits 27. At time t = 0, N1 nuclei of decay constant l1 and N2 nuclei
radiation of intensity which is 64 times the permissible safe
level. The minimum time after which it would be possible to of decay constant l 2 are mixed. The decay rate of mixture
work safely with this source is is
(a) 6 hr (b) 12 hr
(a) - N1 N 2 e -(l1 + l 2 ) t
(c) 24 hr (d) 128 hr
20. The intensity of gamma radiation from a given source is I0. æN ö
On passing through 37.5 mm of lead it is reduced to I0/8. (b) - çç 1 ÷÷e -(l1 + l 2 ) t
The thickness of lead which will reduce it to I0/2 is è N2 ø
(a) (37.5)1/3 mm (b) (37.5)1/4 mm t t
(c) 37.5/3 mm (d) (37.5/4) mm (c) - ( N1l1e -l1 + N1l 2 e -l 2 )
21. In the uranium radioactive series, the initial nucleus is 92U238
and that the final nucleus is 82Pb206. When uranium nucleus (d) - N1l1N 2 l 2 e (- l1 +l 2 ) t
decays to lead, the number of a particles and b particles 28. A radioactive nuclide is produced at the constant rate of n
emitted are per second (say, by bombarding a target with neutrons).
(a) 8a, 6 b (b) 6a, 7b The expected number N of nuclei in existence t seconds
(c) 6a, 8b (d) 4a, 3b
after the number is N0 is given by
22. Radioactive element decays to form a stable nuclide, then
the rate of decay of reactant is (a) N = N 0 e -lt
N N
n
(b) N= + N 0 e - lt
(a) (b) l

t t n æ nö
(c) N= + ç N 0 - ÷ e -lt
N N l è lø

(c) (d) n æ nö
(d) N= + ç N 0 + ÷ e - lt
t t l è lø
23. A radioactive nucleus undergoes a -emission to form a Where l is the decay constant of the sample
stable element. What will be the recoil velocity of the 29. The electrons cannot exist inside the nucleus because
daughter nucleus if v is the velocity of a emission?
(a) de-Broglie wavelength associated with electron
4v 2v
(a) (b) in b -decay is much less than the size of nucleus
A-4 A-4
(b) de-Broglie wavelength associated with electron
4v 2v
(c ) (d) in b -decay is much greater than the size of
A+4 A+4
nucleus
24. If an electron and positron annihilate, then the energy
released is (c) de-Broglie wavelength associated with electron
(a) 3.2 ´ 10 -13 J (b) 1.6 ´ 10 -13 J in b -decay is equal to the size of nucleus

(c) 4.8 ´ 10 -13 J (d) 6.4 ´ 10 -13 J (d) negative charge cannot exist in the nucleus
25. Radium 226 Ra, spontaneously decays to radon with the 30. A radioactive element forms its own isotope after 3
emission of an a-particle and a g ray. If the speed of the a consecutive disintegrations. The particles emitted are
particle upon emission from an initially stationary radium (a) 3 b–particles
nucleus is 1.5 ×107 m/s, what is the recoil speed of the
resultant radon nucleus? Assume the momentum of g ray is (b) 2 b–particles and 1 a–particle
negligible compared to that of a particle. (c) 3 b–particles and 1 a–particle
(a) 2..0 × 105 m/s (b) 2.7 × 105 m/s (d) 2 a–particles and 1 b–particle.
(c) 3.5 × 105 m/s (d) 1.5 × 107 m/s
26. If the end A of a wire is irradiated with a -rays and the other 31. An electron is
end B is irradiated with b -rays. Then (a) hadron (b) baryon
(a) a current will flow from A to B (c) a nucleon (d) a lepton.
(b) a current will flow from B to A 32. If the radius of a nucleus 256X is 8 fermi, then the radius of
(c) there will be no current in the wire 4He nucleus will be
(d) a current will flow from each end to the mid-point (a) 16 fermi (b) 2 fermi (c) 32 fermi (d) 4 fermi
of the wire
Free eBooks on @neetquestionpaper2020

978 Physi cs
33. Which of the following has the mass closest in value to 44. The mass and energy equivalent to 1 amu are respectively
that of the positron? (1 a.m.u. = 931 MeV) (a) 1.67 × 10–27 gm, 9.30 MeV
(a) Proton (b) Electron (b) 1.67 × 10–27 kg, 930 MeV
(c) Photon (d) Neutrino
(c) 1.67 × 10–27 kg, 1 MeV
34. Nucleus of an atom whose atomic mass is 24 consists of
(d) 1.67 × 10–34 kg, 1 MeV
(a) 11 electrons, 11 protons and 13 neutrons
(b) 11 electrons, 13 protons and 11 neutrons 45. From the following equations, pick out hte possible nuclear
(c) 11 protons and 13 neutrons reactions.
(d) 11 protons and 13 electrons (a) 6C13 + 1H1 ® 6C14 + 4.3 MeV
35. Order of magnitude of density of uranium nucleus is (b) 6C12 + 1H1 ® 7N13 + 2 MeV
(mp = 1.67 × 10–27 kg) (c) 7N14 + 1H1 ® 8O15 + 7.3 MeV
(a) 1020 kg / m3 (b) 1017 kg / m3 (d) 92U235 + 0n1 ® 54X140 + 38Si94 + 20n1 + g + 200 MeV
14
(c) 10 kg / m 3 (d) 1011 kg / m3
46. Which of the following statements is true?
36. Outside a nucleus
(a) neutron is stable (a) 78Pt192 has 78 neutrons
(b) proton and neutron both are stable (b) 84Po214 ® 82Pb210 + b–
(c) 238 ® Th 234 + He4
(c) neutron is unstable 92U 90 2
(d) neither neutron nor proton is stable (d) Th 234 ® Pa234 + He4
90 91 2
37. Atomic number of a nucleus is Z and atomic mass is M. The 47. Half life of radioactive element depends upon
number of neutron is (a) amount of element present
(a) M – Z (b) M (c) Z (d) M + Z (b) temperature
38. Which of the following nuclear reactions is not possible? (c) pressure
12 12 20
(a) 6C+ 6 C ¾¾
® 10 Ne + 42 He (d) nature of element
48. They decay constant of radium is 4.28 × 10–4 per year. Its
9 1
(b) 4 Be + 1H ® 63 Li + 42 He
¾¾ half life will be
11 1 (a) 2000 years (b) 1240 years
(c) 5 Be + 1H ® 94 Be + 42 He
¾¾
(c) 63 years (d) 1620 years
7 4
(d) ® 11H + 104 B
3 Li + 2 He ¾¾ 49. Consider a radioactive material of half-life 1.0 minute. If one
of the nuclei decays now, the next one will decay
2 4
39. The binding energy per nucleon for 1H and 2 He
(a) after 1 minute
respectively are 1.1 MeV and 7.1 MeV. The energy released
1
in MeV when two 12 H nuclei fuse to form 42 He is (b) after log 2 minute
e
(a) 4.4 (b) 8.2 (c) 24 (d) 28.4 1
40. Which one is correct about fission? (c) after minute, where N is the number of nuclei
N
(a) Approx. 0.1% mass converts into energy
present at that moment
(b) Most of energy of fission is in the form of heat
(c) In a fission of U235 about 200 eV energy is released (d) after any time
(d) On an average, one neutron is released per fission of 50. If 200 MeV energy is released in the fission of a single U235
U235 nucleus, the number of fissions required per second to
41. The average binding energy per nucleon is maximum for produce 1 kilowatt power shall be (Given 1eV = 1.6 × 10–19 J)
the nucleus (a) 3.125 × 1013 (b) 3.125 × 1014
(a) 2He 4 (b) 8O 16 (c) 3.125 × 1015 (d) 3.125 × 1016
(c) 26 Fe 56 (d) 92 U 238
42. The rest energy of an electron is é1ù
51. The fossil bone has a 14C : 12C ratio, which is ê ú of that
(a) 510 KeV (b) 931 KeV ë 16 û
(c) 510 MeV (d) 931 MeV in a living animal bone. If the half-life of 14C is 5730 years,
43. The mass defect per nucleon is called then the age of the fossil bone is
(a) binding energy (b) packing fraction (a) 11460 years (b) 17190 years
(c) ionisation energy (d) excitation energy
(c) 22920 years (d) 45840 years
Free eBooks on @neetquestionpaper2020

Nuclei 979
TA 60. The radioactivity of a sample is R1 at a time T1 and R2 at a
52. The ratio of half-life times of two elements A and B is T .
B time T2. If the half-life of the specimen is T, the number of
lA
The ratio of respective decay constant , is atoms that have disintegrated in the time (T1 – T2) is
lB
(a) TB / TA (b) TA / TB proportional to
(a) (R1T1 – R2T2) (b) (R1 – R2)
TA + TB TA - TB (c) (R1 – R2)/T (d) (R1 – R2) T
(c) (d)
TA TA 61. A radioactive element X converts into another stable
53. An archaeologist analyses the wood in a prehistoric element Y. Half life of X is 2 hrs. Initially only X is present.
After time t, the ratio of atoms of X and Y is found to be 1 :
structure and finds that C14 (Half life = 5700 years) to C12 is
4, then t in hours is
only one-fourth of that found in the cells of buried plants.
(a) 2 (b) 4
The age of the wood is about
(c) between 4 and 6 (d) 6
(a) 5700 years (b) 2850 years
62. After 150 days, the activity of a radioactive sample is 5000
(c) 11,400 years (d) 22,800 years dps. The activity becomes 2500 dps after another 75 days.
54. A radioactive nucleus undergoes a series of decay The initial activity of the sample is
according to the scheme (a) 20000 dps (b) 40000 dps
a b a g
(c) 7500 dps (d) 10000 dps
A ¾¾
® A1 ¾¾
® A 2 ¾¾
® A 3 ¾¾
® A4 2 4 56 235
63. If the total binding energies of 1 H, 2 He, 26 Fe & 92 U
If the mass number and atomic number of ‘A’ are 180 and 72 nuclei are 2.22, 28.3, 492 and 1786 MeV respectively, identify
respectively, then what are these numbers for A4 the most stable nucleus of the following.
(a) 172 and 69 (b) 174 and 70 56 2 235 4
(a) 26 Fe (b) 1H (c) 92 U (d) 2 He
(c) 176 and 69 (d) 176 and 70
64. A neutron travelling with a velocity v and kinetic energy E
55. 1 g of hydrogen is converted into 0.993 g of helium in a
has a perfectly elastic head-on collision with a nucleus of
thermonuclear reaction. The energy released is
an atom of mass number A at rest. The fraction of total
(a) 63 × 107 J (b) 63 × 1010 J
energy retained by the neutron is approximately
(c) 63 × 1014 J (d) 63 × 1020 J
(a) [(A – 1)/(A + 1)]2 (b) [(A + 1)/(A – 1)]2
56. The mass defect in a particular nuclear reaction is 0.3 grams.
(c) [(A – 1)/A]2 (d) [(A + 1)/A]2
The amount of energy liberated in kilowatt hour is
65. At any instant, the ratio of the amount of radioactive
(Velocity of light = 3 × 108 m/s)
substances is 2 : 1. If their half lives be respectively 12 and
(a) 1.5 × 106 (b) 2.5 × 106
16 hours, then after two days, what will be the ratio of the
(c) 3 × 106 (d) 7.5 × 106
substances ?
57. The binding energies per nucleon for a deuteron and an
(a) 1 : 1 (b) 2 : 1 (c) 1 : 2 (d) 1 : 4
a-particle are x1 and x2 respectively. What will be the energy
66. Half lives for a and b emission of a radioactive material are
Q released in the reaction 1H2 + 1H2 ® 2He4 + Q
16 years and 48 years respectively. When material decays
(a) 4(x1 + x2) (b) 4(x2 – x1)
giving a and b emission simultaneously, time in which 3/
(c) 2(x1 + x2) (d) 2(x2 – x1)
4th material decays is
58. Atomic weight of boron is 10.81 and it has two isotopes
10 11 10 11 (a) 29 years (b) 24 years
5B and 5B . Then ratio of 5B : 5B in nature would
(c) 64 years (d) 12 years
be
67. A sample of radioactive material decays simultaneously by
(a) 19 : 81 (b) 10 : 11
two processes A and B with half lives 1/2 and 1/4 hr.
(c) 15 : 16 (d) 81 : 19
respectively. For first half hour it decays with the process
59. The masses of neutron and proton are 1.0087 a.m.u. and
1.0073 a.m.u. respectively. If the neutrons and protons A, next one hour with the process B and for further half an
combine to form a helium nucleus (alpha particles) of mass hour with both A and B. If originally there were N 0 nuclei,
4.0015 a.m.u the binding energy fo the helium nucleus will the number of nuclei after 2 hour of such decay is
be (1 a.m.u. = 931 MeV) x
æ 1ö
(a) 28.4 MeV (b) 20.8 MeV N 0 ç ÷ then find the value of x.
è 2ø
(c) 27.3 MeV (d) 14.2 MeV (a) 5 (b) 4 (c) 3 (d) 8
Free eBooks on @neetquestionpaper2020

980 Physi cs
68. In an a-decay the kinetic energy of a-particle is 48 MeV electrons is known as plasma. The nuclei move randomly in the
and Q-value of the reaction is 50 MeV. The mass number of reactor core and occasionally come close enough for nuclear
the mother nucleus is X. Find value of X/25. fusion to take place. Usually, the temperatures in the reactor core
(Assume that daughter nucleus is in ground state) are too high and no material wall can be used to confine the
plasma. Special techniques are used which confine the plasma
(a) 2 (b) 4 (c) 6 (d) 8
for a time t0 before the particles fly away from the core. If n is the
69. A heavy nucleus having mass number 200 gets density (number/volume) of deuterons, the product nt0 is called
disintegrated into two small fragments of mass number 80 Lawson number. In one of the criteria, a reactor is termed
and 120. If binding energy per nucleon for parent atom is successful if Lawson number is greater than 5 × 10 14 s/cm3.
6.5 MeV and for daughter nuclei is 7 MeV and 8 MeV
It may be helpful to use the following: Boltzmann constant
respectively, then the energy released in the decay is
e2
X × 110 MeV. Find the value of X. k = 8.6 × 10 – 5 eV/K; = 1.44 × 10–9 eVm.
4pe 0
(a) 3 (b) 4 (c) 2 (d) 1
74. In the core of nuclear fusion reactor, the gas becomes plasma
DIRECTIONS (Qs. 70 to 73) : Each of these question contains
because of
two statements : Statement-1 (Assertion) and Statement-2
(Reason). Answer these questions from the following four (a) strong nuclear force acting between the deuterons
options. (b) coulomb force acting between the deuterons
(a) Statement-1 is True, Statement-2 is True; Statement-2 is a (c) coulomb force acting between deuteron-electron pairs
correct explanation for Statement -1 (d) the high temperature maintained inside the reactor core
(b) Statement-1 is True, Statement -2 is True; Statement-2 is 75. Assume that two deuteron nuclei in the core of fusion
NOT a correct explanation for Statement - 1 reactor at temperature T are moving towards each other,
each with kinetic energy 1.5 kT, when the separation between
(c) Statement-1 is True, Statement- 2 is False
them is large enough to neglect Coulomb potential energy.
(d) Statement-1 is False, Statement -2 is True Also neglect any interaction from other particles in the core.
70. Statement- 1: The binding energy per nucleon, for nuclei The minimum temperature T required for them to reach a
with atomic mass number A > 100, decrease with A. separation of 4 × 10–15 m is in the range
Statement- 2 : The forces are weak for heavier nuclei. (a) 1.0 × 109 K < T < 2.0 × 109 K
71. Statement- 1 : Radioactivity of 108 undecayed radioactive (b) 2.0 × 109 K < T < 3.0 × 109 K
nuclei of half life of 50 days is equal to that of 1.2 × 108
(c) 3.0 × 109 K < T < 4.0 × 109 K
number of undecayed nuclei of some other material with
half life of 60 days. (d) 4.0 × 109 K < T < 5.0 × 109 K
Statement- 2 : Radioactivity is proportional to half-life. 76. Results of calculations for four different designs of a fusion
reactor using D-D reaction are given below. Which of these
72. Statement- 1 : The ionising power of b-particle is less
is most promising based on Lawson criterion?
compared to a-particles but their penetrating power is more.
(a) Deuteron density = 2.0 × 1012 cm–3, confinement time
Statement- 2 : The mass of b-particle is less than the mass
of a-particle. = 5.0 × 10–3 s
73. Statement-I : Energy is released when heavy nuclei (b) Deuteron density = 8.0 × 1014 cm–3, confinement time
undergo fission or light nuclei undergo fusion and = 9.0 × 10–1 s
Statement-II : For heavy nuclei, binding energy per nucleon (c) Deuteron density = 4.0 × 1023 cm–3, confinement time
increases with increasing Z while for light nuclei it decreases = 1.0 × 10–11 s
with increasing Z. (d) Deuteron density = 1.0 × 1024 cm–3, confinement time
DIRECTIONS (Qs. 74 to 78 ) : Read the passages given below = 4.0 × 10–12 s
and answer the questions that follow : PASSAGE-2
PASSAGE-1 Gold nucleus (79Au198) can decay into mercury nucleus (80Hg198)
Scientists are working hard to develop nuclear fusion reactor. by two decay schemes shown in figure. (i) it can emit a b particle
(b1) and come to ground state by either emitting one g ray (g1) or
Nuclei of heavy hydrogen, 12 H, known as deuteron and denoted emitting two g rays (g3 & g4) (ii) it can emit one b particle (b2) and
by D, can be thought of as a candidate for fusion reactor. The come to ground state by emitting g2 ray.

D-D reaction is 2 2 3
1 H + 1 H ®2 He + n + energy. In the core of Atomic masses : 198
Au = 197.9682 amu ,
fusion reactor, a gas of heavy hydrogen is fully ionized into 198
Hg = 197.9662 amu , 1 amu = 930 MeV/c². The energy levels
deuteron nuclei and electrons. This collection of 2 nuclei and
1H of the nucleus are shown in figure.
Free eBooks on @neetquestionpaper2020

Nuclei 981
198
Au 77. What is the maximum kinetic energy of emitted b2 particles?
79
- (a) 1.44 MeV (b) 0.59 MeV
b1 (c) 1.86 MeV (d) 1.46 MeV
-
b2 78. The wavelength of emitted g rays are in the other
E3 = – 1 MeV
Second excited
state (a) l g 2 > l g 3 > l g1
g1 g3
(b) l g 3 > l g 2 > l g1
E2 = –1.6 MeV First excited
state (c) l g1 > l g 2 > l g 3
g2
E1 = – 2 MeV (d) l g 3 = l g 2 = l g1
Ground state
198
80
Hg
Free eBooks on @neetquestionpaper2020

Exercise 1 : NCERT Based Questions


Exercise 28.1
6. Light nuclei are provided in a reactor to slow down neutrons
1. (d) Nuclear radius = 10–14 m. in fission process. These are moderators e.g. Water, graphite
2. (d) Average BE/nucleon increases first, and then decreases, etc.
as is clear from BE curve. 7. The ratio,
3. (c) An electron is accompanied by an antineutrino.
4. (d) All the characteristics given are true for radioactivity. no.of fission produced by a given generation of neutrons
K=
5. (d) g-rays carry no charge. They are neither deflected by an no.of fission of preceeding generation
electric field nor by a magnetic field. is called multiplication factor.
6. (c) b-rays are charged particles emitted by nucleus.
8. Energy released in fission is million times more than other
7. (a) Antiparticle of electron ( –1e0) is positron ( +1e0)
chemical processes.
8. (a) Chemical properties are unaffected with addition of
neutrons to the nucleus. 9. No. of protons = 13, No. of electrons = 13,
No. of neutrons = 27 – 13 = 14
9. (c) Fusion is not a mode of decay.
1
0.693 1 10. N = N0
10. (c) Half life Th = , Tm = Clearly,, Th < Tm . 8
l l 235 1 147 86 1
11. U + 0n ¾¾
® 56 Ba + 36 kr + 3 0 n + 200MeV
11. (c) Control rods are made of cadmium. 92
12. (a) Boron rods absorb excess neutrons. 12. 1 a. m. u. = 1.660565 × 10–27 kg.
13. (d) One curie = 3.71 × 1010 disintegrations/sec
Mass of 6.023 × 1023 atoms of U234 = 234 g 19. = 35.47 u.
Mass of 3.71 × 1010 atoms 20. (c) 21. (b) 22. (a) 23. (b)
10
234 ´ 3.71 ´ 10 24. (d) 25. (d) 26. (d) 27. (d)
= = 1.438 ´ 10 -11 gm
6.023 ´ 10 23
14. (c) Moderator slows down neutrons. Exercise 2 : PAST Competition MCQs
15. (c) Thermal neutrons are slow neutrons having kinetic
energies similar to those of surrounding molecules. dN
16. (b) Nuclear energy is relased in fission because BE/nucleon
1. (c) = l N = activity R
dt
is larger for fission fragments than for parent nucleus.
17. (d) Extremely high temps needed for fusion make K.E. large R0 = lN0 at t = 0, R1 = lN1 at t = 5 minutes
enough to overcome repulsion between nuclei. 2.303 N
where l = log 0
æ A ö
1/ 3 1/ 3 t N1
Rs æ 32 ö
19. (c) =ç s ÷ =ç ÷ =2
R He è A He ø è 4ø 2. (b) Liberated energy Q = 117 × 8.5 + 117 × 8.5 – 236 × 7.6 =
200 MeV. Thus, in fission of one Uranium nuclei nearly
20. (b) As momentum is conserved, therefore,
200 MeV energy is liberated.
m1 A1 v 2 1
= = =
m 2 A 2 v1 2 N A (1 / 2) n A (1 / 2) 4 1
3. (c) = = = , where n A & n B are
N B (1 / 2) n B (1 / 2) 2 4
1/ 3 1/ 3
R1 æ A1 ö æ1ö number of half lives of samples A & B respectively. NA
\ = çç ÷÷ = ç ÷ = 1 : 21 / 3
R 2 è A2 ø è2ø & NB are the remaining numbers of A & B after 80 minutes
28. (a) Increase of charge no. by 1 indicates b emission. in this case.
Decrease of mass number by 4 and charge no. by 2 4. (d) 16
+1 H 2 ® 7 N14 + 2 He 4
8O
indicates a emission. No change of mass no. and charge
no. indicates g-emission. 5. (c)
Free eBooks on @neetquestionpaper2020

0 13. (c) When the coulomb repulsion between the nuclei is


6. (b) -1 e is known as b– particle & n is known as
overcome then nuclear fusion reaction takes place.
antineutrino. Since in this reaction n is emitted with This is possible when temperature is too high.
0 (b– particle or electron), so it is known as b-decay.. 14. (b) Initially P ® 4N0
-1 e
Q ® N0
B.E Half life TP = 1 min.
7. (d) Mass defect =
c2 TQ = 2 min.
Mass of nucleus = Mass of proton Let after time t number of nuclei of P and Q are equal,
+ mass of neutron – mass defect that is
8. (b) B.E. = 0.042 × 931 ; 42 MeV
4N0 N0 4N 1
= Þ = Þ 2t/1 = 4.2t/2
Number of nucleons in 37 Li is 7. 2t /1
2 t/2
2 t /1
2 t/2

2 t/2
2 .2 = 2 (2 + t/2)
42
\ B.E./ nucleon = = 6 MeV ; 5.6 MeV
7 t t t
Þ = 2 + Þ = 2 Þ t = 4 min
1 2 2
9. (d) N = N0 e-lt
(4N0 ) N0
Here, t = 5 minutes NP = =
4 /1 4
2
N0
= N 0 × e -5l at t = 4 min.
e
N0 N0
1 N0 = =
Þ 5l = 1 , or l= , 4 4
5 or population of R
l n2 æ N0 ö æ N0 ö 9N 0
Now, T1/2 = = 5 l n2 çè 4 N 0 - 4 ÷ø + çè N 0 - 4 ÷ø =
l 2
10. (b) Let number of atoms in X = Nx
Number of atoms in Y = Ny 1
15. (b) N1 = N0 e–lt N1 = N0
By question 3
Nx 1 N0
= = N0e -lt2
N y 15 3
1 1 2
\ Part of Nx = (N + N y ) Þ = e -lt ...(i)
16 x 3
1 2
=(Nx + N y ) N2 = N0
24 3
So, total 4 half lives are passed, so, age of rock is 4 ×
2
50 = 200 years N 0 = N0e -lt1
3
11. (b) Momentum
E hn 2
Mu = = Þ = e -lt1 ...(ii)
c c 3
Recoil energy Dividing equation (i) by equation (ii)

1 M2u 2
2
h 2 n2 1
1
Mu 2 = =
1 æ hn ö = e -l (t2 -t1 )
=
2 2 M 2M çè c ÷ø 2Mc2
2
l(t2 - t1 ) = ln2
12. (c) When m emits one a-particle then its atomic mass
n X
ln 2
decreases by 4 units and atomic number by 2. t 2 - t1 = = T1/2 = 50 days
l
Therefore, the new nucleus becomes m - 4 . But as it
n-2 Y 16. (c) The radius of the nuclears is directly proportional to
emitstwo b– particles, its atomic number increases by cube root of atomic number i.e. R µ A1/3
m-4
Þ R = R 0 A 1/3 , where R 0 is a constant of
2. Thus the resulting nucleus is n X. proportionality
Free eBooks on @neetquestionpaper2020

984 Physi cs

1/ 3 1/ 3 N 1 N 1
R2 æ A2 ö æ 64 ö 4 = = =
=ç ÷ ç ÷ = N0 - N 7 N0 8
R1 è A1 ø è 27 ø 3
t = 3T
where R1 = the radius of 27Al, and = 3 × 20 = 60 years
A1 = Atomic mass number of Al 20. (a) Neutrons can’t be deflected by a magnetic field.
R2 = the radius of 64Cu and A2 = Atomic mass number 21. (a) Amount left = N0/2n = N0/8 (Here n=15/5=3)
of C4 22. (a) Applying the principle of conservation of linear
momentum
4
R2 = 3.6 ´ = 4.8m 4u
3 (4) (u) = (v) (238) Þ v =
238
17. (d) Let, the amount of the two in the mixture will become
equal after t years. 1 æ N0 ö 1 æ 5000 ö
23. (a) K = ln çè ÷ Þ K = ln ç ÷
The amount of A1, which remains after t years t Nø 5 è 1250 ø
N 01 1 2
N1 = ln ( 4) = ln 2 = 0.4ln 2
(2)t/20 5 5
The amount of A2, which remains, after t years 24. (b) No. of a particles emitted = 8
No. of b– particles emitted = 4
N 02
N2 = No. of b+ particles emitted = 2
(2)t/10 Z = 92 – 2 × 8 + 4 – 2 = 78
According to the problem 25. (a) 26. (d)
N1 = N2 27. (b) From conservation of momentum m1v1 = m 2 v 2
40 160
= æm ö æv ö v m 1 r3 1
(2)t/20 (2)t/10 Þ ç 1 ÷ = ç 2 ÷ given 1 = 2 Þ 1 = Þ 1 =
è m2 ø è v1 ø v2 m2 2 r23 2
æ t ö
çè - 2÷ø
2t/20
=2 10 æ r ö æ 1 ö 1/ 3
Þç 1÷ = ç ÷
è r2 ø è 2 ø
t t
= -2
20 10 28. (a) The chemical reaction of process is 212 H ® 42 He
t t Energy released = 4 ´ (7 ) - 4(1.1) = 23.6 eV
- =2
20 10
29. (b) R = R 0 (A)1/ 3
t
=2 1/ 3
20 R æA ö æ 27 ö
1/3
3
\ 1 =ç 1÷ =ç =
t = 40 s R2 è A2 ø è 125 ÷ø 5
18. (b) Mass defect Dm = 0.02866 a.m.u.
Energy = 0.02866 × 931 = 26.7 MeV
5
As 1H2 + 1H2 ¾® 2He4 R2 = ´ 3.6 = 6 fermi
Energy liberated per a.m.u = 13.35/2 MeV 3
= 6.675 MeV
7
30. (d) of Cu decays in 15 minutes.
1 8
19. (a) The value of x is
8
3
x x 7 1 æ1ö
= 0 = 30 Þ t = 3T = 3 × 20 = 60 years \ Cu undecayed = N = 1 – = =ç ÷
8 8 8 è2ø
2
Hence the estimated age of the rock is 60 years \ No. of half lifes = 3
ALTERNATE : X ® Y0
at t = 0 N0 0 t 15
n= or 3 =
at t = t N N0 – N T T
Free eBooks on @neetquestionpaper2020

Nuclei 985

15 Þ l X = (0.693). l Y
Þ T = half life period = = 5 minutes
3 \ lX > lY .
Alternative method : N = N0 (1 – e–lt) Now, the rate of decay is given by
N0 - N
Þ = e-lt æ dN ö
N0 - ç ÷ = l X N0
è dt ø

1
\ = e -lt æ dN ö
8 -ç ÷ = l 4 N0
è dt ø4
3 ln 2 = lt
Y will decay faster than X.
3 ´ 0.693 39. (d) For A + B ® C + e, e is positive. This is because Eb
or l= = 0.1386
15
for C is greater than the E b for A and B.
0.693 0.693 Again for F ® D + E + e, e is positive. This is
Half-life period, t½ = = = 5 minutes
l 0.1386
because Eb for D and E is greater than Eb for F .
A 40. (c) In nuclear fission, the binding energy per nucleon of
31. (a) ZX + 0 n1 ¾¾
® 3 Li7 + 2 He 4
daughter nuclei is always greater than the parent
On comparison, nucleus.
A = 7 + 4 – 1 = 10, z = 3 + 2 – 0 = 5 41. (b) By conservation of energy,
It is boron 5B10
2.M 2 1 2M 2
7 1 8 0
( M + Dm) c2 = c + . v ,
32. (c) 3 Li + 1p ¾
¾® 4 Be + 0g
2 2 2
33. (c) The range of energy of b-particles is from zero to some where v is the speed of the daughter nuclei
maximum value.
2 M 2 2 Dm
34. (b) Let E be the energy of proton, then Þ Dmc = v \v=c
2 M
E + 7 ´ 5.6 = 2 ´ [4 ´ 7.06]
42. (b) As a result of emission of 1 a-particle, the mass number
Þ E = 56.48 - 39.2 = 17.28MeV decreases by 4 units and atomic number decreases by
35. (c) The risk posed to a human being by any radiation 2 units. And by the emission of 1 positron the atomic
exposure depends partly upon the absorbed dose, the number decreases by 1 unit but mass number remains
amount of energy absorbed per gram of tissue. constant.
Absorbed dose is expressed in rad. A rad is equal to
\ Mass number of final nucleus = A – 12
100 ergs of energy absorbed by 1 gram of tissue. The
more modern, internationally adopted unit is the gray Atomic number of final nucleus = Z – 8
(named after the English medical physicist L. H. Gray); \ Number of neutrons = (A – 12) – (Z – 8)
one gray equals 100 rad. =A–Z –4
36. (c) Binding energy Number of protons = Z – 8
= [ZMP + (A – Z)MN – M]c2
A-Z -4
= [8MP + (17 – 8)MN – M]c2 \ Required ratio =
Z -8
= [8MP + 9MN – M]c2
= [8MP + 9MN – Mo]c2 43. (b) Number of undecayed atom after time t2 ;
37. (c) There is no change in the proton number and the N0
= N 0 e -lt2 ...(i)
neutron number as the g-emission takes place as a 3
result of excitation or de-excitation of nuclei. Number of undecayed atom after time t1;
g-rays have no change or mass. 2N0
= N 0 e -lt1 ...(ii)
38. (c) According to question, 3

Half life of X, T1/2 = tav , mean life of Y -lt 1


From (i), e 2 =
3
0.693 1
Þ =
lX lY
Free eBooks on @neetquestionpaper2020

986 Physi cs

æ 1ö 1
Þ –lt2 = loge çè 3 ÷ø ...(iii) \ 1010 = ´N
109

From (ii) – e -lt2 =


2 \ N = 1019
3
i.e. 1019 radioactive atoms are present in the freshly
æ 2ö prepared sample.
Þ –lt1 = loge çè 3 ÷ø ...(iv)
The mass of the sample
Solving (iii) and (iv), we get
= 1019 × 10–25 kg = 10–6kg = 1 mg
t2 – t1 = 20 mm
210 206
1 50. (a) 84 Po ¾® 82 Pb + 42 He
44. (a) 0n ® 11H + -1e 0 + n + Q
¾¾
Here Dm = [209.982876 –(205.974455 + 4.002603)] × 932 MeV
The mass defect during the process
= 5.422 MeV = 5422 keV
Dm = mn - mH - me By conservation of linear momentum
= 1.6725 × 10–27 – (1.6725 × 10–27+ 9 × 10–31kg) Linear momentum of a-particle = linear momentum of
lead
= – 9 × 10–31 kg
pa = plead
The energy released during the process
2ma K.E a = 2m lead K.E lead
E = Dmc2
m lead ´ K.E lead 206 ´ K.E lead
E = 9 × 10–31× 9 × 1016 = 81 × 10–15 Joules \K.E = = ...(1)
ma 4
81 ´ 10 -15 Also K.Ea + K.Elead = 5422 keV ...(2)
E= = 0.511MeV
1.6 ´ 10 –19 On solving the above two equations we get
K.Ea = 5319 keV
2.303 A
45. (b) We know that l = log 0 (a) is the correct option.
t A 51. (a) Only in case of c we have m3 + m4 > M¢
where A0 is the initial activity (c) is the correct option.
A is the activity at time t
In other cases of fission m1 + m2 < M and in the other
2.303 A 2.303 A case of fusion m3 + m4 > M¢
\ l= log 0 = log 0
280 6000 420 3000 15
52. (c) 8 O ¾® 15
7 N+
0
1b
On solving we get A0 = 24000 dps.
b + particle
16
46. (c) 4 42 He ¾
¾® 8O
238
¾® 234
+ 4
92 U 90 Th 2 He
B.E = DM × 931.5 MeV a- particle
= (4 × 4.0026 – 15.9994) × 931.5 = 10.24 MeV 185
83 Bi ¾® 184 1
82 Pb + 1 H
47 (b) Radioactive decay is a random process. Each decay is a proton
completely independent event. Therefore, which
239 140
particular nucleus will decay at a given instant of time 94 Pu ¾® + 99
57 La 37 X
cannot be predicted. In other words when a particular (c) is the correct option.
nucleus will decay cannot be predicted. Each nucleus 53. (a) For a radioactive decay
has same probability of disintegration.
N = N 0 e- l t
48. (a) Iodine and Yttrium are medium sized nuclei and
therefore have more binding energy per nucleon as N
\ = e-l t
compared to Uranium which has a big nucleus and N0
less B.E. / nucleon. In other words, Iodine and Yttrium
are more stable and therefore, possess less energy N
\1 - = 1 - e-l t
and less rest mass. Also, when Uranium nuclei N0
explodes, it will convert into I and Y nuclei having
0.693
kinetic energies. - ´t
N -N t1 2
\ 0 = 1- e
dN 1 N
49. (a) We know that , = lN = N
dt Tmean = 1 - e -0.04 = 1 - (1 - 0.04)

» 0.04 = 4% [Q e- x = 1 - x x <<1]
Free eBooks on @neetquestionpaper2020

Nuclei 987
Exercise 3 : Conceptual & Applied MCQs n
æ1ö N 1
18. (b) ç ÷ = = gives n = 4.32
1. (a) E = mc2 = 10–6 × (3 × 108)2 = 10–6 × 9 × 1016 = 9 × 1010 J è2ø N 0 20
2. (c) Energy released per fission is » 200 MeV t = nT = 4.3 × 3.8 = 16.5 days
19. (b) To work safely, intensity must reduce by 1/64
3. (d) E = mc 2 = 10 -3 ´ 10 -3 (3 ´108 ) 2 = 9 × 1010 J t /T 6 t/T
N 1 æ1ö æ1ö æ1ö
4. (a) Each atom of 6C14 contains 6 p, 6 e and 8 n \ = =ç ÷ i.e. ç ÷ = ç ÷
\ In 14 gram of 6C14 N 0 64 è 2 ø è2ø è2ø
t
p = 6 × 6 × 1023 = 36 × 1023 or = 6 or t = 6 T = 12 hrs
T
n = 8 × 6 × 1023 = 48 × 1023
I
e = p = 36 × 1023 20. (c) As = e -mx
I0
3 1
5. (c) Substance left undecayed, N 0 - N0 = N0
1 1 -mx
4 4 \ = e -m 37.5 .......(i) and = e .......(ii)
8 2
n
N 1 æ1ö Put (ii) in (i), e –3 m x = e– m (37.5)
= =ç ÷
N0 4 è 2 ø 37.5
x= = 12.5 mm
\ Number of atoms left undecayed, 3
n = 2 i.e. in two half lives 21. (a) Let no. of a-particles emitted be x and no. of b particles
\ t = nT = 2 × 4 = 8 months emitted be y.
6. (a) Because thermal energy decreases, therefore mass Diff. in mass no. 4x = 238 – 206 = 32 Þ x = 8
Diff. in charge no. 2x – 1y = 92 – 82 = 10
should increase.
16 – y = 10, y = 6
7. (b) 8. (b)
9. (d) Time taken to disintegrate completely by a substance is 22. (c) No. of nuclide at time t is given by N = N o e - lt
N0 Where N o = initial nuclide
infinity as log = lt
N
thus this equation is equivalent to y = ae - kx
N0 thus correct graph is
Þ log = lt
0 N
log ¥ = lt
hence when N ® 0, t ® ¥ .
10. (b) Plutonium 239 is processed by breeder mechanism to
be used as nuclear feul. t
11. (b) 12. (a) 13. (a) 23. (a) We assume that mass number of nucleus when it was at
t 10 rest = A
N æ 1 öT N æ 1 ö 20 Q mass number of a -particle = 4
14. (a) =ç ÷ or =ç ÷
N0 è 2 ø 10000 è 2 ø \ mass number of remaining nucleus = A - 4
As there is no external force, so momentum of the system
10000 ´1 10000 will remain conserved
or N = = = 7070.
2 1.414 4v
Þ 0 = (A - 4)v¢ + 4v Þ v' = -
15. (a) Radius of nucleus R = R0 A1/3 where A is the mass (A - 4)
number of nucleus. negative sign represents that direction is opposite to
the direction of motion of a -particle.
4 3 æ4 ö
\ Volume of nucleus = pR = ç p R 30 ÷ A 24. (b) Energy of electron = m e c 2
3 è3 ø
\ Volume is proportional to A. Energy of positron = m p c 2
16. (b) Energy of g-ray photon = Rest mass energy + K.E.
me = m p , c = speed of light.
= 2 (0.5) + 0.78 = 1.78 MeV
17. (d) Density of nuclear material = mass/volume Thus according to conservation of energy released
= 2m e c 2
10-27 3 ´ 10-27
= 1017 kg / m3
= =
4 3 4p (2 ´ 10-15 )3
pr (
= 2 ´ 9.1 ´ 10 -31 3 ´108 )
2
= 1.6 ´ 10 -13 Joules .
3
Free eBooks on @neetquestionpaper2020

988 Physi cs
25. (b) Conservation of linear momentum requires: 33. (b) Positron is the antiparticle of electron.
mradonvradon= mheliumvhelium with helium identified as the 34. (c) Nucleus does not contain electron.
alpha particle. The nuclear masses can be approximated 35. (b) The order of magnitude of mass and volume of uranium
by their mass numbers (222 and 4). Thus, the recoil nucleus will be
speed of the radon is (4/222) × 1.5 × 107 m/s = 2.7 x 105 m ; A (1.67 × 10–27 kg) (A is atomic number)
m/s.
26. (a) Due to irradiation of a-rays on end A will make it 4 3 4
V= pr ; p[(1.25 ´ 10-15 m)A1/ 3 ]3
(positive) and irradiation of b-rays on end B will make 3 3
it (negative) hence current will flow from A to B (or ; (8.2 × 10–45m3)A
from positive to negative).
27. (d) m A(1.67 ´ 10 -27 kg)
Hence, r = =
dN V (8.2 ´ 10-45 m3 )A
28. (c) = n - lN
dt
; 2.0 × 1017 kg / m3.
dN = (n - lN)dt 36. (c) Out side the nucleus, neutron is unstable (life » 932
N t N
s).
dN 1 - ldN
ò n - lN ò
= dt Þ -
l ò n - lN = t 37.
38.
(a) N = M – Z = Total no. of nucleons – no. of protons.
(c) In this reaction mass is not conserved.
N0 0 N0
39. (c) The chemical reaction of process is 212 H ® 24 He
1
Þ- [log e (n - lN )]NN0 = t Energy released = 4 ´ (7.1) - 4(1.1) = 24 eV
l
40. (a)
1é æ n - lN ö ù 41. (c) Binding energy per nucleon increases with atomic
Þ- êlog e ç ú=t
l ëê è n - lN0 ÷ø ûú number. The greater the binding energy per nucleon
the more stable is the nucleus.
é æ n - lN 0 öù For 26Fe56 number of nucleons is 56.
Þ lt = êlog e ç ÷ú This is most stable nucleus, since maximum energy is
ë è n - lN øû
needed to pull a nucleon away from it.
n - lN 0 42. (a) Rest energy of an electron = mec2
e lt =
n - lN Here me = 9.1 × 10–31 kg and C = velocity of light
\ Rest energy = 9.1 × 10–31 × (3 × 108)2 joule
n - lN = (n - lN 0 ) e - lt

n æn ö 9.1 ´ 10-31 ´ (3 ´ 108 ) 2


- ç - N 0 ÷ e -lt = N = eV » 510 KeV
l èl ø 1.6 ´ 10-19
29. (b) 43. (b) 44. (b) 45. (b, c) 46. (c)
30. (b) A nucleus is denoted by ZXA 47. (d) Half life of a substance doesn’t depends upon amount,
An isotope should have same Z. temperature and pressure. It depends upon the nature
a–particle = 2He4; b–particle = –1b0 of the substance.
The emission of one a particle and the emission of
two b particles maintain the Z same. 0.6931 ´ 1 0.6931
48. (d) T= = year = 1620 years
Hence, for isotope formation 2b particles and one a l 4.28 ´ 10-4
particle are emitted.
49. (d) Because radioactivity is a spontaneous phenomenon.
31. (d) An electron is a lepton.

32. (b) R = R 0 (A)1/ 3 æ Eö n ´ 200 ´ 106 ´ 1.6 ´ 10 -19


50. (a) P = n ç ÷ Þ 1000 =
1/ 3 è tø t
1/ 3
R1 æ A1 ö æ 256 ö
\ =ç = çè ÷ =4
R 2 è A 2 ÷ø 4 ø n
R Þ = 3.125 ´ 1013.
R 2 = 1 = 2 fermi t
4
Free eBooks on @neetquestionpaper2020

Nuclei 989
\ Number of atoms decayed in time
14 1 N
C =
51. (c) (T1 – T2) = (N1 –N2)
12 = 16 N0
C
(R 1 - R 2 ) (R1 - R 2 ) T
N æ 1ö
n = = µ (R1 - R 2 ) T
Q = ç ÷ l 0.693
N0 è 2ø
61. (c) Let N0 be the number of atoms of X at time t = 0.
n 4 n
1 æ 1ö æ 1ö æ 1ö Then at t = 4 hrs (two half lives)
Þ = çè ÷ø Þ çè ÷ø = çè ÷ø
16 2 2 2 N0 3N0
Nx = and N y =
or, n = 4 4 4
t \ Nx/Ny = 1/3
or =4
T and at t = 6 hrs (three half lives)
or t = 4 × T = 4 × 5730 = 22920 years
N0 7N 0
Nx = and N y =
ln 2 ln 2 8 8
52. (a) T1/ 2 = \l =
l T1/ 2 Nx 1
or =
Ny 7
In2 ln 2 l T 1
Þ lA = ,lB = Þ A = B. The given ratio
1 1
lies between and .
TA TB l B TA 4 3 7
53. (c) Therefore, t lies between 4 hrs and 6 hrs.
t / 5700 62. (a) Activity of sample becomes 2500 from 5000 in 75 days
C14 1 æ 1 ö t
= =ç ÷ Þ = 2 Þ t = 11400 years therefore its half life is 75 days, so
C12 4 è 2 ø 5700
R0
R= = 5000 Þ R 0 = 5000 ´ 4 = 20, 000
a b 150
180
54. (a) 72 A ® 70 A1176
¾¾ ® 71 A 2176
¾¾ 2 75

a g
®69 A3172 ¾¾®69 A 4172
¾¾ 2.22
63. (a) B.E H = = 1.11
2
55. (b) Dm = 1 – 0.993 = 0.007 gm
28.3
\ E = (Dm)c2 = (0.007 ×10–3 ) (3× 108)2 = 63 × 1010 J. B.E He = = 7.08
4
0.3
56. (d) E = Dm.c2 Þ E = ´ (3 ´ 108 ) 2 = 2.7 ´ 1013 J 492
1000 B.E Fe = = 8.78 = maximum
56

2.7 ´ 1013 1786


= 6
= 7.5 ´ 106 kWh. B.E U = = 7.6
3.6 ´ 10 235

57. (b) Q = 4 (x2 – x1) 56 is most stable as it has maximum binding energy
26 Fe
58. (a) Let the percentage of B10 atoms be x, then average per nucleon.
atomic weight
( m1 - m2 )v1 + 2 m2 v2
10x + 11(100 - x) 64. (a) v¢1 =
= = 10.81 Þ x = 19 m1 + m2
100
As v2 is zero, m2 > m1, v '1 is in the opposite direction.
N 19 m1 = 1, m2 = A.
\ B10 =
N 81 ( A - 1)
B11 \ | v ¢1 |= v
( A + 1) 1
59. (a) B.E. = Dmc2 = D × 931 MeV The fraction of total energy retained is
= [2(1.0087 + 1.0073) – 4.0015] × 931 = 28.4 MeV 1/ 2mv¢12 ( A - 1) 2
=
60. (d) Radioactivity at T1 , R1 = l N1 1/ 2mv12 ( A + 1) 2

Radioactivity at T2, R2 = l N2
Free eBooks on @neetquestionpaper2020

990 Physi cs
65. (a) For substance A : 72. (b) b-particles, being emitted with very high speed
æ 1ö
48/12
N0 N0 compared to a-particles, pass for very little time near
2N0 ® N A = 2N 0 ç ÷ = = the atoms of the medium. So the probability of the
è 2ø 23 8
For substance B : atoms being ionised is comparatively less. But due to
48/16 this reason, their loss of energy is very slow and they
æ 1ö N0 N0
N0 ® N B = N 0 ç ÷ = = can penetrate the medium through a sufficient depth.
è 2ø 2 3 8
NA : NB = 1 : 1 73. (d) We know that energy is released when heavy nuclei
undergo fission or light nuclei undergo fusion.
66. (b) Effective half life is calculated as Therefore statement (1) is correct.
1 1 1
= + The second statement is false because for heavy nuclei
T T1 T2 the binding energy per nucleon decreases with
1 1 1 increasing Z and for light nuclei, B.E/nucleon increases
= + Þ T = 12 years
T 16 48 with increasing Z.
3
Time in which will decay is 2 half lives = 24 years
4 74. (d) The collection of 12 H nuclei and electron is known as
1
67. (d) After first half hrs N = N0 plasma which is formed due to high temperature inside
2
the reactor core.
1 1
for t = to t = 1 75. (a) Applying conservation of mechanical energy, we get,
2 2
4 5
Loss of kinetic energy of two deuteron nuclei
æ 1ö é 1 ù æ 1ö
N = ç N0 ÷ ê ú = N0 ç ÷ = Gain in their potential energy.
è 2ø ë 2 û è 2ø
1 1 e´e
for t = 1 to t = 2 hrs. [for both A and B 2 ´ 1.5kT =
2 4pe0 r
1 1 1
= + = 2 + 4 = 6 ; t = 1/6 hrs] æ eV ö (1.44 ´ 10-9 eVm)
t1/ 2 1/ 2 1/ 4 1/2 Þ2 ´ 1.5 ´ ç 8.6 ´ 10-5 ÷ ´ T =
è Kø 4 ´ 10-15 m
é æ 1 ö5 ù æ 1ö 3 æ 1ö
8
N = ê N0 ç ÷ ú ç ÷ = N 0 ç ÷ 1.44 ´ 10-9
êë è 2 ø úû è 2 ø è 2ø Þ T= -15
2 ´ 1.5 ´ 8.6 ´ 10-5 ´ 4 ´ 10
my
68. (b) We have K a = .Q = 0.0139 × 1011 = 1.4 × 109 K
m y + ma
76. (b) For the reading given in option (b), we get, nto > 5 ×
A-4 A-4
Þ Ka = .Q Þ 48 = .50 Þ A = 100 1014 which is the Lawson criterion for a reactor to
A A
work successfully.
69. (c) Energy released = (80 × 7 + 120 × 8 – 200 × 6.5)
70. (c) Nuclear force is nearly same for all nucleus. 77. (d) Total energy released from Au198 ® Hg198 in ground
state = ( Dmloss ) c 2 = (197.9682 – 197.9662) (930) = 1.86
dN 0.693N
71. (c) Radioactivity = - = lN = MeV
dt T1/ 2
Energy released from 198Hg in first excited state ® Hg
0.693 ´ 108 0.693 ´ 1.2 ´ 108 in ground state = (–1.6) – (–2) MeV
= = = 0.693 ´ 2 ´ 106.
50 60 = 0.4 MeV
Radioactivity is proportional to 1/T1/2, and not to T1/2. Þ Energy released from Ag 198 ® Hg198 second
excited state
= 1.86 – 0.4 = 1.46 MeV = max K.E. of b2 particle.
78. (a)
Free eBooks on @neetquestionpaper2020

29
Semiconductor Electronics :
Materials, Devices and Simple
Circuits
SOLIDS Lattice Parameters
All solids are made up of atoms and molecules but due to different In a three dimensional crystal structure, the translational vectors
arrangement of the molecules inside them, they are divided into
(a , b , c ) along the x, y and z axis and the angles (a, b and g) in
two classes (a) crystalline and (b) amorphous.
between the translational vectors are called lattice parameters.
Properties of Crystalline Solids
Y
(i) The atoms and molecules are arranged in a definite or regular
order. b
(ii) Crystalline solids are bounded by flat surfaces.
(iii) They posses uniform chemical composition.
g
(iv) They have sharp melting point. a a
X
(v) They are anisotropic.
(vi) They have symmetry. b
c
Properties of Amorphous Solids Z
(i) The atoms and molecules are arranged in an irregular Unit Cell
manner. A unit cell is the smallest geometrical unit in three dimensions.
(ii) These are isotropic i.e. they have same physical properties The repetition of which will give the entire crystal. The crystalline
in all directions. solid is said to be consisting of a large no. of unit cells, each one
(iii) They do not have a sharp melting point. in contact with immediate neighbours.
(iv) They do not have any symmetry. Y
CRYSTAL LATTICE
It is defined as the infinite array of atoms and molecules in b
space (three dimensions) such that at every point, an atom or
the molecule has got the identical surroundings. a g
X
Lattice Vectors a
b
Let a and b be the distance between the two successive atoms c
(points) of the lattice along the x-axis and y-axis, then the lattice
Z
vector l is given by
ur ur ur
l = n1 a + n2 b where a and b are called translational vectors Crystal System
Based on the different combination of lattice parameters (a, b, c)
and n 1 and n2 are the integers.
ur ur ur ur and the angles (a, b and g) we can define crystal system as
For three dimensional lattice, l = n1 a + n2 b + n3 c follows.
Free eBooks on @neetquestionpaper2020

992 Ph ysi cs
S. Crystal Lattice type Unit cell Examples
No. system characteristics
1. Cubic (i) Simple cubic (sc) a=b= c (i) CsCl, ZnS, NaCl
(ii) Face centered cubic (fcc) s = b = g = 90° (ii) Silver, copper, gold, lead, nickel and platinum
(iii) Body centered cubic (bcc) (iii) Sodium, barium, iron, tungsten and uranium
2. Tetragonal (i) Simple cubic (sc) a=b¹c NiSO4, SnO2 and white tin
(ii) Body centered cubic (bcc) s = b = g = 90°
3. Hexagonal Simple cubic (sc) a=b¹c Zn, ZnO, Cd, Ni, graphite and quartz
s = b = 90°
g = 120°
4. Rhombohedral Simple cubic (sc) a=b= c Sb, Bi, calcite
(Trigonal) s = b = g ¹ 90° < 120°
5. Orthorhombic (i) Simple cubic (sc) a¹b¹c KNO3, gallanium, mercury chloride, Rhombic
(Rhombic) (ii) Base centred cubic s = b = g = 90° sulphur
(iii) Body centered cubic (bcc)
(iv) Face centered cubic (fcc)
6. Monoclinic (i) Simple cubic (sc) a¹b¹c Na2SO4, KClO3, FeSO4
(ii) Base centered or s = g = 90° ¹ b
end centered cubic (ecc)
7. Triclinic Simple cubic (sc) a¹b¹c CuSO4, K2Cr2O7
s ¹ b ¹ g ¹ 90°

Characteristics of the unit cell of a cubic system Atomic Packing Factor or Density of Packing
(i) Volume of a unit cell (V) = (a × b × c) or V = a × b × c It is defined as ratio of the volume occupied by all the atoms in
(ii) Mass and density of a unit cell : a unit cell to the total volume of the unit cell.
The mass of unit cell = number of atoms or molecules in the
cell × mass of each atom
M
or, m = p × ma or m = p ´ where M = molecular wt. of
Na
the substance

M S.C. B.C.C F.C.C


p
\ density d =
Na pM
= Volume of atoms per unit cell
V VN a Packing factor ( P.F .) =
Volume of unit cell
If edge a, is given in picometre then
pM
d= 3 -30
g / cm 3
a N a ´ 10
(iii) Number of atoms (N) in a unit cell or the total number of
Nf Nc
atoms per unit cell is given by N = N i + +
2 8
where Ni = no. of body centred or interior atoms
Nf = no. of face centered atoms
Nc = no. of corner atoms.
Example 1.
Coordination Number
In a cubic unit cell of bcc structure, the lattice points
The number of nearest neighbours of a given atom in the crystal i.e. number of atoms are
is called the coordination number.
(a) 2 (b) 6
Half of the distance between the centres of two neighbouring
atoms is called atomic radius . It is also called lattice constant or (c) 8 (d) 12
lattice parameter.
Free eBooks on @neetquestionpaper2020

Semi conductor Electroni cs : Materials, Devi ces an d Simple Circuits 993


Solution : (a) Example 3.
No. of lattice points in a crystal structure will be Determine the atomic packing factor for a face centred
cubic cell.
N C N F Ni
n= + + Solution :
8 2 1 Atomic packing factor
In bcc crystal NC = 8 and NF = 0 and N i = 1 ; Volume occupied by the atoms in a unit cell
=
8 0 1 Volume of the unit cell
\ n= + + =2
8 2 1 For fcc sturcture, no. of atoms in a unit cell = 4
Example 2. a
Sodium has body-centred packing. Distance between two radius of each atom r = ,
2 2
nearest atoms is 3.7 Å. Determine the lattice parameter.
where a is the lattice parameter.
Solution :
Atomic radius for bcc structure is 4pr 3 p 2a 3
Volume of 4 atoms = 4 ´ =
a 3 or a = 4r = 4(3.7 / 2) = 4 ´ 1.75 = 4.3Å 3 6
r=
4 3 3 3 ´ 1.732 p 2a 3 / 6
so, atomic packing factor = p 2
=
i.e., Lattice parameter = 4.3 Å a3 6

29.1
Solve following problems with the help of above text and 8. A solid which is not transparent to visible light and whose
examples. electrical conductivity decreases with increase in
1. Which of the following is a crystalline solid ? temperature is formed by
(a) Graphite (b) Calcite (a) ionic bonding (b) metallic bonding
(c) Gallium (d) Oxygen (c) covalent bonding (d) vander Wall bonding
2. Crystalline solids do not have 9. The density for simple cubic lattice is (where A is atomic
(a) a definite latent heat of fusion weight, N Avogadro number and a is lattice parameter)
(b) irregular atomic arrangement 4A 2A
(c) incompressibility (a) 3 (b)
Na Na 3
(d) free movement of atoms
3. Identify the system of crystal structure, if a = b ¹ c and A A
a = b = 90º and g = 120º. (c) 3 (d)
Na Na 2
(a) Monoclinic (b) Triclinic 10. A solid which is not transparent to visible light and whose
(c) Hexagonal (d) Rhombohedral electrical conductivity increases with temperature is
4. The space lattice of diamond is formed by
(a) fcc (a) ionic bonding (b) metallic bonding
(b) bcc with two atoms per unit cell (c) covalent bonding (d) van der wall bonding
(c) bcc with 4 atoms per unit cell 11. What makes the molecular crystals solid at low
(d) simple cubic temperature?
5. Which of the following is not an amorphous substance ? (a) Sharing of electrons
(a) Copper (b) Rubber (b) Transfer of electrons
(c) Glass (d) Polymers (c) Vander Wall’s force arising out of polarisation
6. The number of atoms per unit cell in a sc, bcc and fcc (d) All of the above
system are respectively 12. The density for an fcc lattice is (A = atomic wt,
(a) 1, 2 and 4 (b) 8, 6 and 10 N = Avogadro’s number, a = lattice parameter)
(c) 1, 4 and 2 (d) 2, 4 and 1
7. Which of the following bonds produces a solid that reflects 4A 2A
(a) 3 (b)
light in the visible region and whose electrical conductivity Na Na 3
decreases with temperature and has high melting point?
A A
(a) Metallic bonding (b) Ionic bonding (c) (d)
3
(c) Covalent bonding (d) Vander Wall’s bonding Na Na 2
ANSWER KEY
1. (b) 2. (d) 3. (c) 4. (a) 5. (a) 6. (a) 7. (b) 8. (b) 9. (c) 10. (a) 11. (c) 12. (a)
Free eBooks on @neetquestionpaper2020

994 Ph ysi cs
ENERGY BAND THEORY OF SOLIDS Formation of 3s band by a larger number of atoms.(G-state)
When two identical atoms, are far apart, the electronic levels (or E
electronic wave functions) in one are not disturbed by the Energy
presence of other atom fig 1(a). For example, the 3s electron of
each sodium atom has a single energy with respect to its nucleus.
But when the two atoms come closer, these electron’s wave
function starts to overlap and each electron revolves round both 3s
nuclei.
Y
Y Atomic
Fig.2(c) separation
Probability of
finding the electron (symmetric wave The wave function of each electron is roughly, a combination of
YA YB is maximum the wave functions of isolated atoms : Fig 1(b) & (c) show two
Y A+ Y Bfunction)
r r possible wave functions. In the first, the two isolated wave
Fig. 1(a) Fig. 1(b) functions are combined with (make symmetric state) same sign,
Y
in the second with opposite signs(antisymmetic state). The
symmetric state has lower energy as compared to antisymmetric
state, because the electron is more likely to be found midway
Probability of between the nuclei, which leads to lower coulomb energy. This is
finding the the effect, responsible for molecular bonding. Representation of
electron is zero
energy level (when two sodium atoms come closer) is shown in
r fig 2(a).
Fig. 1(c)
YA – YB As we bring together a large number of atoms to form a solid, the
(Antisymmetric same sort of effect occurs. When the sodium atoms are far apart,
wave function)
all 3s electrons have same energy & as we begin to move them
Fig. 1 : Wave functions of two sodium atoms (each in G-state) : together, the energy levels begin to “split”. The situation for four
(a) : At wide separation sodium atoms is shown in fig 2(b). As the number of atoms is
(b) : Small separation : Isolated wave function combined with increased (may be 1020 atoms), the levels become so numerous
same sign (i.e., lower energy)
& so close that we can no longer distinguish the individual levels
(c) : Small separation : Isolated wave function combined with
as shown in fig 2(c). We can regard the N atoms as forming an
opposite sign i.e., antisymmetric state of joint wave
functions Y A & Y B(higher energy state) almost continuous band of energy. Since those levels were
Splitting of 3s levels when two atoms are brought together. identified with 3s atomic levels of sodium, we refer to the 3s band.
E 3p
Energy
N 3s

6N 2p
3s
2N 2s

Atomic 1N 1s
Fig.2(a) separation Fig. 3- Energy bands in sodium metal (G-state)
Splitting of 3s levels when four atoms are brought together. Each band has a total of N individual levels. Each level can hold
E 2 × (2l + 1) electrons (l is azimuthal quantum number), so the
Energy capacity of each band is 2(2l + 1)N electrons.
Fig - 3 shows a complete representation of energy bands in sodium
metal. The 1s, 2s & 2p bands are each full, 3s band is half & 3p
band is completely empty. The 1s & 2s bands each contain 2N
electrons & 2p band contain 6N electrons. The 3s band contain
3s
N electron, so it is half filled. The band, which can hold 6N
electrons is completely empty.
Atomic When we add energy to a system i.e., to sodium metal, the electron
Fig.2(b) separation
can move from filled state to empty state. In this case, the electron
Free eBooks on @neetquestionpaper2020

Semi conductor Electroni cs : Materials, Devi ces an d Simple Circuits 995


can move from partially full states of 3s band to empty states of Semiconductors
3s band by absorbing small amount of energy or move to 3p A semiconductor, has a completely filled valence band i.e., it
band by absorbing larger amount of energy. resembles an insulator at zero temperature. However, the gap
In a solid at zero temperature, the electron settle into the available between this filled valence band & next band (C.B.) is small, about
states of lowest energy. The lower energy bands will therefore be 1eV or less. Hence electrons can easily make the transitions from
completely filled & the upper most energy band will be either one band to another at room temperature & then carry an electric
completely filled or partially filled, depending on the number of current (Silicon and germanium are semiconductors).
electrons & on the number of available states. The diffence In a semiconductors, in thermal equilibrium, nenh = ni2
between conductor & insulator arises from a partially filled or a
There are two types of semiconductor.
completely filled upper most energy band.
(i) Intrinsic (or pure) semiconductors : These semiconductors
CONDUCTORS, INSLULATORS AND are pure materials in which the thermal vibrations of the
SEMICONDUCTORS lattice have liberated charge carriers (i.e., electrons & holes).
Conductors In intrinsic semiconductor, the number of electrons (ne) are
A conductor such as sodium, has a partially filled band (in sodium, equal to the number of holes (nh).
the upper most band 3s is half filled). In these substance, electrons (ii) Extrinsic (or impure or doped) semiconductors : They are
are free to move by applying an electric field, because un- impure semiconductors in which minutes traces of impurity
occupied states are available in upper most band (in 3s band of introduces mobile charge carriers [which may be +ive (holes)
sodium only half states are completely filled & half states are or -ive (electrons)] in addition to those liberated by thermal
empty or un-occupied). Therefore these electrons are mobile & vibration.
can contribute to electrical & thermal conductivity. For example Again there are two types of extrinsic semiconductors.
the energy bands of sodium metal are shown in fig. 4.
(a) N-type semiconductor : When we add a pentavalent
E
impurity in intrinsic semiconductor, then we obtain N-
Energy type semi-conductor. The pentavalent impurity
3p substances are P, As & Sb. In N-type semi-conductors,
the electrons are majority carriers & holes are
3s minority carriers.
Actual separation i.e., ne >> nh
of atom in crystal
(b) P-type semiconductor : When we add a trivalent
2p
impurity in intrinsic semiconductor (such as B, Al, In),
R
O R0 we obtain p-type semiconductor.
Fig.4 In p-type semiconductors, the holes are majority
Origin of energy bands in sodium metals, as atoms move together carriers & electrons are minority carriers.
the energy level spreads into bands.
i.e., nh >> ne
In an isolated sodium atom, the six 3p lowest excited state for
In acceptor impurity (trivalent element) energy level is just above
valence electrons are about 2.1 eV above two 3s G-states. But in
the top of valence band (V.B.)
solid sodium the 3s & 3p bands are spread out enough so that
they actually overlap, forming a single band.
C.B.
Because each sodium atom has only one valence electron but
eight 3s & 3p states, that single band is only 1/8 filled & other
states are unoccupied. *EF
Acceptor energy level
V.B.
Insulators
In these substances (such as diamond), the upper most level is (*EF is called fermi level & at T = 0 all the states below are
compeletely filled i.e., no unoccupied state is available for electron completely filled. In N-type semiconductor it lies between
to move. The nearest unoccupied states are in next band(called conduction band & donor energy level Ed & in case of P-type, it
C.B.), but this is separated from filled band (called V.B.) by an lies between acceptor level Ea & valence band. In intrinsic
energy gap (Eg) of about 6eV. Hence electron refuses to carry an semiconductor the EF level lies in midway between C.B. & V.B.)
electric current.
Free eBooks on @neetquestionpaper2020

996 Ph ysi cs
Comparison Between Conductors, Semiconductors and Insulators
Property Conductors Semiconductors Insulators
Resistivity range 10–6 – 10–8 Wm 10–4 – 0.5 Wm 107 – 1016 Wm
Conductivity range 10–6 – 10–8 mho/m 104 – 0.5 mho/m 10–7 – 10–16 mho/m
Temp. coefficient Positive Negative Negative
of resistance (a)
Flow of current Due to free electrons Due to electrons and holes No current flow

Conduction band Conduction band


(CB) Conduction band
(CB)
(CB)
Electron energy

Ele ctron e ne rgy


Electron energy
Energy band diagram
No gap Overlapping
Forbidden
region
gap D Eg 1eV Forbidden
gap DEg 6eV

Valence band
(VB)
Valence band
(VB) Valence band
(VB)

Forbidden energy gap @ 0eV @ 0.1 – 3eV ³ 6eV


Examples Pt, Al, Cu, Ag etc. Ge, Si, C, Ga, As Wood, plastic
GaF2 etc. diamond, mica
COMMON DEFAULT 5. Holes doesn’t exist in conductors but only in semi-
´ Incorrect. N–type semiconductors are negatively and P– conductors. They are deflected by electric and magnetic
type semiconductors are positively charged. fields like electrons. Their movement contributes to electric
current similar to electrons.
Ö Correct. Both N–type and P–type semiconductors are
neutral because these are made up of neutral atoms. Example 4.
Find the current produced at the room temperature in a
´ Incorrect. Mobility of holes is greater than mobility of pure germanium plate of area 2 × 10–4 m2 and of thickness
electrons 1.2 × 10–3 m when a potential of 5 V is applied across the
Ö Correct. Mobility of holes is less than mobility of electrons faces. Concentration of carriers in germanium at room
because mobility of holes takes place in valance band and temperature is 1.6 × 106 per cubic metre. The mobility of
mobility of electrons takes place in conduction band. electrons and holes are 0.4 m2V–1 s–1 and 0.2 m2V–1 s–1
respectively. How much heat is generated in the plate in
Keep in Memory 100 second?
Solution :
1. Width of forbidden energy gap Here, ni = 1.6 × 106 m–3
= Spacing between the top of valence band and the me = 0.4 m2V–1s–1
bottom of conduction band. mh = 0.2 m2 V2 s–1 ;
2. At absolute zero temperature (0 K), there are no free A = 2 × 10–4 m2 and d = 1.2 × 10–3m ; V = 5 V
electrons in the conduction band of a semi-conductor. As s= n i e (m e + m h )
3. A good number of semi-conductors (elemental and
compound) have been discovered but the most frequently
( )
1.6 ´ 106 ´ (1.6 ´ 10-19 )(0.4 + 0.2)
=

used are germanium (Ge) and silicon (Si) of group-IV of = 1.53 × 10–13 Sm–1
periodic table because the forbidden energy gap of them is Current produced in germanium plate
small (of the order of 1 eV) so that at ordinary room æVö
Ι = JA = sEA = s ç ÷ A
temperature covalent bonds break easily and free electrons èdø
become available in the conduction band. 5
4. Important compound semi-conductors are gallium arsenide = 1.53 ´10 -13 ´ ´ 2 ´ 10- 4 = 1.28 ´10 -13 A
(GaAs), lead sulphide (Pbs), cadmium sulphide (Cds), (1.2 ´10 -3 )
Heat generated in plate,
indium phosphide (InP), etc. H = VI t = 5 × 1.28 × 10–13 ×100 = 6.4 ×10–11 J.
Free eBooks on @neetquestionpaper2020

Semi conductor Electroni cs : Materials, Devi ces an d Simple Circuits 997

29.2
Solve following problems with the help of above text and (c) the electron concentration increases
examples. (d) the electron concentration decreases
1. In a semiconductor [more than one option may be correct] 11. The resistivity of a semiconductor at room temperature is
(a) there are no free electrons at 0 K in between
(b) there are no free electrons at any temperature (a) 10–2 to 10–5 W cm (b) 10–3 to 106 W cm
(c) the number of free electrons increases with (c) 106 to 108 W cm (d) 1010 to 1012 W cm
temperature 12. Number of electrons in th e valence shell of a
(d) the number of fre electrons is less than that in a semiconductor is
conductor (a) 1 (b) 2 (c) 3 (d) 4
2. Let np and ne be the number of holes and conduction 13. In a semiconductor, the forbidden energy gap between
electrons in an extrinsic semiconductor. Then the valence band and the conduction band is of the order
(a) np > ne (b) np = ne (c) np < ne (d) np ¹ ne
is
3. A p-type semiconductor is
(a) 1 MeV (b) 0.1 Mev
(a) positively charged
(c) 1 eV (d) 5 eV
(b) negatively charged
(c) uncharged 14. The forbidden energy gap for germanium crystal at 0 K is
(d) uncharged at 0K but charged at higher temperatures (a) 0.071 eV (b) 0.71 eV
4. When an impurity is doped into an intrinsic semiconductor, (c) 2.57 eV (d) 6.57 eV
the conductivity of the semiconductor 15. In an insulator, the forbidden energy gap between the
(a) increases (b) decreases valence band and conduction band is of the order of
(c) remains the same (d) becomes zero (a) 1 MeV (b) 0.1 MeV(c) 1 eV (d) 5 eV
5. Electric conduction in a semiconductor takes place due to 16. What is the resistivity of a pure semiconductor at absolute
(a) electrons only zero ?
(b) holes only (a) Zero
(c) both electrons and holes (b) Infinity
(d) neither electrons nor holes (c) Same as that of conductors at room temperature
6. An electric field is applied to a semiconductor. Let the (d) Same as that of insulators at room temperature
number of charge carriers be n and the average drift speed 17. Temperature coefficient of resistance of semiconductor is
be v. If the temperature is increased (a) zero (b) constant
(a) both n and v will increase (c) positive (d) negative
(b) n will increase but v will decrease 18. In a p-type semiconductor, the acceptor valence band is
(c) v will increase but n will decrease (a) close to the valence band of the host crystal
(d) both n and v will decrease (b) close to conduction band of the host crystal
7. Let np and n e be the number of holes and conduction (c) below the conduction band of the host crystal
electrons in an intrinsic semiconductor. Then (d) above the conduction band of the host crystal
(a) np > ne (b) np = ne (c) np < ne (d) np ¹ ne
19. In an n-type semiconductor, donor valence band is
8. The impurity atoms with which pure silicon may be doped
(a) above the conduction band of the host crystal
to make it a p-type semiconductor are those of
(b) close to the valence band of the host crystal
[more than one option may be correct]
(a) phosphorus (b) boron (c) close to the conduction band of the host crystal
(c) antimony (d) aluminium (d) below the valence band of the host crystal
9. The electrical conductivity of pure germanium can be 20. The mobility of free electrons is greater than that of free
increased by holes because
(a) increasing the temperature (a) they are light
(b) doping acceptor impurities (b) they carry negative charge
(c) doping donor impurities (c) they mutually colllide less
(d) all of the above (d) they require low energy to continue their motion
10. A semiconductor is doped with a donor impurity then 21. The relation between number of free electrons (n) in a
[more than one option may be correct] semiconductor and temperature (T) is given by
(a) the hole concentration increases (a) n µ T (b) n µ T2
(b) the hole concentration decreases
(c) nµ T (d) n µ T3/2
ANSWER KEY
1. (a, c, d) 2. (d) 3. (c) 4. (a) 5. (c) 6. (b) 7. (b) 8. (b, d) 9. (d) 10 (b,c) 11. (b) 12. (d)
13. (c) 14. (b) 15. (d) 16. (b) 17. (d) 18. (a) 19. (c) 20. (a) 21. (d)
Free eBooks on @neetquestionpaper2020

998 Ph ysi cs
p-n JUNCTION DIODE But, if e-h pair is created in depletion region, the electron is quickly
If we join a piece of n-type to a piece of p-type semiconductor by pushed by electric field towards n-side & holes towards p-side.
appropriate method, then we obtain p-n junction diode. The two As e-h pairs are continuously created in depletion region, there
ends of p-n junction is provided with metallic conductors for the is a regular flow of electron towards n-side & holes towards
application of an external voltage. It is clear that p-type has more p-side & so current flows from n-side to p-side. This current is
holes concentration but less concentration of electrons than drift current.
n-type semicondutor. In steady state the magnitude of drift current is equal to diffusion
current & since they are in opposite direction, hence there is no
net transfer of charge at any cross section.
This recombination produces narrow region near junction called
depletion layer. Since this region is depleted of free or mobile
charge carrier and contains only immobile charge carriers, hence
it is called depletion region. Due to these immobile charge carriers,
a potential barrier VB is established & further diffusion is stopped
& equilibrium is reached (shown by fig. 1(b)). The sign of VB is
+ive towards the n-type & –ive towards p-type.
The schematic symbol of diode & corresponding equivalance in
terms of VB are shown below in fig. 2 (a) & (b).
Depletion
VB layer
Cathode Anode
(n) VB
(p) (b)
(a)
o x
Fig (2) (c)

In fig-2(a) the triangle shows the direction of current. For Si


diode the value of VB is 0.7V & for Ge diode the value of VB is
0.3V.
FORWARD AND REVERSE BIAS OF p-n JUNCTION
DIODE
Forward Bias
When p-side of the p-n junction is connected to the +ve terminal
Due to this difference in concentration, density gradient is of a battery and n to –ve terminal, conduction takes place and
established across the junction resulting in majority carriers the diode is said to be forward biased.
diffusion : Holes (+ive ions) diffuses from p-type to n-type &
electrons (–ive ions) from n-type to p-type (shown in fig (a)) & p n
terminating their existence by recombination.
Diffusion and Drift Current
Diffusion current : We know that due to concentration difference,
+ –
holes diffuse from p-side to n-side (in figure -1(a)) they move
from left to right) & electron diffuse from n-side to p-side (in fig Reverse Bias
1(a) they move from right to left). But electric field at junction When p-side of he p-n junction is connected to the -ve terminal
repels the holes as they come to depletion layer & only those of a battery and n-side to the +ve terminal, there is no conduction
holes which have high kinetic energy are able to cross the takes place and diode is said to be reverse biased.
potential barrier. Similarly electric field at junction repels electrons
p n
& those having high kinetic energy cross the junction.
The electric potential of n-side is more +ive (or higher) than
p-side –ive (or lower). The variation of potential barrier VB is
shown in fig (c).
The potential difference created across the p-n junction due to – +
the diffusion of electrons and holes is called potential barrier. Effect of biasing : In forward bias, the thickness of depletion
So diffusion results in an electric current from p-side to n-side layer decreases and potential barrier reduces, while in reverse
known as diffusion current. bias, the thickness of depletion layer increases and potential
Drift current : Due to thermal collision, some times a covalent barrier also increases. In forward bias the current increases
bond is broken & electron-hole pair is created. But those electron- exponentially & in reverse bias, the current remains constant at
hole pairs are destroyed easily due to recombination. This process very small magnitude upto breakdown voltage Vo & after that it
[generation of electron-hole (e-h) pair] occurs in the whole part increases sharply without any further increase of reverse voltage
of material. shown in fig.1.
Free eBooks on @neetquestionpaper2020

Semi conductor Electroni cs : Materials, Devi ces an d Simple Circuits 999


V-I characteristics of a p-n junction diode in forward and reverse eV
bias. In forward bias the expression of current is I = I o (exp - 1)
I K BT
Forward where V is applied voltage
Breakdown
bias Io is reverse saturation current
voltage
V0 e is fundamental electronic charge
V KB is Boltzmann constant and
knee
T is temperature
Reverse voltage
bias Keep in Memory

Fig (1) 1
The effect of forward biasing & reverse biasing on potential barrier 1. Width of depletion layer µ
Doping
are shown in fig.(2).
2. Depletion is directly proportional to temperature.
Depletion layer Barrier voltage
Junction
Deple 3. Resistance is forward bias Rforward » 10W – 25W and
Idiffusion
-tion Idrift resistance in reverse bias Rreverse » 105W.
p n VB(x) layer Inet
=0
4. Under suitable reverse bias break down occurs and voltage
gets stabilized at Zener voltage.
No biasing x 5. Junction diode (p-n junction) can be used to convert the
Fig.(2)(a) p-n junction without biasing a.c. current to d.c. current i.e. as a rectifier. The process of
conversion from a.c. to d.c. is called rectification.
Depletion layer 6. p-n junction diode can be used as a half wave or full wave
Junction Barrier voltage
rectifier. A device used to convert dc into ac is called
Idiffusion inverter.
p n Deple
Idrift
-tion
Inet
layer
VB'(x) 7. Avalanche or zener diode or breakdown
x
Forward biasing diode used in voltage stabilisation.

Fig.(2)(b) p-n junction with forward biasing


8. Photodiode, used in automatic switching
Junction
Barrier voltage
Deple Idiffusion
of light, fire alarm, burglar alarm
p n -tion
Idrift
layer
VB''(x) Inet = Indrift – Idrift LED (Light emitting diode) indicator, and
Depletion layer
x
Reverse biasing optical fibre communication.
(c)
9. A zener diode operating in the breakdown region is
Fig.(2) (c) p-n junction with reverse biasing equivalent to a battery. This property of zener diode makes
it suitable for voltage regulation (holding voltage constant).
Some Important Results of Half-wave and Full-wave Rectifier
Circuit and Quantity Half wave rectifier Full wave rectifier

D1 RL V0
diode
I/P
1. Circuit I/P RL V 0

D2

2. Input voltage Output voltage Output voltage

Vin V0 V0

t
t t
Free eBooks on @neetquestionpaper2020

1000 Physi cs

3. The value of Irms of I0 / 2 I0 / 2


input alternating current
I = I0 sin wt
4. Peak inverse voltage E0 2E0
(i.e. maximum voltage
across diode when it
is not conducting)
5. Output direct current I0 / p 2I 0 / p

2
æ Irms ö
6. Ripple factor g = ç -1 1.21 0.48
è Idc ÷ø

Output d.c. power


7. Efficiency h = 40.6% 81.2%
Input a.c. power

Irms E rms
8. Form factor = = 1.57 1.11
Idc E dc
9. The value of d.c. component Less More
in output voltage as compared
to input a.c. voltage

Example 5. 150W
The diode used in the circuit shown has a constant voltage D1
drop at 0.5 V at all currents and a maximum power rating
of 100 milli-watt. What should be the value of the resistor 50W
D2
R, connected in series with diode to obtain maximum
100W
current?
6V
Fig. (a)
R 0.5V
Solution :
In this circuit, diode D1 is froward biased and offers a
resistance of 50W. The diode D2 is reverse biased and offers
infinite resistance to the current i.e. no current flows through
1.5V D2. Thus the equivalent circuit is as shown in fig. (b)

50W 150W
Solution : D1
VP = 0.5 V, Maximum power rating P = 100 mW = 100×10–3 W I I
and source voltage Vs = 1.5 V.
100W
Vp2 (0.5) 2 I
Now diode resistance R D = = = 2.5 W 6V
Fig. (b)
P 100 ´10 -3
Total resistance of circuit = 50 + 150 + 100 = 300W
V 0.5 6
Current in diode ΙD = D = = 0.2A Current through 100W is, Ι = = 0.02 A .
R D 2.5 300
\ Total resistance in the circuit Example 7.
Vs 1.5 In p-n junction diode the reverse saturation current is
RT = = = 7.5 W 10–5 A at 27ºC. Find the forward current for a voltage of
Ι D 0.2 0.2 V. [exp (7.62) = 2038.6, k = 1.4 × 10–23 J/K]
Solution :
So, resistance R in circuit = R T - R D = 7.5 - 2.5 = 5 W
Example 6. Q Ι = Ι0 (e eV / kT - 1)
The circuit shown in the fig. (a) contains two diodes each By solving we get,
with a forward resistance of 50W and with infinite reverse I = 10-5 [e7.62 - 1] = 10 -5 [ 2038.6 - 1] = 20.376 ´ 10 -3 A
resistance. If the battery voltage is 6 V, find the current
So the forward current for voltage 0.2V is 20.376 × 10–3A
through the 100W resistance.
Free eBooks on @neetquestionpaper2020

Semi conductor Electroni cs : Materials, Devi ces an d Simple Circuits 1001

29.3
Solve following problems with the help of above text and 7. In a p-n junction [more than one option may be correct]
examples. (a) new holes and conduction electrons are produced
1. A semiconductor device is connected in a series circuit continuously throughout the material
with a battery and a resistance. A current is found to pass (b) new holes and conduction electrons are produced
through the circuit. If the polarity of the battery is continously throughout the material except in the
reversed, the current drops to almost zero. The device depletion region
may be a/an (c) holes and conduction electrons recombine
(a) intrinsic semiconductor continuously throughout the material
(b) p-type semiconductor (d) holes and conduction electrons recombine
(c) n-type semiconductor continuously throughout the material except in the
(d) p-n junction diode depletion region
2. If the two ends of a p-n junction are joined by a wire 8. Forward biasing is that in which applied voltage
(a) there will not be a steady current in the circuit (a) increases potential barrier
(b) there will be a steady current from the n-side to the (b) cancels the potential barrier
p-side (c) is equal to 1.5 volt
(c) there will be a steady current from the p-side to the (d) None of these
n-side 9. In V-I characteristic of a p-n junction, reverse biasing
(d) there may or may not be a current depending upon results in
the resistance of the connecting wire (a) leakage current
3. The drift current in a p-n junction is (b) the current barrier across junction increases
(a) from the n-side to the p-side (c) no flow of current
(d) large current
(b) from the p-side to the n-side
10. In reverse biasing
(c) from the n-side to the p-side if the junction is forward-
(a) large amount of current flows
biased and in the opposite direction if it is reverse
(b) potential barrier across junction increases
biased
(c) depletion layer resistance increases
(d) from the p-side to the n-side if the junction is forward-
(d) no current flows
biased and in the opposite direction if it is reverse-
11. Zener diode is used for
biased
(a) amplification (b) rectification
4. The diffusion current in a p-n junction is
(c) stabilisation (d) all of the above
(a) from the n-side to the p-side
12. Bridge type rectifier
(b) from the p-side to the n-side (a) uses four diodes (b) uses six diodes
(c) from the n-side to the p-side if the junction is forward- (c) uses two diodes (d) uses one diode
biased and in the opposite direction if it is reverse- 13. Filter circuit
biased (a) eliminates a.c. component
(d) from the p-side to the n-side if the junction is (b) eliminates d.c. component
forward-biased and in the opposite direction if it is (c) does not eliminate a.c. component
reverse-biased (d) None of these
5. Diffusion current in a p-n junction is greater than the drift 14. A sinusoidal voltage of peak value 200 volt is connected
current in magnitude to a diode and resistor R in the circuit shown so that
(a) if the junction is forward-biased halfwave rectification occurs. If the forward resistance of
(b) if the junction is reverse-biased the diode is negligible compared to R, the r.m.s. voltage
(c) if the junction is unbiased (in volt) across R is approximately
(d) in no case
6. In a p-n junction with open ends
[more than one option may be correct]
(a) there is no systematic motion of charge cariers E0 = 200 R
(b) holes and conduction electrons systematically go Volt
from the p-side to the n-side and from the n-side to
the p-side respectively.
(c) there is no net charge transfer between the two sides (a) 200 (b) 100
at steady state 200
(d) there is a constant electric field near the junction (c) (d) 280
2
Free eBooks on @neetquestionpaper2020

1002 Physi cs

15. In bridge rectifier circuit, (see fig.), the input signal should (a) I0 /p (b) I0/2
be connected between (c) p I0/2 (d) 2 I0/p
18. If the forward bias on p-n junction is increased from zero
A to 0.045 V, then no current flows in the circuit. The contact
potential of junction i.e. VB is
(a) zero (b) 0.045 V
D B (c) more than 0.045 V (d) less than 0.045 V
19. For a junction diode the ratio of forward current (I f) and
reverse current (Ir) is
[e = electronic charge,
C
V = voltage applied across junction,
(a) A and D (b) B and C k = Boltzmann constant,
(c) A and C (d) B and D T = temperature in kelvin]
16. The average value of output direct current in a half wave (a) e –V/kT (b) eV/kT
rectifier is (c) (e –eV/kT + 1) (d) (eeV/kT – 1)
(a) I0 /p (b) I0/2 20. In a half wave rectifier, the r.m.s. value of the a.c.
(c) p I0/2 (d) 2 I0/p component of the wave is
17. The average value of output direct current in a full wave (a) equal to d.c. value (b) more than d.c. value
rectifier is (c) less than d.c. value (d) zero
ANSWER KEY
1. (d) 2. (a) 3. (a) 4. (b) 5. (a) 6. (b, c, d) 7. (a, d) 8. (b) 9.(a) 10. (b)
11. (c) 12. (a) 13. (a) 14. (b) 15. (d) 16. (a) 17. (d) 18. (c) 19. (d) 20. (b)

TRANSISTOR Collector (C) : It is a region on other side of base. It has maximum


Transistors are three terminal (solid state) devices just like triode. area out of other sections(emitter & base) of transistor to dissipate
It can be assumed to consist of two back to back p-n junctions. the heat. It collects the charge carriers & it is always reverse
In practice a junction transistor (p-n-p) consists of silicon (or biased.
germanium) bar crystal in which a layer of n-type silicon (or Ge) is Base (B) : It is middle region which forms the two junctions
sandwiched between two layers of p-type silicon & we get p-n-p
between emitter & collector. It is very lightly doped.
transistor. Alternatively it may consist of a layer of p-type between
two layers of n-type material & we get a n-p-n transister as shown Biasing of Transistor
by fig. (1). In proper biasing of transistor the input (i.e., base emitter junction)
is always forward biased & output (i.e., collector base junction)
B(Base) B(Base)
is always reverse biased as shown in fig.(3)
This scheme of biasing is same for all three transistor
E(Emitter)

p n p C n p n C configurations; Common base configuration (CB), common


(Collector) (Collector) emitter configuration (CE) & common collector configuration (CC)]
(P – n – P) transistor (n – P – n) transistor
E (Emitter)
p-n-p n-p-n
Fig. (1) E C E C
The schematic symbols of transistor are IE IE
p p n n
E C E C
B B

n B p B
(n-p-n) transistor (a) (b)
(p-n-p) transistor Fig. (3)
(a) (b)
Fig. (2) Working of Transistor
Components of Transistor : Fig. (4) shows a common base (C.B.) configuration of p-n-p
Emitter (E) : It supplies charge carriers (electron in n-p-n transistor. The forward biasing of emitter junction lowers the
transistor and holes in p-n-p transistor) & it has high density of emitter base potential barrier, whereas the reverse biasing of
impurity concentration i.e., highly doped. It is always forward collector junction increases the collector-base potential barrier.
biased. Hence holes (majority carriers in p-type) flows through emitter to
Free eBooks on @neetquestionpaper2020

Semi conductor Electroni cs : Materials, Devi ces an d Simple Circuits 1003


base & constitutes an emitter current IE. Since emitter is heavily
doped in comparison to base, so approximately (only 5% holes C IC
recombine with electrons in base region & constitute base current
IB) 95% holes reach to collector & constitute collector current IC. B VCC

p-n-p IB E
E C IC
IE
IE VCC
VBB Ground
B Fig. (6)
IB
(iii) Common collector configuration (CC) : Here the collector
VEE
terminal is common to both input as well as output terminals
Fig. (4) as shown in fig. (7). The base terminal is input & emitter is
output terminal.
From Kirch off’s current law,
IE = IC + I B ...(1) E IE
Eq. (1) holds true regardless of circuit configuration or transistor B VEE
type (p-n-p or n-p-n) that is used. IB
The current gain (B) of transistor is defined as ratio of collector C
current IC to base current IB VBB IC
IC
i.e., b = ...(2)
IB
Fig. (7)
The value of b lies between 10 and 100. Since I E » IC
Current gain (a) is defined as the ratio of collector current IC to
Transistor as an Amplifier
emitter current IE. The value of a is always less than unity.
Amplification is the process of linearly increasing the amplitude
IC of a signal. It is one of the major properties of a transistor.
a= ...(3) This amplifing action was produced by transfering a current from
IE
a low (base emitter loop is forward biased & hence provide low
Relation between a and b resistance path and collector base junction is reverse biased &
b a hence gives high resistance path in common emitter configuration)
a= or b= ...(4) to a high resistance circuit.
1+ b 1- a Common- Emitter (CE) Amplifier : Fig. 8 shows a basic common-
Configurations of Transistor emitter amplifier circuit in which we connect a signal source Vs.
There are three types of transistor configuration. We can take The input voltage loop is Vs, then from base to emitter through
either terminal as input terminal and other terminal as output. transister to ground (common refrence point) & then through
(i) Common base configuration (CB) : Here base terminal is VBE back to Vs. The output voltage loop consists ground, then
common to both input & output terminals. The emitter from emitter to collector through RC. The emitter is connected to
terminal is taken as input. ground. During a +ive half cycle of Vs, the forward bias across
the emitter base junction will be increased, while during – ive half
p-n-p cycle of Vs (signal source), the forward bias across emitter base
E C IC
junction will be decreased. Hence more electrons flow during
IE +ive half cycle and so we obtain more collector current and so
VCC
large voltage drop across RC and less value and VCE. In negative
B half cycle collector current decreases.
IB
VEE Ground
RC
Fig. (5)
VCE Vout
(ii) Common emitter configuration (CE) : Here emitter terminal ns +
is common to both input and output terminals as shown in –
fig. (6). The base terminal is taken as input and collector is
VBE VCC
taken as output.
Fig. (8)
Free eBooks on @neetquestionpaper2020

1004 Physi cs
In the absence of input signal vs, a D.C collector current IC flows
Base
in collector circuit due to forward biased battery VBE. This is current
called zero signal collector current. So total collector current is
I total collector current = IC + Ic
d.c. collector a .c. collector 80mA
current current
ib(signal
60mA IB current) DIB
vs Vout
40mA iB= IB+ ib
Curve & Vs= Vce Curve of IB=60mA
input signal output (d.c value) (d.c value + signal)
+ vs
signal
t
VBE VCE t (a)
–vs
–vs=–vce
t 4mA
O ic(signal
(a) (b) current)
Fig. (9) DIC
(a) input signal superimposed on V IC
(a) Input signal superimposed on V BE
(b) Output amplifying signal wave from super imposed on V CE
2mA IC=3mA
The voltage from the collector to ground determines the output i =I +i
(d.c value) C c c
signal. When IC (during +ive half cycle) increases, VCE decreases (d.c value + signal)
& when IC decreases (during – ive half cycle) VCE increase. So t
(b)
output voltage is 180º out of phase with input signal in CE
amplifier: Fig. (a) Base wave forms or input wave forms
(b) Collector wave froms or output wave forms.
The current gain Ai is defined as,
ICRC = .003A × 3000W = 9.0V
DI Change in load current/collector current
Ai = C = So by K.V.L the voltage from collector to emitter is
DI B Change in input current/base current VCE = VCC – ICRC = 20 – 9 = 11.0V
The voltage gain Av is defined as, Now we introduce signal voltage vs, which has peak value 50mV.
DVCE Change in load voltage During +ive half cycle of vs = + 50mV, IB rises to 80mA & during
Av = = –ive half cycle of vs = – 50mV. IB decreases to 40mA.
DVBE Change in input voltage
So change in base current is DIB (80 – 60) mA = 20mA
The power gain Ap is defined as, Ap = Ai × Av and change in collector current is DIC (bac DIB) = 50 × 20 = 1mA
For example : Let VBE =0 .7V, VCC = 20V It means during +ive half cycle IC becomes 4mA hence
IB = 60mA, IC = 3m A, RC = 3 kW VCE = VCC – ICRC = 20 – 12 = 8.V
IC 3mA During the negative half cycle IC becomes 2mA, hence
Then bd.c = = = 50 VCE = 20 – 6 = 14V.
I B 60mA
so DVCE = 3V
The collector current flowing through RC results in voltage drop:
DI C 1mA
vBE (signal)= vs so Ai = = = 50
DI B 20mA
DVBE
DVCE 3V
VBE =0.7V v BE = VBE + vbe Av = = = 60
0.5v–
dc bias DVBE 50mV
bias + signal
and Ap = Ai × Av = 3000
time
Collector to
VCC = 20V emitter voltage
20V
ICR C = 9V 14.0V vce
v (signal)= vout
(d.c value) ce
DVCE 11.0V
10V–
VCE=11V vCE= VCE + vce 8.0V
(d.c value) (d.c value + signal) VCE =11.0V nCE = VCE + vce

time
time
Free eBooks on @neetquestionpaper2020

Semi conductor Electroni cs : Materials, Devi ces an d Simple Circuits 1005


Keep in Memory Solution : (c)
1. Amplifier is a device used to increase the amplitude of input a
b= or b(1 - a) = a or b - ba = a
signal. 1- a
2. Amplification factor of triode valve m = gm × Rp where or b = a (1 + b) ; \ a = b /(1 + b)
gm = mutual or transconductance and Rp = plate resistance. Example 9.
3. Transistor is also used as an oscillator. The current gain of a transistor in common base mode is
Frequency of transistor LC oscillator is given by, 0.99. To change the emitter current by 5 mA, what will be
1 the necessary change in collector current?
f= Solution :
2p LC
4. In phase shift oscillator (which consists of three RC section), DΙ c
= a or DΙ c = a D Ι e ; D Ι c = .99 ´ 5 = 4.95 mA
1 DΙ e
the frequency is given by f =
2p 6RC Example 10.
For a transistor, the current amplification factor is 0.8,
5. Oscillator gives continuous, undamped oscillating output.
the transistor is connected in common emitter
6. Inverter : An oscillator which converts d.c. into a.c.
configuration. Find the change in the collector current
7. An amplifier with feed back in proper phase and magnitude when the base current changes by 6 mA.
behaves as an oscillator. Solution :
8. Berkhausen criteria : for amplifier to act as oscillator :
a 0.8
Gain with feed back, A´ = A/[1 - KA], K = feed back ratio. a = 0.8 ; b = = =4;
Criteria to become oscillator, (1 - KA) = 0. 1 - a 1 - 0.8
“Where A' is called closed loop gain & A is called open b = DΙ c / DΙ b or DΙ c = b DΙ b = 4 ´ 6 = 24 mA
loop gain.
9. The basic rules for normal operation of a transistor are Example 11.
(i) the emitter-base junction is forward blased to offer a The current gain of a transistor in a common emitter
low resistance to the flow of current. configuration is 40. If the emitter current is 8.2 mA, then
(ii) the collector-base junction is reverse biased to offer a find the base current.
high resistance to the flow of current. Solution :
Ι c Ιe - Ιb Ιe Ι
Comparative study of CB, CE and CC Amplifier b= = = - 1 or e =1 + b
Ιb Ιb Ιb Ιb
Use and property Common base Common emitter Common collector
(CB) amplifier (CE) amplifier (CC) amplifier Ιe 8.2 8.2
or Ιb = = = = .20 mA
Power Current 1 + b 1 + 40 41
Used for Voltage
amplification amplification amplification Example 12.
In a common emitter transistor amplifier b = 60,
Impedance Low High Low R0 = 5000 W and internal resistance of a trnasistor is
Output
500 W. What will be the voltage amplification of amplifier?
High High Low Solution :
Impedance

Ai< 1 Ai (B + 1)> 1 R0 5000


Current gain Ai< 1 Av = b = 60 ´ = 600
(a) (b ) (b » 20 – 100) (20 – 200) Ri 500
Voltage gain Av > 1 Av < 1
Example 13.
Av > 1
In a n-p-n transistor 1010 electrons enter the emitter in
10–6 s. 4% of the elctrons are lost in the base. What will be
Power gain Ap > 1 Ap > 1 Av > 1
the current transfer ratio?
Solution :
Phase shift Nil 180° Nil No. of electrons reaching the collector,
96
nc = ´ 1010 = 0.96 ´ 1010
Example 8. 100
The correct relationship between two current gains a and
ne ´e
b in a transistor is Emitter current, Ι e = ;
t
a 1+ a
(a) b = (b) b = nc ´e
1+ a a Collector current, Ι c =
t
b 1+ b
(c) a = (d) a = Ι c n c 0.96 ´ 1010
1+ b b \ Current transfer ratio, a = = = = 0.96
Ιe ne 1010
Free eBooks on @neetquestionpaper2020

1006 Physi cs
Example 14. Solution :
A p-n-p transistor is used in common-emitter mode in an
amplifier circuit. A change of 40 mA in the base current AT12e - W1 / kT1
2
J1 æ T1 ö - W1 / kT1 + W2 / kT2
brings a change of 2 mA in collector current and 0.04 V in = ç ÷ e
J 2 AT22 e - W2 / kT2 = çè T2 ÷ø
base emitter voltage. Find the (i) input resistance Ri (ii)
the base current amplification factor (b). (iii) If a load
resistance of 6 kW is used, then also find the voltage gain 2 æ W2 T1 ö
of the amplifier. æT ö - W1 / kT1ççè1- W1 ´ T2 ÷÷ø
= çç 1 ÷ e
÷
Solution : è T2 ø
Here, DΙ b = 40 ´ 10 -6 A ; DΙ c = 2 ´10 -3 A ;
2
æ 1ö 1 1
DVi = 0.04 V ; R0 = 6000W = ç ÷ e - W / kT1 (1- 2 /1´1/ 2) = e0 =
è 2ø 4 4
DVi 0.04
(i) Input resistance, R i = = = 103 W Example 16.
DI b 40 ´10 -6
In presence of interspace charge, at plate voltage of
(ii) Base current amplification factor, 200 V, the current is 80 mA. Then find the current in mA at
400 V.
DI c 2 ´ 10 -3
b= = = 50 Solution :
DI b 40 ´10 -6
3/ 2 3/ 2
R 6000 Ι 2 æ V2 ö æV ö
(iii) Voltage gain, Av = b 0 = 50 ´ = 300 . =ç ÷ or Ι 2 = Ι1 ´ ç 2 ÷÷
Ι1 çè V1 ÷ø çV
Ri 103 è 1 ø
Example 15.
3/ 2
The ratio of work functions and temperatures of two æ 400 ö
= 80ç ÷ = 80 ´ 2 2 = 160 2
emitters are 1 : 2, then what will be the ratio of current è 200 ø
densities obtained by them?

29.4
Solve following problems with the help of above text and 6. Amplifier may be
examples. (a) multi stage (b) single stage
1. In a transistor (c) both (a) & (b) (d) None of these
(a) the emitter has the least concentration of impurity 7. In common emitter cirucit, current gain is
(b) the collector has the least concentration of impurity (a) zero
(c) the base has the least concentration of impurity (b) same as in other configuration
(d) all the three regions have equal concentrations of
(c) lowest (d) highest
impurity
2. Current gain in common emitter configuration is more than 8. In common base circuit, output resistance is
1 because (a) very high (b) low
(a) Ic < Ib (b) Ic < Ie (c) Ic > Ie (d) Ic > Ib (c) very low (d) moderate
3. Current gain in common base configuration is less than 1 9. In common collector circuit, voltage gain is
because (a) very high (b) moderate
(a) Ie < Ib (b) Ib < Ie (c) Ic < Ie (d) Ie < Ic (c) low (d) very low
4. Operating point of a transistor is 10. The main defference between voltage and power amplifiers
(a) zero signal value of VCC and Ib is that
(b) zero signal value of Ic (a) power amplifier handles current
(c) zero signal value of Vcc
(b) power amplifier handles large voltage
(d) zero signal value of Ic and VCE
(c) power amplifier handles large power
5. A transistor is essentially
(a) a current operated device (d) None of these
(b) power driven device 11. Three amplifier stages each with a gain of 10 are cascaded.
(c) a voltage operated device The overall gain is
(d) resistance operated device (a) 10 (b) 30 (c) 1000 (d) 100
Free eBooks on @neetquestionpaper2020

Semi conductor Electroni cs : Materials, Devi ces an d Simple Circuits 1007

12. In a transistor 14. Negative feed back


(a) both emitter and collector have same length (a) increases stability (b) decreases stability
(c) produces oscillation (d) stops current in the tube
(b) length of emitter is greater than that of collector
15. In class A amplifiers, current through load flows for
(c) length of collector is greater than that of emitter (a) full cycle of the signal (b) half cycle of the signal
(d) any one of emitter and collector can have greater
1
length (c) cycle of the signal (d) None of these
4
13. In a transistor, the base is
16. Tuned amplifiers have
(a) a conductor of low resistance (a) tuned LC circuit as collector load
(b) a conductor of high resistance (b) LC circuit
(c) an insulator (c) RC circuit
(d) an extrinsic semiconductor (d) LR circuit

ANSWER KEY
1. (c) 2. (d) 3. (c) 4. (d) 5. (d) 6. (c) 7. (d) 8. (d) 9. (a) 10 (b) 11. (c) 12. (c)
13. (d) 14. (a) 15. (a) 16. (a)

LOGIC GATES NAND gate is called the building block of all digital circuits.
A logic gate is a digital circuit that follow certain logical NOR gate : It is the combination of NOT and OR gate. The
relationship between input and output voltages. They control output is high, when all inputs are low.
the flow of information. Truth table Logic symbol
Basic logic gates are : AND, OR and NOT gates. A B Y
AND gate : The output is high or 1 when all inputs are high. 0 0 1 A
1 0 0 Y
Truth table Logic symbol 0 1 0 B
A B Y 1 1 0
A
0 0 0
Y
0 1 0 Boolean expression : A + B = Y
1 0 0 B
Exclusive OR (XOR) gate : The output of a two input exclusive
Boolean expression : A . B = y OR gate is at logical 1 if one and only one input accepts logical
OR gate : Output is high even if one of the inputs is high. 0 (zero)
Truth table Logic symbol
A B Y A Y=A+B
0 0 0 A Logic symbol:
0 1 1 B
1 0 1
B
1 1 1
Truth table
Boolean expression : A + B = y
NOT gate : Output is not the input. Input Output
Logic symbol A B Y=A ÅB
Truth table 0 0 0
A Y A
0 1 0 1 1
1 0 B 1 0 1
1 1 0
Boolean expression : y = A
Logic expression : Y = A Å B = AB + AB
NAND gate : It is the combinatin of AND and NOT gate. The
output is high, even if all inputs are low (i.e., 0) or one input is Note : This circuit is also called inequality comparator or
low. detector because it produces an output only when the two
inputs are different.
Truth table
A B Y
Logic symbol BINARY NUMBERS AND DECIMAL NUMBERS
0 0 1 A
· In a binary system, a number is expressed by only two
1 0 1 Y digits 0 and 1. The base of the binary system is thus 2.
0 1 1
B · 0 and 1 represent the coefficients of powers of 2.
1 1 0
· A binary digit (0 or 1) is known as a bit.
Boolean expression : A.B = Y · A group of 4 bit is called a nibble & a group of 8 bit is
called byte.
Free eBooks on @neetquestionpaper2020

1008 Physi cs
Decimal to Binary Conversion Some basic boolean identies :
(A) When the decimal number is an integer – OR operation AND operation NOT operation
Divide the decimal number progressively by 2 until the A+ 0 = A A.0 = 0 A+ A =1
quotient is zero. The remainders of the successive divisions, A+1=1 A.1= A A A=0
written in the reverse order, give the binary number.
A+A =A A.A = A A=A
For example : We want to convert (9)10 into binary system.
A+ A=1 A. A = 0
A +B =B +A A. B=B .A
2 9 1 ® First remainder
A(B + C) = AB + ACA + BC = (A + B) (A + C)
4
0 ® Second remainder A + AB = A A(A + B) = A
2
( ­ Read up) A+ AB=A +B A( A + B) = AB
1
0 ® Third remainder These above theorems, identities & relations can be easily proved.
0
1 ® Fourth remainder Keep in Memory

1. The basic concept of digital circuit has been provided by


So (9)10 = (1001)2 George Boole.
(B) When the decimal number is a fraction – 2. Claude shanon established an analogy between function
The number is multiplied repeatedly by 2, and the carry in of mechanical switches and Boolean algebra.
the integer position is recorded each time. The process is 3. Series combination of switches is equivalent to AND logic
continued until the fractional part is zero or sufficient binary operation.
4. Parallel combination of switches is equivalent to OR logic
bits have been obtained.
operation.
Let we want to convert (0.85)10 into binary no. then 5. NOT logic operation is performed on a single variable. That’s
0.85 × 2 = 1.7 = 0.7 with a carry 1 why it is called unary operation.
0.7 × 2 = 1.4 = 0.4 with a carry 1 6. AND, OR and NOT logic operations follow closure property,
i.e., input as well as output are in either of the binary states.
0.4 × 2 = .8 = 0.8 with a carry 0 (¯ Read down)
7. NAND and NOR gates are universial gates
0.8 × 2 = 1.6 = 0.6 with a carry 1 8. If the logic gate is changed from positive to negative or
0.6 × 2 = 1.2 =0 .2 with a carry 1 vice-versa; AND changes into OR, OR changes into AND,
0.2 × 2 = .4 = 0.4 with a carry 0 NAND changes into NOR and NOR charges into NAND.
So (0.85)10 = (0.110110)2 Example 17.
Binary to Decimal Conversion Identify the gate represented by the block diagram shown
in fig. Write the Boolean expression and truth table.
(A) When the binary number is an integer –
Binary can be converted into its decimal equivalent by
noting that the successive digits from the extreme right of a A
I y
binary number are the coefficients of ascending power of 2,
beginning with the zeroth power of 2 at the extreme right. III
B
(B) When the binary number is a fraction –
II
The decimal equivalent of the binary number is found by
multiplying each digit in fraction sucessively by Solution :
2–1, 2–2, 2–3...etc. Here for the input, the two NOR gates have been used as
Let we want to convert (101.011)2 into its decimal equivalent. NOT gates (by joining the input terminals of NOR gate).
Their outputs are jointly fed to the NOR gate. From the
Then
(101.011)2 = 1 × 22 + 0 × 21 + 1 × 20 NOR gate I, for the input A, the output is A . From the NOR
+ 0 × 2–1 + 1 × 2–2 + 1 × 2–3 = (5.375)10 gate II, for the input B, the output is B . From NOR gate III,
5.375 = 5 × 100 + 3 × 10–1 + 7 × 10–2 + 5 × 10–3 the output is given by Y = A + B = A.B
Demorgan Theorems Thus Boolean expression for this combination of gate is
Y = A + B = A.B
(i) A.B = A + B (ii) A + B = A . B
which is for AND gate. Thus the combination will work as
These theorems are self proved. AND gate. The truth table of the combination of gates is
Free eBooks on @neetquestionpaper2020

Semi conductor Electroni cs : Materials, Devi ces an d Simple Circuits 1009


shown in fig. (a) an ‘OR’ gate and an ‘AND’ gate respectively
A B Y (b) an ‘AND’ gate and a ‘NOT’ gate respectively
A B
(c) an ‘AND’ gate and an ‘OR’ gate respectively
0 0 1 1 0
(d) an ‘OR’ gate and a ‘NOT’ gate respectively
1 0 0 1 0
Solution : (a)
0 1 1 0 0
1 1 0 0 1 For first case, C1 = A . B = ( A + B) (by Demorgan's
theorem)
Example 18.
The truth table is shown below
The combinations of the ‘NAND’ gates shown here in fig.
are equivalent to A B A B A.B A.B
1 0 0 1 0 1
A A 0 1 1 0 0 1
C1 0 0 1 1 1 0
B B
1 1 0 0 0 1
This is truth table for C1 = A + B i.e. OR gate
A C2
B For second case, C 2 = A .B = A . B i.e., AND gate.

29.5
Solve following problems with the help of above text and 4. To which logic gate does the truth table given below
examples. correspond ?
1. What is the value of A + A in the Boolean algebra ? A B Y
(a) 0 (b) 1 (a) OR gate 0 0 0
(c) A (d) A (b) AND gate 1 0 0
(c) NAND gate 0 0 0
2. What is the value of A.A in Boolean algebra?
(d) NOR gate 1 1 1
(a) 0 (b) 1
5. To which logic gate does the truth table given below
(c) A (d) A correspond?
3. To which logic gate does the truth table given below A B Y
correspond?
(a) OR gate 0 0 1
A B Y
(a) AND gate 0 0 0 (b) AND gate 1 0 1
(b) NAND gate 0 1 1 (c) NAND gate 0 1 1
(c) NOR gate 1 0 1 (d) NOR gate 1 1 0
(d) XOR gate 1 1 0
ANSWER KEY

1. (b) 2. (a) 3. (d) 4. (b) 5. (c)


Free eBooks on @neetquestionpaper2020

1010 Physi cs

Very Short/ Short Answer Questions 13. Derive a relation between current gain of a common base
amplifier and current gain of common emitter amplifier.
1. How does the depletion region of a p-n junction diode get
14. Discuss how AND gate is realised from NAND gate?
affected under reverse bias?[Delhi Board-2012 COMPTT.]
Long Answer Questions
2. The figure shows the V – I characteristics of a
semiconductor device. [Delhi Board-2012 COMPTT.] 15. (a) Describe briefly with the help of a necessary circuit
I (mA) diagram, the working principle of a solar cell.
(b) Why are Si and GaAs preferred materials for solar
Reverse bias cells? Explain. [Outside Delhi-2011 COMPTT.]
Vz Forward bias
V (V) 16. Draw the general shape of the ‘transfer characteristics’ of
a transistor in its CE configuration.
Which regions of this characteristic of a transistor, are
I(mA) used when it works (a) as a switch, the (b) as an amplifer?
3. (a) Which semiconductors are preferred to make LED’s Why is the output voltage of the CE amplifier opposite in
and why? phase with the input voltage?
(b) Give two advantages of using LED’s over [Outside Delhi-2010 COMPTT.]
conventional incandescent low power lamps.
17. State the principle of working of p-n diode as a rectifier.
[Delhi Board-2011 COMPTT.]
Explain, with the help of a circuit diagram, the use of p-n
4. Identify the logic gates ‘X’ and ‘Y’ in the figure. Write diode as a full wave rectifier. Draw a sketch of the input
down the truth table for output Z for all possible inputs ‘A’ and output wave forms. [Delhi Board-2007]
and ‘B’. [Delhi Board-2011 COMPTT.]
18. Discuss the working of transistor as an amplifier. Determine
A various gains.
X Y Z 19. Discuss how transistor when given a feedback in the
B
amplifier works as an oscillator.
5. Write the main use of the [Outside Delhi-2010 COMPTT.] 20. A transistor has a current gain of 50. If the collector
(i) photodiode and (ii) zener diode. resistance 5K W and the input resistance is 1K W. Calculate
6. You are given a circuit below. Write its truth table. Hence, the output voltage if input voltage is 0.01V.
identify the logic operation carried out by this circuit. Multiple Choice Questions
[Outside Delhi-2011]
21. Reverse bias applied to a junction diode
(a) it increases the minority carrier current
A X (b) it lowers the potential barrier
Z (c) it raises the potential barrier
(d) it increases the majority carrier current
B Y
22. In semiconductors, at room temperature
(a) the conduction band is completely empty
7. What is drift current?
(b) the valence band is partially empty and the
8. What is barrier potential? conduction band is partially filled
9. When is a diode reverse biased?
(c) the valence band is completely filled and the
10. What do you understand by breakdown voltage?
conduction band is partially filled
11. Why is base of a transistor made thin?
(d) the valence band is completely filled
12. Three amplifiers are connected in series. The voltage gain
of each is 5. What is the final voltage amplification?
Free eBooks on @neetquestionpaper2020

Semi conductor Electroni cs : Materials, Devi ces an d Simple Circuits 1011


23. Choose the only false statement from the following. 26. In a semiconductor diode, the barrier potential offers
(a) In conductors, the valence and conduction bands opposition to
may overlap. (a) holes in P-region only
(b) Substances with energy gap of the order of 10 eV are (b) free electrons in N-region only
insulators. (c) majority carriers in both regions
(c) The resistivity of a semiconductor increases with (d) majority as well as minority carriers in both regions
increase in temperature.
(d) The conductivity of a semiconductor increases with 27. What is the value of A.C + A.B.C where A, B and C are
increase in temperature. inputs ?
24. Application of a forward bias to a p–n junction (a) A.C (b) A.B
(a) widens the depletion zone.
(c) A (d) B
(b) increases the potential difference across the depletion
zone 28. In Boolean algebra, Y = A + B implies that
(c) increases the number of donors on the n side. (a) output Y exists when both inputs A and B exist
(d) increases the electric field in the depletion zone.
(b) output Y exists when either input A exists or input B
25. A transistor has three impurity regions. All the three regions exists or both inputs A and B exist
have different doping levels. In order of increasing doping
(c) output Y exists when either input A exists or input B
level, the regions are
exists but not when both inputs A and B exist
(a) emitter, base and collector
(b) collector, base and emitter (d) output Y exists when both inputs A and B exists but
(c) base, emitter and collector not when either input A or B exist
(d) base, collector and emitter

1. The forward biasd diode is [CBSE PMT 2000] 4. In the case of a common emitter transistor amplifier the
ratio of the collector current to the emitter current Ic /Ie is
(a) –2V +2V (b) –4V –3V
0.96. The current gain of the amplifier is
(c) 3V 5V (d) 0V –2V [CBSE PMT 2002]
(a) 6 (b) 48
2. In a P -N junction [CBSE PMT 2002]
(a) the potential of P & N sides becomes higher (c) 24 (d) 12
alternately 5. Barrier potential of a P-N junction diode does not depend
(b) the P side is at higher electrical potential than N side. on [CBSE PMT 2003]
(c) the N side is at higher electric potential than P side. (a) doping density (b) diode design
(d) both P & N sides are at same potential. (c) temperature (d) forward bias
3. A d.c. battery of V volt is connected to a series combination 6. A n-p-n transistor conducts when [CBSE PMT 2003]
of a resistor R and an ideal diode D as shown in the figure (a) both collector and emitter are negative with respect
below. The potential difference across R will be to the base
[CBSE PMT 2002] (b) both collector and emitter are positive with respect to
the base
R D (c) collector is positive and emitter is negative with
respect to the base
(d) collector is positive and emitter is at same potential
as the base
7. Following diagram performs the logic function of
[CBSE PMT 2003]
V
A
Y
(a) 2V when diode is forward biased B
(b) zero when diode is forward biased
(c) V when diode is reverse biased (a) XOR gate (b) AND gate
(d) V when diode is forward biased (c) NAND gate (d) OR gate
Free eBooks on @neetquestionpaper2020

1012 Physi cs
8. Reverse bias applied to a junction diode 15. The device that can act as a complete electronic circuit is
[CBSE PMT 2003] [CBSE PMT 2010]
(a) increases the minority carrier current (a) junction diode (b) integrated circuit
(b) lowers the potential barrier (c) junction transistor (d) zener diode
(c) raises the potential barrier 16. A common emitter amplifier has a voltage gain of 50, an
(d) increases the majority carrier current input impedance of 100W and an output impedance of 200W.
9. In semiconductors at a room temperature The power gain of the amplifier is [CBSE PMT 2010]
[CBSE PMT 2004] (a) 500 (b) 1000
(a) the conduction band is completely empty (c) 1250 (d) 50
(b) the valence band is partially empty and the 17. For transistor action [CBSE PMT 2010]
conduction band is partially filled (1) Base, emitter and collector regions should have similar
(c) the valence band is completely filled and the size and doping concentrations.
conduction band is partially filled (2) The base region must be very thin and lightly doped.
(d) the valence band is completely filled (3) The eimtter-base junction is forward biased and base-
10. The peak voltage in the output of a half-wave diode collector junction is reverse based.
rectifier fed with a sinusoidal signal without filter is 10V. (4) Both the emitter-base junction as well as the base-
The d.c. component of the output voltage is collector junction are forward biased.
[CBSE PMT 2004] (a) (3) and (4) (b) (4) and (1)
(a) 20/p V (b) 10/Ö2 V (c) (1) and (2) (d) (2) and (3)
(c) 10/p V (d) 10V 18. The following figure shows a logic gate circuit with two
11. In a p-n junction photo cell, the value of the photo- inputs A and B and the output Y. The voltage waveforms of
electromotive force produced by monochromatic light is A, B and Y are given [CBSE PMT 2010]
proportional to [CBSE PMT 2004] A Logic gate
(a) the voltage applied at the p-n junction circuit Y
(b) the barrier voltage at the p-n junction B
(c) the intensity of the light falling on the cell
(d) the frequency of the light falling on the cell
12. Of the diodes shown in the following diagrams, which 1
A
one is reverse biased ? [CBSE PMT 2004] 0
+10 V

R B 1
(a) (b)
+5 V –12 V R
0
–5 V
+5 V

R
1
(c) R
(d)
Y 0
–10 V

13. Choose the only false statement from the following.


[CBSE PMT 2005] t1 t2 t3 t4 t5 t6
(a) In conductors the valence and conduction bands may The logic gate is :
overlap. (a) NAND gate (b) NOR gate
(b) Substances with energy gap of the order of 10 eV are (c) OR gate (d) AND gate
insulators. 19. A transistor is operated in common emitter configuration at
(c) The resistivity of a semiconductor increases with VC = 2V such that a change in the base current from 100 mA
increase in temperature. to 300 mA produces a change in the collector current from
(d) The conductivity of a semiconductor increases with 10mA to 20 mA. The current gain is [CBSE PMT 2011]
increase in temperature. (a) 50 (b) 75
14. Which one of the following statement is false ? (c) 100 (d) 25
[CBSE PMT 2010] 20. In forward biasing of the p–n junction[CBSE PMT 2011]
(a) Pure Si doped with trivalent impurities gives a p-type (a) the positive terminal of the battery is connected to
semiconductor p–side and the depletion region becomes thick
(b) Majority carriers in a n-type semiconductor are holes (b) the positive terminal of the battery is connected to
(c) Minority carriers in a p-type semiconductor are n–side and the depletion region becomes thin
electrons (c) the positive terminal of the battery is connected to
(d) The resistance of intrinsic semiconductor decreases n–side and the depletion region becomes thick
with increase of temperature (d) the positive terminal of the battery is connected to
p–side and the depletion region becomes thin
Free eBooks on @neetquestionpaper2020

Semi conductor Electroni cs : Materials, Devi ces an d Simple Circuits 1013


21. Symbolic representation of four logic gate are shown as is 1kW and the current amplification of the transistor is
[CBSE PMT 2011] 100, the input signal voltage is [CBSE PMT 2012 S]
(a) 0.1 V (b) 1.0 V
(c) 1 mV (d) 10 mV
(i) (ii)
27. Transfer characteristics [output voltage (V0 ) vs input
voltage (V1)] for a base biased transistor in CE configuration
is as shown in the figure. For using transistor as a switch, it
(iii) (iv) is used [CBSE PMT 2012 S]
Pick out which ones are for AND, NAND and NOT gates, V0 I II
respectively III
(a) (ii), (iii) and (iv) (b) (iii), (ii) and (i)
(c) (iii), (iii) and (iv) (d) (ii), (iv) and (iii)
22. If a small amount of antimony is added to germanium crystal
[CBSE PMT 2011] Vi
(a) it becomes a p–type semiconductor (a) in region (III)
(b) the antimony becomes an acceptor atom (b) both in region (I) and (III)
(c) there will be more free electrons than holes in the (c) in region (II)
semiconductor (d) in region (I)
(d) its resistance is increased 28. The figure shows a logic circuit with two inputs A and B
23. Pure Si at 500K has equal number of electron (ne) and hole and the output C. The voltage wave forms across A, B and
(nh) concentrations of 1.5 × 1016 m–3. Doping by indium C are as given. The logic gate circuit is:
increases n h to 4.5 × 1022 m–3. The doped semiconductor is [CBSE PMT 2012 S]
of [CBSE PMT 2011 M]
(a) n–type with electron concentration
ne = 5 × 1022 m–3
(b) p–type with electron concentration
ne = 2.5 ×1010 m–3 A
(c) n–type with electron concentration
ne = 2.5 × 1023 m–3
(d) p–type having electron concentration
ne = 5 × 109 m–3 B
24. A zener diode, having breakdown voltage equal to 15V, is
used in a voltage regulator circuit shown in figure. The
current through the diode is [CBSE PMT 2011 M]
C
t1 t2 t3 t4 t5 t6
250W (a) OR gate (b) NOR gate
20V 15V 1kW (c) AND gate (d) NAND gate
29. The input resistance of a silicon transistor is
100 W. Base current is changed by 40 mA which results in a
change in collector current by 2 mA. This transistor is used
(a) 10 mA (b) 15 mA
as a common emitter amplifier with a load resistance of 4
(c) 20 mA (d) 5 mA
KW. The voltage gain of the amplifier is
25. Two ideal diodes are connected to a battery as shown in
[CBSE PMT 2012 M]
the circuit. The current supplied by the battery is
(a) 2000 (b) 3000
[CBSE PMT 2012 S]
(c) 4000 (d) 1000
D1 10W 30. In a n-type semiconductor, which of the following statement
A B
is true? [NEET 2013]
D2 20W (a) Electrons are minority carriers and pentavalent atoms
C D
are dopants.
E
5V
F
(b) Holes are minority carriers and pentavalent atoms are
dopants.
(a) 0.75 A (b) zero (c) Holes are majority carriers and trivalent atoms are
(c) 0.25 A (d) 0.5 A dopants.
26. In a CE transistor amplifier, the audio signal voltage across (d) Electrons are majority carriers and trivalent atoms are
the collector resistance of 2kW is 2V. If the base resistance dopants.
Free eBooks on @neetquestionpaper2020

1014 Physi cs
31. In a common emitter (CE) amplifier having a voltage gain G, (a) – 24.8 (b) – 15.7
the transistor used has transconductance 0.03 mho and (c) – 5.2 (d) – 48.78
current gain 25. If the above transistor is replaced with 41. When p-n junction diode is forward biased then
another one with transconductance 0.02 mho and current [AIEEE 2004]
gain 20, the voltage gain will be [NEET 2013] (a) both the depletion region and barrier height are reduced
(b) the depletion region is widened and barrier height is
1 reduced
(a) 1.5 G (b) G
3 (c) the depletion region is reduced and barrier height is
increased
5 2 (d) Both the depletion region and barrier height are increased
(c) G (d) G
4 3 42. The electrical conductivity of a semiconductor increases
32. The output(X) of the logic circuit shown in figure will be when electromagnetic radiation of wavelength shorter than
[NEET 2013] 2480 nm is incident on it. The band gap in (eV) for the
semiconductor is [AIEEE 2005]
(a) 2.5 eV (b) 1.1 eV
(c) 0.7 eV (d) 0.5 eV
43. In a common base amplifier, the phase difference between
the input signal voltage and output voltage is
(a) X = A.B (b) X = A.B
[AIEEE 2005]
p
(c) X = A + B (d) X = A, B (a) p (b)
4
33. At absolute zero, Si acts as [AIEEE 2002]
p
(a) non-metal (b) metal (c) (d) 0
(c) insulator (d) none of these 2
34. By increasing the temperature, the specific resistance of a 44. In a common base mode of a transistor, the collector current
is 5.488 mA for an emitter current of 5.60 mA. The value of
conductor and a semiconductor [AIEEE 2002]
the base current amplification factor (b) will be [AIEEE 2005]
(a) increases for both (b) decreases for both (a) 49 (b) 50
(c) increases, decreases (d) decreases, increases (c) 51 (d) 48
35. The energy band gap is maximum in [AIEEE 2002] 45. If in a p-n junction diode, a square input signal of 10 V is
(a) metals (b) superconductors applied as shown [AIEEE 2007]
(c) insulators (d) semiconductors.
36. The part of a transistor which is most heavily doped to 5V
produce large number of majority carriers is [AIEEE 2002] RL
(a) emmiter
(b) base -5V
(c) collector
Then the output signal across RL will be
(d) can be any of the above three.
37. A strip of copper and another of germanium are cooled 10 V
+5V
from room temperature to 80K. The resistance of
[AIEEE 2003] (a) (b)
(a) each of these decreases
(b) copper strip increases and that of germanium decreases
(c) copper strip decreases and that of germanium increases
(d) each of these increases (c) (d)
-5V
38. In the middle of the depletion layer of a reverse- biased -10 V
p-n junction, the [AIEEE 2003] 46. Carbon, silicon and germanium have four valence electrons
(a) electric field is zero (b) potential is maximum each. At room temperature which one of the following
(c) electric field is maximum (d) potential is zero statements is most appropriate ? [AIEEE 2007]
39. When npn transistor is used as an amplifier [AIEEE 2004] (a) The number of free electrons for conduction is
(a) electrons move from collector to base significant only in Si and Ge but small in C.
(b) holes move from emitter to base (b) The number of free conduction electrons is significant
(c) electrons move from base to collector in C but small in Si and Ge.
(d) holes move from base to emitter (c) The number of free conduction electrons is negligibly
40. For a transistor amplifier in common emitter configuration small in all the three.
for load impedance of 1k W (hfe = 50 and h oe = 25ms) (d) The number of free electrons for conduction is
the current gain is [AIEEE 2004] significant in all the three.
Free eBooks on @neetquestionpaper2020

Semi conductor Electroni cs : Materials, Devi ces an d Simple Circuits 1015


47. A working transistor with its three legs marked P, Q and R is
tested using a multimeter. No conduction is found between
P and Q. By connecting the common (negative) terminal of
the multimeter to R and the other (positive) terminal to P or (a)
Q, some resistance is seen on the multimeter. Which of the
following is true for the transistor? [AIEEE 2008] I
(a) It is an npn transistor with R as base
(b) It is a pnp transistor with R as collector (b)
(c) It is a pnp transistor with R as emitter
(d) It is an npn transistor with R as collector t
48. In the circuit below, A and B represent two inputs and C
represents the output. [AIEEE 2008]

A (c)
C
I
B

(d)
The circuit represents
t
(a) NOR gate (b) AND gate
(c) NAND gate (d) OR gate 51. The combination of gates shown below yields[AIEEE 2010]
49. The logic circuit shown below has the input waveforms ‘A’
and ‘B’ as shown. Pick out the correct output waveform. A
[AIEEE 2009]
A X
Y
B B
Input A
(a) OR gate (b) NOT gate

Input B (c) XOR gate (d) NAND gate


52. Truth table for system of four NAND gates as shown in
figure is [AIEEE 2012]
Output is
A
(a)

Y
(b)

(c)
B

(d) A B Y A B Y
0 0 0 0 0 0
50. A p-n junction (D) shown in the figure can act as a rectifier.
An alternating current source (V) is connected in the circuit. 0 1 1 0 1 0
(a) (b)
[AIEEE 2009] 1 0 1 1 0 1
1 1 0 1 1 1

D A B Y A B Y
V ~ R 0 0 1 0 0 1
0 1 1 0 1 0
(c) (d)
1 0 0 1 0 1
The current (I) in the resistor (R) can be shown by : 1 1 0 1 1 1
Free eBooks on @neetquestionpaper2020

1016 Physi cs
53. The I-V characteristic of an LED is [JEE Main 2013]
V O
R YG B
I R
B
G (c) (d) Y I
G
(a) I (b) Y
R B
O V
O V O V

1. If lattice parameter for 3a crystalline structure is 3.6Å, 9. Distance between body centred atom & a corner atom in
then atomic radius in fcc crystal in Å is sodium(a = 4.225 Å) is
(a) 7.20 (b) 1.80 (a) 3.66 Å (b) 3.17 Å
(c) 1.27 (d) 2.90 (c) 2.99 Å (c) 2.54 Å
10. If the forward voltage in a semiconductor diode is changed
2. A crystal has bcc structure and its lattice constant is 3.6Å.
from 0.5V to 0.7 V, then the forward current changes by
What is the atomic radius ? 1.0 mA. The forward resistance of diode junction will be
(a) 3.6Å (b) 1.8Å (a) 100 W (b) 120 W
(c) 1.27 Å (d) 1.56 Å (c) 200 W (d) 240 W
3. A crystal has scc structure and its lattice constant is 11. Current gain of a transistor in common base mode is 0.95.
3.5 Å. What is the atomic raidus ? Its value in common emitter mode is
(a) 3.5 Å (b) 1.75 Å (a) 0.95 (b) 1.5
(c) 19 (d) (19)–1
(c) 1.237 Å (d) 1.52 Å
12. A transistor has b = 40. A change in base current of 100 m
4. The energy gap of silicon is 1.14 eV. The maximum A, produces change in collector current
wavelength at which silicon starts energy absorption, will (a) 40 × 100 microampere (b) (100 – 40) microampere
be (h = 6.62 × 10–34 Js ; c = 3 × 108 m/s) (c) (100 + 40) microampere (d) 100/40 microampere
(a) 10.888 Å (b) 108.88 Å 13. A transistor has a base current of 1 mA and emitter current
(c) 1088.8 Å (d) 10888 Å 90 mA. The collector current will be
5. On doping germanium with donor atoms of density (a) 90 mA (b) 1 mA
(c) 89 mA (d) 91 mA
1017 cm–3 its conductivity in mho/cm will be
14. In a common emitter transistor amplifier b = 60, Ro = 5000
[Given : me = 3800 cm2/V–s and ni = 2.5 × 1013 cm–13] W and internal resistance of a transistor is 500 W. The
(a) 30.4 (b) 60.8 voltage amplification of amplifier will be
(c) 91.2 (d) 121.6 (a) 500 (b) 460
6. The ratio of electron and hole currents in a semiconductor (c) 600 (d) 560
is 7/4 and the ratio of drift velocities of electrons and 15. For a common base amplifier, the values of resistance gain
holes is 5/4, then the ratio of concentrations of electrons and voltage gain are 3000 and 2800 respectively. The
current gain will be
and holes will be
(a) 1.1 (b) 0.98
(a) 5/7 (b) 7/5 (c) 0.93 (d) 0.83
(c) 25/49 (d) 49/25 16. In the binary number system 100 + 1011 is equal to
7. In the half wave rectifier circuit operating from 50 Hz mains (a) 1000 (b) 1011
frequency, the fundamental frequency in the ripple would (c) 1110 (d) 1111
be 17. The decimal number 16 in binary number is
(a) 25 Hz (b) 50 Hz (a) 1000 (b) 10000
(c) 70.7 Hz (d) 100 Hz (c) 1010 (d) 11000
18. The decimal number 605 in binary number will be
8. In a full wave rectifier circuit operating from 50 Hz mains
(a) 100101 (b) 10010111
frequency, the fundamental frequency in the ripple would (c) 1001011101 (d) 100101101
be 19. The binary number 1000 represented by
(a) 25 Hz (b) 50 Hz (a) 8 (b) 16
(c) 70.7 Hz (d) 100 Hz (c) 32 (d) 64
Free eBooks on @neetquestionpaper2020

Semi conductor Electroni cs : Materials, Devi ces an d Simple Circuits 1017


20. The binary number of decimal number (9.25)10 is 31. The truth-table given below is for which gate?
(a) 1101.01 (b) 1001.01
(c) 1001.10 (d) 1110.010 A B C
21. In a common base amplifier the phase difference between 0 0 1
the input signal voltage and the output voltage is 0 1 1
(a) 0 (b) p/4 1 0 1
1 1 0
(c) p/2 (d) p
22. The current gain in transistor in common base mode is (a) XOR (b) OR
0.99. To change the emitter current by 5 mA, the necessary
(c) AND (d) NAND
change in collector will be
(a) 0.196 mA (b) 2.45 mA 32. The cause of the potential barrier in a p-n diode is
(c) 4.95 mA (d) 5.1 mA (a) depletion of positive charges near the junction
23. The electrical conductivity of a semiconductor increases (b) concentration of positive charges near the junction
when electromagnetic radiation of wavelength shorter than (c) depletion of negative charges near the junction
2480 nm is incident on it. The band gap (in eV) for the
semiconductor is (d) concentration of positive and negative charges near
the junction
(a) 0.9 (b) 0.7
(c) 0.5 (d) 1.1 33. The value of b
24. What is the voltage gain in a common emitter amplifier, (a) is always less than 1
where input resistance is 3 W and load resistance 24 W, b
(b) lies between 20 and 200
= 0.6 ?
(a) 8 . 4 (b) 4 . 8 (c) is always greater than 200
(c) 2 . 4 (d) 480 (d) is always infinity
25. A half-wave rectifier is being used to rectify an alternating 34. In a bridge rectifier, the number of diodes required is
voltage of frequency 50 Hz. The number of pulses of (a) 1 (b) 2
rectified current obtained in one second is
(c) 3 (d) 4
(a) 50 (b) 25
35. When n-P-n transistor is used as an amplifier, then
(c) 100 (d) 2000
26. In a triode, gm = 2 × 10–3 ohm–1 ; m = 42; resistance of (a) electrons move from collector to emitter
load, R = 50 kilo ohm. The voltage amplification obtained (b) electrons move from emitter to collector
from this triode will be (c) electrons move from collector to base
(a) 30.42 (b) 29.57 (d) holes move from emitter to collector
(c) 28.18 (d) 27.15 36. The intrinsic conductivity of germanium at 27° is 2.13 mho
27. In a p-n junction having depletion layer of thickness m–1 and mobilities of electrons and holes are 0.38 and
10–6 m the potential across it is 0.1 V. The electric field is 0.18 m2V–1s–1 respectively. The density of charge carriers
(a) 107 V/m (b) 10–6 V/m is
5
(c) 10 V/m (d) 10–5 V/m (a) 2.37 × 1019 m–3 (b) 3.28 × 1019 m–3
28. In a junction diode, the holes are due to
(c) 7.83 × 1019 m–3 (d) 8.47 × 1019 m–3
(a) protons (b) extra electrons
37. An oscillator is nothing but an amplifier with
(c) neutrons (d) missing electrons
29. The difference in the variation of resistance with (a) positive feedback (b) large gain
temperature in a metal and a semiconductor arises (c) no feedback (d) negative feedback
essentially due to the difference in the 38. Minority carriers in a p-type semiconductor are
(a) crystal structure (a) free electrons
(b) variation of the number of charge carriers with (b) holes
temperature
(c) neither holes nor free electron
(c) type of bonding
(d) both holes and free electrons.
(d) variation of scattering mechanism with temperature
39. In a reverse biased diode when the applied voltage changes
Ic by 1 V, the current is found to change by 0.5 µA. The reverse
30. In common emitter amplifier the is 0.98. The current
Ie bias resistance of the diode is
gain will be
(a) 4.9 (b) 7.8 (a) 2 × 105 W (b) 2 × 106 W
(c) 49 (d) 78 (c) 200 W (d) 2 W.
Free eBooks on @neetquestionpaper2020

1018 Physi cs
40. Which of the following represents NAND gate ? 49. The ratio of work function and temperature of two emitters
are 1 : 2, then the ratio of current densities obtained by
them will be
(a) (a) 4 : 1 (b) 2 : 1
(c) 1 : 2 (d) 1 : 4
50. The transfer ratio b of transistor is 50. The input resistance
(b) of a transistor when used in C.E. (Common Emitter)
configuration is 1kW. The peak value of the collector A.C
current for an A.C input voltage of 0.01V peak is
(a) 100 mA (b) .01 mA
(c)
(c) .25 mA (d) 500 mA
51. The intrinsic semi conductor becomes an insulator at
(a) 0ºC (b) 0 K
(d) (c) 300 K (d) –100ºC
52. The manifestation of band structure in solids is due to
41. In insulator (a) Bohr’s correspondence principle
(a) valence band is partially filled with electrons (b) Pauli’s exclusion principle
(b) conduction band is partially filled with electrons (c) Heisenberg’s uncertainty principle
(c) conduction band is filled with electrons and valence (d) Boltzmann’s law
band is empty 53. The frequency response curve of RC coupled amplifier is
(d) conduction band is empty and valence band is filled shown in figure. The band with of the amplifier will be
with electrons.
42. When the forward bias voltage of a diode is changed from
0.6 V to 0.7 V, the current changes from 5 mA to 15 mA. A max
Then its forward bias resistance is
0.707 A max

(a) 0.01 W (b) 0.1 W


(c) 10 W (d) 100 W
43. The current gain of a transistor in common base mode is
0.995. The current gain of the same transistor in common
emitter mode is
(a) 197 (b) 201 f1 f 2 f3 f 4
(c) 198 (d) 199 (a) f3 – f2 (b) f4 – f1
44. The gate for which output is high if atleast one input is
f4 - f2
low? (c) (d) f3 – f1
2
(a) NAND (b) NOR
54. Sum of the two binary number (100010)2 and (11011)2 is
(c) AND (d) OR (a) (111101)2 (b) (111111)2
45. What is the conductivity of a semiconductor if electron (c) (101111)2 (d) (111001)2
density = 5 × 1012/cm3 and hole density = 8 × 1013/cm3 55. A diode having potential difference 0.5 V across its
(µe = 2.3 m2 V–1 s–1, µh = 0.01 m2V–1 s–1) junction which does not depend on current, is connected
(a) 5.634 (b) 1.968
(c) 3.421 (d) 8.964. in series with resistance of 20 W across source. If 0.1 A
46. In a p-type semiconductor the acceptor level is situated 60 current passes through resistance then what is the voltage
meV above the valence band. The maximum wavelength of of the source?
light required to produce a hole will be (a) 1.5 V (b) 2.0 V
(a) 0.207 × 10–5 m (b) 2.07 × 10–5 m (c) 2.5 V (d) 5 V
(c) 20.7 × 10–5 m (d) 2075 × 10–5 m 56. Copper has face centered cubic (fcc) lattice with
10 interatomic spacing equal to 2.54 Å. The value of lattice
47. In a npn transistor 10 electrons enter the emitter in
constant for this lattice is
10–6 s. 4% of the electrons are lost in the base. The current (a) 2.54 Å (b) 3.59 Å
transfer ratio will be (c) 1.27 Å (d) 5.08 Å
(a) 0.98 (b) 0.97 57. Carbon, Silicon and Germanium atoms have four valence
(c) 0.96 (d) 0.94 electrons each. Their valence and conduction bands are
48. The grid voltage of any triode valve is changed from –1 separated by energy band gaps represented by (Eg)C,
volt to –3 volt and the mutual conductance is 3 × 10–4 mho. (Eg)Si and (Eg)Ge respectively. Which one of the following
The change in plate circuit current will be relationship is true in their case?
(a) 0.8 mA (b) 0.6 mA (a) (Eg)C > (Eg)Si (b) (Eg)C < (Eg)Si
(c) 0.4 mA (d) 1 mA (c) (Eg)C = (Eg)Si (d) (Eg)C < (Eg)Ge
Free eBooks on @neetquestionpaper2020

Semi conductor Electroni cs : Materials, Devi ces an d Simple Circuits 1019


58. Application of a forward bias to a p–n junction 64. NAND and NOR gates are called universal gates primarily
(a) widens the depletion zone because they
(b) increases the potential difference across the depletion (a) are available universally
zone (b) can be combined to produce OR, AND and NOT gates
(c) increases the number of donors on the n side (c) are widely used in Integrated circuit packages
(d) increases the electric field in the depletion zone. (d) are easiest to manufacture
65. Two identical pn junctions may be connected in series,
59. A semi-conducting device is connected in a series circuit
with a battery in three ways as shown in figure. The
with a battery and a resistance. A current is found to
potential drops across the two pn junctions are equal in
pass through the circuit. If the polarity of the battery is
reversed, the current drops to almost zero. The device PN NP PN PN NP NP
may be
(a) a p-n junction
(b) an intrinsic semi-conductor Circuit 1 Circuit 2 Circuit 3
(c) a p-type semi-conductor (a) circuit 1 and circuit 2
(d) an n-type semi-conductor (b) circuit 2 and circuit 3
60. Which of the following gates will have an output of 1? (c) circuit 3 and circuit 1
(d) circuit 1 only
1 0 66. Metallic solids are always opaque because
1 1 (a) they reflect all the incident light.
(A) (B) (b) they scatter all the incident light.
0 0 (c) the incident light is readily absorbed by the free
1 0 electrons in a metal.
(C) (D) (d) the energy band traps the incident.
67. Assuming that the silicon diode having resistance of
(a) D (b) A 20 W , the current through the diode is
(c) B (d) C (knee voltage 0.7 V)
61. The following circut represents R =180W
A 2V 0V
(a) 0 mA (b) 10 mA
Y (c) 6.5 mA (d) 13.5 mA
68. The following configuration of gate is equivalent to
B
A OR
(a) OR gate (b) XOR gate B
(c) AND gate (d) NAND gate Y
62. The diagram of a logic circuit is given below. The output AND
F of the circuit is represented by NAND

W (a) NAND gate (b) XOR gate


(c) OR gate (d) NOR gate
X 69. One serious drawback of semi-conductor devices is
F (a) they do not last for long time.
W (b) they are costly
(c) they cannot be used with high voltage.
Y (d) they pollute the environment.
70. The ratio of forward biased to reverse biased resistance
(a) W . (X + Y) (b) W . (X . Y)
for pn junction diode is
(c) W + (X . Y) (d) W + (X + Y)
63. If A is the atomic mass number of an element, N is the (a) 10–1 : 1 (b) 10–2 : 1
Avogadro number and a is the lattice parameter, then the 4
(c) 10 : 1 (d) 10–4 : 1
density of the element, if it has bcc crystal structure, is 71. In germanium the energy gap is about 0.75 eV. The
wavelength of light which germanium starts absorbing is
A 2A 3A 2 2A
(a) (b) (c) (d) (a) 5000 Å (b) 1650 Å
Na 3
Na 3 Na 3 Na 3
(c) 16500 Å (d) 165000 Å
Free eBooks on @neetquestionpaper2020

1020 Physi cs
72. Which one is the weakest type of bonding in solids ? 78. Which one of the following gives the 2's complement of
(a) Ionic (b) Covalent decimal number 13?
(c) Metallic (d) Vander Wall’s (a) 0010 (b) 0011
(c) 1100 (d) 1101.
73. Two junction diodes one of Germanium (Ge) and other of
79. In a transistor, the change in base current from 100 µA to
silicon (Si) are connected as shown in figure to a battery
125 µA causes a change in collector current from 5 mA to
of emf 12 V and a load resistance 10 k W . The germanium 7.5 mA, keeping collector-to-emitter voltage constant at 10
diode conducts at 0.3 V and silicon diode at 0.7 V. When a V. What is the current gain of the transistor?
current flows in the circuit, the potential of terminal Y will (a) 200 (b) 100
be (c) 50 (d) 25.
Ge 80. The truth table given below is for
(a) NOR A B Y
Y (b) AND
(c) XOR 0 0 1
Si (d) NAND 0 1 1
12 V 10 kW 1 0 1
1 1 0
81. In the following circuit, the output Y for all possible
inputs A and B is expressed by the truth table.
A
(a) 12 V (b) 11 V A
Y
(c) 11.3 V (d) 11.7 V B
B
74. The current gain b may be defined as
(a) A B Y (b) A B Y
(a) the ratio of change in collector current to the change
0 0 1 0 0 1
in emitter current for a constant collector voltage in a
0 1 1 0 1 0
common base arrangement.
(b) the ratio of change in collector current to the change 1 0 1 1 0 0
in the base current at constant collector voltage in a 1 1 0 1 1 0
common emitter circuit (c) A B Y (d) A B Y
(c) the ratio of change in emitter current to the change in 0 0 0 0 0 0
base current for constant emitter voltage in common 0 1 1 0 1 0
emitter circuit. 1 0 1 1 0 0
(d) the ratio of change in base current to the change in
1 1 1 1 1 1
collector current at constant collector voltage in
82. In common emitter amplifier, the current gain is 62. The
common emitter circuit. collector resistance and input resistance are 5 kW an 500W
75. A solid that is not transparent to visible light and whose respectively. If the input voltage is 0.01 V, the output voltage
electrical conductivity increases with temperature is is
formed by (a) 0.62 V (b) 6.2 V
(c) 62 V (d) 620 V
(a) ionic binding (b) covalent binding
83. The real time variation of input signals A and B are as shown
(c) metallic binding (d) vander Waal’s binding below. If the inputs are fed into NAND gate, then select the
76. A metallic bond differs form a covalent bond in that output signal from the following.
(a) it is not directional
(b) it is not saturable
(c) the valence electrons are not attached to any particular A
A
atom B Y
(d) all of the above B
77. The transistor are usually made of t (s)
(a) metal oxides with high temperature coefficient of
resistivity
Y Y
(b) metals with high temperature coefficient of resistivity
(c) metals with low temperature coefficient of resistivity (a) (b)
(d) semiconducting materials having low temperature t (s) t (s)
0 2 4 6 8 0 2 4 6 8
coefficient of resistivity
Free eBooks on @neetquestionpaper2020

Semi conductor Electroni cs : Materials, Devi ces an d Simple Circuits 1021


88. Statement-1 : When two semi conductor of p and n type
are brought in contact, they form p-n junction which act
like a rectifier.
(c) Y (d) Y Statement-2 : A rectifier is used to convent alternating
t (s) t (s) current into direct current.
0 2 4 6 8 0 2 4 6 8
89. Statement-1 : NOT gate is also called invertor circuit.
84. The time variations of signals are given as in A, B and C.
Point out the true statement from the following : Statement-2 : NOT gate inverts the input order.
e e e 90. Statement 1 : Diode lasers are used as optical sources in
optical communication.
Statement 2 : Diode lasers consume less energy.
1.0 1.0 1.0
0 t 91. Statement 1 : A pure semiconductor has negative
0 t 0 t
(A) (B) (C) temperature coefficient of resistance.
(a) A, B and C are analogue signals Statement 2 : In a semiconductor on raising the temperature,
(b) A and B are analogue, but C is digital signal more charge carriers are released, conductance increases
(c) A and C are digital, but B is analogue signal and resistance decreases.
(d) A and C are analogue, but B is digital signal 92. Statement 1 : A transistor amplifier in common emitter
configuration has a low input impedence.
85. If the ratio of the concentration of electrons to that of holes Statement 2 : The base to emitter region is forward biased.
7 7 93. Statement 1 : If the temperature of a semiconductor is
in a semiconductor is and the ratio of currents is ,
5 4 increased then it’s resistance decreases.
Statement 2 : The energy gap between conduction band
then what is the ratio of their drift velocities?
and valence band is very small.
5 4 Directions for Qs. (94 to 101) : Read the following passage(s)
(a) (b)
8 5 carefully and answer the questions that follows:
5 4 PASSAGE-1
(c) (d)
4 7
86. The circuit has two oppositively connected ideal diodes in A student performs an experiment for drawing the static
parallel. What is the current flowing in the circuit? characteristic curve of a triode value in the laboratory. The
following data were obtained from the linear portion of the curves:
4W Grid voltage Vg (volt) –2.0 –3.5 –2.0
D1 D2 Plate voltage Vp (volt) 180 180 120
Plate current IP (mA) 15 7 10
12V 94. Calculate the plate resistance rp of the triode valve?
3W 2W
(a) 0.12 × 104 ohm (b) 1.2 × 104 ohm
4
(c) 1.3 × 10 ohm (d) 1.4 × 104 ohm
(a) 1.71 A (b) 2.00 A 95. Calculate the mutual conductance gm of the triode valve?
(c) 2.31 A (d) 1.33 A (a) 5.33 × 10–3 ohm–1 (b) 53.3 × 10–3 ohm–1
Directions for Qs. (87 to 93) : Each question contains –3
(c) 4.32 × 10 ohm –1 (d) 5.00 × 10–3 ohm–1
STATEMENT-1 and STATEMENT-2. Choose the correct answer 96. Calculate the amplification factor µ, of the triode valve?
(ONLY ONE option is correct ) from the following-
(a) 64 (b) 52
(a) Statement -1 is true, Statement-2 is true; Statement -2 is a (c) 54 (d) 62
correct explanation for Statement-1
PASSAGE-2
(b) Statement -1 is true, Statement-2 is true; Statement -2 is not
a correct explanation for Statement-1 Doping changes the fermi energy of a semiconductor. Consider
(c) Statement -1 is true, Statement-2 is false silicon, with a gap of 1.11 eV between the top of the valence bond
(d) Statement -1 is false, Statement-2 is true and the bottom of the conduction band. At 300K the Fermi level
87. Statement-1 : NAND or NOR gates are called digital building of the pure material is nearly at the midpoint of the gap. Suppose
blocks. that silicon is doped with donor atoms, each of which has a state
Statement-2 : The repeated use of NAND (or NOR) gates 0.15 eV below the bottom of the silicon conduction band, and
can produce all the basis or complicated gates. suppose further that doping raises the Fermi level to 0.11 eV
below the bottom of that band.
Free eBooks on @neetquestionpaper2020

1022 Physi cs
minimum current of 1 mA to be above the knee point 0.7 V of its
V-I characteristic curve. Also assume that the voltage V across
the diode is independent of current above the knee (cut-off) point
R

+ –
VB
97. For both pure and doped silicon, calculate the probability
that a state at the bottom of the silicon conduction band is 99. If VB = 5V, then the maximum value of R so that the voltage
occupied? V is above the knee point voltage, should be
(a) 5.20 × 10–2 (b) 1.40 × 10–2 (a) 0.7kW (b) 4.3 kW (c) 5 kW (d) 5.7 kW
(c) 10.5 × 10 –2 (d) 14 × 10–2
98. Calculate the probability that a donor state in the doped 100. If VB = 5V, then the value of R in order to establish a current
material is occupied? of 5 mA in the circuit, will be
(a) 0.824 (b) 0.08 (a) 140 W (b) 215 W (c) 430 W (d) 860 W
(c) 0.008 (d) 8.2
101. If VB = 6V and 5mA current flows through the circuit, then
PASSAGE-3 the power dissipated in R will be
A Si diode (p-n junction) is connected to a resistor and a biasing (a) 32.6 mW (b) 26.4 mW
battery of variable voltage VB. Assume that the diode requires a
(c) 3.26 mW (d) 2.6 mW
Free eBooks on @neetquestionpaper2020

Semi conductor Electroni cs : Materials, Devi ces an d Simple Circuits 1023

Exercise 29.1 6. (b) When we increase the temperature, then more charge
carriers are produced due to the breaking of covlent
1. (b) Calcite is a crystalline solid. bands & average drift speed [(vd)avg] decreases with
2. (d) In a crystalline solid, there is no free movement of atoms. temperature due to increase of vrms & decrease of
relaxation time t.
3. (c) For hexagonal crystal structure, a = b ¹ c and
a = b = 90º but g = 120º . r æ et ö r l
( v d ) av = - ç ÷E & t =
4. (a) The diamond is fcc structure èmø v rms

N c Nf Ni 3kT
6. (a) Number of atoms per unit cell, n = + + where l is mean free path, v rms =
8 2 1 m
For scc system, Nc = 8, Nf = 0 and Ni = 2, so n = 2 7. (b) In intrinsic semi conductor, the number of holes are
For f.c.c. system, Nc = 8, Nf = 0 and Ni = 4, so n = 4 equal to number of electrons i.e., n p = ne
7. (b) The given characteristics are of ionic bonding. 11. (b) Resistivity of a semiconductor at room temp. is in
8. (b) A solid is not transparent to visible light if the value of between 10–5 Wm to 104 Wm i.e. 10–3 to 106 Wcm
wavelength of light is greater than the bond length 12. (d) The valency of semiconductor (Ge or Si) is four, hence
between the atoms/molecules/ions of material. it has 4 valence electrons in the outermost orbit of the
Conductivity depends upon the no. of free charge Ge or Si-atom
carriers present in the substance at a given temperature 14. (b) The forbidden energy gap for germanium crystal is 0.71
or resistance of that material at that temperature. eV.
9. (c) In simple cubic lattice, volume, V = a3 16. (b) The electrical conductivity of a semiconductor at 0 K is
mass of unit cell A/N A zero. Hence resistivity (= 1/electrical conductivity) is
Density = = = infinity.
volume of unit cell V Na 3 17. (d) The temperature coefficient of resistance of a
11. (c) Vander Waal’s forces make the molecular crystals, solids semiconductor is negative. It means that resistance
at low temperature decrease with increase of temperature.
12. (a) In fcc lattice, the no. of atoms per unit cell = 4 18. (a) The acceptor valence band is close to the valence band
mass of unit cell 4A / N 4A of host crystal
Density = = = 19. (c) The donor valence band lies little below the conduction
3
volume of unit cell a Na 3 band of the host crystal
Eg
-
Exercise 29.2 21. (d) For semiconductor, n = AT 3/ 2
e 2KT ;

so n µ T 3 / 2
2. (d) In extrinsic semi conductor the number of holes are not
Exercise 29.3
equal to number of electrons i.e.,
np ¹ ne 8. (b) Forward bias opposes the potential barrier and if the
np > ne applied voltage is more than knee voltage it cancels the
In p - type
potential barrier.
In n - type ne > np 9. (a) Leakage current is the name given to the reverse current.
10. (b) In the reverse biasing of p-n junction, the voltage
But over all both p & n - type semi conductor are
applied supports the barrier voltage across junction,
uncharged.
which increases the width of depletion layer and hence
3. (c) By doping, the band gap reduce from 1eV to 0.3 to 0.7
increases its resistance.
eV & electron can achieve this energy (0.3eV to 0.7eV)
11. (c) Zener diode is used as a voltage regulator i.e. for
at room temperature & reach in C.B (conduction band).
stabilization purposes.
5. (c) Electric conduction, in a semi conductors occurs due to
13. (a) filter circuit eliminates a.c. component of rectified voltage
both electrons & holes.
obtained from p-n junction as a rectifier.
Free eBooks on @neetquestionpaper2020

1024 Physi cs
14. (b) The output r.m.s. voltage in a half wave rectifier circuit
Exercise 29.5
is Vrms = V0 / 2 = 200 / 2 = 100 V
15. (d) The input signal should be connected between two
1. (b) When A = 1, then A + A = 1 + 0 = 1
points of bridge rectifier such that in positive half wave
of input signal, one p-n junction should be forward and when A = 0, then A + A = 0 + 1 = 1
biased and other should be reverse biased and in
2. (a) When A = 1, then A.A = 1 .0 = 0
negative half wave of input signal, the reverse should
take place. It will be so when input is connected between and when A = 0, then A.A = 0.1 = 0
B and D. 3. (d) For this truth table, Boolean expression is
16. (a) The average value of output direct current in a half
Y = A.B + A.B which is for XOR gate
wave rectifier is = (average value of current over a cycle)/ 4. (b) For this truth table, Boolean expression is Y = A.B which
2 = (2 I0/p)/2 = I0/p is for AND gate
17. (d) The average value of output direct current in a full wave
rectifier = average value of current over a cycle = 2 I0/p 5. (c) For this truth table, Boolean expression is Y = A.B
18. (c) When no current flows at the junction plane, then contact which is for NAND gate
potential of junction plane is equal to the forward
Exercise 1 : NCERT Based Questions
voltage applied = 0.045 V
19. (d) Current in junction diode, I = I 0 (e eV / kT - 1) 7. The motion of charge carriers due to the electric field at the
junction is called drift current.
In forward biasing, V is positive ; In reverse bias V
9. When an external voltage is applied across the diode such
is negative. Then Ir = I0
that n – side is positive and p – side is negative.
I F I 0 (e eV / kT - 1) 10. After which reverse current increases sharply.
= = (e eV / kT - 1)
Ir I0 12. 125
20. (b) The r.m.s. value of a.c. component of wave is more than a
13. b=
d.c. value due to barrier voltage of p-n junction used as 1- a
rectifier. 20. 2.5 V
Exercise 29.4 21. (c) 22. (c) 23. (c) 24. (c)
1. (c) In transistor base is least doped, so that most of electrons 25. (d) 26. (c) 27. (a) 28. (b)
emitted (in case of npn) from emitter reach to collector
& less number of electrons are destroyed due to
recombination with holes in base. Exercise 2 : PAST Competition MCQs

Ic 1. (d) 0V –2V It can be sketched as


2. (d) b = > 1 or Ic > Ib
Ib Immobile ions
Ic
3. (c) a= < 1 or I < I – +
Ie c e
P – + N
4. (d) Operating point of a transistor is zero signal value of I c 0V – +
and VCE. depletion
5. (d) A transistor is a current operating device in which the layer –2V
emitter current controls the collector current 2. (b) [Hint : For easy flow of current the P side must be
6. (c) An amplifier can be both a single stage and multistage connected to +ive terminal of battery i.e., it is connected
7. (d) In common emitter circuit, the current gain is highest to higher potential in comparison to N. This connection
10. (b) The power amplifier handles large power is called forward biased. In this case the input resistance
11. (c) If K is a gain of one stage, then total gain of n stages = is very low.
(K)n = 103 = 1000 In reverse-biased, the P-side is connected to –ive
12. (c) The size (or length) of collector is large in comparison terminal & N side to (+ive) terminal to battery. In this
to emitter (base is very small in comarison to both case input resistance is very high.
collector & emitter) to dissipate the heat. n
14. (a) Negative feed back to a transistor increases stability in dow I
k e forward
the working of transistor. ea ag
Br volt biased
15. (a) In class A amplifier, current through load flows for full VB
cycle of the signal. V
16. (a) A tuned amplifier has a tuned L-C circuit as collector VT
reverse
load. biased
Free eBooks on @neetquestionpaper2020

Semi conductor Electroni cs : Materials, Devi ces an d Simple Circuits 1025


3. (d) In forward biasing, the diode conducts. For ideal 16. (c) Power gain = voltage gain × current gain
junction diode, the forward resistance is zero; therefore,
entire applied voltage occurs across external resistance V0 I0
= VG × IG = ×
R i.e., there occur no potential drop, but potential across Vi Ii
R is V in forward biased.
V02 Ri 100
= × = 50 ´ 50 ´
Ic 2 R 200
= 0.96 Þ Vi 0
4. (c) I c = 0.96 I e
Ie 2500
= = 1250
But I e = I c + I b = 0.96 I e + I b Þ I b = 0.04 I e 2
17. (d) For transistor action, the base region must be very
I 0.96 I e thin and lightly doped. Also, the emitter-base junction
\ Current gain b = c = = 24
I b 0.04I e is forward biased and base-collector junction is reverse
biased.
5. (b) [Hint : Barrier potential depends on, doping density,
18. (a) From the given waveforms, the truth table is as follows.
temperature, forward/reverse bias but does not depend
on diode design.]
6. (c) When the collector is positive and emitter is negative A B Y
w.r.t. base it causes the forward biasing for each junction, 1 1 0
which causes conduction of current. 0 0 1
7. (b) A 0 1 1
Y 1 0 1
B X
The above truth table is for NAND gate.
X = AB Therefore, the logic gate is NAND gate.
\ Y = X = AB 19. (a) The current gain
Y = AB by Demorgan theorem DIC 10mA 10 ´ 103
\ This diagram performs the function of AND gate. b= = = = 50
DIB 200mA 200
8. (c) In reverse biasing, the conduction across the p-n junction
does not take place due to majority carriers, but takes place 20. (d) In forward biasing of the p-n junction, the positive
due to minority carriers if the voltage of external battery is terminal of the battery is connected to p-side and the
large. The size of the depletion region increases thereby negative terminal of the battery is connected to n-
increasing the potential barrier. side. The depletion region becomes thin.
9. (c) In semiconductors, the conduction band is empty and 21. (d)
the valence band is completely filled at 0 K. No electron 22. (c) When small amount of antimony (pentavalent) is added
from valence band can cross over to conduction band to germanium crystal then crystal becomes n-type semi
at 0 K. But at room temperature some electrons in the conductor. Therefore, there will be more free electrons
valence band jump over to the conduction band due to than holes in the semiconductor.
the small forbidden gap, i.e., 1 eV. 23. (d) ni2 = nenh
(1.5 × 1016)2 = ne (4.5 × 1022)
10. (c) V = Vo = 10 V
p p Þ ne = 0.5 × 1010
11. (c) Electromotive force depends upon intensity of light or ne = 5 × 109
falling, it does not depend on frequency of barrier Given nh = 4.5 × 1022
voltage. Þ nh >> ne
12. (d) Positive terminal is at lower potential (0V) and negative \ Semiconductor is p-type and
terminal is at higher potential 5V. ne = 5 × 109 m–3.
13. (c) Semiconductors have –ve temperature coefficient of 24. (d) Voltage across zener diode is constant.
resistivity.
14. (b) Majority carriers in an n-type semiconductor are 250W i i1kW
electrons.
5V i–i1kW
15. (b) Integrated circuit can act as a complete electronic 1kW
circuit. 20v
15V
15V
Free eBooks on @neetquestionpaper2020

1026 Physi cs
Current in 1kW resistor,
Vout I out Rout
15volt 29. (a) Voltagegain (AV) = V = I × R
in in in
(i)1kW = = 15 mA
1kW
Current in 250W resistor, 2 ´ 10 –3 4 ´ 103
AV = -6
´ = 2 × 100 = 2000
40 ´ 10 100
(20 - 15)V 5V
(i)250W = = 30. (b) In a n-type semiconductor holes are minority carriers
250W 250W
and pentavalent atoms are dopants.
20 R out
= A = 20 mA 31. (d) Voltage gain DÚ = b
1000 R in
\ (i) zener diode = (20 - 15) = 5mA.
R out
25. (d) Here D1 is in forward bias and D2 is in reverse bias so, Þ G = 25 R ...(i)
in
D1 will conduct and D2 will not conduct. Thus, no
current will flow through DC. b
Transconductance gm =
R in
V 5 1
I= = = Amp. b 25
R 10 2 Þ Rin = =
gm 0.03

26. (d) IC Putting this value of Rin in eqn. (i)


R out
RC O/P = 2 Volt G = 25 × 0.03 ...(ii)
25
Input RB R out
\ G' = 20 × 0.02 ...(iii)
20
From eqs. (ii) and (iii)
2
The output voltage, across the load RC Voltage gain of new transistor G' = G
3
V0 = IC RC = 2
32. (b)
The collector current (IC)

2
IC = 3
= 10-3 Amp
2 ´ 10
i.e., output X = A.B
Current gain (b) ALTERNATE :
IC
(b) current gain = = 100 A B X
IB
0 0 0
IC 10 -3 1 0 0
IB = = = 10 -5 Amp 0 1 0
100 100
1 1 1
Input voltage (Vi)
Vi = RB IB = 1 × 103 × 10–5 = 10–2 Volt 33. (c) Semiconductors are insulators at low temperature
Vi = 10 mV 34. (c) The resistivity of conductor increases with increase in
27. (b) I ® ON temperature. The resistivity of semiconductor decreases
as the temperature increases
II ® OFF
35. (c) Maximum in insulators and overlaping in metals
In IInd state it is used as a amplifier it is active region.
36. (a)
A 0 1 1 0 37. (c) Copper is a conductor so its resistance decreases on
28. (a) B 0 0 1 1
decreasing temperature as thermal agitation decreases
C 1 1 1 1
whereas germanium is semiconductor therefore on
OR gate
decreasing temperature resistance increases.
38. (c)
Free eBooks on @neetquestionpaper2020

Semi conductor Electroni cs : Materials, Devi ces an d Simple Circuits 1027


39. (d) Holes move from base to emmitter
49. (d) Here Y = ( A + B ) = A.B = A × B . Thus it is an AND
40. (d) In common emitter configuration current gain gate for which truth table is
-h f e -50
Ai = = A B Y
1 + h oe R L 1 + 25 ´ 10-6 ´ 1 ´ 103
0 0 0
= – 48.78 0 1 0
Where hfe= forward current ratio 1 0 0
hoe= output admittance. 1 1 1
41. (a) Both the depletion region and barrier height are reduced.
50. (b) We know that a single p-n junction diode connected
42. (d) Band gap = energy of photon of wavelength 2480 nm. to an a-c source acts as a half wave rectifier [Forward
hc biased in one half cycle and reverse biased in the other
So, DE = half cycle].
l
51. (a) The final boolean expression is,
æ 6.63 ´10 - 34 ´ 3 ´108 ö 1 X = ( A . B ) = A + B = A + B Þ OR gate
=ç ÷´ eV = 0.5 eV
ç 2480 ´10 -9 ÷ -19
è ø 1.6 ´10 52. (a)
43. (d) Zero; In common base amplifier circuit, input and A
output voltage are in the same phase. Y2 = A.AB
44. (a) IC = 5.488 mA, Ie = 5.6 mA Y = A.AB B.AB
Y1 = AB
5.488 a
a= ,b= = 49
5. 6 1- a B
Y3 = B.AB
45. (a) The current will flow through RL when the diode is
By expanding this Boolen expression
forward biased.
46. (a) Si and Ge are semiconductors but C is an insulator. Y = A.B + B.A
Also, the conductivity of Si and Ge is more than C Thus the truth table for this expression should be (a).
because the valence electrons of Si, Ge and C lie in 53. (a) For same value of current higher value of voltage is
third, fouth and second orbit respectively. required for higher frequency hence (a) is correct
answer.
47. (N) It is a p-n-p transistor with R as base. None of the
option is correct. Exercise 3 : Conceptual & Applied MCQs
1. (c) Atomic radius for fcc crystal is
48. (d) A
a 3. 6 Å
r= = = 1.27 Å
2 2 2 2
C
2. (d) Atomic radius for bcc structure,
B
a 3 3.6 3
r= = = 1.56 Å
The truth table for the above logic gate is : 4 4
3. (b) Atomic radius for scc structure,
A B C a 3. 5
r= = = 1.75 Å
1 1 1 2 2
1 0 1
-34
0 1 1 hc 6.62 ´ 10 ´ 3 ´108
4. (a) l = = = 10.888 Å
0 0 0 E (1.14 ´1.6 ´10 -19 )

5. (b) Conductivity s = n i em e = 1017 ´ (1.6 ´10 -19 ) ´ 3800


This truth table follows the boolean algebra
= 60.8 mho/cm
C = A + B which is for OR gate
Free eBooks on @neetquestionpaper2020

1028 Physi cs

6. (b) I = nA evd or I µ nvd 18. (c)


19. (a) (1000)2 = 0 × 20 + 0 × 21 + 0 × 22 + 1 × 23 = 8
Ie n v n I v 7 4 7
\ = e e or e = e ´ h = ´ = 20. (b) (9.25)10 = (9)10 + (0.25)10 = (1001)2 + (0.01)2 = (1001.01)2
Ih n h vh n h Ih ve 4 5 5 21. (a) The phase difference between output voltage and input
7. (b) In half wave rectifier, we get the output only in one half signal voltage in common base transistor circuit is zero
cycle of input a.c. therefore, the frequency of the ripple
22. (c) DI C = a DI E = 0.99 ´ 5 = 4.95 m A
of the output is same as that of input a.c. i.e. 50 Hz
8. (d) In full wave rectifier, we get the output for the positive 23. (c) Band gap,
and negative cycle of input a.c. Hence the frequency of
the ripple of the output is twice than that of input a.c. hc (6.6 ´ 10 -34 ) (3 ´ 108 )
Eg = = eV = 0.49 eV
i.e. 100 Hz l 2480 ´ 10 -9 ´ 1.6 ´ 10 -19

3 RL 24
9. (a) [Hint Þ for B.C.C cell r = a 24. (b) Voltage gain, A v = b R = 0.6 ´ 3 = 4.8
4 i
so distance between body centered atom & a corner 25. (b) In half wave rectifier only half of the wave is rectified
3 mR mR
atom is 2r = ´ a ´ 2 = 3.66A º ]
4 26. (b) A n = r + R = (m / g ) + R
p m
DV 0.7 - 0.5
10. (c) Forward resistance = = = 200 W
DI 1.0 ´ 10-3 42 ´ (50 ´10 3 )
= = 29.57
42 /(2 ´10 -3 ) + 50 ´10 3
11. (c) b = a = 0.95 = 0.95 = 19
1 - a 1 - 0.95 0.05 V 0.1
27. (c) E= = - 6 = 105 V / m
DI C d 10
12. (a) b = or DIC = b DIB = 40 ´ 100 m A
DI B 28. (d)
13. (c) IC = IE –IB = 90 – 1 = 89 m A 29. (b)
30. (c) We know that for common base
Ro 5000
14. (c) Voltage amplification A v = b = 60 ´ = 600
i c collector current
Ri 500
a= = = 0.98
ie emiter current
A V 2800
15. (c) Current gain, a = = = 0.93 & for common emitter
A R 3000
16. (d) (100)2 + (1011)2 = (0 × 20 + 0 ×21 + 1×22) i c collector current
b= =
+ (1 × 20 + 1 × 21 + 0 × 22 + 1 × 23) ib base current
= (0 + 0 + 4) + (1 + 2 + 0 + 8) = (15)10
a 0.98 0.98
2 15 b= = = = 49.
1 - a 1 - 0.98 0.02
2 7 –1
2 3 –1 ­ readup 31. (d) Relation between A, B and C shows that C = AB
2 1 –1 the remainders. So NAND Gate
0 –1 or (15)10 = (1111)2 32. (d) During the formation of a junction diode, holes from p-
ALTERNATIVELY : region diffuse into n-region and electrons from n-region
0100 + 1011 = (1111)2 diffuse into p-region. In both cases, when an electrons
(1111)2 = 1× 23 + 1 × 22 + 1 × 21 + 1 × 20 = (15)10 meets a hole, they cancel the effect at each other and as
17. (b) 2 16 a result, a thin layer at the junction becomes free from
2 8 –0 any of charges carriers. This is called depletion layer.
2 4 –0 There is a potential gradient in the depletion layer,
­ read up the remainders
negative on the p-side, and positive on the n-side. The
2 2 –0
potential difference thus developed across the junction
1 –0
is called potential barrier.
0 –1
33. (b) 34. (d) 35. (b)
So (16)10 = (10000)2
Free eBooks on @neetquestionpaper2020

Semi conductor Electroni cs : Materials, Devi ces an d Simple Circuits 1029

1 IC n C 0.96 ´ 1010
36. (a) Conductivity, σ = = e(n eμ e + n h μ h ) a= = = = 0.96
ρ IE n E 1010
ie, 2.13 = 1.6 × 10–19(0.38 + 0.18) ni
(Since in intrinsic semi-conductor, ne = nh= ni) DI p
48. (b) g m = DV
\ density of charge carriers, n i g

2.13 or DIp = gm ´ DVg = 3 ´ 10-4 ´ [-3 - (-1)]


= -19
= 2.37 ´ 1019 m -3 .
1.6 ´ 10 ´ 0.56
37. (a) = –0.6 × 10–3 A = shortage of 0.6 × 10–3 A
38. (a) Minority carriers in a p-type semiconductor are free
J1 AT12 e - W1 / kT1
electrons. 49. (d) =
J 2 AT22e - W2 / kT2
39. (b) Reverse resistance

DV 1 W1 W2
= = = 2 ´ 106 W æ T1
2
ö - kT1 + kT2 æ 1 ö 0 1
2
D I 0.5 ´10-6 = çç ÷ e =ç ÷ e =
÷
è T2 ø è2ø 4
40. (d) NAND is combination of AND gate and NOT gate.
41. (d)
Vs 0.01
50. (d) [Hint Þ i B = = = 1 ´ 10-5 A
DV R in 103
42. (c) Forward bias resistance =
DI
ic
(0.7 - 0.6)V 0.1 Now b of transistor is defined as bac =
= = = 10 W. ib
(15 - 5 ) mA 10 ´10-3
43. (d) Current gain in common emitter mode or i c = 50 ´10 -5 = 500mA ]

a 0.995 0.995 51. (b) [Hint : At 0K (–273ºC) motion of free electron stop i.e.,
= = = = 199. there is no electron in conduction band therefore at 0K
1 - a 1 - 0.995 0.005
intrinsic semiconductor becomes insulator.]
44. (a)
52. (b) Pauli’s exclusion principle.
45. (b) Given : µe = 2.3 m2 V–1 s–1
53. (b)
µh = 0.01 m2 V–1 s–1, ne = 5 × 1012 / cm3
54. (a) 100010 + 11011 = (111101)2
= 5 × 1018/m3 nh = 8 × 1013/cm3 = 8 × 1019/m3.
Conductivity s = e[neµe + nhµh] 55. (c) V' = V + IR = 0.5 + 0.1 × 20 = 2.5 V
= 1.6 × 10–19 [5 × 1018 × 2.3 + 8 × 1019 × 0.01]
0.5V 20W
= 1.6 × 10–1 [11.5 + 0.8]
0.1A
= 1.6 × 10–1 × 12.3 = 1.968 W–1 m–1.

hc 6.62 ´10 -34 ´ 3 ´108


46. (b) l = = = 2.07 ´ 10 -5 m V
-3 -19
E (60 ´ 10 ´ 1.6 ´10 )
47. (c) No. of electrons reaching the collector, 56. (b) Given interatomic spacing = 2r = 2.54 Å
96 4r = 2 a, where a is lattice constant
nC = ´ 1010 = 0.96 ´ 1010
100
2a a
n ´e 2r = =
Emitter current, IE = E 2 2
t

n ´e
Collector current, IC = C
t
\ Current transfer ratio,
a = 2r 2 = (2.54 Å)(1.414) = 3.59 Å
Free eBooks on @neetquestionpaper2020

1030 Physi cs
57. (a) Due to strong electronegativity of carbon. 62. (c) (W + X) . (W + Y) = W + (X . Y)
58. (c)
P N Z´ A
63. (b) d =
N ´a3
For bcc, Z = 2

2A
d=
Na 3
64. (b) Combination of NAND & NOR gates can produce OR,
AND & NOT gates
65. (b) In circuit 2, each p-n junction is forward biased, hence
same current flows giving same potential difference
across p-n junction.
In circuit 3, each p-n junction is reverse biased, and
due to the flow of same leakage current, giving equal
potential difference across p-n junction.
66. (c)
67. (c) Here diode is forward biased with
voltage = 2 – 0 = 2 V.
VB = Vknee+ IR
2 = 0.7 + I× 200
(\ Total resistance = 180 + 20 = 200W)

1.3
\ I= = 6.5mA
Number of donors is more because electrons from – 200
ve terminal of the cell pushes (enters) the n side and
decr ea ses t h e n um ber of un compen sat ed 68. (b) A Y1
pentavalent ion due to which potential barrier is B
reduced. The neutralised pentavalent atom are again
Y
in position to donate electrons.
59. (a) In reverse bias the current through the p-n junction is
almost zero. Y2

60. (d) (A) is a NAND gate so output is 1´ 1 = 1 = 0


Y1 = A + B, Y2 = A . B
(B) is a NOR gate so output is 0 + 1 = 1 = 0

(C) is a NAND gate so output is 0 ´1 = 0 = 1 Y = (A + B)gAB = AgA + AgB + BgA + BgB

(D) is a XOR gate so output is 0 Å 0 = 0 = 0 + AgB + BgA + 0 = AgB + BgA


This expression is for XOR
0 69. (c) 70. (d)
1
1 hc
71. (c) E=
l
Following is NAND Gate Y = AB
hc 6.63 ´ 10 -34 ´ 3 ´ 108
61. (b) Output of upper AND gate = AB Þl= = = 16500Å
E 0.75 ´ 1.6 ´ 10 -19
Output of lower AND gate = AB
72. (d) 73. (d) 74. (b)
\ Output of OR gate, Y = A B + BA 75. (b) 76. (d)
This is boolean expression for XOR gate.
Free eBooks on @neetquestionpaper2020

Semi conductor Electroni cs : Materials, Devi ces an d Simple Circuits 1031


77. (a) Metal oxides with high temperature coefficient of 84. (d) A and C are analogue but B is digital signal.
resistivity.
78. (b) I e n e eAv e 7 7 v
85. (c) = Þ = ´ e
I h n h eAv h 4 5 vh
2 13 1
2 6 0 ve 5
Þ =
2 3 1 vh 4
1 86. (b) D2 is forward biased whereas D1 is reversed biased.
So effective resistance of the circuit
The binary representation of 13 is 1101. The 1's
complement of 13 is 0010. The 2's complement of 13 is R = 4 + 2 = 6W
0010 + 1 = 0011.
12
\i = =2A
D IC 6
79. (b) Current gain = when VCE is constant.
DI B 87. (a) These gates are called digital building blocks because
using these gates only (either NAND or NOR) we can
2.5 ´ 10 -3 compile all other gates also (like OR, AND, NOT, XOR)
= = 0.1´ 103 = 100
-6
25 ´ 10 88. (b) Study of junction diode characteristics shows that
[DIB = 125 µA – 100 µA = 25 µA the junction diode offers a low resistance path, when
DIC = 7.5 mA – 5 mA = 2.5 mA] forward biased and high resistance path when reverse
80. (d) The given truth table is for NAND gate. biased. This feature of the junction diode enables it to
81. (c) be used as a rectifier.
89. (a) A NOT gate puts the input condition in the opposite
A
A Y' order, means for high input it give low output and for
Y
B low input it give high output. For this reason NOT
B gate is known as invertor circuit.

Y ' = A + B. Y = A + B = A + B. 90. (c) Statement - 1 is True, Statement- 2 is False


91. (a) In semiconductors, by increasing temperature,
Truth table of the given circuit is given by
covalent bond breaks and conduction hole and
A B Y' Y electrons increase.
0 0 1 0 92. (a) Input impedance of common emitter configuration.
0 1 0 1
DVBE
1 0 0 1 =
DiB V
1 1 0 1 CE = constant
where DVBE = voltage across base and emitter (base
Vo R 5 ´103 ´ 62
82. (b) = o ´b = = 10 ´ 62 = 620 emitter region is forward biased)
Vin R in 500
DiB = base current which is order of few microampere.
Vo = 620 × Vin= 620 × 0.01 = 6.2 V 93. (a) In semiconductors the energy gap between conduction
\ Vo = 6.2 volt. band and valence band is small (»1 eV). Due to
83. (b) From input signals, we have, temperature rise, electron in the valence band gain
thermal energy and may jumpy across the small energy
A B Output NAND gate
gap, (to the conduction band). Thus conductivity
0 0 1 increases and hence resistance decreases.
1 0 1
0 0 1 DVp (180 - 120)
94. (b) rp = = -3
= 1.2 ´ 10-4 ohm
1 1 0 DIp (15 - 10) ´ 10
0 0 1 DI p (15 - 7) ´ 10-3
95. (a) gm = = = 5.33 ´ 10-3 ohm -1
The output signal is shown at B. DVg (-2.0) - (-3.5)

96. (a) m = rp ´ g m = (1.2 ´ 104 ) ´ (5.33 ´ 10 -3 ) = 64


Free eBooks on @neetquestionpaper2020

1032 Physi cs
97. (b) The probability that a state with energy E is occupied 98. (a) The energy of the donor state, relative to the top of
is given by the valence bond, is 1.11 eV – 0.15 eV = 0.96 eV. The
1 Fermi energy is 1.11 eV – 0.11 eV = 1.00 eV. Hence,
P(E) = (E - E ) / K , where EF is the Fermi
e F T +1 (E - E F ) / kT = (0.96eV - 1.00eV)
energy, T is the temperature on the Kelvin scale, and
K is the Boltzmann constant. If energies are measured /(0.02586eV) = -1.547
from the top of the valence band, then the energy
associated with a state at the bottom of the conduction 1
and P(E) = = 0.824
band is E = 1.11 eV. Furthermore, KT = (8.62 × 10 –5 e - 1.547
+1
eV/K) (300K) = 0.02586 eV. For pure silicon, EF = 0.555
eV and (E – EF)/kT = (0.555eV) / (0.02586eV) = 21.46. -3 æ 0.7 ö
99. (b) VB = I (R D + R) Þ 5 = 1 ´ 10 çè -3 + R ÷ø
Thus, 10
Þ R = 4.3 kW
1
P(E) = = 4.79 ´ 10 - 10
21.46 æ 0.7 ö
e +1 100. (d) VB = I (R D + R) Þ 5 = 5 ´ 10-3 ç + R÷
è 5mA ø
For the doped semi-conductor,
Þ 1 kW = 180W + R
(E – EF) / kT = (0.11 eV) / (0.02586 eV) = 4.254
Þ R = 860W
1 -2 101. (b) Power = VI = (6 – 0.7) × 5 × 10–3 W = 26.4 mW
and P(E ) = = 1.40 ´ 10 .
e 4.254 + 1
Free eBooks on @neetquestionpaper2020

30
Communication
Systems
COMMUNICATION SYSTEM Block diagram of high level broadcast transmitter :
The terms communication signifies Transmitting
(i) transmission Antenna
(ii) reception RF Voltage
(iii) processing of information by electrical means. Master RF buffer RF O/P
amplifier & power
Oscillator amplifier Power amplifier
Some examples of communication system are:
(i) Telephony and telegraphy
(ii) Mobile communication Modulation Modulation Modulation Modulation
frequency frequency power
(iii) TV-broadcasting process amplifier
signal voltage amp
(iv) Radar
(v) Radio aids to navigation and aircraft. Function of each block :
The basic constituents of communication system are: (i) Transducer : converts the information into electrical signal.
(a) Collation (sorting), processing and storage of information (ii) Modulation processing stage: compressed and limit the
(b) Actual transmission of information audio frequency in amplitude range.
(c) Reception of information involving decoding, storage and (iii) Modulation frequency voltage and Power amplifier : for
interpretation.
amplification purpose.
Basic block diagram of a communication system :
(iv) Master oscillator : generates stable carrier voltage of desired
Encoding Decoding frequency.
modulation modulation
(v) RF buffer amplifier : isolate master oscillator from influence
Information Transmitter Channel Receiver of modulation done at later stage.
distortion Destination
source (distortion) distortion
(vi) RF voltage and power amplifier : Carrier gets amplitude
modulated
Noise (vii) Transmitting antenna: radiate information in form of e.m.
source
waves.
• Information source: A communication system serves to • Channel and noise source: Channel in the medium through
communicate a message or information. Messages are in which message travels from the transmitter to the receiver.
form of words, group of words, code, symbols etc. Out of Noise source may get added to signal at any point in the
these messages, only the desired message known as commnication system.
information is selected and conveyed in form of bits or
• Receiver: They may be of widely different varieties. The
digits.
simplest radio receiver is the crystal receiver having one
• Transmitter : A high or low level broadcast transmitter can
be take in use as per the requirement. semiconductor crystal and head phone.
Free eBooks on @neetquestionpaper2020

1034 Physi cs
Block diagram of superheterodyne receiver : 2. Signal is the information converted in electrical form and
Receving suitable for transmission.
antenna
(a) Analog signal : It is the continuous variations of
Loudspeakers
Freq. IF Audio freq voltage or current.
RF detector voltage&
mixer AMP
amplifiers power Amp (b) Digital signal : It can take only discrete stepwise
values. 'O' correspond to a low level of voltage or
Local
oscillator current and 1 correspond to a high level of voltage or
current.
The basic function of each block :
3. Attenuation : It is the loss of strength of a signal while
(i) RF amplifier : amplifies weak RF signal
propagating through a medium.
(ii) Local oscillator : generates frequency to get IF at later
stage. Example 1.
(iii) Mixer: mixes RF signal with local oscillator–o/p. thereby Calculate the linear speed of communication satellite,
producing IF given that height of satellite above the earth's surface is
(iv) IF amplifier : amplifies IF signal. 36000 km, radius of earth is 6400 km, period of revolution
(v) Detector : demodulates original signal of satellite is 24 hours.
Solution :
(vi) Audio frequency voltage & power amplifier : amplifies
Radius of orbit of satellite
signals
r = R + h = (6400 + 36000) km = 4.24 × 107 m
(vii) Loudspeaker: reproduce original sound signal.
Period of revolution T = 24 hours = 24 × 60 × 60 s
Keep in Memory = 86400 s
If linear speed of satellite is v then
1. There are two basic modes of communication :
(i) Point to point mode : It is when single receiver 2pr 2pr
T= Þv =
correspond to a single transmitter e.g. telephony. v T
(ii) Broadcast mode : It is when number of receiver 2 ´ 3.14 ´ 4.24 ´ 107
\ v=
correspond to a single transmitter e.g. radio and 86400
television. = 3.1 × 103 m/s = 3.1 km/sec.

30.1
Solve following problems with the help of above text and 5. Buffer amplifier is used at the transmitting end to
examples : (a) feed carrier frequency to master oscillator
1. The science of communication involving long distances (b) amplify carrier frequency
is known as (c) mix modulating signal with carrier frequency
(a) mass-communication
(d) isolate master oscillator from other stages of
(b) distant-communication
(c) tele- communication transmitter.
(d) trans-communication 6. Loudspeaker is a transducer used at
2. The part of the total message that is conveyed during (a) receiving end (b) transmitting end
communication is known as (c) both (a) & (b) (d) neither (a) nor (b)
(a) text (b) electric signal
7. The purpose of a detector at the receiving end is
(c) bytes (d) information.
3. A transducer used at the transmitting end, serves the (a) to amplify signal
purpose of converting (b) to reduce its frequency level
(a) electrical signal to sound form (c) to modulate signal
(b) sound signal to electrical form (d) to demodulate signal
(c) electrical signal to magnetic form
(d) sound signal to magnetic form 8. For transmission of TV- signal, sound-part is
4. During the process of transmission & reception the signal (a) amplitude modulated (b) frequency modulated
gets deteriorated due to (c) phase modulated (d) pulse modulated
(a) noise introduced in the system 9. Reception of information involves
(b) distortion in the system
(a) decoding of signal (b) storage of signal
(c) both (a) & (b)
(d) neigher (a) nor (b) (c) interpretation of signal (d) all of the above
Free eBooks on @neetquestionpaper2020

Communication Systems 1035

10. Collation is a process of (c) phase modulated


(a) sorting at transmitting end (d) pulse modulated
(b) decoding at receiving end 13. The term channel is used to indicate
(c) encoding at transmitting end (a) the amplitude range allocated to a given source
(d) storing at receiving end (b) the frequency range allocated to a given source
11. Encoding of signal is required for (c) the voltage-range allocated to a given source
(a) modulation at transmitting end (d) all of the above
(b) modulation at receiving end 14. For transmission of speeches, talks, music, dramas etc----
(c) demodulation at receiving end ---- is used
(d) demodulation at transmitting end (a) radio broadcast transmitter
12. Picture signal of TV-signal is (b) radio telegraph transmitter
(a) amplitude modulated (c) navigation transmitter
(b) frequency modulated (d) None of these

ANSWER KEY
1. (c) 2. (d) 3. (b) 4. (c) 5. (d) 6. (a) 7. (d) 8. (b) 9. (d) 10. (a) 11. (a) 12. (a)
13. (b) 14. (a)

PROPAGATION OF ELECTROMAGNETIC WAVES IN Surface Wave Propagation


ATMOSPHERE In this mode, the radio waves are directed by the earth and they
• Radio waves also known as electromagnetic waves which travel along the curved surface of earth from the transmitter to
when radiated from transmitting antenna, travel through the receiver.
space to distant places where they are picked up by • Surface wave propagations are of two types.
receiving antenna . (i) Short – distance surface wave propagation
• These waves are unaffected by the atmospheric pressure,
(ii) Long – distance surface wave propagation.
rain, snow etc.
• When the distance between the transmitting and receiving
• They can cross the non-metallic objects but are highly
attenuated by the metals. antenna is short the field strength for surface wave
• These waves travel with a speed of light 3 × 108 m/s in E
vacuum. propagation for a flat surface is E S = O A ...(1)
• They do not require any material medium for their d
propagation. EO : the unit distance field strength at the surface of earth.
• On the basis of different behaviour of atmosphere of radio d : the distance from transmitting to receiving antenna
waves of different wavelengths, there are the following three A : the factor taking into account loses caused by the
modes of propagation of radio waves. earth.
(i) Ground or suface wave propagation • The above equation (1) is applicable for distance upto
(ii) Space or tropospheric wave propagation
(iii) Sky or ionospheric wave propagation. 1003 f km (f = frequency in MHz).
Different layers of atmosphere and their interaction with the
propagating electromagnetic waves • · Dis tance > 100 3 f km
Distance
equation (1) has to be modified to take into account
curvature of earth.
• For long distance propagation reduction in field strength is
due to curvature of earth.
• Surface wave propagation is marked as wave A in
figure (1).
Keep in Memory
Surface wave propagation is possible for frequencies upto 500
kHz, that can propagate upto a distance of 1500 km because
attenuation of ground waves increases with increase in frequency
of waves.
For local region propagation upto 15 MHz frequency is possible.
Free eBooks on @neetquestionpaper2020

1036 Physi cs
Space Wave Propagation Substituting Î0 = 8.854 × 10–12 farad/m
Also known as tropospheric propagation find its wide scope at e = 1.6 × 10 –19 coulomb
frequencies like V.H.F, U.H.F and microwaves. m = 9.1 × 10 –31 kg
• In this mode, the radio waves are transmitted from the
w = 2pf
transmitting antenna at a small angle with the earths surface,
which reaches the receiving antenna in following three ways: æ 80.56N ö æ 81N ö
(i) Some part of the transmitted waves get reflected by Then Î=Î0 ç1 - ÷ »Î0 ç1 - 2 ÷
è f2 ø è f ø
troposphere and reach the receiving antenna (shown
as wave B in fig (1) . The refractive index of ionised medium
(ii) Some part of the radio waves reaches directly to the c 1 m0 Î0 Ne 2
receiving antenna (shown as wave C in fig (1) h= = = 1-
vp æ Ne 2 ö Î0 mw2
(iii) Some part reaches after reflection from the earths 1 m0 Î0 ç 1 - ÷
surface (shown as wave D in fig (1)) ç Î mw2 ÷
è 0 ø
For n < 1 the ionised medium behaves as raw medium, so waves
Ionosphere entering the medium deviates away from straight path.
E E
If fi is angle of incidence, fr angle of refraction, then
Sky wave
Troposphere
B sin fi 1 - Ne 2 1 - 81N
B =n = = .
Space wave
sin fr Î0 mw 2
f2
C
D 1 - 81N
D For T.I.R fr = 90° ® h = sin fi =
////////////////// f2
//// ///
///
A (Ground wave) 81N
For fi = 0° sin fi = 0 = 1 - or fc = 9 N
Fig (1) : Modes of Propagation of radio waves. f2
When e.m. wave passes through the ionosphere, the electric This frequency fc is the critical frequency of e.m. wave which is
– component of waves exerts a force on the electrons and max-frequency for which the wave gets reflected back.
thus electrons starts oscillating along path parallel to electric Keep in Memory
field of wave.
Let electric field across m 3 of space in ionosphere 1. During night F1 and F2 layers merge to form one single
be E = Em sinwt layer-F2 due to this, propagation of radio waves gets
Force on each e– is F = – qe newton affected/varies from day to night.
Instantaneous velocity of e– is 2. As the angle of incidence fi is reduced, the distance from
transmitting to receiving point decreases to a minimum and
æ q ö then again increase. The minimum distance from the
v= ç ÷ E m cos wt m / s
è mw ø transmitter at which sky wave of given frequency is returned
\ Instantaneous electric current by N electrons is to earth by ionosphere is known as skip-distance.
ie = – qN v and capacitive current is i c 3. If h is the height of antenna then the distance covered by
transmission d = 2Rh where R = 6.4 × 106 m is the radius
é æ Nq 2 ö ù of the earth.
Total curent i = ie + ic = w êÎ0 -ç ÷ ú E cos wt
ê ç mw2 ÷ ú m
ë è øû
If the transmitting and receiving antennas are placed at heights
Nq 2 Ht and Hr , the maximum range of space wave communication is
The effective value of dielectric constant Î=Î0 -
mw 2
d = 1.23 éë H t + H r ùû ....(2)
é Nq 2 ù
or Î = Î0 ê1 - ú
êë Î0 mw2 úû

N OT E:

(i) ie lags behind electric field by 90°


ic leads electric field by 90°.
(ii) The reduction in the value of Î causes bending of
path of radio waves. Fig (2) : Direct and ground reflected wave of space wave.
Free eBooks on @neetquestionpaper2020

Communication Systems 1037


For calculation of distance between the transmitting and receiving Solution :
antenna, equation (2) is further modified as fc = 2.8 MHz
4
d = ´ 1.23 é H t + H r ù miles ....(3) f = 4.0 MHz
3 ë û
h = 120 km
considering that the space wave propagate along straight path
and the earths effective radius is 4/3 times the actual radius. 2
f2 æ 4.0 ö
Sky-Wave Propagation Dskip = 2h - 1 = 2 ´ 120 ´ ç - 1 = 249.8 km
2
fc è 2.8 ÷ø
Also known as ionospheric propagation are used for long-distance
radio communication. Example 4.
• In this type of propagation, the energy content of the wave Is it necessary for a transmitting antenna to be at the same
reaches the receiving antenna as a result of bending of height as that of the receiving antenna for line-of-sight
path due to ionization in the upper atmospheric layer communication? A TV transmitting antenna is 81m tall.
(ionosphere). How much service area can it cover if the receiving antenna
• The ionization in the ionosphere is not uniform but it is is at the ground level ?
stratified, i.e. in the form of layers. The layers are designed Solution :
as D, E, F and F2 layer.
No. Service area will be A = pd 2T
Example 2.
The electron density of a layer of ionosphere at a height 22
= ´ 162 ´ 6.4 ´ 106 = 3258 km 2
150 km from the earth surface is 9 ×1010 per m3. For 7
the sky wave transmission from this layer upto a range of Example 5.
250 km, find A T.V. tower has a height of 70m. Calculate the population
(i) critical frequency of the layer covered by T.V. broadcast if the population density around
(ii) the maximum useable frequency the tower is 1000/km2. Radius of earth is 6.4 × 106 m.
(iii) angle of incidence of this layer Solution :
Solution : Height of antenna h = 70m, radius of earth = 6.4 × 106 m.
h = 150 km = 150 × 103 m Population density = 1000 km–2
Nm = 9 × 1010 per m3 = 1000 × 10–6 /m2 = 10–3/m2
D = 250 km = 250 × 103 m The distance covered by the transmission is
(i) Critical frequency of layer
d = 2Rh or d2 = 2Rh
fc = 9 N m = 9 ´ 9 ´ 1010 = 2.7 ´ 106 Hz. Area covered = pd2
(ii) Maximum useable frequency Population covered = pd2 × Population density
= 3.14 × 2Th × 10–3
2
D2 æ 250 ´ 103 ö = 3.14 × 2 × 6.4 × 106 × 700 × 10–3
6
f = fc 1 + = 2.7 ´ 10 ´ 1 + ç
3÷ = 28.16 × 105 = 28.16 lakh.
4h 2 è 4 ´ 150 ´ 10 ø
Example 6.
= 3.17 × 106 Hz. Electromagnetic waves with frequencies greater than
(iii) If angle of incidence at this layer is fi, then from the critical frequency of ionosphere cannot be used for
f = fc sec fi. communication using sky wave propagation because
(a) the refractive index of the ionosphere becomes very
f 3.17 ´ 106
sec fi = = = 1.174 high for f > fc
fc 2.7 ´ 106 (b) the refractive index of the ionosphere becomes very
-1
Þ fi = sec (1.174) = 31.6° low for f > fc
Example 3. (c) the refractive index of the ionosphere very high for
f < fc
The height of E layer atmosphere above the earth's surface
(d) None of these
is 120 km and the critical frequency of this layer is
Solution : (a)
28 MHz. Calculate the maximum distance between the
For total internal reflection of sky waves, the refractive
transmitter and receiver for the monochromatic
index of the ionosphere should be less. If f > fc, the refractive
transmission of e.m.wave of frequency 4.0 MHz on the
index will increase. By this waves will just pass without
surface of earth.
reflection taking place.
Free eBooks on @neetquestionpaper2020

1038 Physi cs

30.2
Solve following problems with the help of above text and 8. Long range propagation is not possible by space wave
examples : propagation because
1. Electromagnetic waves of audible frequency ranges from (a) height of troposhere is quite small
(a) 10 Hz to 10,000 Hz (b) 20 Hz to 20,000 Hz (b) height of troposhere is large
(c) 30 Hz to 30,000 Hz (d) None of these. (c) troposphere absorbs transmitted wave
2. E.m.wave of audible frequency cannot be directly (d) None of these.
propagated over a long distance because 9. The wave reaching the receiving antenna will have
(a) they have vary small energy content maximum intensity if the direct wave & wave after reflection
from earth's surface
(b) the length of antenna required for transmission of
(a) are out of phase by 90°
these wave is too large
(b) are out of phase by 270°
(c) both ( a) and (b)
(c) are in phase
(d) neither (a) nor (b)
(d) None of these.
3. For transmission of e.m.wave of audible frequency, these
10. In an ionized medium, the phase velocity vp, group velocity
waves are superimposed with waves of
vg and the speed of light are related as
(a) frequency less than 20 Hz
(a) vp > vg > c (b) vp = vg = c
(b) frequency less than 10 KHz. (c) vp < vg < c (d) vp > c, vg< c .
(c) frequency in the audible range. 11. For a radio wave reaching the ionised medium
(d) radio-frequency. (a) will bend away from normal
4. Wave obtained on superimposition of audible frequency (b) will bend towards normal
e.m. wave is known as (c) will bend follow a straight-path
(a) carrier wave (b) high frequency wave (d) None of these.
(c) modulating wave (d) modulated wave 12. Skip distance does not depend on
5. Earth's atmosphere is transparent for (a) the frequency of signal transmitted
(a) only the visible light (b) electron density of the layer
(b) only the X-rays (c) they get absorbed by troposphere
(c) only the g- rays (d) they get reflected by stratosphere.
(d) visible light and microwaves. 13. Sky wave propagation is not possible for waves of
6. Ground wave propagation is possible for frequency > 30 MHz because
(a) low radio frequency over a short range (a) these waves do not have much energy to reach
(b) high radio frequency over a short range ionosphere
(c) high radio frequency over a long range (b) they are not reflected by ionosphere
(d) low radio frequency over a short range. (c) they get absorbed by troposphere
7. During ground wave propagation the transmitted waves (d) they get reflected by stratosphere
gets attenuated because 14. Long range transmission of TV-signal is done by
(a) earth surface absorbs the waves (a) space-wave (b) sky waves
(b) frequency of the waves are too low (c) ground wave (d) artificial satellite.
(c) energy content of these waves are high
(d) earth surface offers resistance.
ANSWER KEY
1. (b) 2. (c) 3. (d) 4. (d) 5. (d) 6. (a) 7. (d) 8. .(a) 9. (c) 10. (d) 11. (a)
12. (d) 13. (b) 14. (d)

MODULATION AND ITS NEEDS During the process of modulation : (i) the audio frequency wave
For long range transmission of audio frequency e.m.waves, that needs to be transmitted is known as modulating wave, (ii) the
before transmission are superposed with radio frequency waves. radio frequency wave superimposed on it is known as carrier
The process of superposition of audio frequency waves with wave and (iii) the resultant wave obtained due to superimposition
radio frequency waves is called modulation. is called modulated wave.
Free eBooks on @neetquestionpaper2020

Communication Systems 1039


Modulation of audio frequency waves are done because of the Depth of modulation
following two reasons :
(i) The energy content of these waves is quite low and é Max.variation in amplitude of carrier ù
ê from its unmodulated value úú
therefore due to attenuation while travelling over a long
ma = ê ´100%
distance their amplitude is greatly reduced. êë Amp.of unmodulated carier wave úû
(ii) The length of antenna required for the transmission must
be of order of the wavelength of waves to be transmitted is Greater the value of ma, more clear and intense is the superposition
very larger around 104 m, which is practically not possible. of modulating wave over the carrier.
1
(i) ma = Þ 50% modulation,
Modulation is Done in Two Ways 2
1
(i) By changing amplitude of carrier wave then KaEm = E c , i.e. max-variation of amplitude of carrier
2
(ii) By changing total phase of carrier waves wave is half the amplitude of unmodulated carrier wave.
Modulation

Amplitude Angular
modulation modulation

DSB-AM SSB-AM Frequency Phase


modulation modulation (ii) ma =1 Þ 100% modulation then KaEm = Ec , i.e. when the
modulating wave has maximum + ve amplitude, the
N OTE: By the process of modulation, the properties of high amplitude of modulated wave is twice the amplitude of carrier
frequency carrier wave like amplitude, frequency, phase etc are wave.
changed corresponding to the instantaneous value of low
frequency audio waves.
AMPLITUDE MODULATION (AM)
Amplitude modulation is the process in which one modulating
wave of low frequency are superposed with carrier waves of
high frequency such that the amplitude of carrier waves changes
in accordance with the amplitude of modulating waves.
when one modulating wave has maximum-ve amplitude the
amplitude of modulated wave is zero.
(iii) m a >1Þ modulation is greater than 100% then
kaEm > Ec. In this condition amplitude of modulated wave
remains zero in some part of modulating wave, as a
consequence there is distortion which is undesirable.

Let, modulating signal be em = Em coswmt


Frequency spectrum :
carrier signal be ec = Ec cos wct
Since (e) AM = Ec( 1 + ma coswmt) coswct
wm<<wc(angular frequency)
Þ (e)AM = Ec coswct + Ecma coswmt coswct
Modulated signal (e)AM = [Ec + ka Emcoswmt] coswct
m a Ec m E cos(wc – wm )t
= E c cos wc t + cos(wc + wm )t + a c
é Ka E m ù 2 2
= E c ê1 + E coswm t ú cos wc t
ë c û The above expression shows that AM-wave comprises of
following there frequencies :
= Ec[1 +macoswmt]coswct
(i) Carrier wave of frequency wc and amplitude EC
K a Em ma Ec
ma = is called the modulation factor /depth of modulation.
Ec (ii) wave of frequency (wc + wm) and amplitude
2
Free eBooks on @neetquestionpaper2020

1040 Physi cs
ma Ec • To avoid wastage of power carrier voltage which contain
(iii) Wave of frequency (wc – wm) and amplitude maximum power is suppressed while modulation process
2
(iv) Frequency ( wc + wm) is the upper side band frequency. as it does not contain any desired information. Similarly
one of the side bands can also be suppressed so as to
(v) Frequency (wc – wm) is the lower side band frequency.
avoid wastage of power because each side- band carry the
AM-Frequency spectrum same desired information.
Ec
FREQUENCY MODULATION (FM)
Relative amp

maEc
2 maEc In this type of modulation the frequency of the carrier voltage is
2 varied in accordance with the instantaneous value of the
modulating voltage.
wm wc – wm wc wc+ wm Let modulating signal em = Em coswmt
w
Carrier signal ec = Ec sin (wc t + q)
CAUTION : \ Modulated signal
Please take into consideration that the audio signal is inherent é K f Em ù
only in the side frequency spectrum and it does not lie in the (e)FM = E c sin êwc t + sin wm t ú
ë wm û
carrier frequency component.
or (e)FM= Ec sin (wc t + mf sinwm t).
Methods of AM
mf is known as the modulation index/depth of modulation
AM can be done by the following two methods :
(i) Linear modulation : This method uses linear portion of The ratio of frequency deviation to modulation frequency is
characteristics of tube or transistor. known as d.
(ii) Square law modulation : This method uses non-linear k f Em
portion of characteristics of tube/transistor. d = mf = .
wm
Keep in Memory
The total variation in frequency from the lowest to the highest is
1. Carrier wave- frequency wc, amplitude – Ec, r.m.s. value called carrier swing (CS)
CS = 2 × frequency deviation
Ec 2 Þ CS = 2 × d
2. Lower side wave frequency - frequency : wc – wm, amplitude
Keep in Memory
ma Ec m E
– , r.m.s. value a c 1. For commercial FM- transmission, maximum frequency
2 2 2 deviation allowed is 75 kHz
3. Upper side wave frequency-frequency :
2. FM channel width is 2 × 75 = 150 kHz.
m E m E 3. In FM, the highest audio frequency transmitted is 15kHz
wc +wm, amplitude – a c , r.m.s. value, a c
2 2 2 4. Percentage modulation of FM is the ratio of actual frequency
2 deviation to maximum allowed frequency-deviation.
E
Power consumed by carrier wave : Pc = c
i.e. m% = d actual For 100% modulation dactual = d max.
4.
2R
d max
5. Power consumed by lower frequency wave :
Merits and Demerits of FM
ma 2 E c 2 m a 2
PLSF= = Pc Merits of FM
8R 4
1. There is a large decrease in noise
6. Power consumed by upper frequency wave : 2. S/N ratio is appreciable
ma 2 E c 2 m a 2 3. In FM, frequency allocation allows for a gaurd-band which
PUSF = = Pc helps in reducing adjacent channel interference
8R 4
4. FM permits use of several independent transmitters in the
7. Total power consumed by side-frequency same frequency.
ma 2 5. The amplitude of FM-modulated wave remains unaffected.
PS F = PLSF + PUSF = Pc . Demerits of FM
2
1. Transmitting and receiving equipements are complex &
costly
N OT E: • If ma = 0 then PSF = 0 2. Reception using conventional method is limited to line of
ma = 0.5 then PSF = 0.11P sight.
ma = 1.0 then PSF = 0.33 P 3. A much wider channel of 200 kHz is required.
Free eBooks on @neetquestionpaper2020

Communication Systems 1041


Application of FM- Transmission Application of SSB-System
1. FM broadcast band 88- 108 MHz with 200 kHz channels in 1. Police wireless communication
which commercial FM station broadcasts audio signals. 2. SSB telegraph system
2. Sound signal in TV-broadcast is freuency modulator. 3. Point to point radio telephone communication
3. FM is used in mobile & emergency services in the frequency 4. V.H.F. and V.H.F. communication.
range 20-4000 Hz. Generation of SSB-system
4. FM is also used in amateur band where only voice 1. Filter method
frequencies are transmitted. 2. Phase concellation method
FM-Side Band
SIDE-BAND AND BANDWIDTH
The frequency components of FM wave are :
In actual practice the modulating signal (i.e. audio signal is not
wo/f o
perfectly sinusoidal, but consists of various nearly freuency
wo± wm / f0 ± fm
signal, on either side of carrier wave, use a pair of side frequency
wo ± 2wm / fo ± 2fm
(wc – wm) and (wc + wm). These groups of side frequencies are
called side bands. wo ± 3wm / fo ± 3fm and so on.
• The group on higher frequency side of the carrier is called • Bandwidth occupied by spectrum; BW = 2n fm.
higher sideband. n is the highest order of the significant side band.
• The group on lower frequency side of the carrier is called • BW occupied by spectrum = 2(mf + 1)fm = 2(Df + fm)
lower side band. The above expression requires that mf > 6 i.e. the side bands of
AM-Side bands amplitudes < 5% of unmodulated carrier is negligible.
• Lower frequency band of AM signal = (wc – wm) FM-Frequency spectrum
• Higher frequency band of AM signal = (wc + wm)
• (Angular bandwidth)AM
= (wc + wm) – (wc – wm)= (2wm)max.
• (Bandwidth)AM or channel width

æ 2w ö
=ç m÷ = ( 2 f m )max fo –3fm fo –2fm fo –fm fo fo+fm fo+2fm fo+3fm
è 2p ømax
Keep in Memory
Let modulating signal be of 5 kHz
carrier singal of 1 MHz 1. No. of side band of FM signal is proportional to amplitude
\ Side-frequency components are of modulating signal
LSF = 1MHz – 5 kHz = 995 kHz 2. No. of side band of FM signal is proportional to
USF =1MHz + 5 kHz= 1005 kHz 1/frequency of modulating signal
\ Channel width = 1005 – 995 = 10 kHz i.e. pair of sidebands
AM-Side Band Transmission: (i) increase as frequency deviation increase
It is found that it is not necessary to transmitt all the signals one (ii) increase as modulating signal frequency decrease
carrier and two sidebands) to enable the receiver to reconstruct Example 7.
the original signal. A carrier voltage of peak value 10 volt and frequency
Suppression of carrier and sidebands may be done by two 2.0 MHz is used to modulate the audio signal of frequency
process : in range20 Hz to 20 KHz with modulation depth 60%.
(i) DSB-SC (double side-band supressed carrier system) Which frequency components will contain the modulating
(ii) SSB-SC (single sideband supressed carrier system) signal? What will be the amplitude of these components?
Former supresses only the carrier while later supresses one of What will be the channel width required for transmission
the sideband and the carrier. of the AM-wave ?
Merits of SSB - SC : Solution :
1. Bandwidth requirement is reduced by half so twice as many The modulating signal will be present in the components
channels can be multiplexed in a given frequency range of band range ® fc + fm
2. The size of power supply required is very small = (2000 + 0.02) kHz = 2000.02 kHz
3. Since no carrier is transmitted so there is the possibility of to (2000 + 20) = 2020 kHz
interference Amplitude of these components
4. SSBsystem eliminates the possibility of distortion. m E 0.6 ´10
5. It provides an improvement to S/N of at least 9 dbs = a c = = 3 volt
2 2
Demerits of SSB -SC : Channel width required = (fc + fm) – (fc – fm)
Transmitter and receiver become more complex.
= 2(fm) = 2 × 20 kHz = 40 kHz.
Free eBooks on @neetquestionpaper2020

1042 Physi cs
Example 8. Solution :
What will be the power of audio signal to get completely (i) Modulating signal amplitude Em = 30
modulated for the given carrier of 50 kW if the efficiency Carrier wave amplitude EC = 60
of modulation is 72%?
Percentage modulation
Solution :
Given Pc = 50 kW Em 30
ma = 1 mv = ´ 100 = ´ 100 = 50%
EC 60
ma 2
Total power in side bands = PSF = - PC (ii) Frequency of audio signal 200 Hz
2
1 Frequency of signal carrier wave = 200000 Hz.
= ´ 50 = 25 kW Example 12.
2
Q Efficiency of modulation is 72% or 0.72 The maximum peak-to-peak voltage of an AM wave is
16 mV and the minimum peak-to-peak voltage is 4mV.
Total power inside bands
Þ = 0.72 Calculate the modulation factor.
Power of audio wave
Solution :
25
Þ Power of audio signal PA = kW = 34.7 kW 16
0.72 Maximum voltage of AM wave, Vmax = = 8mV ,
Example 9. 2
Why does FM give noiseless reception ? 4
Solution : Minimum voltage of AM wave, Vmin = = 2mV
2
Noise is a form of amplitude variation and an FM receiver
Vmax - Vmin 8- 2 6
will reject such signals. ma = = = 0.6
Example 10. Vmax + Vmin = 8 + 2 10
For an amplitude modulated wave, the maximum amplitude Example 13.
is found to be 10V while the minimum amplitude is found A sinusoidal carrier voltage of frequency 1 MHz and
to be 2V. Determine the modulation index, m. What would amplitude 100 volts is amplitude modulated by sinusoidal
be the value of m if the minimum amplitude is zero volt ? voltage of frequency 5 kHz producing 50% modulation.
Solution : Calculate the frequency and amplitude of lower and upper
Since the AM wave is given by (Ac + Am sin wm t) cos wct, sidebands terms.
the maximum amplitude is M1 = Ac + Am while the minimum Solution :
amplitude is M2 = Ac – Am. Frequency of carrier, fc = 1 MHZ = 1000 kHz
Hence the modulation index is Frequency of signal, fs = 5 kHZ
A m M1 - M 2 8 2 Modulation factor, ma = 50% = 0.5
m= = = = Amplitude of carrier, EC = 100 V
Ac M1 + M 2 12 3
The lower and upper sideband frquencies are
With M2 = 0, clearly, m =1, irrespective of M1. fc – fs and fc + fs
Example 11. or (1000 – 5) kHz and (1000 +5) kHz
An audio signal is given by 30 sin (2p × 2000t) is used for or 995 kHz and 1005 kHz respectively
modulating a carrier wave given by the equation Amplitude of each sideband term
60 sin (2p × 200000t) ma E C 0.5 ´ 100
Find (i) percentage modulation, (ii) frequency of signal = = = 25V
2 2

30.3
Solve following problems with the help of above text and (d) actual frequency deviation < maximum allowed
examples : frequency deviation
1. 100% modulation in FM means 2. An audio frequency of 10 kHz is transmitted by SSB after
(a) actual frequency deviation > maximum allowed AM with carrier waves of frequency 1MHz. The frequency
frequency deviation of current in output load is
(b) actual frequency deviation = maximum allowed (a) 1010 kHz or 990 kHz
frequency deviation (b) 1010 MHz or1010 MHz
(c) actual frequency deviation ³ maximum allowed (c) 110 kHz or 990 kHz
frequency deviation (d) 110 MHz or 990 MHz
Free eBooks on @neetquestionpaper2020

Communication Systems 1043

Statement for Q 3 to 7 : (c) 2.505× 106 kHz, 2.495 × 106 kHz


The experssion for an AM voltage is (d) 2.505 MHz, 2.495 kHz
e = 5 (1 + 0.6 cos 1000 p t) cos 5 ×106 p t volt. 8. An AM wave is expressed as e = 10 (1 + 0.6 cos 2000 p t)
3. The peak value of carrier wave is cos 2 × 108 pt volts, the minimum and maximum value of
(a) 30V (b) 6 V modulated carrier wave are respectively.
(c) 3 V (d) 5V
(a) 10 V and 20 V (b) 4V and 8V
4. The percentage of modulation is
(a) 0.6% (b) 6% (c) 16 V and 4V (d) 8 V and 20 V
(c) 60% (d) 66% 9. The ratio of Emax – Emin to Emax + Emin is known as
5. The frequency of audio signal is (a) range of modulating signal
(a) 1000 Hz (b) 5000Hz (b) amplitude variation of modulating signal
(c) 500 Hz (d) 250 Hz (c) depth of modulation
6. Frequency of carrier signal is (d) None of these.
(a) 5 × 106Hz (b) 1000 Hz
10. An AM wave varies from 10V to 4V. Its percentage
3
(c) 2.50× 10 Hz (d) 2.5 × 106 Hz modulation is
7. Frequencies of sideband is (a) 36% (b) 42.8%
(a) 2.50005 × 106 Hz, 2.49995 × 106 Hz
(c) 54% (d) 68 %
(b) 2.505× 106 Hz, 2.495 × 106 Hz
ANSWER KEY
1. (b) 2. (a) 3. (d) 4. (c) 5. (c) 6. (d) 7. (a) 8. (c) 9. (c) 10. (b)
Free eBooks on @neetquestionpaper2020

1044 Physi cs

Very Short/Short Answer Questions 18. Why modulation is necessary? What factors prevent
efficient transmission of signals? How can we overcome
1. Which mode of wave propagation is suitable for television these?
broadcast and satellite communication and why?
19. How is an amplitude modulated wave detected and data
[Outside Delhi - 2012 COMPTT.] retrieved from it?
2. (a) Write two factors which justify the need for modulating
20. A T.V tower has a height of 75 m. What is the maximum
a signal.
distance and area upto which this T.V. transmission can be
(b) Calculate the length of a dipole antenna for a carrier received . Take radius of the earth as 6.4 × 10 6m
wave of frequency 6 × 108 Hz.
[Outside Delhi - 2011 COMPTT.] Multiple Choice Questions
3. What does the term ‘transducer’ mean in an electronic 21. Which mode of communication is most suitable for carrier
communication system? [Delhi Board- 2009 COMPTT.] wave of frequencies around 100 MHz?
4. Write the function of (i) Transducer and (ii) Repeater in the (a) Satellite (b) Ground wave
context of communication system. [Delhi Board- 2009]
(c) Line of sight (d) Ionospheric
5. Write two factors justifying the need of modulation for
22. Communication on ground is through electromagnetic
transmission of a signal. [Delhi Board- 2009]
waves of wavelength
6. What is the role of transmitter? (a) larger than 600 m
7. What is bandwidth of a signal? (b) between 200 and 600 m
8. What is modulation? (c) between 1 and 5 m
9. Describe space wave propagation? (d) between 10 – 3 and 0.1
10. Define modulation index? 23. Ground waves are polarised
(a) parallel to the earth's surface
11. What is amplification? Why is it done?
(b) normal to the earth's surface
12. Differentiate between an analog and digital signal. (c) at an angle 45° from earth's surface
13. How does an AM signal propagate? (d) in any direction.
14. What is the significance of modulation index? 24. Intensity of electric field obtained at receiver antenna for a
space wave propagation is
Long Answer Questions (a) directly proportional to the perpendicular-distance
from transmitter to antenna
15. (a) Distinguish between sinusoidal and pulse-shaped
(b) inversely proportional to the perpendicular-distance
signals.
from transmitter to antenna
(b) Explain, showing graphically, how a sinusoidal carrier (c) directly proportional to the square perpendicular-
wave is superimposed on a modulating signal to obtain distance from transmitter to antenna
the resultant amplitude modulated (AM) wave.
(d) inversely proportional to the square perpendicular-
[Outside Delhi - 2012 COMPTT.] distance from transmitter to antenna
16. Draw a schematic sketch showing the (i) ground wave, 25. The tank circuit used in a radio transmitter should have
(ii) sky wave and (iii) space wave modes of propagation.
(a) high effective Q
Write the frequency range for each of these modes of
propagation. [Outside Delhi - 2011 COMPTT.] (b) low effective Q
17. State the two main reasons explaining the need of (c) loosely coupled load
modulation for transmission of audio signals. (d) Both (a) and (c)
26. The picture signal in TV-broadcast is modulated in
1 (a) SSB (b) VSB (c) FM (d) DSB
c(t) 0
–1 27. In PCM if the transmission path is very long
0 0.5 1 1.5 2 2.5 3
1 (a) pulse spacing is reduced
m(t) 0 (b) pulse amplitude is increased
–1
0 0.5 1 1.5 2 2.5 3 (c) pulse width is increased
The diagrams, given above, show a carrier wave c(t), that is (d) repeater stations are used.
to be (amplitude) modulated by a modulating signal m(t). 28. Field strength of tropospheric TV signal is proportional to
Draw the general shape of the resulting AM wave. Define 1 1
its ‘modulation index’. [Outside Delhi - 2010 COMPTT.] (a) (b) l (c) (d) l2
l l2
Free eBooks on @neetquestionpaper2020

Communication Systems 1045

1. The process of superimposing signal frequency (i.e., audio 10. Consider telcommunication through optical fibres. Which
wave) on the carrier wave is known as [AIIMS 1987] of the following statements is not true? [AIEEE 2003]
(a) Transmission (b) Reception (a) Optical fibres may have homogeneous core with a
(c) Modulation (d) Detection suitable cladding
2. A laser beam is used for carrying out surgery because it (b) Optical fibres can be of graded refractive index
(a) is highly monochromatic [AIIMS 2003] (c) Optical fibres are subject to electromagnetic
(b) is highly coherent interference from outside
(c) is highly directional (d) Optical fibres have extremely low transmission loss
(d) can be sharply focussed 11. This question has Statement – 1 and Statement – 2. Of the
3. Long distance short-wave radio broadcasting uses four choices given after the statements, choose the one
[AFMC 1996] that best describes the two statements. [AIEEE 2011]
(a) ground wave (b) ionospheric wave Statement – 1 : Sky wave signals are used for long distance
(c) direct wave (d) sky wave radio communication. These signals are in general, less
4. Broadcasting antennas are generally [AFMC 2003] stable than ground wave signals.
(a) omnidirectional type (b) vertical type Statement – 2 : The state of ionosphere varies from hour to
(c) horizontal type (d) None of these hour, day to day and season to season.
5. The attenuation in optical fibre is mainly due to (a) Statement–1 is true, Statement–2 is true, Statement–2
(a) absorption [AFMC 2003] is the correct explanation of Statement–1.
(b) scattering (b) Statement–1 is true, Statement–2 is true, Statement–2
(c) neither absorption nor scattering is not the correct explanation of Statement – 1.
(d) both (a) and (b) (c) Statement – 1 is false, Statement – 2 is true.
6. In frequency modulation [ Kerala PMT 2005] (d) Statement – 1 is true, Statement – 2 is false.
(a) the amplitude of modulated wave varies as frequency 12. A radar has a power of 1kW and is operating at a frequency
of carrier wave of 10 GHz. It is located on a mountain top of height 500 m.
(b) the frequency of modulated wave varies as amplitude The maximum distance upto which it can detect object
of modulating wave located on the surface of the earth is (Radius of earth
= 6.4 × 106m) [AIEEE 2012]
(c) the frequency of modulated wave varies as frequency
(a) 80 km (b) 16 km
of modulating wave
(c) 40 km (d) 64 km
(d) the frequency of modulated wave varies as frequency
13. The fundamental radio antenna is a metal rod which has a
of carrier wave
length equal to [VITEEE 2006]
7. Audio signal cannot be transmitted because
(a) l in free space at the frequency of operation
(a) the signal has more noise [Kerala PMT 2005]
(b) l/2 in free space at the frequency of operation
(b) the signal cannot be amplified for distance
communication (c) l/4 in free space at the frequency of operation
(c) the transmitting antenna length is very small to design (d) 3l/4 in free space at the frequency of operation
(d) the transmitting antenna length is very large and 14. The maximum range dmax , of radar is [VITEEE 2006]
impracticable (a) proportional to the cube root of the peak transmitted
8. In which of the following remote sensing technique is not power
used? [Kerala PMT 2005] (b) proportional to the fourth root of the peak transmitted
(a) Forest density (b) Pollution power
(c) Wetland mapping (d) Medical treatment (c) proportional to the square root of the peak transmitted
9. Of the following which is preferred modulation scheme for power
digital communication? [Kerala PMT 2006] (d) Not related to the peak transmitted power at all
(a) Pulse Code Modulation (PCM) 15. The phenomenon by which light travels in an optical fibres
(b) Pulse Amplitude Modulation (PAM) is [VITEEE 2008]
(c) Pulse Position Modulation (PPM) (a) Reflection (b) Refraction
(d) Pulse Width Modulation (PWM) (c) Total internal reflection (d) Transmission
Free eBooks on @neetquestionpaper2020

1046 Physi cs

1. Television signals are 12. A transmitter radiates 10 kW of power with the carrier
(a) frequency modulated unmodulated and 11.8 kW with the carrier sinusoidally
(b) amplitude modulated modulated. The modulation factor is
(c) both frequency and amplitude modulated (a) 56% (b) 60 % (c) 72 % (d) 84%
(d) phase modulated 13. Array gain of an antenna is
2. In a super heterodyne receiver, the IF is 455 kHz. If it is
(a) directly proportional to power radiated by isotropic
tuned 1200 kHz, the image frequency will be
antenna
(a) 1562 kHz (b) 1675 kHz
(c) 2110 kHz (d) 2025 kHz (b) invesely proportional to power radiated by isotropic
3. Space wave communication is limited antenna
(a) to the line of sight distance (c) directly proportional to power radiated by practical
(b) by earth’s curvature antenna
(c) either (a) or (b) (d) inversely proportional to square of power radiated by
(d) both (a) and (b) practical antenna.
4. The service area of space wave communication increases 14. The maximum value of induced emf in loop antenna occurs
by (a) when plane of coil is along direction of transmitting
(a) increasing the height of transmitting antenna signal
(b) decreasing the height of receiving antenna (b) when plane of coil is in perpendicular direction of
(c) increasing the height of both transmitting and transmitting signal
receiving antenna (c) when plane of coil is perpendicular to 45°
(d) decreasing the distance between transmitting and
(d) None of these
receiving antenna
5. Sky wave propagation is not possible for frequencies 15. What will be the image frequency of an FM radio receiver
(a) equal to 30 MHz (b) less than 30 MHz that is tuned to 98.6 MHz broadcast station?
(c) greater than 30 MHz (d) None of these (a) 111.8 MHz (b) 108 MHz
6. The picture signal in TV-broadcast is modulated in (c) 121.6 MHz (d) 132 MHz
(a) SSB (b) VSB (c) FM (d) DSB 16. Main function of the RF amplifiers in a superheterodyne
7. The radio altimeter on an aircraft is a receiver is to
(a) MTI radar (a) amplify signal
(b) continuous wave radar (b) reject unwanted signal
(c) FM-continuous wave radar (c) discriminate against image frequency signal and IF-
(d) Doppler radar signal
8. Field strength of tropospheric TV signal is proportional to
(d) all of the above
1 1 17. Optical fibre are used for long distance communication
(a) (b) l (c) (d) l2
l l2 because
9. A broadcast radio transmitter radiates 12 kW when (a) it amplifies signals to be transmitted
percentage of modulation is 50%, then the unmodulated (b) it transfer signals faster than electrical cables
carrier power is (c) it pre-emphasise weak signals
(a) 5.67 kW (b) 7.15 kW (d) it provide little attenuation as compared to electrical
(c) 9.6 kW (d) 12 kW cable for light propagation
10. Shanon’s law relates 18. Optical fibres transmit light along its axis, by the process of
(a) information carrying capacity to S/N ratio (a) total internal reflection
(b) frequency to antenna gain (b) refraction
(c) amplitude to antenna gain (c) interference
(d) antenna gain to channel bandwidth (d) diffraction
11. Magnetic-coil are used in camera tube to 19. Modem is a short form of
(a) increase electron-beam intensity (a) modulator-demodulator
(b) decrease electron-beam intensity (b) multiplexer-demultiplexer
(c) give horizontal and vertical motion to the beam (c) multivibrator-degenerator
(d) None of these (d) None of these
Free eBooks on @neetquestionpaper2020

Communication Systems 1047


20. An audio signal represented as 25 sin 2p (2000 t) amplitude 33. The electron density in all the layers of ionosphere
modulated by a carrier wave : 60 sin 2p (100, 000)t. The (a) is the same
modulation index of the modulated signal is (b) decreases with altitude
(a) 25% (b) 41.6 % (c) 50 % (d) 75 % (c) increases with altitude
21. Critical frequency that gets reflected back from ionosphere (d) sometimes decreases sometimes increases
is 34. Ionosphere as a whole is
(a) same for all layers of the ionosphere
(a) +vely charges (b) –vely charges
(b) different for different layers of the ionosphere
(c) not dependent on layers of the ionosphere (c) electrically neutral (d) can't say
(d) None of these 35. In an AM wave, the information is contained within
22. In sky-wave propagation, skip-distance depends on (a) r.f. carrier wave
(a) frequency of e.m. waves transmitted (b) only lower and upper side frequencies
(b) critical frequency of the layer (c) both r.f. carrier and side frequencies
(c) height of layer above earth’s surface (d) None of these
(d) all of the above 36. The frequency deviation in a FM transmission is 18.75 KHz.
23. For a single side band transmission a balanced modulator If it broadcasts in 88-108 MHz band, then the percent
is used to modulation is
(a) increase power of carrier wave (a) 10 % (b) 25 % (c) 50 % (d) 75 %
(b) increase amplitude of carrier wave 37. In FM, when frequency deviation is doubled, then
(c) suppress audio signal (a) modulation is halved
(d) suppress carrier component (b) carrier swing is halved
24. The tank circuit used in a radio transmitter should have (c) modulation is doubled
(a) high effective Q (d) modulation index is decreased
(b) low effective Q 38. The height of a television tower is 100 m. If radius of earth
(c) loosely coupled load is 6.4 × 106 and average-population density surrounding
(d) both (a) and (c) the tower is 1000 per km2, then the population covered by
25. For an AM wave, the maximum voltage was found to be 10 the television transmission is
V and minimum voltage was 4 V. The modulation index of (a) 2.06 × 106 (b) 4.02 × 106
the wave is (c) 5.18 × 109 (d) 6.04 × 109
(a) 0.33 (b) 0.43 (c) 0.56 (d) 0.64 39. Pre-emphasis in FM system is done to
26. For an AM-system the total power of modulated signal is (a) compress modulating signal
600 W and that of carrier is 400 W, the modulation index is (b) expand modulating signal
(a) 0.25 (b) 0.36 (c) 0.54 (d) 1 (c) amplify lower frequency component of the modulating
27. Which of the following AM-scheme requires the minimum signal
transmitted power & minimum channel bandwidth? (d) amplify higher frequency component of the modulating
(a) VSB (b) DSB-SC (c) AM (d) SSB signal
28. The rms value of a carrier voltage is 100 volts. Compute its 40. A 10 kW carrier is sinusoidally modulated by two carriers
rms value when it has been amplitude modulated by a corresponding to a modulation index of 30% and 40%
sinusoidal audio voltage to a depth of 30%. respectively then total power radiated by the modulator is
(a) 94 V (b) 104 .5V (c) 114.4 V (d) 124 V (a) 10.25 kW (b) 11.25 kW
29. A 1 kW carrier is modulated to a depth of 80%. The total (c) 12.75 kW (d) 17 kW
power in the modulated wave is 41. In a radar system if peak transmitted power is increased by
(a) 1.32 (b) 1.56 (c) 1.84 (d) 1.96 a factor of 81, then the maximum-range will be increased by
30. Calculate the power developed by an amplitude modulated a factor of
wave in a load resistance of 100 W, if the peak voltage of (a) 3 (b) 6 (c) 9 (d) 27
carrier wave is 100 V and modulation index is 0.4. 42. An optical fibre is made of quartz filaments of refractive
(a) 50 watt (b) 54 watt index 1.70 and it has a coating of material whose refractive
(c) 104 watt (d) 4 watt index is 1.45. The range of angle of incidence for one laser
beam to suffer total internal reflection is
31. The maximum and minimum amplitude of an AM wave are
(a) 0° to 56.8° (b) 0° to 62.6°
90 mV and 30 mV respectively. The depth of modulation is
(c) 0° to 90° (d) 0° to 180°
(a) 0.6 (b) 0.5 (c) 0.4 (d) 0.3 43. For 100% modulation (AM), the useful part of the total
32. Ground waves are polarised power radiated is
(a) Parallel to the earth's surface 1 1
(a) of the total power (b) of the total power
(b) normal to the earth's surface 2 3
(c) at an angle 45° from earth's surface 1 2
(d) in any direction. (c) of the total power (d) of the total power
4 3
Free eBooks on @neetquestionpaper2020

1048 Physi cs
44. In PCM if the transmission path is very long 54. Intensity of electric field obtained at receiver antenna for a
(a) pulse spacing is reduced space wave propagation is
(b) pulse amplitude is increased (a) directly proportional to the perpendicular-distance
(c) pulse width is increased from transmitter to antenna
(d) repeater stations are used. (b) inversely proportional to the perpendicular-distance
45. Buffer amplifier is used between one master oscillator and
from transmitter to antenna
Harmonic generator in AM-transmitter because
(c) directly proportional to the square perpendicular-
(a) it reduces S/N ratio
distance from transmitter to antenna
(b) it amplifies signal
(d) inversely proportional to the square perpendicular-
(c) it increases signal frequency
distance from transmitter to antenna
(d) it avoids frequency deviation in tank-circuit
55. The function of an amplitude limitter in an FM-receiver is
46. Synchronizing pulses are transmitted during
(a) to reduce the amplitude of the signal to suit IF amplifier
(a) fly-back intervals of horizontal motion of camera
(b) to amplify low frequency signal
scanning
(c) to eliminate any change in amplitude of receiver FM
(b) fly-back intervals of vertical motion of camera scanning
signal
(c) both (a) and (b)
(d) None of these
(d) None of these
56. A 1000 kHz carrier is simultaneously modulated with 300
47. An FM signal has a resting frequency of 105 MHz and
highest frequency of 105.03 MHz when modulated by a Hz, 800 Hz and 2 kHz audio waves. The frequencies present
signal of frequency 5 kHz. The carrier swing is in the output is
(a) 25 kHz (b) 54 kHz (c) 60 kHz (d) 75 kHz (a) 999.7 kHz, 100.3 kHz, 999.2 kHz
(b) 1000.8 kHz, 998 kHz, 1002 kHz
48. An antenna has a radiation resistance of 68 W, a load
(c) 1002.8 kHz, 996 kHz, 1106 kHz
resistance of 10 W, and power gain of 16. The directive (d) both (a) and (b)
gain of the antenna is 57. A super heterodyne receiver is designed to receive
(a) 15 (b) 16.02 (c) 17.08 (d) 18.35 transmitted signals between 5 and 10 MHz. The tuning
49. Folded dipole is preferred over a simple dipole because it range of the local oscillate for IF frequency 600 kHz for
provides high-side tuning would be
(a) high input impedance for best matching
(a) 4.6 to 9.6 MHz (b) 5.6 to 10.6 MHz
(b) increased bandwidth
(c) 4.6 to 10.6 MHz (d) 5.6 to 9.6 MHz
(c) both (a) and (c)
58. Which of the following pair is correctly matched
(d) None of these
(a) Radio telegraph-VSB (b) Television-SSB
50. 12 signals each band limited to 5 kHz are to be transmitted
by frequency-division multiplexer. If AM-SSB modulation (c) Radio broadcast-AM (d) Radar-AM
guard band of 1 kHz is used then the bandwidth of 59. Consider the following amplitude modulated (AM) signal ,
multiplexed signal is where fm < B
(a) 101 kHz (b) 99 kHz xAM (t) = 10 (1 + 0.5 sin 2pfmt) cos 2pfct
(c) 84 kHz (d) 71 kHz The average side-band power for the AM signal given
51. In a radar system, peak transmitted power is increased a above is
factor of 81 and the antenna diameter is increased by a (a) 25 (b) 12.5 (c) 6.25 (d) 3.125
factor of 3, then the maximum-range will increase by a factor 60. An AM- signal is given as
of
xAM (t) = 100 [p(t) + 0.5g(t)] cos wct
(a) 81 (b) 27 (c) 9 (d) 3
in interval 0 £ t £ 1. One set of possible values of the
52. A carrier frequency of 10 kV at 1 MHz is amplitude modulated
modulating signal and modulation index would be
by a 1 kHz signal of 6 kV peak voltage. When the modulation
pattern is observed on a calibrated CRO, the voltage (a) t, 0.5 (b) t, 1.0 (c) t, 1.5 (d) t2, 2.0
indicated by the CRO will be 61. A device with input x(t) and outputy(t) is characterized by:
(a) 16 (b) 32 (c) 864 (d) 84 y(t) = x2 (t).
53. The height of satellite above earth surface is 36000 km and An FM signal with frequency deviation of 90 kHz and
radius of earth is 6400 km and period of revolution of satellite modulating signal bandwidth of 5 kHz is applid to this
is 24 hrs. The linear speed of the satellite will be device. The bandwidth of the output signal is
(a) 3.1 × 108 m/s (b) 4.1 × 106 m/s (a) 370 kHz (b) 190 kHz
(c) 6 × 10 m/s8 (d) 5.8 × 106 m/s
(c) 380 kHz (d) 95 kHz.
Free eBooks on @neetquestionpaper2020

Communication Systems 1049


62. A sinusoidal carrier voltage of frequency 10 MHz and 70. The depth of modulation is
amplitude 200 volts is amplitude modulated by a sinusoidal (a) 8 (b) 0.8
voltage of frequency 10 kHz producing 40% modulation. (c) 800 (d) None of these
Calculate the frequency of upper and lower sidebands. 71. The frequency of carrier wave is
(a) 10010 kHz, 9990 kHz (b) 1010 kHz, 990 kHz (a) 3× 106 Hz (b) 1.5 × 103Hz
3
(c) 3× 10 Hz (d) 1.5 × 106 Hz
(c) 10100 Hz,9990 Hz (d) 1010 MHz, 990 MHz
72. The minimum and maximum value of modulated carrier wave
63. If modulation is 100%, what will be the useful part of total
is
power ?
(a) 2V, 18 V (b) 4V, 16V
(a) 1/2 (b) 2/3 (c) 1/3 (d) 4/3 (c) 2V, 8 V (d) 4V, 12 V
64. Depth of modulation in terms of Emax and Emin is 73. The frequency of modulating signal is
(a) ma = Emax + Emin/Emin (a) 1000 Hz (b) 100 Hz
(b) ma = Emax – Emin/Emax (c) 10 MHz (d) 100 MHz
(c) ma = Emax – Emin/Emax + Emin 74. The bandwidth of modulated signal is
(d) ma = Emax + Emin/Emax – Emin (a) 1000 Hz (b) 2000 Hz
65. The maximum range for the tropospheric transmission of (c) 1500 Hz (d) 2500 Hz
radio wave of wavelength 3m using the transmitting antenna PASSAGE-2
and receiving antenna of heights 100 m and The electron density of a layer of ionosphere at a height 150 km
60 m respectively is from the earth's surface is 9 × 109 per m3. For the sky transmission
(a) 8 m (b) 800 m (c) 8 km (d) 80 km from this layer up to a range of 250 km,
Directions for Qs. (66 to 68) : Each question contains
75. The critical frequency of the layer is
STATEMENT-1 and STATEMENT-2. Choose the correct answer
(a) 2 Hz (b) 2.7 Hz
(ONLY ONE option is correct ) from the following-
(c) 2.78 kHz (d) 2.7 MHz
(a) Statement -1 is false, Statement-2 is true 76. Maximum useable frequency is
(b) Statement -1 is true, Statement-2 is true; Statement -2 is a (a) 3.17 Hz (b) 3.17 × 106 HZ
correct explanation for Statement-1 (c) 3.17×10 Hz 3 (d) 3.17 × 1010Hz
(c) Statement -1 is true, Statement-2 is true; Statement -2 is not 77. Angle of incidence of this layer is
a correct explanation for Statement-1 (a) 0° (b) 38° (c) 31.6° (d) 42°
(d) Statement -1 is true, Statement-2 is false PASSAGE-3
66. Statement 1 : Electromagnetic waves with frequencies more Communication system means the set up used to transfer the
than the critical frequency of ionosphere cannot be used information or view from one place to another place. In the modern
for communication using sky wave propagation. days the communication may be electrical, electronics or optical
Statement 2 : The refractive index of the ionosphere communication. The three component of a modern communication
becomes very high for frequencies higher than the critical are, transmitter, channel and receiver. A signal is defined as a
frequency. single valued function of time. This function has a unique value
67. Statement 1 : Diode lasers are used as optical sources in at every instant of time. Communication channel carries the
optical communication. modulated wave from transmitter to receiver. The air transmission
Statement 2 : Diode lasers consume less energy. lines and free space are the common communication channels.
68. Statement 1 : Short wave bands are used for transmission The receiver consists of a pickup antenna, a demodulator, an
of radio waves to a large distance. amplifier and a transducer.
Statement 2 : Short waves are reflected by ionosphere. 78. Which of the following is correct regarding a signal?
Directions for Qs. (69 to 80) : Read the following passage(s) (a) Maximum value (b) Single value
carefully and answer the questions that follows: (c) Receiver (d) Two values
PASSAGE-1 79. Some of the important communication channels are:
An amplitude modulated voltage is expressed as (a) air (b) transmission line
(c) free pace (d) any of the above
e = 10 (1 + 0.8 cos 2000 pt) cos 3 ´ 10 6 pt volt 80. Which of the following is not a part of receiver?
69. The peak value of carrier wave is (a) Receiving antenna (b) Amplifier
(a) 5 V (b) 8 V (c) 10 V (d) 100 V (c) Modulator (d) Demodulator
Free eBooks on @neetquestionpaper2020

1050 Physi cs

Exercise 30.1 Exercise 30.2

1. (c) Tele means long distance. 1. (b) Sound waves such as speech or song etc. that a human
2. (d) Out of the total message usually a part is conveyed. being can hear ranges from 20 Hz to 20 kHz frequencies
This part of message in form of bits is called above 20 kHz can not be heard by human ear.
information. 2. (c) Electromagnetic wave of audible frequency have vary
small energy content » 10–12 eV and their amplitude
3. (b) The message from the information source may not be
is greatly reduced due to dissipations of ennergy in
in electrical form so to convert these information i.e. in
travelling a long distance.
form of sound to electrical form a transducer like a
Secondly for frequency range 20 Hz to 20kHz the length
microphone is used.
of antenna required is of the order of wavelength of
4. (c) Noise the unwanted energy and distortion both wave to be transmited
occuring at various stage of a system leads to
i.e. length of antenna » l = c/n(m) = 1.5 × 107m
deterioration of signal as signal to noise ratio becomes
for n in the range 20 to 20 kHz
so poor that signal becomes unintelligble and useless.
and c = 3 × 108 m/s.
5. (d) Buffer amplifier isolate master oscillator from the
An antenna of length 1.5 × 104 m is not practically
influence of modulation done at a later stage.
possible.
6. (a) Loudspeaker is a transducer which converts electrical
3. (d) Since radio frequency waves can travel long distances
signal transmitted by the transmitter to sound form at
because these waves are of wave length of the order
the receiving end. of 100 m and their energy content is quite large
7. (d) The RF-signal coming from the transmitter needs to therefore e.m.wave of audible frequency are
be demodulated by a detector inorder to remove the superimposed with radio frequency waves.
carrier frequency and receive back the low-frequency 4. (d) On superimposition of two waves the audible
original signal. frequency wave is the modulating wave and radio-
8. (b) Due to several advantage of FM over AM, to get better wave is the carrier wave, thus the resultant wave
quality signal the sound part of TV-signal is frequency obtained is known as modulated wave as it is obtained
modulated. by the process of modulation.
9. (d) The received signals is either AM or FM so it needs to 5. (d) Earth's atmosphere is divided into different layers out
be demodulated ie. decoded to get back original signal. of which the ozone layer absorbs the X-rays and g-
It also needs to be stored and interpreted at receiving rays where visible light and microwaves are the waves
end. which are not absorb by any layer of atmosphere.
10. (d) Collation is a process of sorting information done 6. (a) Since the attenuation of ground waves increases with
before transmitting the required signal. increase in frequencies so only low frequency- radio
11. (a) Encoding modulation of signal i.e. to be transmitted is waves uses this mode of propagation for short
done with carrier frequency at transmitting end to avoid distances.
interference with other signals th at are also 7. (d) During ground wave propagation of radio waves, a
charge induced on the earth's surface which takes the
transmitted.
form of current as the wave propagate. The earth offers
12. (a) Picture signal in amplitude modulated to avoid
resistance in the flow of induced current due to which
complication in development of transmitter & receiver
the waves are attenuated.
structure.
8. (a) space wave propagation takes place in such a way
13. (b) Channel indicate frequency range at which different
that the radio waves transmitted at an angle from
R.F. signals all transmitted. earth's surface gets reflected by the troposphere and
14. (a) Radio broadcast transmitter are used for above then reaches the receiving antenna since the height
purpose. AM transmitter operating on long wave, of troposphere is quite small, long-range propagation
medium & short wave are used. by this mode is not possible.
Free eBooks on @neetquestionpaper2020

Communication Systems 1051


9. (a) If the direct wave and reflected (from earth's surface) 4. (c) Percentage of modulation (ma %) = 0.6 × 100
wave are in phase i.e. the path difference (between i.e. 60%
these waves is l) then they interfere constructively 5. (c) e = Ec ( 1+ ma coswmt) coswct
and thus resultant wave obtaind at the receivers end
is maximum. 1000p
Þ wm = = 500Hz
2p
1
10. (d) Phase velocity of e.m.wave in free space c =
m 0 Î0 5 ´ 106 p
6. (d) wc = = 2.5 ´ 106 Hz
Phase velocity of e.m.wave in ionised medium 2p

1 7. (a) wc + wm
vp = = 2.5 × 106 + 0.0005 × 106Hz
æ Ne2 ö
m 0 e0 ç 1 - ÷÷ wc – wm
ç e mw2
è 0 ø = 2.5 × 106 – 0.0005 × 106 Hz
\ vp >c but vg < c in ionised medium. = 2.4995 × 106 Hz
11. (a) Radio wave enters from an un-ionised medium to an 8. (c) E max = ( 1+ ma) Ec = ( 1+ 0.6) × 10 = 16 V
ionised medium, the wave incident on the boundary Emin = ( 1– ma) Ec = ( 1– 0.6) × 10 = 4V.
of the medium deviates from its straight path i.e. will
E max – E min
bend away from normal because ionised medium 9. (c) = m a -Depth of modulation.
behaves as a rarer medium w.r.t. to unionised medium. E max + E min

f2 E max – E min 10 – 4 6
12. (d) Skip distance, Dskip = 2h -1 10. (b) ma = = = = 0.428 = 42.8%
2 E max + E min 10 + 4 14
f0
Where f is the frequency of e.m wave transmitted, fc is Exercise 1 : NCERT Based Questions 
the critical frequency of the layer, h is the height of
the layer above earth's surface. 7. Frequency range over which an equipment operates or the
13. (b) Stratosphere and troposphere allows the radio waves portion of the spectrum occupied by the signal.
to pass through it but they are not reflected back to 8. When low frequency message signal is superimposed on a
earth's surface by ionosphere. high frequency.
Only frequency 1500 kHz to 30 MHz can be propagated 10. Modulation index is ratio of amplitude of message and
by this mode.
A
14. (d) TV-signal ( 30 MHz -300 MHz) are not reflected by carrier wave m = m .
ionosphere. Therefore, sky-wave propagation is not Ac
possible and similarly ground & space wave is also 14. Modulation index determines the strength and quality of
not feasible hence they are transmitted to artificial transmitted signal.
satellite from where they are transmitted back to the 20. 31 km; 3018 km2
earth. 21. (c) 22. (d) 23. (b) 24. (d)
Exercise 30.3 25. (d) 26. (b) 27. (d) 28. (a)
Exercise 2 : PAST Competition MCQs
actual frequency deviation 1. (c) Carrier + signal ® modulation.
1. (b) m= ´ 100%
max.allowed frequency deviation 2. (d) Surgery needs sharply focused beam of light and laser
can be sharply focussed.
(Df )actual
= ´100% 3. (c) 4. (b)
( Df )max
5. (d) A very small part of light energy is lost from an optical
if (Df) actual = (Df ) max fibre due to absorption or due to light leaving the fibre
m = 100 % as a result of scattering of light sideways by impurities
2. (a) SSB transmission to signal are possible at load in the glass fibre.
wc + wm or wc – wm 6. (b) In frequecy modulation th e frequency of the
Þ (1000 + 10) kHz or (100 – 10) kHz. modulated wave is the linear function of the amplitude
Þ 1010 kHz or 990 kHz of the modulating wave.
3. (d) Comparing to general equation of AM- wave 7. (d) Following are the problems which are faced while
e = Ec (1 + ma coswmt)coswct, peak value of carrier transmitting audio signals directly,
wave is Ec = 5V. (i) These signals are relatively of short range.
Free eBooks on @neetquestionpaper2020

1052 Physi cs
(ii) If every body started transmitting these low 3. (d) The space wave propagation is limited due to the line
frequency signals directly, mutual interference will of sight distance and by the curvature of the earth.
render all of them ineffective. 4. (c) Maximum range of space wave propagation
(iii) Size of antenna required for their efficient radiation
would be larger, i.e., about 75 km. 4
d= ´ 1.23 é H t + H r ù
8. (d) Remote sensing is the technique to collect information 3 ë û
about an object in respect of its size, colour, nature,
location, temperature etc. without physically touching Þ d ¥ Ht
it. There are some areas or locations which are
inaccessible. So to explore these areas or locations, a d ¥ Hr
technique known as remote sensing is used. Remote
sensing is done through a satellite. \ d increases if H t and Hr i.e. height of transmitting
9. (a) and receiving antenna increases.
10. (c) Optical fibres are not subjected to electromagnetic 5. (c) Sky wave propagation is not possible for frequency >
interference from outside. 30 MHz because they are not reflected by ionosphere.
11. (b) For long distance communication, sky wave signals 6. (b) Bandwidth is reduced using VSB
are used. SSB is not used because removal of one side band is
Also, the state of ionosphere varies every time. So, practically impossible.
both statements are correct. Bandwidth is reduced because same information is
12. (a) Let d is the maximum distance, upto it condetict the
present in both for picture signal is amplitude
objects C
modulated.
From DAOC
7. (c)
OC 2 = AC 2 + AO 2 h
d 88 p h t h r
(h + R )2 = d 2 + R 2 B 8. (a) Electric field strength E =
ld 2
2
Þ d = (h + R ) - R2 2 A q R 1
R
Þ Eµ
l
d = ( h + R) 2 - R 2 O
Pt 12 12
2 9. (c) Pc = = = = 9.6 kW
d = h + 2hR
1+
m a2
1+
( 0.5) 2 1.25
d = 500 + 2 ´ 6.4 ´ 10 = 80 km
2 6 2 2
13. (c) 10. (a) Channel capacity
14. (b) Maximum Range of the radar is given by C = BW log10 (1 + S / N )
1/ 4
æ P A2 S ö 11. (c) The electron-beam coming out of the electron gun
Rmax = ç t 2
ç 4 pl P ÷÷ needs to scan each element on the photoconductive
è min ø

Where Pt : peak value of transmitted power layer from top to bottom so for scanning entire target
A : caputre area of the receiving antenna electron beam is deflected by these magnetic
S : radar cross-sectional area deflecting coils.
l : wavelength of RADAR wave æ m2 ö
Pmin : minimum receivable power of the receiver 12. (b) Pc = Pt ç1 + ÷
ç 2 ÷ø
è
15. (c) In optical fibre, light travels inside it, due to total
internal reflection.
æ m2 ö
Þ 11.8 = 10 ç1 + ÷
Exercise 3 : Conceptual & Applied MCQs ç 2 ÷
è ø
1. (c) TV signal comprises of video and audio signals. Þ m = 0.6 Þ % modulation = 60%.
Video signal is AM and sound signal is FM.
2. (c) Given fi = 455 kHz æP ö
13. (a) G =10 log10 ç 0 ÷ dbs
fs = 1200 kHz è Pn ø
\ Image frequency = f s + 2f i G µ P0 ( ® power radiated by isotropic antenna)
= 1200 + 2 ´ 455
1

Pn ®
= 2110 kHz ( power radiated by practical antenna)
Free eBooks on @neetquestionpaper2020

Communication Systems 1053

2 pNA cos q æ m 2ö
14. (a) Induced emf e = E m volts 26. (d) PT = PC ç1 + a ÷
l ç 2 ÷ø
è
Þ e µ q æ m 2ö 3 ma 2
\ 600 = 400 ç1 + a ÷ Þ = 1 +
maximum value of e occurs when ç 2 ÷ø 2 2
è
q = 0, p, 2p.........
m 2 1
Þ when plane of the coil is along the direction of or a = Þ m a = 1
2 2
transmitting signal.
27. (d) Since in SSB transmision only one side band is
15. (c) FM (I.F)=11.5MHz transmitted while in other 3-cases more than a side
F(image) = fs+2 fIF = 98.6 + 2 ´11.5 = 121.6 MHz band is transmitted, so minimum power is transmitted
16. (d) 17. (d) for SSB. Similarly SSB bandwidth is minimum BW=
wm .
18. (a) The side-wall of optical fibre provides total internal
reflection of beam of light incident on it. é m 2ù V 2rms V 2c é ma 2 ù
19. (a) 28. (b) Pt = Pc ê1 + a ú Þ = ê1 + ú
ë 2 û 2 2 ë 2 û
B
20. (b) Modulation index = é
A m 2ù
V 2 rms = V 2 c ê1 + a ú
B = 25, A = 60 ë 2 û

25 ma 2
Þ M.I. = = 0.416 Þ m% = 41.6% (0.3)2
60 Þ Vrms = Vc 1 + Þ Vrms = 100 1 +
2 2
21. (b) Critical frequency fc = 9 N m = 104.5 volts.

where N m represents election density of layers é m 2 ù é ( 0.8 )2 ù


29. (a) P = Pc ê1 + a ú = 1 ê1 + ú = 1.32kW.
ë 2 û ë 2 û
Q fc µ N m
30. (b) Ec = 100 V, ma = 0.4, R = 100 W,
Þ fc is different for different layers.
22. (d) Skip distance is the minimum distance on earth’s E c 2 (100) 2
Pc = = = 50 watt
surface from the transmitter where e.m. wave of a 2R 2 ´ 100
definite frequency can reach after reflection from the
ionosphere æ m 2 ö é (0.4) 2 ù
P = ç1 + a ÷ Pc = ê1 + ú ´ 50 = 54 watt
ç 2 ÷ êë 2 ûú
è ø
f2
It is given by Dskip = 2h -1
fc 2 E max - E min 90 - 30 60 6
31. (b) ma = = = = = 0.5
E max + E min 90 + 30 120 12
Þ Dskip is dependent on h, f and fc.
32. (b) Ground waves have the property of being polarized
23. (d) Since maximum part of the power of modulated wave
normal to the earth's surface.
is contained with the carrier wave which does not
33. (c) Electron density of each layer of ionosphere is different
transmit any desired information, hence to avoid from the other. i.e. they are stratified. As we move
wastage of power to suppress carrier balanced upward density increases.
modulator is used. 34. (c) Ionsphere contain free electron & + ve ions. In
24. (d) Harmonics and fundamental frequency voltages cross- equillibrium, the no. of free electron is equal to the
modulate with each other producing fundamental number of positive ions. So as a whole it is elecrically
frequency currents whose relation with the normal neutral.
fundamental current vary with amount of harmonics 35. (b) The modulated voltage comprises of
in the oscillator circuit. The phase angle of the resultant (i) Carrier wave of frequency wc
fundamental frequency current thus varies resulting
in frequency variation. Such frequency variation can (ii) Lower side frequency ( wc - wm ) wave
be minimized by using a tank circuit having high (iii) Upper side frequency ( wc + wm ) wave
effective Q having loosely coupled load.
Thus in an AM wave information is contained in lower
V - Vmin 10 - 4 6
25. (b) ma = max = = = 0.43 ( wc - wm ) and upper ( wc + wm ) side frequencies.
Vmax + Vmin 10 + 4 14
Free eBooks on @neetquestionpaper2020

1054 Physi cs
36. (b) For given tran smission band 88-108 MHz 1
( Df )max = 75 kHz 41. (a) R max . range µ P4

given ( Df )actual = 18.75 kHz 1


R 4
( Df )actual Þ = ( 81) = 3
18.75 R1
\ % modulation m = ´100 = =25%
( Df )max 75
Þ R = 3R1
( Df )actual 42. (b) i–angle of incidence of the laser beam
37. (c) m= ´100 Þ m ¥ ( Df )actual
( Df )max r–angle of refraction
i.e. if frequency deviation is doubled then modulation i¢–angle of incidence of the laser beam inside the fibre
is doubled. ic–critical angle
38. (b) h = 100m, R = 6.4 ´ 106 m By definition of critical angle

1 1
sin ic = = = 0.856
d l mg 1.70 /1.45
d h
Þ i c = sin -1 0.856 = 58.5o

Thus if i ' > 58.5o ® r = 90 - r '


R
or r < 90o - 58.5o

Average population density = 1000 km -2 Þ r < 31.5o


sin i
= 10-3 m -2 By snell’s law, =a m g
sin r
Maximum range of TV transmission
1
d = 2hR = (2 ´100 ´ 6.4 ´106 ) 2
Þ sin i = 1.70 ´ sin 31.5o = 1.70 ´ 0.524 = 0.89

\ Maximum area covered by T.V. transmission Þ i = sin -1 0.89 = 62.6o


= pd 2 = p ´ 2hR = 2 ´ 3.14 ´ 100 ´ 6.4 ´ 106
\ range is 0o to 62.6o
= 4.02 ´109 m 2
43. (b) 100% modulation Þ ma = 1
\ the population covered by T.V. transmission
ma 2
( )
= 4.02 ´ 109 ´10-3 = 4.02 ´ 106
useful power
= =
1
=
total power radiated 2 + m 2 2 + 1 3
1
a
39. (d) Pre-emphasis of higher frequency component is
required in FM-system because high frequency terms 1
of modulating signal have small amplitude and Þ Useful power = ( total power radiated )
3
therefore small power relative to those of low
frequency term. 44. (d) When transmission path is long more repeater stations
In the reproduced program at the o/p, these high are needed at intermediate points as repeater receives
frequency terms have poor S/N ratio and at time noise signal, remove the noise, amplify it and retransmit it
may completely mask the signal at these high along the channel.
frequencies, so it is necessary to provide pre-emphasis 45. (d)
of high frequencies. 46. (c) Synchronization of both vertical and horizontal motion
of camera scanning is required to ensure perfect
E c2 synchronization between the scene being televised
40. (b) Pc = = 10 kW
2 and picture produced on the raster.
ma = m12 + m 22 = 0.30 2 + 0.402 = 0.50 47. (c) Carrier Swing = 2 ´ Df
= 2 ´ 105.03 - 105 = 2 ´ 0.03 MHz
æ m 2 ö æ ( 0.5 )2 ö
\ Pt = Pc ç1 + a ÷ = 10 ç 1 + ÷ = 11.25 kW = 0.06 MHz = 60 kHz
ç 2 ÷ ç 2 ÷
è ø è ø
Free eBooks on @neetquestionpaper2020

Communication Systems 1055

R 68 56. (d) Frequencies present in the output = 100 ± 0.3


48. (d) h= = = 0.872
R + R L 68 + 10 1000 ± 21 i.e. ®
Þ Power gain = h G 999.7 KHz, 100.3 KHz, 999.2 KHz
Power gain 16 1000.8 KHz, 998 KHz, 1002 KHz
Þ directive gain G = = = 18.35 57. (b) For high-side tunning
h 0.872
49. (c) fLO = fm + fIF , PIF = 600 kHz = 0.6 MHz
50. (d) Total signal B.W= 12 ´ 5 = 60 kHz
Þ f LOL = 5 + 0.6 = 5.6 MHz
11 guard band are required between 12 signal
\ guard bandwidth = 11 ´ 1kHz = 11kHz f LOU = 10 + 0.6 = 10.6 MHz
\ total bandwidth = 60 + 11 = 71kHz 58. (c) Radio telegraph–AM and FM is used
1 Television–VSB is used
51. (c) R max µ d µ P 4 Radar–PM or FM is used
and Radio broadcast–AM and FM is used
R 2 (new range) D2 (new diameter)
Þ R (original range) = D (original diameter) So correct pair is (c).
1 1
ma 2
1 59. (c) Average side-band power Pav = Pc
4
P24 (new transmitted power)
= Here ma = 0.5
1
Pc = 10
P14 (original transmitted power)
0.5 ´ 10 ´ 10
1 \ Pav = = 6.25
4
3D1 ( )
81 P1 4
= ´ = 3´ 3 = 9 60. (a) Comparing (x AM)t = 100 [1 + 0.5 t] coswct for 0<t<1
D1 1
4 with standard AM signal x AM = Ec [1+ma cos wmt]
cos wct
52. (b) Voltage observed on CRO of modulated signal (peak
to peak) We have modulating signal t and ma = 0.5.
= (carrier voltage + modulating signal voltage) ´ 2 61. (c) For x(t), BW = 2(Dw + w)
= (10 + 6 ) ´ 2 = 16 ´ 2 = 32 kV Dw is deviation and w is the band width of modulating
signal.
53. (a) Radius of orbit of satellite
\ BW = 2(90 + 5) = 190
= R + h = ( 6400 + 36000 ) km 2
For x (t), BW = 2 × 190 = 380
= 42400 km = 4.24 ´ 107 m 62. (a) Modulating signal frequency ® 10kHz
Carrier signal frequency® 10 MHz
Period of revolution
\ Side band frequency are
= 24 hrs = 24 ´ 60 ´ 60 = 86400 sec
USB = 10 MHz + 10 kHz = 10010 kHz
Þ linear speed
LSB = 10MHz – 10 kHz = 9990 kHz
2pr 2 ´ 3.14 ´ 4.24 ´ 10 7
v= = = 3.1´108 m / s Useful power ma2
T 86400 63. (c) =
Total power 2 + ma2
4pHhE
54. (d) Electric Field = -
ld 2 Useful power 1 1
for ma = 100%, = =
Total power 2 +1 3
1
Þ E.F. µ
d2 1
55. (c) The limitter removes from the carrier all amplitude Þ Useful power = total power..
3
variations which may caused by changes in the
transmission path, by man-made static or natural static. 64. (c)
This suppression of amplitude variation is necessary 65. (c) l = 3m, H = 100 m, h = 60 m
because FM-receives, a vary large improvement in S/ Path difference between direct wave and wave obtained
N results from this. after reflection from earth surface is
Free eBooks on @neetquestionpaper2020

1056 Physi cs

2 p æ 2Hh ö 2000p
+ p = 2p 73. (a) wm = 2000p \ Iff == 1000Hz .
l çè d ÷ø 2p
74. (b) BW = 2fm = 2 × 1000 = 2000 Hz.
4Hh 4Hh
or = 1 or d=
ld l 75. (d) f c = 9 N m = 9 ´ 9 ´ 1010

4 ´ 100 ´ 60 = 2.7 ´ 106 Hz = 2.7 MHz


Þd= = 8 ´ 103 m or 8km
3
66. (b) 67. (c) 68. (b) D2 6 ( 250 ´ 103 )2
76. (b) f = fc 1 + = 2.7 ´ 10 ´ 1+
4h 2 4 ´ (150 ´ 103 )
69. (c) Comparing given expression with 2

(e)AM = Ec(1 + ma cos wmt) cos wct


= 3.17 × 106Hz
peak value of carrier wave, Ec= 10V.
77. (c) f = fc = sec fi
70. (b) depth of modulation = m a = 0.8 or 80%
3 ´ 106 p f 3.17×106
= 1.5 ´106 Hz Secfi = = = 1.174
71. (a) Freq. of carrier wave fc = f c 2.7 ´106
2p
72. (a) Emax = (1 +ma) EC = (1 + 0.8) × 10 = 18 V
fi = sec –1 (1.174) = 31.6° .
Emin =(1–ma)EC = ( 1– 0.8) ×10 = 2V. 78. (b) 79. (d) 80. (c)

Вам также может понравиться